A PYQ Workbook of General Geography

Download as pdf or txt
Download as pdf or txt
You are on page 1of 351

PREVIOUS YEAR QUESTIONS

(PYQ) WORKBOOK OF
GENERAL
GEOGRAPHY
For Civil Services Prelims Examination

((1000+ Topic Wise Solved Objective Questions from UPSC CSE Prelims (1991 to 2023),
CAPF & CDS EXAM (2004 to 2023) and STATE PCS Prelims Exams (1991 to 2023))

PYQ WORKBOOK

Forum Learning Centres


DELHI (ORN) MUKHERJEE PATNA HYDERABAD GURGAON
NAGAR
19, Pusa Road, 2nd 862, Banda 2nd floor, AG 1st Floor, SM Plaza, Property No. 894,
Floor, IAPL House, Bahadur Marg, Palace, E Boring RTC X Rd, Indira G.F., Saraswati
Opposite Metro Ist Floor, Dr. Canal Road, Patna, Park Road, Jawahar Vihar, Chakkarpur,
Pillar #95-96, Mukherji Nagar, Bihar - 800001 Nagar, Hyderabad, Near MG Rd
Karol Bagh, New Near Batra Telangana - Metro Station,
Delhi-05 Cinema, Delhi - 500020 Sector-28,
110009 Gurgaon, Haryana
https://blog.forumias.com | www.academy.forumias.com | 9311740400
PREVIOUS YEAR QUESTIONS (PYQ) WORKBOOK OF
GENERAL GEOGRAPHY
For Civil Services Prelims Examination

First Edition (Print): 2024

No part of this publication may be reproduced or transmitted, in any form or by any means,
electronic, mechanical, photocopying, recording or otherwise, or stored in any retrieval system of
any nature without the written permission of the copyright holder and the publisher, application
for which shall be made to the publisher.
©Forum (Flaviant Network Pvt. Ltd.)

This book/material is sold subject to the conditions that it shall not, by way of trade or otherwise,
be lent, re-sold, hired out or otherwise circulated without the publisher's prior consent in any
form of binding or cover other than that in which it is published and without a similar condition
including this condition being imposed on the subsequent purchaser.

We are continuously working on improving our quality. If you are unhappy or happy with the
quality of this study material, want to report an error, or want to leave us feedback, you can email
us at [email protected]

© 2024 1st Print Edition | For feedback email us : [email protected]


The PYQ WORKBOOK
Dear Aspirants,

ForumIAS is glad to inform you that we have come up with a PYQ Workbook series
to provide the aspirants with a holistic idea regarding challenges and opportunities
associated with the changing pattern and difficulty level of UPSC CSE Prelims Exam.
The PYQ Workbook includes the questions asked in different objective questions-based
exams conducted by Union Public Service Commission like UPSC CSE Prelims Exam,
CAPF Exam and CDS Exam. It will also include the solved previous year questions from
different State PCS Prelims Examinations to equip the aspirants with every relevant fact
and concept needed for their UPSC CSE Prelims exam. This PYQ Workbook Series, along
with ForumIAS Practice books and Full-Length Test book, will help the aspirants to get
the required factual as well as conceptual clarity regarding the different subjects and to
crack their UPSC CSE Prelims Exam.

Trend of General Geography Questions in recent years:

GENERAL GEOGRAPHY

18 16
No. of Questions

15 14
12
13
10 9 9
11 8
7 7
5
4
2012 2014 2016 2018 2020 2022 2023
Year
Number of Geography questions asked in UPSC CSE GS-1 Prelims from 2011 till 2023

The trend analysis shown above suggests that the questions from General Geography in last
few years experienced a dip from double digit to single digit, but again, there was a trend
reversal in 2023 with at least 16 questions. It confirms again that there is no set pattern of
asking questions from a particular subject in UPSC Prelims Exam and the candidates must
devote equal time to every subject.
The questions asked in general geography include both static questions as well as few map-
based questions from current affairs covering the significant events around the globe. The
difficulty level of fundamental topics has remained easy to moderate. Through basic reading
of NCERTs and a standard book of geography, one would be able to solve most of the basic
static and conceptual questions asked in the papers. Also, one must be aware with day-to-
day significant events to solve the questions related to map-based locations, Lakes, rivers,
straits, regions of conflict etc. This demands a good reading of a leading newspaper and
connecting it with static part of general geography.

Happy Learning and Happy Testing!!!


Your team @ForumIAS
INDEX
UNIT 1. UNIVERSE & THE EVOLUTION OF EARTH .....1 – 29
1.1. UPSC CSE Previous Years' Questions .....1
1.2. Other Examination Previous Years' Questions .....4
SOLUTIONS .....11

UNIT 2. GEOMORPOLOGY .....30 – 63


2.1. UPSC CSE Previous Years' Questions .....30
2.2. Other Examination Previous Years' Questions .....33
SOLUTIONS .....41

UNIT 3. OCEANOGRAPHY .....64 – 79


3.1. UPSC CSE Previous Years' Questions .....64
3.2. Other Examination Previous Years' Questions .....65
SOLUTIONS .....69

UNIT 4. CLIMATOLOGY .....80 – 133


4.1. UPSC CSE Previous Years' Questions .....80
4.2. Other Examination Previous Years' Questions .....86
SOLUTIONS .....100

UNIT 5. INDIA PHYSICAL ENVIRONMENT .....134 – 227


5.1. UPSC CSE Previous Years' Questions .....134
5.2. Other Examination Previous Years' Questions .....146
SOLUTIONS .....167

UNIT 6. ECONOMIC & HUMAN GEOGRAPHY (INDIA AND WORLD) .....228 – 282
6.1. UPSC CSE Previous Years' Questions .....228
6.2. Other Examination Previous Years' Questions .....233
SOLUTIONS .....249

UNIT 7. MISCELLANEOUS .....283 – 346


7.1. UPSC CSE Previous Years' Questions .....283
7.2. Other Examination Previous Years' Questions .....289
SOLUTIONS .....302
GENERAL GEOGRAPHY

GENERAL GEOGRAPHY
UNIVERSE & THE EVOLUTION OF EARTH
*This unit consists of questions from Universe Solar System, Evolution of Earth and Latitudes &
Longitude.

1.1. UPSC CSE Previous Years’ Questions (a) Earth and Jupiter
(b) Jupiter and Saturn
1. In the northern hemisphere, the longest day (c) Saturn and Earth
of the year normally occurs in the : (d) Saturn and Neptune
[UPSC CSE Pre 2022] 5. Who of the following scientists proved that
(a) First half of the month of June the stars with mass less than 1.44 times the
(b) Second half of the month of June mass of the Sun end up as White Dwarfs
(c) First half of the month of July when they die? [UPSC CSE 2009]
(d) Second half of the month of July (a) Edwin Hubble
2. Consider the following statements : (b) S Chandrashekhar
(c) Stephen Hawking
1. High clouds primarily reflect solar
(d) Steven Weinberg
radiation and cool the surface of the
Earth. 6. Consider the following statements:
2. Low clouds have a high absorption of [UPSC CSE Pre 2008]
infrared radiation emanating from the 1. The albedo of an object determines its
Earth’s surface and thus cause warming visual brightness when viewed with
effect. reflected light.
Which of the statements given above is/are 2. The albedo of Mercury is much greater
correct ? [UPSC CSE Pre 2022] than the albedo of the Earth.
(a) 1 only Which of the statements given above is/are
(b) 2 only correct?
(c) Both 1 and 2 (a) 1 only
(d) Neither 1 nor 2 (b) 2 only
(c) Both 1 and 2
3. What is the difference between asteroids (d) Neither 1 nor 2
and comets? [UPSC CSE 2011]
1. Asteroids are small rocky planetoids, 7. Consider the following statements:
while comets are formed of frozen gases Assertion (A): To orbit around the Sun, the
held together by rocky and metallic planet Mars takes less time than the time
material. taken by the Earth.
2. Asteroids are found mostly between the Reason (R): The diameter of the planet Mars
orbits of Jupiter and Mars, while comets is less than that of Earth.
are found mostly between Venus and Code: [UPSC CSE Pre 2006]
Mercury. (a) Both (A) and (R) are individually true
3. Comets show a perceptible glowing tail, and (R) is the correct explanation of (A).
while asteroids do not. (b) Both (A) and (R) are individually true but
Which of the statements given above is/are (R) is not the correct explanation of (A).
correct? (c) (A) is true, but (R) is false.
(d) (A) is false, but (R) is true.
(a) 1 and 2 only
(b) 1 and 3 only 8. Consider the following statements:
(c) 3 only Assertion (A): Existence of human life on
(d) 1, 2 and 3 Venus is highly improbable.
4. In order of their distance from the Sun, Reason (R): Venus has an extremely high
which of the following planets lie between level of carbon dioxide in its atmosphere.
Mars and Uranus? [UPSC CSE 2009] Code: [UPSC CSE Pre 2005]

1 PYQ Workbook
GENERAL GEOGRAPHY

(a) Both A and R are true, and R is the correct 12. Among the following which planet takes
explanation of A maximum time for one revolution around
(b) Both A and R are true, and R is not the the Sun? [UPSC CSE Pre 2003]
correct explanation of A (a) Earth
(c) A is true, but R is false (b) Jupiter
(d) A is false, but R is true (c) Mars
(d) Venus
9. Consider the following statements:
[UPSC CSE Pre 2005] 13. Which one of the following statements is
1. The axis of the earth’s magnetic field is correct with reference to our solar system?
inclined at 23 and a half degrees to the [UPSC CSE Pre 2002]
geographic axis of the earth. (a) The Earth is the densest of all the planets
2. The earth’s magnetic pole in the northern in our solar system.
(b) The predominant element in the
hemisphere is located on a peninsula in
composition of Earth is silicon.
northern Canada.
(c) The Sun contains 75 percent of the mass
3. Earth’s magnetic equator passes through of the solar system.
Thumba in South India. (d) The diameter of the Sun is 190 times that
Which of the Statements given above is/are of the Earth.
correct? 14. Sun’s halo is produced by the refraction of
(a) 1, 2 and 3 light in- [UPSC CSE Pre 2002]
(b) 2 and 3 only (a) Water vapour in stratus clouds
(c) 2 only (b) Ice crystals in Cirro-Cumulus clouds
(d) 3 only (c) Ice crystals in Cirrus clouds
10. Consider the following statements: (d) Dust particles in Stratus clouds
Assertion (A): The same face of the Moon 15. Consider the following statements:
always faces the Earth. Assertion (A): Artificial satellites are always
Reason (R): The Moon completes one launched from the earth in the eastward
rotation around its own axis in 23 and half direction.
days which is about the same time that it Reason (R): The earth rotates from west to
takes to orbit the Earth. east and so the Satellite attains the escape
Code: [UPSC CSE Pre 2005] velocity.
(a) Both (A) and (R) are true, and (R) is the Code: [UPSC CSE Pre 2002]
correct explanation of (A). (a) Both (A) and (R) are true, and (R) is the
(b) Both (A) and (R) are true, but (R) is not correct explanation of (A).
the correct explanation of (A). (b) Both (A) and (R) are true, but (R) is not a
(c) (A) is true, but (R) is false. correct explanation of (A).
(d) (A) is false, but (R) is true. (c) (A) is true, but (R) is false.
(d) (A) is false, but (R) is true.
11. Consider the following statements:
16. Which of the following planets is the
[UPSC CSE Pre 2003]
farthest planet of the solar system?
Assertion (A): Moving from one place on [UPSC CSE Pre 2002]
earth to another place of high latitude, the (a) Neptune
weight of an object decreases. (b) Pluto
Reason (R): Earth is not a perfect sphere. (c) Sometimes Neptune and sometimes Pluto
Choose the correct answer by using the code (d) Mars
given below. 17. If the stars are seen to rise perpendicular to
(a) (A) and (R) both are true, and (R) explains the horizon by an observer, he is located on
(A) properly. the- [UPSC CSE Pre 2001]
(b) (A) and (R) both are true, but (R) does (a) Equator
not explain (A) properly. (b) Tropic of Cancer
(c) (A) is true, but (R) is wrong. (c) South Pole
(d) (A) is false, but (R) is true. (d) North Pole

PYQ Workbook 2
GENERAL GEOGRAPHY

18. A ship is moving from the eastern side of (c) Occurrence of ice cap and frozen water
Aleutian Islands towards Dutch Harbour. (d) Occurrence of ozone
It crosses 1800 meridian on 1st January 24. The group of small pieces of rock revolving
1999 at 23:30 hr. What time and date will round the sun between the orbits of Mars
be recorded by the captain of the ship in his and Jupiter are called:
diary after one-hour journey from the point
[UPSC CSE Pre 1997]
of crossing of the meridian?
(a) Meteors
[UPSC CSE Pre 1999]
(b) Comets
(a) January 1, 00:30 hr (c) Metroid
(b) January 2, 00:30 hr (d) Asteroids
(c) January 3, 00:30 hr
(d) January 4, 00:30 hr 25. The tail of comet is directed away from the
Sun, because– [UPSC CSE Pre 1997]
19. Which of the following scholars has
(a) As the Comet rotates around the Sun, the
suggested that the Earth originated from
lighter mass of Comet is pushed away due
the gases and the dust particles?
to centrifugal force alone.
[UPSC CSE Pre 1999] (b) As the Comet rotates, the lighter mass
(a) James Jeans of the Comet is attracted by some stars
(b) H. Alfven situated in the direction of its tail.
(c) F. Hoyal (c) The radiation emitted by the Sun exerts a
(d) O. Schmidt radial pressure on the comet throwing its
20. One Astronomical unit is average distance- tail away from the Sun.
[UPSC CSE Pre 1998] (d) The Tail of the Comet always exists in the
(a) Between Earth and Sun same orientation.
(b) Between Earth and Moon 26. If the earth’s direction of rotation is
(c) Between Jupiter and Sun reversed, what should be the IST when it is
(d) Pluto and Sun noon at the International Date Line?
21. Consider the following statements [UPSC CSE Pre 1997]
regarding asteroids: [UPSC CSE Pre 1998] (a) 06.30 hrs
1. Asteroids are rocky debris of varying sizes (b) 05.30 hrs
orbiting the sun. (c) 18.30 hrs
2. Most of the asteroids are small but some (d) 17.30 hrs
have diameters as large as 1000 km. 27. Consider the following Statements:
3. The orbit of asteroids lies between the [UPSC CSE Pre 1996]
orbits of Jupiter and Saturn. A person in spacecraft situated at the mid of
Of these statements: earth and Sun will see that–
(a) 1, 2 and 3 are correct 1. Sky is black
(b) 2 and 3 are correct 2. Stars do not twinkle
(c) 1 and 2 are correct 3. The temperature outside the spacecraft is
(d) 1 and 3 are correct more than that of the earth surface
22. Which one of the following Stars is nearest Which of the following is true?
to the Earth? [UPSC CSE Pre 1997] (a) Only 3
(a) Polaris (b) Only 1 and 2
(b) Alpha Centauri (c) Only 1 and 3
(c) Sun (d) 1, 2 and 3
(d) Sirius
28. Meteor is a- [UPSC CSE Pre 1995]
23. Which one of the following conditions is (a) Fast moving star.
most relevant for the presence of life on (b) A piece of matter which has entered the
Mars? [UPSC CSE Pre 1997] earth’s atmosphere from outer space.
(a) Atmospheric composition (c) Part of the stellar system.
(b) Thermal conditions (d) Tailless Comet.

3 PYQ Workbook
GENERAL GEOGRAPHY

29. The standard time of the following countries 34. The Solar Eclipse achieves totality only in
is ahead of or behind Greenwich Mean limited geographical regions because-
Time depending on whether they are East [UPSC CSE Pre 1993]
or West of the longitude passing through (a) The size of the shadow of the Moon on
Greenwich. the Earth is small as compared to the
1. Cuba cross section of the Earth.
2. Greece (b) The Earth is not a smooth flat surface but
3. Iraq has elevations and depressions.
4. Costa Rica (c) The trajectories of the Earth around the
5. Japan Sun and the Moon around the Earth are
Which one of the following sequential orders not perfect circles.
gives the correct arrangement of the countries (d) Sun rays can reach most of the peripheral
according to their standard time from ahead regions of the shadow of the Moon due to
to behind GMT? [UPSC CSE Pre 1995] atmospheric refraction.
(a) 5, 3, 2, 1, 4 35. Consider one special point at the Earth’s
(b) 2, 4, 1, 3, 5 surface (e.g., City Delhi) its temperature
(c) 4, 1, 3, 2, 5 at noon (e.g., 12 o’clock) will be higher in
(d) 3, 5, 4, 1, 2 Summer in comparison to winter because-
30. The colour of the star indicates its- [UPSC CSE Pre 1993]
[UPSC CSE Pre 1994] (a) Earth is much closer to Sun in Summer in
(a) Distance from Sun comparison to winter.
(b) Lighting or glow (b) In winter the Sun rays falling on the Earth
(c) Distance from earth bends more towards the Earth’s surface in
(d) Temperature comparison to the summer.
(c) The vaporization of water happens only in
31. A plane starts flying from 300 northern winter due to environmental precipitation
latitude and 500 eastern longitude and lands (d) In winters, the Earth’s axis becomes more
at the opposite end of the earth, where will inclined towards the Sun.
it land– [UPSC CSE Pre 1994]
(a) 300 Northern latitude, 500 Western 1.2. Other Examination Previous Years’
longitude Questions
(b) 300 Southern latitude, 500 Western
36. Black-hole is: [UPPCS (Pre) 2019]
longitude
(c) 500 Northern latitude, 300 Western (a) a flight recorder in aeroplane
longitude (b) a spot on the sun
(d) 300 Southern latitude, 1300 Western (c) a place in Antarctica
longitude (d) a collapsed star

32. When it is noon along 820 30’ E longitude 37. Consider the following events:
along what degree of longitude it will be 1. First reptiles
6.30 a.m.? [UPSC CSE Pre 1994] 2. First insects
0
(a) At 165 East 3. Shelled animals
(b) At 670 30’ West 4. First mammals
(c) At 00 East or West Arrange the above events in Chronological
(d) At 820 30’ West order of their origin on the earth and select
33. Which one of the following is not an the correct answer from the codes given
Astronomical object? [UPSC CSE Pre 1993] below: [U.P. P.C.S. (Pre) 2018]
(a) Pulsar (a) 2, 1, 3, 4
(b) Brittle star (b) 2, 3, 1, 4
(c) Black hole (c) 3, 2, 1, 4
(d) Quasar (d) 2, 4, 1, 3

PYQ Workbook 4
GENERAL GEOGRAPHY

38. Which of the following pairs is not properly 44. Which of the following planets takes the
matched? [U.P.PC.S. (Mains) 2011] least time to orbit the sun?
(a) The largest planet of the Solar System - [U.P.P.C.S. (Pre) 2002]
Jupiter (a) Pluto
(b) The smallest planet of the Solar System - (b) Mercury
Mercury (c) Earth
(c) The brightest planet of the Solar System (d) Saturn
- Venus
(d) The slowest moving planet of Solar 45. The milky way is classified as-
System - Mars [U.P.P.C.S. (Pre) 2001]
39. The ‘blue moon’ phenomenon occurs when– (a) Spiral Galaxy
[U.P.P.C.S. (Mains) 2009] (b) Elliptical Galaxy
(a) Two full moons occur in the same month. (c) Irregular Galaxy
(b) Four full moons appear in two consecutive (d) Round Galaxy
months of the same calendar year. 46. Which one of the following longitudes
(c) Two full moons appear in the same along with the Prime Meridian forms a
month, thrice in a calendar. great circle on the globe?
(d) None of the above. [U.P.P.C.S. (Pre) 2000]
40. Solar energy is due to- (a) 00
[U.P.P.C.S. (Pre) 2006] (b) 900 E
(a) Ionization (c) 900 W
(b) Nuclear Fusion (d) 1800
(c) Nuclear Fission 47. The limit beyond which stars suffer internal
(d) Oxidation
collapse is called: [U.P.P.C.S. (Pre) 1997]
41. When any object is taken from the Earth to (a) Chandrashekhar limit
the Moon? [U.P.P.C.S. (Pre) 2006] (b) Eddington limit
(a) Its weight increases (c) Hoyle limit
(b) Its weight decreases (d) Fowler limit
(c) There is no change in its weight
(d) It becomes totally weightless 48. A Ball thrown outside from an artificial
satellite revolving around the Earth will-
42. On the surface of the moon, two persons [U.P.P.C.S. (Pre) 1995]
cannot listen to each other as–
(a) Go to the Sun
[U.P.P.C.S. (Pre) 2003] (b) Go to the Moon
(a) Their ears stop working at the Moon. (c) Fall on the Earth
(b) There is no atmosphere on the Moon. (d) Revolve around the Earth in the same
(c) They wear special types of space suits on orbit with the same time period as satellite
the Moon.
(d) Sound travels very slowly on the Moon. 49. Which is true about planets?
43. Consider the following statements: [U.P.P.C.S. (Pre) 1992]
[U.P.P.C.S. (Pre) 2003] (a) Planets are non-luminous bodies and
don’t shine
Assertion (A): A black hole is an astronomical
entity that cannot be seen by telescope. (b) Planets shine though they are non-
luminous bodies
Reason (R): The gravitational field on a black (c) Planets do not shine though they are the
hole is so strong that it does not allow even luminous body
light to escape. (d) Planets are the luminous body and also
In the context of the above statements, which shine
one of the following is correct:
(a) Both (A) and (R) are true, and (R) is the 50. The solar eclipse occurs on-
correct explanation of (A). [U.P.P.C.S. (Pre) 1991]
(b) Both (A) and (R) are true, but (R) is not (a) Quarter Moon Day
the correct explanation of (A). (b) New Moon Day
(c) (A) is true, but R is false. (c) Any day
(d) (A) is false, but R is true. (d) Full Moon Day (Poornima)

5 PYQ Workbook
GENERAL GEOGRAPHY

51. Which of the following groups of planets is (a) The South Pole experiences 24 hours of
termed as ‘gas planets’ as they are composed darkness.
primarily of lighter ice, liquids and gases? (b) It occurs on 21st June.
[CDS 2023 (I)] (c) The North Pole experiences 24 hours of
(a) Mars, Jupiter, Neptune, Uranus darkness.
(b) Jupiter, Uranus, Neptune, Saturn (d) The Sun is at aphelion.
(c) Saturn, Mars, Jupiter, Neptune
57. Which of the following statements
(d) Neptune, Saturn, Mars, Uranus
concerning atmosphere of the Earth are
52. Ozone layer, which absorbs the ultra-violet correct? [CDS 2016 (II)]
radiation, is found in which one of the 1. In stratosphere, temperature increases
following layers of the atmosphere? with altitude.
[CDS 2021 (I)] 2. In mesosphere, temperature decreases
(a) lonosphere with altitude.
(b) Troposphere. 3. The lowest temperature of the
(c) Mesosphere
atmosphere is recorded in the upper part
(d) Stratosphere
of mesosphere.
53. The four planets closest to the Sun are called 4. Tropopause is an isothermal zone.
[CDS 2020 (II)] Which of the statements given above are
(a) Terrestrial planets correct?
(b) Giant planets
(a) 1 and 2
(c) Dwarf planets
(b) 1, 2 and 3
(d) Gas planets
(c) 3 and 4
54. Which one among the following stars is (d) All of these
nearest to the Earth? [CDS 2019 (I)]
58. Which one of the following statement are
(a) Sirius
(b) Arcturus not correct? [CDS 2016 (II)]
(c) Spica (a) Position of the Sun, Earth and Moon in
(d) Proxima Centauri a straight line of 180° angle is known as
Syzygy
55. Match List I with List II and select the (b) Syzygy conjunction causes solar eclipse
correct answer using the codes given below (c) Syzygy opposition causes lunar eclipse
the lists. (d) Syzygy conjunction occurs at the time of
List I (Hypothesis) List II perihelion only
(Propounder) 59. Which of the following statement(s) is/are
A. Planetesimal 1. Kober correct? [CDS 2016 (I)]
Hypothesis 1. The Earth is nearest to the Sun at
B. Thermal 2. Chamberline Perihelion, which generally occurs on 3rd
Contraction January.
Theory 2. The Earth is farthest away from the Sun
C. Grosynclinal 3. Daly at Perihelion, which generally occurs on
Orogen theory 4th July.
3. The Earth is farthest away from the Sun at
D. Hypothesis of 4. Jeffrey
Aphelion, which generally occurs on 4th
Sliding Continent
July.
Code: [CDS 2018 (I)] 4. The Earth is nearest to the Sun at
A B C D Aphelion, which generally occurs on 3rd
(a) 2 4 1 3 January.
(b) 2 1 4 3 Select the correct answer using the codes
(c) 3 1 4 2 given below:
(d) 3 4 1 2
(a) Only 1
56. Which one of the following statements with (b) 2 and 4
regard to the winter solstice is correct? (c) 1 and 3
[CDS 2017 (I)] (d) 1 and 2

PYQ Workbook 6
GENERAL GEOGRAPHY

60. Which one of the following statements (c) 2 and 3


about the atmosphere is correct? (d) All of these
[CDS 2015 (II)] 64. Match List I with List II and select the
(a) The atmosphere has definite upper limits, correct answer using the codes given below
but gradually thins until it becomes the lists :
imperceptible
(b) The atmosphere has no definite upper List I (Phenomenon) List II (Date)
limits, but gradually thins until it becomes A. Summer Solstice 1. 21st June
imperceptible B. Winter Solstice 2. 22nd December
(c) The atmosphere has no definite upper
limits, but gradually thickens until it C. Vernal Equinox 3. 23rd September
becomes imperceptible D. Autumnal Equinox 4. 21st march
(d) The atmosphere has definite upper limits, Code: [CDS 2014 (II)]
but gradually thickens until it becomes A B C D
imperceptible
(a) 1 4 2 3
61. If 82° 30’ East longitude (Allahabad) shows (b) 1 2 4 3
6:00 am of Sunday (local time), what would (c) 3 2 4 1
be the local time of Florida (USA) located at (d) 3 4 2 1
82° West longitude? [CDS 2015 (I)]
65. Which of the following statements regarding
(a) 6:58 pm of Saturday
(b) 7:02 pm of Sunday the duration of day and night is correct?
(c) 6:58 am of Sunday [CDS 2014 (I)]
(d) 7:02 am of Saturday (a) Difference is least near the Equator and
progressively increases away from it
62. The following item consist of two
(b) Difference is maximum at the Equator
statements, Statement I and Statement II.
and progressively decreases away from it
You have to examine these two statements
(c) Difference is least at the Tropics and
carefully and select the answers from these
items using the codes given below: progressively increases towards the
Equator and Poles
Statement I: Sidereal day is shorter than (d) Difference is maximum at the Tropics
Solar day.
and progressively decreases towards the
Statement II: The motion of the Earth in its orbit Equator and Poles
around the Sun is termed as revolution.
Codes [CDS 2015 (I)] 66. Which one of the following statements is
(a) Both the statements are individually true correct with reference to normal lapse rate?
and statement II is the correct explanation [(CAPF 2022)]
of statement I (a) Temperature is highest at ground level
(b) Both the statements are individually and decreases with increasing altitude.
true, but statement II is not the correct (b) Temperature is lowest at ground level and
explanation of statement I increases with increasing altitude.
(c) Statement I is true, but statement II is (c) Temperature remains stable with
false increasing altitude.
(d) Statement I is false, but statement II is (d) Temperature first increases with
true increasing altitude and gradually starts
63. The Earth without rotational movement decreasing.
would result into [CDS 2015 (I)] 67. Which one of the following geologic
1. no Sun-rise and Sun-set. time periods is associated with the first
2. no occurrence of day and night cycle. appearance of the marsupials?
3. only one season. [(CAPF 2022)]
Select the correct answer using the codes (a) Lower Cretaceous
given below: (b) Upper Cretaceous
(a) Only 1 (c) Early Jurassic
(b) 1 and 2 (d) Middle Jurassic

7 PYQ Workbook
GENERAL GEOGRAPHY

68. In order to find out the absolute location of 75. Which of the following statements relating
a place on the map, which of the following to the solar system is/are correct?
will be required? [CAPF 2021] 1. Venus and Uranus rotate opposite to the
(a) Latitude of the place alone direction of rotation of Earth.
(b) Longitude of the place alone 2. Venus is the nearest planet to Earth.
(c) Both latitude and longitude of the place Select the correct answer using the code given
(d) Neither latitude nor longitude of the place below. [CAPF 2016]
69. Which one of the following represents an (a) 1 only
inner planet? [CAPF 2019] (b) 2 only
(c) Both 1 and 2
(a) Planets between the Sun and the Earth
(d) Neither 1 nor 2
(b) Planets between the Sun and the belt of
asteroids 76. Intertropical Convergence Zone (ITCZ), a
(c) Planets near the Earth low-pressure zone, is located:
(d) Planets around the Sun [M.P.P.C.S. (Pre) 2018]
70. If the equatorial plane of Earth is not (a) between trade winds and westerlies belt
(b) between westerlies and polar winds belt
inclined to its orbit, then [CAPF 2019]
(c) nearby the poles
(a) duration of the year would be longer (d) at the equator
(b) summer would be warmer
(c) there would be no change of seasons 77. India was the part of ancient Gondwana
(d) winter would be longer land Supercontinent. It includes the
following landmass of the present:
71. Which one of the following processes is not [M.P.P.C.S. (Pre) 2008]
a part of long-wave radiation? (a) South America
[CAPF 2018] (b) Africa
(a) Conduction (c) Australia
(b) Scattering (d) All of above
(c) Convection
78. The “Sea of Tranquility” lies on-
(d) Radiation
[M.P.P.C.S. (Pre) 1995]
72. The lowest temperature is observed/ (a) Earth
recorded in which one of the following (b) Sun
layers of the atmosphere? (c) Jupiter
[CAPF 2018] (d) Moon
(a) Stratosphere 79. “Light Year” is the unit of-
(b) Mesosphere [M.P.P.C.S. (Pre) 1994]
(c) Thermosphere (a) Time
(d) Troposphere (b) Distance
73. Presence of ozone in the atmosphere is (c) Luminosity
important because it absorbs (d) None of these
[CAPF 2017] 80. The earth is nearest to the sun on-
(a) ultraviolet-A and ultraviolet-B radiations [Uttarakhand P.C.S. (Mains) 2006]
(b) ultraviolet-B radiations only (a) 3rd January
(c) infrared-B radiations only (b) 4th July
(d) outgoing ultraviolet-B radiations and (c) 22nd March
incoming ultraviolet-A radiations (d) 21st September
74. Which one of the following layers of 81. Zero-degree latitude and zero-degree
atmosphere has a high concentration of longitude lies in the-
ions? [CAPF 2016] [Uttarakhand P.C.S. (Pre) 2005]
(a) Stratosphere (a) Atlantic Ocean
(b) Exosphere (b) Arctic Ocean
(c) Thermosphere (c) Indian Ocean
(d) Troposphere (d) Pacific Ocean

PYQ Workbook 8
GENERAL GEOGRAPHY

82. When the length of day and night is equal (a) Asteroids
the Sunrays falls perpendicular at– (b) Comets
[Uttarakhand P.C.S. (Pre) 2002] (c) Planets
(a) North pole (d) Nebula
(b) Equator 89. Continents have drifted apart because of-
(c) South pole [53rd to 55th B.P.S.C. (Pre) 2011]
(d) Tropic of cancer (a) Volcanic eruptions
(b) Tectonic activities
83. The causes of Day and Night is-
(c) Folding and faulting of rocks
[R.A.S./R.T.S. (Pre) 2000] (d) All of the above
(a) Earth-rotation
90. Which planet is known as the ‘Evening
(b) Rotation of Earth on its axis
Star’? [44th B.P.S.C. (Pre) 2000]
(c) Tilt of Earth’s axis
(a) Mars
(d) Moon’s rotation
(b) Jupiter
84. The latitude at which ‘Annual Range of (c) Venus
Temperature’ remains minimum is- (d) Saturn
[R.A.S./R.T.S. (Pre) 1999] 91. The possibility of a desert on earth is more-
(a) Equator [41st B.P.S.C. (Pre) 1996]
(b) Tropic of Cancer (a) Nearby 0° latitude
(c) Tropic of Capricorn (b) Nearby 23° latitude
(d) North Pole Circle (c) Nearby 50° latitude
(d) Nearby 70° latitude
85. If the distance between the sun and the earth
is reduced by quarter to present distance, 92. Which of the following statements are
then probability of which one will increase- correct regarding the solar system?
[R.A.S./R.T.S. (Pre) 1993] I. Mercury is the hottest planet in the solar
(a) Earth would fall into the Sun. system
(b) Earth shall be burned. II. Ganymede, satellite of Saturn, is the largest
(c) The duration of our year will be decreased. satellite in the solar system
(d) Earth will fly into space. III. Neptune is surrounded by methane gas
rings of sub-zero temperature
86. Seasonal changes are caused by which one
IV. Phobos and Deimos are two satellites of
of the special characteristics of Earth’s
Mars
motion? [R.A.S./R.T.S. (Pre) 1992]
Code: [Chhattisgarh P.C.S. (Pre) 2020]
(a) Revolution around Sun
(a) Only I and II are correct
(b) Axial tilt by 23½ degree (b) Only II and III are correct
(c) Both (a) and (b) (c) Only III and IV are correct
(d) Rotation on its axis (d) I, II, III and IV all are correct
87. The country which has the longest north- 93. Which of the following phenomenon
south (latitudinal) extension of its territory happens in India when the sun-shines
is: [65th B.P.S.C. (Pre) 2019] vertically over the Tropic of Capricorn?
(a) Russia [Chhattisgarh P.C.S. (Pre) 2015]
(b) Chile (a) High pressure develops over North-West
(c) China India
(d) Brazil (b) Low pressure develops over North-West
(e) None of the above/More than one of the India
above (c) No change in pressure occurs over North-
West India
88. Which of the following does not belong to (d) Temperature increases in North-West
the solar system? India
[53rd to 55th B.P.S.C. (Pre) 2011] (e) None of these

9 PYQ Workbook
GENERAL GEOGRAPHY

94. Which of the following statements is true (c) Both 1 and 2 are false
regarding earth movement? (d) Only 1 is true
1. Earth rotates on its axis from west to east 95. Equinox, the two periods in the year when
resulting in seasonal changes. day and night are equal, occurs on:
2. Earth revolution around sun results in [U.P. Lower Sub. (Spl) (Pre) 2004]
occurrence of day and night on earth. (a) 21 March and 23 September
Code: [Jharkhand P.C.S. (Pre) 2021] (b) 22 February and 23 August
(a) Both 1 and 2 are true (c) 15 October and 23 April
(b) Only 2 is true (d) 22 July and 22 December

PYQ Workbook 10
GENERAL GEOGRAPHY

SOLUTIONS
miles to tens of miles wide, but as they orbit closer to the
1.1. UPSC CSE Previous Years’ Questions sun, they heat up and spew gases and dust into a glowing
1. Solution (b) head that can be larger than a planet. This material forms a
Exp) Option b is the correct answer. tail that stretches millions of miles.

In the northern hemisphere, the longest day of the year is Statement 2 is incorrect.
called the summer solstice. The asteroid belt is located roughly between the orbits of the
planets Jupiter and Mars.
The maximum amount of sunlight received by the Northern
Hemisphere during this time is usually on June 20, 21 or 22. Comets are not limited to any specific region; they are spread
all over the solar system.
The longest day of 2021 for those living north of the Equator
is June 21. This day is referred to as the summer solstice, the 4. Solution: (b)
longest day of the summer season. It occurs when the sun is Exp) Option b is the correct answer.
directly over the Tropic of Cancer, or more specifically right
Jupiter and Saturn lie between Mars and Uranus.
over 23.5 degree north latitude.
The International Astronomical Union defines a true
planet as a body that circles the sun without being some
other object’s satellite; is large enough to be rounded by its
own gravity (but not so big that it begins to undergo nuclear
fusion, like a star); and has “cleared its neighbourhood” of
most other orbiting bodies.
The order of the planets in the solar system, starting nearest
the sun and working outward is the following: Mercury,
Venus, Earth, Mars, Jupiter, Saturn, Uranus and Neptune.

2. Solution (d)
Exp) Option d is the correct answer.
Whether a given cloud will heat or cool the surface depends
on several factors, including the cloud’s altitude, its size, and
the make-up of the particles that form the cloud.
Statement 1 is incorrect: Low, thick clouds primarily
5. Solution: (b)
reflect solar radiation and cool the surface of the Earth.
Exp) Option b is the correct answer.
Statement 2 is incorrect: High, thin clouds primarily
transmit incoming solar radiation; at the same time, they Subrahmanyam Chandrasekhar, an Indian-born scientist is
trap some of the outgoing infrared radiation emitted by the most famous for coming up with the theory that explains the
death of the universe’s most massive stars. Chandrasekhar
Earth and radiate it back downward, thereby warming the
showed that it is impossible for a white dwarf star, which
surface of the Earth.
is supported solely by a degenerate gas of electrons, to
3. Solution: (b) be stable if its mass is greater than 1.44 times the mass
Exp) Option b is the correct answer. of the Sun. If such a star does not completely exhaust its
thermonuclear fuel, then this limiting mass may be slightly
Asteroids, sometimes called minor planets, are rocky, airless
larger.
remnants left over from the early formation of our solar
system about 4.6 billion years ago. All direct mass determinations of actual white dwarf stars
have resulted in masses less than the Chandrasekhar limit.
Comets are cosmic snowballs of frozen gases, rock, and dust A star that ends its nuclear-burning lifetime with a mass
that orbit the Sun. greater than the Chandrasekhar limit must become either a
Statement 1 and 3 are correct. neutron star or a black hole.
Asteroids are small, rocky objects that orbit the Sun. Comets Before Chandrasekhar, scientists assumed that all stars
are frozen leftovers from the formation of the solar system collapsed into white dwarfs when they died. He determined
composed of dust, rock and ices. They range from a few this isn’t so.

11 PYQ Workbook
GENERAL GEOGRAPHY

6. Solution (a) Statement 2 is correct: The earth’s magnetic pole in the


Exp) Option a is the correct answer. northern hemisphere is located on a peninsula in northern
Canada, near Ellesmere Island.
Statement 1 is correct: Albedo is the fraction of the incident
sunlight that is reflected from the surface of an object. Statement 3 is correct: Earth’s magnetic equator passes
When an object reflects most of the light that hits it, it looks through Thumba in South India, which is a rocket launching
bright and has a high albedo. For example, snow and ice station near Thiruvananthapuram.
have high albedos. When an object absorbs most of the light 10. Solution (c)
that hits it, it looks dark and has a low albedo. For example,
Exp) Option c is the correct answer.
asphalt and charcoal have low albedos. The albedo of an
object also depends on the wavelength of the light, as some Assertion (A) is true: The same face of the Moon always
objects may reflect more or less light at different parts of the faces the Earth because the Moon rotates on its axis at the
electromagnetic spectrum. same rate as it orbits the Earth. This is called synchronous
rotation or tidal locking.
Statement 2 is incorrect: Mercury, the closest planet to the
Sun, has a very low albedo, because it is mainly composed of Reason (R) is false: The Moon does not complete one
dark porous rock that reflects very little light. The albedo of rotation around its own axis in 23 and half days, but in
Mercury is 0.142, which is lower than the albedo of Earth, 27.322 days, which is a sidereal month. A sidereal month is
which is 0.367. This means that Mercury appears darker than the time it takes for the Moon to return to the same position
Earth when viewed with reflected light. The albedo of Earth in the sky relative to the stars. A synodic month, which is
is higher than Mercury’s because Earth has clouds, oceans, the time it takes for the Moon to return to the same phase
ice caps, and vegetation that reflect lighter than Mercury’s (such as full moon or new moon), is slightly longer, about
barren surface. 29.53 days, because the Earth-Moon system is also orbiting
the Sun.
7. Solution (d)
11. Solution (d)
Exp) Option d is the correct answer.
Exp) Option d is the correct answer.
Assertion (A) is false: The orbital period of a planet depends
on its distance from the Sun, not on its diameter. The farther Assertion (A) is false: Weight of an object depends on the
a planet is from the Sun, the longer it takes to complete one gravitational force exerted by the Earth, which varies with
orbit. Mars is about 1.5 times farther from the Sun than the distance from the center of the Earth. Moving from one
Earth, so it takes about 1.88 times longer to orbit the Sun place on Earth to another place of high latitude means
than Earth. moving closer to the poles, where the radius of the Earth
is smaller than at the equator. Therefore, the distance from
Reason (R) is true: The diameter of a planet affects its surface the center of the Earth is smaller, and the gravitational force
gravity, rotation speed, and atmospheric pressure, but not its is stronger. This means that the weight of an object increases
orbital period. Mars has a smaller diameter than Earth, so as it moves from the equator to the poles.
it has a lower surface gravity, a faster rotation speed, and a
Reason (R) is true: Earth is not a perfect sphere, but an
thinner atmosphere than Earth.
oblate spheroid. This means that it is slightly flattened at the
8. Solution (a) poles and bulges at the equator due to the centrifugal force
Exp) Option a is the correct answer. caused by its rotation. The difference between the equatorial
radius and the polar radius of the Earth is about 21 km.
The existence of human life on Venus is highly improbable
because of the extreme conditions on the surface of the 12. Solution (b)
planet, which are largely caused by the high level of carbon Exp) Option b is the correct answer.
dioxide in its atmosphere. Carbon dioxide is a greenhouse
Jupiter takes the longest time to complete one revolution
gas that traps heat and raises the temperature of the planet.
around the Sun among the four planets given in the options.
Venus has about 154,000 times more carbon dioxide in its
The orbital period of Jupiter is about 4,333 days or 11.86
atmosphere than Earth, producing a runaway greenhouse
years i n Earth days. This means that Jupiter takes about 12
effect and a surface temperature hot enough to melt lead.
times longer than Earth to orbit the Sun once.’
The atmospheric pressure on Venus is also 93 times that of
Earth, making it very difficult for any living organism to Important Tips
survive. Therefore, the high level of carbon dioxide in Venus’ • The orbital period of a planet depends on its distance
atmosphere is the correct explanation for why human life is from the Sun and its orbital speed.
unlikely to exist there.
• The farther a planet is from the Sun, the longer its
9. Solution (b) orbital path and the slower its orbital speed.
Exp) Option b is the correct answer. • Jupiter is the fifth planet from the Sun and has an
average distance of about 778 million kilometers,
Statement 1 is incorrect: The axis of the earth’s magnetic
which is about 5.2 times farther than Earth’s distance
field is not inclined at 23 and a half degrees to the geographic
from the Sun.
axis of the earth, but at about 11 degrees.

PYQ Workbook 12
GENERAL GEOGRAPHY

13. Solution (a) Pluto is classified as a dwarf planet, not a planet. However,
Exp) Option a is the correct answer. Pluto has an elliptical orbit that sometimes brings it closer to
the Sun than Neptune. The last time this happened was from
Option a is correct: The Earth is the densest of all the
1979 to 1999, when Pluto was the farthest planet from the
planets in our solar system. Earth has a mean density of
Sun for most of that period. Therefore, the farthest planet
5.513 g/cm3, which is higher than any other planet in the
from the Sun changes depending on the position of Pluto
solar system.
and Neptune in their orbits.
Option b is incorrect: The predominant element in the
composition of Earth is not silicon, but iron. Iron makes up 17. Solution (a)
about 35% of Earth’s mass, followed by oxygen (30%), silicon Exp) Option a is the correct answer.
(15%), and magnesium (13%).
If the stars are seen to rise perpendicular to the horizon
Option c is incorrect: The Sun contains not 75 percent, by an observer, it means that he is located at the Earth’s
but about 99.86% of the mass of the solar system. The Sun’s
equator. This is because the Earth’s equator is the only
mass is about 330,000 times that of Earth, and it accounts for
location where the celestial equator (the projection of the
almost all of the mass in the solar system.
Earth’s equator onto the celestial sphere) is perpendicular to
Option d is incorrect: The diameter of the Sun is not 190 the horizon
times, but about 109 times that of the Earth. The Sun’s
At the Earth’s poles, the stars do not rise or set, but circle
diameter is about 864,938 miles (1.392 million km), while
Earth’s diameter is about 7,917 miles (12,742 km). around the zenith (the point directly above the observer).
At any other latitude, the stars rise and set at an angle to the
14. Solution (c) horizon, depending on the declination (the angular distance
Exp) Option c is the correct answer. of a star from the celestial equator) and the observer’s
A sun halo is a circular rainbow-like ring that sometimes position.
appears around the sun when there are thin, high-altitude 18. Solution (b)
clouds in the sky. These clouds, called cirrus clouds, are
composed of tiny ice crystals that act like prisms and split Exp) Option b is the correct answer.
the sunlight into different colors. The angle of refraction When crossing the International Date Line (IDL), the change
is about 22 degrees, which determines the radius of the sun in the date and time depends on the direction of travel. If a
halo. ship is moving from the eastern side of the Aleutian Islands
Sun halos are not very common, but they can be seen from towards Dutch Harbour (westward), it will gain a day when
anywhere in the world, depending on the weather conditions it crosses the 180° meridian. So, if the ship crosses the
and the position of the sun. They are often considered as a sign meridian on 1st January 1999 at 23:30 hr, after one-hour
of impending rain or snow, as cirrus clouds usually precede journey, it will be recorded as January 2, 00:30 hr in the
a storm front. Sun halos are also sometimes accompanied captain’s diary. The ship moves into the next day as it travels
by other optical phenomena, such as sun dogs, which are westward.
bright spots on either side of the halo, or a circumzenithal
Important Tips
arc, which is an inverted rainbow above the halo.
Time and date across IDL:
15. Solution (d)
• Crossing the IDL results in either gaining or losing a
Exp) Option d is the correct answer. day depending on the direction of travel.
Assertion (A) is false: Artificial satellites are launched from • Traveling westward across the IDL leads to gaining a
the earth in the eastward direction. This statement is true, day, while traveling eastward results in losing a day.
as most satellites are launched in the eastward direction to
take advantage of the Earth’s rotation and save fuel. However, • Example: If someone travels eastward from Wake
there are some exceptions, such as polar satellites, which Island to the Hawaiian Islands on June 25, they will
are launched in the northward or southward direction. shift their calendar back to June 24 upon crossing the
IDL. Traveling westward in the opposite direction
Reason (R) is true: The earth rotates from west to east and
would mean arriving at Wake Island on June 26.
so the Satellite attains the escape velocity. This statement
is also true, as the Earth’s rotation provides an initial boost • When moving westward from the IDL, travelers gain
to the satellite’s velocity, which helps it reach the escape one hour for every 15° of longitude crossed.
velocity. However, this is not the only factor that determines • Travelers moving eastward lose one hour for every 15°
the escape velocity, which also depends on the mass of the of longitude crossed.
Earth, the distance of the satellite from the Earth’s center,
• If travelers journey westward without adjusting
and the direction of the launch.
their time by one hour for every 15° meridian, they
16. Solution (c) will find that they have gained an extra day when
Exp) Option c is the correct answer. returning to their starting point.

13 PYQ Workbook
GENERAL GEOGRAPHY

19. Solution (d) The Sun is the nearest star to Earth, as it is the central star
Exp) Option d is the correct answer. of our solar system. The Sun is about 8.5 light-minutes away
from Earth, which means that it takes 8.5 minutes for the
The scholar who suggested that the Earth originated from
light from the Sun to reach us. The Sun is a yellow dwarf star
gases and dust particles is Otto Schmidt. He proposed the
that provides heat and light for life on Earth.
Interstellar Dust Hypothesis in 1943 to explain the complex
problems of the origin and characteristics of the solar Important Tips
system and Earth. According to this hypothesis proposed, • Polaris is a yellow supergiant star that is about 433
the sun’s gravitational attraction initially drew gases and light-years away from Earth. It is also known as the
dust particles from the universe, forming a cloud and dust North Star, as it lies very close to the north celestial
sheet revolving around the sun. Over time, these particles pole.
consolidated, forming embryos of planets, which eventually
• Alpha Centauri is a triple star system that is about
became asteroids and then planets. Satellites also formed
from remaining scattered material around the planets. This 4.37 light-years away from Earth. It consists of two
hypothesis explains the complex problems of the origin and sun-like stars, Alpha Centauri A and B, and a red dwarf
characteristics of the solar system and the Earth. star, Proxima Centauri. Proxima Centauri is slightly
closer to Earth, at 4.24 light-years away, and may have
20. Solution (a) a rocky planet orbiting it
Exp) Option a is the correct answer. • Sirius is a binary star system that is about 8.6 light-
One astronomical unit (AU) is the average distance from years away from Earth. It consists of a bright white
the Earth to the Sun, approximately 93 million miles (150 main sequence star, Sirius A, and a faint white dwarf
million km), or 8 light-minutes. One astronomical unit is a star, Sirius B. Sirius A is the brightest star in the night
convenient unit of length for measuring distances within the sky.
Solar System or around other stars. It is also a fundamental
23. Solution (c)
component in the definition of another unit of astronomical
length, the parsec. Exp) Option c is the correct answer.
One of the most important conditions for the presence of
21. Solution (c) life on Mars is the availability of liquid water, which is
Exp) Option c is the correct answer. essential for all known forms of life. Orbiting spacecraft
Statement 1 is correct: Asteroids are rocky debris of varying and mobile laboratories on the Martian surface have showed
sizes orbiting the sun. They are also known as minor planets that important prerequisites for life still exist today – an
or planetoids. They are remnants of the early formation of atmosphere, elements such as carbon, hydrogen, oxygen,
the Solar System, when planets were formed from collisions nitrogen, Sulphur and phosphorus, and even water, at least
of smaller bodies. Some asteroids have been visited by in the form of ice. The occurrence of ice caps and frozen
spacecraft, such as NASA’s Dawn mission, which explored water on Mars indicates that there may be liquid water under
the two largest asteroids, Ceres and Vesta. the surface or during warmer seasons, which could provide a
Statement 2 is correct: Most of the asteroids are small but suitable environment for life.
some have diameter as large as 1000 km. The largest asteroid
24. Solution (d)
in the belt is Ceres, which is about 950 km in diameter.
Exp) Option d is the correct answer.
Statement 3 is incorrect: The orbit of asteroids lies between
the orbits of Mars and Jupiter, not Jupiter and Saturn. The Asteroids are a group of small pieces of rock revolving
asteroid belt is a torus-shaped region in the Solar System, around the sun between the orbits of Mars and Jupiter.
centered on the Sun and roughly spanning the space between They are also called minor planets or planetoids, and they
the orbits of the planets Jupiter and Mars. form a region in the Solar System known as the asteroid belt.
The asteroid belt contains millions of asteroids, ranging in
Important Tips size from boulders to hundreds of kilometers in diameter. The
• Some asteroids follow the same path around the Sun largest asteroid in the belt is Ceres, which is also classified
as planets. These are called co-orbital asteroids. ‘Earth as a dwarf planet. Asteroids are made up of different types
has a few co-orbital asteroids, such as 3753 Cruithne of materials, such as carbon, silicate, or metal, depending on
and 2010 TK7. their origin and history.
• Some asteroids also cross the orbits of planets. These
25. Solution (c)
are called near-Earth objects (NEOs).
Exp) Option c is the correct answer.
• NASA has sent spacecraft to fly by, orbit, and even
land on asteroids. Some of these missions are NEAR The radiation emitted by the Sun exerts a radial pressure
Shoemaker, Dawn, OSIRIS-REx, and DART. on the comet throwing its tail away from the Sun. This is
because the comet’s tail is composed of dust and gas that
22. Solution (c) are released from the comet’s nucleus by the Sun’s heat and
Exp) Option c is the correct answer. driven away by sunlight (solar radiation pressure) and the

PYQ Workbook 14
GENERAL GEOGRAPHY

solar wind. The solar wind is a stream of charged particles • Japan (UTC+09:00) - Standard Time in Japan is 9
that flows outward from the Sun at high speeds. The solar hours ahead of GMT.
wind interacts with the gas in the comet’s tail, causing it to • Iraq (UTC+03:00) - Standard Time in Iraq is 3 hours
glow and point along the streamlines of the solar wind. The
ahead of GMT.
tail always points away from the Sun, whether the comet is
approaching or receding from it. • Greece (UTC+02:00) - Standard Time in Greece is 2
hours ahead of GMT.
26. Solution (c)
• Cuba (UTC-05:00) - Standard Time in Cuba is 5
Exp) Option c is the correct answer. hours behind of GMT.
As Earth rotates from west to east, the International Date
• Costa Rica (UTC-06:00) - Standard Time in Costa
Line (IDL) is 6 hours 30 minutes ahead of Indian Standard
Rica is 6 hours behind of GMT.
Time (IST). So, when the time at IDL is 12 PM, the time at
IST would be 5:30 AM. Important Tips
Now if the earth starts rotating from east to west, IDL will be Greenwich Mean Time (GMT):
6 hours 30 minutes behind the IST. Thus, when the time at
• GMT is the mean solar time at the Royal Observatory
IDL is 12 PM, the time at IST would be at 6:30 PM.
in Greenwich, London, counted from midnight.
27. Solution (b) • GMT has been calculated in different ways in the past,
Exp) Option b is the correct answer. including being calculated from noon. This historical
A person in spacecraft situated in the middle of earth and variation means that it cannot specify a particular time
Sun will see that the sky is black, and the stars do not without context.
twinkle, but the temperature outside the spacecraft is not • GMT is also used as one of the names for the time
necessarily more than that of the earth surface.
zone UTC+00:00. In the United Kingdom, it is the
• The sky is black because there is no atmosphere to scatter basis for civil time.
the sunlight and make it appear blue.
• English speakers often use GMT interchangeably
• The stars do not twinkle because there is no atmospheric with Coordinated Universal Time (UTC).
turbulence to distort their light.
• For navigation purposes, GMT is considered equivalent
• The temperature outside the spacecraft depends on several
to UT1, which is the modern form of mean solar time at
factors, such as the distance from the Sun, the angle of
0° longitude. However, there can be slight differences
incidence of the sunlight, the albedo of the spacecraft,
and the presence of any shielding or insulation. (up to 0.9 seconds) between GMT and UTC. Thus,
GMT should not be used for applications requiring
28. Solution (b) high precision due to variations caused by Earth’s
Exp) Option b is the correct answer. elliptical orbit and axial tilt.
A meteor is a piece of matter which has entered the earth’s • Noon GMT is rarely the exact moment when the
atmosphere from outer space. This is because a meteor is Sun crosses the Greenwich Meridian and reaches its
a streak of light in the sky caused by a meteoroid crashing highest point in the sky due to Earth’s uneven angular
through Earth’s atmosphere. velocity. This event can occur up to 16 minutes before
Important Tips or after noon GMT, and the annual average of these
• Meteoroids are lumps of rock or iron that orbit the sun. moments is what GMT represents, explaining the
The other options are incorrect because they do not “mean” in “Greenwich Mean Time.”
match the definition of a meteor.
30. Solution (d)
• A fast-moving star is a star that moves rapidly across
Exp) Option d is the correct answer.
the sky, but it is not a meteor.
The colour of a star indicates its temperature, which is
• A part of the stellar system is a vague term that
could refer to any object in the universe, but it does related to the wavelength of light it emits. The shorter the
not specify that it enters the earth’s atmosphere and wavelength, the hotter the star. Blue-white stars are much
produces a bright light. hotter than the Sun, whereas red stars are cooler. The
• A tailless comet is a comet that does not have a visible colour of a star can also suggest its age, as older stars tend to
tail, but it is still a comet and not a meteor. be redder than younger ones. The distance from the Sun or
the Earth does not affect the colour of a star, but it may affect
29. Solution (a) its brightness or apparent size. The lighting or glow of a star
Exp) Option a is the correct answer. is not a well-defined term, but it may refer to the luminosity
Countries according to their standard time from ahead of or magnitude of a star, which are also influenced by other
GMT to behind GMT are as follows: factors besides the colour.

15 PYQ Workbook
GENERAL GEOGRAPHY

31. Solution (d) • In terms of the usual way these geographic coordinates
Exp) Option d is the correct answer. are given, this transformation can be expressed
The opposite end of the Earth from 30° North Latitude, 50° symbolically as
East Longitude is 30° South Latitude, 130° West Longitude. • x° N/S y° E/W ↦ x° S/N (180 − y) ° W/E,
When a plane aims to reach the opposite end of its starting • That is, for the latitude (the north–south coordinate)
point, it is essentially trying to reach the antipodal point of the magnitude of the angle remains the same, but N
its initial location. The antipode of any location on Earth is changed to S and vice versa, and for the longitude
is the point that is diametrically opposite to it on Earth’s (the East/West coordinate) the angle is replaced by its
surface, and a straight line connecting the two points would supplementary angle while E is exchanged for W.
pass through the center of the Earth.
32. Solution (c)
To find the antipode of a given location on Earth, you
need to consider the geographic coordinates (latitude and Exp) Option c is the correct answer.
longitude). If the starting point is at 30 degrees northern To determine the degree of longitude where it will be 6:30
latitude and 50 degrees eastern longitude, then the antipodal a.m. when it is noon along 82 degrees 30’ E longitude, we can
point would have: follow these steps:
• The latitude would change to its opposite, so 30 degrees • Earth completes a full rotation of 360 degrees in 24 hours,
northern latitude becomes 30 degrees southern which means that each hour corresponds to 15 degrees of
latitude. longitude.
• The longitude would change to its opposite by adding or • Consequently, a location situated 15 degrees west of 82°
subtracting 180 degrees, so 50 degrees eastern longitude 30’ E longitude will be one hour behind in time, while
becomes 50 degrees - 180 degrees = -130 degrees a location 15 degrees east of this longitude will be one
western longitude. hour ahead.
• Given that 6:30 a.m. is 5 hours and 30 minutes behind
12 noon, we can calculate that this time difference
corresponds to 82 degrees and 30 minutes of longitude.
Therefore, the place where it is 6:30 a.m. is located 82
degrees and 30 minutes west of 82° 30’ E longitude. This is
equivalent to 0° East or West longitude which is precisely
on the Prime Meridian.

33. Solution (b)


Exp) Option b is the correct answer.
A brittle star is not an astronomical object, but a marine
animal that belongs to the class Ophiuroidea, closely
related to starfish. A brittle star has a central disk and five
long, slender arms that can break off easily. Brittle stars can
be found in many marine habitats, from shallow waters to
deep sea.
Important Tips
• A pulsar is a highly magnetized rotating neutron star
that emits beams of electromagnetic radiation.
• A black hole is a region of space-time where gravity
is so strong that nothing, not even light, can escape
from it.
• A quasar is a very luminous and active galactic nucleus
powered by a supermassive black hole.
Important Tips
Mathematical description of Antipodes: 34. Solution (a)
• If the geographic coordinates (latitude and longitude) Exp) Option a is the correct answer.
of a point on the Earth’s surface are (φ, θ), then A total solar eclipse occurs when the Moon completely covers
the coordinates of the antipodal point are (−φ, θ ± the Sun’s disk, creating a dark shadow on the Earth called
180°). This relation holds true whether the Earth is the umbra. However, the umbra is very narrow, typically
approximated as a perfect sphere or as a reference only about 150 km (90 miles) wide, and it only covers a
ellipsoid. small area of the Earth at a time. Therefore, only people who

PYQ Workbook 16
GENERAL GEOGRAPHY

are located within this narrow path of totality can see the
total eclipse, while others may see a partial eclipse or no
eclipse at all.

35. Solution (d)


Exp) Option d is the correct answer.
In winters, the Earth’s axis becomes more inclined towards
the Sun. This is because the Earth’s axis is tilted by 23.5
degrees from the perpendicular to the plane of its orbit
around the Sun. This tilt causes different parts of the Earth
to receive different amounts of sunlight throughout the year,
resulting in the seasons.
In summer, Delhi is tilted towards the Sun, so it receives
more direct and intense sunlight than in winter, when it is
tilted away from the Sun. This makes Delhi hotter in summer
than in winter. The distance between the Earth and the Sun
does not have a significant effect on the seasons, as it varies
only slightly throughout the year.

1.2. Other Examination Previous Years’


Questions
36. Solution (d)
Exp) Option d is the correct answer.
A black hole is a region of space where gravity is so strong
that nothing, including light, can escape. A black hole can
be formed by the death of a massive star, when its core
collapses under its own weight and creates a singularity, a
38. Solution (d)
point of infinite density and zero volume.
Exp) Option d is the correct answer.
37. Solution (c)
Mars is not the slowest moving planet in the Solar System,
Exp) Option c is the correct answer. but the fourth slowest. The slowest moving planet in the
The chronological order of the origin of the four events is as Solar System is Neptune, which takes 165 Earth years to
follows (This order is based on the fossil record): revolve around the Sun. Mars takes about 1.88 Earth years
to complete one orbit, which is faster than Jupiter, Saturn,
• Shelled animals (520 million years ago): The oldest
Uranus and Neptune. The orbital speed of Mars is 24.077
known shelled animals are trilobites, which date back to
km/s, or 53,858 miles per hour.
the Cambrian period, around 520 million years ago.
Important Tips
• First insects (410 million years ago): Rhyniognatha
hirsti has been dated to nearly 410 million years ago, • Jupiter is the fifth planet from the Sun and the largest
in the Solar System. It has a diameter of 142,800
making it the oldest known definitive insect specimen in
kilometers, which is about 11 times the diameter of
existence. Earth. It is composed mainly of hydrogen and helium
• First reptiles (320 million years ago): The origin of and has a powerful magnetic field and a fast rotation. It
the reptiles lies about 320–310 million years ago, in has at least 79 moons, including four large ones called
Io, Europa, Ganymede and Callisto.
the swamps of the late Carboniferous period, when
• Mercury is the closest planet to the Sun and the
the first reptiles evolved from advanced reptiliomorph
smallest in the Solar System. It has a diameter of 4,879
labyrinthodonts.
kilometers, which is only 1.4 times larger than the
• First mammals (225 million years ago): The oldest Moon. It is a rocky planet with a large iron core and
known mammals are small, shrew-like animals that lived a thin atmosphere. It has no moons and rotates very
225 million years ago. slowly, taking 243 Earth days to complete one spin.

17 PYQ Workbook
GENERAL GEOGRAPHY

• Venus is the second closest planet to the Sun and the 42. Solution (b)
brightest planet in the Solar System. It can be seen with Exp) Option b is the correct answer.
the unaided eye from Earth and is sometimes called There is no atmosphere on the Moon. Sound waves need a
the evening star or the morning star because of its medium to travel, such as air, water, or solid objects. On
consistent presence in the sky. It is similar in size and the Moon, there is no air or other medium to carry the sound
shape to Earth but has a very different environment. It waves, so they cannot be heard. Astronauts use radio waves
has a thick atmosphere of carbon dioxide and sulfuric to communicate with each other on the surface of the Moon,
acid, which reflects a lot of sunlight and creates a because radio waves can travel through vacuum.
greenhouse effect.
43. Solution (a)
39. Solution (a)
Exp) Option a is the correct answer.
Exp) Option a is the correct answer. Assertion (A) is correct: It states that a black hole is an
Two full moons occur in the same month. This is the more astronomical entity that cannot be seen by a telescope.
popular definition of a Blue Moon, which is also known as This is true because black holes do not emit any light or other
a monthly Blue Moon. It happens when there are two full forms of electromagnetic radiation. They are essentially
moons in a calendar month, which occurs about once every invisible to conventional telescopes.
2.7 years. The other definition of a Blue Moon is a seasonal Reason (R) is correct: It states that the gravitational field on
Blue Moon, which is the third full moon in an astronomical a black hole is so strong that it does not allow even light to
season that has four full moons. This happens about once escape. This is also true. According to Einstein’s theory of
every 2.5 years. general relativity, a black hole is formed when a massive
star collapses under its own gravity, creating a region in
40. Solution (b) space where the gravitational pull is so intense that nothing,
Exp) Option b is the correct answer. not even light, can escape from it.
Solar energy is the energy that is produced by the Sun The reason (R) provides the correct explanation for why a
through a process called nuclear fusion. Nuclear fusion is a black hole cannot be seen through a telescope. The intense
reaction in which two or more atomic nuclei fuse together to gravitational field of a black hole prevents any form of
form a heavier nucleus, releasing a huge amount of energy in electromagnetic radiation from escaping, making it appear
the form of electromagnetic radiation. completely dark and effectively invisible.

In the Sun, the nuclear fusion process involves hydrogen 44. Solution (b)
atoms fusing into helium atoms in the core. This process, Exp) Option b is the correct answer.
known as the proton-proton chain reaction, converts about 4 Mercury is the planet that takes the least time to orbit the
million tons of matter into energy every second. The energy Sun, because it is the closest planet to the Sun and has the
travels outward from the core through the radiative and highest orbital speed. Mercury’s orbital period is 87.97 days
convective zones of the Sun, and then reaches the surface as (0.2 years), which is much shorter than any other planet’s
visible light and other forms of electromagnetic radiation. orbital period.
41. Solution (b) Important Tips
Exp) Option b is the correct answer. • Pluto is not a planet, but a dwarf planet, and it takes
the longest time to orbit the Sun, because it is the
When any object is taken from the Earth to the Moon,
farthest object in the Solar System and has the lowest
its weight decreases. Weight is the force of gravity acting
orbital speed. Pluto’s orbital period is 248.09 Earth
on an object. The weight of an object depends on the mass
years, which is much longer than any other planet’s or
and the distance from the center of the object that exerts
dwarf planet’s orbital period.
the gravitational force. The moon has a much smaller mass
• Earth is the third planet from the Sun, and it takes one
and radius than the Earth, so its gravitational force is much
year to orbit the Sun, which is the definition of a year.
weaker. Therefore, any object taken from the Earth to the
Earth’s orbital period is 365.26 days (1 year), which is
moon will experience a decrease in weight, but not in mass. longer than Mercury’s and Venus’ orbital periods, but
You can use this formula to calculate your weight on the shorter than Mars’ and the other outer planets’ orbital
moon: periods
Wmoon = (Wearth/9.81) ∗ 1.62 • Saturn is the sixth planet from the Sun, and it takes
where Wmoon is your weight on the moon, Wearth is your almost 30 years to orbit the Sun, because it is one of
weight on Earth, 9.81 is the gravitational acceleration on the outer planets and has a lower orbital speed than
Earth, and 1.62 is the gravitational acceleration on the moon. the inner planets. Saturn’s orbital period is 10,759 days
For example, if the weight is 70 kg on Earth, you would weigh (29.5 years), which is longer than any other planet’s
orbital period except Neptune’s
about 11.5 kg on the moon.

PYQ Workbook 18
GENERAL GEOGRAPHY

45. Solution (a) 47. Solution (a)


Exp) Option a is the correct answer. Exp) Option a is the correct answer.
The Milky Way is classified as a spiral galaxy, which means The Chandrasekhar limit is the maximum mass that a
that it has a flattened disk shape with spiral arms that extend white dwarf star can have before it collapses under its own
from the central bulge. The Milky Way also has a bar-like gravity and becomes a neutron star or a black hole. The
structure in the middle, which makes it a barred spiral galaxy. Chandrasekhar limit is about 1.4 times the mass of the Sun,
Spiral galaxies are characterized by having a lot of gas and or about 2.8 x 10^30 kg. The Chandrasekhar limit was named
dust, which allows them to form new stars and have a blue after the Indian American astrophysicist Subrahmanyan
color. Elliptical galaxies are more spherical and have less Chandrasekhar, who calculated it in 1930.
gas and dust, which makes them older and redder. Irregular
Important Tips
galaxies have no definite shape and are often the result of
collisions or mergers between other galaxies. • The Eddington limit is the maximum luminosity
that a star can have before it is blown apart by its own
46. Solution (d) radiation pressure
Exp) Option d is the correct answer. • The Hoyle limit is the minimum mass that a star must
The Prime Meridian, at 0 degrees longitude, and 180 have to ignite nuclear fusion in its core
degrees longitude together form a great circle on the globe. • The Fowler limit is the maximum mass that a star can
A Great Circle is any circle that circumnavigates the Earth have before it becomes unstable and pulsates violently.
and passes through the center of the Earth. A great circle
48. Solution (d)
always divides the Earth in half; thus, the Equator is a great
circle (but no other latitudes) and all lines of longitude are Exp) Option d is the correct answer.
great circles. A Ball thrown outside from an artificial satellite revolving
around the Earth will revolve around the Earth in the same
orbit with the same time period as satellite. This is because
the ball will have the same orbital speed as the satellite,
and thus will behave as a satellite itself. The ball will not be
affected by the sun, the moon, or the earth’s gravity, as long
as it is in a stable orbit.

49. Solution (b)


Exp) Option b is the correct answer.
Planets shine though they are non-luminous bodies.
Planets do not produce their own light, but they reflect the
light from the Sun or other stars. The apparent brightness of
a planet depends on its distance from the Sun, its distance
from the Earth, and its albedo (the fraction of light that it
reflects). The planets also have phases, like the Moon, which
affect how much of their illuminated surface is visible from
Earth.

50. Solution (b)


Exp) Option b is the correct answer.
A solar eclipse occurs when the Moon passes between the
Earth and the Sun, blocking some or all of the Sun’s light from
Important Tips
reaching the Earth. A solar eclipse can only happen during
• Circles which do not pass through the center of the a new moon, which is when the Moon is directly between the
earth are small circles. All the parallels of latitudes are Earth and the Sun and not visible from our planet
small circles.
However, not every new moon produces a solar eclipse,
• Navigators use great circles to find the shortest distance
because the Moon’s orbit is tilted at about 5 degrees to the
between any two points on the earth’s surface.
Earth’s orbit. This means that the Moon’s shadow usually
• Great circles are used by meteorologists to determine
misses the Earth, unless the new moon coincides with one of
the climate and weather conditions in a region.
the two points where the orbits intersect, called nodes. When
• Great circle routes are especially important for places this happens, a solar eclipse can occur somewhere on Earth,
that are on the opposite sides of the globe. depending on the alignment of the Sun, Moon and Earth.

19 PYQ Workbook
GENERAL GEOGRAPHY

Important Tips • Spica: Spica is a bright star in the constellation Virgo.


Different types of solar eclipses: It is a relatively distant star compared to Proxima
Centauri and is not among the closest stars to Earth.
• A total solar eclipse occurs when the Moon completely
covers the Sun, creating a brief period of darkness 55. Solution (a)
called totality. Exp) Option a is the correct answer.
• An annular solar eclipse occurs when the Moon is too • Planetesimal Hypothesis - Propounder: Thomas
far away to cover the Sun completely, leaving a ring of Chrowder Chamberlin
sunlight around the Moon’s edge.
• This hypothesis suggests that the formation of the
• A partial solar eclipse occurs when the Moon covers solar system occurred from the aggregation of small
only part of the Sun’s disk solid particles called planetesimals.

51. Solution (b) • Thermal Contraction Theory - Propounder: Harold


Jeffreys
Exp) Option b is the correct answer
• The Thermal Contraction Theory proposes that the
Gas planets are planets that are composed primarily of Earth and other celestial bodies formed from the
lighter ice, liquids and gases. They have no solid surface, and contraction and cooling of a hot, initially molten
their atmospheres are very thick. The four gas planets in mass.
our solar system are Jupiter, Saturn, Uranus, and Neptune.
• Geosynclinal Orogen theory - Propounder: Antoni
52. Solution (d) Kober
Exp) Option d is the correct answer. • The Geosynclinal Orogen theory postulates that
major mountain-building events, known as orogenies,
The ozone layer, which absorbs ultraviolet (UV) radiation,
are a result of the accumulation and subsequent
is located in the stratosphere, the second layer of the Earth’s
compression of sedimentary rocks in geosynclines
atmosphere between the troposphere and the mesosphere. (depression in the Earth’s crust).
The ozone layer contains a high concentration of ozone
• Hypothesis of Sliding Continent - Propounder:
(O3) molecules and acts as a shield against the Sun’s
Reginald Daly
harmful UV radiation. By absorbing and filtering out a
significant portion of UV rays, the ozone layer protects • The Hypothesis of Sliding Continent, also known
living organisms from the detrimental effects of excessive as the Continental Drift theory, proposed that the
UV exposure, including health issues like skin cancer. continents were once connected and have since
drifted apart over geological time.
53. Solution (a)
56. Solution (c)
Exp) Option a is the correct answer.
Exp) Option c is the correct answer.
The four planets closest to the Sun are called terrestrial
During the winter solstice, which typically occurs around
planets. Terrestrial planets, also known as inner planets
December 21st in the Northern Hemisphere, the North Pole
or rocky planets, are characterized by their solid, rocky
experiences 24 hours of darkness. This happens because the
surfaces. The four terrestrial planets in our solar system tilt of the Earth’s axis is oriented away from the Sun, causing
are Mercury, Venus, Earth, and Mars. These planets are the Sun’s rays to be at their lowest angle and the North Pole
composed primarily of rock and metal and have relatively to be tilted away from the Sun. As a result, the North Pole
high densities. remains in continuous darkness for the entire day.
54. Solution (d) 57. Solution (d)
Exp) Option d is the correct answer. Exp) Option d is the correct answer.
Proxima Centauri is the star that is nearest to the Earth. Statement 1 is correct- In the stratosphere, temperature
Proxima Centauri is a red dwarf star located in the Alpha increases with altitude due to the presence of the ozone
Centauri star system, which is the closest star system to our layer, which absorbs ultraviolet radiation from the Sun,
solar system. It is approximately 4.24 light-years away from leading to the heating of the surrounding air.
Earth, making it the closest known star to us. Statement 2 is correct- The mesosphere is the third layer of
Important Tips the Earth’s atmosphere, located above the stratosphere. As
altitude increases, the density of air molecules decreases,
• Sirius: Sirius is a bright star in the constellation Canis
resulting in a decrease in temperature.
Major, and it is the brightest star visible from Earth.
Statement 3 is correct- The lowest temperature of the
• Arcturus: Arcturus is a bright star in the constellation
atmosphere is recorded in the upper part of the mesosphere.
Boötes. It is a relatively nearby star in astronomical Known as the mesopause, this is the coldest region of the
terms, but it is not as close to Earth as Proxima Earth’s atmosphere with temperatures reaching as low as -90
Centauri. degrees Celsius (-130 degrees Fahrenheit).

PYQ Workbook 20
GENERAL GEOGRAPHY

Statement 4 is correct- The tropopause is the boundary eventually becomes indistinguishable from the surrounding
between the troposphere (the lowest layer of the atmosphere) space.
and the stratosphere. In this region, the temperature remains
61. Solution (a)
relatively constant with increasing altitude, resulting in an
isothermal zone. Exp) Option a is the correct answer.
To determine the local time in Florida (USA) located at 82°
Important Tips
West longitude, we need to account for the time difference
Other layers of Earth Atmosphere: between the two locations. The time difference between
• Troposphere: The lowest layer where weather each degree of longitude is approximately 4 minutes.
phenomena occur and temperature generally • Since Allahabad (82° 30’ East longitude) is ahead of
decreases with increasing altitude. Florida (82° West longitude) by 164 degrees (82° 30’ +
• Thermosphere: A layer with high temperatures due 82°), we multiply the time difference (4 minutes) by 164.
to the absorption of extreme ultraviolet radiation, but • 164 degrees * 4 minutes = 656 minutes
it would feel cold to humans due to low density. • Now, we convert 656 minutes into hours by dividing it
• Exosphere: The outermost layer that gradually by 60:
transitions into space, consisting of a very thin • 656 minutes / 60 = 10.93 hours
atmosphere and the location where satellites orbit
• Since the time in Allahabad is 6:00 am on Sunday, by
the Earth.
subtracting 10.93 hours from it, we get the local time in
58. Solution (d) Florida.

Exp) Option d is the correct answer • 6:00 am - 10.93 hours = approximately 7:02 pm of
Saturday
Syzygy conjunction can occur at any time, not just at the
Therefore, the correct answer is (a) 6:58 pm of Saturday.
time of perihelion
Important Tips 62. Solution (b)

• Syzygy refers to the alignment of three celestial Exp) Option b is the correct answer.
bodies, such as the Sun, Earth, and Moon, in a Both the statements are individually true, but statement II is
straight line. not the correct explanation of statement I.
• During a syzygy conjunction, if the Moon is Statement I is correct: A sidereal day is the time it takes
positioned directly between the Earth and the Sun, it for the Earth to complete one rotation on its axis relative
can block the sunlight from reaching certain areas on to the stars, which is approximately 23 hours, 56 minutes,
Earth, resulting in a solar eclipse. and 4 seconds. A solar day, on the other hand, is the time it
takes for the Earth to complete one full rotation relative to
• During a syzygy opposition, when the Earth is
the Sun, which is approximately 24 hours.
positioned between the Sun and the Moon, the Earth’s
shadow can fall on the Moon, causing a lunar eclipse. Statement II is correct: The motion of the Earth in its orbit
around the Sun is termed as revolution. However, it does not
59. Solution (c) directly explain why a sidereal day is shorter than a solar
Exp) Option c is the correct answer day. The difference in duration between the two types of
days is primarily due to the Earth’s axial tilt and the elliptical
Statement 1 is correct: The Earth is closest to the Sun at
shape of its orbit around the Sun.
perihelion, which occurs generally on 3 rd January.
Statement 2 is incorrect: The Earth is farthest from the Sun 63. Solution (b)
at Aphelion not Perihelion, which occurs generally on 4 th Exp) Option b is the correct answer.
July. If the Earth did not have rotational movement, it would
Statement 3 is correct: The Earth is farthest away from the result in the following consequences:
Sun at Aphelion, which generally occurs on 4th July. Statement 1 is correct: The rotation of the Earth on its axis
Statement 4 is incorrect: The Earth is nearest from the Sun causes the apparent rising and setting of the Sun. Without
at Perihelion, not Aphelion, which occurs generally on 3 rd the Earth’s rotation, there would be no daily movement of
January the Sun across the sky, resulting in the absence of Sun-rise
and Sun-set.
60. Solution (b)
Statement 2 is correct: The rotation of the Earth is
Exp) Option b is the correct answer responsible for the alternation of day and night. As the
The Earth’s atmosphere does not have a definitive Earth rotates, different parts of it are exposed to sunlight,
boundary where it abruptly ends. Instead, it gradually leading to the day and night cycle. Without rotational
thins as altitude increases until it merges with the vacuum movement, there would be a fixed illumination pattern, with
of space. This transition is not clearly perceptible, as the one side of the Earth always facing the Sun (perpetual day)
density of gases decreases with increasing altitude, and and the other side always in darkness (perpetual night).

21 PYQ Workbook
GENERAL GEOGRAPHY

Statement 3 is incorrect: The Earth’s axial tilt and its 67. Solution (a)
orbit around the Sun are primarily responsible for the Exp) Option a is the correct answer.
occurrence of different seasons. Even without rotation,
The first known marsupial fossils date back to the Lower
the axial tilt and orbital characteristics would still lead to
Cretaceous period, which was about 125 million years ago.
variations in the amount of sunlight received by different
The Lower Cretaceous period occurred approximately 145
regions, resulting in the occurrence of different seasons.
to 100 million years ago. These fossils were found in North
64. Solution (b) America and Asia. Marsupials eventually spread to Australia
Exp) Option b is the correct answer. and South America, where they diversified into a wide
variety of species.
• Summer Solstice corresponds to 21st June
• Winter Solstice corresponds to 22nd December
• Vernal Equinox corresponds to 21st March
• Autumnal Equinox corresponds to 23rd September
Important Tips
• Summer Solstice - It is the phenomenon when the
Sun reaches its highest position in the sky, resulting
in the longest day and shortest night of the year. In the
Northern Hemisphere, it occurs on 21st June.
• Winter Solstice - It is the phenomenon when the Sun
reaches its lowest position in the sky, resulting in
the shortest day and longest night of the year. In the
Northern Hemisphere, it occurs on 22nd December.
• Vernal Equinox - It is the phenomenon when day
and night are approximately of equal length all over
the world. It marks the beginning of spring in the
Northern Hemisphere. In the Northern Hemisphere, it
occurs on 21st March.
• Autumnal Equinox - It is the phenomenon when day
and night are approximately of equal length all over
the world. It marks the beginning of autumn in the
68. Solution (c)
Northern Hemisphere. In the Northern Hemisphere, it
occurs on 23rd September. Exp) Option c is the correct answer.
Latitude and longitude are two sets of coordinates that
65. Solution (a) are used to determine the absolute location of a place on
Exp) Option a is the correct answer. the map. Latitude is the measure of a place’s distance north
The duration of day and night is determined by the tilt of or south of the equator, while longitude is the measure of a
the Earth’s axis and its rotation around the Sun. Near the place’s distance east or west of the Prime Meridian.
Equator, which is located at 0 degrees latitude, the difference
between day and night duration is relatively small throughout
the year. This is because the Equator receives nearly equal
amounts of sunlight year-round due to its proximity to the
Sun’s direct rays.
As one moves away from the Equator towards higher
latitudes, such as towards the Poles, the difference in the
duration of day and night progressively increases. This
is because the angle at which sunlight reaches the Earth’s
surface becomes more oblique, resulting in shorter days and
longer nights during certain times of the year.

66. Solution (a)


Exp) Option a is the correct answer.
The normal lapse rate is the rate at which temperature
decreases with increasing altitude in the troposphere. The
normal lapse rate is approximately 6.5°C per kilometer
(18.8°F per mile). This means that for every kilometer you
go up in altitude, the temperature will decrease by 6.5°C.

PYQ Workbook 22
GENERAL GEOGRAPHY

69. Solution (b) In the context of long-wave radiation, which refers to the
Exp) Option b is the correct answer. emission of infrared radiation from the Earth’s surface,
Inner planets refer to the planets that are located between scattering is not involved.
the Sun and the asteroid belt. These planets include Long-wave radiation primarily involves the transfer of heat
Mercury, Venus, Earth, and Mars. They are called inner energy as electromagnetic waves with longer wavelengths. It
planets because they are closer to the Sun compared to the
occurs through the process of radiation, where the Earth’s
outer planets (Jupiter, Saturn, Uranus, and Neptune) which
are located beyond the asteroid belt. surface emits infrared radiation into the atmosphere.
This emitted radiation can be absorbed or re-emitted by
Important Tips
greenhouse gases in the atmosphere, contributing to the
The inner and outer planets are two groups of planets in
greenhouse effect.
our solar system. They have different characteristics and
locations relative to the sun. Here are some of the main 72. Solution (b)
differences between them:
Exp) Option b is the correct answer.
• The inner planets are Mercury, Venus, Earth, and
Mars. The outer planets are Jupiter, Saturn, Uranus, The lowest temperature is observed in the mesosphere of the
and Neptune. atmosphere. The mesosphere is the third layer of the Earth’s
• The inner planets are closer to the sun and have atmosphere, located above the stratosphere and below the
shorter orbits. The outer planets are farther away and thermosphere. As we move higher into the mesosphere,
have longer orbits. the temperature decreases with increasing altitude. The
• The inner planets are smaller and rockier. They are coldest temperatures in the atmosphere, reaching as low as
also called terrestrial planets because they have solid -90 degrees Celsius (-130 degrees Fahrenheit), are found
surfaces. The outer planets are larger and gassier. They in the mesosphere. This layer is also known for its unique
are also called gas giants because they are made mostly
phenomena, such as the occurrence of noctilucent clouds
of hydrogen and helium.
and meteors burning up upon entry.
• The inner planets have either no moons or few moons.
Earth has one moon, Mars has two, and Mercury and 73. Solution (b)
Venus have none. The outer planets have numerous
moons. Jupiter has 79, Saturn has 82, Uranus has 27, Exp) Option b is the correct answer.
and Neptune has 143. The ozone layer is a region of the upper atmosphere, between
• The inner planets have no rings. The outer planets roughly 15 and 35 km above Earth’s surface, containing
have rings made of dust, ice, and rocks. Saturn’s rings relatively high concentrations of ozone molecules (O3). The
are the most visible and complex. ozone layer effectively blocks almost all solar radiation of
70. Solution (c) wavelengths less than 290 nm, which includes UVC and
Exp) Option c is the correct answer. most of UVB.
Earth’s tilt causes the seasons because it affects how much Important Tips
sunlight each hemisphere receives throughout the year. • The Sun emits different types of ultraviolet radiation,
When one hemisphere is tilted toward the Sun, it receives classified according to their wavelengths.
more direct and intense rays, making it warmer and longer
• The shortest wavelength is ultraviolet-C (UVC),
in daylight hours. This is summer for that hemisphere. When
which ranges from 100 to 280 nanometers (nm).
the same hemisphere is tilted away from the Sun, it receives
less direct and weaker rays, making it colder and shorter in • The next is ultraviolet-B (UVB), which ranges from
daylight hours. This is winter for that hemisphere. 280 to 315 nm.
If the equatorial plane of Earth (the plane perpendicular • The longest is ultraviolet-A (UVA), which ranges
to its axis) was not inclined to its orbit, then there would from 315 to 400 nm
be no seasons. The amount of sunlight each hemisphere
74. Solution (c)
receives would be constant throughout the year, and there
would be no solstices or equinoxes. Exp) Option c is the correct answer.

71. Solution (b) The thermosphere is the layer of the atmosphere that is
Exp) Option b is the correct answer. located above the mesosphere and below the exosphere. It
is the hottest layer of the atmosphere, and it has a high
Scattering is the process by which particles or molecules
in the atmosphere redirect the incoming solar radiation in concentration of ions. The ions in the thermosphere are
different directions. It occurs when light interacts with small formed by the interaction of solar radiation with the atoms
particles or gas molecules and changes its direction. and molecules in the atmosphere.

23 PYQ Workbook
GENERAL GEOGRAPHY

Important Tips
Layer Altitude Temperature Characteristics
Densest layer
of atmosphere,
contains most
of the Earth’s
Troposphere 0-12 km -56 to 15 °C atmosphere
Ozone layer
located in this
Stratosphere 12-50 km -56 to 0 °C layer
Coldest layer of
Mesosphere 50-85 km -56 to -89 °C atmosphere
Ionosphere
-89 to 1500 located in this
Thermosphere 85-600 km °C layer
Very thin layer
77. Solution (d)
of atmosphere,
6 0 0 - 1 0 0 0 1 5 0 0 - 1 0 0 0 0 no definite upper Exp) Option d is the correct answer.
Exosphere km °C limit India was part of the ancient Gondwana supercontinent.
The Gondwana supercontinent included South America,
75. Solution (c) Africa, Arabia, Madagascar, India, Australia, and
Antarctica. It was a vast landmass that existed around 600
Exp) Option c is the correct answer. million years ago and started breaking apart during the Early
Jurassic Period, about 180 million years ago, leading to the
Statement 1 is correct- Venus and Uranus are the only
formation of the continents we recognize today.
two planets in the solar system that rotate opposite to the
Important Tips
direction of rotation of Earth. Venus rotates clockwise,
The assembly and breakup of Gondwana:
while Earth rotates counterclockwise. Uranus rotates on its • Gondwana was assembled through continental
side, and its rotation is also retrograde. collisions during the Late Precambrian, roughly 1
billion to 542 million years ago.
Statement 2 is correct- Venus is the second planet from
• It eventually collided with other landmasses, including
the Sun, and it is the closest planet to Earth. The average North America, Europe, and Siberia, forming the
distance between Venus and Earth is about 41 million supercontinent of Pangea.
kilometers. • The breakup of Gondwana occurred in stages,
primarily during the Mesozoic Era.
Important Tips • Around 180 million years ago, during the Jurassic
Uranus, with an orbital period of 84 years, was discovered Period, the western half of Gondwana (comprising
by William Herschel and has a total of 27 moons. On the Africa and South America) began to separate from the
other hand, Venus has an orbital period of 225 days and eastern half (including Madagascar, India, Australia,
no confirmed moons. It was Galileo Galilei who made and Antarctica).
the first accurate observation of Venus in 1610. • This separation led to the opening of the South
Atlantic Ocean about 140 million years ago as Africa
separated from South America.
76. Solution (d)
• Around the same time, India, still attached to
Exp) Option d is the correct answer. Madagascar, started to separate from Antarctica
and Australia, initiating the formation of the central
The Intertropical Convergence Zone (ITCZ) is a low-
Indian Ocean.
pressure zone located at the equator. It is the area where • During the Late Cretaceous Period, about 50 million
the northeast and southeast trade winds converge. Sailors years ago, India finally broke away from Madagascar.
often refer to it as the ‘doldrums’ due to its windless • Australia also gradually rifted away from Antarctica.
conditions. The ITCZ encircles the Earth near the thermal • India eventually collided with Eurasia, forming the
Himalayan Mountain range.
equator but can vary in its specific position seasonally. It is
• The northward movement of the Australian plate began
a significant meteorological feature that influences weather its collision along the southern margin of Southeast
patterns and precipitation in tropical regions. Asia, an ongoing process.

PYQ Workbook 24
GENERAL GEOGRAPHY

78. Solution (d)


Exp) Option d is the correct answer.
The Sea of Tranquility is a dark spot located in the northern
hemisphere of the Moon. It is not a body of water, but a
lower altitude plain filled with dark, solidified lava. It is also
the first location on another celestial body to be visited by
humans, as the Apollo 11 mission landed there in 1969.

79. Solution (b)


Exp) Option b is the correct answer.
A light-year is a unit of length used to express astronomical
distances and is equivalent to about 9.46 trillion kilometers
(9.46 × 10 km), or 5.88 trillion miles (5.88 × 10 mi). As
defined by the International Astronomical Union (IAU), a
light-year is the distance that light travels in a vacuum in
one Julian year (365.25 days). A light-year is not a unit of
time, as the name might imply.
A light-year is most often used when expressing distances
to stars and other distances on a galactic scale, especially
in non-specialist contexts and popular science publications.
The unit most commonly used in professional astronomy is
the parsec (symbol: pc, about 3.26 light-years) which derives
from astrometry; it is the distance at which one astronomical
unit (au) subtends an angle of one second of arc.

80. Solution (a)


Exp) Option a is the correct answer.
The Earth is closest to the Sun, or at its perihelion, about
two weeks after the December solstice, when it is winter in
the Northern Hemisphere. The date of the perihelion varies
slightly from year to year, but it usually occurs around
Important Tips
January 3 or 4. On January 4, 2023, the Earth was about
• The closest country to 0°, 0° is the African country of
147.1 million km (91.4 million mi) away from the Sun. This
Ghana which lies about 614 km (382 mi) to the north
is about 5 million km (3 million mi) closer than the average of this confluence.
distance between the Earth and the Sun, which is about 150 • The closest piece of land to 0°, 0° is a small islet
million km (93 million mi). However, the Earth’s distance offshore of Ghana, between Akwidaa and Dixcove at
from the Sun does not cause seasons or affect the global the latitude and longitude coordinates of 4°45′30″N,
temperature significantly. Seasons are mostly caused by the 1°58′33″W. The distance from this islet to 0°, 0° is
tilt of the Earth’s axis, which affects the amount of sunlight about 570 km (354.2 mi).

each hemisphere receives. • Concept of Null Island:


• Null Island is a fictional geographic location
81. Solution (a) situated at the precise intersection of zero degrees
Exp) Option a is the correct answer. latitude (the Equator) and zero degrees longitude
(the Prime Meridian). It gained recognition with
The intersection of zero-degree latitude (the Equator) and the release of publicly available Natural Earth GIS
zero-degree longitude (the Prime Meridian) is located in data.
the Atlantic Ocean, specifically in the Gulf of Guinea off • This one-square-meter “island” serves as a
the coast of western Africa. This point serves as a reference troubleshooting tool for identifying geocoding
for global navigation and is where the coordinates (0°N 0°E) failures, where geographic data cannot be
meet. accurately located due to errors.

25 PYQ Workbook
GENERAL GEOGRAPHY

• In cases where GIS software or mapping tools Important Tips


struggle to assign coordinates to an address, the Kepler’s law of planetary motion: Kepler’s laws of planetary
record may be temporarily assigned a coordinate of motion are three scientific laws that describe how planets
0,0 at Null Island to signal a potential issue in the orbit the sun. They were discovered by the German
data. astronomer Johannes Kepler in the early 17th century,
based on the observations of his mentor Tycho Brahe.
82. Solution (b)
• Kepler’s first law: The orbit of each planet around the
Exp) Option b is the correct answer. sun is an ellipse with the sun at one of the foci. This
The equinoxes are the only times of the year when the means that the distance between the planet and the
subsolar point is directly on the Equator. The subsolar point sun varies during the orbit, and that the planet moves
is an area where the sun’s rays shine perpendicular to Earth’s faster when it is closer to the sun and slower when it
surface—a right angle. This means that the length of day and is farther away.
night is equal for both hemispheres on the equinoxes. The • Kepler’s second law: The line joining a planet to the
equinoxes occur around March 21 and September 22 every sun sweeps out equal areas in equal intervals of time.
This means that the planet covers the same amount of
year.
space in its orbit regardless of its position, and that it
83. Solution (b) takes the same time for the planet to go from one point
to another if the area enclosed by the line is the same.
Exp) Option b is the correct answer.
• Kepler’s third law: The square of the period of a
Day and night are caused by the rotation of the Earth on planet’s orbit is proportional to the cube of its average
its axis, which makes different parts of the planet face the distance from the sun. This means that there is a
Sun or away from it at different times. The Earth rotates once mathematical relationship between the time it takes
every 24 hours, which means that every part of the planet for a planet to complete one orbit and its distance from
experiences a period of daylight and a period of darkness in the sun, and that planets that are farther away from the
that time frame. sun have longer periods than planets that are closer to
the sun.
84. Solution (a)
86. Solution (c)
Exp) Option a is the correct answer.
Exp) Option c is the correct answer.
The latitude at which the annual range of temperature
remains minimum is the Equator. At the Equator, the Seasonal changes are caused by both the revolution of
Sun’s rays strike almost vertically year-round, resulting Earth around the Sun and the axial tilt of Earth by 23.5
in consistent and nearly equal day and night lengths degrees. The revolution of Earth around the Sun determines
throughout the year. This near-vertical angle of sunlight the length of the year and the seasons, while the axial tilt of
leads to minimal variation in temperature between seasons. Earth determines how different parts of Earth receive more
Consequently, the Equator experiences a relatively stable or less solar energy at different times of the year.
and warm climate with a minimal annual temperature range 87. Solution (b)
compared to regions at higher latitudes.
Exp) Option b is the correct answer.
85. Solution (c) Chile is the country with the longest north-south
Exp) Option c is the correct answer. (latitudinal) extension of its territory. It is located in
If the distance between the sun and the earth is reduced by South America and extends from a latitude of 17° South to
a quarter to the present distance, then the duration of our Cape Horn at 56° South, covering a distance of over 4,270
year will be decreased. This is because the orbital period of kilometers (2,653 miles).
a planet depends on its distance from the Sun, according to Important Tips
Kepler’s third law of planetary motion. The law states that Indian Geographical Extent:
the square of the orbital period is proportional to the cube of
• India is located entirely in the northern hemisphere;
the semi-major axis of the orbit. specifically in the south-central part of the continent
If the distance between the Sun and the Earth is reduced by of Asia, the mainland extends between latitudes
a quarter, then the semi-major axis of the Earth’s orbit will 8°4’N and 37°6’N and longitudes 68°7’E and 97°25’E,
also be reduced by a quarter. This means that the orbital area of the world.
period will be reduced by a factor of 0.5^ (3/2), which is • The southernmost point of the country is the
approximately 0.35. Therefore, the duration of our year will Pygmalion Point or Indira Point is located at 6° 45′
be decreased by 65%. N latitude.

PYQ Workbook 26
GENERAL GEOGRAPHY

• The north-south extent from Indira Col in Kashmir to


Kanyakumari is 3,214 km.
• East-west width from the Rann of Kutch to Arunachal
Pradesh is 2,933 km.

88. Solution (d)


Exp) Option d is the correct answer.
A nebula is not a part of the solar system, but a giant cloud
of dust and gas in interstellar space. A nebula can be the
result of a dying star, such as a supernova, or a region where
new stars are forming. Nebulae are very far away from the 90. Solution (c)
Earth and can only be seen with powerful telescopes. Exp) Option c is the correct answer.
Venus is the planet that is most often called the ‘Evening
Important Tips
Star’, because it is the brightest object in the night sky after
• Asteroids are rocky objects that orbit the Sun, mostly the Moon and the Sun, and it usually appears in the western
between Mars and Jupiter. sky after sunset. Venus is also sometimes called the ‘Morning
• Comets are icy bodies that have highly elliptical orbits Star’, because it can also appear in the eastern sky before
around the Sun and develop tails when they approach sunrise. Whether Venus is an evening star, or a morning star
it. depends on its position relative to the Earth and the Sun.
• Planets are large spherical objects that orbit the Sun When Venus is on the same side of the Sun as the Earth, it is
and clear their vicinity of other smaller objects. a morning star; when it is on the opposite side of the Sun, it
is an evening star. Venus alternates between being an evening
• The solar system consists of eight planets: Mercury,
star and a morning star every 584 days, or about 19 months.
Venus, Earth, Mars, Jupiter, Saturn, Uranus and
Neptune. Important Tips
Facts on Venus Planet:
89. Solution (b)
• Venus has a thick atmosphere that consists mainly of
Exp) Option b is the correct answer.
carbon dioxide and sulfuric acid clouds. The atmosphere
Continents have drifted apart because of tectonic traps heat and creates a runaway greenhouse effect,
activities, which are the movements of the Earth’s tectonic making Venus the hottest planet in our solar system.
plates. Tectonic plates are large slabs of rock that float on • Venus also rotates in the opposite direction of most
the Earth’s mantle. They are constantly moving, and their other planets, including Earth. This means that the Sun
movements can cause continents to drift apart, collide, or rises in the west and sets in the east on Venus.
slide past each other. Tectonic activities are caused by the • Venus rotates very slowly, taking 243 Earth days to
heat that is generated in the Earth’s core. This heat causes complete one spin. This makes a day on Venus longer
the mantle to flow, which in turn moves the tectonic plates. than a year on Venus, which lasts 225 Earth days
The theory of continental drift, which was first proposed by
91. Solution (b)
Alfred Wegener in 1912, states that the continents were once
all joined together in a single supercontinent called Pangaea. Exp) Option b is the correct answer.
Over time, Pangaea broke apart and the continents drifted to Most of the world’s deserts are located near 30 degrees north
their current positions. latitude and 30 degrees south latitude, where the heated

27 PYQ Workbook
GENERAL GEOGRAPHY

equatorial air begins to descend. Sinking, dry air currents and the stars across the sky, as well as the Coriolis effect,
occurring at 30° north and south of the equator produce which influences the wind and ocean currents.
trade winds that create deserts like the African Sahara and
Statement 2 is false: Earth’s revolution around the Sun
Australian Outback. Therefore, the possibility of a desert
does not cause the occurrence of day and night on Earth,
on earth is nearer 23° latitude than the other options.
but it causes seasonal changes and variations in day length
92. Solution (c) throughout the year. Earth revolves around the Sun in
Exp) Option c is the correct answer. an elliptical orbit once every 365.25 days, or one year.
Statement I is incorrect: Mercury is the closest planet to This means that Earth’s distance from the Sun changes
the Sun, but it is not the hottest. Mercury has a very thin slightly during its orbit, but this does not affect the global
atmosphere that cannot trap heat, so its surface temperature temperature significantly. What causes seasonal changes
varies greatly from day to night. The hottest planet in the is Earth’s axial tilt, which is about 23.5 degrees from the
solar system is Venus, which has a thick atmosphere that vertical. This means that Earth’s axis always points in the
creates a greenhouse effect and keeps the surface temperature same direction in space, regardless of its position in its orbit.
at a constant 462 °C (864 °F) As a result, different parts of Earth receive different amounts
Statement II is incorrect: Ganymede is the largest satellite of sunlight and heat throughout the year, creating seasons
in the solar system, but it is not a satellite of Saturn.
Ganymede is a moon of Jupiter, and it is larger than Mercury 95. Solution (a)
and Pluto, and three-quarters the size of Mars. Exp) Option a is the correct answer.
Statement III is correct: Neptune has six rings that are
Equinoxes, the two periods in the year when day and night
composed mainly of dust and ice particles, some of which
are nearly equal in duration, occur on 21 March (Vernal
contain methane gas. The rings are very faint and hard to
see, and they orbit at a very cold distance from the Sun, equinox) and 23 September (Autumnal equinox). These
about 30 times farther than Earth. The average temperature equinoxes mark the times when the Earth’s axis is neither
on Neptune is about -200 °C (-328 °F) tilted toward nor away from the Sun, resulting in nearly
Statement IV is correct: Phobos and Deimos are the equal hours of daylight and darkness at all latitudes. The
two moons of Mars, and they are named after the Greek term “equinox” comes from the Latin words “aequus” (equal)
mythological twin characters Phobos (fear and panic) and and “nox” (night), reflecting the balanced distribution of
Deimos (terror and dread) who accompanied their father daylight and nighttime during these periods.
Ares (Mars) into battle. Both Phobos and Deimos were
discovered in 1877 by American astronomer Asaph Hall.
They are irregular in shape and have surface materials
similar to many asteroids in the outer asteroid belt, which
suggests that they are captured asteroids.

93. Solution (a)


Exp) Option a is the correct answer.
When the sun shines vertically over the Tropic of
Capricorn, it is summer in the Southern Hemisphere
and winter in the Northern Hemisphere. This means that
the air in North-West India is cooler than the air in the
surrounding regions. As a result, the air in North-West
India contracts and sinks, creating a high-pressure area. Important Tips
This high-pressure area is known as the subtropical high- • Equinoxes mark the transition between seasons,
pressure belt or the anticyclone. It is responsible for the with the March equinox marking the start of spring
clear skies and dry weather that are typically experienced in (in the Northern Hemisphere) and the September
North-West India during the winter months. equinox marking the start of autumn (in the Northern
94. Solution (c) Hemisphere).

Exp) Option c is the correct answer. • During equinoxes the Sun crosses the Earth’s equator,
and it appears to rise “due east” and set “due west” on
Statement 1 is false: Earth’s rotation on its axis does not
the equator. This happens because the plane of Earth’s
cause seasonal changes, but it causes the day and night
equator aligns with the center of the Sun’s disk.
cycle on Earth. Earth rotates on its axis once every 24 hours,
making one complete turn from west to east. This means that • The “nearly” equal hours of day and night are due to
different parts of Earth face the Sun at different times of the refraction of sunlight or a bending of the light’s rays
day, creating periods of light and darkness. Earth’s rotation that causes the sun to appear above the horizon when
also affects the apparent movement of the Sun, the Moon, the actual position of the sun is below the horizon.

PYQ Workbook 28
GENERAL GEOGRAPHY

• Additionally, the days become a little longer at the • Therefore, on the equinox and for several days before
higher latitudes (those at a distance from the equator) and after the equinox, the length of day will range
because it takes the sun longer to rise and set. from about 12 hours and six and one-half minutes at
the equator, to 12 hours and 8 minutes at 30 degrees
latitude, to 12 hours and 16 minutes at 60 degrees
latitude.

29 PYQ Workbook
GENERAL GEOGRAPHY

GENERAL GEOGRAPHY
GEOMORPOLOGY
*This unit consists of questions from Earth’s Interor and its Structure, Volcanism and Earthquakes,
Minerals and Rock system, Continental drift theory, SFS Theory, Plate Tectonics theory and Geomorphic
process & landforms.

2.1. UPSC CSE Previous Years’ Questions 2. The amount of water in polar ice caps
and glaciers is more than the amount of
1. Which one of the following is a part of the groundwater.
Congo Basin? [UPSC CSE Pre 2023] Which of the statements given above is/are
(a) Cameroon correct? [UPSC CSE Pre 2021]
(b) Nigeria (a) 1 only
(c) South Sudan (b) 2 only
(d) Uganda (c) Both 1 and 2
(d) Neither 1 nor 2
2. With reference to the Earth’s atmosphere,
which one of the following statements is 5. Consider the following statements:
correct? [UPSC CSE Pre 2023] [UPSC CSE Pre. 2018]
(a) The total amount of insolation received at 1. The Earth’s magnetic field has reversed
the equator is roughly about 10 times of every few hundred thousand years.
that received at the poles. 2. When the Earth was created more than
(b) Infrared rays constitute roughly two- 4000 million years ago, there was 54%
thirds of insolation. oxygen and no carbon dioxide.
(c) Infrared waves are largely absorbed by 3. When living organisms originated, they
water vapour that is concentrated in the modified the early atmosphere of the
Earth.
lower atmosphere.
(d) Infrared waves are a part of visible Which of the statements given above is/are
spectrum of electromagnetic waves of correct?
solar radiation. (a) 1 only
(b) 2 and 3 only
3. Consider the following statements : (c) 1 and 3 only
1. In a seismograph, P waves are recorded (d) 1, 2 and 3
earlier than S waves. 6. Which of the following phenomena
2. In P waves, the individual particles might have influenced the evolution of
vibrate to and fro in the direction of organisms? [UPSC CSE Pre. 2014]
wave propagation whereas in S waves, 1. Continental drift
the particles vibrate up and down at 2. Glacial cycles
right angles to the direction of wave
Select the correct answer using the code
propagation. given below.
Which of the statements given above is/are (a) 1 only
correct? [UPSC CSE Pre 2023] (b) 2 only
(a) 1 only (c) Both 1 and 2
(b) 2 only (d) Neither 1 nor 2
(c) Both 1 and 2 7. Which one of the following pairs is correctly
(d) Neither 1 nor 2 matched? [UPSC CSE Pre. 2013]
4. With reference to the water on the planet Geographical Feature Region
Earth, consider the following statements:
(a) Abyssinian Plateau Arabia
1. The amount of water in the rivers
and lakes is more than the amount of (b) Atlas Mountains North-Western
groundwater. Africa

PYQ Workbook 30
GENERAL GEOGRAPHY

(c) Guiana Highlands South-Western 13. Consider the following statements:


Africa [UPSC CSE Pre 2009]
(d) Okavango Basin Patagonia 1. In the world, the tropical deserts occur
along the western margins of continents
8. Consider the following: within the trade wind belt.
[UPSC CSE Pre. 2013] 2. In India the East Himalayan region gets
1. Electromagnetic radiation high rainfall from north-east winds.
2. Geothermal energy Which of the statements given above is/are
3. Gravitational force correct?
4. Plate movements (a) 1 only
5. Rotation of the earth (b) 2 only
6. Revolution of the earth (c) Both 1 and 2
(d) Neither 1 nor 2
Which of the above are responsible for
bringing dynamic changes on the surface of 14. Match List 1 and List II and select the
the earth? correct answer using the code given below:
(a) 1, 2, 3 and 4 only [UPSC CSE Pre 2009]
(b) 1, 3, 5 and 6 only
(c) 2, 4, 5 and 6 only List I List II
(d) 1, 2, 3, 4, 5 and 6 (Geographic feature) (Country)
9. Which one of the following sets of elements A. Great Victoria 1. Australia
was primarily responsible for the origin of Desert
life on the Earth? [UPSC CSE 2012] B. Grand Canyon. 2. Canada
(a) Hydrogen, Oxygen, Sodium C. Lake Winnipeg. 3. New Zealand
(b) Carbon, Hydrogen, Nitrogen
(c) Oxygen, Calcium, Phosphorous D. Southern Alps. 4. USA
(d) Carbon, Hydrogen, Potassium A B C D
10. A person stood alone in a desert on a dark (a) 1 2 4 3
night and wanted to reach his village which (b) 1 4 2 3
was situated 5km east of the point where he (c) 3 2 4 1
was standing. He had no instruments to find (d) 3 4 2 1
the direction but he located the polestar, 15. Which of the following pairs is/are correctly
the most convenient way now to reach his matched? [UPSC CSE Pre 2008]
village is to walk in the
[UPSC CSE Pre. 2012] Theory/Law Associated
(a) facing the polestar Scientist
(b) opposite to the polestar 1. Continental Drift Edwin Hubble
(c) keeping the polestar to his left 2. Expansion of Alfred Wegener
(d) keeping the polestar to his right Universe
11. Through which one of the following groups 3. Photoelectric Effect Albert Einstein
of countries does the Equator pass? Select the correct answer using the code given
[UPSC CSE Pre. 2011] below:
(a) Brazil, Zambia and Malaysia (a) 2 and 3 only
(b) Colombia, Kenya and Indonesia (b) 3 only
(c) Brazil, Sudan and Malaysia (c) 2 only
(d) Venezuela, Ethiopia and Indonesia
(d) 1 only
12. In the structure of the planet Earth, below
16. Which of the following is/are the geologic
the mantle, the core is mainly made up of
which one of the following? features that can be seen off a Rocky
Coast? [UPSC CSE 2008]
[UPSC CSE Pre 2009]
1. Wave-cut terrace
(a) Aluminium
(b) Chromium 2. Sea Arches and Stacks
(c) Iron 3. Barrier Bars
(d) Silicon 4. Lagoons

31 PYQ Workbook
GENERAL GEOGRAPHY

Select the correct answer using the code given 22. Consider the following statements about
below: the sedimentary rocks:
(a) 1 and 2 only 1. Sedimentary rocks are formed at the
(b) 3 and 4 only earth’s surface by the hydrological system.
(c) 1, 3 and 4 only 2. The formation of sedimentary rocks
(d) 1, 2, 3 and 4 involves the weathering of pre-existing
rocks.
17. Which one of the following cities is nearest
3. Sedimentary rocks contain fossils.
to the equator? [UPSC CSE 2008]
4. Sedimentary rocks typically occur in
(a) Colombo layers.
(b) Jakarta
(c) Manila Which of these statements are correct?
(d) Singapore [UPSC CSE Pre 2001]
(a) 1 and 2
18. Which one of the following straits is nearest (b) 1 and 4
to the International Date Line? (c) 2, 3 and 4
[UPSC CSE 2008] (d) 1, 2, 3 and 4
(a) Malacca Strait 23. Volcanic eruptions do not occur in the:
(b) Bering Strait [UPSC CSE Pre 2001]
(c) Strait of Florida (a) Baltic Sea
(d) Strait of Gibraltar (b) Black Sea
19. Consider the following statements: (c) Caribbean Sea
[UPSC CSE Pre. 2005] (d) Caspian Sea
1. Total land area of Earth is approximately 24. Consider the following statement about
1475 lakh square kilometres. earthquake: [UPSC CSE Pre 2001]
2. Ratio of land area to water area of Earth is 1. Intensity of earthquake is measured on
approximately 1:4. Mercalli Scale.
3. Maximum percentage of Earth’s water is 2. Earthquake’s magnitude is the
in the Pacific Ocean. measurement of energy released.
Which of the statements given above is/are 3. Magnitude of earthquake depends directly
correct? upon Amplitude of an earthquake waves.
(a) 1 and 3 4. Every integer on Richter Scale shows 100
times increase in energy released.
(b) 2 and 3
(c) 1 only Which of the statements are true?
(d) 3 only (a) 1, 2 and 3
(b) 2, 3 and 4
20. Which one of the following is the correct (c) 1 and 4
sequence of the given Continents in the (d) 1 and 3
decreasing order of their percentage of
Earth’s land? [UPSC CSE Pre. 2005] 25. Match list-I with list-II and select the
correct answer using the codes given below
(a) North America - Africa - South America the lists:
– Europe
(b) Africa - North America - South America List-I (Volcano) List-II (Country)
- Europe
(c) North America - Africa - Europe - South A. Mount Rainier 1. Italy
America B. Etna 2. Mexico
(d) Africa - North America - Europe - South C. Paricutin 3. Philippines
America
D. Taal 4. USA
21. Where is Mount Saint Helens situated? Code: [UPSC CSE Pre 2001]
[UPSC CSE Pre 2005] A B C D
(a) Chile (a) 4 2 1 3
(b) Japan (b) 4 1 2 3
(c) Philippines (c) 2 1 4 3
(d) United State of America (d) 4 3 2 1

PYQ Workbook 32
GENERAL GEOGRAPHY

2.2. Other Examination Previous Years’ A B C D


Questions (a) 3 1 4 2
(b) 3 2 1 4
26. Which one of the following natural regions (c) 2 1 4 3
is known as ‘Land of Big Games’? (d) 4 3 2 1
[U.P.P.C.S. (Pre) 2019] 31. Mauna Loa is an active volcano in-
(a) Temperate Grassland [U.P.P.C.S. (Mains) 2014]
(b) Tropical Monsoon Region (a) Alaska
(c) Hot Desert Region (b) Hawaii
(d) Tropical Savanna Region (c) Italy
27. Which of the following instruments is used (d) Japan
to record seismic waves? 32. Consider the following statements:
[UPPCS (Mains) 2017] Statement (A): Deserts can be effective
(a) Seismogram sources for the production of perennial
(b) Seismograph energy.
(c) Seismoscope Reason (R): As much energy is consumed
(d) Seismometer by mankind in one year, Desert can receive
more energy than that in only 6 hours from
28. Which one of the following is the highest
the Sun.
volcanic mountain in the world?
Choose the correct answer from the following
[U.P.P.C.S. (Pre) 2015]
code. [UPPCS (Pre) 2013]
(a) Mount Pinatubo (a) (A) and (R), both are correct and (R) is
(b) Mount Kilimanjaro the correct explanation of (A).
(c) Mount Tall (b) (A) and (R), both are correct, but (R) is
(d) Mount Cotopaxi not the correct explanation of (A).
29. Match list-I with list-II and select the (c) (A) is correct, but (R) is false.
correct answer using the codes given below (d) (A) is false but (R) is correct.
the lists: 33. Which of the following is not a temperate
grassland? [U.P.P.C.S. (Pre) 2012]
List-I List-II
(a) Pampas
A. Mount Kinabalu 1. Argentina (b) Veld
B. Elburz 2. Malaysia (c) Downs
C. Aconcagua 3. Tanzania (d) Savanna
D. Kilimanjaro 4. Iran 34. ‘Death Valley’ situated in South California
Code: [UPPCS (Mains) 2015] in the U.S.A is an example of which of the
following? [U.P.P.C.S. (Pre) 2012]
A B C D (a) Anticlinal Valley
(a) 1 4 2 3 (b) Indian Valley
(b) 3 2 1 4 (c) Pristiclinal Valley
(c) 2 4 3 1 (d) Rift Valley
(d) 2 4 1 3
35. The Savannas are most widespread in–
30. Match list-I with list-II and select the [U.P.P.C.S. (Mains) 2011]
correct answer using the codes given below (a) Africa
the lists: (b) Asia
(c) North America
List-I (Volcano) List-II (Country)
(d) South America
A. Sabankaya 1. Italy
36. The ‘Ring of Fire’ is associated with-
B. Mount Etna 2. Peru
1. Earthquake
C. Colima 3. Indonesia 2. Volcanoes
D. Merapi 4. Mexico 3. Pacific Ocean
Code: [UPPCS (Pre) 2015] 4. Forest Fires

33 PYQ Workbook
GENERAL GEOGRAPHY

Choose your answer with the help of the 43. The probability of having a desert on the
codes given below. [U.P.P.C.S. (Pre) 2008] earth is more- [41st BPSC (Pre) 1996]
(a) 1, 2 and 3 (a) Near Zero-degree latitude
(b) 2 and 3 (b) Near 230 latitude
(c) 2 and 4 (c) Near 500 latitude
(d) 1, 2, 3 and 4 (d) Near 700 latitudes
37. Which of the following are active volcanoes? 44. The greatest delta of the world is formed
1. Aconcagua by– [41st B.P.S.C. (Pre) 1996]
2. Cotopaxi (a) Ganga and Brahmaputra
3. Etna (b) Mississippi-Missouri
4. Fujiyama (c) Yangtze-Kiang
Select the correct answer from the code given (d) Huang-He
below: [U.P.P.C.S. (Mains) 2007]
(a) 1 and 2 45. Which statement of the following is true for
(b) 3 and 4 igneous rocks? [40th B.P.S.C. (Pre) 1995]
(c) 1, 2 and 3 (a) These have little fossils
(d) 2, 3 and 4 (b) They have porous for water
(c) They are both crystal and non-crystal
38. The North-Western Region of the Indian
(d) These rocks have no silicas
Sub-continent is susceptible to earthquake
activity because of- 46. Based on following statements choose the
[U.P.P.C.S. (Mains) 2005] correct option:
(a) Volcanic activity Statement I: Theory of Plate tectonics was
(b) Plate tectonic activity given by H.H. Hess in 1967.
(c) Coral formation activity
(d) All of the above Statement II: Theory of Plate Tectonics is
based on Theory of Sea Floor Spreading.
39. Which Volcano is called ‘the lighthouse’ of Code: [Chhattisgarh PCS (Pre) 2021]
Mediterranean Sea?
(a) Statement I and Statement II both are true
[U.P.P.C.S. (Mains) 2004]
(b) Statement I and Statement II both are
(a) Etna false
(b) Pele
(c) Statement I is true, but Statement II is
(c) Stromboli
(d) Vesuvius false
(d) Statement I is false, but Statement II is
40. Which one of the following is not correctly true
matched? [U.P.P.C.S. (Mains) 2002]
(a) Downs - Tropical grassland 47. Read the following statement related to
(b) Steppes - Temperate grassland ‘Earthquake shadow zone’ and choose the
(c) Selva - Tropical Forest correct option:
(d) Taiga - Temperate Forest Statement I: The shadow zone of one
41. Desert is defined as the area where- earthquake is totally different from the
[U.P. Lower Sub. (Pre) 2009] shadow zone of another earthquake.
(a) Temperature remains more than 420 C Statement II: Seismometers record both ‘P’
(b) No plants are found and ‘S’ waves at any distance out of 105° from
(c) Annual rain occurs less than 25 cm earthquake’s epicenter.
(d) Prominence of sand barrows Code: [Chhattisgarh P.C.S. (Pre) 2020]
42. Highest peaks of the world are mostly found (a) Statement I and Statement II both are true
in which type of Mountains? (b) Statement I is true, but Statement II is
[45th B.P.S.C. (Pre) 2001] false
(a) Old folded mountains (c) Statement I is false, but Statement II is
(b) Young folded mountains true
(c) Residual mountains (d) Statement I and Statement II both are
(d) Block mountains false

PYQ Workbook 34
GENERAL GEOGRAPHY

48. Which of the following landforms is found (a) wind


in Karst region? (b) underground water
[Jharkhand PCS (Mains) 2016] (c) glacier
(a) Pothole (d) river
(b) Sink hole 55. Formation of delta is affected by:
(c) Blow hole [M.P.P.C.S. (Pre) 1993]
(d) Playa (a) Rocks
49. The most abundant gas emitted from (b) Tide-ebb
volcanoes is- [Jharkhand P.C.S. (Pre) 2013] (c) Strong winds
(a) Water Vapor (d) Deep sea
(b) Helium 56. Sandstone is flaky rock, because-
(c) Sulphur Dioxide [M.P.P.C.S. (Pre) 1993]
(d) Carbon Dioxide (a) It is formed in the desert
50. Origin of Metamorphic rocks occurs from (b) It is formed due to Heat
……... rock? [Jharkhand P.C.S. (Pre) 2011] (c) It is formed underwater
(a) Igneous (d) It is formed on the mountain
(b) Sedimentary 57. Which of the following is a basic landform?
(c) Both Igneous and Sedimentary [Uttarakhand P.C.S. (Pre) 2016]
(d) None of the above
(a) Volcanic cone
51. In which belt, the volcanic eruption is (b) Residual mountain
maximum? [MPPCS (Pre) 2021] (c) Monadnock
(a) Mid Continental (d) Erosional waterfall
(b) Circumpacific 58. Which one of the following is not an
(c) Mid Atlantic example of Metamorphic rock?
(d) Intraplates Volcanoes [Uttarakhand P.C.S. (Pre) 2006]
52. Match list-I with list-II and select the (a) Marble
correct answer using the codes given below (b) Quartzite
the lists: (c) Slate
(d) Granite
List-I List-II
(Grasslands) (Country/Continent) 59. The molten material found inside the earth
is called- [Uttarakhand P.C.S. (Pre) 2006]
A. Steppes 1. United States of America
(a) Lava
B. Prairies 2. South Africa (b) Basalt
C. Velds 3. Russia (c) Obsidian
D. Downs 4. Australia (d) None of the above
Code: [MPPCS Pre) 2020] 60. Which of the following statements about
A B C D volcanoes are correct? [CDS-2023(I)]
(a) 2 1 3 4 1. The strength of a volcano is measured by
(b) 1 4 2 3 the Volcanic Explosive Index.
(c) 3 1 2 4 2. Iceland and Philippines were created by
(d) 4 2 3 1 volcanic activity.
3. Volcanic soils are rich, deep and fertile
53. Which of the following pairs is not correctly
and allow intensive agriculture to take
matched? [M.P.P.C.S. (Pre) 2017]
place.
(a) Canyon - River
Select the correct answer using the code
(b) Zeugen - Wind
given below.
(c) Inselberg - Glacier
(d) Moraine - Glacier (a) 1 and 2 only
(b) 2 and 3 only
54. The Peneplain is related to: (c) 1 and 3 only
[M.P.P.C.S. (Pre) 2017] (d) 1, 2 and 3

35 PYQ Workbook
GENERAL GEOGRAPHY

61. A peripediment in a desert or semi-desert (a) Hydrolysis


region is also known as: [CDS-2022(I)] (b) Hydration
(a) Inselberg (c) Oxidation
(b) Zeugen (d) Carbonation
(c) Bajada 66. When the rivers discharge their waters from
(d) Playa all directions into a lake or depression, the
62. Individual lava flows are normally only a pattern is known as: [CDS 2021(I)]
few feet thick, but over a long period of time, (a) Trellis
repeated flows may build up a volcano. Such (b) Dendritic
volcanoes are termed as: [CDS-2022(I)] (c) Radial
(a) Shield volcano. (d) Centripetal
(b) Composite volcano. 67. Which one of the following is a type of
(c) Strato-volcano. igneous rock? [CDS 2021 (I)]
(d) Cinder-cone volcano. (a) Marble
63. Which one of the following sedimentary (b) Halite
rocks has not been formed mechanically? (c) Granite
(d) Shale
[CDS-2022(II)]
(a) Sandstone 68. Which one of the following statements
(b) Conglomerate about primary waves of earthquakes is not
(c) Loess correct? [CDS 2021 (I)]
(d) Geyserites (a) They are similar to sound waves.
(b) They can travel only through solid
64. Match List I with List II Match List I with materials.
List II and select the correct answer using (c) They travel through gaseous, liquid and
the code given below the Lists: solid materials.
List I Drainage List IIFeature) (d) They move faster and are the first to
Pattern of River) arrive at the surface.
A. Insequent 1. Whose direction of 69. The extent of damage caused by earthquake
flow is controlled by is not influenced by which one of the
the rock structure following? [CDS 2021 (II)]
B. Consequent 2. No apparent reason (a) Strength of earthquake
why it follows the (b) Population density
path it takes (c) Type of building
(d) Climate of the area
C. Subsequent 3. Flow in the same
direction as the 70. Geomorphic factors influencing plant and
consequent stream animal distributions are [CDS 2020(I)]
but at a lower level (a) slope angle and relief only
D. Resequent 4. Whose course is (b) slope aspect and relative relief
determined by the (c) slope angle, slope aspect and relief
original slope of (d) slope angle, slope aspect and relative
the land relief
Code : [CDS-2022(II)] 71. Match List-I with List-II and select the
A B C D correct answer using the codes given below
the Lists.
(a) 2 1 4 3
(b) 2 4 1 3 List-1 (Active List-II (Location)
(c) 3 4 1 2 Volcano)
(d) 3 1 4 2 A. Mount Merapi 1. Hawaii
65. Which one of the following processes leads B. Sakurajima 2. Italy
to expansion of certain minerals as they C. Mount Vesuvius 3. Japan
take up water, causing additional stress in
the rock? [CDS-2022(II)] D. Mauna Loa 4. Indonesia

PYQ Workbook 36
GENERAL GEOGRAPHY

Codes: [CDS 2020 (II)] (b) Both the statements are individually
A B C D true, but statement II is not the correct
(a) 1 2 3 4 explanation of statement I
(b) 1 3 2 4 (c) Statement I is true, but statement I is false
(c) 4 2 3 1 (d) Statement I is false, but statement II is
(d) 4 3 2 1 true

72. Which one of the following is not a major 77. Statement I: Incised meanders are formed in
tectonic plate? [CDS 2020 (II)] the mature stage of a river.
(a) Saudi Arabian plate Statement II: Incised meanders are
(b) Antarctica and the surrounding oceanic characterised by rejuvenation and upliftment
plate of land.
(c) India-Australia-New Zealand plate Codes: [CDS 2018 (I)]
(d) Pacific plate (a) Both the statements are individually true
73. Which one of the following is not a fluvial and statement II is the correct explanation
landform? [CDS 2020 (II)] of statement I
(b) Both the statements are individually
(a) Cirque
true, but statement II is not the correct
(b) Gorge
explanation of statement I
(c) Braids
(c) Statement I is true, but statement I is false
(d) Canyon
(d) Statement I is false, but statement II is
74. Statement I: By far the most common true
topographic form in a Karst terrain is the
78. Which one of the following statements with
sinkhole.
regard to volcanoes is not correct?
Statement II: Topographically, a sinkhole is a [CDS 2017 (I)]
depression that varies in depth from less than
(a) Stratovolcanoes produce lava flows that
a meter to few hundred meters.
initially follow valleys but are highly
Codes: [CDS 2018 (I)] resistant to erosion.
(a) Both the statements are individually true (b) The surrounding areas can remain
and statement II is the correct explanation highlands, lava ridges or mesas.
of statement I (c) Hawaiian shield volcanoes are eroded by
(b) Both the statements are individually streams that form deeply carved valleys
true, but statement II is not the correct with steeply sloping heads.
explanation of statement I (d) The system of streams on a dissected
(c) Statement I is true, but statement I is false volcano cone is not a radial drainage
(d) Statement I is false, but statement II is pattern.
true
79. With reference to earthquakes, which of the
75. Extrusive volcanoes are not found in, which following statements are correct?
one of the following mountains? 1. Earthquakes largely occur along the
[CDS 2018 (I)] converging plate boundaries.
(a) Alaska 2. Point of origin of earthquake in
(b) Rocky lithosphere is known as focus/hypocentre.
(c) Andes 3. Intensity of earthquake decreases with
(d) Himalayas distance from the epicentre.
76. Statement I: Portions of glacial troughs may 4. Epicentre of earthquake always remains
exhibit remarkably flat floors. over continents only.
Statement II: The falt floor in a glacial trough Select the correct answer using the code given
is produced by uniform glacial erosion. below: [CDS 2016 (II)]
Codes: [CDS 2018 (I)] (a) 1 and 2
(a) Both the statements are individually true (b) 1, 2 and 3
and statement II is the correct explanation (c) 2 and 3
of statement I (d) 1 and 4

37 PYQ Workbook
GENERAL GEOGRAPHY

80. Which one of the following is not related to statements regarding plate tectonics is not
wind erosion? [CDS 2016 (II)] correct? [CDS 2014 (II)]
(a) Wind gap (a) Tectonic plates are composed of oceanic
(b) Zeugen lithosphere and thicker Continental
(c) Dreikanter lithosphere.
(d) Demoiselle (b) Tectonic plates are able to move
because the Earth’s lithosphere has a
81. Stalactites and stalagmites are features of higher strength than the underlying
[CDS 2016 (I)] asthenosphere.
(a) glacial topography (c) The Earth’s lithosphere is broken up into
(b) volcanic topography tectonic plates.
(c) karst topography (d) Along divergent plate boundaries,
(d) fluvial topography subduction carries plates into the mantle.
82. Which of the following elements are found 86. Arrange the following features formed by
in highest and lowest quantities respectively rivers in its course starting from upstream:
in the crust of the Earth? [CDS 2015 (II)] 1. Meanders
(a) Oxygen and silicon 2. Falls
(b) Calcium and sodium 3. Delta
(c) Sodium and magnesium 4. Oxbow Lake
(d) Oxygen and magnesium Select the correct answer using the codes
83. Match List I with List II and select the given below: [CDS 2014 (II)]
correct answer using the codes given below (a) 2, 1, 3, 4
the lists. (b) 2, 1, 4, 3
(c) 1, 2, 3, 4
List I (Weathering List II (Landform/ (d) 1, 4, 2, 3
Type) Process)
87. Which of the following statement(s) relating
A. Chemical 1. Till to Earthquakes is/are correct?
weathering 1. The point of origin of Earthquake is
B. Mechanical 2. Oxidation called epicenter.
weathering 2. The lines joining the places which were
C. Glacial deposits 3. Plant roots affected Earthquake at the same point of
time are called homoseismal lines.
D. Deposition by 4. Stalactite
ground water Select the correct answer using the codes
given below: [CDS 2014 (I)]
Codes: [CDS 2015 I(II)]
(a) Only 1
A B C D (b) Only 2
(a) 2 3 1 4 (c) Both 1 and 2
(b) 2 1 3 4 (d) Neither 1 nor 2
(c) 4 1 3 2
88. Trough and ridge are [CAPF 2022]
(d) 4 3 1 2
(a) elongated area of low pressure and of high
84. Seismic gaps are [CDS 2015 I)] pressure respectively
(a) parts of plate boundaries in oceans where (b) elongated areas of low pressure
tsunamis occur frequently (c) elongated areas of high pressure
(b) sections of plate boundaries that have (d) elongated area of high pressure and of
ruptured repeatedly in the recent past low pressure respectively
(c) sections of plate boundaries that have not 89. Solifluction is a geomorphic process involving a
ruptured in the recent past special type of soil flow that is noticed in
(d) plate boundaries having no volcanic
[CAPF 2022]
activity.
(a) desert regions
85. Plate tectonics is a scientific theory that (b) karst regions
describes the large scale motions of Earth’s (c) permafrost regions
lithosphere. Which one among the following (d) savanna regions

PYQ Workbook 38
GENERAL GEOGRAPHY

90. What is the depression at the top of a volcano List I (Landform) List II (Agent)
following a volcanic eruption called?
A. Loess 1. Wind deposited
[CAPF 2021]
(a) Chamber B. Tombolo 2. Glacier deposited
(b) Vent C. Point bars 3. River Deposited
(c) Cinder D. Moraines 4. Sea deposited
(d) Crater Codes: [CAPF 2019]
91. Caves, arches, stacks and stumps are the A B C D
landscape features of which one of the (a) 1 4 3 2
following? [CAPF 2021] (b) 1 3 4 2
(a) River (c) 2 3 4 1
(b) Wave (d) 2 4 3 1
(c) Limestone
(d) Wind 98. The gently sloping accumulation of coarse
alluvium deposits by a braided stream is
92. The heaviest rocks, which are rolled along known as [CAPF 2019]
the riverbed in river transport, come under (a) sand bar
which type of load? [CAPF 2021] (b) alluvial fan
(a) Solution load (c) bajada
(b) Suspension load (d) diaras
(c) Saltation load
(d) Traction load 99. Springs are common in which of the
following areas?
93. Convection currents, which move the
1. Well jointed rocks
tectonic plates, are found in which layer of
the Earth? [CAPF 2021] 2. Arid areas with underlying rocks
(a) Crust 3. Karst topography
(b) Mantle 4. Tilted strata
(c) Outer core Select the correct answer using the codes
(d) Inner core given below. [CAPF 2018]
94. In India, Cardamom Hills are regarded as a (a) 1 and 3 only
continuation of the [CAPF 2021] (b) 1, 3 and 4
(a) Eastern Ghats (c) 2 and 4 only
(b) Mizo Hills (d) 3 and 4 only
(c) Naga Hills 100. Which one of the following is not an igneous
(d) Western Ghats rock? [CAPF 2018]
95. A long narrow stretch of sand and/or shingle (a) Granite
with one end attached to the mainland is (b) Gneiss
called [CAPF 2020] (c) Pumice
(a) Sand spit (d) Basalt
(b) Sand bar 101. Consider the following statements about
(c) Sand dune ‘Ring of Fire’ :
(d) Tombolo 1. It is also known as Circum-Pacific belt of
96. Which one of the following karst landforms volcanoes
is the largest in size? [CAPF 2020] 2. It is an active seismic zone
(a) Polje 3. It is a zone of divergent place margin
(b) Uvala Select the correct answer using the code given
(c) Swallow Hole below: [CAPF 2017]
(d) Sink Hole (a) Only 1
97. Match List I with List II and select the (b) 1 and 2
correct answer using the code given below (c) 2 and 3
the lists. (d) 1, 2 and 3

39 PYQ Workbook
GENERAL GEOGRAPHY

102. Which of the following statements List I (Landform List II (Location)


concerning circum-pacific belt are correct? feature)
1. It is an active volcanic realm.
A. Alluvial fans 1. Mountainous areas
2. It is an active seismic realm.
3. It is a divergent plate boundary. B. ‘V’-shaped 2. Coasts
valleys
4. It is prone to Tsunami.
Select the correct answer using the code given C. Deltas 3. Lower reaches of
the river
below. [CAPF 2016]
(a) 1 and 2 only D. Ox-bow lakes 4. Mountain foothills
(b) 3 and 4 only Codes: [CAPF 2015]
(c) 1, 2 and 4 only A B C D
(d) 1, 2, 3, 4 (a) 4 1 2 3
103. Various landforms observed on the surface (b) 4 2 1 3
of the Earth are due to the (c) 3 2 1 4
1. differences in the type and structure of (d) 3 1 2 4
the crustal material.
106. Match List I with List II and select the
2. differences in the land forming processes.
correct answer using the code given below
3. differential rates of the processes at
the lists.
different places on the Earth’s surface.
Select the correct answer using the code given List I (Tropical List II (Location)
below. [CAPF 2015] cyclone)
(a) 1 only A. Cyclones 1. USA
(b) 1 and 2 only B. Hurricanes 2. East Asia
(c) 2 and 3 only
(d) 1, 2 and 3 C. Typhoons 3. Australia
104. Match List I with List II and select the D. Willy-willies 4. India
correct answer using the code given below Codes: [CAPF 2015]
the lists. A B C D
List I (Volcano List II (Location) (a) 4 2 1 3
Type) (b) 4 1 2 3
A. Shield Volcano 1. Indonesia (c) 3 1 2 4
(d) 3 2 1 4
B. Composite 2. India
Volcano 107. Which one among the following is NOT a
C. Caldera 3. Hawaii minor plate? [CAPF 2015]
(a) Nazca
D. Flood Basalt 4. Philippines
Provinces (b) Arabia
(c) Philippines
Codes: [CAPF 2015]
(d) Antarctica
A B C D
(a) 2 4 1 3 108. Which one among the following plains is
(b) 2 1 4 3 associated with Limestone Topography?
(c) 3 1 4 2 [CAPF 2015]
(d) 3 4 1 2 (a) Bajada plain
105. Match List I with List II and select the (b) Alluvial plain
correct answer using the code given below (c) Karst plain
the lists. (d) Pene plain

PYQ Workbook 40
GENERAL GEOGRAPHY

SOLUTIONS
nanometers and includes the colors we perceive, such as red,
2.1. UPSC CSE Previous Years’ Questions orange, yellow, green, blue, and violet. Infrared radiation,
1. Solution: (a) on the other hand, has longer wavelengths than visible light
and is not visible to the human eye. It lies just beyond the
Exp) Option a is the correct answer.
red end of the visible spectrum and is typically divided into
Option a is correct: The Congo Basin is a vast region
near-infrared, mid-infrared, and far-infrared regions based
in Central Africa, encompassing multiple countries. It
on wavelength.
is primarily characterized by the Congo Rainforest, the
second-largest rainforest in the world. The Congo Basin 3. Solution: (c)
spans across six countries—Cameroon, Central African Exp) Option c is the correct answer
Republic, Democratic Republic of the Congo, Republic of Statement 1 is correct: P waves, or Primary waves, are the
the Congo, Equatorial Guinea and Gabon. first waves to arrive at a seismograph. P waves are the fastest
Option b is incorrect: Nigeria is located in West Africa and seismic waves and can move through solid, liquid, or gas.
is not part of the Congo Basin. It is situated to the west of They leave behind a trail of compressions and rarefactions
Cameroon. on the medium they move through. P waves are also called
pressure waves for this reason.
Option c is incorrect: South Sudan is located in East-
Central Africa, and while it is geographically closer to the
Congo Basin compared to Nigeria, it does not fall within the
basin itself. The Congo Basin is mainly found to the south of
South Sudan.
Option d is incorrect: Uganda is also located in East Africa,
north of Rwanda and Tanzania. While it is relatively close to
the eastern edge of the Congo Basin, it does not lie entirely
within the basin.

2. Solution: (c)
Exp) Option c is the correct answer.
Option a is incorrect: While it is true that the equator
receives more direct sunlight compared to the poles, the Statement 2 is correct: P waves are longitudinal waves,
actual difference in insolation is not as large as ten times. which means that the particle motion occurs parallel to the
The difference in insolation between the equator and poles direction of wave propagation. These waves are compression
is primarily due to the curvature of the Earth and the tilt of waves, where particles oscillate back and forth in the
its axis, resulting in varying angles at which sunlight reaches same direction that the wave is traveling. As a result, the
different latitudes. individual particles vibrate to and fro in the direction of
Option b is incorrect: Insolation refers to the total solar wave propagation. On the other hand, S waves are transverse
radiation received at the Earth’s surface, which includes a waves, where particle motion occurs perpendicular at right
broad spectrum of electromagnetic radiation. While infrared angles) to the direction of wave propagation. These waves
cause the particles to move up and down or side to side
radiation is a significant component of solar radiation, it
perpendicular to the direction of the wave. Therefore, in S
does not account for two-thirds of the insolation. Insolation
waves, the particles vibrate up and down at right angles to
includes visible light, ultraviolet UV) radiation, and other
the direction of wave propagation.
wavelengths as well.
Option c is correct: Water vapor is a potent absorber of 4. Solution: (b)
infrared radiation, particularly in certain wavelength bands. Exp) Option b is the correct answer.
In the lower atmosphere, where water vapor concentration is Statement 1 is incorrect. Of the total freshwater, 69% resides
relatively higher, it absorbs a significant amount of infrared in glaciers, 30% underground, and less than 1% is located
radiation emitted by the Earth’s surface. This absorption in lakes, rivers, and swamps. Thus, the amount of water in
contributes to the greenhouse effect and plays a crucial role rivers and lakes is less than amount of groundwater.
in regulating the Earth’s temperature. Statement 2 is correct. Of the total freshwater, 69% resides
Option d is incorrect: Infrared waves and visible light in glaciers and 30% underground. Polar ice caps and
waves are distinct parts of the electromagnetic spectrum. glaciers has more water than groundwater.
The visible spectrum ranges from approximately 400 to 700
5. Solution: (c)

41 PYQ Workbook
GENERAL GEOGRAPHY

Exp) Option c is the correct answer. Option 3 is correct. Due to the gravitational forces, the Sun,
Statement 1 is correct. The study of paleomagnetism (the planets, and their moons interact with each other in a way,
magnetism residual in rocks) has provided evidence that so that the Solar system is stable. The tides in the oceans are
the earth’s magnetic field has reversed direction every few caused by gravitational force which leads to erosion and
hundred thousand years— i.e., the north and south poles deposition.
have switched polarity. Some scientists as well as psychics Option 4 is correct. Movements of plates leads to formation
predict this phenomenon may occur in the near geologic of islands, mountains, volcanoes, earthquakes.
future. Option 5 is correct. Rotation of earth causes day and night, it
Statement 2 is incorrect. When Earth was first created, also results in pressure differences due to differential heating
there was no oxygen whatsoever. Our Earth’s atmosphere at of earth surface. This cause winds to flow and weathering
that time consisted of mostly methane and carbon dioxide. of surfaces, as prominently seen in deserts. Rotation also
The early atmosphere largely contained water vapour, produces Coriolis affect, which affects the movement of
nitrogen, carbon dioxide, methane, and ammonia. These ocean currents.
gases and were released from the interior solid earth during Option 6 is correct. Revolution of the earth results in
its cooling off. The process through which the gases were different seasons. When surfaces are exposed to different
outpoured from the interior is called degassing. conditions it affects their structural integrity and aids in
Statement 3 is correct. The process of photosynthesis erosion.
modified the early atmosphere of the Earth. About 2.4
9. Solution: (b)
billion years ago, a type of organism called cyanobacteria
evolved on the early Earth and began carrying out Exp) Option b is the correct answer.
photosynthesis. Photosynthesis uses carbon dioxide and Carbon, Hydrogen, Nitrogen were primarily responsible for
energy from the Sun to produce sugar and oxygen. The the origin of life on the Earth.
cyanobacteria were very simple organisms but performed an Hydrogen, Nitrogen, Oxygen, Phosphorus and Sulfur
important role in changing Earth’s early atmosphere. They combined with Carbon generated the first group of
carried out photosynthesis to produce the materials they compounds that eventually formed the chemical basis of life.
needed to grow. They gave off oxygen to the atmosphere as
The matter within every living Earth creature mainly consists
they did this.
of just four chemical elements: hydrogen, oxygen, carbon,
6. Solution: (c) and nitrogen.
Exp) Option c is the correct answer. 10. Solution: (c)
Continental drift and Glacial cycles had an impact on the Exp) Option c is the correct answer.
climate of different regions and thus influenced the evolution
The pole star also called the North Star indicates the North
of organisms on this planet.
direction. Therefore, if the person walks keeping the polestar
7. Solution: (b) in his left, he will be travelling in the East direction.
Exp) Option b is the correct answer. 11. Solution: (b)
Geographical Feature Region Exp) Option b is the correct answer.
Abyssinian Plateau Ethiopia The equator is an imaginary line that circles the Earth,
Atlas Mountains North-Western Africa splitting it into the Northern and Southern hemispheres.
Because of this, the intersection point of any location by
Guiana Highlands Omicoo basin (Brazil)
the equator is equidistant from the North and South poles.
Okavango Basin South-West Africa Find out what life is like for countries along the equator.
Abyssinian Plateau is also known as Ethiopian Highlands. Although the equator stretches 40,075 kilometres around the
world, it travels through just 13 countries, though only the
8. Solution: (d) water controlled by two of these rather than the landmasses
Exp) Option d is the correct answer answer. themselves.
All the above are responsible for bringing dynamic changes
on the surface of the earth. Transfer from potential to kinetic
energy results in dynamic changes.
Option 1 is correct. Electromagnetic radiation from the sun
is source of heat and energy on Earth. This energy is source
of all life on earth. It leads to precipitation and weathering of
different surfaces.
Option 2 is correct. Geothermal energy is heat derived
within the sub-surface of the earth. It leads to melting of ice
caps, volcanism and other dynamic changes.

PYQ Workbook 42
GENERAL GEOGRAPHY

The Equator passes through 13 countries: Ecuador, side of the mountains and hence the region is also hot and
Colombia, Brazil, Sao Tome & Principe, Gabon, Republic dry.
of the Congo, Democratic Republic of the Congo, Uganda,
Kenya, Somalia, Maldives, Indonesia and Kiribati. At least
half of these countries rank among the poorest in the world.

12. Solution: (c)


Exp) Option c is the correct answer.
The core is made almost entirely of metal—specifically, iron
and nickel.
The Core of Earth is the very hot, very dense centre of our
planet. The ball-shaped core lies beneath the cool, brittle
crust and the mostly-solid mantle. The core is found about
2,900 kilometres below Earth’s surface, and has a radius of
about 3,485 kilometres.
When Earth was formed about 4.5 billion years ago, it was a Statement 2 is incorrect. The average annual rainfall in East
uniform ball of hot rock. Radioactive decay and leftover heat Himalayan region is 10,000 mm (390 inches). The Bay of
from planetary formation caused the ball to get even hotter. Bengal Branch of Southwest Monsoon is the cause of high
Eventually, after about 500 million years, Earth’s temperature rainfall in the north-east Himalayas. The northeast monsoon
heated to the melting point of iron—about 1,538° Celsius. is confined to south India and brings rainfall from October
This pivotal moment in Earth’s history is called the iron to December over Tamil Nadu, Puducherry, Karaikal, Yanam,
catastrophe. Andhra Pradesh, Kerala, Mahe and south interior Karnataka.
The shorthand used for the core’s iron-nickel alloys is simply 14. Solution: (b)
the elements’ chemical symbols—NiFe.
Exp) Option b is the correct answer.
The Great Victoria Desert is a sparsely populated desert
ecoregion and interim Australian bioregion in Western
Australia and South Australia. It is the largest desert in
Australia and consists of many small sandhills, grassland
plains, areas with a closely packed surface of pebbles (called
desert pavement or gibber plains), and salt lakes.
The Grand Canyon is a steep-sided canyon carved by the
Colorado River in Arizona, United States. The Grand
Canyon is 277 miles (446 km) long, up to 18 miles (29 km)
wide and attains a depth of over a mile (6,093 feet or 1,857
meters).
Elements that dissolve in iron, called siderophiles, are also Lake Winnipeg is a very large, relatively shallow
found in the core. Because these elements are found much 24,514-square-kilometre (9,465 sq mi) lake in North America,
more rarely on Earth’s crust, many siderophiles are classified in the province of Manitoba, Canada. Its southern end is
as “precious metals.” Siderophile elements include gold, about 55 kilometres (34 mi) north of the city of Winnipeg.
platinum, and cobalt. Another key element in Earth’s core Lake Winnipeg is Canada’s sixth-largest freshwater lake and
is sulphur—in fact 90% of the sulphur on Earth is found the third-largest freshwater lake contained entirely within
in the core. Canada. It is the eleventh-largest freshwater lake on Earth.
The lake’s east side has pristine boreal forests and rivers
13. Solution: (a) that were in 2018 inscribed as Pimachiowin Aki, a UNESCO
Exp) Option a is the correct answer. World Heritage Site.
Statement 1 is correct. Tropical deserts are mostly located The Southern Alps are a mountain range extending along
between 10 0 to 30 0 north and south of Equator on the much of the length of New Zealand’s South Island, reaching
western margins of the continent. These regions lie in its greatest elevations near the range’s western side. The
the belt of the trade winds. These winds blow from east name “Southern Alps” generally refers to the entire range,
to west. These winds shed their moisture on the eastern although separate names are given to many of the smaller
side of the continents. By the time they reach to the western ranges that form part of it.
margin of the continents, they had shed their moisture and
15. Solution: (b)
hence do not cause any rainfall. Thus, most of the tropical
deserts like The Sahara Desert, the Kalahari Desert, West Exp) Option b is the correct answer.
Australian desert etc. are located on the western margin of Pair 1 is incorrectly matched. Continental drift is the
the continent. Moreover, these deserts also lie on the leeward hypothesis that the Earth’s continents have moved over

43 PYQ Workbook
GENERAL GEOGRAPHY

geologic time relative to each other, thus appearing to currents and waves to deposit them as beaches along the
have “drifted” across the ocean bed. The speculation that shore and as bars (long ridges of sand and/ or shingle parallel
continents might have ‘drifted’ was first put forward by to the coast) in the nearshore zone. Bars are submerged
Abraham Ortelius in 1596. A pioneer of the modern view features and when bars show up above water, they are
of mobilise was the Austrian geologist, Otto Ampferer. The called barrier bars.
concept was independently and more fully developed by Option 4 is correct - Barrier bar which gets keyed up to
Alfred Wegener in 1912, but his hypothesis was rejected by the headland of a bay is called a spit. When barrier bars
many for lack of any motive mechanism. Arthur Holmes and spits form at the mouth of a bay and block it, a lagoon
later proposed mantle convection for that mechanism. The form. The lagoons would gradually get filled up by sediments
idea of continental drift has since been subsumed into the from the land giving rise to a coastal plain.
science of plate tectonics, which studies the movement of the
continents as they ride on plates of the Earth’s lithosphere. 17. Solution: (d)

Edwin Hubble contribution was in describing the Exp) Option d is the correct answer.
universe’s rate of expansion. The equator is an imaginary line of latitude on the spheroid
Pair 2 is incorrectly matched. The expansion of the (Earth), dividing it into southern and northern hemispheres.
universe is the increase in distance between any two given On earth, this line is approximately 24,901 miles of which
gravitationally unbound parts of the observable universe only 21.3% lies on land and the rest on the water. The Equator
with time. Edwin Hubble contributed in development of runs through several countries, mainly in Africa and South
America. However, Indonesia (Southeast Asia) is straddled
expansion of universe theory. Hubble’s brilliant observation
the most by the equator.
was that the red shift of galaxies was directly proportional
to the distance of the galaxy from earth. That meant that Singapore is around one-and-a-half degrees north of the
things farther away from Earth were moving away faster. In equator, lying entirely between the 1st and 2nd parallels.
other words, the universe must be expanding. He announced It is nearest to the equator among the above given cities.
his finding in 1929.
Pair 3 is correctly matched. Photoelectric effect is
phenomenon in which electrically charged particles
are released from or within a material when it absorbs
electromagnetic radiation. The effect is often defined as the
ejection of electrons from a metal plate when light falls on it.
In a broader definition, the radiant energy may be infrared,
visible, or ultraviolet light, X-rays, or gamma rays; the
material may be a solid, liquid, or gas. The released particles
may be ions (electrically charged atoms or molecules) as
well as electrons. The phenomenon was fundamentally
significant in the development of modern physics because
of the puzzling questions it raised about the nature of
light—particle versus wavelike behaviour—that were finally Jakarta is the capital of Indonesia. Lying on the northwest
coast of Java (the world’s most populous island), its
resolved by Albert Einstein in 1905.
Coordinates are 6°12S 106°49E.
16. Solution: (d) Colombo is the capital of Sri Lanka by population. Its
Exp) Option d is the correct answer Coordinates: 6°5604N 79°5034E.
Option 1 is correct - Along high rocky coasts, waves break Manila, is the capital of the Philippines, and its coordinates
with great force against the land shaping the hill sides into are 14°36N 120°59E.
cliffs. With constant pounding by waves, the cliffs recede
leaving a wave-cut platform in front of the sea cliff. After 18. Solution: (b)
a considerable period of cliff development, a wave-built Exp) Option b is the correct answer.
terrace would develop in front of wave-cut terrace. The international date line passes through the mid-Pacific
Option 2 is correct - The constant erosion of rocky headlands Ocean and roughly follows a 180 degrees longitude north-
may produce a variety of particular geomorphic structures, south line on the Earth. It is located halfway around the
including sea arches and sea stacks. With prolonged world from the prime meridian — the 0 degrees longitude
erosion, sea arches may collapse to form sea stacks—steep line in Greenwich, England.
pillars of rock a short distance from the mainland. Both sea Bering Strait: The Bering Strait is a strait between the
stacks and sea arches are impermanent features that will Pacific and Arctic oceans, separating the Chukchi Peninsula
eventually disappear with continued erosion. of the Russian Far East from the Seward Peninsula of Alaska.
Option 3 is correct - As the erosion along the coast takes Bering Strait is nearest to the International Date line. The
place a good supply material becomes available to longshore International Date Line runs between the Diomede Islands

PYQ Workbook 44
GENERAL GEOGRAPHY

and St. Lawrence Islands located in the middle of the strait. Strait of Gibraltar: The Strait of Gibraltar also known as
The present Russia-United States maritime boundary is at the Straits of Gibraltar is a narrow strait that connects the
168° 58’ 37” W longitude, slightly south of the Arctic Circle Atlantic Ocean to the Mediterranean Sea and separates the
at about 65° 40’ N latitude. Iberian Peninsula in Europe from Morocco in Africa.

19. Solution: (a)


Exp) Option a is the correct answer.
Malacca Strait: The Strait of Malacca Straits of Malacca “Land area” refers to the total surface area of the land
is a narrow stretch of water, 580 mi (930 km) in length, of a geographical region or country (which may include
between the Malay Peninsula (Peninsular Malaysia) and the discontinuous sections of land such as islands).
Indonesian island of Sumatra.
Statement 1 is correct. Earth’s total planimetric (flat) land
area is approximately 147,539,063.133 km2 (57,505,693.767
sq. mi) which is about 29.2% of its total surface, including
that which is covered by ice. Water covers approximately
70.8% of planimetric Earth’s surface, mainly in the form of
oceans and ice formations; but this proportion is decreased
by the land’s increased terrain.
Statement 2 is incorrect. The ratio of land and water on
earth is approximately in the ratio of 1:2.
About 71 percent of the Earth’s surface is water-covered,
and the oceans hold about 96.5 percent of all Earth’s water.
Water also exists in the air as water vapor, in rivers and lakes,
Strait of Florida: The Straits of Florida, Florida Straits, or in icecaps and glaciers, in the ground as soil moisture and in
Florida Strait is a strait located south-southeast of the North aquifers, and even in you and your dog.
American mainland, generally accepted to be between the Statement 3 is correct. Pacific Ocean is the largest water
Gulf of Mexico and the Atlantic Ocean, and between the mass on the planet. It covers more than 30 percent of the
Florida Keys (U.S.) and Cuba. It is 93 mi (150 km) wide at the Earth’s surface. With a surface area of more than 155 million
narrowest point between Key West and the Cuban shore, and square kilometres (60 million square miles), this ocean basin
has been sounded to a depth of 6,000 feet. The strait carries is larger than the landmass of all the continents combined.
the Florida Current, the beginning of the Gulf Stream, from Additionally, it contains almost twice as much water as the
world’s second largest body of water, the Atlantic Ocean.
the Gulf of Mexico.
20. Solution: (b)
Exp) Option b is the correct answer.
The correct sequence of the given Continents in the
decreasing order of their percentage of Earth’s land are
Africa – North America – South America – Europe.
Asia: It is a landmass variously described as part of Eurasia
or as Earth’s largest and most populous continent in its
own right, located primarily in the Eastern and Northern
Hemispheres. It shares the continental landmass of Eurasia
with the continent of Europe and the continental landmass
of Afro-Eurasia with both Europe and Africa. Asia covers
an area of 44,579,000 square kilometres (17,212,000 sq mi),
about 30% of Earth’s total land area and 8.7% of the Earth’s
total surface area.

45 PYQ Workbook
GENERAL GEOGRAPHY

Africa is the world’s second-largest and second-most diplomat Lord St. Helens, a friend of explorer George
populous continent, after Asia in both cases. At about 30.3 Vancouver who surveyed the area in the late 18th century.
million km2 (11.7 million square miles) including adjacent It is part of the Cascade Volcanic Arc, a segment of the
islands, it covers 6% of Earth’s total surface area and 20% of Pacific Ring of Fire. The Cascade Volcanic Arc is a chain of
its land area. volcanoes that extends from British Columbia to California.
North America is a continent in the Northern Hemisphere Mount St. Helens is one of the most active volcanoes in the
and almost entirely within the Western Hemisphere. It can Cascade Volcanic Arc, and it has erupted over 20 times in the
also be described as the northern subcontinent of a single past 2,000 years.
continent, America. North America covers an area of about
24,709,000 square kilometres (9,540,000 square miles), 22. Solution: (d)
about 16.5% of the Earth’s land area and about 4.8% of its Exp) Option d is the correct answer.
total surface. North America is the third-largest continent by Statements 1 and 2 are correct: Sedimentary rocks are
area, following Asia and Africa, and the fourth by population formed when sediments are deposited by the hydrological
after Asia, Africa, and Europe. system. Sediments are weathered and eroded from preexisting
South America is a continent entirely in the Western rocks and then transported by water, wind, or ice. When the
Hemisphere and mostly in the Southern Hemisphere, with a sediments settle in a body of water, they form layers. Over
relatively small portion in the Northern Hemisphere. time, these layers are compacted and cemented together to
Antarctica is Earth’s southernmost continent. It contains the form sedimentary rock.
geographic South Pole and is situated in the Antarctic region Statement 3 is correct: Fossils are the preserved remains
of the Southern Hemisphere, almost entirely south of the of plants and animals. Fossils can be found in sedimentary
Antarctic Circle, and is surrounded by the Southern Ocean. rocks because sedimentary rocks are formed from sediments
At 14,200,000 square kilometres (5,500,000 square miles), that were deposited in water. When plants and animals die in
it is the fifth-largest continent and nearly twice the size of water, their remains can be buried in sediment. Over time,
Australia.
the sediment can harden into rock, preserving the fossils of
Europe is a landmass variously recognised as part of Eurasia the plants and animals.
or a continent in its own right, located entirely in the Northern
Statement 4 is correct: Sedimentary rocks are often
Hemisphere and mostly in the Eastern Hemisphere. Europe
described as being bedded or stratified. This is because
covers about 10.18 million km 2 (3.93 million sq mi), or 2% of
they are made up of layers of sediment. The layers can be of
the Earth's surface (6.8% of land area), making it the second-
different thicknesses and colors, and they can be made up of
smallest continent.
different types of sediment.
Australia is a sovereign country comprising the mainland
of the Australian continent, the island of Tasmania, and Important Tips
numerous smaller islands.[13] It is the largest country The hydrological system is the cycle of water on Earth.
by area in Oceania and the world’s sixth-largest country. It includes the evaporation of water from the oceans,
Australia’s population of nearly 26 million, in an area of the transportation of water vapor in the atmosphere, the
7,617,930 square kilometres. precipitation of water back to Earth, and the runoff of
water back to the oceans.
Weathering is the process by which rocks are broken down
into smaller pieces. Weathering can be caused by physical
processes, such as changes in temperature or pressure, or
by chemical processes, such as the reaction of rocks with
water or acids.

23. Solution: (a)


Exp) Option a is the correct answer.
Option a is correct: The Baltic Sea is a shallow, brackish sea
that is located between northern Europe and Scandinavia. It
is the world’s largest enclosed inland sea, and it is connected
to the North Sea by the Kattegat and Skagerrak straits. The
Baltic Sea is not volcanically active because it is not located
21. Solution: (d) on a plate boundary. The Baltic Sea is part of the Eurasian
Exp) Option d is the correct answer. Plate, and there are no active volcanoes on that plate.
Mount St. Helens is an active stratovolcano located in Options b, c, and d are incorrect: The Black Sea, Caribbean
Washington, in the Pacific Northwest region of the United Sea, and Caspian Sea are all located on plate boundaries,
States. It takes its English name from that of the British and they are all volcanically active.

PYQ Workbook 46
GENERAL GEOGRAPHY

The Black Sea is located on the boundary between the Option C matches with option 2: Paricutin is a cinder cone
Eurasian Plate and the Anatolian Plate. There are several volcano located in the state of Michoacán, Mexico. It was
active volcanoes in the Black Sea region, including Mount created by a volcanic eruption in 1943, and it is one of the
Etna and Mount Vesuvius. youngest volcanoes in the world.
The Caribbean Sea is located on the boundary between Option D matches with option 3: Taal Volcano is a complex
the North American Plate and the Caribbean Plate. There volcano located on Taal Island in Taal Lake, Philippines. It
are several active volcanoes in the Caribbean Sea region, is one of the most active volcanoes in the Philippines, and it
including Mount Pelee and Soufriere Hills. has erupted over 30 times in recorded history.
The Caspian Sea is located on the boundary between the
Eurasian Plate and the Arabian Plate. There are several 2.2. Other Examination Previous Years’
active volcanoes in the Caspian Sea region, including Mount Questions
Damavand and Mount Sabalan. 26. Solution: (d)
24. Solution: (a) Exp) Option d is the correct answer.
Exp) Option a is the correct answer. The Tropical Savanna Region is known as the ‘Land of
Big Games’. The savanna is known as the ‘land of big game
Statement 1 is correct: The intensity of an earthquake is
country’ as thousands of animals are trapped or killed each
measured by the Mercalli Scale. It measures the energy
year by people from all over the world. Carnivorous animals
released during earthquakes. The intensity scale takes into
like lions, leopards, and tigers are present in the region and
account the visible damage caused by the event. feed on deer, zebra and other herbivores.
Statement 2 is correct: Earthquake’s magnitude is the
measurement of energy released. The more magnitude 27. Solution: (b)
on the Richter scale, the more energy released by the Exp) Option b is the correct answer.
earthquakes. A Seismograph is an instrument used to detect and record
Statement 3 is correct: The Richter magnitude of an seismic waves. Seismic waves are propagating vibrations that
earthquake is determined from the logarithm of the carry energy from the source of an earthquake outward in all
amplitude of waves recorded by seismographs. Earthquake directions. They travel through the interior of the Earth and
magnitude is a measure of the size or amplitude of the can be measured with seismographs. A Seismometer is the
seismic waves generated by an earthquake. internal part of the seismograph.

Statement 4 is incorrect: Increase in magnitude by one Important Tips


integer on Richter scale represents a tenfold increase in the A Seismogram is the recording of the ground shaking at
measured amplitude and 32 times not 100 times) more the specific location of the instrument. On a seismogram,
energy released. the horizontal axis measures time in seconds and the
vertical axis measures ground displacement in millimeters.
Important Tips
A Seismoscope is an instrument that gives a qualitative
Difference between Richter Scale and Mercalli scale:
measure of the oscillatory motion produced by an
Richter Scale is used to measure the scale of absolute earthquake or other disturbance of the earth’s surface.
‘Magnitude’ of an earthquake based on the amount of Unlike the seismograph, it lacks a device to calibrate the
seismic energy released whereas Mercalli scale measures time.
the ‘Intensity’ of an earthquake based on based on the
amount of destruction caused. 28. Solution: (c)
The Richter scale ranges from a value of 1 to 10. It is a Exp) Option c is the correct answer.
logarithmic scale. The Mercalli scale ranges from a value Option a is incorrect: Mount Pinatubo is a stratovolcano
of 1 to 12. located in the Zambales Mountains of Luzon, Philippines,
about 100 northwest of Manila. The current height of Mount
25. Solution: (b)
Pinatubo is 1,486 meters 4,875 feet). This is lower than
Exp) Option b is the correct answer. its pre-eruption height of 1,745 meters 5,725 feet), as the
Option A matches with option 4: Mount Rainier is eruption caused the summit of the volcano to collapse.
a stratovolcano located in the Cascade Mountains of Option b is incorrect: Mount Kilimanjaro is a dormant
Washington, USA. It is the highest mountain in the state, stratovolcano in the Kilimanjaro National Park, Tanzania. It
and it is also one of the most active volcanoes in the Cascade is the highest mountain in Africa at 5,895 meters 19,341 ft)
Volcanic Arc. above sea level, and the highest free-standing mountain in
Option B matches with option 1: Etna is a stratovolcano the world.
located on the island of Sicily, Italy. It is one of the most Option c is correct: Mount Tall, also known as Ojos del
active volcanoes in Europe, and it has erupted over 200 times Salado, is the highest volcanic mountain in the world,
in recorded history. with a summit elevation of 6,893 meters 22,615 feet). It is

47 PYQ Workbook
GENERAL GEOGRAPHY

located on the border of Argentina and Chile in the Andes Important Tips
Mountains.
Mauna Loa:
Option d is incorrect: Mount Cotopaxi is an active • It is a shield volcano, which means that it has a broad,
stratovolcano in the Andes Mountains, located in central gentle slope.
Ecuador, about 50 kilometers 31 miles) south of Quito. It • Shield volcanoes are formed by the eruption of fluid
is the second highest summit in Ecuador, reaching a height lava that flows long distances before it cools and
of 5,897 meters 19,347 feet). Cotopaxi is among the highest solidifies.
active volcanoes in the world. • Mauna Loa’s lava flows have created the majority of the
island of Hawaii.
29. Solution: (d)
• Mauna Loa last erupted in 1984, and it is possible that
Exp) Option d is the correct answer.
it could erupt again at any time.
Option A matches with option 2: Mount Kinabalu is the • It rises to a height of 4,170 meters 13,680 feet) above
highest mountain in Malaysia and Borneo. The mountain sea level, making it the second-tallest mountain in
is part of the Kinabalu National Park, which was designated Hawaii, after Mauna Kea.
a UNESCO World Heritage Site in 2000.
32. Solution: (a)
Option B matches with option 4: Elburz is the highest
Exp) Option a is the correct answer.
mountain range in Iran.
Statement 1 is correct: Deserts can be effective sources
Option C matches with option 1: Aconcagua is the highest
for the production of perennial energy. Solar energy
mountain in the Americas, with a summit elevation of 6,961 and Wind energy is abundantly available in this region
meters 22,838 feet). It is located in the Andes Mountains, which can be exploited for as perennial source of energy
on the border between Argentina and Chile. by required infrastructure. Deserts are heat producers,
Option D matches with option 3: Kilimanjaro is the highest reflecting around 60% to 70% of the solar energy that falls
mountain in Africa, with a summit elevation of 5,895 meters on them straight back into the atmosphere.
19,341 feet). It is located in northeastern Tanzania, near the Statement 2 is correct: Deserts can be a potential source for
border with Kenya. Kilimanjaro is a dormant volcano. generating sustainable energy because Deserts receive more
energy from the Sun in six hours than mankind consumes in
30. Solution: (c) a year. Statement 2 provides correct reason for statement 1.
Exp) Option c is the correct answer. Hence, option a is the correct answer.
Option A matches with option 2: Sabankaya is an active 33. Solution: (d)
stratovolcano located in the Andes Mountains of southern Exp) Option d is the correct answer.
Peru. It is one of the most active volcanoes in Peru, and it has
• The Savanna is not a temperate grassland. Savannas are
erupted over 20 times in recorded history. another name for tropical grasslands. Tropical grasslands
Option B matches with option 1: Mount Etna is an active are located near the equator, between the Tropic of
stratovolcano located on the island of Sicily, Italy. It is one Cancer and the Tropic of Capricorn.
of the most active volcanoes in Europe, and it has erupted • The main temperate grasslands are the Steppe in
over 200 times in recorded history. Eurasia, the Prairies of North America, the Downs of
Option C matches with option 4: Colima is an active Australia and New Zealand, Veld in South Africa and the
stratovolcano located in western Mexico. It is one of the Pampas of Argentina.
most active volcanoes in Mexico, and it has erupted over 40
times in recorded history
Option D matches with option 3: Merapi is an active
stratovolcano located on the border between Central Java
and Yogyakarta provinces in Indonesia. It is one of the
most active volcanoes in Indonesia, and it has erupted over
60 times in recorded history.

31. Solution: (b)


Exp) Option b is the correct answer.
Mauna Loa is an active volcano located on the island of
Hawaii in the Pacific Ocean. It is the largest subaerial 34. Solution: (d)
volcano on Earth, with a volume of over 18,000 cubic miles. Exp) Option d is the correct answer.
Mauna Loa is also one of the most active volcanoes on Earth, Death Valley is a structural depression rift valley) in south
having erupted over 30 times in the past 1,500 years. California in the USA. It is the lowest, hottest, and driest

PYQ Workbook 48
GENERAL GEOGRAPHY

portion of the North American continent. A rift valley is a 36. Solution: (a)
lowland region that forms where Earth’s tectonic plates move Exp) Option a is the correct answer.
apart. The ‘Ring of Fire’ is an arc around the Pacific Ocean where
many volcanoes and earthquakes are formed. 75% of Earth’s
volcanoes and 90% of Earth’s earthquakes are located along
the Ring of Fire. Ring of Fire is not associated with forest
fire.

Important Tips
Anticlinal Valley: A valley occupies the center of the
anticline. An anticlinal valley is where a fault runs through
the center of the anticline, and the fault makes the rock
37. Solution: (d)
erode into a valley. An anticlinal valley is where the
geological structure is reversed from the normal. Exp) Option d is the correct answer.
Option 1 is incorrect: Aconcagua is a dormant volcano,
which means that it has not erupted in recent history, but it
is possible that it could erupt again in the future.
Option 2 is correct: Cotopaxi is a stratovolcano located
in Ecuador. It is one of the most active volcanoes in South
America, and it has erupted over 50 times in recorded
history. The last eruption of Cotopaxi occurred in 2015.
Option 3 is correct: Etna is a stratovolcano located in Italy.
It is one of the most active volcanoes in Europe, and it has
erupted over 200 times in recorded history. The last eruption
of Etna occurred in 2022.
Option 4 is correct: Fujiyama is a stratovolcano located in
Japan. It is the highest mountain in Japan, and it is also one
35. Solution: (a) of the most active volcanoes in the country. Fujiyama has
erupted over 100 times in recorded history. The last eruption
Exp) Option a is the correct answer.
of Fujiyama occurred in 1707.
A savanna biome is a widespread grassland area that contains
small shrubs and a few trees. The largest savanna is located 38. Solution: (b)

in Africa. Nearly half of the continent of Africa is covered Exp) Option b is the correct answer.
with savanna grasslands. Other major savannas are located The Northwestern Indian Subcontinent NWIS) referred to
South America, India, and northern Australia. a region including Southeast Afghanistan, Pakistan, and
northwestern India. The North-Western Region of the Indian
Sub-continent is susceptible to earthquake activity because
of plate tectonic activity. The main cause of earthquakes is
the Indian plate’s gradual shift towards the Eurasian plate.

39. Solution: (c)


Exp) Option c is the correct answer.
Mount Stromboli is a volcano located in the Aeolian Islands
of Italy. It is one of the most active volcanoes in the world,
and it is known for its frequent eruptions, which can be seen
from a distance at night. Stromboli has been nicknamed

49 PYQ Workbook
GENERAL GEOGRAPHY

“The lighthouse of the Mediterranean” because its 42. Solution: (b)


eruptions have helped sailors navigate the sea for centuries. Exp) Option b is the correct answer.
The volcano’s eruptions are typically small and produce ash
and lava bombs, but it has also erupted more violently in the Young Fold mountains are formed when Earth’s tectonic
past. plates collide against each other, and one plate is forced
under the other. This process, called subduction, creates a
Important Tips massive fold in the Earth’s crust. Rocks near the continental
• Mount Etna is a volcano located in Sicily, Italy. It is crust’s edge are weaker as compared to that of the
one of the most active volcanoes in Europe, but it is not continental’s center. Fold mountains are found near the edges
as active as Stromboli. of continents because that’s where these plate collisions
• Mount Pele is a volcano located in Hawaii. It is one happen. In these areas, the Earth’s crust gets squished
of the most active volcanoes in the world, but it is not together, forming rocky outcrops, hills, mountains, and
located in the Mediterranean Sea. even entire mountain ranges. The Himalayas, the Alps,
• Mount Vesuvius is a volcano located in Italy. It is one and the Andes are all examples of young folded mountains.
of the most famous volcanoes in the world, but it is not The highest peak in the world, Mount Everest, is located in
as active as Stromboli. the Himalayas, a young folded mountain range. Other high
peaks in the world, such as K2, Kangchenjunga, and Lhotse,
40. Solution: (a)
are also located in young folded mountain ranges.
Exp) Option a is the correct answer.
Pair a is incorrect: Downs are the temperate grassland not
Tropical grassland) of Australia.
Pair b is correct: Steppes are the temperate grassland.
Steppe grasslands are found in Eastern Europe and Central
Asia.
Pair c is correct: Selva is a Tropical Forest. It is thick
equatorial forest or rain forest. It is found in the Amazon
basin. Important Tips
Pair d is correct: Taiga is a Temperate Forest. Taigas are cold Some other types of mountains:
biomes where few plants grow due to the nutrient-poor soil, Block Mountains: Block Mountains are formed when
coniferous trees are able to survive and cover most of the extensive areas of land break and shift vertically. The
biome. raised blocks are known as horsts, while the lowered
Important Tips ones are called graben. An illustration of Block Mountain
systems can be seen in the Rhine Valley and the Vosges
Savannah, Campos, and Llanos are examples of tropical
Mountains in Europe.
grasslands.
Volcanic Mountains: Volcanic mountains are the result
Pampas, Prairie, Veld, Steppe, and Down are examples of
of volcanic activity. For instance, Mt. Kilimanjaro in
temperate grasslands.
Africa and Mt. Fujiyama in Japan are prime examples of
41. Solution: (c) volcanic mountains.
Exp) Option c is the correct answer.
43. Solution: (b)
Option a is incorrect: Temperature of deserts can be less
Exp) Option b is the correct answer.
than 420 C. Temperature generally falls at night in tropical
deserts. Most of the world’s deserts are located near 230 latitude. The
Option b is incorrect: Plants are also found in deserts heated equatorial air begins to descend in the area near 230
but adapted as per the environment. For e.g. the leaves of north latitude and 230 south latitudes. The descending air is
desert plants are modified into spines to reduce the rate of dense and begins to warm again, evaporating large amounts
transpiration from leaves. of water from the land surface. The resulting climate is very
Option c is correct: Most experts agree that a desert is an dry.
area of land that receives not more than 25 cm 10 inches) of 44. Solution: (a)
precipitation a year. The amount of evaporation in a desert
often greatly exceeds the annual rainfall. In all deserts, there Exp) Option a is the correct answer.
is little water available for plants and other organisms. The Ganges Brahmaputra Delta is the largest delta in
Option d is incorrect: Sand barrows are commonly found in the world, covering an area of more than 105,000 square
deserts, but it is a necessary condition but not sufficient kilometers. It is formed by the confluence of the Ganges, the
condition. Sand burrows can also be found in the region Brahmaputra and the Meghna rivers, which drain a catchment
outside the deserts such in the coastal areas, but they are not of about 1.72 million square kilometers in the south of the
considered deserts. Himalayas. The delta is shared by India and Bangladesh and

PYQ Workbook 50
GENERAL GEOGRAPHY

is one of the most fertile and densely populated regions in 47. Solution: (b)
the world. Exp) Option b is the correct answer.
Important Tips Statement 1 is correct: The shadow zone of one earthquake is
totally different from the shadow zone of another earthquake
• Mississippi-Missouri is a river system formed by the
as they are on different plates.
confluence of the Mississippi River and the Missouri
River in the United States. The Mississippi River is Statement 2 is incorrect: Seismometers do not record ‘S’
the second-longest river in North America, after the waves beyond 105° from earthquake’s epicenter whereas
Missouri River, and the fourth-longest river in the the seismographs located beyond 145° from the epicenter
world, after the Nile, the Amazon, and the Yangtze. record the arrival of P-waves. The seismographs located at
• The Yangtze River is the longest river in Asia and any distance within 105° from the epicenter recorded the
the third-longest in the world, after the Nile and the arrival of both P and S-waves. A zone between 105° and 145°
Amazon. The Yangtze originates from the Tanggula from epicenter is identified as the shadow zone for both the
Mountains in Qinghai province and flows eastward for types of waves.
about 3,915 miles 6,300 km) to the East China Sea. Important Tips
• Huang He is another name for the Yellow River, which • Earthquake waves get recorded in seismographs
is the second-longest river in China, after the Yangtze, located at far off locations. However, there exist some
and the sixth-longest river system in Earth, after specific areas where the waves are not reported. Such a
the Nile, the Amazon, the Yangtze, the Mississippi- zone is called the ‘shadow zone’.
Missouri, and the Ob-Irtysh.
• The shadow zone of S-wave is much larger than that of
45. Solution: (c) the P-waves.

Exp) Option c is the correct answer.


Option c is correct: Igneous rocks are formed when molten
rock magma) cools and solidifies. The rate at which the
magma cools determines whether the rock will be crystalline
or non-crystalline. If the magma cools slowly, the minerals
in the magma have time to crystallize, resulting in a
crystalline rock. If the magma cools quickly, the minerals
do not have time to crystallize, resulting in a non-crystalline
rock. Examples of crystalline igneous rocks include granite,
basalt, and gabbro. Examples of non-crystalline igneous
48. Solution: (b)
rocks include obsidian and pumice.
Exp) Option b is the correct answer.
Option a is incorrect: Fossils are the preserved remains
of plants and animals, and they are most commonly found The Karst region is a landscape which is underlain by
in sedimentary rocks. Sedimentary rocks are formed when limestone which has been eroded by dissolution producing
sediments are deposited and compacted over time. various landforms. Karst region is characterized by barren
Option b is incorrect: Igneous rocks are not typically porous. and rocky ground, caves, sink holes, stalactites, stalagmites,
Porosity is the amount of open space in a rock. Igneous rocks underground rivers, and the presence of surface streams and
are typically very dense and have very little open space. lakes.
Option d is incorrect: Igneous rocks are made up of silicates. Important Tips
Silicates are minerals that contain silicon and oxygen. • Potholes are circular depressions formed because
46. Solution: (d) of stream erosion aided by the abrasion of rock
fragments. These are common on the surface at lower
Exp) Option d is the correct answer.
side of waterfall where water falls from a height.
Statement 1 is incorrect: German meteorologist Alfred
• Blowholes are mainly located in regions with crevices
Wegener (not H.H. Hess) is credited as the first to give a
in the coastal rocks. Sea water enters these crevices
theory of Plate tectonics. He is also known as the ‘Father of
when there are powerful waves in the sea, and it is
Plate Tectonics’. Henry H. Hess gave the theory of ‘seafloor
released in the form of jet sprays through the blowhole.
spreading’.
• Playa is a dry, vegetation-free, flat area at the lowest
Statement 2 is correct: Seafloor spreading helps to explain
part of an undrained desert basin. A dry Playa is a
continental drift in the theory of plate tectonics. When
lakebed formed at the bottom of a desert basin,
oceanic plates diverge along the mid ocean ridge system,
sometimes temporarily covered with water.
tensional stress causes fractures to occur in the lithosphere.
This idea plays an important role in the development of the 49. Solution: (a)
theory of Plate Tectonics. Exp) Option a is the correct answer.

51 PYQ Workbook
GENERAL GEOGRAPHY

Option a is correct: Water vapor is the most abundant gas Option d is incorrect: Intraplate volcanoes are volcanoes
emitted from volcanoes, making up about 60% of all volcanic that occur within tectonic plates. They are less common than
gases. Other gases that are emitted from volcanoes include volcanoes that occur at plate boundaries, but they can be
carbon dioxide, sulfur dioxide, hydrogen sulfide, and very destructive.
hydrogen halides.
52. Solution: (c)
Option b is incorrect: Helium is a rare gas that is not
typically emitted from volcanoes. Exp) Option c is the correct answer.
Options c and d are incorrect: Sulfur dioxide and carbon Steppes: The world’s largest steppe region is often referred to
dioxide are both greenhouse gases that can contribute to as “the Great Steppe”. They are found in Eastern Europe and
climate change. Sulfur dioxide SO2) and carbon dioxide Central Asia., including Russia.
CO2) are both gases that are emitted from volcanoes. SO2 is Prairies: Prairies are found in United States of America.
a short-lived climate pollutant, meaning that it stays in the The Prairies are known as the ‘Granaries of the World’
atmosphere for only a few days. CO2 is a long-lived climate
because of the huge production of wheat.
pollutant, meaning that it can stay in the atmosphere for
hundreds or even thousands of years. Velds: The veld grasslands of Africa occur in the most
southern portion of the continent spreading through the
50. Solution: (c) political boundaries of South Africa, Lesotho, Swaziland,
Exp) Option c is the correct answer. Mozambique, Zimbabwe and Botswana.
Metamorphic rocks are formed when existing igneous, Downs: It is a pastoral and agricultural region in Australia.
sedimentary, or metamorphic rocks are subjected to high This area experiences a mostly humid sub-tropical climate.
heat, pressure, or hot mineral-rich fluids. This process Agricultural practice is mostly done here.
is called metamorphism. Metamorphism can change the
mineral composition and texture of the original rock. For 53. Solution: (c)
example, limestone, which is a sedimentary rock, can be Exp) Option c is the correct answer.
metamorphosed into marble.
An inselberg is not related to a glacier. An inselberg is an
51. Solution: (b) isolated hill or mountain that rises abruptly from a gently
Exp) Option b is the correct answer. sloping or level surrounding plain. It is formed by the erosion
of softer rocks around a resistant core of hard rock.
Options a and c are incorrect: The Mid-Continental Belt
and the Mid-Atlantic Belt are both regions of tectonic Important Tips
activity, but they are not as volcanically active as the Circum- • A canyon is a deep, narrow valley with steep sides
Pacific Belt. that is carved by a river over time. A river is a natural
The Mid-Continental Belt is a region of divergence, where stream of water that flows in a channel from a source to
the North American Plate and the Eurasian Plate are moving a mouth. Some examples of canyons formed by rivers
away from each other. This type of plate movement typically are the Grand Canyon in Arizona, USA.
does not produce volcanic activity. However, there are • A zeugen is a type of landform that consists of a ridge
some volcanoes in the Mid-Continental Belt, such as the or table of hard rock with a flat top and steep sides
Yellowstone Caldera in the United States. that is eroded by wind. Wind is a natural movement
The Mid-Atlantic Belt is also a region of divergence, where of air across the surface of the earth. Some examples of
the North American Plate and the African Plate are moving zeugens formed by wind are the Mushroom Rocks in
away from each other. Again, this type of plate movement Egypt and the Yardangs in China.
typically does not produce volcanic activity. However, there • A moraine is an accumulation of unconsolidated
are some volcanoes in the Mid-Atlantic Belt, such as the debris carried by a glacier and deposited along its
Azores Islands and Iceland. margins or at its terminus. A glacier is a large mass of
ice that moves slowly over land.
Option b is correct: The Circum-Pacific Belt, also known
as the Ring of Fire, is a region of intense volcanic and 54. Solution: (d)
seismic activity that circles the Pacific Ocean. It is home to
Exp) Option d is the correct answer.
about 90% of the world’s active volcanoes and 75% of the
world’s earthquakes. It is a region of convergence, where The term “peneplain” is related to river processes and
tectonic plates are moving towards each other. This type erosion. In geomorphology and geology, a peneplain is a low
of plate movement often produces volcanic activity. Some relief plain that is formed by protracted erosion, particularly
of the examples of volcanoes in the Circum-Pacific Belt: by river processes. It represents a stage of fluvial erosion
Mount Fuji, Japan, Mount St. Helens, United States, Mount during periods of tectonic stability when a landscape
Pinatubo, Philippines Krakatoa, Indonesia, Mount Ruapehu, is gradually worn down, creating a near level or gently
New Zealand. sloping plain.

PYQ Workbook 52
GENERAL GEOGRAPHY

57. Solution: (a)


Exp) Option a is the correct answer.
Option a is correct: A volcanic cone is a hill-shaped
landform that forms around a volcano. It is a basic landform
because it is one of the four major types of landforms, along
with mountains, hills, plateaus, and plains.
Option b, c, and d are incorrect: The other options are
examples of minor landforms, which are landforms that are
created by the erosion of other landforms.
A residual mountain is a mountain that remains after the
55. Solution: (b)
surrounding land has been eroded away.
Exp) Option b is the correct answer.
A monadnock is a type of residual mountain that is isolated
The formation of delta is affected by many factors, such from other mountains.
as climatic conditions, geologic setting, sediment sources,
An erosional waterfall is a waterfall that is formed by the
tectonic stability, river slope, flooding characteristics,
erosion of the land by water.
depositional and erosional processes, tidal range and offshore
energy conditions1. Among the given options, tide-ebb is Important Tips
the most relevant factor that affects the shape and type of Three main types of volcanic cones:
delta. Tide-ebb is the period when the tide level falls, and the • Cinder cones: Cinder cones are the most common
water flows back to the sea. This can create a tide-dominated type of volcanic cone. They form from ash and magma
delta, which is characterized by long, linear islands and cinders, which are partly-burned, solid pieces of
channels that are parallel to the coast. An example of a magma that fall to the ground following a volcanic
tide-dominated delta is the Ganges-Brahmaputra delta in eruption. Cinder cones are typically steep-sided and
Bangladesh and India. have a central crater.

Tides also limit where deltas can form. The Amazon, the • Broad cones: Broad cones, also known as shield
volcanoes, are the largest type of volcanic cone. They
largest river in the world, is without a delta. The tides of the
form from runny lava that flows easily from the
Atlantic Ocean are too strong to allow silt to create a delta
volcano. Broad cones have gently sloping sides and a
on the Amazon.
broad base.
56. Solution: (c) • Composite cones: Composite cones, also known
Exp) Option c is the correct answer. as stratovolcanoes, are the most dangerous type of
volcanic cone. They form from alternating layers of
Option a is incorrect: Sandstone is not typically formed lava and ash. Composite cones are typically tall and
in the desert. Deserts are dry and arid environments, and have steep sides.
there is not enough water to transport sand grains and form Examples of volcanic cones:
sandstone.
• Mount Fuji, Japan (composite cone)
Option b is incorrect: Sandstone is not typically formed due
• Mauna Loa, Hawaii (broad cone)
to heat. Heat can cause rocks to expand and contract, but it
• Parícutin, Mexico (cinder cone)
is unlikely to cause sandstone to be flaky.
Option c is correct: Sandstone is a sedimentary rock that is 58. Solution: (d)
formed from sand grains that have been cemented together. Exp) Option d is the correct answer.
Sand grains are typically made of quartz, which is a hard
Option d is correct: Granite is an igneous rock, not a
and durable mineral. However, sandstone can also contain metamorphic rock. Igneous rocks are formed when molten
other minerals, such as feldspar, mica, and iron. Sandstone rock magma) cools and solidifies. Metamorphic rocks are
is formed when sand grains are deposited on the bottom formed when existing igneous, sedimentary, or metamorphic
of a body of water. Over time, the sand grains are buried rocks are subjected to high heat, pressure, or hot mineral-
by other sediments and compacted. The pressure from the rich fluids.
overlying sediments causes the sand grains to fuse together,
Option a, b, and c are incorrect: The other options are all
forming sandstone and this makes the sandstone flaky.
examples of metamorphic rocks:
Option d is incorrect: Sandstone is not typically formed on
• Marble is a metamorphic rock formed from limestone.
the mountain. Mountains are high-altitude environments,
and there is not enough water to transport sand grains to the • Quartzite is a metamorphic rock formed from sandstone.
top of mountains. • Slate is a metamorphic rock formed from shale.

53 PYQ Workbook
GENERAL GEOGRAPHY

59. Solution: (d) Important Tips


Exp) Option d is the correct answer. Additional details about bajadas:
Option a is incorrect: Lava is molten rock that has erupted • Bajadas are common in arid and semi-arid regions,
from a volcano. It is typically very hot, with temperatures such as the southwestern United States, Mexico, and
ranging from 700 to 1,200 degrees Celsius. Lava can be fluid Australia.
or viscous, depending on its composition and temperature. • Bajadas are formed by the deposition of sediment from
Fluid lava can flow long distances, while viscous lava is more streams and rivers that flow down from the mountains.
likely to form thick piles or domes around the volcano. • The sediment is deposited in a fan-shaped pattern,
with the apex of the fan at the base of the mountains
Option b is incorrect: Basalt is a dark-colored, fine-grained
and the base of the fan extending outward.
igneous rock formed from the rapid cooling of low-viscosity
• Bajadas can be several miles long and hundreds of feet
lava rich in magnesium and iron mafic lava) exposed at or
thick.
very near the surface of a rocky planet or moon. More than
90% of all volcanic rock on Earth is basalt.
Option c is incorrect: Obsidian is a volcanic glass that is
formed when lava cools very quickly. It is typically black or
dark gray in color, but it can also be green, brown, or red.
Obsidian is very hard and sharp. It is formed when lava flows
over water or other cold surfaces. The rapid cooling of the
lava prevents the formation of crystals, resulting in a glassy
texture. Obsidian can also be formed when lava erupts from
a volcano and is cooled by the air.
Option d is correct: The molten material found inside
the Earth is called magma. Magma is a mixture of molten
and semi-molten rock, minerals, and dissolved gases. When
magma rises to the surface of the Earth and erupts from a
volcano, it is called lava.

60. Solution: (d)


Exp) Option d is the correct answer
Statement 1 is correct: The VEI is a relative scale that
measures the explosiveness of volcanic eruptions based on
factors such as eruption cloud height, volume of ejected
material, and other qualitative observations. 62. Solution: (a)
Statement 2 is correct: Iceland is located on the Mid-
Exp) Option a is the correct answer
Atlantic Ridge, where the Eurasian and North American
tectonic plates are spreading apart, leading to volcanic Shield volcanoes are formed by the eruption of fluid lava,
activity. The Philippines is part of the Pacific Ring of Fire, an
which flows easily and builds up broad, gently sloping
area characterized by intense volcanic and seismic activity
due to the movement of several tectonic plates. mountains. Individual lava flows from a shield volcano

Statement 3 is correct: When volcanoes erupt, they release are typically only a few feet thick, but over time, repeated
minerals and nutrients from the Earth’s mantle, which enrich flows can build up a volcano that is several miles wide and
the surrounding soil. These soils are highly fertile and
thousands of feet high.
support intensive agriculture.
Important Tips
61. Solution: (c)
• Composite volcanoes are also known as
Exp) Option c is the correct answer
stratovolcanoes. They are built up by alternating
A bajada is a gently sloping, piedmont-like slope of alluvial layers of lava and pyroclastic material fragments of
debris that forms at the base of a mountain range in a rock, ash, and cinders). Composite volcanoes have a
desert or semi-arid region. The term “bajada” is Spanish for steep-sided cone shape and are often explosive. Mount
“descent” or “slope.” It is also known as Perpediment. Fuji in Japan is an example of a composite volcano.

PYQ Workbook 54
GENERAL GEOGRAPHY

• Cinder-cone volcanoes are the simplest type of point, they merge and contribute their water into the lake or
volcano. They are built up by pyroclastic material that depression.
is ejected from a single vent. Cinder-cone volcanoes Important Tips
have a bowl-shaped crater at the summit and rarely
Other drainage patterns:
rise more than a thousand feet or so above their
surroundings. Sunset Crater in Arizona is an example Trellis pattern - It is characterized by parallel main rivers
of a cinder-cone volcano. with shorter tributaries joining them at right angles.
• Lava domes are formed when lava is very viscous thick) Dendritic pattern - It is characterized by a branching
and does not flow easily. Lava domes are typically small network of rivers resembling the branches of a tree.
and steep-sided. Mount Etna in Italy is an example of Radial pattern - It is characterized by rivers flowing
a lava dome. outward in different directions from a central high point,
• Calderas are large, bowl-shaped depressions that often found in volcanic regions.
are formed when the top of a volcano collapses.
67. Solution: (c)
Calderas can be formed by explosive eruptions or by
the withdrawal of magma from below the volcano. Exp) Option c is the correct answer.
Yellowstone National Park is an example of a caldera. The type of igneous rock among the given options is granite.
Granite is a common type of intrusive igneous rock that is
63. Solution: (d) composed primarily of quartz, feldspar, and mica minerals.
Exp) Option d is the correct answer It is formed from the solidification of magma beneath the
Geyserites are sedimentary rocks that are formed in the Earth’s surface.
vicinity of geysers and hot springs. They are typically Important Tips
composed of silica silicon dioxide) deposits that precipitate
Some examples of Igneous rocks are- Granite, Basalt,
from the hot water or steam as it emerges from the ground.
Obsidian, Pumice, Rhyolite, Andesite, Diorite, Gabbro,
Geyserites are formed through chemical processes rather
Peridotite, Tuff.
than mechanical processes.
Some examples of Metamorphic Rocks are- Marble,
64. Solution: (b) Slate, Quartzite, Schist, Gneiss, Anthracite, Hornfels,
Exp) Option b is the correct answer Serpentinite, Phyllite, Amphibolite
A: Insequent (2) - No apparent reason why it follows the path Some examples of sedimentary rocks are- Limestone,
it takes Sandstone, Shale, Conglomerate, Chalk, Coal, Dolomite,
Siltstone, Gypsum, Breccia
B: Consequent (4) - Whose course is determined by the
original slope of the land 68. Solution: (b)
C: Subsequent (1) - Whose direction of flow is controlled by Exp) Option b is the correct answer.
the rock structure
Primary waves, also known as P-waves, are one of the two
D: Resequent (3) - Flow in the same direction as the main types of seismic waves generated by earthquakes.
consequent stream but at a lower level P-waves can propagate through the Earth’s interior, including
gaseous, liquid, and solid substances.
65. Solution: (b)
Exp) Option b is the correct answer Important Tips

Hydration is the process by which a substance combines Seismic Waves are of two types- Body waves and Surface
with water. In the case of minerals, hydration can cause Waves
the minerals to expand. This is because the water molecules Body waves are generated by the energy released at the
can get trapped in the crystal structure of the mineral, which earthquake’s focus and propagate in various directions as
can cause the crystal to grow. The expansion of the mineral they travel through the Earth’s interior. It is of two types -
can put stress on the rock, which can eventually lead to the P Waves and S-Waves.
rock breaking. P Waves are longitudinal in nature whereas S Waves are
transverse in nature.
66. Solution: (d)
Exp) Option d is the correct answer. 69. Solution: (d)
Exp) Option d is the correct answer.
When rivers discharge their waters from all directions into
a lake or depression, the pattern is known as a centripetal The climate, including factors such as temperature, rainfall,
pattern. In a centripetal drainage pattern, the rivers converge or humidity, does not directly influence the extent of damage
towards a central point or basin. This occurs when the land caused by an earthquake. Earthquakes are geological events
around a lake or depression is lower in elevation, causing and their impact primarily depends on the geophysical
the rivers to flow inward. As the rivers approach the central characteristics of the area and the built environment.

55 PYQ Workbook
GENERAL GEOGRAPHY

70. Solution: (d) characterized by soluble rocks such as limestone, sinkholes


Exp) Option d is the correct answer. are the most common topographic feature. Sinkholes are
formed when the underlying limestone is dissolved, leading
The geomorphic factors influencing plant and animal
to the collapse of the surface layer.
distributions are slope angle, slope aspect, and relative relief.
Slope angle determines steepness, affecting soil moisture, Statement II is correct: Topographically, a sinkhole is a
drainage, and sunlight exposure. Slope aspect determines depression that varies in depth from less than a meter to a few
the direction a slope faces, influencing sunlight, hundred meters. Sinkholes can have varying depths, ranging
temperature, and moisture availability. Relative relief from shallow depressions to deep cavities, depending on
refers to elevation differences, affecting water drainage, soil the extent of the dissolution and collapse of the underlying
formation, and microclimate. rock. However, statement II does not explain or provide a
correct explanation of statement I.
71. Solution: (d) Sinkholes are the most common topographic form in Karst
Exp) Option d is the correct answer. terrains because the process of dissolution and collapse
Mount Merapi is an active volcano located in Indonesia occurs extensively in these areas. The soluble nature of
option A - (4). the rocks, combined with the action of water, leads to the
Sakurajima is an active volcano located in Japan option B - formation of numerous sinkholes of various sizes and depths.
(3). 75. Solution: (d)
Mount Vesuvius is an active volcano located in Italy option Exp) Option d is the correct answer.
C - (2).
The Himalayas are not associated with volcanic activity,
Mauna Loa is an active volcano located in Hawaii option D and therefore extrusive volcanoes are not found in this
- (1). mountain range. The Himalayas, which are a mountain range
72. Solution: (a) in Asia, were primarily formed by the collision of the Indian
and Eurasian tectonic plates. The formation of the Himalayas
Exp) Option a is the correct answer.
is primarily due to the convergence of these plates, resulting
Tectonic plates are large, rigid pieces of Earth’s lithosphere in the uplift of the crust and the creation of fold mountains.
that interact and move, causing earthquakes, volcanic
activity, and the formation of mountains. While there Important Tips
are smaller plates and microplates in the region, the Saudi • Alaska: Alaska is a region known for its volcanic
Arabian plate is not considered one of the major tectonic activity. It is located in the “Ring of Fire,” which is
plates like the African Plate or Eurasian Plate. an area in the Pacific Ocean where many volcanic
eruptions and earthquakes occur. Alaska has numerous
Important Tips volcanoes, including some active ones such as Mount
The major tectonic plates: Redoubt and Mount Augustine.
• African Plate • Rocky Mountains: The Rocky Mountains stretch
• Antarctic Plate across western North America. While there are some
volcanic rocks in the Rocky Mountains, they are
• Eurasian Plate
primarily formed through uplift and erosion rather
• North American Plate than volcanic activity.
• South American Plate • Andes: The Andes Mountains in South America
• Indo-Australian Plate are associated with a significant amount of volcanic
• Pacific Plate activity. They are part of the “Ring of Fire” and are
characterized by a volcanic arc along the western coast
73. Solution: (a) of the continent.
Exp) Option a is the correct answer.
76. Solution: (b)
Cirque is not a fluvial landform. Fluvial landforms are
Exp) Option b is the correct answer.
created or influenced by rivers or streams through erosion,
transportation, and deposition. Examples include gorges, Both statements are individually true, but Statement II is not
braids, and canyons. Cirques, however, are associated with the correct explanation of Statement I.
glacial activity and are bowl-shaped hollows formed by Statement I is correct: Glacial troughs, which are U-shaped
glaciers in mountainous regions. They are not formed by valleys formed by the erosion of glaciers, can have flat floors
rivers or streams. in certain sections. This is due to the extensive glacial erosion
that smoothens and levels the valley bottom.
74. Solution: (b)
Statement II is correct: While glacial erosion does
Exp) Option b is the correct answer. contribute to the formation of flat floors in glacial troughs,
Statement I is correct: In Karst terrains, which are the main factor responsible for the flatness is the deposition

PYQ Workbook 56
GENERAL GEOGRAPHY

of sediments by the glacier as it retreats. As the glacier melts, Important Tips


it deposits the material it carries, creating a relatively flat
• Zeugen is a table-shaped area of rock found in arid
surface within the valley.
and semi-arid areas formed when more resistant rock
77. Solution: (a) is reduced at a slower rate than softer rocks around it
under the effects of wind erosion.
Exp) Option a is the correct answer.
• Dreikanter is a type of ventifact that typically forms
Statement I is correct: Incised meanders are characteristic of in the desert or periglacial environments due to the
mature rivers where the river has eroded vertically, creating abrasive action of blowing sand.
deep, narrow valleys with meandering patterns.
• Demoiselle is a rock pillar which stands as resistant
Statement II is correct: Incised meanders are characterized rocks above soft rocks as a result of differential erosion
by rejuvenation and upliftment of land. This statement is the of hard and soft rocks.
correct explanation of Statement I. Incised meanders often
occur when the land is uplifted or rejuvenated, causing the 81. Solution: (c)
river to downcut into its own bed, forming the deep, narrow Exp) Option c is the correct answer
valleys. Stalactites and stalagmites are features of karst topography.
Both the statements are individually true, and Statement II is Stalactites and stalagmites are formed when the water that
the correct explanation of Statement I drips from the ceiling of a cave leaves behind minerals that
solidify.
78. Solution: (d)
Important Tips
Exp) Option d is the correct answer.
• Karst is a topography formed from the dissolution
Streams on a dissected volcano cone generally follow a of soluble rocks such as limestone, dolomite, and
radial drainage pattern, flowing outward from the central gypsum. It is characterized by underground drainage
vent. This pattern is shaped by the underlying geology and systems with sinkholes and caves.
the natural flow of water down the slopes. • Karst is most strongly developed in dense carbonate
79. Solution: (b) rock, such as limestone, that is thinly bedded and
highly fractured.
Exp) Option b is the correct answer.
• Karst is not typically well developed in chalk, because
Statement 1 is correct- Earthquakes largely occur along
chalk is highly porous rather than dense, so the flow of
the converging plate boundaries where tectonic plates,
groundwater is not concentrated along with fractures.
which make up the Earth’s crust, collide, subduct one
plate goes beneath another), or slide past each other • Karst is also most strongly developed where the
water table is relatively low, such as in uplands with
along faults. These plate boundaries, such as subduction
entrenched valleys, and where rainfall is moderate
zones, transform boundaries, and collision zones, are
to heavy.
characterized by intense geological activity, leading to the
release of accumulated stress and resulting in seismic events 82. Solution: (d)
known as earthquakes. Exp) Option d is the correct answer.
Statement 2 is correct- The point of origin of an earthquake
within the lithosphere is called the focus or hypocenter. It
is the location within the Earth where the seismic energy is
released.
Statement 3 is correct- The intensity of an earthquake
does indeed decrease with distance from the epicenter.
The intensity measures the amount of energy released by
the earthquake and its effects on the Earth’s surface. As the
distance from the epicenter increases, the intensity decreases.
Statement 4 is incorrect- The epicenter of an earthquake is
the point on the Earth’s surface directly above the focus
or hypocenter. It can be located anywhere on the Earth’s
surface, including both land and oceans. 83. Solution: (a)
Exp) Option a is the correct answer
80. Solution: (a)
A. Chemical weathering - 2. Oxidation
Exp) Option a is the correct answer
B. Mechanical weathering - 3. Plant roots
Wind gap is not related to wind erosion because it is
C. Glacial deposits - 1. Till
formed by the capture of a river by another river, not by
wind. D. Deposition by groundwater - 4. Stalactite

57 PYQ Workbook
GENERAL GEOGRAPHY

84. Solution: (c) Oxbow Lake: An oxbow lake is a U-shaped Lake that forms
Exp) Option c is the correct answer. when a meandering river cuts off a meander bend, leaving
Seismic gaps refer to areas along plate boundaries where a curved lake behind. This occurs when a meander gets
tectonic stress has been building up over time but no cutoff from the main channel due to erosion and deposition
significant earthquake activity has occurred.
processes.
Important Tips Delta: A delta is a landform that forms at the mouth of a
• Seismic gaps are considered to be “locked” or “stuck,” river where it meets a body of water, such as a lake, sea, or
indicating a potential for future earthquakes.
ocean. Deltas are created when the river deposits sediment
• The accumulation of stress in seismic gaps can
eventually be released through a major earthquake and builds up a triangular or fan-shaped land area.
when the built-up strain exceeds the strength of the
rocks, causing a rupture along the fault. 87. Solution: (b)

• Therefore, seismic gaps are areas of concern for Exp) Option b is the correct answer.
potential future earthquakes. Statement 1 is incorrect: The point of origin of an
85. Solution: (d) earthquake is actually called the hypocenter or focus, which
Exp) Option d is the correct answer. is the location within the Earth where the seismic energy is
Plate tectonics is a scientific theory that describes the large- released. The epicenter, on the other hand, refers to the point
scale movements and interactions of Earth’s lithosphere. on the Earth’s surface directly above the hypocenter.
The lithosphere is broken up into tectonic plates, and these
Statement 2 is correct: Homoseismal lines are the lines
plates are composed of both oceanic lithosphere and thicker
continental lithosphere. drawn on the maps joining places experiencing an
earthquake at the same time. They are also called as co-
Option d is incorrect: Along divergent plate boundaries,
plates move away from each other, creating new crust as seismal lines.
magma rises to fill the gap. Subduction, on the other hand,
occurs along convergent plate boundaries where one plate 88. Solution: (a)
sinks beneath another and is recycled back into the mantle. Exp) Option a is the correct answer.
Subduction does not occur along divergent plate boundaries.
A trough is an elongated area of low pressure with a dip
Option a is correct: Tectonic plates are composed of both
in the isobars on a weather map. It causes rising air and the
oceanic lithosphere found beneath the oceSolution: (a)nd
continental lithosphere found beneath the continents). formation of clouds and precipitation. A ridge, on the other
Option b is correct: Tectonic plates are able to move because hand, is an elongated area of high pressure with a crest in
the Earth’s lithosphere, which is composed of the crust and the isobars, leading to sinking air and clear skies.
uppermost part of the mantle, has a higher strength than the
underlying asthenosphere. The asthenosphere is a partially
molten, weak layer beneath the lithosphere that allows for
the movement of the plates.
Option c is correct: The Earth’s lithosphere is indeed broken
up into tectonic plates, which are distinct segments that
interact with each other along their boundaries.

86. Solution: (b)


Exp) Option b is the correct answer.
The correct arrangement of the features formed by rivers in
its course, starting from upstream, is 89. Solution: (c)

Falls: Falls, also known as waterfalls, are formed when a Exp) Option c is the correct answer.
river flows over a steep vertical drop in its course. This Solifluction is the slow movement of soil and loose
occurs when a river encounters resistant rock layers or a
materials downslope in permafrost regions due to the
sudden change in the gradient of the land.
freezing and thawing of water. The permanence of frozen
Meanders: Meanders are large bends or curves formed by
ground in permafrost areas prevents water drainage, leading
a river in its middle or lower course. These are created
due to the erosional and depositional activities of the river. to soil saturation and eventual movement caused by the
Meanders typically occur in areas with gentle slopes. expansion and contraction of freezing water.

PYQ Workbook 58
GENERAL GEOGRAPHY

92. Solution: (d)


Exp) Option d is the correct answer.
Traction load is the heaviest type of load that is transported
by a river. It consists of large, heavy rocks that are rolled
90. Solution: (d)
along the riverbed. Traction load is only transported by
Exp) Option d is the correct answer. rivers with a high velocity.
A crater is a depression at the top of a volcano following a Important Tips
volcanic eruption. It is formed when the magma chamber • Solution load: This refers to dissolved minerals and
beneath the volcano collapses. Craters can be very large, salts that are carried within the water. These particles
some reaching up to several kilometers in diameter. They are are not visible and are transported in a dissolved state.
often filled with water, forming crater lakes. • Suspension load: This refers to small particles, such as
silt and clay, that are suspended in the water column.
These particles are transported by the flow of water
without directly contacting the riverbed. They can
cause turbidity and give the water a murky appearance.
• Saltation load: This refers to the transportation of
relatively larger particles, such as sand and pebbles,
that are lifted and moved intermittently by the force
of the flowing water. These particles bounce and hop
along the riverbed in a series of leaps rather than
rolling continuously.

93. Solution: (b)


Exp) Option b is the correct answer.
Convection currents are found in the mantle, which is the
layer of the Earth that lies between the crust and the core.
The mantle is made up of hot, molten rock that is constantly
moving. The convection currents are caused by the uneven
heating of the mantle. The hot rock rises to the top, cools,
and then sinks back down. This movement of the mantle is
what drives the tectonic plates.

91. Solution: (b) 94. Solution: (d)


Exp) Option d is the correct answer.
Exp) Option b is the correct answer.
The Cardamom Hills are a mountain range in the Western
The landscape features mentioned in the question, such as
Ghats in India. They are located in the states of Kerala
caves, arches, stacks, and stumps, are primarily associated and Tamil Nadu. The Cardamom Hills are regarded as a
with wave action. These formations are commonly found continuation of the Western Ghats because they share the
along coastlines and are a result of erosion caused by the same geological formations and are home to similar plant
continuous pounding of waves against rock formations. and animal species.

59 PYQ Workbook
GENERAL GEOGRAPHY

95. Solution: (a) 96. Solution: (a)


Exp) Option a is the correct answer. Exp) Option a is the correct answer.
A sand spit is a long, narrow, finger-like landform that is A polje is a large, flat-floored depression in karst terrain.
attached to the mainland at one end and extends into the It is the largest type of karst landform, and it can be up to
water. It is formed by the deposition of sand and shingle by
100 square kilometers in size. An uvala is a smaller type of
longshore currents. Sand spits are often found in bays and
karst depression than a polje. A swallow hole is a small hole
along coasts that are exposed to strong winds and waves.
in the ground that leads to an underground cave system. A
Important Tips sinkhole is a small depression in the ground that is caused by
• A sand bar is a submerged ridge of sand or gravel the collapse of limestone bedrock.
that runs parallel to the shore.
• A sand dune is a hill or ridge of sand that is formed
by the wind.
• A tombolo is a sandbar that connects an island to the
mainland.

97. Solution: (a)


Exp) Option a is the correct answer.

Landform Definition Example Agent Definition Example

A fine-grained wind-
The movement of air Dust storms, sand
deposited sediment, The Loess Plateau in
Loess Wind caused by differences in dunes, deflation
usually composed of silt China.
atmospheric pressure. hollows.
and clay particles.

A narrow strip of sand or Waves, tides,


The Tombolo of The body of salt water
gravel that connects an currents, coastal
Tombolo Sainte-Marie-de-la- Sea that covers most of
island to the mainland or erosion and
Mer in France. Earth’s surface.
another island. deposition.

Curved ridges of sand A natural stream of water


The Point Bar State Fluvial erosion and
or gravel that form on that flows in a channel
Point bars Recreation Area in River deposition, floods,
the inner bank of a toward a larger body of
Colorado. deltas, oxbow lakes.
meandering river. water.

Ridges or mounds of Glacial erosion


A large mass of ice
glacial debris that are The lateral moraines and deposition,
that moves slowly over
Moraines deposited along the of the Khumbu Glacier ice sheets, valley
land due to gravity and
margins or at the end of a Glacier in Nepal. glaciers, cirques,
pressure.
glacier. horns, fjords.

98. Solution: (b) rocks) carried by a fast-flowing river or stream. When a river
Exp) Option b is the correct answer. flows down a steep slope and then enters a flatter area, it
slows down and deposits its load of sediments. This causes
An alluvial fan is a geological feature formed by the
the sediments to spread out in a fan-like shape.
deposition of sediments such as sand, gravel, and smaller

PYQ Workbook 60
GENERAL GEOGRAPHY

Important Tips 101. Solution: (b)

• A sand bar is a landform that is made of sand, gravel, Exp) Option b is the correct answer.
or fine sediment that is built by waves offshore from Statement 1 is correct: It is also known as Circum-Pacific
the beach. belt of volcanoes. This name reflects the fact that the Ring of
• A bajada is an alluvial plain formed at the base of a Fire follows the outline of the Pacific Plate and its margins,
mountain by the combination of several alluvial fans. where most of the volcanic activity occurs. Volcanoes are
formed by the melting of subducted crust or by the upwelling
• A diara is a piece of land that was created in the middle
of magma from hot spots or mantle plumes.
of river Ganga due to the deposition of sand over the
decades. Diaras are temporary islands formed within Statement 2 is correct: It is an active seismic zone. This
the river channel due to deposition of silt and sand means that it is a region where earthquakes are frequent
and intense. Earthquakes are caused by the sudden release of
99. Solution: (b) energy from the movement or deformation of tectonic plates
Exp) Option b is the correct answer. along faults or fractures. The Ring of Fire contains about 90%
of Earth’s earthquakes, including some of the most powerful
A spring is a natural exit point at which groundwater emerges
and devastating ones, such as the 1960 Valdivia earthquake,
out of the aquifer and flows onto the surface. Springs are
the 2004 Indian Ocean earthquake and tsunami, and the
driven out onto the surface by various natural forces, such as
2011 Tohoku earthquake and tsunami.
gravity and hydrostatic pressure.
Statement 3 is incorrect: The Ring of Fire is mostly a zone of
Some of the areas where springs are common are:
convergent plate margin, where tectonic plates are moving
• Well jointed rocks: These are rocks that have many toward each other, creating subduction zones or continental
cracks or joints that provide pathways for groundwater collisions. Convergent plate margins are usually associated
to flow and reach the surface. Examples of well jointed with volcanic activity, earthquakes, and subduction zones.
rocks are basalt, granite, and limestone.
• Karst topography: This is a landscape formed by the
dissolution of soluble rocks such as limestone, dolomite,
and gypsum. Karst topography is characterized by
sinkholes, caves, underground rivers, and springs. The
water dissolves the rock and creates openings that allow
water to flow out.
• Tilted strata: These are layers of sedimentary rocks that
have been tilted or folded by tectonic forces. Tilted strata
can create aquifers that slope towards the surface and
discharge water as springs. The water can also emerge
along faults or fractures that cut across the strata.
Arid areas with underlying rocks are not common areas for
springs. Arid areas have low rainfall and high evaporation 102. Solution: (c)
rates, which limit the recharge of groundwater. Exp) Option c is the correct answer.
Statement 1 is correct- The circum-Pacific belt is an active
100. Solution: (b)
volcanic realm because of the subduction of oceanic plates
Exp) Option b is the correct answer. beneath continental plates. When an oceanic plate subducts
Gneiss is not an igneous rock, but a metamorphic rock beneath a continental plate, the oceanic plate melts and the
that is formed from high temperature and pressure on other magma rises to the surface, forming volcanoes.
rocks. It forms when pre-existing rocks, such as granite, Statement 2 is correct- The circum-Pacific belt is an active
undergo high temperatures and pressures, causing them to seismic realm because of the subduction of oceanic plates
recrystallize and change their mineral composition. beneath continental plates. The subduction of oceanic
plates results in earthquakes
Important Tips
Statement 3 is incorrect- The circum-Pacific belt is a
• Granite is a coarse-grained igneous rock with a
convergent plate boundary, where two plates are moving
composition similar to that of continental crust.
towards each other.
• Pumice is a light, frothy volcanic rock that forms
Statement 4 is correct- The circum-Pacific belt is prone to
from the rapid cooling of lava with high gas content.
tsunamis because of the earthquakes and the underwater
• Basalt is a dense, fine-grained volcanic rock that landslides that are caused by the subduction. Tsunamis
forms from the solidification of lava on the Earth’s are large waves that are generated by earthquakes or
surface. underwater landslides.

61 PYQ Workbook
GENERAL GEOGRAPHY

103. Solution: (d) A. Alluvial fans are associated with - 4. mountain foothills.
Exp) Option d is the correct answer. Alluvial fans are fan-shaped deposits of sediment that
Statement 1 is correct- The Earth’s crust is made up of are formed where a river enters a plain. The sediment is
different types of rocks, each with its own properties. These deposited by the river as it slows down and spreads out.
properties affect how the rocks are weathered and eroded, Alluvial fans are often found at the foot of mountains.
which in turn affects the formation of landforms. For B. ‘V’-shaped valleys are associated with - 1. mountainous
example, sandstone is more easily eroded than granite, so areas.
sandstone landforms are typically more rounded and less V-shaped valleys are valleys that have been formed by the
rugged than granite landforms. erosion of a river. The river cuts into the bedrock, creating a
Statement 2 is correct- There are many different processes narrow, steep-sided valley. V-shaped valleys are often found
that can shape the Earth’s surface, including weathering, in mountainous areas.
erosion, deposition, and tectonic movements. The rates and C. Deltas are associated with - 2. coasts.
intensity of these processes vary from place to place, which Deltas are triangular deposits of sediment that are formed
is why we see different landforms in different parts of the where a river meets the sea. The sediment is deposited by
world. For example, mountains are formed by tectonic the river as it slows down and spreads out. Deltas are often
movements, while deserts are formed by the deposition of found at the mouths of rivers
sand and dust.
D. Ox-bow lakes are associated with - 3. lower reaches of
Statement 3 is correct- Even if the type and structure of the the river.
crustal material and the land forming processes are the same
Ox-bow lakes are crescent-shaped lakes that are formed
in two different places, the rates at which these processes
when a meander of a river is cut off. The meander is cut
occur may be different. This is because factors such as
off when the river changes course, leaving a loop of water
climate, vegetation, and human activity can affect the rates
behind. Ox-bow lakes are often found in floodplains.
of weathering, erosion, deposition, and tectonic movements.
For example, mountains in a humid climate will erode 106. Solution: (b)
more quickly than mountains in a dry climate.
Exp) Option b is the correct answer.
104. Solution: (d) A. Cyclones are associated with - 4. India.
Exp) Option d is the correct answer. Cyclones are tropical cyclones that occur in the Indian
A. Shield Volcano is associated with - 3. Hawaii. Shield Ocean and the Bay of Bengal. They are typically less intense
Volcano are formed by the eruption of large amounts than hurricanes, but they can still cause significant damage.
of lava, which flows out and builds up a broad, gently B. Hurricanes are associated with - 1. USA.
sloping cone. Hurricanes are tropical cyclones that occur in the Atlantic
B. Composite Volcano is associated with location - 4. Ocean and the Gulf of Mexico. They are the most intense
Philippines. Composite Volcano are formed by the type of tropical cyclone, and they can cause catastrophic
eruption of both lava and ash. The ash builds up around damage.
the lava, creating a steep-sided cone. C. Typhoons are associated with - 2. East Asia.
C. Caldera is associated with location - 1. Indonesia. Typhoons are tropical cyclones that occur in the western
Caldera can be caused by a large eruption, or by the Pacific Ocean. They are similar to hurricanes, but they are
withdrawal of magma from beneath the volcano. typically more intense.
D. Flood Basalt Provinces are associated with - 2. India. D. Willy-willies are associated with - 3. Australia.
Flood Basalt Province lava flows can be very thick, and Willy-willies are tropical cyclones that occur in Australia.
they can cover an area of hundreds of thousands of square They are less intense than hurricanes or typhoons, but they
kilometers.
can still cause significant damage.
Important Tips
107. Solution: (d)
The Hawaiian Islands are home to many shield volcanoes,
Exp) Option d is the correct answer.
including Mauna Loa and Kilauea.
The Antarctic Plate is one of the seven major tectonic
Mount Fuji in Japan and Mount Pinatubo in the
plates that make up the Earth’s lithosphere. Major tectonic
Philippines are examples of composite volcanoes.
plates are the largest units of the Earth’s lithosphere. They
The caldera of Mount Pinatubo is an example of a caldera. are made up of the uppermost part of the mantle and the
The Deccan Traps in India is an example of a flood basalt crust. The minor plates are smaller than the major plates,
province. and they are often located near the boundaries of the major
plates. The minor plates are more active than the major
105. Solution: (a) plates, and they are more likely to experience earthquakes
Exp) Option a is the correct answer. and volcanic eruptions.

PYQ Workbook 62
GENERAL GEOGRAPHY

The Karst plain is the type of plain associated with


Important Tips
limestone topography. Limestone, being a soluble rock, is
Major Plates: gradually dissolved by water and carbonic acid, leading to
• North American Plate the formation of unique landforms such as sinkholes, caves,
• South American Plate and underground drainage systems.
• Pacific Plate Important Tips
• India-Australia-New Zealand Plate Bajada plain: Formed by the merging of alluvial fans.
• Africa with the eastern Atlantic floor plate Alluvial plain: Created by the deposition of sediments
• Eurasia and the adjacent oceanic plate carried by rivers.
• Antarctica and surrounding oceanic plates Pene plain: A nearly level plain resulting from the erosion
Minor Plates: and deposition of rocks.

• Cocos Plate
• Nazca Plate
• Arabian Plate
• Philippine Plate
• Caroline Plate
• Fuji Plate

108. Solution: (c)


Exp) Option c is the correct answer.

63 PYQ Workbook
GENERAL GEOGRAPHY

GENERAL GEOGRAPHY
OCEANOGRAPHY
*This unit consists of questions from Movement, Relief, Temperature and Salinity of the Ocean.

3.1. UPSC CSE Previous Years’ Questions Which of the statements given above is/are
correct?
1. Consider the following statements (a) 1 and 2 only
Statement-I: The temperature contrast (b) 3 only
between continents and oceans is greater (c) 1 and 3 only
during summer than in winter. (d) 1, 2 and 3
Statement-II: The specific heat of water is 4. Tides occur in the oceans and seas due to
more than that of land surface. which among the following?
Which one of the following is correct in [UPSC CSE Pre. 2015]
respect of the above statements? 1. Gravitational force of the Sun
[UPSC CSE Pre 2023] 2. Gravitational force of the Moon
(a) Both Statement-I and Statement- are 3. Centrifugal force of the Earth
correct and Statement- is the correct Select the correct answer using the code
explanation for Statement-I. given below.
(b) Both Statement-1 and Statement- are
correct and Statement-I is not the correct (a) 1 only
explanation for Statement-I. (b) 2 and 3 only
(c) Statement-l is correct but Statement-II is (c) 1 and 3 only
incorrect. (d) 1, 2 and 3
(d) Statement-I is incorrect but Statement-II 5. Which of the following have coral reefs?
is correct. [UPSC CSE Pre. 2014]
2. With reference to Ocean Mean Temperature 1. Andaman and Nicobar Islands
(OMT), which of the following statements 2. Gulf of Kachchh
is/are correct? [UPSC CSE Pre. 2020] 3. Gulf of Mannar
1. OMT is measured up to a depth of 26°C 4. Sunderbans
isotherm which is 129 meters in the south Select the correct answer using the code
-western Indian Ocean during January– given below.
March.
(a) 1, 2 and 3 only
2. OMT collected during January –March (b) 2 and 4 only
can be used in assessing whether the (c) 1 and 3 only
amount of rainfall in monsoon will be less (d) 1, 2, 3 and 4
or more than a certain long -term mean.
Select the correct using the code given below: 6. The most important fishing grounds of the
(a) 1 only world are found in the regions where
(b) 2 only [UPSC CSE Pre. 2013]
(c) Both 1 and 2 (a) warm and cold atmospheric currents
(d) Neither 1 nor 2 meet
(b) rivers drain out large amounts of fresh
3. Consider the following statements: water into the sea
[UPSC CSE Pre. 2018] (c) warm and cold oceanic currents meet
1. Most of the world’s coral reefs are in (d) continental shelf is undulating
tropical waters.
7. Consider the following factors:
2. More than one-third of the world’s coral
reefs are located in the territories of [UPSC CSE Pre. 2012]
Australia, Indonesia and Philippines. 1. Rotation of the Earth
3. Coral reefs host far a greater number 2. Air pressure and wind
of animal phyla than those hosted by 3. Density of ocean water
tropical rainforests. 4. Revolution of the Earth

PYQ Workbook 64
GENERAL GEOGRAPHY

Which of the above factors influence the 4. Ocean currents are affected by the
ocean currents? configuration of the ocean.
(a) 1 and 2 only Which of these statements are correct?
(b) 1, 2 and 3 only
(c) 1 and 4 only [UPSC CSE Pre 2002]
(d) 2, 3 and 4 only (a) 1 and 2
(b) 2, 3 and 4
8. Between India and East Asia, the navigation-
time and distance can be greatly reduced by (c) 1, 3 and 4
which of the following? (d) 1, 2, 3 and 4
[UPSC CSE Pre. 2011] 12. Which one of the following factors is
1. Deepening the Malacca straits between responsible for the change in the regular
Malaysia and Indonesia. direction of the ocean currents in the Indian
2. Opening a new canal across the Kra Ocean? [UPSC CSE Pre 1997]
isthmus between the Gulf of Siam and (a) Indian Ocean is half an Ocean
Andaman Sea.
(b) Indian Ocean has Monsoon drift
Which of the statements given above is/are (c) Indian Ocean is a land-locked ocean
correct?
(d) Indian Ocean has greater variation in
(a) 1 only
salinity
(b) 2 only
(c) Both 1 and 2 13. Which of the following pair of oceanic
(d) Neither 1 nor 2 currents is shown in the given map:
9. Consider the following statements: [UPSC CSE Pre 1999]
1. On the planet Earth, the fresh water
available for use amounts to about less
than 1% of the total water found.
2. Of the total freshwater found on the
planet Earth, 95% is bound up in polar
ice caps and glaciers.
Which of the statements given above is/are
correct? [UPSC CSE Pre 2010]
(a) 1 only
(b) 2 only
(c) Both 1 and 2
(d) Neither 1 nor 2
(a) Benguela and Falkland
10. Consider the following statements:
(b) Canary and Humboldt
[UPSC CSE Pre. 2007]
(c) Agulhas and Guinea
1. The annual range of temperature is
greater in the Pacific Ocean than that in (d) Benguela and Guinea
the Atlantic Ocean
2. The annual range of temperature is
3.2. Other Examination Previous Years’
greater in the Northern Hemisphere than Questions
that in the Southern Hemisphere. 14. Match list-I with list-II and select the
Which of the statements given above is/are correct answer using the codes given below
correct? the lists:
(a) 1 only
(b) 2 only List-I List-II
(c) Both 1 and 2 (Ocean) (Maximum Deepest
(d) Neither 1 nor 2 Point)
11. Consider the following statements: A. Pacific 1. Sundra Trench
1. Ocean currents are the slow-surface B. Arctic 2. Puerto Rico
movement of water in the ocean.
Trench
2. Ocean currents assist in maintaining the
Earth’s heat balance. C. Indian 3. Mariana Trench
3. Ocean currents are set in motion D. Atlantic 4. Molloy Deep
primarily by prevailing winds. Code: [U.P.P.C.S. (Pre) 2022]

65 PYQ Workbook
GENERAL GEOGRAPHY

A B C D 22. Which is the world’s deepest trench?


(a) 3 2 1 4 [U.P.P.C.S. (Pre) 1991]
(b) 3 4 1 2 (a) Northern
(c) 4 3 2 1 (b) Challenger
(d) 1 2 3 4 (c) Manhattan
(d) Richards
15. Which one of the following denotes water
salinity gradient? 23. The country benefiting the most out of the
[U.P.P.C.S. (Pre) (Re-Exam) 2015] North Atlantic Drift is:
(a) Thermocline [R.A.S./R.T.S. (Pre) 1999]
(b) Halocline (a) Poland
(c) Pycnocline (b) Portugal
(d) Chemocline (c) Norway
(d) Nigeria
16. Which of the following ocean currents does
not belong to the Indian Ocean? 24. The ocean current different from the
[U.P.P.C.S. (Mains) 2014] remaining three is:
(a) Agulhas Current [R.A.S./R.T.S.(Pre) 1999]
(b) Mozambique Current (a) Benguela
(c) South Indian Ocean Current (b) Brazilian
(d) Benguela Current (c) South Equatorial
(d) Peruvian
17. Which one of the following is a cold current
of the South Atlantic Ocean? 25. Which of the following oceanic currents
[U.P.P.C.S. (Pre) 2005] does not flow in the North Atlantic Ocean?
(a) Canary current [Uttarakhand PCS (Pre) 2021]
(b) Benguela current (a) Gulf Stream
(c) Agulhas current (b) Banguela
(d) Brazil current (c) Norwegian
(d) Florida
18. Which one of the following currents does
not contribute to forming a complete circle 26. Which one of the following Oceanic
of currents in the South Atlantic Ocean? currents is not associated with the Pacific
[U.P.P.C.S. (Pre) 2000] Ocean? [Uttarakhand P.C.S. (Pre) 2016]
(a) Benguela (a) Canaries
(b) Brazil (b) Kuroshio
(c) Canary (c) California
(d) West wind drift (d) Humboldt
19. Which one of the following salts contributes 27. Which one is the most extensive part of the
maximum to the salinity of sea water? oceanic floor?
[U.P.P.C.S. (Pre) 2000] [Uttarakhand P.C.S. (Pre) 2012]
(a) Calcium sulphate (a) Ocean Deeps
(b) Magnesium chloride (b) Oceanic Slope
(c) Magnesium sulphate (c) Oceanic Shelves
(d) Sodium chloride (d) Deep Sea Plains
20. The highest salinity is found in: 28. Match list-I with list-II and select the
[U.P.P.C.S. (Pre) 1995] correct answer using the codes given below
the lists:
(a) Dead sea
(b) Red sea List-I List-II
(c) Great Salt Lake in the U.S.A.
(d) Lake Van in Turkey A. Gulf Stream 1. Pacific Ocean
B. West Wind Drift 2. A slow eastward
21. When the density in the sea increases, then? movement of
[U.P.P.C.S. (Pre) 1990] water over the
(a) Salinity and depth decreases. zone of westerly
(b) Salinity increases but depth decreases. wind
(c) Both salinity and depth increases.
(d) Salinity decreases and depth increases. C. Peru Current 3. Indian Ocean

PYQ Workbook 66
GENERAL GEOGRAPHY

D. West Australian 4. Warm current 34. Isohalines are lines joining equal:
Current [CDS-2022(I)]
Code: [Jharkhand PCS (Pre) 2013] (a) temperature.
(b) pressure.
A B C D (c) rainfall.
(a) 4 2 1 3 (d) salinity.
(b) 1 3 4 2
(c) 4 3 1 2 35. Which one of the following do not influence
(d) 1 2 4 3 the ocean currents? [CDS-2022(II)]
(a) Heating by solar energy
29. Main Source of Salinity of the Sea is– (b) Wind
[53rd to 55th B.P.S.C. (Pre) 2011] (c) Gravitational pull by Sun and Moon
(a) Rivers (d) Coriolis force
(b) Land
(c) Wind 36. Which one of the following is a cold ocean
(d) Ash ejected from the Volcano current? [CDS 2021 (I)]
(a) Brazilian Current
30. The distance between two successive crests (b) Gulf Stream
or troughs of sea waves is called as (c) North Equatorial Current
[CAPF 2020] (d) California Current
(a) Wavelength
(b) Wave height 37. Which of the following are warm ocean
(c) Wave frequency currents? [CDS 2020 (I)]
(d) Fetch (a) Kuroshio and California Current
(b) North Atlantic Drift and Brazil Current
31. Consider the following statements relating (c) Canaries and Benguela Current
to Sea Salinity: (d) West Wind Drift and Falkland Current
1. The ocean salinity depends on evaporation
and precipitation. 38. Which one of the following is considered as
2. Any change in the temperature or density the deepest point of the oceans?
influences the salinity. [CDS 2020 (II)]
3. Major source of sea salinity is terrestrial (a) Tonga Trench
discharge by rivers. (b) Mariana Trench
Which of the statements given above are (c) Philippine Trench
correct? [CAPF 2016] (d) Kermadec Trench
(a) 1 and 2 only 39. Which one of the following is not correct
(b) 2 and 3 only about Sargasso Sea? [CDS 2019 (I)]
(c) 1 and 3 only (a) It is characterised with anti-cyclonic
(d) 1, 2 and 3 circulation of ocean currents.
32. Which of the following statements with (b) It records the highest salinity in Atlantic
regard to the continental shelf is NOT ocean.
correct? [CAPF 2015] (c) It is located West of Gulf stream and East
(a) Shelves are absent close to plate of Canary current.
boundaries (d) It confined in gyre of calm and motionless
(b) They are highly sedimented water.
(c) They have abrupt falls towards the 40. The Gulf Stream is a poleward flowing
continental slopes current in the Atlantic Ocean. Which one
(d) They are rich fishing areas of the following statements with regard to
33. It is reported that there is an ongoing this is not correct? [CDS 2017 (I)]
decrease in the pH value of ocean water (a) It is similar to the Kuroshio current in the
because of global warming. It happens due North Pacific Ocean
to [CAPF 2015] (b) It transports warm, tropical water towards
(a) larger uptake of CO2 by ocean water polar region
(b) lesser uptake of CO2 by ocean water (c) This current is a major factor in weather
(c) larger uptake of atmospheric nitrogen by along the East coast of the USA
ocean water (d) The warm water of the Gulf Stream
(d) lesser uptake of atmospheric nitrogen by sustains the coral reefs of West Pacific
ocean water. Coast.

67 PYQ Workbook
GENERAL GEOGRAPHY

41. Statement I: The Kuroshio is a warm North- (d) Statement I is false, but Statement II is
flowing ocean current on the West side of the true.
North Pacific ocean.
Statement II Presence of a number of 42. In the absence of Cold Labrador Current,
volcanoes at the bottom of the sea of Japan is which one among the following would
responsible for the Kuroshio becoming warm. happen? [CDS 2015 (I)]
Codes: [CDS 2015 (II)] (a) There will be no North-East Atlantic
(a) Both the statements are individually true fishing grounds
and Statement II is the correct explanation (b) There will be no North-West Atlantic
of Statement I. fishing grounds
(b) Both the statements are individually
true, but Statement II is not the correct (c) There will be no fishing ground in the
explanation of Statement I. North Atlantic ocean
(c) Statement I is true, but Statement II is (d) Semi-arid condition of the Atlantic coast
false. of the USA and Canada would prevail

PYQ Workbook 68
GENERAL GEOGRAPHY

SOLUTIONS

3.1. UPSC CSE Previous Years’ Questions Important Tips


Coral reefs are large underwater structures composed
1. Solution: (d)
of the skeletons of colonial marine invertebrates called
Exp) Option d is the correct answer coral. Coral polyps, the animals primarily responsible for
Statement 1 is incorrect: This statement is incorrect as the building reefs, can take many forms: large reef building
temperature contrast between the continents and oceans is colonies, graceful flowing fans, and even small, solitary
more during the winter rather than summer. organisms.
Statement 2 is correct: This statement is true. Specific heat is
4. Solution: (d)
the amount of heat energy required to raise the temperature
of a substance by a certain amount. Water has a relatively Exp) Option d is the correct answer.
high specific heat compared to land surfaces. This means Options 1 and 2 are correct. Tides are the rise and fall of
that it takes a larger amount of heat energy to raise the sea levels caused by the combined effects of the gravitational
temperature of water compared to an equal mass of land forces exerted by the moon and the sun, and the rotation of
surface material by the same amount. The high specific heat
the earth.
of water is due to its molecular structure and the presence of
hydrogen bonding, which allows water to absorb and store a Option 3 is correct. Tides occur due to an imbalance between
significant amount of heat energy. the various forces acting on the ocean water at a point in
time. In general, the tide-generating force is the difference
2. Solution: (b)
between these two forces; i.e. the gravitational attraction
Exp) Option b is the correct answer due to the mass of the moon and the centrifugal force due
Ocean Mean Temperature (OMT) is an important climatic to rotation of the earth. Together, the gravitational pull
parameter required for atmospheric and oceanic studies like and the centrifugal force are responsible for creating the
cyclone and monsoon predictions and ocean heat transport two major tidal bulges on the earth. On the side of the earth
estimations.
facing the moon, a tidal bulge occurs while on the opposite
Statement 1 is incorrect. OMT is measured up to a depth of side though the gravitational attraction of the moon is less
26°C isotherm, is seen at depths varying from 50-100 metres. as it is farther away, the centrifugal force causes tidal bulge
During January-March, the mean 26°C isotherm depth in the
on the other side.
South-western Indian ocean is 59 metres.
Statement 2 correct. OMT collected during January — March 5. Solution: (a)
can be used in assessing whether the amount of rainfall in Exp) Option a is the correct answer.
monsoon will be less or more than a certain long-term mean.
Options 1, 2 and 3 are correct. Coral reefs ideally require
Using OMT data collected during January-March 2018, it
was able to predict with greater probability of the pattern of sunlight, clean water to sunlight through, Salt water, and
monsoon. abundant Planktons. In India, the major reef formations
are found in the Gulf of Mannar, Palk bay, Gulf of Kutch,
3. Solution: (d) Andaman and Nicobar Islands and the Lakshadweep
Exp) Option d is the correct answer islands.
Statement 1 is correct: Coral reefs are predominantly Option 4 is incorrect. Due to the presence of high volume of
present in tropical shallow waters less than 50 meters deep.
freshwater, coral reefs are not formed in Sunderbans.
Coral reefs can be found in tropical destinations around the
world, mostly in areas around the equator where the water
is warmer. More than 100 countries have a coral reef within
their borders, and over half of the world’s coral reefs are
found within six countries: Australia, Indonesia, Philippines,
Papua New Guinea, Fiji, and the Maldives.
Statement 2 is correct: Global distribution of corals-
Australia-17% Indonesia-16% Philippines-9% So, Australia,
Indonesia and Philippines together host more than one-third
of world’s corals.
Statement 3 is correct: It is estimated that more than 25,000
described species from thirty-two of the world’s thirty-three
animal phyla live in coral reef habitats. It is around four times
the number of animal phyla found in tropical rain forests. Fig: Distribution of coral reefs along Indian Coast

69 PYQ Workbook
GENERAL GEOGRAPHY

6. Solution: (c)
Exp) Option c is the correct answer.
The meeting of the cold and warm water, produces favourable
conditions for the growth of plankton, on which fish depend
directly or indirectly for their food supply. Thus, the areas
where warm and cold currents meet are the best fishing
grounds in the world.
For instance, Grand Banks in North America is an
international fishing ground where the cold Labrador
Current and the relatively warm Gulf Stream.

7. Solution: (b)
Exp) Option b is the correct answer.
Option 1 is correct. Rotation of Earth leads to Coriolis
10. Solution: (b)
effect which influences direction of ocean currents. The
Coriolis effect causes air and water to move towards the Exp) Option b is the correct answer.
right in the northern hemisphere and the left in the southern Statement 1 is incorrect. The size of the oceans and the seas
hemisphere. affects annual range of temperature e.g., bigger the size,
Option 2 is correct. Wind is the primary force driving lower the annual range and vice versa. The Atlantic Ocean
surface currents in the ocean. Warm air masses form where records relatively higher annual range of temperature than
the sun’s radiation is most intense, which is at the equator, the Pacific Ocean due to this reason.
this becomes an area of low pressure. Cold air masses form Statement 2 is correct. The temperature of ocean water
at the poles, where the sun’s radiation is less intense, this
varies in the northern and the southern hemispheres because
becomes an area of high pressure. Rising warm air travels
of dominance of land in the former and water in the latter.
from low pressure to high pressure, resulting in wind. The
The oceans in the northern hemisphere receive more heat
dominant wind patterns drive oceanic currents.
due to their contact with larger extent of land than their
Option 3 is correct. The density of ocean water influences
counterparts in the southern hemisphere and thus the
the speed of the ocean currents. Also, denser water tends to
temperature of surface water is comparatively higher in the
sink, while relatively lighter water tends to rise.
former than the latter.
Option 4 is incorrect. Revolution of Earth does not influence
the movement of Ocean Currents. Important Tips
Insolation, Planetary winds, Gravity, Salinity of ocean water, Other factors affecting the distribution of temperature
Shape of coastline are some other important factors which of ocean water include:
influence the movement of ocean currents. • Latitudes: The temperature of surface water decreases
from equator towards the poles because the sun’s rays
8. Solution: (b) become more and more slanting and thus the amount
Exp) Option b is the correct answer. of insolation decreases poleward accordingly.
Statement 1 is incorrect. The current depth of Strait of • Prevailing wind: Wind direction largely affects the
Malacca is 25 metres, which prohibits entry of some of distribution of temperature of ocean water. The winds
the biggest ships in the world. However, deepening it will blowing from the land towards the oceans and seas
not reduce navigation-time and distance to be travelled. (e.g., offshore winds) drive warm surface water away
Statement 2 is correct. The Kra Isthmus is the narrowest from the coast resulting into upwelling of cold bottom
part of the Malay Peninsula. Kra canal is a proposed canal water from below.
which would reduce the travel time by eliminating need • Ocean currents: Surface temperatures of the oceans are
of going around the Malay peninsula, and reducing a controlled by warm and cold currents. Warm currents
distance of nearly 1,200 kilometres. raise the temperature of the affected areas whereas cool
currents lower down the temperature. For example, the
9. Solution: (a) Gulf Stream raises the temperature near the eastern
Exp) Option a is the correct answer. coasts of N. America and the western coasts of Europe.
Statement 1 is correct: Only about 3% of the Earth’s water is
11. Solution: (d)
fresh, and of that, only about 0.5% is readily available. The
rest is locked up in glaciers, ice caps, and groundwater. Exp) Option d is the correct answer.
Statement 2 is incorrect: Of the total freshwater on Earth, Statement 1 is correct: Ocean currents are indeed slow-
about 68% is in glaciers and ice caps, and about 30% is in surface movements of water in the ocean, often covering vast
groundwater. Only a small percentage of freshwater is found distances and playing a crucial role in the Earth’s climate
in lakes, rivers, and streams. system.

PYQ Workbook 70
GENERAL GEOGRAPHY

Statement 2 is correct: Ocean currents assist in maintaining in the Indian Ocean is primarily due to the presence of
the Earth’s heat balance by redistributing heat from the the Monsoon drift. Monsoons are seasonal wind patterns
equator toward the poles and vice versa. For example, the that bring about a reversal in wind direction in the
Gulf Stream is a warm current that flows from the Gulf of Indian Ocean region. During the summer monsoon, winds
Mexico to the North Atlantic Ocean. It helps to keep Western blow from the southwest to the northeast, causing surface
Europe much warmer than it would be otherwise. currents to flow in the same direction. Conversely, during
the winter monsoon, winds reverse direction, blowing from
Statement 3 is correct: Prevailing winds are the winds that
the northeast to the southwest, and this change in wind
blow in a consistent direction over a particular region. The
patterns leads to a reversal of ocean currents in the Indian
wind exerts frictional drag on the ocean’s surface, causing
Ocean. This seasonal variability in winds and currents is a
the movement of surface waters in the direction of the wind.
distinctive feature of the Indian Ocean.
For example, the Trade Winds are steady winds that blow
from east to west in the tropics. They create a strong current 13. Solution: (d)
in the Atlantic Ocean called the North Equatorial Current. Exp) Option d is the correct answer.
Statement 4 is correct: Ocean currents are affected by the Benguela and Guinea oceanic currents have been shown in
configuration of the ocean, including factors like the shape the given map.
of coastlines, the presence of underwater features, and the The Benguela Current is a large, northward-flowing cold
depth contours of the ocean floor. These factors can influence ocean current that makes up the eastern part of the South
the direction, strength, and behavior of ocean currents. Atlantic Ocean gyre. The current stretch about from Cape
Important Tips Point in the south to the Angola-Benguela border in the
north. The prevailing south easterly trade winds propel the
Factors Influencing Ocean Currents:
stream.
Primary Forces:
The Guinea Current is a warm oceanic current in the
• Heating by Solar Energy: Solar heating causes water
Atlantic Ocean, extending eastward from the Gulf of
to expand, creating a slight gradient where water flows
Guinea along the western coast of Africa. It remains north
downhill. Near the equator, ocean water is about 8 of the equator, shifting between approximately 7° N in winter
cm higher in level than in mid-latitudes due to this and 15° N in summer. This warm, saline current reaches
expansion. depths of around 660 feet (200 meters).
• Wind: Surface winds exert frictional drag on the
ocean’s surface, propelling the water beneath them. 3.2. Other Examination Previous Years’
Wind direction and strength influence the movement Questions
of ocean currents.
14. Solution: (b)
• Gravity: Gravity’s pull on water results in variations
in gradient, contributing to the movement of ocean Exp) Option b is the correct answer.
currents. The correct matching of List-I (Ocean) with List-II
• Coriolis Force: The Coriolis effect causes moving water (Maximum Deepest Point) is as follows:
to deflect to the right in the Northern Hemisphere and A. Pacific - 3. Mariana Trench
to the left in the Southern Hemisphere, influencing B. Arctic - 4. Molloy Deep
the direction of ocean currents. Gyres, large circular C. Indian - 1. Sunda Trench
currents, are formed as a result. D. Atlantic - 2. Puerto Rico Trench
Secondary Forces: Important Tips
• Differences in Water Density: Variations in
• The Mariana Trench is an oceanic trench located
water density, primarily influenced by salinity and in the western Pacific Ocean, about 200 km east of
temperature differences, affect vertical ocean current the Mariana Islands; it is the deepest oceanic trench
movement. Dense, cold, and saline water tends to sink, on Earth. It is crescent-shaped and measures about
while lighter, warmer, and less saline water rises. 2,550 km in length and 69 km in width. The maximum
• Temperature of Water: Cold-water currents form known depth is 10,984 m at the southern end of a small
when polar cold water sinks and gradually moves slot-shaped valley in its floor known as the Challenger
toward the equator, while warm-water currents flow Deep.
from the equator towards the poles, replacing sinking • The Puerto Rico Trench is located on the boundary
cold water. These temperature-driven movements help between the Caribbean Sea and the Atlantic Ocean.
shape ocean currents. The trench is 800 km long and has a maximum depth of
8,376 m. This constitutes the single deepest point in
12. Solution: (b)
the Atlantic Ocean. This point is commonly referred
Exp) Option b is the correct answer. to as the Milwaukee Deep, with the Brownson Deep
The change in the regular direction of ocean currents naming the seabed surrounding it.

71 PYQ Workbook
GENERAL GEOGRAPHY

• The Molloy Deep (also known as the Molloy Hole) Important Tips
is a bathymetric feature in the Fram Strait, within the • A pycnocline is a layer where density gradients are
Greenland Sea east of Greenland and about 160 km greatest in water bodies. Pycnoclines, driven by density
west of Svalbard. It is the location of the deepest point gradients, impact ocean flows and vertical profiles,
in the Arctic Ocean. The outer rim of the trench is at
influencing the transport of heat, salt, nutrients, and
a depth of 2,700 m and contains about 600 km2 inside
upwelling. Vertical mixing across pycnoclines occurs
the rim, descending to approximately 5,550 m at its
through turbulence, playing a crucial role in nutrient
greatest depth. The basin floor measures about 220
transport in the ocean.
km2 and is the deepest point in the Arctic Ocean.
• A thermocline (also known as the thermal layer or
15. Solution: (b) the metalimnion in lakes) is a distinct layer based on
Exp) Option b is the correct answer. temperature within a large body of fluid (e.g. water,
as in an ocean or lake; or air, e.g. an atmosphere) with
“Halocline” denotes the water salinity gradient. The
a high gradient of distinct temperature differences
halocline is a zone in the ocean where there is a rapid change
associated with depth. In the ocean, the thermocline
in salinity with increasing depth. In this region, salinity divides the upper mixed layer from the calm deep
increases significantly at higher latitides and decreases water below.
significantly at lower latitudes, creating a distinct layer • A chemocline is a distinct layer within a body of
of water with different salinity characteristics. This is water characterized by a significant vertical gradient
typically found at depths ranging from 300 meters to 1000 in chemical properties. It separates upper and lower
meters and is particularly prominent in high and low layers with variations in the concentration of dissolved
latitudes due to variations in water density and temperature. gases and solids.

16. Solution: (d)


Exp) Option d is the correct answer.
The Benguela Current does not belong to the Indian
Ocean. Benguela Current is a branch of the West Wind
Drift of the Southern Hemisphere. It is cold current that
flows northward in the South Atlantic Ocean along
the west coast of southern Africa nearly to the Equator
before merging with the westward-flowing Atlantic South
Equatorial Current. The Benguela Current is known for its
cool temperatures, relatively low salinity, and high plankton
concentration, creating favorable conditions for fishing.
Agulhas Current, Mozambique Current and South Indian
Ocean Current are ocean currents of Indian Ocean.

PYQ Workbook 72
GENERAL GEOGRAPHY

17. Solution: (b) Important Ocean Currents:


Exp) Option b is the correct answer. • The Canary Current is a wind-driven surface cold
The cold current of the South Atlantic Ocean is the current that is part of the North Atlantic Gyre. This
Benguela Current. This current flows northward along eastern boundary current branches south from the
the eastern portion of the South Atlantic Ocean gyre, North Atlantic Current and flows southwest about
extending from Cape Point in the south to the Angola- as far as Senegal where it turns west and later joins
Benguela front in the north (around 16°S latitude). The the Atlantic North Equatorial Current. The current is
named after the Canary Islands.
Benguela Current is driven by prevailing south easterly
trade winds and is associated with coastal upwelling, which • The Agulhas Current is the western boundary warm
brings cold, nutrient-rich waters to the surface, supporting current of the southwest Indian Ocean. It flows south
along the east coast of Africa from 27°S to 40°S. It
a productive marine ecosystem along the southwestern
is narrow, swift and strong. It is the largest western
African coast.
boundary current in the world ocean.
18. Solution: (c) • The Brazil Current is a warm water current that flows
Exp) Option c is the correct answer. south along the Brazilian south coast to the mouth of
the Río de la Plata. This current is caused by diversion
The Canary Current is a cold current that flows along the
of a portion of the Atlantic South Equatorial Current
western coast of Africa, from the Canary Islands to Cape from where that current meets the South American
Verde. It is part of the North Atlantic Ocean gyre, not the continent.
South Atlantic Ocean gyre. Benguela current, Brazil current
and west wind draft are part of South Atlantic Ocean gyre. 19. Solution: (d)
Exp) Option d is the correct answer.
“Sodium chloride” contributes the maximum to the
salinity of seawater. Sodium chloride, commonly known
as table salt, accounts for approximately 77.7% of the total
salts in seawater.
• Share of different salts is as shown below—
• sodium chloride — 77.7%
• magnesium chloride—10.9%
• magnesium sulphate —.4.7%
Important Tips • calcium sulphate — 3.6%
West Wind Drift: • potassium sulphate — 2.5%
• West Wind Drift, also known as the Antarctic
Circumpolar Current, is a major cold ocean current
in the southern hemisphere.
• It flows from west to east, generally between 40 and
60 degrees south latitude, encircling Antarctica.
• It spans across the Atlantic, Pacific, and Indian Oceans,
connecting these major ocean basins.
• The lack of landmasses in its route allows it to form a
Circumpolar loop around Antarctica.
• It plays a crucial role in preserving Antarctica’s
massive ice sheet by keeping warm ocean waters Important Tips
away from the continent. Factors Affecting Ocean Salinity:
• The West Wind Drift influences the formation of the • Evaporation and Precipitation: Surface salinity
Ross and Weddell gyres in the Southern Ocean. is primarily influenced by the balance between
• Strong westerly winds and temperature differences evaporation and precipitation in a given region.
between the Equator and poles drive the formation and Increased evaporation leads to higher salinity, while
intensity of this current. heavy precipitation dilutes seawater and lowers salinity.
• The westerlies and trade winds, particularly the • Freshwater Inflow: Coastal regions are influenced by
roaring forties and furious fifties, contribute to the freshwater input from rivers, which can significantly
current’s flow, reaching speeds of 15 to 25 knots. reduce salinity near river mouths.

73 PYQ Workbook
GENERAL GEOGRAPHY

• Polar Processes: In polar regions, freezing and thawing (36,037 feet), making it the deepest point on Earth’s ocean
of ice affect salinity as ice formation excludes salt, floor, situated more than 2 kilometers (1.2 miles) below sea
increasing salinity, while ice melting dilutes seawater. level, even deeper than the peak of Mount Everest is above
sea level. The pressure at the bottom is immense, at 1,086
• Wind Transport: Wind can transport water from one
bar (15,750 psi), over a thousand times the atmospheric
area to another, affecting the salinity of both regions.
pressure at sea level.
• Ocean Currents: Ocean currents redistribute water
with varying salinities, contributing to regional 23. Solution: (c)
salinity variations. Exp) Option c is the correct answer.
• Temperature and Density: Salinity, temperature, and Norway benefits the most from the North Atlantic Drift.
density are interconnected; changes in temperature or The North Atlantic Drift, an extension of the Gulf Stream,
density influence local salinity levels. brings warm waters from the Gulf of Mexico into the North
Atlantic Ocean. This warm ocean current has a significant
20. Solution: (d) moderating effect on Norway’s climate, keeping its coastal
Exp) Option d is the correct answer. regions warmer in winter and cooler in summer than their
“Lake Van in Turkey” has the highest salinity. Lake Van, high northern latitude would typically suggest. Without
situated in eastern Turkey, is renowned for its exceptional this warm current, Norway’s climate would be much colder,
salinity level, measuring approximately 330 parts per making it a crucial factor in the country’s relatively mild
thousand (ppt). The lake’s high salinity is attributed to its climate.
endorheic nature, where it has no outlet, and its geological
history, which includes the blocking of its original outlet due
to a volcanic eruption.

Sea Salinity (in ppt)

Baltic Sea 7

Red sea 39

Caspian Sea 180

Dead Sea 250

Lake Van 330

21. Solution: (b)


Exp) Option b is the correct answer. 24. Solution: (d)
When the density in the sea increases, then Salinity Exp) Option d is the correct answer.
increases but depth decreases. When sea density increases,
it’s primarily due to elevated salinity. This higher salinity,
caused by dissolved salts, adds mass to the water, making it
denser. Denser water, being heavier, sinks beneath less dense
water. This occurs because more particles, like dissolved
salts, are packed into the same volume of water. This
increased particle concentration makes the water more
compact, occupying less space for the same mass. Think
of it as denser water squeezing into the space occupied by
less dense water, effectively displacing it. Consequently,
the surface layer becomes shallower in the region where
densification happens, leading to a reduction in the sea
column’s depth.

22. Solution: (b)


Exp) Option b is the correct answer.
The world’s deepest trench is the Challenger Deep in the
Mariana Trench, located in the western Pacific Ocean.
Mariana Trench stretches approximately 2,550 km in length
and 69 km in width. The Challenger Deep within this
trench reaches a maximum known depth of 10,984 meters

PYQ Workbook 74
GENERAL GEOGRAPHY

The Peruvian Current is a cold current that flows along • The Norwegian Current, a dominant Arctic inflow,
the western coast of South America, from Peru to Chile, originates near Shetland, north of Scotland, and
the eastern North Sea at depths of up to 100 meters.
in southeast Pacific Ocean. It is caused by the upwelling of It eventually flows into the Barents Sea, an Arctic
cold water from the deep ocean. The other three currents Ocean outcrop. Compared to its partial source, the
North Atlantic Current, the Norwegian Current is
are all warm currents. colder and less salty. Other sources include the North
and Baltic seas, Norwegian fjords, and rivers.
Important Tips
• The Florida Current is a thermal ocean current that
Peru Current: flows from the Straits of Florida around the Florida
• The Humboldt Current, also known as the Peru Peninsula and along the southeastern coast of the
Current, is a cold and low-salinity ocean current. United States before joining the Gulf Stream Current
• It flows northward along the western coast of South near Cape Hatteras. Its contributing currents are the
America in south east pacific ocean. The current is Loop Current and the Antilles Current.
an eastern boundary current and moves equatorially. 26. Solution: (a)
• It significantly cools the climates of Chile, Peru, and Exp) Option a is the correct answer.
Ecuador.
The Canary Current is a wind-driven surface cold current
• It contributes to the aridity of regions such as associated with the North Atlantic Ocean, specifically the
northern Chile’s Atacama Desert and coastal areas of eastern part. It flows southwestward from the North Atlantic
Peru and southern Ecuador. Current, extending as far as Senegal before turning west. The
• The Humboldt Current carries nutrient-rich waters Canary Current is named after the Canary Islands.
from the depths, supporting a productive marine In contrast, the other currents mentioned are associated
ecosystem. Phytoplankton thrive in these nutrient- with the Pacific Ocean:
rich waters, forming the basis of the food chain. • The Kuroshio Current is a warm, north-flowing current
This current is one of the world’s most productive in the western North Pacific.
ecosystems and sustains significant fisheries. • The California Current is a cold, south-flowing current
• Trade winds drive the circulation of the Humboldt along the western coast of North America.
Current, influenced by the Intertropical Convergence • The Humboldt Current, also known as the Peru Current,
flows northward along the western coast of South
Zone and trade winds’ latitudinal movements.
America.
25. Solution: (b) Important Tips
• The Kuroshio Current , also known as the Black
Exp) Option b is the correct answer. or Japan Current or the Black Stream, is a north-
The Benguela Current is the oceanic current that does flowing, warm ocean current on the west side of
the North Pacific Ocean basin. It was named for
not flow in the North Atlantic Ocean. It is, in fact, a the deep blue appearance of its waters. Similar to the
branch of the Southern Hemisphere’s West Wind Drift and Gulf Stream in the North Atlantic, the Kuroshio is a
powerful western boundary current that transports
flows northward along the west coast of southern Africa, warm equatorial water poleward and forms the western
extending into the South Atlantic Ocean. This current is limb of the North Pacific Subtropical Gyre. Off the
East Coast of Japan, it merges with the Oyashio
responsible for the eastern limit of a vast gyre in the South Current to form the North Pacific Current.
Atlantic Ocean and influences the coastal regions of South • The California Current is a cold water Pacific Ocean
current that moves southward along the western
Africa, Namibia, and Angola, not the North Atlantic Ocean.
coast of North America, beginning off southern British
Columbia and ending off southern Baja California Sur.
Important Tips
It is considered an Eastern boundary current due to
• The Gulf Stream, together with its northern the influence of the North American coastline on its
extension the North Atlantic Drift, is a warm and course. It is also one of five major coastal currents
swift Atlantic ocean current that originates in the affiliated with strong upwelling zones, the others
Gulf of Mexico and flows through the Straits of Florida being the Humboldt Current, the Canary Current,
and up the eastern coastline of the United States, then the Benguela Current, and the Somali Current. The
veers east near 36°N latitude (North Carolina) and California Current is part of the North Pacific Gyre, a
moves toward Northwest Europe as the North Atlantic large swirling current that occupies the northern basin
Current. of the Pacific.

75 PYQ Workbook
GENERAL GEOGRAPHY

27. Solution: (d) 31. Solution: (a)


Exp) Option d is the correct answer. Exp) Option a is the correct answer.
The most extensive part of the oceanic floor is the “Deep Statement 1 is correct- Evaporation of ocean water leaves
Sea Plains,” which covers more than 50% of the Earth’s behind dissolved salts, which increases the salinity.
surface. These plains are characterized by their flatness and
Precipitation of water dilutes the salinity.
are located at depths between 3,000 and 6,000 meters below
the ocean surface. Abyssal plains result from the spreading Statement 2 is correct- As the temperature of water
of the seafloor and the deposition of fine-grained increases, its density decreases. This means that the water
sediments, mainly clay and silt. They play a crucial role in can hold less dissolved salts, so the salinity decreases.
oceanic biodiversity and carbon cycling. Conversely, as the temperature of water decreases, its density
increases. This means that the water can hold more dissolved
28. Solution: (a) salts, so the salinity increases.
Exp) Option a is the correct answer.
Statement 3 is incorrect- The major source of sea salinity is
The correct match for List-I with List-II is as follows: evaporation. Terrestrial discharge by rivers does contribute
A. Gulf Stream - 4. Warm current to salinity, but it is a much smaller factor than evaporation.
B. West Wind Drift - 2. A slow eastward movement of water
over the zone of westerly wind 32. Solution: (c)

C. Peru Current - 1. Pacific Ocean Exp) Option c is the correct answer.

D. West Australian Current - 3. Indian Ocean The continental shelves are characterized by a gentle slope
rather than abrupt falls towards the continental slopes.
29. Solution: (b)
The continental shelves are often covered with varying
Exp) Option b is the correct answer. thicknesses of sediment that have been transported by
“Land” is considered the main source of sea salinity. The rivers, glaciers, and other natural processes.
land is the origin of many minerals and salts that find their
way into the ocean over geological timescales. As rocks on Important Tips
land weather and erode, they release dissolved salts and Some other major oceanic landforms:
minerals, which are carried by rainwater into rivers and • Continental slope: It is a steep drop-off that leads
eventually reach the sea. Geological processes like volcanic from the continental shelf to the deep ocean. It can
activity and salt domes also contribute to the introduction of be up to 4 kilometers deep and is home to a variety of
salts from the land into the ocean, highlighting the significant deep-sea creatures, such as whales, sharks, and giant
role of terrestrial sources in determining sea salinity levels. squid.
• Continental rise: It is a gently sloping area that lies
30. Solution: (a)
at the base of the continental slope. It is typically
Exp) Option a is the correct answer. 2 kilometers to 4 kilometers deep and is home to a
Wavelength is the distance between two successive crests variety of deep-sea sediments.
or troughs of a wave. • Abyssal plain: It is the flattest and deepest part of the
ocean floor. It covers about 40% of the Earth’s surface
and is typically 4 kilometers to 6 kilometers deep. The
abyssal plain is home to a variety of deep-sea creatures,
such as sea cucumbers, sea stars, and sea anemones.
• Mid-ocean ridge: It is a continuous mountain range
that runs through the center of all the major oceans.
It is formed by the upwelling of molten rock from
the Earth’s mantle. The mid-ocean ridge is home to a
variety of hydrothermal vents, which are areas where
hot, mineral-rich water flows out of the ocean floor.
Hydrothermal vents support a diverse range of marine
Important Tips
life, including tubeworms, clams, and crabs.
• Wave height is the distance between a crest and a
• Ocean trench: It is a long, narrow depression in the
trough of a wave.
ocean floor. Ocean trenches are formed when one
• Wave frequency is the number of waves that pass a tectonic plate slides underneath another. The deepest
certain point in a specified amount of time. ocean trench in the world is the Mariana Trench, which
• Fetch is the distance over which the wind blows over is located in the western Pacific Ocean and is over 11
the water surface before it reaches the coast. kilometers deep.

PYQ Workbook 76
GENERAL GEOGRAPHY

Important Tips
Factors influencing ocean currents:
• Solar heating: Sun’s heat warms surface water, making
it less dense and causing it to rise and create currents
that transport warm water away from the equator
towards the poles.
• Wind: Wind blows surface water, generating waves and
currents. Variations in wind direction and speed lead
to changes in ocean currents.
• Coriolis force: Earth’s rotation deflects moving water,
33. Solution: (a) causing ocean currents to curve as they travel across
the Earth’s surface.
Exp) Option a is the correct answer.
• Topography: Ocean floor topography, like the Mid-
The ongoing decrease in the pH value of ocean water, often Atlantic Ridge, can obstruct or redirect the flow of
referred to as ocean acidification, is primarily caused by a water, influencing ocean currents.
larger uptake of carbon dioxide (CO2) by ocean water. As • Salinity: Differences in water salinity affect density,
the concentration of CO2 increases in the atmosphere due with saltier water sinking and creating currents that
to human activities such as the burning of fossil fuels, a move from the surface to the ocean bottom.
portion of that CO2 is absorbed by the ocean. This leads to 36. Solution: (d)
a chemical reaction in the water, resulting in the formation Exp) Option d is the correct answer.
of carbonic acid. The increased carbonic acid concentration
The cold ocean current among the given options is
lowers the pH of the ocean, making it more acidic. the California Current. Ocean currents are continuous
movements of seawater driven by various factors such as
34. Solution: (d)
wind, temperature, and the Earth’s rotation. The California
Exp) Option d is the correct answer Current flows southward along the western coast of North
Isohalines are lines that join points of equal salinity in an America. It is influenced by the cold waters of the California
aquatic system. They are used to map the distribution of Current System, which originates from the subarctic region
and moves southward along the California coastline.
salinity in the ocean and other bodies of water.
Important Tips
Important Tips
Some Important Cold Currents:
• Isobars: Isobars are lines that connect points of equal
atmospheric pressure. Atmospheric pressure is the California Current: A cold ocean current that flows
force exerted by the atmosphere on the Earth’s surface. southward along the western coast of North America,
It is measured in millibars mb). bringing cold water from higher latitudes.

• Isohyets: Isohyets are lines that connect points of equal Benguela Current: A cold ocean current that flows
rainfall. Rainfall is the amount of water that falls from northward along the southwestern coast of Africa,
the sky as rain, snow, sleet, or hail. It is measured in influencing the marine ecosystems and climate in the
millimeters mm) or inches in). region.

• Isotherms: Isotherms are lines that connect points of Canary Current: A cold ocean current that flows
equal temperature. Temperature is a measure of how southward along the western coast of North Africa,
hot or cold something is. It is measured in degrees affecting the weather patterns and marine life in the area.
Celsius °C) or degrees Fahrenheit °F). Peru Current: A cold ocean current that flows northward
along the western coast of South America, leading to
35. Solution: (c) the upwelling of nutrient-rich waters and supporting a
Exp) Option c is the correct answer productive ecosystem.
Labrador Current: A cold ocean current that flows
Gravitational pull by the Sun and Moon primarily affects
southward along the eastern coast of Canada, bringing
ocean tides. The gravitational forces exerted by these cold water from the Arctic and influencing the climate of
celestial bodies cause the water levels to rise and fall in the the region.
oceans, resulting in the formation of tides. While tides can Falkland Current: A cold ocean current that flows
indirectly affect ocean currents by altering water levels, northward along the eastern coast of Argentina and the
they do not directly drive the movement of water over Falkland Islands, contributing to the unique marine
environment in the area.
large distances as ocean currents do.

77 PYQ Workbook
GENERAL GEOGRAPHY

The warm water of the Gulf Stream does not sustain the coral
Oyashio Current: A cold ocean current that flows
southward along the eastern coast of Japan, bringing cold reefs of the West Pacific Coast. Coral reefs are primarily
water from the North Pacific and impacting the climate found in tropical and subtropical regions, particularly in
and fisheries in the region. the Indo-Pacific region, including the West Pacific Coast.
The Gulf Stream, on the other hand, is a strong, warm ocean
37. Solution: (b) current that flows from the Gulf of Mexico along the
Exp) Option b is the correct answer. eastern coast of the United States and then moves across
Among the given options, the warm ocean currents are the the Atlantic Ocean towards Europe. It plays a significant
North Atlantic Drift and Brazil Current. The North Atlantic role in shaping the weather patterns along the East coast
Drift carries warm waters from the Gulf of Mexico into the of the USA.
North Atlantic Ocean, moderating the climate of European
41. Solution: (c)
countries. The Brazil Current runs along the eastern coast
of South America, transporting warm waters from the Exp) Option c is the correct answer
equatorial regions southward and impacting the climate of Statement I is true: The Kuroshio is a warm North-flowing
countries like Brazil, Uruguay, and Argentina. ocean current on the West side of the North Pacific ocean. It
Important Tips is formed by the North Equatorial Current, which is a warm
current that flows from the west to the east in the Pacific
Some other warm ocean currents along with their location:
Ocean.
Gulf Stream: It originates in the Gulf of Mexico and flows
Statement II is false: The presence of volcanoes at the bottom
along the eastern coast of North America towards Europe.
of the Sea of Japan does not contribute to the warmth of the
Kuroshio Current: It flows along the eastern coast of
Kuroshio Current. The Kuroshio Current is warm because
Asia, particularly affecting the waters near Japan.
it originates in the tropics, where the water is warm.
Agulhas Current: It flows along the eastern coast of South
Africa and influences the weather patterns in the region. Important Tips
North Equatorial Current: It flows westward across the • The Kuroshio Current begins near the Philippines and
tropical Pacific Ocean, influencing the weather patterns flows past Taiwan and Japan.
in the region. • It then merges with the Oyashio Current off the coast
East Australian Current: It flows southward along of Japan to form the North Pacific Current.
the eastern coast of Australia, impacting the marine • The Kuroshio Current is about 1,000 kilometers wide
ecosystems and weather patterns in the region. and 200 meters deep. It flows at speeds of up to 2
meters per second.
38. Solution: (b)
• The Kuroshio Current is a warm current, with
Exp) Option b is the correct answer. temperatures that can reach up to 25 degrees Celsius.
The Mariana Trench is located in the western Pacific Ocean • This warm water helps to keep the coastal regions of
and is known for its extreme depth. It reaches a maximum Japan warm, even though they are located at similar
depth of approximately 10,994 meters (36,070 feet) at a latitudes to Alaska and Siberia.
location called the Challenger Deep. This makes it the
deepest known point in the world’s oceans.

39. Solution: (c)


Exp) Option c is the correct answer.
Gulf Stream flows along the western boundary of the Sargasso
Sea, while the Canary Current is located further east, along
the coast of northwest Africa.
Statement a is correct: The Sargasso Sea is known for its
anti-cyclonic circulation, where ocean currents move in a
clockwise direction.
Statement b is correct: It records the highest salinity in
Atlantic Ocean. 42. Solution: (b)
Statement d is correct: he Sargasso Sea is located within
Exp) Option b is the correct answer.
a gyre, which is an area of calm and relatively motionless
water. The Cold Labrador Current is a cold ocean current that flows
southward along the coast of Labrador and Newfoundland in
40. Solution: (d) Canada, affecting the temperature and nutrient distribution
Exp) Option d is the correct answer. in the North Atlantic Ocean. It creates favorable conditions

PYQ Workbook 78
GENERAL GEOGRAPHY

for the formation of productive fishing grounds in the Important Tips


North-West Atlantic. • The Labrador Current is a cold current in the North
If the Cold Labrador Current were absent, the absence of Atlantic Ocean that flows from the Arctic Ocean south
the cold water flow would disrupt the current patterns and through Labrador’s coast.

alter the temperature and nutrient distribution in the region. • This cold water stream joins the warm, northward-
moving Gulf Stream at Nova Scotia.
This change in oceanic conditions would likely lead to the
• The confluence of these two currents creates dense fog
disappearance of the North-West Atlantic fishing grounds,
and one of the world’s most productive fishing places.
impacting the availability of fish stocks in that area.

79 PYQ Workbook
GENERAL GEOGRAPHY

GENERAL GEOGRAPHY
CLIMATIOLOGY
*This unit consists of questions from Structure and Composition of the atmosphere, Temperature
distribution and Heat budget of the earth, Pressure belts and Wind system, Humidity and Precipitation,
Climatic process like Cylone, El Nino etc, Classification of Climates and Climatic types, Natural
Vegetation.

4.1. UPSC CSE Previous Years’ Questions (a) 1 only


(b) 2 only
1. Consider the following trees: (c) Both 1 and 2
1. Jackfruit (Artocarpus heterophyllus) (d) Neither 1 nor 2
2. Mahus (Madhuca indica) 4. “Leaf litter decomposes faster than in
3. Teak (Tectona grandis) any other biome and as a result the soil
How many of the above are deciduous trees? surface is often almost bare. Apart from
[UPSC CSE Pre 2023] trees, the vegetation is largely composed of
(a) Only one plant forms that reach up into the canopy
(b) Only two vicariously, by climbing the trees or growing
(c) All three as epiphytes, rooted on the upper branches
(d) None of trees,” This is the most likely description
of [UPSC CSE Pre 2021]
2. Consider the following statements: (a) coniferous forest
Statement-I: The soil in tropical rain forests (b) dry deciduous forest
is rich in nutrients (c) mangrove forest
Statement-II: The high temperature and (d) tropical rain forest
moisture of tropical rain forests cause dead 5. The Vegetation of savannah consists of
organic matter in the soil to decompose grassland with scattered small trees, but
quickly. extensive areas have no trees. The forest
Which one of the following is correct in development in such areas is generally kept
respect of the above statements? in check by one or more or a combination of
[UPSC CSE Pre 2023] some conditions.
(a) Both Statement-I and Statement- are Which of the following are such conditions?
correct and Statement- is the correct 1. Burrowing animals and termites
explanation for Statement-I. 2. Fire
(b) Both Statement-1 and Statement- are
3. Grazing herbivores
correct and Statement-I is not the correct
explanation for Statement-1. 4. Seasonal rainfall
(c) Statement-l is correct but Statement-II is 5. Soil properties
incorrect. Select the correct answer using the code given
(d) Statement-I is incorrect but Statement-II below: [UPSC CSE Pre 2021]
is correct. (a) 1 and 2
3. Consider the following statements: (b) 4 and 5
(c) 2, 3 and 4
1. In the tropical zone, the western sections
(d) 1, 3 and 5
of the oceans are warmer than the eastern
sections, owing to the influence of trade 6. Consider the following statements:
winds. 1. Jet streams occur in the Northern
2. In the temperate zone, westerlies make Hemisphere only.
the eastern sections of oceans warmer 2. Only some cyclones develop an eye.
than the western sections. 3. The temperature inside the eye of a
Which of the statements given above is/are cyclone is nearly 10°C lesser than that of
correct? [UPSC CSE Pre 2021] the surroundings.

PYQ Workbook 80
GENERAL GEOGRAPHY

Which of the statements given above is/are 2. The moist air masses that cause winter
correct? [UPSC CSE Pre. 2020] rains in North-Western region of India
(a) 1 only are part of westerlies.
(b) 2 and 3 only Which of the statements given above is/are
(c) 2 only correct?
(d) 1 and 3 only
(a) 1 only
7. Consider the following statements : (b) 2 only
1. High clouds primarily reflect solar (c) Both 1 and 2
radiation and cool the surface of the (d) Neither 1 nor 2
Earth.
12. “Each day is more or less the same, the
2. Low clouds have a high absorption of
morning is clear and bright with a sea
infrared radiation emanating from the
Earth’s surface and thus cause warming breeze; as the Sun climbs high in the sky,
effect. heat mounts up, dark clouds form, then
rain comes with thunder and lightning. But
Which of the statements given above is/are
rain is soon over.” Which of the following
correct ? [UPSC CSE Pre. 2020]
regions is described in the above passage?
(a) 1 only
(b) 2 only [UPSC CSE Pre. 2015]
(c) Both 1 and 2 (a) Savannah
(d) Neither 1 nor 2 (b) Equatorial
(c) Monsoon
8. Why are dewdrops not formed on a cloudy (d) Mediterranean
night? [UPSC CSE Pre. 2019]
(a) Clouds absorb the radiation released 13. Variations in the length of daytime and
from the Earth’s surface. night time from season to season are due
(b) Clouds reflect back the Earth’s radiation. to [UPSC CSE Pre. 2013]
(c) The Earth’s surface would have low (a) the earth’s rotation on its axis.
temperature on cloudy nights. (b) the earth’s revolution round the sun in an
(d) Clouds deflect the blowing wind to elliptical manner.
ground level. (c) latitudinal position of the place.
9. In the South Atlantic and South-Eastern (d) revolution of the earth on a tilted axis.
Pacific regions in tropical latitudes, cyclone 14. On the planet earth, most of the freshwater
does not originate. What is the reason? exists as ice caps and glaciers. Out of
[UPSC CSE Pre. 2015] the remaining freshwater, the largest
(a) Sea surface temperatures are low. proportion [UPSC CSE Pre. 2013]
(b) Inter-tropical Convergence Zone seldom (a) is found in atmosphere as moisture and
occurs.
clouds.
(c) Coriolis force is too weak.
(b) is found in freshwater lakes and rivers.
(d) Absence of land in those regions.
(c) exists as groundwater.
10. What explains the eastward flow of the (d) exists as soil moisture.
equatorial counter-current?
[UPSC CSE Pre. 2015] 15. Which of the following leaf modifications
occurs/occur in desert areas to inhibit water
(a) The Earth’s rotation on its axis.
loss? [UPSC CSE Pre 2013]
(b) Convergence of the two equatorial
currents. 1. Hard and waxy leaves
(c) Difference in salinity of water. 2. Tiny leaves or no leaves
(d) Occurrence of the belt of calm near the 3. Thorns instead of leaves
equator. Select the correct answer using the codes
11. Consider the following statements: given below.
[UPSC CSE Pre. 2015] (a) 1 and 2 only
1. The winds which blow between 30 N and (b) 2 only
60 S latitudes throughout the year are (c) 1 and 3 only
known as westerlies. (d) 1, 2 and 3

81 PYQ Workbook
GENERAL GEOGRAPHY

16. The annual range of temperature in Select the correct answer using the codes
the interior of the continents is high as given below.
compared to coastal areas. What is/are the (a) 1 only
reason/reasons? [UPSC CSE Pre. 2013] (b) 2 and 3
1. Thermal difference between land and (c) 1 and 3
water. (d) None of the above produces the thunder
2. Variation in altitude between continents 20. The 2004 Tsunami made people realize that
and oceans. mangroves can serve as a reliable safety
3. Presence of strong winds in the interior. hedge against coastal calamities. How do
4. Heavy rains in the interior as compared mangroves function as a safety hedge?
to coasts. [UPSC CSE Pre. 2011]
Select the correct answer using the codes (a) The mangrove swamps separate the
given below. human settlements from the sea by a
(a) 1 only wide zone in which people neither live
(b) 1 and 2 only nor venture out.
(c) 2 and 3 only (b) The mangroves provide both food and
(d) 1, 2, 3 and 4 medicines which people are in need of
after any natural disaster.
17. Which of the following is/are unique (c) The mangrove trees are tall with dense
characteristic/characteristics of equatorial canopies and serve as an excellent shelter
forests? [UPSC CSE Pre. 2013] during a cyclone or tsunami.
1. Presence of tall, closely set trees with (d) The mangrove trees do not get uprooted
crowns forming a continuous canopy. by storms and tides because of their
2. Coexistence of a large number of species. extensive roots.
3. Presence of numerous varieties of 21. The jet aircrafts fly very easily and smoothly
epiphytes. in the lower stratosphere. What could be
Select the correct answer using the code given the appropriate explanation?
below: [UPSC CSE Pre. 2011]
(a) 1 only 1. There are no clouds or water-vapour in
(b) 2 and 3 only the lower stratosphere.
(c) 1 and 3 only 2. There are no vertical winds in the lower
(d) 1, 2 and 3 stratosphere.
18. “Climate is extreme, rainfall is scanty and Which of the statements given above is/are
the people used to be nomadic herders.” correct in this context?
[UPSC CSE Pre. 2013] (a) 1 only
(b) 2 only
The above statement best describes which of (c) Both 1 and 2
the following regions? (d) Neither 1 nor 2
(a) African Savannah
(b) Central Asian Steppe 22. What could be the main reason/reasons
(c) North American Prairie of the formation of African and Eurasian
(d) Siberian Tundra desert belt? [UPSC CSE Pre. 2011]
1. It is located in the sub-tropical high-
19. During a thunderstorm, the thunder in the pressure cells.
skies is produced by the 2. It is under the influence of warm ocean
[UPSC CSE Pre. 2013] currents.
1. meeting of cumulonimbus clouds in the Which of the statements given above is/are
sky. correct in this context?
2. lightning that separates the nimbus (a) 1 only
clouds. (b) 2 only
3. violent upward movement of air and (c) Both 1 and 2
water particles. (d) Neither 1 nor 2

PYQ Workbook 82
GENERAL GEOGRAPHY

23. La Nina is suspected to have caused recent 27. Normally, the temperature decreases with
floods in Australia. How is La Nina different the increase in height from the Earth’s
from El Nino? [UPSC CSE Pre. 2011] surface, because [UPSC CSE Pre. 2012]
1. La Nina is characterised by unusually cold 1. the atmosphere can be heated upwards
ocean temperature in equatorial Indian only from the Earth’s surface.
Ocean whereas El Nino is characterised 2. there is more moisture in the upper
by unusually warm ocean temperature in atmosphere.
the equatorial Pacific Ocean.
3. the air is less dense in the upper
2. El Nino has adverse effect on south-west
atmosphere.
monsoon of India, but La Nina has no
effect on monsoon climate. Select the correct answer using the codes
Which of the statements given above is/are given below:
correct? (a) 1 only
(a) 1 only (b) 2 and 3 only
(b) 2 only (c) 1 and 3 only
(c) Both 1 and 2 (d) 1, 2 and 3
(d) Neither 1 nor 2 28. Which one of the following is the
24. Westerlies in southern hemisphere are characteristic climate of the Tropical
stronger and persistent than in northern Savannah Region? [UPSC CSE Pre. 2012]
hemisphere. Why? [UPSC CSE Pre. 2011] (a) Rainfall throughout the year
1. Southern hemisphere has less landmass (b) Rainfall in winter only
as compared to northern hemisphere. (c) An extremely short dry season
2. Coriolis force is higher in southern (d) A definite dry and wet season
hemisphere as compared to northern
hemisphere. 29. A geographic region has the following
distinct characteristics:
Which of the statements given above is/are
correct? [UPSC CSE Pre 2010]
(a) 1 only 1. Warm and dry climate.
(b) 2 only 2. Mild and wet winter.
(c) Both 1 and 2 3. Evergreen oak trees.
(d) Neither 1 nor 2 The above features are the distinct
25. The jet aircrafts fly very easily and smoothly characteristics of which one of the following
in the lower stratosphere. What could be regions?
the appropriate explanation? (a) Mediterranean
[UPSC CSE Pre. 2011] (b) Eastern China
1. There are no clouds or water-vapour in (c) Central Asia
the lower stratosphere. (d) Atlantic coast of North America
2. There are no vertical winds in the lower
30. Which one of the following reflects more
stratosphere.
sunlight as compared to the three?
Which of the statements given above is/are
[UPSC CSE Pre (Pre) 2010]
correct in this context?
(a) Sand desert
(a) 1 only
(b) 2 only (b) Paddy cropland
(c) Both 1 and 2 (c) Land covered with fresh snow
(d) Neither 1 nor 2 (d) Prairie land

26. The seasonal reversal of winds is the typical 31. Which type of forests are found near the
characteristic of [UPSC CSE Pre. 2012] equator? [UPSC CSE Pre 2007]
(a) Equatorial climate (a) Deciduous forests
(b) Mediterranean climate (b) Coniferous forest
(c) Monsoon climate (c) Meadows forest
(d) All of the above climates (d) Tropical forest

83 PYQ Workbook
GENERAL GEOGRAPHY

32. Consider the following statements: (a) Both (A) and (R) are true, and (R) is the
[UPSC CSE Pre 2007] correct explanation of (A).
1. The annual range of temperature is (b) Both (A) and (R) are true, but (R) is not
greater in the Pacific Ocean than that in the correct explanation of (A).
the Atlantic Ocean. (c) (A) is true, but (R) is false.
2. The annual range of temperature is (d) (A) is false, but (R) is true.
greater in the Northern Hemisphere than 37. Which one among the following covers
that in the Southern Hemisphere. the highest percentage of forest area in the
Which of the statements given above is/are world? [UPSC CSE Pre 2003]
correct? (a) Temperate Coniferous forests
(a) 1 only (b) Temperate Deciduous forests
(b) 2 only (c) Tropical Monsoon forests
(c) Both 1 and 2 (d) Tropical Rain forests
(d) Neither 1 nor 2
38. Consider the following statements:
33. Consider the following statements:
Assertion (A): Unlike temperate forests,
1. Either of the two belts over the oceans the tropical rain forests, if cleared, can
at about 300 to 350 N and S latitudes is
yield productive farmland that can support
known as Horse Latitudes.
intensive agriculture for several years even
2. Horse Latitudes are low pressure belts. without chemical fertilizers.
Which of the statements given above is/are Reason (R): The primary productivity of the
correct? [UPSC CSE Pre 2007]
tropical rain forest is very high compared to
(a) 1 only that of temperate forests.
(b) 2 only
Code: [UPSC CSE Pre 2003]
(c) Both 1 and 2
(d) Neither 1 nor 2 (a) Both (A) and (R) are true, and (R) is the
correct explanation of (A).
34. Dalbergia species is associated with which (b) Both (A) and (R) are true, but (R) is not
one of the following? [UPSC CSE Pre 2007] the correct explanation of (A).
(a) Cashew nut (c) (A) is true, but (R) is false.
(b) Coffee (d) (A) is false, but (R) is true.
(c) Tea
(d) Rosewood 39. Consider the following statements:
Assertion (A): The surface winds spiral
35. Consider the following statements:
inwards upon the centre of the cyclone.
Assertion (A): The amount of moisture in
the atmosphere is associated with latitude. Reason (R): Air descends at the centre of the
cyclone.
Reason (R): The ability to keep the moisture
in the form of water vapour is related to Code: [UPSC CSE Pre 2002]
temperature. (a) Both (A) and (R) are true, and (R) is the
Code: [UPSC CSE Pre 2006] correct explanation of (A).
(b) Both (A) and (R) are true, but (R) is not
(a) Both (A) and (R) are true, and (R) is the
the correct explanation of (A).
correct explanation of (A).
(b) Both (A) and (R) are true, but (R) is not (c) (A) is true, but (R) is false.
the correct explanation of (A). (d) (A) is false, but (R) is true.
(c) (A) is true, but (R) is false. 40. Consider the following statements:
(d) (A) is false, but (R) is true. Assertion (A): 600–650 latitudes in both the
36. Consider the following statements: hemispheres have a low-pressure belt instead
Assertion (A): Wind patterns are clockwise in of high pressure.
the northern hemisphere and anti-clockwise Reason (R): The low-pressure areas are stable
in the southern hemisphere. over oceans rather than on land.
Reason (R): The directions of wind patterns Code: [UPSC CSE Pre 2002]
in the northern and the southern hemisphere (a) Both (A) and (R) are true, and (R) is the
are governed by the Coriolis effect. correct explanation of (A).
Code: [UPSC CSE Pre 2005] (b) Both (A) and (R) are true, but (R) is not

PYQ Workbook 84
GENERAL GEOGRAPHY

the correct explanation of (A). 4. Overcast skies, rain and raw weather are
(c) (A) is true, but (R) is false. generally associated with them.
(d) (A) is false, but (R) is true. Which of these statements are correct?
41. Consider the following statements: (a) 1, 2 and 3
[UPSC CSE Pre 2002] (b) 2, 3 and 4
1. In Equatorial regions, the year is divided (c) 1, 3 and 4
into four main seasons. (d) 1, 2 and 4
2. In the Mediterranean region, summer 45. Consider the following statements:
receives more rain. Statement (A): During the times of neap-
3. In China type climate, rainfall occurs tide, high-tide is below normal and low-tide
throughout the year. is above normal.
4. Tropical highlands exhibit vertical Reason (R): Neap-tides occurs during the
zonation of different climates. new-moon instead of full-Moon.
Which of these statements are correct? Choose the correct answer using following
(a) 1, 2, 3 and 4 options. [UPSC CSE Pre 1998]
(b) 1, 2 and 3 (a) Both (A) and (R) are true, and (R) is the
(c) 1, 2 and 4
correct explanation of (A).
(d) 3 and 4
(b) Both (A) and (R) are true, but (R) is not
42. For short-term climatic predictions, which the correct explanation of (A).
one of the following events, detected in the (c) (A) is true, but (R) is false.
last decade, is associated with occasional (d) (A) is false, but (R) is true.
weak monsoon rains in the India sub-
46. The correct sequence of different layers
continent? [UPSC CSE Pre 2002]
of the atmosphere from the surface of the
(a) La Nina Earth upwards is: [UPSC CSE Pre 1998]
(b) Movement of Jet Streams
(c) El-Nino and Southern Oscillations (a) Troposphere, Stratosphere, Ionosphere,
(d) Greenhouse effect on a global level Mesosphere.
(b) Stratosphere, Troposphere, Ionosphere,
43. Match list-I with list-II and select the Mesosphere.
correct answer using the codes given below (c) Troposphere, Stratosphere, Mesosphere,
the lists: Ionosphere.
List-I List-II (d) Stratosphere, Troposphere, Mesosphere,
Ionosphere.
(Local Wind) (Region)
A. Foehn 1. Argentina 47. Which one of the following countries is the
largest producer of fuelwood in the world?
B. Simoom 2. Kurdistan [UPSC CSE Pre 1997]
C. Santa Ana 3. California (a) Indonesia
D. Zonda 4. Alps (b) Russia
Code: [UPSC CSE Pre 2001] (c) India
(d) China
A B C D
(a) 2 4 1 3 48. Given below are two statements one
(b) 4 2 3 1 labelled as Assertion and the other labelled
(c) 2 4 3 1 as Reason (R). [UPSC CSE Pre 2003]
(d) 4 2 1 3 Assertion (A): Areas lying within five to
44. Consider the following statements about eight-degree latitude on either side of the
the ‘Roaring Forties’: [UPSC CSE Pre 2000] equator receive rainfall throughout the year.
1. They blow uninterrupted in the northern Reason (R): High temperature and high
and southern hemispheres. humidity cause convectional rainfall mostly
2. They blow with great strength and in the noon near the equator.
constancy. By using the codes given below, select the
3. Their direction is generally from north- correct answer:
west to east in the southern hemisphere. (a) Both (A) and (R) are true, and (R) is the

85 PYQ Workbook
GENERAL GEOGRAPHY

correct explanation of (A). (a) Transmitting power


(b) Both (A) and (R) are true, but (R) is not (b) Absorbing power
the correct explanation of (A). (c) Emissive power
(c) (A) is true, but (R) is false. (d) Reflecting power
(d) (A) is false, but (R) is true. 53. Match list-I with list-II and select the
49. Match list-I with list-II and select the correct answer using the codes given below
correct answer using the codes given below the lists:
the lists:
List-I List-II
List-I List-II (Vegetation Type) (Region)
(Timber) (Country) A. Maquis 1. California
A. Cedar 1. Myanmar B. Fynbos 2. M e d i t e r r a n e a n
B. Douglas Fir 2. Canada Sea Coastal
Region
C. Mahogany 3. Mexico
C. Chaparral 3. South Africa
D. Teak 4. Honduras
D. Matorral 4. Chile
Code: [UPSC CSE Pre 1999] Code: [U.P.P.C.S. (Pre) 2019]
A B C D A B C D
(a) 3 2 1 4 (a) 2 3 1 4
(b) 3 2 4 1 (b) 4 3 2 1
(c) 2 3 4 1 (c) 3 4 1 2
(d) 2 3 1 4 (d) 4 2 3 1
4.2. Other UPSC CSE Previous Years’ 54. Match list-I with list-II and select the
Questions correct answer using the codes given below
50. Given below are two statements, one is the lists:
labelled as Assertion and the other as List-I List-II
Reason (R). (Natural (Region)
Assertion (A): Rainfall occurs in the winter Vegetation)
season in Mediterranean climate.
A. Epiphytes 1. Mediterranean
Reason (R): In summer these regions remain
B. Acacia 2. Equatorial
under the influence of dry terrestrial winds.
Select the correct answer from the codes C. Baobab 3. Sahara
given below. [U.P.P.C.S. (Pre) 2022] D. Cedars 4. Savanna
(a) (A) is true but (R) is false. Code: [U.P.P.C.S. (Pre) 2019]
(b) Both (A) and (R) are true, and (R) is the A B C D
correct explanation of (A). (a) 2 3 4 1
(c) (A) is false but (R) is true. (b) 2 3 1 4
(d) Both (A) and (R) are true, but (R) is not (c) 2 4 3 1
the correct explanation of (A). (d) 2 4 1 3
51. Which one of the following pairs is not 55. Match list-I with list-II and select the
correctly matched? [U.P.P.C.S. (Pre) 2021] correct answer using the codes given below
Name of local Places the lists:
winds List-I List-II
(a)Leveche Spain (Different name of (Country)
(b) Brick fielder Australia Tropical cyclone)
(c) Black roller North America A. Willy-Willies 1. Philippines
(d)Shamal Austria B. Taifu 2. Australia
C. Baguio 3. Japan
52. Which one of the following is associated
with ‘Albedo’? [U.P.P.C.S. (Pre) 2019] D. Hurricanes 4. USA

PYQ Workbook 86
GENERAL GEOGRAPHY

Code: [U.P.P.C.S. (Pre) 2019] (d) Khamsin - Egypt


A B C D 62. Which of the following countries has a
(a) 3 4 1 2 constitutional provision for maintaining
(b) 2 3 4 1 70% of its geographical area under forest?
(c) 1 3 2 4 [U.P.P.C.S. (Mains) 2016]
(d) 2 3 1 4 (a) Maldives
56. Which of the following is not correctly (b) Nepal
matched? [U.P.P.C.S. (Pre) 2019] (c) Bhutan
(d) Afghanistan
Wind Country
63. Which of the following scales is used to
(a)Santa Ana California
measure the damage by a hurricane?
(b) Haboob Sudan [U.P.P.C.S. (Mains) 2016]
(c) Yamo Japan (a) Saffir-Simpson Scale
(d)Mistral Australia (b) Mercalli Scale
(c) Fujita Scale
57. Which of the following is concerned with (d) Richter scale
soil? [U.P.P.C.S. (Pre) 2018]
(a) Edaphic 64. Taiga forests are characteristics of–
(b) Climatic [U.P.P.C.S. (Mains) 2016]
(c) Biotic (a) Equatorial region
(d) Topography (b) Tropical region
(c) Sub-tropical region
58. As a result of global warming the frequency (d) Temperate region
and severity of which of the following are
increasing due to global warming? 65. Consider the following statements:
[U.P.P.C.S (Pre) 2018] Assertion (A): Of the soil species, clay holds
(a) Cyclones only maximum water.
(b) Storms only Reason (R): Porosity is found to be of large
(c) Hurricanes only size in clay.
(d) All of the above Code: [U.P.P.C.S. (Pre) 2015]
59. Arrange in chronological order: (a) (A) and (R) both are correct, and (R) is
1. Patpara formation the correct explanation of (A).
2. Khetaunhi formation (b) (A) is correct, but (R) is wrong.
3. Baghor formation (c) (A) and (R) both are correct, but (R) is
4. Sihawal formation not the correct explanation of (A).
(d) (A) is wrong, but (R) is true.
Code: [U.P.P.C.S. (Mains) 2017]
(a) 1, 4, 2, 3 66. Air pressure is lowest in:
(b) 4, 1, 3, 2 [U.P.P.C.S. (Pre) 2014]
(c) 1, 2, 3, 4 (a) Winter season
(d) 4, 3, 2, 1 (b) Spring season
60. Which of the following continents has all (c) Autumn season
types of climatic zones? (d) Summer season
[U.P.P.C.S. (Mains) 2016] 67. Which one of the following climates are
(a) South America associated with the shifting of the pressure
(b) North America belts? Select the correct answer from the
(c) Australia code given below:
(d) Asia 1. Equatorial climate
61. Which one of the following pairs is not 2. Mediterranean climate
correctly matched? [U.P.P.C.S. (Pre) 2016] 3. Monsoon climate
(a) Foehn - Alps Mountain 4. Western European climate
(b) Bora - Poland Code: [U.P.P.C.S. (Mains) 2010]
(c) Mistral - Rhine Valley (a) 1 and 2 only

87 PYQ Workbook
GENERAL GEOGRAPHY

(b) 1 and 3 only Select the correct answer from the code given
(c) 2 and 3 only below:
(d) 2 and 4 only (a) Both (A) and (R) are true, and (R) is the
68. Which layer of atmosphere is responsible correct explanation of (A).
for Aurora Borealis? [U.P.P.C.S (Pre) 2010] (b) Both (A) and (R) are true, but (R) is not
(a) Troposphere the correct explanation of (A).
(b) Thermosphere (c) (A) is true, but (R) is false.
(c) Ionosphere (d) (A) is false, but (R) is true.
(d) Exosphere 74. Which of the following statements is not
69. Which one of the following layers of the true? [U.P.P.C.S. (Pre) 2002]
atmosphere is responsible for the deflection (a) Presence of water vapour is highly
of radio waves? [U.P.P.C.S. (Mains) 2005] variable in the lower atmosphere.
(a) Troposphere (b) The zone of maximum temperature is
(b) Stratosphere located along the equator.
(c) Mesosphere (c) Frigid zones are located in both the
(d) Ionosphere hemispheres between the polar circles
and the poles.
70. Match list-I with list-II and select the (d) Jet streams are high altitude winds
correct answer using the codes given below affecting the surface weather conditions.
the lists:
75. Consider the following statements:
List-I List-II [U.P.P.C.S. (Mains) 2002]
A. Australia Hurricanes Assertion (A): The ports of north-west
B. China Willy-Willy Europe are open throughout the year.
C. India Typhoons Reason (R): The south-westerly winds blow
over north-west Europe throughout the year.
D. USA Cyclones
Code: [U.P.P.C.S. (Mains) 2005] Select the correct answer using the code given
below.
A B C D
(a) Both (A) and (R) are true, and (R) is the
(a) 1 2 3 4 correct explanation of (A).
(b) 2 3 4 1 (b) Both (A) and (R) are true, but (R) is not
(c) 3 2 1 4 the correct explanation of (A).
(d) 4 3 2 1 (c) (A) is true but (R) is false.
71. Which one of the following regions of the (d) (A) is false, but (R) is true.
U.S.A. is known as ‘Tornado Alley’? 76. Which one of the following is the correct
[U.P.P.C.S. (Pre) 2005] sequence of the climatic regions while going
(a) Atlantic Seaboard from Zaire to the Netherlands?
(b) Pacific Coast 1. Equatorial climate
(c) Mississippi Plains 2. Mediterranean climate
(d) Alaska
3. Hot desert climate
72. Which one of the following is not correctly 4. West European climate
matched? [U.P.P.C.S. (Pre) 2005] Code: [U.P.P.C.S. (Pre) 2002]
(a) Chinook - U.S.A. (a) 1, 3, 2, 4
(b) Sirocco - Sicily (b) 1, 4, 2, 3
(c) Blizzard - Chile (c) 2, 3, 4, 1
(d) Norwesters - India (d) 3, 2, 1, 4
73. Consider the following statements: 77. The coniferous forests are not found in:
[U.P.P.C.S. (Mains) 2004] [U.P.P.C.S. (Pre) 2001]
Assertion (A): The Mediterranean regions (a) Amazonia
have rainy winters. (b) Scandinavia
Reason (R): During winter, they lie in the (c) Canada
belt of the Westerlies. (d) Finland

PYQ Workbook 88
GENERAL GEOGRAPHY

78. Consider the following statements: 83. Soil leaching is a major problem
Assertion (A): The atmosphere receives most [U.P.P.C.S. (Pre) 1998]
of the heat only indirectly from the sun and (a) In tropical rainforests
directly from the earth’s surface. (b) In African savanna
Reason (R): The conversion from shortwave (c) In Mediterranean regions
solar to long wave terrestrial energy takes (d) In Australian Dawns
place at the earth’s surface. 84. The winds flowing from high-pressure area
Code: [U.P.P.C.S. (Pre) 1997] toward the Mediterranean Sea are-
(a) Both (A) and (R) are true, and (R) is the [U.P.P.C.S. (Pre) 1992]
correct explanation of (A). (a) Westerly
(b) Both (A) and (R) are true, but (R) is not (b) Trade wind
the correct explanation of (A). (c) Monsoonal wind
(c) (A) is true, but (R) is false. (d) Sea wind
(d) (A) is false, but (R) is true.
85. What are the causes of high-tide low tide
79. The Tornadoes are very strong tropical formation in the oceans?
cyclones originating in:
[U.P.P.C.S. (Pre) 1991]
[U.P.P.C.S. (Pre) 1996]
(a) Due to the effect of the sun.
(a) Caribbean Sea
(b) Due to the rotation of the Earth.
(b) China Sea
(c) Due to the combined effect of sun and the
(c) Arabian Sea
moon.
(d) Black Sea
(d) Due to Gravitation, Centripetal force and
80. Spring Tide occurs [U.P.P.C.S. (Pre) 1999] the centrifugal force.
(a) When the Sun the Earth and Moon are in
86. Hekistotherms are plants growing in–
a straight line.
(b) When the Sun and Moon make a right [U.P. Lower Sub. (Pre) 2013]
angle. (a) bright light
(c) When a strong wind blows. (b) high temperature
(d) When the night is very cold. (c) very low temperature
(d) very low light
81. Match list-I with list-II and select the
correct answer using the codes given below 87. To measure the intensity of Tornadoes we
the lists: use- [U.P. Lower Sub. (Pre) 2009]
(a) Mercalli scale
List-I List-II (b) Fujita Scale
(Region) (Type of Climate) (c) Saffir-samson scale
A. California 1. Mediterranean (d) Richter scale
B. West Australia 2. Tropical Monsoon 88. Highest altitude clouds are-
C. Bangladesh 3. Cold Temperate [U.P. Lower Sub. (Pre) 2009]
D. Siberia 4. Hot Desert (a) Altocumulus
Code: [U.P.P.C.S. (Pre) 1999] (b) Altostratus
(c) Cumulus
A B C D (d) Cirrostratus
(a) 1 2 3 4
(b) 1 4 2 3 89. By which of the following is the maximum
(c) 2 3 4 1 percentage of Earth covered?
(d) 2 4 1 3 [U.P. Lower Sub. (Pre) 2002]
82. In which of the following countries of (a) Arid Region
Africa, dense, hot and humid forests are (b) Semi-arid Region
found? [U.P.P.C.S. (Pre) 1998] (c) Sub-humid land
(d) Humid Region
(a) Ivory Coast
(b) Kenya 90. Match list-I with list-II and select the
(c) Zimbabwe correct answer using the codes given below
(d) South Africa the lists:

89 PYQ Workbook
GENERAL GEOGRAPHY

List-I List-II List-I List-II


A. Monsoon Forest 1. Plum and Olive (Soils) (Climate region)
B. Equatorial Forest 2. Pine and Spruce A. Podzol 1. Temperate cold
C. Mediterranean 3. Teak and Sal steppe
Forest B. Chernozem 2. Cold temperate
D. Coniferous 4. Mahogany and C. Spodosols 3. Hot and humid
Forest Rosewood D. Laterite 4. Humid cold
Code: [UP RO/ARO (Mains) 2013] temperate
A B C D Code: [R.A.S./RTS (Pre) (Re-Exam) 2013]
(a) 3 4 1 2 A B C D
(b) 2 3 4 1 (a) 2 1 4 3
(c) 4 2 3 1 (b) 3 4 2 1
(d) 1 3 2 4 (c) 2 3 4 1
91. Coniferous forests are mainly found in– (d) 4 1 3 2
[Uttarakhand Lower (Sub.) (Pre) 2010] 97. Which continent is known as ‘the Cradle of
(a) Tropical region Mankind’? [R.A.S./R.T.S. (Pre) 2012]
(b) Temperate region (a) Asia
(c) Coastal region (b) Africa
(d) None of the above (c) Europe
(d) South America
92. Clouds are the result of–
[Uttarakhand P.C.S. (Pre) 2016] 98. In which atmospheric layer are
communication satellites located?
(a) Evaporation
(b) Normal temp. lapse rate [Uttarakhand P.C.S. (Pre) 2005, R.A.S./
(c) Catabatic lapse rate R.T.S. (Pre) 1997]
(d) Condensation (a) Exosphere
(b) Stratosphere
93. The function of the ozone layer in the (c) Ionosphere
stratosphere is– (d) Troposphere
[Uttarakhand P.C.S. (Pre) 2002]
99. The cause of producing indirect high tide
(a) To stabilize the global temperature. is– [R.A.S./R.T.S. (Pre) 1999]
(b) To reduce the frequency of earthquake. (a) Gravitational force of the Moon.
(c) To avoid failure of monsoons. (b) Gravitational force of the Sun.
(d) To prevent ultra-violet radiation effect on (c) Centrifugal force of the Earth.
the ground. (d) Gravitational force of the Earth.
94. A sudden fall of Mercury in Barometer 100. Most weather activity occurs in which
indicates: atmospheric layer:
[Uttarakhand P.C.S. (Mains) 2002] [R.A.S./R.T.S. (Pre) 1999]
(a) Fair weather (a) Ozonosphere
(b) Stormy weather (b) Ionosphere
(c) Cold weather (c) Troposphere
(d) Hot weather (d) Exosphere
95. The evergreen rainforests are found in: 101. Intertropical Convergence Zone (ITCZ), a
[Uttarakhand P.C.S. (Pre) 2002] low-pressure zone, is located:
(a) Australia [M.P.P.C.S. (Pre) 2018]
(b) Brazil (a) between trade winds and westerlies belt
(c) Canada (b) between westerlies and polar winds belt
(d) France (c) nearby the poles
96. Match list-I with list-II and select the (d) at the equator
correct answer using the codes given below 102. What are roaring forties, furious fifties and
the lists: shrieking sixties? [M.P.P.C.S. (Pre) 2015]

PYQ Workbook 90
GENERAL GEOGRAPHY

(a) Oceanic storms (c) Stratosphere


(b) Westerlies in Southern hemisphere (d) Photosphere
(c) Westerlies in Northern hemisphere
109. Consider the following characteristics of a
(d) Pacific Ocean currents
cloud type: [CDS-2023(I)]
103. Halophytes grow well in– 1. They are born through convection.
[M.P.P.C.S. (Pre) 2014] 2. Only cloud type that can produce hail,
(a) Acidic soils thunder and lightning.
(b) Cold soils 3. They are large cauliflower-shaped towers,
(c) soil containing calcium often anvil tops’.
(d) Saline soils
Identify the type of cloud on the basis of the
104. What is the effect of Chinook winds in the given characteristics.
central plains in the USA? (a) Stratocumulus
[Jharkhand P.C.S. (Pre) 2013] (b) Cumulonimbus
(a) Temperature of winter rises (c) Cirrocumulus
(b) Temperature of summer reduces (d) Nimbostratus
(c) Uniform temperature 110. Which one of the following is not a resultant
(d) No effect on temperature
of the El Nino effect?
105. Consider the following statements: [CDS-2022(I)]
Assertion (A): Earthworms are not good for (a) Distortion of equatorial atmospheric
agriculture. circulation.
Reason (R): Earthworms break down the soil (b) Flow of the South-East Trade Winds
into fine particles and make it soft. towards the Indian Ocean.
Code: [Jharkhand P.C.S. (Pre) 2013] (c) Irregularities in the evaporation of sea
(a) (A) and (R) both are correct, and (R) is water.
the correct explanation of (A). (d) Reduction in the amount of planktons
(b) (A) is correct, and (R) is wrong. which reduces the number of fish in the
(c) (A) and (R) both are correct, but (R) is sea.
not the correct explanation of (A). 111. Which one of following is the most
(d) (A) is wrong, but (R) is true. important factor for the formulation of
106. The stratosphere is said to be ideal for smog? [CDS-2022(I)]
flying jet aircraft. This is because- (a) Long winter nights
[Jharkhand P.C.S. (Pre) 2010] (b) Formation of an ‘inversion lid’
(a) This layer is rich in ozone which reduces (c) Presence of many air pollutant resources
fuel consumption. (d) Rapid fall in temperature with increasing
(b) The temperature is constant and ideal for height above the sea level
aircraft engine efficiency. 112. Which one of the following land surfaces
(c) This layer is out of the firing range of has the highest albedo? [CDS-2022(I)]
antiaircraft guns. (a) A snow covered mountain
(d) The absence of clouds and other weather (b) A barren rocky piedmont surface
phenomena. (c) A sandy desert
107. Which of the following clouds is responsible (d) A mangrove forest
for highly intense rain? 113. Which of the following statements about
[Jharkhand P.C.S. (Pre) 2003] inversion of temperature is/are correct?
(a) Cumulus [CDS-2022(II)]
(b) Cumulonimbus 1. Temperature increases with increasing
(c) Nimbostratus altitude.
(d) Cirrostratus
2. A long winter night with clear skies is an
108. The ozone layer lies in the: ideal situation.
[Chhattisgarh P.C.S. (Pre) 2011] 3. It is a short term phenomenon and is
(a) Troposphere common all over the globe except at the
(b) Tropopause poles.

91 PYQ Workbook
GENERAL GEOGRAPHY

Select the correct answer using the code given Sea, a cold and dry wind blowing down
below: from the mountain is known as
(a) 1 and 2 only [CDS 2020 (I)]
(b) 2 and 3 only (a) Mistral
(c) 1 only (b) Bora
(d) 1, 2 and 3 (c) Bise
(d) Blizzard
114. Match List I with List II and select the
correct answer using the code given below 118. In the Hadley cell thermal circulation, air
the lists: rises up and finally descends at
[CDS 2020 (I)]
List I List II
(Characteristic) (a) intertropical convergence zone
(Cloud Form) (b) doldrums
A. Cirrus 1. Storm Clouds (c) sub-tropical high-pressure cells
B. Cumulus 2. High fog “Low (d) equatorial troughs
Ceiling” 119. Timber vegetation is generally not found in
C. Nimbostratus 3. F a i r - w e a t h e r which of the following regions?
clouds [CDS 2020 (II)]
D. Stratus 4. Entirely of ice (a) Subtropical region
particles (b) Temperate region
Code : [CDS-2022(II)] (c) Alpine region
(d) Tundra region
A B C D
(a) 2 1 3 4 120. Which one of the following indicates the
(b) 2 3 1 4 Tropical Savannah climate? [CDS 2020 (II)]
(c) 4 3 1 2 (a) Aw
(d) 4 1 3 2 (b) Dfc
(c) Cwg
115. Which one of the following statements (d) Am
about the Coriolis force is not correct?
[CDS 2021 (I)] 121. Which one of the following countries does
not have Tundra vegetation?[CDS 2020 (II)]
(a) It is maximum at the Poles.
(b) It is absent at the Equator. (a) Belarus
(c) It deflects the wind to the right direction (b) USA
in the southern hemisphere. (c) Russia
(d) It deflects the wind to the right direction (d) Canada
in the northern hemisphere. 122. Which one of the following is the
most noticeable characteristic of the
116. Match List I with List II and select the
Mediterranean climate? [CDS 2019 (II)]
correct answer using the codes given below
the lists: (a) Limited geographical extent
(b) Dry summer
List I List II (c) Dry winter
A. Cyclones 1. Western Australia (d) Moderate temperature
B. Hurricanes 2. South China Sea 123. Which one of the following regions is an
C. Typhoons 3. Indian Ocean important supplier of citrus fruits?
[CDS 2019 (II)]
D. Willy-willies 4. Atlantic Ocean
(a) Equatorial region
Codes [CDS 2021 (I)] (b) Mediterranean region
A B C D (c) Desert region
(a) 3 2 4 1 (d) Sub-humid region
(b) 3 4 2 1 124. Which one of the following may be the true
(c) 1 4 2 3 characteristic of cyclones? [CDS 2019 (I)]
(d) 1 2 4 3
(a) Temperate cyclones move from West to
117. In the region of Eastern shore of Adriatic East with Westerlies whereas tropical

PYQ Workbook 92
GENERAL GEOGRAPHY

cyclones follow trade winds. lands of Africa.


(b) The front side of cyclone is known as the
128. Tsunami waves are the undersea occurrence
‘eye of cyclone’.
of earthquake exceeding 7.5 on Richter
(c) Cyclones possess a centre of high pressure
scale. Which one of the following statements
surrounded by closed isobars.
regarding this is not correct?
(d) Hurricanes are well-known tropical
cyclones which develop over mid- [CDS 2017 (I)]
latitudes. (a) It often generates strong waves.
(b) The Pacific coasts are most vulnerable to
125. Statement I: In Tundra climate, biodiversity Tsunami waves.
is comparatively less. (c) Tsunami waves are also called high-
Statement II: Tundra climate has less energy tidal waves or seismic sea waves.
reproductive warm period. (d) Tsunami is a Latin word
Codes [CDS 2018 (I)] 129. Which of the following statements with
(a) Both the statements are individually true regard to cloudburst is/are correct?
and statement II is the correct explanation [CDS 2017 (I)]
of statement I. 1. It is defined as sudden localised very
(b) Both the statements are individually heavy downpour with cloud thunder and
true, but statement II is not the correct lightning.
explanation of statement I.
(c) Statement I is true, but statement I is false. 2. It mostly occurs in the hilly areas.
(d) Statement I is false, but statement II is 3. It results into very high intensity of
true. rainfall, i.e. 250 mm-300 mm in a couple
of hours.
126. Statement I: Tides are the rise and fall of sea 4. It occurs only during daytime.
levels caused by the combined effects of the
gravitational forces exerted by the Moon, Select the correct answer using the code given
Sun and the rotation of the Earth. below
(a) 1, 2 and 3
Statement II: The Earth rotates from West to (b) 1, 3 and 4
East. (c) 2 and 3
Codes : [CDS 2018 (I)] (d) Only 2
(a) Both the statements are individually true
and statement II is the correct explanation 130. Which one of the following statements
of statement I. with regard to Jet stream, an upper-level
(b) Both the statements are individually tropospheric wave, is not correct?
true, but statement II is not the correct [CDS 2017 (I)]
explanation of statement I. (a) It is narrow band of high-velocity wind.
(c) Statement I is true, but statement I is false. (b) It follows the wave path near the
(d) Statement I is false, but statement II is tropopause at elevations of 8 km to 15
true. km.
(c) Jet streams are typically continuous over
127. Tropical evergreen rainforest biome long distances.
provides optimum environmental (d) In summer, the polar front jet achieves its
conditions for the growth of plants and maximum force.
animals. Which one among the following
statements regarding this is not correct? 131. Which of the following statements regarding
[CDS 2017 (I)] Mediterranean and Monsoon climate is/are
(a) It has heavy rainfall and high temperature correct?
throughout the year. 1. Precipitation in Mediterranean climate is
(b) This biome is also called mega-thermal in winter while in Monsoon climate it is
biome. mostly in summer.
(c) The evergreen rain forest biome extends 2. The annual range of temperature in
between 10° N and 10° S latitudes. Mediterranean climate is higher than the
(d) The maximum development of this biome Monsoon climate.
has taken place in central and Southern 3. Rainy and dry seasons are found in both
California and North-Western coastal the climates.

93 PYQ Workbook
GENERAL GEOGRAPHY

Select the correct answer using the codes C. Walnut Alpine


given below: [CDS 2016 (I)]
D. Birch Tropical evergreen
(a) Only 1
(b) 2 and 3 Codes : [CDS 2015 (II)]
(c) 1 and 3 A B C D
(d) 1, 2 and 3 (a) 4 1 2 3
132. Which one of the following is the reason (b) 4 2 1 3
due to which the wind in the Southern (c) 3 2 1 4
hemisphere is deflected towards its left? (d) 3 1 2 4
[CDS 2016 (I)] 137. Match List I with List Il and select the
(a) Difference in the water masses of correct answer using the codes given below
Northern and Southern hemisphere. the lists,
(b) Temperature and pressure variations. List I List II
(c) Inclined axis of the Earth.
(d) Rotation of the Earth. (Climate) (Characteristic)
A. Mediterranean 1. Temperature cycle
133. The ‘eye’ of the cyclone has [CDS 2016 (I)]
is moderated by
(a) abnormally high temperature and lowest marine influence.
pressure.
(b) abnormally low temperature and pressure B. Marine West 2. Warm summers
(c) clear sky and lowest temperature. Coast and cold winters
(d) dense cloud cover and low pressure. with 3 months
below freezing
134. Which one of the following is the pattern of very large annual
circulation around a low-pressure area in temperature range.
the Northern hemisphere? [CDS 2015 (II)]
C. Dry Mid- 3. S t r o n g
(a) Counter-clockwise and away from the Latitude temperature cycle
centre. with large annual
(b) Clockwise and away from the centre. range. Warm
(c) Counter-clockwise and towards the summers to hot
centre. and cold winters to
(d) Clockwise and towards the centre. very cold.
135. Which one of the following statement is D. Moist 4. T e m p e r a t u r e
correct? [CDS 2015 (II)] Continental range is moderate
(a) Cold fronts move at slower rate than warm with warm to hot
fronts and therefore, cannot overtake the summers and mild
warm fronts. winters.
(b) Cold fronts normally move faster than Codes : [CDS 2015 (II)]
warm fronts and therefore, frequently A B C D
overtake the warm fronts.
(c) Cold fronts move at slower rate and (a) 2 1 3 4
eventually, they are overtaken by the (b) 2 3 1 4
warm fronts. (c) 4 3 1 2
(d) Cold fronts move faster than warm fronts, (d) 4 1 3 2
but they cannot overtake the warm fronts. 138. Match List I with list II and select the
136. Match List I with List II and and select the correct answer using the codes given below
correct answer using the codes given below the lists:
the lists. List I (Type of List II (Country)
List I List II Grass)
(Plant) (Natural Vegetation A. Lianos 1. Australia
Type) B. Prairies 2. Venezuela
A. Ebony Moist deciduous C. Pampas 3. USA
B. Shisham Himalayan moist D. Downs 4. Argentina

PYQ Workbook 94
GENERAL GEOGRAPHY

Codes [CDS 2015 (I)] in a water body caused by displacement of a


A B C D large volume of water, generally an ocean or
(a) 1 4 3 2 a large lake.
(b) 1 3 4 2 Statement II: A tsunami can be generated
(c) 2 4 3 1 when thrust faults associated with convergent
(d) 2 3 4 1 or destructive plate boundaries move
abruptly.
139. The following item consist of two
statements, Statement I and Statement II. Codes: [CDS 2014 (II)]
You have to examine these two statements (a) Both the statements are individually true
carefully and select the answers from these and statement II is the correct explanation
items using the codes given below: of statement I.
(b) Both the statements are individually
Statement I: Tides are the rise and fall of sea
true, but statement II is not the correct
levels caused by the combined effects of the
explanation of statement I.
gravitational forces exerted by the Moon and
(c) Statement I is true, but statement II is
the Sun and the rotation of the Earth.
false.
Statement II: The Earth rotates from the (d) Statement I is false, but statement II is
West towards the East once in 24 hours with true.
respect to the Sun.
Codes : [CDS 2015 (I)] 143. Statement I: The Atacama is the driest among
the deserts of the World.
(a) Both the statements are individually true
and statement II is the correct explanation Statement II: The aridity of the Atacama
of statement I. is explained by its location between two
(b) Both the statements are individually mountain chains of sufficient height to
true, but statement II is not the correct prevent moisture advection from either the
explanation of statement I. Pacific or the Atlantic Ocean.
(c) Statement I is true, but statement II is Codes: [CDS 2014 (II)]
false. (a) Both the statements are individually true
(d) Statement I is false, but statement II is and statement II is the correct explanation
true. of statement I.
140. Collision-Coalescence process of (b) Both the statements are individually
precipitation is applicable to true, but statement II is not the correct
explanation of statement I.
[CDS 2014 (II)] (c) Statement I is true, but statement II is
(a) clouds which extend beyond freezing false.
level. (d) Statement I is false, but statement II is
(b) those clouds which do not extend beyond true.
the freezing level.
(c) all types of clouds. 144. The vegetation type characterized by
(d) cumulonimbus cloud. (i) a large expanse of grassland with scattered
141. The Equatorial region has no other season trees and shrubs,
except summer. What could be the reason? (ii) lying between tropical rain forest and
1. The length of day and night is more or tropical steppes and deserts and
less equal over the year. (iii) flat-topped trees.
2. The Earth’s rotational velocity is is called: [CDS 2014 (I)]
maximum at the Equator. (a) Mid-latitude broad-leaf mixed forest
3. The Coriolis force is zero at the Equator. (b) Temperate rain forest
Select the correct answer using Dir the (c) Tropical savanna
codes given below: [CDS 2014 (II)] (d) Mid-latitude grassland
(a) Only 1 145. Which of the following statements is/are
(b) 1 and 2 correct?
(c) 2 and 3 1. The jet streams are high altitude easterly
(d) 1, 2 and 3 winds.
142. Statement I: A tsunami is a series of waves 2. El Niño is a warm ocean current.

95 PYQ Workbook
GENERAL GEOGRAPHY

3. El Niño appears along the Peru coast. (c) 1 and 3


Select the correct answer using the code given (d) 2, 3 and 4
below. (CAPF 2022) 150. In mid-latitude regions, the diurnal
(a) 1 only variation in weather is due to [CAPF 2018]
(b) 2 and 3 only (a) raising air /convection
(c) 1 and 3 only (b) sinking air/conduction
(d) 1, 2 and 3 (c) advection
146. Which of the following statements about (d) radiation
tropical rain forests are correct? 151. Which of the following statements are
1. The soils of tropical rain forests are quite appropriate to Mangrove Sites?
infertile. 1. Mangrove plants require an appropriate
2. The vegetation is evergreen, enabling mix of saline water and freshwater.
photosynthesis to take place year around. 2. Mangrove plants require mudflats to
3. They have been described as ‘deserts enable them to grow and develop.
covered by trees’. 3. Mangrove plants are found in the Inter-
4. They are most productive land-based tidal zones of sheltered coasts.
ecosystem. 4. Mangrove vegetation has been reported in
Select the correct answer using the codes all the coastal States including Andaman
given below. [CAPF 2021] and Nicobar Islands.
(a) 2 and 4 Select the correct answer using the codes
(b) 1, 3 and 4 given below. [CAPF 2018]
(c) 1, 2 and 3 (a) 1 and 2 only
(d) 1, 2, 3 and 4 (b) 2, 3 and 4 only
147. Which one of the following statements is (c) 1, 3 and 4 only
NOT correct? [CAPF 2020] (d) 1, 2, 3 and 4
(a) Mahogany is a timber species of tropical 152. The tides whose height is 20 percent more
rainforest. than normal tide are called [CAPF 2018]
(b) Chaparral vegetation is found in taiga (a) Spring tides
climate. (b) neap tides
(c) Lichens are predominantly found in (c) apogean and perigean tides
tundra region. (d) daily and semi-diurnal tides
(d) Sal is a representative species of moist
deciduous forest. 153. Which of the following statements about
tropical cyclone are correct?
148. Which one of the following does not fall 1. It originates and develops over warm
in the areas of tropical deciduous forest oceanic surfaces.
biomes? [CAPF 2019]
2. Presence of Coriolis force is necessary for
(a) The Neotropics mainly West Indies. it.
(b) Indo-Malaysian zone mainly in South
3. It occurs in the middle latitudinal region.
and South-East Asia, except equatorial
evergreen rainforest areas. 4. It develops ‘eye’ with calm and descending
(c) Eastern Africa and Northern Australia. air condition.
(d) North America mainly South-West Select the correct answer using the codes
America. given below. [CAPF 2018]
(a) 1 and 2 only
149. Doldrum are
(b) 1, 2 and 4
1. equatorial calms (c) 3 and 4 only
2. calm and light winds (d) 2, 3 and 4
3. roaring forties
154. Match List I with List II and select the
4. variable both in position and in extent
correct answer using the code given blow
Select the correct answer using the code given the Lists:
below: [CAPF 2019]
(a) 1, 2 and 4 List-I (Vegetation) List-II (Region)
(b) 2 and 3

PYQ Workbook 96
GENERAL GEOGRAPHY

A. Chaparrals North America 3. When warm air mass moves to cold air
mass.
B. Maquis Southern Europe 4. When an air mass is fully lifted above the
C. Fynbos South Africa land surface.
D. Malle scrubs Australia Select the correct answer using the code given
Codes: [CAPF 2017] below. [CAPF 2015]
A B C D (a) 1 and 2
(a) 1 2 3 4 (b) 2 and 3
(b) 1 3 2 4 (c) 3 and 4
(c) 4 3 2 1 (d) 4 only
(d) 4 2 3 1 159. Which one of the following factors is NOT
155. Which of the following statements about connected with planetary wind system?
the Equatorial region are correct? [CAPF 2015]
1. It is a mega thermal region. (a) Latitudinal variation of the atmospheric
2. It is a sclerophyll region. heating.
3. It is a region of high development. (b) Emergence of the pressure belt.
(c) Earth’s revolution around the Sun.
4. It is a region of aerial streamlets. (d) Migration of the pressure belts due to
Select the correct answer using the code given apparent path of the Sun.
below: [CAPF 2017]
(a) 1 and 4 160. The trees of tropical rainforest have buttress
(b) 1 and 2 only roots because [CAPF 2015]
(c) 2, 3 and 4 (a) they help to provide aeration to soils.
(d) 1, 2 and 3 (b) the organisms found in the buttresses
have a symbiotic relationship.
156. Which one of the following is not a (c) the trees belong to gramineae family.
favourable condition for occurrence of a (d) the buttresses have to bear the mechanical
thunderstorm? [CAPF 2017] load of hardwoods.
(a) Conditional and convective instability.
(b) Adequate supply of moisture in the lower 161. Consider the following statements relating
atmosphere. to cyclone, anti-cyclone and trade wind.
(c) Advection of cold dry air in the lower 1. The wind direction is clockwise in the
troposphere and warm moist air in the cyclone of Northern hemisphere.
upper troposphere. 2. The planetary wind that blows from
(d) A synoptic situation of low level North-East to North-West is known as
convergence and upper level divergence. North-East trade wind.
3. The wind direction is anti-clockwise in
157. Consider the following statements: the anti-cyclone of Southern hemisphere.
1. Areas where rainfall exceeds 250 cm. 4. Both westerlies and trade winds originate
2. Annual temperature 25-27o C. from sub-tropical highs.
3. Average humidity exceeds 75%. Which of the statements given above is/are
4. Trees do not shed the leaves. correct? [CAPF 2015]
To which one of the following types of (a) 3 and 4 only
vegetation does the above represent? (b) 1, 3 and 4
[CAPF 2017] (c) 4 only
(a) Tropical wet evergreen (d) 1, 2 and 3
(b) Tropical semi-evergreen 162. The following item consists of two
(c) Tropical moist deciduous Statements I and II. You are to examine
(d) Tropical dry evergreen these two statements carefully and select
158. Under which of the following condition(s) the answer to these items using the code
are the occluded fronts created? given below.
1. When the front remains stationary. Statements:
2. When cold air mass moves to warm air I. The Doldrums is a low-pressure area
mass. around the Equator where the prevailing

97 PYQ Workbook
GENERAL GEOGRAPHY

winds are calm. 165. The inner most region of a hurricane is


II. Low pressure is caused by the heat at the known as hurricane-eye where
Equator which makes the air rise and [CAPF 2014]
travel both Northwards and Southwards. (a) temperature is highest and pressure is the
Codes: [CAPF 2014] lowest.
(a) Both the statements are individually true (b) temperature is lowest and pressure is the
and Statement II is the correct explanation lowest.
of Statement I. (c) sky is clear and temperature is the lowest.
(b) Both the statements are individually (d) sky is under dense cloud cover and
true but Statement II is not the correct pressure is the lowest.
explanation of Statement I. 166. The Saragasso Sea is a vast patch of Atlantic
(c) Statement I is true and Statement II is Ocean characterized by an abundance of
false. floating brown seaweed called ‘Sargassum’.
(d) Statement I is false but Statement II is This is formed due to the deposit of marine
true. plants and refuse carried by the
163. The following item consists of two 1. North Atlantic current.
Statements I and II. You are to examine 2. Canary current.
these two statements carefully and select 3. North Atlantic Equatorial current.
the answer to these items using the code Select the correct answer using the codes
given below. given below: [CAPF 2014]
Statements I: Roaring Forties are strong Codes
Westerly winds found in the Southern
(a) 1 only
Hemisphere, generally between the latitudes
(b) 2 and 3 only
of 40° and 50°.
(c) 1 and 3 only
Statements II: The strong East to West air (d) 1, 2 and 3
currents are caused by the combination of air
being displaced from the Equator towards the 167. Capillaries are most effective in–
South Pole and the Earth’s rotation and there [53rd to 55th B.P.S.C. (Pre) 2011]
are few landmasses to serve as wind breaks. (a) Clayey soil
Codes: [CAPF 2014] (b) Silt soil
(a) Both the statements are individually true (c) Sandy soil
and Statement II is the correct explanation (d) Loamy soil
of Statement I. 168. Match list-I with list-II and select the
(b) Both the statements are individually correct answer using the codes given below
true but Statement II is not the correct the lists:
explanation of Statement I.
(c) Statement I is true and Statement II is List-I List-II
false. A. The hottest 1. Chile
(d) Statement I is false but Statement II is B. The coldest 2. Cherrapunji
true.
C. The wettest 3. Antarctica
164. The Kuroshio is a North flowing Ocean
current on the West side of the North Pacific D. The driest 4. Sahara
Ocean. Which one among the following Code: [48th to 52nd B.P.S.C. (Pre) 2008]
statements regarding this is not correct? A B C D
[CAPF 2014] (a) 1 2 3 4
(a) It is similar to the Gulf Stream in the (b) 4 3 2 1
North Atlantic ocean. (c) 2 3 1 4
(b) It transports warm, tropical water (d) 3 2 4 1
northward towards the Polar region. 169. Dense-forests on the earth are mostly
(c) The warm water of the Kuroshio current found– [41st B.P.S.C. (Pre) 1996]
sustains the coral reefs of Japan. (a) Nearby Equator
(d) It begins off the coast of Japan and flows (b) Nearby tropic of Cancer
Northward. (c) Nearby tropic of Capricorn

PYQ Workbook 98
GENERAL GEOGRAPHY

(d) Nearby Poles (c) Monsoon Zone - Heavy rain throughout


170. Clear sky nights are cooler than cloudy sky the year.
nights due to- [39th B.P.S.C. (Pre) 1994] (d) Desert zone - Winter rain.
(a) condensation 172. The formal development of Terra Rossa
(b) radiation
takes place in that part of land which
(c) induction
(d) conduction consists of-
[39th B.P.S.C. (Pre) 1994]
171. Which of the following pair matchings is
correct? [39th B.P.S.C. (Pre) 1994] (a) Limestone
(a) Mediterranean Sea Zone - Summer rain. (b) Cynite
(b) Equatorial Zone - Rain with thunder in (c) Granite
the noon. (d) Sandstone

99 PYQ Workbook
GENERAL GEOGRAPHY

SOLUTIONS

4.1. UPSC CSE Previous Years’ Questions


1. Solution: (b)
Exp) Option b is the correct answer.
Option 1 is incorrect: Jackfruit, (Artocarpus heterophyllus)
is a tropical evergreen tree (family Moraceae) native to
tropical Asia and widely grown throughout the wetland
tropics for its large fruits and durable wood. The greenish
unripe fruit is cooked as a vegetable, and the brown ripened
fruit is eaten fresh for the sweetly acid but insipid pulp
surrounding the seeds.
Option 2 and 3 are correct: The Moist deciduous forests
are more pronounced in the regions which record rainfall
between 100-200 cm. These forests are found in the
northeastern states along the foothills of Himalayas, eastern Statement 2 is correct. Similarly, the Westerlies play an
important role in carrying the warm, equatorial waters and
slopes of the Western Ghats and Odisha. Teak, sal, shisham,
winds to the western coasts of continents (that is eastern
hurra, mahua, amla, semul, kusum, and sandalwood etc. are section of the Oceans in temperate zone). Thus, in temperate
the main species of these forests. zones, westerlies make the eastern section of the ocean
warmer than the western sections.
2. Solution: (d)
4. Solution: (d)
Exp) Option d is the correct answer.
Exp) Option d is the correct answer.
Statement 1 is incorrect: Tropical rain forest soil is very
In tropical rainforest biome, hot, moist atmosphere and all
poor in nutrient which are required by plants to grow. This
the dead plant material create the perfect conditions in which
is due to acidic nature of soil and incessant rains. Potassium, bacteria and other microorganisms can thrive. This leads
calcium, magnesium and phospohorous are lost away due to to rapid decomposition of leaf litter material. The tropical
leaching. Thus regeneration is very slow. rainforest is known for its dense canopies of vegetation that
form three different layers. The top layer or canopy contains
Statement 2 is correct: Tropical rainforests typically
giant trees that grow to heights of 75 m (about 250 ft) or
experience warm and consistent temperatures throughout more. Thick, woody vines are also found in the canopy.
the year. These elevated temperatures enhance the activity They climb trees in the canopy to reach for sunlight.
of soil microorganisms responsible for decomposing
5. Solution: (c)
organic matter. Increased microbial activity leads to
Exp) Option c is the correct answer.
faster decomposition rates. Rainforests receive abundant
Statement 1 is incorrect. Burrowing animals and termites
rainfall, creating high levels of soil moisture. Moisture is
have not checked the extensive forest cover in savannah.
crucial for microbial activity and the breakdown of organic
A burrow is a tunnel or hole that an animal digs for
matter. It facilitates the enzymatic processes involved habitation (a place to live) or as a temporary refuge (a place
in decomposition, providing a suitable environment for of protection). Burrows provide shelter from predators and
decomposers like bacteria and fungi. extreme temperatures. For these reasons, animals have used
burrowing behavior for a very long time.
3. Solution: (c) In savannah, an unusually large proportion of dead organic
Exp) Option c is the correct answer. matter—approximately 30 percent—is decomposed through
the feeding activities of termites. In savannas, soil fertility
Statement 1 is correct. Warmer water is transported
can be markedly improved by mechanically breaking up
westward in the ocean by the Northeast trade winds in the termite mounds and spreading the material across the soil
Northern hemisphere of the tropical zone (as the figure given surface.
below shows). So, in tropical zones, the western section of Statement 2 is correct. Fire has a large effect on structure
ocean is warmer than eastern sections due to trade winds. in the densely wooded granitic landscapes in Savannah.

PYQ Workbook 100


GENERAL GEOGRAPHY

Although proportionally, the woody vegetation was more Statement 1 is incorrect: Low, thick clouds primarily
reduced in the drier than in the wetter landscapes. Frequent reflect solar radiation and cool the surface of the Earth.
fires reduced woody vegetation and dry-season fires have Statement 2 is incorrect: High, thin clouds primarily
reduced woody vegetation more than wet-season fires. transmit incoming solar radiation; at the same time, they
Statement 3 is correct. Grazing herbivores like giraffe and trap some of the outgoing infrared radiation emitted by the
elephant rely on foliage or fruit from the thorny trees. Thus, Earth and radiate it back downward, thereby warming the
their population has impact on tree growth. For example, surface of the Earth.
a steady rise in the elephant population between 1934 and
1959 in Virunga National Park, Congo (Kinshasa), led to an 8. Solution: (b)
increase in the destruction of woody plants and transformed Exp) Option b is the correct answer.
a heavily wooded savanna into a grass savanna with very few Dew is small drops of water that form on the ground and
trees.
other surfaces outdoors during the night.
Statement 4 is correct. Seasonal rainfall has affected the
Dewdrops are formed when the moisture is deposited in the
extensive forest areas of savannah. Savannah experience
form of water droplets on cooler surfaces of solid objects
frequent dry periods. The dry season is typically longer
such as stones, grass blades and plant leaves. The ideal
than the wet season, but it varies considerably, from 2 to 11
conditions for its formation are clear sky, calm air, high
months.
relative humidity, and cold and long nights. When the sky
Statement 5 is incorrect. The Savannah consists of very is clear and the trees and plants are cooler at nights, there is
deep, moderately well drained, moderately slowly permeable
more evaporation of water and hence more dew formation.
soils on uplands and terraces in the Southern Coastal Plain.
For the formation of dew, it is necessary that the dew
They formed in loamy marine or fluvial terrace deposits.
point is above the freezing point. The air containing
They are beneficial for trees and do not check their growth.
moisture to its full capacity at a given temperature is said
The forested areas are dominated by shortleaf, longleaf,
to be saturated. The temperature at which saturation
slash, and loblolly pines; white, post, and red oaks; hickory,
occurs in a given sample of air is known as dew point.
and sweetgum.
Dew forms when the temperature becomes equal to the
6. Solution: (c) dewpoint. This often happens first at ground level for two
Exp) Option c is the correct answer. reasons. First, longwave emission causes the earth’s surface
to cool at night. Condensation requires the temperature
Statement 1 is incorrect. Jet Streams occur in both the
to decrease to the dewpoint. Second, the soil is often the
hemisphere. Jet streams are high altitude westerly wind
system blows at a height of 6 to 14 km, with very high speed moisture source for the dew. Warm and moist soils will help
up to 450 km/hr in wavy form in both hemispheres. with the formation of dew as the soil cools overnight. Cloudy
skies reflect back the Earth’s radiation and that prevents
At most times in the Northern and Southern Hemispheres,
earth’s surface to cool at night.
there are two jet streams: a subtropical jet stream centered
at about 30 degrees latitude and a polar-front jet stream 9. Solution: (b)
whose position varies with the boundary between polar and
Exp) Option b is the correct answer.
temperate air.
The South Atlantic and South-Eastern Pacific are as void of
Statement 2 is correct. Only tropical cyclones have eye.
cyclonic activity largely due to the ITCZ having a tendency
The eye is a region of mostly calm weather at the centre of
to stay near or north of the equator.
tropical cyclones. A mature tropical cyclone is characterised
by the strong spirally circulating wind around the centre, For a tropical cyclone to occur in South Atlantic and South-
called the eye. The diameter of the circulating system can Eastern Pacific regions, a broad convergence zone is needed
vary between 150 and 250 km. to reach about 5 degrees of latitude away from the equator
Statement 3 is incorrect. The eye experiences the higher in order for the Coriolis Force to have sufficient intensity
temperature and lower pressure as compared to the to organize a full-fledged tropical cyclone, and the Atlantic
surroundings. The eye is a region of calm with subsiding ITCZ almost never shifts that far south.
air. Around the eye is the eye wall, where there is a strong 10. Solution: (b)
spiralling ascent of air to greater height reaching the
Exp) Option b is the correct answer.
tropopause. The wind reaches maximum velocity in this
region, reaching as high as 250 km per hour. Torrential rain There are two equatorial current flowing from east to west
occurs here. - the North EC and South EC. In between the two, there
flows a equatorial counter current in the opposite direction,
7. Solution: (d) i.e from west to east. Most of the water from North EC and
Exp) Option d is the correct answer. South EC travels polewards, however some of the water gets
Whether a given cloud will heat or cool the surface depends deflected towards equators leading to piling up of waters in
on several factors, including the cloud’s altitude, its size, and the area near Brazil, due to convergence of the two equatorial
the make-up of the particles that form the cloud. currents give rise to the equatorial counter current.

101 PYQ Workbook


GENERAL GEOGRAPHY

Of freshwater, 69% resides in glaciers, 30% exists as


groundwater, and less than 1% is located in lakes, rivers,
and swamps.

15. Solution: (d)


Exp) Option d is the correct answer.
Following are leaf modifications of desert plant to inhibit
water loss for adaptations and survival in deserts:
Fig. Flow of Equatorial Counter Current Statement 1 is correct: Wax coatings on leaves prevent water
loss through evaporation, which in the hot desert can cause
11. Solution: (b) loss of water from both the surface and the inside of leaves.
Exp) Option b is the correct answer. Leaves are also smaller on desert plants, further reducing
Statement 1 is incorrect. The Westerlies are prevailing the possibility for water loss.
winds from the west toward the east in the middle latitudes Statement 2 is correct: Deciduous plants in desert
between 30°N and 60°N, and 30°S and 60° S. They originate ecosystems have adapted through the activity of their leaves.
from the high-pressure areas in the horse latitudes and tend Leaves on these plants are typically smaller and coated with
towards the poles and steer extra tropical cyclones in this wax to prevent evaporation.
general manner. Plants such as aloes are equipped with fleshy leaves that
Statement 2 is correct. Western disturbances which contain much of their water supply. Because of their moist
originate over the Mediterranean Sea are steered towards inner bodies, these plants are called succulents. They
North western India by the Westerly Jet Stream causing typically feel spongy and when cut open are filled with a
winter rains. pulpy flesh, protected by a waxy outer layer.
12. Solution: (b) Statement 3 is correct: Many plants in the desert conserve
water by not having any leaves at all. Cacti are the most
Exp) Option b is the correct answer.
prolific of this plant type. Many cacti have spines in place of
A tropical rainforest climate or equatorial climate is a
leaves, which conduct photosynthesis and catch dew when
tropical climate usually found within 10 to 15 degrees
the climate is right. These small structures also reflect light,
latitude of the equator. They experience high mean
further reducing water loss. During heavy rains, cacti will
annual temperatures and small temperature ranges. Annual
grow temporary root systems and absorb water. They will
rainfall is high, amounting to more than 2000 mm that falls
then shed the roots when the ground has dried.
throughout the year, there is almost absence of any seasonal
variation. A tropical rainforest climate is typically hot, very 16. Solution: (a)
humid, and wet. Due to high temperature and high humidity, Exp) Option a is the correct answer.
and convergence of trade winds which uplift the air, there is
The annual range of temperature is defined as the difference
rainfall throughout the year.
between the hottest and coldest months at a place, taking
13. Solution: (d) monthly mean temperatures in each case. It is given
Exp) Option d is the correct answer. approximately by the difference between the average of the
January maximum and minimum temperatures, and the
Throughout the year, different parts of Earth receive the
corresponding average for July.
Sun’s most direct rays. So, when the North Pole tilts toward
the Sun, it’s summer in the Northern Hemisphere and the Statement 1 is correct. The continents get heated faster and
days are longer in the Northern Hemisphere. Similarly, get cooled faster in comparison to the Oceans. The annual
when the South Pole tilts toward the Sun, it’s winter in the range of temperature is high in the interior of the continent
Northern Hemisphere and nights are longer in the Northern because places located in the interior of the continent
Hemisphere. Thus, Revolution of Earth on it’s tilted axis are far away from the moderating influence of the sea.
causes variations in the length of daytime and night time Statements 2 is incorrect. Altitude affects the daily
from season to season. range of temperature and annual mean temperature but
If the Earth was not tilted on its axis, it would be permanently has negligible impact on annual range of temperature.
hot at the equator and cold at the poles. Also, the precipitation Latitude affects the annual range of temperature. The annual
patterns would remain the same throughout the year. range of temperature increases with increasing latitude.
Statement 3 is incorrect. Wind is generally stronger near
14. Solution: (c) the coasts compared to interior areas of continents.
Exp) Option c is the correct answer. Statement 4 is incorrect. Rainfall in the interiors of the
The distribution of water on the Earth’s surface is extremely Continents is generally low as compared to Coasts. It is
uneven. Only 3% of water is fresh and the remaining 97% because the rain bearing winds from seas and oceans tend to
resides in the ocean. loss most of the moisture till they reach the interiors.

PYQ Workbook 102


GENERAL GEOGRAPHY

The continents get heated faster and get cooled faster in of the cloud. Since opposites attract, that causes a positive
comparison to the Oceans. The annual range of temperature charge to build up on the ground beneath the cloud.
is high in the interior of the continent because places The grounds electrical charge concentrates around anything
located in the interior of the continent are far away from the that sticks up, such as mountains, people, or single trees.
moderating influence of the sea. Once the negative charge at the bottom of the cloud gets
17. Solution: (d) large enough, a flow of negative charge called a stepped
leader rushes toward the Earth. The positive charges at the
Exp) Option d is the correct answer.
ground are attracted to the stepped leader, so positive charge
Statement 1 is correct. Equatorial forests are found within flows upward from the ground. When the stepped leader and
5 degrees north and south of the equator. Primary tropical the positive charge meet, a strong electric current carries
rainforest is vertically divided into at least five layers: the positive charge up into the cloud. This electric current is
overstory, the canopy, the understory, the shrub layer, and known as the return stroke. We see it as the bright flash of a
the forest floor. The canopy is the dense ceiling of closely lightning bolt.
spaced trees and their branches. It is the primary layer of
the forest and resembles a roof of the forest. 20. Solution: (d)
Statement 2 is correct. Tropical rainforests support the Exp) Option d is the correct answer.
greatest diversity of living organisms on Earth. Although The sturdy root system of mangrove trees help form a
they cover less than 2 percent of Earth’s surface, rainforests natural barrier against disasters such as tsunamis and
house more than 50 percent of all plants and animals found floods. River and land sediment is trapped by the roots,
on land. The primary reason behind this abundance of which stabilizes shoreline, coastline areas and slows erosion.
diversity is the high amount of sunlight this region receives.
21. Solution: (c)
Statement 3 is correct. An important characteristic of the
canopy system is the presence of plants known as epiphytes, Exp) Option c is the correct answer.
that grow on canopy trees. Epiphytes are not parasitic because Statement 1 is correct. The stratosphere is very dry, air
they draw no nutrients away from the host, but climb the in lower stratosphere contains little water vapor and there
host tree to access direct sunlight. is almost complete absence of thunderstorms. Less water
vapour means less drag which helps maintain fuel efficiency.
18. Solution: (b)
Statement 2 is correct. Unlike the troposphere, there is
Exp) Option b is the correct answer.
almost complete absence of vertical winds or thunderstorms
Central Asian Steppe are dry, grassy plain. Steppes are in the lower stratosphere.
located in the heart of the continents, meant they have
little maritime influence. Their climate is thus continental 22. Solution: (a)
with extremes of temperature. With recorded temperatures Exp) Option a is the correct answer.
as high as 45 °C in summer and in winter falls to −55 °C. Statement 1 is correct. The sub-tropical high-pressure
Trees are very scarce in the steppes, because of the scanty belts are adiabatic in nature. Hence, they bring little rainfall
rainfall (25-50 cm annually). to the African and Eurasian desert region. Majority of
The lands do not allow people to settle permanently and deserts are located in the subtropics due to the warming and
the thinly scattered population live in small nomadic drying effect of the subsidence generated by high pressure.
groups, herding cattle, goats, horses, camels, and sheep. The Sahara Desert and Arabian Deserts are good examples
These grasslands were dominate by nomadic and semi- of subtropical deserts that largely owe their existence to this
nomadic peoples like the kirghiz. climatological feature.
19. Solution: (d) The air converges and rise at the equator due to intense
heating, and circulate to the edge of the subtropics where it
Exp) Option d is the correct answer.
descends again. This is called the Hadley Cell, and it creates
All the statements are incorrect. the subtropical high-pressure belt, where high pressure and
Thunder is caused by the rapid expansion of the air descending air are found for much of the year.
surrounding the path of a lightning bolt. It is caused by Statement 2 is incorrect. Warm ocean currents aren’t a
intense convection on moist hot days. A thunderstorm is a necessary condition for the desert belt. For instance, Gobi
well-grown cumulonimbus cloud producing thunder and cold desert doesn’t come under the influence of any warm
lightning. currents. Moreover, African and Eurasian desert belts lie
Many small bits of ice or water droplets(frozen raindrops) under the influence of cold ocean currents.
bump into each other as they move around in the air. All of
those collisions create an electric charge. After a while, the 23. Solution: (d)
whole cloud fills up with electrical charges. Exp) Option d is the correct answer.
The positive charges or protons form at the top of the cloud Statement 1 is incorrect. La Nina is characterised by
and the negative charges or electrons form at the bottom unusually cold ocean temperatures in the Equatorial Pacific

103 PYQ Workbook


GENERAL GEOGRAPHY

compared to El Nino Whereas El Nino is characterized low pressure and hence, less heat can be absorbed. Also, as
by unusually warm ocean temperatures in the Equatorial the warm air rises it cools down adiabatically. \
Pacific.
Statement 2 is incorrect. La Nina is favourable to Indian 28. Solution: (d)
monsoon whereas, El Nino has adverse effect on south-west Exp) Option d is the correct answer.
monsoon.
The tropical savanna climate has alternating dry and wet
24. Solution: (a) seasons. The wet summer season lasts 6 to 8 months and
Exp) Option a is the correct answer. during these days, there is plenty of rainfall. Winter lasts for
Statement 1 is correct. Land comprises only 19.1% of 4 to 6 months and there might be no rains in winter this
Southern Hemisphere, In Northern Hemisphere, the results in frequent forest fires.
majority of area is composed of land masses. Less landmass
Option a is incorrect. Rainfall throughout the year is a
in Southern Hemisphere results in lesser frictional drag
in motion of westerlies, thus resulting in stronger and characteristic of Equatorial region.
persistent Westerlies. Option b is incorrect. The Mediterranean region falls under
Statement 2 is incorrect. Coriolis force is equal in both the the influence of wet westerlies during winter season and
hemispheres at their respective latitudes. The magnitude of receive rainfall in winters only.
the Coriolis force depends on the speed of the object and
Option c is incorrect. Tropical Savannah region experience
its latitude. The Coriolis force is zero at the equator and
increases toward the poles. dry conditions for a longer duration than wet conditions.
Extremely short dry season is a characteristic of regions
25. Solution: (c)
experiencing Tropical Monsoon climate.
Exp) Option c is the correct answer.
The stratosphere is found above the tropopause and extends 29. Solution: (a)
up to a height of 50 km. One important feature of the Exp) Option a is the correct answer.
stratosphere is that it contains the ozone layer.
The Mediterranean region has a warm and dry climate,
Statement 1 is correct. The stratosphere is very dry and air
with mild and wet winters, and evergreen oak trees, such
there contains little water vapor and there is almost complete
absence of clouds and thunderstorms. Less water vapour and as the holm oak (Quercus ilex). This type of climate is also
absence of clouds means less drag which helps maintain fuel known as the Mediterranean climate or the dry summer
efficiency. subtropical climate. It is characterized by hot, dry summers
Statement 2 is correct. Unlike the troposphere, there is and mild, rainy winters, with a mean annual temperature of
almost complete absence of vertical winds or thunderstorms 15–20 °C (59–68 °F). The evergreen oak trees are adapted
in the lower stratosphere. to this climate by having thick, leathery leaves that reduce
26. Solution: (c) water loss and protect them from fire. The Mediterranean
Exp) Option c is the correct answer. region covers parts of southern Europe, northern Africa,
Monsoon refers to the seasonal change in the direction of the and southwestern Asia, and is one of the world’s biodiversity
prevailing winds of a region. Monsoons cause wet and dry hotspots.
seasons throughout much of the tropics. Monsoons are most
often associated with the Indian Ocean. Monsoons always 30. Solution: (c)
blow from cold to warm regions. The summer monsoon and Exp) Option c is the correct answer.
the winter monsoon determine the climate for most of India
Fresh snow has a high albedo, which is the percentage of
and Southeast Asia.
sunlight that is reflected by a surface. Albedo values range
27. Solution: (c) from 0 to 1, with a value of 1 indicating that all sunlight
Exp) Option c is the correct answer. is reflected and a value of 0 indicating that all sunlight is
Statement 1 is correct. The atmosphere is generally heated absorbed. Fresh snow has an albedo of 0.85-0.90, meaning
from the terrestrial radiation emitted by the Earth’s surface that it reflects 85-90% of sunlight. This is because snow
as gases in the atmosphere do not absorb the incoming
is made up of small crystals of ice, which are very good at
solar radiation. Therefore, temperature is maximum near
reflecting light.
the surface of the Earth.
Statement 2 is incorrect. As the air rises upwards it cools Sand deserts have an albedo of 0.25-0.35, paddy cropland
down, as result of this its water holding capacity decreases. has an albedo of 0.15-0.25, and prairie land has an albedo
Statement 3 is correct. Due to decreased atmospheric of 0.10-0.20. This means that they reflect less sunlight than
pressure the density of air decreases as we go upward due to fresh snow.

PYQ Workbook 104


GENERAL GEOGRAPHY

Important Tips latitude. They are so named because they were once a
common place for sailing ships to become becalmed, or
Prairies:
stuck, for weeks or even months. This was because the high
• Prairies are temperate grasslands that are characterized pressure in the Horse Latitudes creates light winds and calm
by tall grasses, few trees, and a relatively dry climate. seas.
They are found in North America, South America,
Asia, and Australia. Important Tips
• Prairies are home to a variety of plants and animals, Horse Latitudes:
including bison, prairie dogs, coyotes, and wolves. The Horse Latitudes are caused by the descending branch
• Tall grasses: Prairies are dominated by tall grasses, of the Hadley circulation. The Hadley circulation is a
such as big bluestem and Indian grass. These grasses global atmospheric circulation pattern that is driven by
can grow up to 6 feet tall. the uneven heating of the Earth’s surface.
• Few trees: Prairies have few trees, because the dry The sun heats the tropics more than the poles, which creates
climate and poor soil conditions make it difficult for a temperature gradient. The warm air at the equator rises,
trees to grow. cools, and condenses, forming clouds and precipitation.
• Relatively dry climate: Prairies receive between 10 and This process releases heat into the atmosphere, which
30 inches of rainfall per year. This is enough to support helps to drive global circulation.
tall grasses, but it is not enough to support trees. The rising air at the equator eventually descends into the
subtropics. This descending air is dry and warm. As it
31. Solution: (d) descends, it warms even further. This warming compresses
Exp) Option d is the correct answer. the air, which increases its pressure. The high pressure in
Tropical forests are found near the equator. They are the Horse Latitudes prevents the warm, moist air from the
characterized by their high biodiversity and their dense tropics from moving poleward, and it also prevents the
vegetation. Scientifically referred to as Low land Equatorial cold, dry air from the poles from moving equatorward.
Evergreen Rainforests, these forests thrive in regions 34. Solution: (d)
characterized by a tropical rainforest climate, where a
Exp) Option d is the correct answer.
constant, lush environment prevails with no dry seasons,
and every month receives an average precipitation of at least Dalbergia is a genus of trees and shrubs that includes
60 mm. These forests are notably humid, with an annual over 300 species. Dalbergia species are found in tropical
rainfall approaching nearly 2,000 mm. The daily average and subtropical regions around the world. Rosewood is
temperature hovers around 28°C, consistently warm and a valuable timber that is prized for its beautiful grain and
rarely dipping below 20°C or rising above 35°C. The climate rich color. Rosewood is used to make a variety of products,
is consistently hot and humid year-round, devoid of distinct including furniture, musical instruments, and luxury
seasonal variations. goods.

32. Solution: (b) Important Tips


Exp) Option b is the correct answer. Some of the most popular rosewood species include:
Statement 1 is incorrect: The Atlantic Ocean has a • Brazilian rosewood (Dalbergia nigra)
greater annual range of temperature than the Pacific • Indian rosewood (Dalbergia sissoo)
Ocean. This is because the Atlantic Ocean is narrower and • Madagascar rosewood (Dalbergia madagascariensis)
more landlocked than the Pacific Ocean. As a result, the
• Honduras rosewood (Dalbergia stevensonii)
Atlantic Ocean is more influenced by the temperature of the
surrounding landmasses, which have a greater annual range 35. Solution: (a)
of temperature than the ocean.
Exp) Option a is the correct answer.
Statement 2 is correct: In most regions of the world, the
Assertion is correct: The amount of moisture in atmosphere
annual range of temperature (the difference between the
is related to latitude because the temperature is a function
highest and lowest temperatures in a year) is greater in the
Northern Hemisphere than in the Southern Hemisphere. of latitude and moisture holding capacity depends upon
This is primarily due to the differences in landmass temperature.
distribution and geographical factors between the two Reason is correct explanation for Assertion: The amount
hemispheres. of moisture that the atmosphere can hold is related to
temperature. Warm air can hold more moisture than cold air.
33. Solution: (a)
This is why the air in tropical regions is more humid than
Exp) Option a is the correct answer. the air in polar regions. As a result, the tropics have more
The Horse Latitudes are subtropical high-pressure belts moisture in the atmosphere than the poles. This is why there
that are located at approximately 30°–35° north and south is more rainfall in the tropics than at the poles.

105 PYQ Workbook


GENERAL GEOGRAPHY

36. Solution: (a) at the center of the cyclone is cooler and denser than the air
Exp) Option a is the correct answer. at the surrounding edges. As a result, the air at the center of
the cyclone sinks. This sinking air then spreads out at the
Assertion(A) is correct: Wind patterns are clockwise in the
surface, causing the surface winds to spiral inwards.
northern hemisphere and anti-clockwise in the southern
hemisphere. This is due to the Coriolis effect, which is a 40. Solution: (b)
result of the Earth’s rotation. The Coriolis effect deflects
Exp) Option b is the correct answer.
moving objects to the right in the northern hemisphere and
to the left in the southern hemisphere. Assertion (A) is correct. The 60°–65° latitudes in both
hemispheres have a low-pressure belt instead of a high-
Reason(R) is correct explanation for Assertion (A): The
pressure belt. This is known as the subpolar low-pressure
directions of wind patterns in the northern and the
belt.
southern hemisphere are governed by the Coriolis effect.
The Coriolis effect is the main factor that determines the Reason (R) is correct. Low-pressure areas are more stable
direction of wind patterns on Earth. The Earth’s rotation over oceans than on land. This is because water has a higher
causes the air to move at different speeds at different heat capacity than land, meaning that it takes more energy
latitudes. The air at the equator moves faster than the air at to heat water than land. As a result, water heats up and cools
the poles. This difference in speed causes the air to deflect down more slowly than land. This means that low-pressure
to the right in the northern hemisphere and to the left in the areas over oceans are more likely to persist than low-pressure
southern hemisphere. areas over land.
Reason (R) is not the correct explanation for Assertion (A).
37. Solution: (a)
The subpolar low-pressure belt is not caused by the stability
Exp) Option a is the correct answer. of low-pressure areas over oceans. Instead, it is caused by
Temperate coniferous forests, also known as boreal forests the convergence of warm, moist air from the tropics and
or taigas, cover the 25% of the world’s forest land which is cold, dry air from the poles. The convergence of these two
the highest percentage of forest area in the world. They are air masses creates a zone of rising air. As the air rises, it
found in the Northern Hemisphere (usually Alaska, Siberia, cools and condenses, forming clouds and precipitation. This
Canada, and Scandinavia), in regions with cold winters and process releases heat into the atmosphere, which helps to
short summers. Temperate coniferous forests are dominated maintain the low pressure in the subpolar low-pressure belt.
by evergreen trees such as pines, firs, and spruces.
41. Solution: (d)
38. Solution: (d)
Exp) Option d is the correct answer.
Exp) Option d is the correct answer.
Statement 1 is incorrect: Equatorial regions have a very
Assertion (A) is false- Tropical rainforests are not well- small variation in temperature and day length throughout
suited for intensive agriculture. The soil in tropical the year. There is no clear distinction of seasons in the
rainforests is nutrient-poor, and the high temperatures and equatorial regions, except for the wet and dry periods that
humidity promote rapid leaching of nutrients from the
depend on the rainfall pattern.
soil. High temperature and intense bacterial activity leads to
decrease in humus content. As a result, intensive agriculture Statement 2 is incorrect: Mediterranean region has a dry
is not possible without the chemical fertilizers. and hot summer, and a mild and wet winter. The rainfall is
mostly concentrated in the winter months, while the summer
Reason (R) is true- Tropical rainforests have very high
months are usually dry.
primary productivity. Primary productivity is the rate at
which plants produce biomass. Tropical rainforests have high Statement 3 is correct: China type climate is influenced by
primary productivity because they have warm temperatures, the monsoon winds that change direction with the seasons.
abundant sunlight, and high rainfall. These conditions The China type climate has a hot and rainy summer, and a
allow plants to grow rapidly. cold and dry winter. The rainfall is distributed throughout
the year, but it is more abundant in the summer months.
39. Solution: (a)
Statement 4 is correct: Tropical highlands have a large
Exp) Option a is the correct answer. variation in altitude, which affects the temperature
Assertion is correct: The surface winds spiral inwards upon and precipitation. The higher the altitude, the lower the
the centre of the cyclone. This is true because the air pressure temperature and the higher the precipitation. Therefore, the
at the center of a cyclone is lower than the air pressure at the tropical highlands have different climate zones depending on
surrounding edges. This difference in air pressure creates a the elevation.
pressure gradient, which causes the air to flow from the areas
of high pressure to the areas of low pressure. As the air flows 42. Solution: (c)
towards the center of the cyclone, it spirals inwards due to Exp) Option c is the correct answer.
the Coriolis effect. The phenomenon of El-Nino and Southern Oscillations,
Reason is correct explanation for Assertion: Air descends which is detected in the last decade, is associated with
at the centre of the cyclone. This is also true because the air occasional weak monsoon rains in the Indian subcontinent.

PYQ Workbook 106


GENERAL GEOGRAPHY

ENSO is associated with occasional weak monsoon rains in are generally associated with them.
the Indian subcontinent, especially during El-Nino years. Statement 1 is incorrect. The Roaring Forties only blow in
This is because El-Nino reduces the temperature contrast the Southern Hemisphere.
between the land and sea, which weakens the low-pressure
system over India that attracts moist winds from the Indian 45. Solution: (c)
Ocean. El-Nino also shifts the rainfall patterns over the Exp) Option c is the correct answer.
Pacific and Indian Oceans, resulting in less rainfall over Statement (A) is correct: Neap tides occur when the sun
India and more rainfall over Indonesia and Australia. and moon are at right angles to each other (during the first
Important Tips and third quarter moon phases). The gravitational pull of the
sun partially cancels out the gravitational pull of the moon,
• El-Nino refers to the abnormal warming of the sea
resulting in lower high tides and higher low tides than usual.
surface temperature in the eastern and central Pacific,
while La Nina refers to the abnormal cooling of the Reason (R) is false: Neap tides actually occur during the
same region. first and third quarters of the moon when it appears “half
full,” not during the new moon. During the new moon,
• Southern Oscillations refer to the changes in the
spring tides occur when the gravitational forces of the sun
atmospheric pressure between the eastern and western
and moon combine to create higher high tides and lower low
Pacific.
tides.
• ENSO can alter the strength and direction of the
trade winds, which affect the moisture transport and
rainfall distribution over these regions. ENSO can also
influence the position and intensity of the jet streams,
which are high-altitude winds that steer weather
systems across the continents.

43. Solution: (b)


Exp) Option b is the correct answer.
The correct answer is -
List-I List-II
A. Foehn 4. Alps
B. Simoom 2. Kurdistan
C. Santa Ana 3. California
46. Solution: (c)
D. Zonda 1. Argentina
Exp) Option c is the correct answer.
Foehn is a dry, warm wind that blows down the leeward side
of a mountain range. It is common in the Alps, but it can also Troposphere, Stratosphere, Mesosphere, Ionosphere. This
be found in other mountainous regions, such as the Rockies is the sequence of the layers of the atmosphere from the
and the Andes. surface of the Earth upwards.
Simoom is a hot, dry wind that blows in the deserts of North • The troposphere is the lowest layer, where most weather
Africa and the Middle East. It is particularly common in occurs.
Kurdistan. • The stratosphere is above the troposphere and contains
Santa Ana is a warm, dry wind that blows from the interior the ozone layer.
of California to the coast. It is most common in the fall and • The mesosphere is above the stratosphere and is where
winter months. most meteors burn up.
Zonda is a warm, dry wind that blows from the Andes • The ionosphere is above the mesosphere and is where
Mountains to the plains of Argentina. It is most common in auroras and radio waves occur.
the winter months. • The exosphere is the outermost layer, where the
atmosphere gradually fades into space.
44. Solution: (b)
Exp) Option b is the correct answer. Important Tips
Statements 2,3 and 4 are correct: The Roaring Forties are the The troposphere: This is the lowest layer of the
prevailing westerly winds that blow between 40-50 degrees atmosphere, where most weather occurs. The troposphere
south latitude. They are strong and persistent, and they can extends from the surface to about 10 to 15 km (6 to 9 mi)
reach speeds of up to 100 knots. The westerlies are caused by above sea level, depending on the latitude and season. The
the temperature gradient between the equator and the poles. temperature in the troposphere decreases with altitude,
Their direction is generally from north-west to east in the reaching the lowest point at the tropopause.
southern hemisphere. Overcast skies, rain and raw weather

107 PYQ Workbook


GENERAL GEOGRAPHY

The stratosphere: This is the second layer of the of applications, including furniture making, construction,
atmosphere, where the ozone layer is located. The and shipbuilding. Mexican cedar is also used to make
stratosphere extends from the tropopause to about 50 essential oils, which are used in aromatherapy and other
km (31 mi) above sea level. The temperature in the products. The essential oil of cedarwood has a warm, woody
stratosphere increases with altitude, due to the absorption scent that is said to be calming and relaxing.
of ultraviolet radiation by ozone. B. Douglas Fir - 2. Canada
The mesosphere: This is the third layer of the atmosphere, Douglas fir (Pseudotsuga menziesii) is a coniferous tree
where most meteors burn up. The mesosphere extends that is native to western North America, including Canada.
from the stratopause, which is the boundary between It is one of the most important timber species in Canada,
the stratosphere and the mesosphere, to about 85 km (53 and it is also used to make a variety of other products, such
mi) above sea level. The temperature in the mesosphere as paper, pulp, and plywood. Douglas fir trees can grow up
decreases with altitude, reaching the lowest point at the to 100 meters tall and live for over 1,000 years. They are
mesopause. found in a variety of climates, from coastal rainforests to
The ionosphere: This is not a distinct layer of the dry interior forests. Douglas fir trees are an important part
atmosphere, but rather a region that overlaps with parts of the Canadian ecosystem. They provide food and shelter
of the upper mesosphere and lower thermosphere. The for a variety of wildlife, and they help to regulate the climate.
ionosphere extends from about 48 km (30 mi) to 965 km C. Mahogany - 4. Honduras
(600 mi) above sea level. The ionosphere is ionized by solar Mahogany is a tropical hardwood timber that is native to
radiation, creating a layer of electrically charged particles. Central and South America. The most prized mahogany
The thermosphere: This is the fourth layer of the species is Honduran mahogany (Swietenia macrophylla),
atmosphere, where auroras occur. The thermosphere which is known for its rich color, beautiful grain, and
extends from the mesopause to about 600 km (370 mi) durability. Honduran mahogany is a large tree that can
above sea level. The temperature in the thermosphere grow up to 40 meters tall and live for over 500 years. It is
increases with altitude, reaching very high values due to found in tropical rainforests and lowland areas. Honduran
solar radiation and auroral activity. mahogany is a slow-growing tree, and it can take up to 100
years for a tree to reach maturity.
47. Solution: (b)
D. Teak - 1. Myanmar
Exp) Option b is the correct answer.
Teak is a tropical hardwood timber that is native to
Russia is the largest producer of fuelwood in the world, Southeast Asia, including Myanmar. It is one of the most
accounting for over 20% of global fuelwood production. valuable timbers in the world, prized for its durability,
Fuelwood is used for cooking, heating, and other household water resistance, and beautiful grain. Teak is used to
purposes in many rural areas of Russia. make a variety of products, including furniture, musical
48. Solution: (a) instruments, flooring, and boatbuilding.

Exp) Option a is the correct answer. 4.2. Other UPSC CSE Previous Years’
Assertion (A) is true: Areas lying within five to eight- Questions
degree latitude on either side of the equator receive
50. Solution: (d)
rainfall throughout the year. It is the region of Intertropical
Convergence Zone (ITCZ), which is a belt of low pressure Exp) Option d is the correct answer.
where the trade winds from both hemispheres meet and Assertion (A) is true: It states that rainfall occurs in the
cause rising air, condensation, and precipitation. winter season in Mediterranean climate. Mediterranean
Reason (R) is true: High temperature and high humidity climates are characterized by mild, wet winters and hot, dry
cause convectional rainfall mostly in the noon near the summers. The majority of precipitation in these regions falls
equator, as the air on getting heated becomes light and rises during the winter months.
in convection currents, forming cumulus clouds and heavy Reason (R) is true: It states that in summer these regions
rainfall. remain under the influence of dry terrestrial winds. During
the summer, Mediterranean regions are often influenced by
49. Solution: (b) subtropical high-pressure systems that bring dry, warm air
Exp) Option b is the correct answer. from over the continents.
A. Cedar - 3. Mexico While both statements are true, the reason (R) doesn’t
Cedar is a type of coniferous tree that is found in many directly explain why rainfall occurs in the winter season in
parts of the world, including Mexico. The most common Mediterranean climates.
type of cedar in Mexico is the Mexican cedar, also known Rainfall occurs in the winter season in Mediterranean
as the cedar-of-Goa (Cupressus lusitanica). Mexican cedar climate because of the shifting of the subtropical ridge and
is a fast-growing tree that can reach heights of up to 30 the westerly winds. The subtropical ridge is a belt of high
meters. It has soft, aromatic wood that is used in a variety pressure that extends towards the poles during the summer

PYQ Workbook 108


GENERAL GEOGRAPHY

and towards the equator during the winter. This causes the Maquis, a form of scrubland vegetation, is prevalent in
prevailing winds to change direction and bring moisture regions surrounding the Mediterranean, encompassing
from the oceans to the land. The moisture cools, condenses Europe, North Africa, and Asia Minor, while in Chile, a
and forms clouds and rain. comparable type is known as Matorral. The Mediterranean
Shrublands, also referred to as the Chaparral biome, are
51. Solution: (d) present along the coastlines of the Mediterranean Sea,
Exp) Option d is the correct answer. California, Central Chile, the southwestern parts of South
The Shamal is a hot, dry wind that blows from the north Africa, and the southwestern regions of Australia.
or northwest in Iraq, Iran, and the Arabian Peninsula. It is B. Fynbos - 3. South Africa
most common in the summer months and can reach speeds Fynbos, a narrow strip of indigenous shrubland and
of up to 70 kilometers per hour (43 miles per hour). heathland vegetation, can be found in the South African
Austria is located in Central Europe and does not experience provinces of the Western Cape and Eastern Cape. This region
the Shamal wind. is primarily characterized by its coastal and mountainous
terrain, experiencing a Mediterranean climate with wet
Important Tips winters.
Local Winds: C. Chaparral - 1. California
• Leveche is a hot, dry wind that blows from the north Chaparral is a plant community consisting of shrubland and
or northeast in Spain. heathland, predominantly located in the state of California
• Brickfielder is a hot, dry wind that blows from the in the United States and the northern region of the Baja
interior of Australia to the coast. California Peninsula in Mexico.
• Black roller is a type of dust storm that occurs in D. Matorral - 4. Chile
North America. Matorral is typically defined by a moderate Mediterranean
climate, featuring wet winters and arid summers.
52. Solution: (d)
54. Solution: (c)
Exp) Option d is the correct answer.
Exp) Option c is the correct answer.
Albedo is the fraction of sunlight that is reflected by the
surface. It is a measure of how reflective a surface is. Albedo A. Epiphytes - 2. Equatorial
is important in climatology because it affects the amount The equatorial region, characterized by its hot and humid
of solar radiation that is absorbed by the Earth’s surface. climate, is situated approximately between 5 degrees and 10
Surfaces with high albedo, such as snow and ice, reflect degrees north and south of the equator. Its most extensive
more sunlight back into space, which helps to keep the Earth areas can be observed in the lowlands of the Amazon, the
cool. Surfaces with low albedo, such as asphalt and dark soil, Congo, Malaysia, and the East Indies. This region is home
absorb more sunlight, which helps to warm the Earth. to a diverse array of evergreen trees, including those that
produce valuable tropical hardwood such as mahogany,
Important Tips ebony, greenheart, and cabinet wood. Additionally, one can
Albedo: find lianas, epiphytic plants, and parasitic plants thriving
Albedo is also affected by the angle of the sun’s rays. When in this environment.
the sun is high in the sky, the sun’s rays hit the Earth’s B. Acacia - 4. Savanna
surface more directly, which results in higher albedo. The savannah terrain is distinguished by its tall grass and
When the sun is low in the sky, the sun’s rays hit the Earth’s low trees. It experiences a climate marked by clearly defined
surface at an angle, which results in lower albedo. wet and dry seasons, with an average high temperature
Here are some examples of surfaces with different albedo ranging from 24°C to 27°C throughout the year. This
values: ecosystem features various elements, including bulbous
• Fresh snow: 0.85-0.90 cacti, prickly shrubs, long-rooted wiry grasses, and
scattered dwarf acacia trees.
• Sand desert: 0.25-0.35
C. Baobab - 3. Sahara
• Paddy cropland: 0.15-0.25
The Sahara Desert stretches across extensive regions of
• Prairie land: 0.10-0.20
Algeria, Chad, Egypt, Libya, Mali, Mauritania, Morocco,
• Asphalt: 0.05-0.10 Niger, Western Sahara, Sudan, and Tunisia, encompassing
• Dark soil: 0.03-0.05 a vast area of 9 million square kilometers (3,500,000 sq mi),
which accounts for 31% of Africa’s landmass. The Baobab
53. Solution: (a) tree is commonly encountered in the arid sub-Saharan
Exp) Option a is the correct answer. African areas within the Sahara region. The local flora here
A. Maquis - 2. Mediterranean Sea Coastal Region comprises olive, cypress, and mastic trees.
(California) D. Cedars - 1. Mediterranean

109 PYQ Workbook


GENERAL GEOGRAPHY

The climate in this area is marked by warm and arid Global warming is causing the Earth’s temperature to rise,
summers, contrasting with cooler and wetter winters. It is which is leading to more extreme weather events, including
situated in the latitudes between approximately 30° and 45° cyclones, storms, and hurricanes. Global warming is making
north and south of the Equator, primarily on the western
cyclones, storms, and hurricanes more frequent and more
coastlines of continents. Cedar trees are indigenous to the
mountainous regions of the western Himalayas as well as severe. This is because warm ocean waters provide the energy
the Mediterranean area. for these weather systems to form and grow. Additionally,
global warming is causing the sea level to rise, which makes
55. Solution: (d)
storm surges more dangerous.
Exp) Option d is the correct answer.
The correct match is- Important Tips
A. Willy-Willies - 2. Australia Here are some of the ways that global warming is
B. Taifu - 3. Japan affecting cyclones, storms, and hurricanes:
C. Baguio - 1. Philippines • Increased frequency: Global warming is causing
cyclones, storms, and hurricanes to form more often.
D. Hurricanes - 4. USA
This is because warm ocean waters provide the energy
Willy-willies are tropical cyclones that occur in the North- for these weather systems to form and grow.
West region of Australia.
• Increased intensity: Global warming is also causing
Taifu is another name for a typhoon, which is a tropical
cyclones, storms, and hurricanes to become more
cyclone that occurs in the western North Pacific Ocean
intense. This is because warm ocean waters provide
including Japan.
more energy for these weather systems to develop.
Baguio is a type of tropical cyclone that occurs in the
• Increased storm surge: Global warming is causing
Philippines. It is characterized by strong winds and heavy
the sea level to rise, which makes storm surges more
rain.
dangerous. Storm surges are the walls of water that
Hurricanes are tropical cyclones that occur in the North are pushed onshore by the strong winds of a cyclone,
Atlantic Ocean and the eastern North Pacific Ocean
storm, or hurricane.
including the USA. They are characterized by strong winds,
heavy rain, and storm surges. 59. Solution: (b)
56. Solution: (d) Exp) Option b is the correct answer.
Exp) Option d is the correct answer. The Sihawal, Patpara, Baghor, and Khetaunhi formations
The Mistral is a cold, dry wind that blows from the north are all part of the Vindhyan Supergroup of rocks, which
into the Rhone Valley in France. It is most common in the
are found in central India.
winter and spring months.
The correct chronological order of these formations is:
Important Tips
Option 4: Sihawal Formation (Middle Pleistocene): The
Santa Ana winds are warm, dry winds that blow from the
interior of California to the coast. They are most common Sihawal Formation is the oldest of the four formations, and it
in the fall and winter months. is composed of sandstone, shale, and limestone. The Sihawal
Haboobs are strong dust storms that occur in the Sahara Formation is known for its fossils of stromatolites, which are
Desert and other arid regions. They can travel long microbial mats that are considered to be some of the oldest
distances and can cause significant damage. life forms on Earth.
Yamo winds are seasonal winds that blow from the Pacific Option 1: Patpara Formation (Upper Pleistocene): The
Ocean to Japan. They are strongest in the summer months. Patpara Formation is composed of sandstone, shale, and
57. Solution: (a) dolomite. The Patpara Formation is known for its fossils of
Exp) Option a is the correct answer. marine animals, such as trilobites, brachiopods, and crinoids.
Edaphic factors pertain to soil-related characteristics that Option 3: Baghor Formation (Terminal Pleistocene): The
influence the types of microorganisms and plants thriving Baghor Formation is composed of shale, sandstone, and
in a specific soil environment. These factors encompass
limestone. The Baghor Formation is known for its fossils of
soil type, texture, structure, pH level, moisture content,
salinity, temperature, organic matter, nitrogen content, land plants, such as ferns and lycopods.
and the presence of heavy metals. They play a crucial Option 2: Khetaunhi Formation (Late Holocene): The
role in shaping ecological communities within a given soil Khetaunhi Formation is the youngest of the four formations,
ecosystem.
and it is composed of sandstone, shale, and limestone. The
58. Solution: (d) Khetaunhi Formation is known for its fossils of dinosaurs,
Exp) Option d is the correct answer. such as sauropods and theropods.

PYQ Workbook 110


GENERAL GEOGRAPHY

Important Tips Mongolia, Russia, Kazakhstan, Kyrgyzstan, Tajikistan,


and Uzbekistan.
• Polar climates: These are climates that have very low
temperatures throughout the year due to the high
latitudes and low solar radiation. Some examples of
Asian regions that have polar climates are Siberia, Tibet,
Himalayas, and parts of China.

61. Solution: (b)

0. Middle Pleistocene gravels and clays of the Sihawal Exp) Option b is the correct answer.
Formation The Bora is a cold, dry wind that blows from the northeast
into the Adriatic Sea. It is most common in the winter
1. Upper Pleistocene sands and gravels of the Khuntelli
months and can reach speeds of up to 200 kilometers per hour
Formation
(124 miles per hour). The Bora is caused by the temperature
2. Lower Proterozoic metasediments gradient between the cold, dry air over the Balkans and the
3. Upper Pleistocene gravels, sands and clays of the warm, moist air over the Adriatic Sea.
Patpara Formation Poland is located in Central Europe, far from the Adriatic
4. Terminal Pleistocene sands and clays of the Baghor Sea. Therefore, the Bora is not common in Poland.
Formation
Important Tips
5. Late Holocene clays, silts and sands of the Khetaunhi
Local Winds:
formation
• The Foehn is a warm, dry wind that blows down the
6. Present-day channel sands and point-bars of the
leeward side of the Alps Mountains.
river Son
• The Mistral is a cold, dry wind that blows from the
60. Solution: (d) north into the Rhine Valley.
Exp) Option d is the correct answer. • The Khamsin is a hot, dry wind that blows from the
Asia is the only continent that has all types of climatic Sahara Desert into Egypt.
zones, according to the Köppen climate classification • Chinook winds are warm, dry winds that blow down
system. Asia has tropical, dry, temperate, continental, and the eastern slopes of the Rocky Mountains in the
polar climates, as well as some subtypes of these climates. United States.
Here are some examples of the climatic zones in Asia: • Sirocco winds are hot, dry winds that blow from the
• Tropical climates: These are climates that have a monthly Sahara Desert into Sicily.
mean temperature of 18 °C (64.4 °F) or more throughout • Norwesters are thunderstorms that occur in India
the year. They are found in regions near the equator or during the pre-monsoon season.
within 10° to 25° latitude in both hemispheres. Some
examples of Asian countries that have tropical climates 62. Solution: (c)
are India, Indonesia, Malaysia, Philippines, Sri Lanka, Exp) Option c is the correct answer.
Thailand, and Vietnam. Bhutan’s constitution contains a vital provision that mandates
• Dry climates: These are climates that have very low a minimum of 60% forest cover, ensuring the nation’s
annual precipitation, usually less than the potential unwavering commitment to environmental preservation.
evapotranspiration. Some examples of Asian countries This constitutional safeguard has allowed Bhutan not only
that have dry climates are Afghanistan, Iran, Iraq, to maintain but also increase its forest cover, currently
Kazakhstan, Mongolia, Pakistan, Saudi Arabia, standing at an impressive 70%. To complement this,
Turkmenistan, and Uzbekistan. Bhutan has implemented measures like restricting timber
exports, promoting community forestry, and declaring
• Temperate climates: These are climates that have
protected forest areas linked with wildlife corridors.
moderate temperatures and precipitation throughout the Bhutan’s unique governance approach, emphasizing Gross
year. They are found in regions that are influenced by National Happiness (GNH) over Gross Domestic Product
maritime air masses or ocean currents. Some examples of (GDP), ensures that every policy is evaluated based on
Asian countries that have temperate climates are China, good governance, sustainable socioeconomic development,
Japan, Korea, Nepal, Taiwan, and Turkey. cultural preservation, and environmental conservation.
• Continental climates: These are climates that have While facing challenges like climate change impacts and
a significant annual variation in temperature due to demographic shifts, Bhutan explores innovative solutions
the lack of moderating influence from oceans. They such as the “Bhutan for Life” project and partnerships with
are found in regions that are located in the interior of automobile manufacturers to promote sustainability and
large landmasses or at high latitudes. Some examples maintain its green cover, making it a global model for forest
of Asian countries that have continental climates are conservation and balanced growth.

111 PYQ Workbook


GENERAL GEOGRAPHY

Important Tips permeability, meaning that the small pores are not well-
connected, hindering the flow of fluids through it. Thus, low
The Gross National Happiness Index (GNH Index)
permeability explains high water holding capacity of clay
is a measure of a country’s well-being and progress,
and not high porosity.
considering both economic and non-economic factors. It
was developed in Bhutan in the 1970s as an alternative
to the Gross Domestic Product (GDP) as a measure of
national progress. The GNH Index is based on four pillars:
• Good governance.
• Sustainable socio-economic development
• Cultural preservation
• Environmental conservation
The GNH Index is a more holistic measure of well-being
than the GDP because it considers both economic and
non-economic factors. The GDP is a measure of the total
value of goods and services produced in a country in a 66. Solution: (d)
given year. It does not consider factors such as the quality
Exp) Option d is the correct answer.
of life, the environment, or social well-being.
Air pressure is highest in the winter and lowest in the
63. Solution: (a) summer. This is because warm air is less dense than cold
Exp) Option a is the correct answer. air. As a result, warm air rises and cold air sinks. The rising
warm air creates a low-pressure area, while the sinking cold
The Saffir-Simpson Hurricane Wind Scale is a 1 to 5 rating
air creates a high-pressure area.
based on a hurricane’s maximum sustained wind speed.
This scale estimates potential property damage. Hurricanes In the winter, the air is colder and denser, so it sinks and
creates a high-pressure area. In the summer, the air is warmer
reaching Category 3 and higher are considered major
and less dense, so it rises and creates a low-pressure area.
hurricanes because of their potential for significant loss of
life and damage. Important Tips
Relation Between Air Pressure and Seasons:
64. Solution: (d)
• The difference in air pressure between the summer
Exp) Option d is the correct answer.
and winter is one of the factors that drives global wind
Taiga forest/Boreal forests are characteristics of a Temperate circulation.
region. The forest holds the distinction of being the largest
• The winds blow from areas of high pressure to areas of
land biome on Earth. This expansive ecosystem, known as the
low pressure. This is why, the winds tend to blow from
Boreal Ecozone, stretches across eight countries: Canada,
the poles to the equator in the winter and from the
China, Finland, Japan, Norway, Russia, Sweden, and the
equator to the poles in the summer.
United States. Boreal forests are specifically characterized
by their presence in high-latitude environments, enduring • The difference in air pressure between the summer and
freezing temperatures for 6 to 8 months each year. Within winter also affects the weather.
these environments, the trees must attain a minimum height • Low-pressure areas are associated with clouds and
of 5 meters and form a canopy covering at least 10%. This precipitation, while high-pressure areas are associated
forest type predominantly consists of coniferous tree species with clear skies and fair weather. This is why the
like pine, spruce, and fir, with some scattered broadleaf weather is often stormier in the summer than in the
species such as poplar and birch. winter.

65. Solution: (c) 67. Solution: (c)

Exp) Option c is the correct answer. Exp) Option c is the correct answer.
The Mediterranean climate and the Monsoon climate are
Assertion (A) is correct: Clay has the highest water holding
associated with the shifting of the pressure belts.
capacity of all soil types. This is because clay particles are
very small and have a large surface area. The water molecules • Equatorial climate is not affected by the shifting of the
are attracted to the clay particles by adhesive forces and pressure belts. The equatorial climate is influenced by
the equatorial low-pressure belt, which is a permanent
capillary action.
feature of the earth’s atmosphere. The equatorial low-
Reason (R) is true and (R) is not the correct explanation pressure belt is caused by the intense heating of the air
of (A): Porosity refers to the void spaces within a material. near the equator, which makes it rise and create a zone
Clay has numerous small pore spaces between its fine of low pressure. The equatorial climate is characterized
particles, contributing to its high porosity. However, by high temperatures, high humidity, and heavy rainfall
despite its high porosity, clay is often characterized by low throughout the year.

PYQ Workbook 112


GENERAL GEOGRAPHY

• Mediterranean climate is affected by the shifting of the ozone layer, which absorbs most of the Sun’s ultraviolet
pressure belts. The Mediterranean climate is influenced radiation. The stratosphere does not reflect radio waves,
by the subtropical high-pressure belt, which is a seasonal but rather allows them to pass through with little
feature of the earth’s atmosphere. The subtropical high- attenuation.
pressure belt is caused by the sinking of the air that rises • The mesosphere is the third layer of the atmosphere,
from the equatorial low-pressure belt. The subtropical above the stratosphere. The mesosphere is where most
high-pressure belt shifts northward in summer and
meteors burn up as they enter the Earth’s atmosphere.
southward in winter, following the apparent movement
The mesosphere does not reflect radio waves, but rather
of the sun. The Mediterranean climate is characterized by
causes them to scatter or attenuate due to meteor trails
dry and hot summers and mild and wet winters.
and turbulence.
• Monsoon climates are characterized by a seasonal
reversal of winds due to the shifting of pressure belts. 70. Solution: (b)
This leads to distinct wet and dry seasons. The monsoon Exp) Option b is the correct answer.
winds blow from the sea to the land during the wet season The correct match for the given pairs is-
and from the land to the sea during the dry season.
A. Australia - 2. Willy-Willy
• Western European climate is influenced by the North
B. China - 3. Typhoons
Atlantic Drift and prevailing westerlies, but it is not
primarily associated with the shifting of pressure belts. C. India - 4. Cyclones
It brings relatively warm and moist air to the western D. USA - 1. Hurricanes
coasts of Europe, particularly countries like the UK, Willy-willies are dust devils that form in Australia. They are
Ireland, France, etc. The prevailing westerly winds also typically small and weak, but they can sometimes grow larger
play a significant role. The climate is characterized by
and stronger. Willy-willies can be dangerous, and they can
mild winters and relatively cool summers.
cause damage to property and infrastructure.
68. Solution: (c) Typhoons are tropical cyclones that form in the western
Exp) Option c is the correct answer. North Pacific Ocean particularly in China. They are similar
to hurricanes, but they are often stronger and more intense.
The ionosphere is the uppermost layer of the atmosphere,
where the air is ionized by solar radiation. The ionosphere Typhoons can also be very destructive, and they can cause
reflects radio waves back to the Earth, enabling long- significant damage to property and infrastructure.
distance radio communication. The ionosphere varies in Cyclones are tropical cyclones that form in the Indian Ocean
height and density depending on the time of day, season, and and South Pacific Ocean. They are similar to hurricanes
solar activity. and typhoons, but they are often weaker and less intense.
The aurora borealis, or northern lights, is caused by the However, cyclones can still be very destructive, and they can
interaction of energetic particles (electrons and protons) cause significant damage to property and infrastructure.
from the solar wind with atoms and molecules in the Hurricanes are tropical cyclones that form in the North
ionosphere. These particles are guided by Earth’s magnetic Atlantic Ocean and the eastern North Pacific Ocean. They
field and collide with atmospheric gases, causing them to are characterized by strong winds, heavy rain, and storm
emit light of different colors and shapes. The aurora borealis surges. Hurricanes can be very destructive, and they can
occur mostly in high latitudes, near the magnetic poles, cause significant damage to property and infrastructure.
where the magnetic field lines are more vertical and allow
more particles to enter. 71. Solution: (c)
Exp) Option c is the correct answer.
69. Solution: (d)
The Mississippi Plains, also known as Tornado Alley, is
Exp) Option d is the correct answer. a region of the central United States that is known for its
The ionosphere is the uppermost layer of the atmosphere, frequent tornadoes. It includes the states of Texas, Oklahoma,
where the air is ionized by solar radiation. The ionosphere Kansas, and Nebraska. Tornado Alley is characterized by
reflects radio waves back to the Earth, enabling long- its warm, moist air from the Gulf of Mexico and dry, cold
distance radio communication. The ionosphere varies in air from Canada. This combination of air masses creates
height and density depending on the time of day, season, and the perfect conditions for the formation of thunderstorms,
solar activity. which can produce tornadoes.
• The troposphere is the lowest layer of the atmosphere,
72. Solution: (c)
where most weather activity occurs. The troposphere
does not reflect radio waves, but rather causes them to Exp) Option c is the correct answer.
bend or diffract over the horizon. This is called ground Blizzards are rare in Chile. They are more common in colder
wave propagation. climates, such as the United States, Canada, and Europe.
• The stratosphere is the second layer of the atmosphere, Chinook winds are warm, dry winds that blow down the
above the troposphere. The stratosphere contains the eastern slopes of the Rocky Mountains in the United States.

113 PYQ Workbook


GENERAL GEOGRAPHY

Sirocco winds are hot, dry winds that blow from the Sahara • Jet streams are high altitude winds affecting the surface
Desert into Sicily. weather conditions. Jet streams are fast-moving bands of
Norwesters are thunderstorms that occur in India during the air that flow from west to east. They can affect the surface
pre-monsoon season. weather conditions by bringing storms and cold weather.

75. Solution: (b)


73. Solution: (a)
Exp) Option b is the correct answer.
Exp) Option a is the correct answer.
Assertion (A) is correct: The ports of north-west Europe
The Mediterranean regions have rainy winters because they
are open throughout the year. This is because the ports of
lie in the belt of the Westerlies, which are prevailing winds
north-west Europe, such as Rotterdam, Hamburg, Antwerp,
that blow from the west to the east in the middle latitudes.
and London, are located on the coastlines of the North Sea
The Westerlies bring moist and warm air from the oceans to and the English Channel, which are connected to the Atlantic
the land, causing precipitation. In winter, the Westerlies shift Ocean. These water bodies have a moderating influence
equatorward, making them more effective in bringing rain to on the climate of north-west Europe, preventing extreme
the Mediterranean regions. In summer, the Westerlies shift temperatures and ice formation. Therefore, the ports of
poleward, making them less effective in bringing rain to the north-west Europe do not freeze in winter and remain
Mediterranean regions.Hence both (A) and (R) are true, and accessible for shipping all year round.
(R) is correct explanation of (A). Reason (R) is correct: The south-westerly winds blow
Important Tips over north-west Europe throughout the year. This is true
because north-west Europe lies in the belt of the westerlies,
Features of Mediterranean climate:
which are prevailing winds that blow from west to east in the
• It is found between about 30° and 45° latitude north middle latitudes. The westerlies in the northern hemisphere
and south of the Equator and on the western sides of are mostly from the southwest, especially in winter when
the continents they are stronger and more persistent. The westerlies bring
• It has hot, dry summers and cool, wet winters. The moist and warm air from the Atlantic Ocean to north-west
summer months are dominated by the subtropical high Europe, creating a mild and humid climate.
pressure that brings clear skies and high temperatures,
Reason (R) is not the correct explanation of Assertion (A),
while the winter months are influenced by the
because the south-westerly winds are not the main factor that
westerlies and associated storms that bring rainfall.
keeps the ports of north-west Europe open throughout the
• It has abundant sunshine throughout the year, with 90 year. The main factor is the presence of the North Atlantic
per cent in summer and 50 to 60 per cent in winter. Current, which is a warm ocean current that flows from the
• It has a moderate annual temperature range, with Gulf of Mexico to the North Sea and the English Channel.
average annual temperatures around 10°C to 27°C The North Atlantic Current transfers heat from the tropics
depending on the region. to the higher latitudes, raising the water temperature and
preventing ice formation. The south-westerly winds help to
74. Solution: (b) maintain the North Atlantic Current by pushing it eastward,
Exp) Option b is the correct answer. but they are not essential for its existence.
The zone of maximum temperature is located slightly north 76. Solution: (a)
of the equator, at about 20 degrees north latitude. This is
Exp) Option a is the correct answer.
because the equator receives more direct sunlight than any
other latitude, but the Earth’s atmosphere and oceans absorb The correct sequence of the climatic regions while going
some of this sunlight before it reaches the surface. Also, at from Zaire to the Netherlands is equatorial, hot desert,
Mediterranean, and west European.
equator there are more clouds which reflect the sunlight. As
a result, the surface temperature at the equator is slightly Zaire, also known as the Democratic Republic of the Congo,
lower than the surface temperature at 20 degrees north is a country in Central Africa that lies on the equator. It
latitude. has a tropical climate with high temperatures and humidity
throughout the year. The climate of Zaire can be divided into
The other three statements are all true:
two main types: equatorial and tropical wet and dry.
• The presence of water vapor is highly variable in the Equatorial climate: This type of climate is found in the
lower atmosphere. Water vapor is a greenhouse gas that central and northern parts of Zaire, where there is no dry
traps heat, so its presence in the atmosphere can affect season and rainfall is abundant and constant. The average
the temperature. annual temperature is around 25°C and the average annual
• Frigid zones are located in both the hemispheres between rainfall is over 2000 mm.
the polar circles and the poles. The frigid zones are the Tropical wet and dry climate: This type of climate is found
coldest regions on Earth, and they are characterized by in the southern and eastern parts of Zaire, where there is a
long winters and short summers. distinct dry season and a wet season.

PYQ Workbook 114


GENERAL GEOGRAPHY

The Netherlands is a country in Western Europe that lies on Important Tips


the North Sea coast. It has a temperate maritime climate with
mild winters, cool summers, and rainfall in all seasons. Some of the key characteristics of tornadoes:

West European climate: This type of climate is influenced Funnel-shaped appearance: Tornadoes are often visible
by the Atlantic Ocean and the North Sea, which moderate as funnel-shaped clouds that extend from thunderstorms
the temperature and bring moist air masses. The average to the ground. The funnel cloud is caused by the rapid
annual temperature is around 10°C and the average annual rotation of the air in the tornado.
rainfall is around 800 mm. Strong winds: Tornadoes can produce winds that reach
speeds of up to 300 miles per hour. These strong winds can
77. Solution: (a) cause significant damage to property and infrastructure.
Exp) Option a is the correct answer. Low air pressure at the center: The air pressure at the
Amazonia is a tropical rainforest region located in South center of a tornado is very low. This low air pressure
America. Coniferous forests are found in cold regions, such causes the air to expand and cool, which can lead to the
as the northern hemisphere and the high mountains. They formation of a condensation cloud.
are characterized by their needle-like leaves that remain Tornadoes can also be accompanied by other hazardous
green all year round. weather conditions, such as hail, heavy rain, and lightning.
Important Tip
80. Solution: (a)
Characteristics of Coniferous Forests:
Exp) Option a is the correct answer.
In contrast to equatorial rainforests, coniferous forests
Spring tides occur when the Sun, Earth, and Moon are in
exhibit moderate density and greater uniformity in their
a straight line, with the Earth in between the Sun and the
composition. These forests are known for producing tall,
Moon. This alignment results in the combined gravitational
straight trees with leaves that persist for an extended
forces of the Sun and the Moon acting in the same direction,
period, often up to five years on the same tree. The
causing higher high tides and lower low tides. Spring tides
trunks of conifers serve as food storage, and their
typically occur during the full moon and new moon phases,
thick bark provides protection against extreme cold.
when the Sun and Moon are nearly in alignment.
Conifers typically have a conical shape, which prevents
the accumulation of snow and reduces the impact of 81. Solution: (b)
winds. During hot weather, these trees can undergo rapid Exp) Option b is the correct answer.
perspiration. To minimize excessive transpiration,
coniferous leaves are small, dense, leathery, and needle- A. California: Mediterranean climate: This climate is
shaped. characterized by warm to hot summers with basically no
rainfall, and winters that are short, mild and wet.
78. Solution: (a) B. West Australia: Hot desert climate: This climate is
Exp) Option a is the correct answer. generally arid to semi-arid, with very low rainfall and
Assertion is correct: The atmosphere receives most of its heat high evaporation rates.
indirectly from the sun and directly from the Earth’s surface. C. Bangladesh: Tropical monsoon climate: This climate is
This is because the atmosphere is relatively transparent hot and humid all year round, with a distinct wet season
to shortwave solar radiation, but it is opaque to longwave and dry season depending on the movement of the
terrestrial radiation. Shortwave solar radiation from the sun monsoon winds.
passes through the atmosphere and is absorbed by the Earth’s D. Siberia: Cold temperate climate: This climate has long
surface. The Earth’s surface then emits longwave terrestrial
and harsh winters, with very low temperatures and heavy
radiation, which is absorbed by the atmosphere.
snowfall, and short and cool summers, with moderate
Reason is correct explanation for Assertion: The conversion rainfall.
from shortwave solar to longwave terrestrial energy takes
place at the Earth’s surface. This is because the Earth’s 82. Solution: (a)
surface is much warmer than the atmosphere. Therefore, the Exp) Option a is the correct answer.
atmosphere receives most of its heat indirectly from the sun The Ivory Coast, also known as Côte d’Ivoire, is a country
and directly from the Earth’s surface. in West Africa. It has a dense, hot and humid tropical
79. Solution: (a) rainforest climate, with average temperatures ranging from
24°C to 28°C. The country is home to a variety of plant and
Exp) Option a is the correct answer.
animal life, including elephants, chimpanzees, and gorillas.
Tornadoes can occur in the Caribbean Sea. Tornadoes do not
occur in China sea, Arabian sea and Black Sea.Tornadoes are 83. Solution: (a)
rotating columns of air that can form from thunderstorms. Exp) Option a is the correct answer.
They can be very destructive, with winds that can reach
Soil leaching is the process by which nutrients are washed
speeds of up to 300 miles per hour.
out of the soil by rainwater. It is a natural process, but it is

115 PYQ Workbook


GENERAL GEOGRAPHY

accelerated in tropical rainforests due to the high rainfall oceans, leading to the formation of tidal bulges on both the
and the acidic nature of the rainwater. The acidic rainwater near and far sides of the Earth, resulting in the daily rise and
dissolves the nutrients in the soil, and they are then carried fall of sea levels, known as tides.
away by the rainwater. This can lead to nutrient depletion in Centrifugal Force: The Earth and the moon’s orbital motion
the soil, which can make it difficult for plants to grow. generates a centrifugal force that opposes gravity. This force
Important Tips is stronger on the side of the Earth facing away from the
moon and contributes to the formation of the second tidal
Soil Leaching in Tropical Rainforests: bulge (indirect high tide).
• Soil leaching is a major problem in tropical rainforests
because it can lead to deforestation. When the soil
is depleted of nutrients, trees can no longer grow
properly, and they eventually die. This can lead to the
formation of cleared areas in the rainforest, which
are known as canopy gaps.
• Soil leaching is a serious problem that is threatening
the health of tropical rainforests around the world.
There are a number of things that can be done to
reduce soil leaching, such as planting trees, reducing
deforestation, and using sustainable agricultural
practices.

84. Solution: (a)


Exp) Option a is the correct answer.
The winds flowing from high pressure area toward
Mediterranean Sea are the westerlies. The westerlies blow
into the Mediterranean Sea from the west and northwest.
They help to bring moisture to the region, which is important
for agriculture and tourism.
The westerlies are the prevailing winds that blow from west
to east between 30° and 60° latitude in both the Northern and
Southern Hemispheres. They are caused by the temperature
gradient between the equator and the poles. The equator is
warmer than the poles, so the air at the equator rises. As
the air rises, it cools and condenses, forming clouds and
precipitation. The air then sinks at the poles, and it warms as
it sinks. This creates a circulation pattern of warm air rising
at the equator and cool air sinking at the poles.

85. Solution: (d)


Exp) Option d is the correct answer.
Tides are primarily caused by the gravitational interaction
between the Earth, the moon, and the sun. The main factors
contributing to tidal formation are:
Gravitation: The gravitational pull of the moon is stronger
on the side of the Earth facing the moon and weaker on the Important Tips
opposite side. This difference in gravitational force creates Types of Tides based on Frequency:
two tidal bulges in the ocean, resulting in high tides. Tides are • Semi-diurnal Tide:
also influenced by the sun’s gravitational force, with spring
• This is the most common tidal pattern.
tides occurring during full moons and new moons when the
sun, moon, and Earth are aligned, resulting in higher tidal • It features two high tides and two low tides each
ranges. Neap tides occur when the sun and moon are at right day.
angles to each other, causing lower tidal ranges. • Successive high and low tides are approximately of
Centripetal Force: As the Earth and the Moon both orbit the same height.
around their common center of mass, known as the barycenter, • The time interval between tides is not exactly 12
they experience an inward centripetal force directed toward hours but rather 12 hours and 25 minutes due to the
this point. This force creates a slight outward force on Earth’s moon’s eastward movement.

PYQ Workbook 116


GENERAL GEOGRAPHY

• Diurnal Tide: • The Saffir-Simpson scale is used to measure the


• Diurnal tides consist of only one high tide and one intensity of hurricanes.
low tide during each day. • The Richter scale is used to measure the magnitude of
• The successive high and low tides are roughly of the earthquakes.
same height.
88. Solution: (d)
• Mixed Tide:
Exp) Option d is the correct answer.
• Mixed tides exhibit variations in tide heights.
• These tides are commonly observed along the west Cirrostratus clouds are the highest altitude clouds, forming
coast of North America and on numerous Pacific in the upper troposphere at altitudes of 5 to 13 km (16,000 to
Ocean islands. 43,000 ft). They are thin, wispy clouds that are composed of
ice crystals. Cirrostratus clouds often produce a halo around
• Tidal Patterns in Specific Locations:
the sun or moon.
• Some regions, like Southampton in England,
experience tides 6-8 times a day due to the The other options are not correct:
combined effects of tides from the North Sea and Altocumulus clouds form in the middle troposphere at
the English Channel. These complex tidal patterns altitudes of 2 to 7 km (6,500 to 23,000 ft). They are puffy,
result from the interaction of multiple tidal forces. white clouds that are often seen on fair-weather days.
Altostratus clouds form in the middle troposphere at
86. Solution: (c)
altitudes of 2 to 7 km (6,500 to 23,000 ft). They are gray,
Exp) Option c is the correct answer. sheet-like clouds that produce light to moderate rain or snow.
Hekistotherms are plants that grow in regions with very low Cumulus clouds form in the lower troposphere at altitudes
temperature, such as cold deserts or polar regions. of 0.5 to 1.5 km (1,600 to 5,000 ft). They are puffy, white
Important Tips clouds that are often seen on fair-weather days.
Hekistotherm: 89. Solution: (b)
• Hekistotherm is a term coined by A. L. P. de Candolle Exp) Option b is the correct answer.
in 1874 to describe a cold-tolerant plant of polar
regions, according to his classic temperature-based The maximum percentage of Earth is covered by semi-arid
scheme of world vegetation zones. regions, which account for 15.2% of the land surface. Arid
regions cover 10.6% and sub-humid regions cover 8.7%.
• Hekistotherm plants have a requirement for a
Semi-arid regions receive slightly more precipitation than
temperature of less than 10°C in the warmest month,
arid regions, but they still have limited water resources.
typically occurring in regions with a mean annual
temperature below 0°C. Vegetation in semi-arid regions is typically adapted
to drought conditions. These regions are found on all
• Hekistotherm plants are adapted to survive the harsh
continents.
conditions of cold deserts, where the growing season is
short and the precipitation is low. 90. Solution: (a)
• Some examples of hekistotherm plants are mosses, Exp) Option a is the correct answer.
lichens, sedges, grasses, and cushion plants that
A. Monsoon Forest - 3. Teak and Sal
are found in the tundra biome or the alpine zone of
mountains. Teak and sal are two of the most important tree species found
in tropical monsoon forests. They are both deciduous trees,
87. Solution: (b) meaning that they lose their leaves for a period of time each
Exp) Option b is the correct answer. year. This is a way for the trees to conserve water during the
The Fujita scale is used to measure the intensity of tornadoes. dry season. Teak and sal trees are both highly valued for their
It is based on the damage that tornadoes cause to structures timber. Teak is known for its durability, water resistance,
and vegetation. The scale ranges from F0 to F5, with F5 being and beautiful grain. It is used to make a variety of products,
the most intense. including furniture, boats, and musical instruments. Sal is
The Fujita scale was developed by T. Theodore Fujita in the also a valuable timber, but it is not as durable as teak. It is
1970s. It has been updated several times since then, and it often used to make construction materials, such as beams
is now known as the Enhanced Fujita scale. The Enhanced and rafters.
Fujita scale takes into account a wider range of damage B. Equatorial Forest - 4. Mahogany and Rosewood
indicators, and it provides more accurate estimates of
Mahogany and rosewood are two of the most important tree
tornado wind speeds.
species found in equatorial forests. They are both evergreen
Important Tips trees, meaning that they keep their leaves all year round.
This is because equatorial forests have a warm and humid
• The Mercalli scale is used to measure the intensity of
climate all year round. Mahogany and rosewood are both
earthquakes.
highly valued for their timber. Mahogany is known for its

117 PYQ Workbook


GENERAL GEOGRAPHY

beautiful grain, rich color, and durability. It is used to Important Tips


make a variety of products, including furniture, musical
Lapse Rate:
instruments, and luxury goods. Rosewood is also a valuable
timber, but it is more difficult to work with than mahogany. It • Normal temperature lapse rate is the decrease in
is often used to make veneers, inlays, and other decorative temperature with altitude.
items. • Catabatic lapse rate is the increase in temperature with
C. Mediterranean Forest - 1. Plum and Olive altitude.

Mediterranean forests are characterized by hot, dry 93. Solution: (d)


summers and mild, wet winters. They are found in regions Exp) Option d is the correct answer.
around the Mediterranean Sea, as well as in other parts of the The function of the ozone layer in the stratosphere is to
world with similar climates, such as California, Chile, and prevent ultra-violet radiation effect on the ground. The
Australia. Plum and olive trees are two of the most common ozone layer is a thin part of Earth’s atmosphere that absorbs
tree species found in Mediterranean forests. almost all of the sun’s harmful ultraviolet radiation, or UV
D. Coniferous Forest - 2. Pine and Spruce light, which can damage living things. UV light can cause
skin cancer, cataracts, and impaired immune systems in
Coniferous trees are trees that produce cones instead of
humans and animals, as well as harm plants and ecosystems.
flowers. They are also known as evergreens, because they The ozone layer acts as a shield for life on Earth by filtering
keep their leaves all year round. Coniferous trees are found out the most damaging forms of UV light.
all over the world, but they are most common in cold
• The ozone layer does not stabilize the global
climates. Pine and spruce are two of the most common
temperature, but rather contributes to the greenhouse
coniferous trees. Pine trees are known for their long, needle- effect by trapping some of the infrared radiation emitted
like leaves and their woody cones. Spruce trees are known by the Earth’s surface. However, the ozone layer is not the
for their short, prickly needles and their drooping cones. main cause of global warming, which is driven by other
greenhouse gases such as carbon dioxide and methane
91. Solution: (b)
• The ozone layer has no effect on the frequency of
Exp) Option b is the correct answer.
earthquakes, which are caused by the movement of
Coniferous forests are mainly found in the temperate tectonic plates in the Earth’s crust. The ozone layer is
region. The temperate region is characterized by four distinct located in the stratosphere, which is far above the crust
seasons, with warm summers and cold winters. Coniferous and has no direct interaction with it
trees are well-suited to the temperate climate because they • The ozone layer does not affect the monsoon patterns,
can withstand cold winters and heavy snowfall. Coniferous which are determined by the seasonal changes in
forests are found all over the world in the temperate region, atmospheric pressure and wind circulation over land and
including in North America, Europe, and Asia. Some of the ocean. The ozone layer is relatively stable and does not
largest coniferous forests in the world include the Boreal vary significantly with seasons or regions.
Forest in North America, the Scandinavian Taiga in
94. Solution: (b)
Europe, and the Siberian Taiga in Asia.
Exp) Option b is the correct answer.
92. Solution: (d) A barometer is an instrument used to measure atmospheric
Exp) Option d is the correct answer. pressure. As the atmospheric pressure decreases, the mercury
level in the barometer falls. A sudden fall in the mercury
Clouds are formed when water vapor in the atmosphere
level in a barometer indicates that the atmospheric pressure
condenses into tiny water droplets or ice crystals. This
is decreasing rapidly. This is often a sign that a storm is
happens when the air cools to a temperature below its dew approaching.
point. The dew point is the temperature at which the air
The other options are not correct:
can no longer hold all of its water vapor and condensation
begins. • Fair weather is associated with high atmospheric pressure.
• Cold weather is not directly related to atmospheric
There are two main ways that the air can cool to its dew
pressure.
point: through evaporation and through adiabatic cooling.
Evaporation is the process of water turning into a gas. When • Hot weather is not directly related to atmospheric
water evaporates, it takes heat energy with it, which cools the pressure.
air. Adiabatic cooling is the process of air cooling without 95. Solution: (b)
the exchange of heat with the surrounding environment. Exp) Option b is the correct answer.
This happens when the air rises. As the air rises, it expands
Evergreen rainforests are found in tropical regions around
and cools.
the world, where there is high rainfall and a warm & humid
When the air cools to its dew point, water vapor condenses climate all year round. Brazil is home to the largest rainforest
into tiny water droplets or ice crystals. These water droplets in the world, the Amazon rainforest. The Amazon rainforest
and ice crystals form clouds. covers over 5.5 million square kilometers, and it is home to

PYQ Workbook 118


GENERAL GEOGRAPHY

jaguars, harpy eagles, and pink river dolphins, while also 97. Solution: (b)
providing a habitat for sloths, black spider monkeys, and Exp) Option b is the correct answer.
poison dart frogs. This vast rainforest ecosystem harbours
an astonishing diversity, with one-tenth of all known species Africa is known as ‘the Cradle of Mankind’ because it is
on Earth, including 40,000 plant species, 3,000 freshwater where human life originated. The term specifically refers
fish species, and over 370 different types of reptiles. to a region in South Africa, which has yielded fossil remains
of several human ancestors dating back millions of years.
Important Tips This region, including sites like Sterkfontein, Swartkrans,
Some additional information about the Amazon Kromdraai, Drimolen, Bolt’s Farm, and Gondolin holds
Rainforest: crucial evidence for the study of human evolution, and it was
The Amazon rainforest, the world’s largest, covers about designated a UNESCO World Heritage site in 1999.
40% of South America, spanning eight countries: Brazil,
98. Solution: (a)
Bolivia, Peru, Ecuador, Colombia, Venezuela, Guyana,
Suriname, and French Guiana. It consists of various Exp) Option a is the correct answer.
ecosystems, including rainforests, seasonal forests, The exosphere is the outermost layer of the atmosphere,
deciduous forests, flooded forests, and savannas. The where the air is very thin, and molecules can escape into
Amazon River, the largest by discharge and second-longest space. Communication satellites are located in the
globally, drains the basin, with over 1,100 tributaries, exosphere because they need to orbit at high altitudes and
including two (the Negro and the Madeira) larger than avoid atmospheric drag and interference. The exosphere
the Congo River. Due to its unique vegetation, the Amazon extends from the top of the thermosphere, which is about
is often referred to as the ‘lungs of the planet Earth,’ as it 600 km (373 miles) above the Earth’s surface, to about 10,000
continuously recycles carbon dioxide into oxygen. km (6,200 miles) or more, depending on how it is defined.

96. Solution: (a) 99. Solution: (c)


Exp) Option a is the correct answer. Exp) Option c is the correct answer.
The correct match between List-I (Soils) and List-II (Climate The indirect high tide is caused by the centrifugal force
region) is as follows: of the Earth. The centrifugal force is the force that pushes
A. Podzol - 2. Cold temperate an object away from the center of rotation. Because the
Earth is rotating, its oceans are also rotating. This creates
B. Chernozem - 1. Temperate cold steppe
a centrifugal force that pushes the oceans away from the
C. Spodosols - 4. Humid cold temperate center of rotation. On the side of the Earth opposite the
D. Laterite - 3. Hot and humid moon, the centrifugal force is strongest. This is because
the centrifugal force is inversely proportional to the distance
Important Tips from the center of rotation. The farther away an object is
• Podzol soil, prevalent in cool and humid climates, from the center of rotation, the weaker the centrifugal force.
forms under forest or heath vegetation. Slow The stronger centrifugal force on the side of the Earth
decomposition of organic matter leads to acidic opposite the moon creates the indirect high tide by pushing
organic surface layers. Nutrient deficiency and coarse- the ocean away from the Earth’s center. The indirect high
textured, base-poor parent material further inhibit tide is not as high as the direct high tide, which is caused
microbial activity, promoting the development of by the gravitational pull of the moon. This is because the
Podzol soils. centrifugal force is weaker than the gravitational pull of
• Chernozem is a fertile black soil found in temperate the moon.
grasslands of the Russian steppes and North
American prairies. It contains abundant humus and is
underlain by a lighter lime-rich layer, making it highly
suitable for agriculture.
• Spodosols soils are formed in humid, cool temperate
climates. They are characterized by a thick layer of
organic matter on the surface, followed by a bleached
layer where nutrients have been leached out by the
rainwater. Spodosols are often found in coniferous
forests and mixed forests.
• Laterite soils are formed in hot, humid climates. They
are characterized by a high concentration of iron and
aluminum oxides. Laterite soils are often infertile
and difficult to cultivate. However, they can be used
to grow certain crops, such as rubber and cassava.

119 PYQ Workbook


GENERAL GEOGRAPHY

Important Tips
Westerlies:
The westerlies are caused by the temperature gradient
between the equator and the poles. The equator is warmer
than the poles, so the air at the equator rises. As the
air rises, it cools and condenses, forming clouds and
precipitation. The air then sinks at the poles, and it warms
as it sinks. This creates a circulation pattern of warm air
rising at the equator and cool air sinking at the poles.
The westerlies are a major component of the global
atmospheric circulation. They help to transport heat and
moisture around the world. They also play a role in the
formation of weather patterns, such as storms and fronts.

100. Solution: (c) The Roaring Forties, Furious Fifties, and Shrieking Sixties
are particularly strong because there is less land to disrupt
Exp) Option c is the correct answer. the wind flow in the Southern Hemisphere than in the
The troposphere is the lowest layer of the atmosphere, Northern Hemisphere.
where most weather activity occurs. It is where we experience 103. Solution: (d)
clouds, rain, snow, wind, and temperature changes. The
Exp) Option d is the correct answer.
troposphere is heated from below by the Earth’s surface,
Halophytes are plants that thrive in saline or salty soils.
which absorbs solar radiation and emits infrared radiation.
They have adaptations that allow them to tolerate high
This creates convection currents that move air masses and salinity levels, which would be detrimental to most other
create weather patterns. The troposphere is about 8 to 14 km plants. Halophytes are specialized in extracting water
(5 to 9 miles) thick, depending on the latitude and season. from saline soils and to filter out salt. They are commonly
found in environments like salt marshes, mangrove
101. Solution: (d) swamps, and coastal areas where the soil contains high salt
Exp) Option d is the correct answer. concentrations.
The Intertropical Convergence Zone (ITCZ) is a belt of 104. Solution: (a)
converging trade winds and rising air that encircles the Exp) Option a is the correct answer.
Earth near the Equator. The ITCZ shifts north and south
Chinook winds are warm, dry winds that blow down the
seasonally with the Sun, but it is always located near the eastern slopes of the Rocky Mountains in the United States.
Equator. The ITCZ produces high cloudiness, frequent They are caused by the adiabatic compression of air as it
thunderstorms, and heavy rainfall, as well as calm surface air descends the mountains. The descending air warms and
known as the doldrums. dries, and it can cause temperatures to rise by as much as
20°C (36°F) in a matter of minutes.
Chinook winds are most common in the winter months,
when the temperature difference between the mountains and
the plains is greatest. Chinook winds can melt snow and ice,
which can help to replenish water supplies and reduce the
risk of flooding. They can also help to improve crop yields by
increasing the growing season.

105. Solution: (d)


Exp) Option d is the correct answer.
Assertion (A) is incorrect: Earthworms are actually
beneficial for agriculture. They are considered “ecosystem
102. Solution: (b) engineers” because they improve soil structure and fertility.
Exp) Option b is the correct answer. Reason (R) is correct: Earthworms burrow through the
soil, creating channels that improve aeration and water
The Roaring Forties, Furious Fifties, and Shrieking Sixties
penetration. As they consume organic matter, their castings
are the names given to the prevailing westerly winds that
(excretions) are rich in nutrients and help break down larger
blow between 40- 60 degrees south latitude that is in southern soil particles into finer ones. This process results in softer,
hemisphere. These winds are strong and persistent, and they more crumbly soil that is conducive to plant root growth
can reach speeds of up to 100 knots. and nutrient absorption.

PYQ Workbook 120


GENERAL GEOGRAPHY

106. Solution: (d) 110. Solution: (b)


Exp) Option d is the correct answer. Exp) Option b is the correct answer.
The stratosphere is said to be ideal for flying jet aircraft. The South-East Trade Winds, which blow from the southeast
This is because of the absence of clouds and other weather towards the equator in the Pacific Ocean, are not directed
phenomena. The stratosphere is the second layer of the towards the Indian Ocean as a direct result of El Niño.
atmosphere, above the troposphere where most weather Instead, during El Niño, the South-East Trade Winds may
activity occurs. The stratosphere is very dry and has few weaken or even reverse their direction in the central and
clouds, which makes it ideal for flying jet aircraft. Flying eastern Pacific, leading to reduced upwelling of nutrient-
in the stratosphere also reduces turbulence, air resistance, rich waters along the western coast of South America.
and fuel consumption, as well as avoiding bad weather and Important Tips
meteorological phenomena that can affect flight performance El Niño effects on the atmosphere and oceans:
and safety.
• Distortion of equatorial atmospheric circulation:
107. Solution: (b) During El Niño, the trade winds weaken or even
reverse direction. This allows warm water to build up
Exp) Option b is the correct answer.
in the central and eastern Pacific Ocean.
Cumulonimbus clouds are tall, towering clouds that are
• Irregularities in the evaporation of sea water: The
responsible for thunderstorms, hail, and heavy rain. They
warm water in the central and eastern Pacific Ocean
are formed when warm, moist air rises rapidly and cools. As
has a lower salinity than normal, which reduces the
the air cools, water vapor condenses into tiny water droplets
amount of evaporation. This, in turn, reduces the
and ice crystals. These water droplets and ice crystals grow
amount of precipitation that falls in the region.
larger and larger, and eventually they fall to the ground as
rain, hail, or snow. • Reduction in the amount of planktons which reduces
the number of fish in the sea: The warm water in the
Important Tips central and eastern Pacific Ocean is also less nutrient-
Types of Clouds: rich than normal. This, in turn, reduces the amount of
• Cumulus clouds are puffy, white clouds that are often plankton, which are the base of the marine food chain.
seen on fair-weather days. They do not produce rain. As a result, there are fewer fish in the sea.

• Nimbostratus clouds are gray, sheet-like clouds that 111. Solution: (b)
produce light to moderate rain or snow. Exp) Option b is the correct answer.
• Cirrostratus clouds are thin, wispy clouds that are The formation of an ‘inversion lid’ is the most important
often seen at high altitudes. They do not produce rain. factor for the formulation of smog. The presence of an
inversion lid traps the pollutants near the ground and
108. Solution: (c) hinders their dispersion, leading to the accumulation of
Exp) Option c is the correct answer. pollutants in the lower atmosphere. This, combined with
The stratosphere is the second layer of the Earth’s other factors like sunlight, temperature, and the presence
atmosphere, above the troposphere where most weather of air pollutant resources, contributes to the formulation of
activity occurs. The stratosphere contains a high smog.
concentration of ozone (O3) in relation to other parts of Important Tips
the atmosphere, although it is still small in relation to other
• An inversion lid is a layer of warm air that traps
gases in the stratosphere. The ozone layer is mainly found in
pollutants close to the ground. This layer of warm air
the lower portion of the stratosphere, from approximately is created when the temperature of the air decreases
15 to 35 kilometers (9 to 22 mi) above Earth, although its with height. This temperature inversion prevents
thickness varies seasonally and geographically. The ozone the pollutants from rising and dispersing into the
layer absorbs most of the Sun’s ultraviolet radiation, which atmosphere.
otherwise would potentially damage exposed life forms near
• Inversion lid leading to the formation of smog:
the surface.
• Warm air rises: Normally, warm air rises and mixes
109. Solution: (b) with the cooler air above it. This helps to disperse
Exp) Option b is the correct answer. pollutants into the atmosphere.
Cumulonimbus clouds are born through convection, which • Inversion lid forms: When the temperature of the
is the upward movement of warm, moist air. They are air decreases with height, a layer of warm air can
known for producing hail, thunder, and lightning, making form near the ground. This layer of warm air is
them the only cloud type associated with these weather called an inversion lid.
phenomena. Cumulonimbus clouds are characterized by • Pollutants trapped: The inversion lid prevents
their large cauliflower-shaped towers and often have anvil- the pollutants from rising and dispersing into the
shaped tops due to their vertical development. atmosphere. This can lead to the formation of smog.

121 PYQ Workbook


GENERAL GEOGRAPHY

112. Solution: (a) Important Tips


Exp) Option a is the correct answer. • Cirrus clouds: These clouds are made up of ice crystals
Snow has the highest albedo of any natural surface, and are usually found high in the atmosphere. They are
reflecting up to 90% of incoming sunlight. Barren rocky often wispy and feathery in appearance. Cirrus clouds
piedmont surfaces have an albedo of about 30%, sandy are not associated with precipitation.
deserts have an albedo of about 20%, and mangrove forests • Cumulus clouds: These clouds are also made up of
have an albedo of about 10%.
ice crystals, but they are found at lower altitudes than
113. Solution: (a) cirrus clouds. Cumulus clouds are often puffy and
white in appearance. Cumulus clouds can produce
Exp) Option a is the correct answer.
precipitation, but they are usually associated with light
Statement 1 is correct: Inversion of temperature refers to rain or snow showers.
a situation where the normal decrease in temperature with
• Nimbostratus clouds: These clouds are low-level
increasing altitude is inverted, and temperature increases
clouds that are often dark and gray in appearance.
with height. It is a deviation from the typical atmospheric
Nimbostratus clouds are associated with continuous
temperature profile.
rain or snow.
Statement 2 is correct: Inversion of temperature often occurs
• Stratus clouds: These clouds are also low-level clouds,
during clear and calm nights in winter when the ground loses
but they are thinner than nimbostratus clouds. Stratus
heat rapidly through radiation. The cool air near the surface
clouds are often gray or white in appearance and can
becomes denser and sinks, creating a stable layer of cold air
produce light drizzle or snow flurries.
near the ground, which traps warmer air above it. This leads
to the inversion of temperature.
115. Solution: (c)
Statement 3 is incorrect: Inversion of temperature can
occur in various regions and under different atmospheric Exp) Option c is the correct answer.
conditions, including both polar and non-polar regions. It The Coriolis force is an apparent force that affects moving
is not strictly a short-term phenomenon and can persist for objects in a rotating reference frame like the Earth. It
several days or even weeks under favorable conditions. influences the direction of motion for winds and ocean
Important Tips currents. In the northern hemisphere, the Coriolis force
causes deflection to the right, resulting in clockwise rotation
Different types of temperature inversions:
of cyclones and the rightward shift of ocean currents. In the
• Radiation inversion: This type of inversion occurs at southern hemisphere, the Coriolis force causes deflection to
night, when the ground cools off rapidly by radiation. the left, leading to counterclockwise rotation of cyclones
The cool air then sinks, creating a layer of warm air
and the leftward shift of ocean currents.
above it. This type of inversion is most common in
clear, calm nights. 116. Solution: (b)
• Subsidence inversion: This type of inversion occurs Exp) Option b is the correct answer.
when air sinks from higher altitudes. As the air sinks, The correct match is (b) 3-4-2-1
it compresses and warms up. This type of inversion is
most common in areas with high pressure systems. • Cyclones are intense low-pressure systems characterized
by rotating winds, typically found over warm tropical
• Frontal inversion: This type of inversion occurs when
waters, and associated with the Indian Ocean.
a cold air mass undercuts a warm air mass. The front
between the two air masses then has warm air above • Hurricanes are large, powerful tropical cyclones that
and cold air below. This type of inversion is most form over the Atlantic Ocean, known for their activity in
common in areas where cold and warm air masses are the Caribbean Sea and along the eastern coast of North
meeting. America.
• Turbulence inversion: This type of inversion occurs • Typhoons, named for the western Pacific Ocean region,
when turbulent air mixes with calm air. The turbulent are commonly found in the South China Sea, known
air is warmer than the calm air, so the temperature for frequent occurrences that affect countries like the
of the mixed air increases with height. This type of Philippines, Taiwan, and China.
inversion is most common in areas with strong winds. • Willy-willies, or dust devils, occur in arid regions,
114. Solution: (c) particularly in Western Australia, known for its dry and
desert-like landscapes.
Exp) Option c is the correct answer.
A. Cirrus - 4. Entirely of ice particles 117. Solution: (b)
B. Cumulus - 3. Fair-weather clouds Exp) Option b is the correct answer.
C. Nimbostratus - 1. Storm Clouds In the Eastern shore of the Adriatic Sea, a cold and dry
D. Stratus - 2. High fog “Low Ceiling” wind blowing down from the mountain is known as Bora.

PYQ Workbook 122


GENERAL GEOGRAPHY

It is a strong, gusty wind originating from the northeast is typically found in regions near the equator, characterized
and can reach hurricane-like speeds. The Bora brings cold by distinct wet and dry seasons. It is commonly associated
temperatures and clear skies, but it can also cause damage with grasslands, scattered trees, and shrubs.
to structures and trees along the coast.
Important Tips
Important Tips Savanna landscapes are characterized by tall grass and
Some other Cold Winds along with their locations: short trees, also known as “bush-veld”. Trees in the
• Mistral: France savanna are deciduous, with broad trunks and water-
storing adaptations to survive drought. The grass in true
• Nor’easter: Northeastern United States
savanna lands grows tall and coarse, reaching heights of
• Siberian Wind: Siberia, Russia 6 to 15 feet. During the rainy season, the grass appears
• Purga: Russia green and lush, but turns yellow and dies down in the
• Zonda: Argentina subsequent dry season. As rainfall decreases towards
the deserts, the savanna transitions into thorny scrub
• Williwaw: Alaska, United States
vegetation.
• Buran: Central Asia
• Piteraq: Greenland 121. Solution: (a)
• Karaburan: Turkey Exp) Option a is the correct answer.
• Puna: Andean region (Peru, Bolivia, Chile, Argentina) Tundra is a type of biome characterized by cold temperatures,
• Roaring Forties: Southern Hemisphere, particularly short growing seasons, permafrost, and low-growing
around the 40th parallel vegetation. It is typically found in high-latitude regions
near the Arctic, such as northern parts of Russia, Canada,
and the United States. While Russia, Canada, and the USA
118. Solution: (c)
have regions that experience Tundra vegetation due to their
Exp) Option c is the correct answer. northern latitudes, Belarus is located further south and does
In the Hadley cell thermal circulation, the air descends at not have Tundra vegetation.
sub-tropical high-pressure cells. The Hadley cell operates
122. Solution: (b)
between the equator and subtropics, with warm air rising at
the intertropical convergence zone. Exp) Option b is the correct answer.
The most noticeable characteristic of the Mediterranean
Important Tips
climate is a dry summer. In Mediterranean climate regions,
The Hadley cell, present in both hemispheres between such as Southern Europe, California, and parts of Australia
10 and 30 degrees latitude, is a solar cell driven by high and Chile, summers tend to be hot and dry.
solar insolation. At the equator, intense heating leads to
These areas experience a distinct pattern of wet winters and
the rising of air, which then cools and forms the anti-
dry summers, with most of the rainfall occurring during the
trade wind moving away from the pole. This ascending air
cooler months.
reaches the upper atmosphere around 30 degrees latitude,
where it descends, creating subtropical high-pressure Important Tips
systems. The cell is completed by the trade winds, which
Mediterranean climate:
blow from the high-pressure region towards the equator.
The Hadley cell is known for its stability and is associated • The regions are located in the Western Part of the
with tropical monsoons and desert climates. Continent in between 30-45 degrees North and South
of the Equator.
119. Solution: (d) • Pressure belts are anticyclonic, meaning they rotate
Exp) Option d is the correct answer. clockwise in the Northern Hemisphere, and anti-
Timber vegetation is generally not found in the tundra clockwise in the Southern Hemisphere.
region. The tundra region is characterized by extremely cold • Anti-cyclonic conditions are accompanied by a clearer
temperatures, permafrost (permanently frozen ground), sky.
and a short growing season. These conditions are not • Rainfall is uneven, but the regions with Mediterranean
conducive to the growth of large trees and forests. Instead, climates experience winter rainfalls.
the tundra is dominated by low-lying vegetation such as
mosses, lichens, and shrubs. 123. Solution: (b)
Exp) Option b is the correct answer.
120. Solution: (a)
The Mediterranean region is an important supplier of citrus
Exp) Option a is the correct answer. fruits. This region is known for its favorable climate for
The indication of the Tropical Savannah climate is growing citrus fruits such as oranges, lemons, grapefruits,
represented by the code “Aw”. The Tropical Savannah climate and mandarins. The Mediterranean climate, characterized

123 PYQ Workbook


GENERAL GEOGRAPHY

by hot, dry summers and mild, wet winters, provides ideal Both statements are individually true, but Statement II is not
conditions for the cultivation of citrus fruits. the correct explanation of Statement I.

124. Solution: (a) Statement I is correct: Tides are primarily caused by the
gravitational attraction between the Moon and the Earth,
Exp) Option a is the correct answer.
with a secondary influence from the Sun. The rotation of the
Option a is correct: Temperate cyclones, also known as Earth also plays a role in the complex tidal patterns observed.
mid-latitude cyclones, generally move from West to East Statement II is correct: The Earth rotates on its axis from
with the prevailing westerly winds in the middle latitudes. west to east, which is the cause of the daily cycle of day and
These cyclones are typically associated with the polar front night. However, this statement does not directly explain the
and are responsible for the weather patterns experienced in phenomenon of tides mentioned in Statement I. Tides are
the mid-latitudes. primarily caused by gravitational forces, not the rotation
Option b is incorrect: The “eye” of a cyclone refers to the of the Earth.
central part of a tropical cyclone or hurricane, which is
characterized by calm and relatively clear conditions. 127. Solution: (d)
The front side of a cyclone does not correspond to the eye; Exp) Option d is the correct answer.
instead, it is usually associated with the leading edge of the The maximum development of the tropical evergreen
storm where the strongest winds and most severe weather rainforest biome has not taken place in central and Southern
conditions are found. California and North-Western coastal lands of Africa. This
Option c is incorrect: Cyclones are characterized by a biome is primarily found in the equatorial regions and
center of low pressure, not high pressure. In a cyclone, extends between 10° N and 10° S latitudes. It is commonly
the air spirals inward toward the center, creating a region associated with regions such as the Amazon rainforest in
of low atmospheric pressure. The isobars, which are lines South America, the Congo Basin in Africa, and Southeast
connecting points of equal atmospheric pressure, are Asia.
typically arranged in a circular or spiral pattern around the
128. Solution: (d)
center of the cyclone.
Exp) Option d is the correct answer.
Option d is incorrect: Hurricanes are not typically
associated with the mid-latitudes (lies between the tropics The statement “Tsunami is a Japanese word and not a Latin
and the polar circles, or roughly between 30° and 60° north word. The word “tsunami” is derived from the Japanese
or south of the equator). They are intense tropical cyclones words “tsu” meaning “harbor” and “nami” meaning
that primarily develop in the tropical and subtropical “wave.” It refers to a series of ocean waves caused by the
regions, usually between 5- and 30-degrees latitude over displacement of a large volume of water, usually due to an
warm ocean waters. undersea earthquake, volcanic eruption, or landslide.

125. Solution: (a) Important Tips

Exp) Option a is the correct answer. Tsunamis:

Both statements are individually true, and Statement II is the • Tsunamis can travel across the ocean at high speeds,
reaching speeds up to 800 kilometers per hour (500
correct explanation of Statement I.
miles per hour).
Statement I is correct: Tundra climate is characterized by
• As they approach the coastline, the depth of the
extremely cold temperatures and a short growing season,
water decreases, causing the wave to slow down and
which limits the diversity of plant and animal species that
increase in height, potentially causing significant
can survive in such harsh conditions. The low temperatures
damage and destruction.
and frozen soil make it difficult for vegetation to grow,
resulting in a sparse and limited biodiversity. • The Indian Ocean tsunami in 2004 was one of the
deadliest in recorded history, claiming the lives of
Statement II is correct: Tundra climate has less reproductive
more than 230,000 people in 14 countries.
warm period. This statement is the correct explanation
of Statement I. The short growing season in the tundra 129. Solution: (a)
climate restricts the reproductive opportunities for many
Exp) Option a is the correct answer.
organisms. The warm period in the tundra is brief, usually
lasting only a few weeks to a couple of months. This limited Statement 1 is correct- A cloudburst is defined as a sudden,
period of warmth hampers the growth and reproduction of localized, and very heavy downpour accompanied by
thunder and lightning. It occurs over a small geographical
plants and animals, leading to lower biodiversity compared
area.
to regions with longer and more favorable reproductive
periods. Statement 2 is correct- Cloudbursts mostly occur in hilly
or mountainous areas. The complex topography and
126. Solution: (b) orographic lifting in such regions can enhance the convective
Exp) Option b is the correct answer. processes that lead to the formation of cloudbursts.

PYQ Workbook 124


GENERAL GEOGRAPHY

Statement 3 is correct- Cloudbursts are characterized by Important Tips


a very high intensity of rainfall, often ranging from 250
Tropical Cyclone:
mm to 300 mm or even more in just a couple of hours. This
extremely heavy rainfall within a short duration can lead to • The eye is a calm region at the center of a tropical
flash floods, landslides, and other related hazards. cyclone, usually 30-65 kilometers (19-40 miles) wide.

Statement 4 is incorrect- Cloudbursts can happen at any • It is encircled by the eyewall, where the most severe
weather and highest winds occur.
time, although certain atmospheric conditions, such as
convective instability and high moisture content, are more • The eye has the lowest barometric pressure, up to 15
conducive to their formation. percent lower than the outside.
• Strong cyclones have a well-defined eye with light
130. Solution: (d) winds and clear skies, while weaker cyclones may have
Exp) Option d is the correct answer. a less defined eye covered by thick clouds.
The polar front jet stream is stronger and more prominent
134. Solution: (c)
during the winter months when there is a greater temperature
contrast between the polar regions and the mid-latitudes. In Exp) Option c is the correct answer.
summer, the polar front jet weakens and often becomes less In the Northern Hemisphere, air circulates around a low-
defined. pressure area in a counterclockwise direction. This is
due to the Coriolis effect, which is a result of the Earth’s
131. Solution: (c) rotation. The Coriolis effect deflects moving air to the
Exp) Option c is the correct answer. right in the Northern Hemisphere, causing the air to
Statement 1 is correct: Mediterranean climates have wet flow counterclockwise around a low-pressure center and
winters and dry summers, while monsoon climates have wet towards the center.
summers and dry winters.
135. Solution: (b)
Statement 2 is incorrect: The annual range of temperature
Exp) Option b is the correct answer.
is higher in monsoon climates than in Mediterranean
climates. This is because monsoon climates are located Cold fronts, which represent the leading edge of a cold
in the tropics, where the temperature difference between air mass, tend to move faster than warm fronts. As a
summer and winter is greater. The Mediterranean climate result, they often catch up to and overtake the slower-
is characterized by mild, wet winters and hot, dry summers moving warm fronts. This can lead to weather conditions
and where the temperature range is generally moderate characterized by the rapid lifting of warm air, potentially
throughout the year. causing thunderstorms and other atmospheric disturbances.

Statement 3 is correct: Both Mediterranean and monsoon Important Tips


climates have rainy and dry seasons. In the Mediterranean • Warm Front:
climate, the winter season is typically rainy, while the • A warm front occurs when a warm air mass moves
summer season is dry. In the Monsoon climate, there is a towards a cold air mass.
clear distinction between the wet monsoon season and the
• It has a gentle slope and as the warm air rises, it
dry season, with the monsoon bringing the majority of the
condenses and causes precipitation.
annual precipitation.
• Unlike a cold front, temperature and wind
132. Solution: (d) changes are gradual along a warm front. These
Exp) Option d is the correct answer. fronts bring light to moderate precipitation across
a wide area for several hours.
The Coriolis effect is the reason why the wind in the
• Cold Front:
Southern hemisphere is deflected towards its left. The
Coriolis effect is caused by the rotation of the Earth. As • A cold front happens when cold air moves towards
the Earth rotates, it deflects moving objects to the right in a warm air zone. The dense cold air remains near
the Northern hemisphere and to the left in the Southern the ground and uplifts the less dense warm air.
hemisphere. • Cold fronts have a steeper slope and are associated
with a narrow band of clouds and precipitation.
133. Solution: (a)
• Weather along a cold front can include storms with
Exp) Option a is the correct answer. high winds, frequent thunderstorms in the warm
In the eye, the temperature is typically higher compared to sector, and even tornadoes in warm regions.
the surrounding areas, and the pressure is at its lowest. This
136. Solution: (a)
is because the air in the eye descends, causing compression
and warming, while the surrounding areas experience strong Exp) Option a is the correct answer.
updrafts and convergence, leading to lower pressures. The correct answer is 4-1-2-3.

125 PYQ Workbook


GENERAL GEOGRAPHY

• Ebony is a hardwood tree that is found in tropical 140. Solution: (b)


evergreen forests. Exp) Option b is the correct answer.
• Shisham is a hardwood tree that is found in the Moist The Collision-Coalescence process of precipitation is
deciduous forests. applicable to clouds that do not extend beyond the freezing
• Walnut is a nut tree that is found in Himalayan moist level. This process occurs in clouds where the temperature
forest. is above freezing throughout. In these clouds, the water
• Birch is a hardwood tree that is found in alpine forests. droplets collide and coalesce (merge) together to form larger
droplets. As these droplets grow in size, they eventually
137. Solution: (d)
become too heavy to remain suspended in the cloud and fall
Exp) Option d is the correct answer. to the ground as precipitation.
A. Mediterranean – 4. Temperature range is moderate with In clouds that extend beyond the freezing level, another
warm to hot summers and mild winters. process called the Ice Crystal (Bergeron) process is
B. Marine West Coast - 1.Temperature cycle is moderated by responsible for precipitation formation. This process involves
marine influence. the growth of ice crystals at the expense of supercooled water
C. Dry Mid-Latitude - 3.Strong temperature cycle with large droplets.
annual range. Warm summers to hot and cold winters to very
141. Solution: (a)
cold.
Exp) Option a is the correct answer.
D. Moist Continental - 2.Warm summers and cold winters
with 3 months below freezing very large annual temperature Statement 1 is correct: The equator receives nearly the same
range. amount of daylight and darkness throughout the year . This
results in a relatively consistent climate without significant
138. Solution: (d) seasonal variations.
Exp) Option d is the correct answer.
Statement 2 is incorrect: The Earth’s rotational velocity is
• Lianos - Lianos is a type of grassland found in Venezuela. maximum at the equator, but this does not affect the seasons.
It is characterized by vast plains and is an important
Statement 3 is incorrect: The Coriolis force is zero at
region for agriculture and cattle ranching.
the equator, but this affects the wind patterns and ocean
• Prairies - Prairies are grasslands found in the United currents, not the seasons.
States. They are typically characterized by fertile soils
and a mix of grasses, flowers, and shrubs. Prairies are 142. Solution: (a)
known for their agricultural productivity. Exp) Option a is the correct answer.
• Pampas - Pampas is a type of grassland found in Both the statements are individually true and statement II
Argentina. It is one of the largest grassland areas in is the correct explanation of statement I.
the world and is known for its rich soil and extensive
Statement I is correct: Tsunami is a series of waves in a
cattle ranching. The Pampas region is also important
water body caused by the displacement of a large volume of
for agriculture, particularly for wheat and soybean
cultivation. water, typically in an ocean or a large lake.

• Downs - Downs is a term used to refer to open grassy Statement II is correct: Tsunamis can be generated when
areas in Australia. They are often found in the coastal thrust faults associated with convergent or destructive plate
regions and are characterized by low, rolling hills covered boundaries move abruptly. This movement can displace a
with grasses. Downs are used for livestock grazing and significant amount of water, leading to the formation of
are an important part of Australia’s pastoral industry. a tsunami.

139. Solution: (b) 143. Solution: (a)


Exp) Option b is the correct answer. Exp) Option a is the correct answer.
Both the statements are individually true, but statement II Statement I is correct: Atacama Desert is the driest among
is not the correct explanation of statement I. the deserts of the world. The Atacama Desert, located in
Statement I is correct: Tides are the rise and fall of sea levels South America, is known for its extremely low levels of
caused by the combined effects of the gravitational forces precipitation, making it one of the driest places on Earth.
exerted by the Moon and the Sun and the rotation of the Statement II is correct: The desert’s location between
Earth. two mountain chains, the Andes and the Chilean Coast
Statement II is correct: Earth rotates from the West towards Range, plays a significant role in its extreme dryness.
the East once in 24 hours with respect to the Sun. However, it These mountain chains are of sufficient height to create a
does not provide a direct explanation of the phenomenon rain shadow effect. As moist air from the Pacific or Atlantic
of tides. he gravitational forces of the Moon and the Sun are Ocean moves towards the Atacama Desert, it is forced to rise
the primary drivers of tides. due to the presence of the mountains.

PYQ Workbook 126


GENERAL GEOGRAPHY

As the air rises, it cools and loses moisture through • Regional Impacts: El Niño brings rain to South
precipitation on the windward side of the mountains,
America but droughts to Indonesia and Australia. This
leaving little moisture for the Atacama Desert. The rain
affects water resources, agriculture, and overall climate
shadow effect created by these mountain chains prevents
patterns in these regions.
significant moisture advection from either the Pacific or
the Atlantic Ocean, contributing to the extreme aridity of • Positive Impact: El Niño reduces the occurrence
the Atacama Desert. of hurricanes in the Atlantic. It can have a positive
influence on hurricane season activity.
144. Solution: (c)
Exp) Option c is the correct answer. 146. Solution: (d)

Tropical savannas are found in regions with a pronounced Exp) Option d is the correct answer.
wet and dry season, typically in tropical latitudes. They are Statement 1 is correct: The soils of tropical rain forests are
characterized by a mix of grassland and scattered trees quite infertile because the heavy rainfall washes away the
and shrubs. The vegetation in tropical savannas is adapted nutrients in the soil.
to the seasonal changes in rainfall, with grasses dominating Statement 2 is correct: The vegetation is evergreen, enabling
during the wet season, and trees and shrubs surviving the photosynthesis to take place year around because the trees
dry season. in tropical rain forests receive a lot of sunlight and water
throughout the year.
Important Tips
Statement 3 is correct: Tropical rain forests have been
• Mid-latitude broad-leaf mixed forest refers to a forest
described as “deserts covered by trees” because, despite the
type found in mid-latitudes characterized by a mix of
dense vegetation, the soil is often very poor in nutrients.
broad-leafed trees.
Statement 4 is correct: Tropical rain forests are the most
• Temperate rainforest refers to a type of forest found in
productive land-based ecosystems on Earth because they
temperate regions with high rainfall and dominated have a high rate of photosynthesis and a large number of
by evergreen trees. species.
• Mid-latitude grassland refers to grassland vegetation
Important Tips
found in mid-latitudes, characterized by a lack of
trees or shrubs. Tropical Wet Evergreen Forests or Rainforests:
• Climatic Conditions:
145. Solution: (b)
• Abundant Rainfall: Annual rainfall exceeds 250
Exp) Option b is the correct answer.
cm, creating a highly moist environment.
Statement 1 is incorrect: Jet streams are narrow bands
• Moderate Temperature: The annual temperature
of strong winds that flow in the upper atmosphere. While
ranges from about 25°-27°C, providing warm
they are often associated with westerly winds, they can have
conditions throughout the year.
both easterly and westerly components depending on their
location and season. • High Humidity: The average annual humidity
exceeds 77 percent due to the proximity to water
Statement 2 is correct: El Niño is a climate pattern
characterized by the warming of the Pacific Ocean along the bodies.
equator, particularly in the central and eastern regions. It is • Short Dry Season: These forests have a distinctly
associated with the weakening or reversal of trade winds and short dry season, with consistent moisture
can have significant impacts on global weather patterns. availability.
Statement 3 is correct: El Niño events typically manifest • Characteristics:
along the western coast of South America, particularly off • Evergreen Nature: Trees in rainforests do not shed
the coast of Peru. their leaves simultaneously, remaining evergreen
Important Tips year-round due to the high heat and humidity.
• Ocean Temperature and Currents: Warmer surface • Mesophytic Plants: Rainforests support
waters shift from the western Pacific to the eastern mesophytes, plants adapted to moderately moist
Pacific, affecting marine ecosystems. conditions, unlike hydrophytic or xerophytic
extremes.
• Increased Rainfall: El Niño brings increased
precipitation and convection above warmer surface • Tall Trees: The forest canopy comprises lofty trees,
waters. South America experiences heavy rainfall, often reaching heights of 45 to 60 meters.
leading to coastal flooding and erosion. • Thick Canopy: Rainforests present a dense canopy
• Upwelling and Marine Life: El Niño disrupts the of foliage when viewed from above, occasionally
broken by large rivers or cleared areas.
upwelling process along the coasts of Ecuador, Peru,
and Chile. Upwelling, which brings nutrient-rich water • Vertical Growth: Most plants, including epiphytes,
to the ocean surface, is reduced, impacting marine compete for sunlight by growing vertically, creating
ecosystems. distinct layers within the forest.

127 PYQ Workbook


GENERAL GEOGRAPHY

• Limited Undergrowth: Due to the dense canopy, move from one place to another. Advection can change the
sunlight penetration is minimal, resulting in limited temperature, moisture, and pressure of a region, affecting
undergrowth consisting mainly of bamboos, ferns, its weather conditions. Advection is more important than
climbers, and orchids. convection in mid-latitude regions, where most of diurnal
(day and night) variations in daily weather are caused by
147. Solution: (b) advection alone.
Exp) Option b is the correct answer.
151. Solution: (b)
Chaparral vegetation is found in Mediterranean climate,
Exp) Option b is the correct answer.
which is characterized by hot, dry summers and mild,
wet winters. Chaparral is a vegetation community generally Statement 1 is incorrect: Mangroves is a shrub or small tree
composed of hard stemmed, leathery leaved shrubs. Cacti that grows in coastal saline or brackish water.
may also be present. Statement 2 is correct: Mangrove plants are well adapted to
muddy and waterlogged conditions. They have specialized
root systems, such as stilt roots or pneumatophores, which
help them anchor in the muddy substrate and obtain oxygen.
Statement 3 is correct: Mangrove forests typically grow
along sheltered coastlines, estuaries, and river deltas.
They thrive in intertidal zones, where they are periodically
exposed to both tidal and freshwater influences.
Statement 4 is correct: Mangrove forests can be found in
various coastal regions around the world, including all the
coastal states of India. The Andaman and Nicobar Islands,
being an archipelago, also have mangrove ecosystems.

148. Solution: (d) 152. olution: (a)


Exp) Option d is the correct answer. Exp) Option a is the correct answer.
North America mainly South-West America: North America The tides whose height is 20 percent more than normal tide
is a continent that lies mostly in the Northern Hemisphere are called Spring tides.
and covers most of the land area between the Arctic Ocean Spring tides occur when the gravitational forces of the
and the Panama Canal. South-West America is a region that Moon and the Sun align and reinforce each other, resulting
covers the south-western part of the United States of America, in higher high tides and lower low tides. During spring tides,
including states such as California, Arizona, Nevada, Utah, the tidal range is greater than average, with higher high tides
New Mexico, and Colorado. These regions do not have and lower low tides. These tides occur during the new moon
tropical deciduous forests but have other types of biomes and full moon phases.
such as deserts, grasslands, chaparral (Mediterranean
Important Tips
shrubland), temperate coniferous forests (pine forests),
and temperate deciduous forests (oak forests). • Neap tides occur when the gravitational forces of the
Moon and the Sun are perpendicular to each other,
149. Solution: (a) causing the tidal range to be smaller than average.
Exp) Option a is the correct answer. Neap tides occur during the first and third quarter
Doldrums are equatorial calms, characterized by calm phases of the moon.
and light winds. They are regions near the Earth’s equator • Apogean and perigean tides refer to the tides that
where the trade winds from the Northern and Southern occur when the Moon is at its farthest or closest point
Hemispheres converge. These converging winds create an to the Earth in its elliptical orbit.
area of low pressure and weak winds, resulting in a zone of • Daily and semi-diurnal tides refer to the frequency
calm and variable weather conditions. of tides, with daily tides occurring once in a day and
Equatorial calms, calm and light winds, and variable both semi-diurnal tides occurring twice in a day.
in position and in extent are all features of the doldrums.
153. Solution: (b)
Roaring forties are strong westerly winds found in the
Southern Hemisphere, not near the equator. Exp) Option b is the correct answer.
Statement 1 is correct: A tropical cyclone is an intense
150. Solution: (c) circular storm that originates over warm tropical oceans
Exp) Option c is the correct answer. and is characterized by low atmospheric pressure, high
Advection is the process of transferring heat by the winds, and heavy rain.
horizontal movement of air (wind). Advection occurs Statement 2 is correct: It also requires the presence of
when air masses with different temperatures and humidity Coriolis force to make it spin

PYQ Workbook 128


GENERAL GEOGRAPHY

Statement 3 is incorrect: Tropical cyclones do not occur in throughout the year. The average annual temperature is
the middle latitudinal region because the Coriolis force is around 27°C, with little variation between months. The term
too weak there megathermal comes from the Greek words mega (large) and
Statement 4 is correct: A tropical cyclone develops an ‘eye’ thermos (heat).
with calm and descending air condition at the center of the Statement 2 is incorrect:It is not a sclerophyll region,
storm meaning that it does not have vegetation with hard, leathery
leaves that are adapted to dry conditions. Sclerophyll regions
Important Tips
are typically found in areas with a Mediterranean climate,
The conditions that favor the formation and such as southern Europe, California, Chile, and South Africa.
intensification of tropical cyclones are:
Statement 3 is incorrect: It is not a region of high
• Large sea surface with temperature higher than 27° C. development, meaning that it does not have a high level
This provides enough moisture and heat to the storm. of economic growth, industrialization, urbanization,
• Presence of the Coriolis force. This causes the storm to or human well-being. The equatorial region faces many
rotate and create a cyclonic vortex. challenges, such as poverty, inequality, malnutrition, disease,
• Small variations in the vertical wind speed. This deforestation, biodiversity loss, climate change, and political
prevents the storm from being torn apart by wind instability.
shear. Statement 4 is correct: It is a region of aerial streamlets,
• A pre-existing weak low-pressure area or low-level- meaning that it has streams or rivulets of water that
cyclonic circulation. This helps to initiate the storm flow through the air or along the surface of plants. Aerial
development. streamlets are formed by the high rainfall and humidity
• Upper divergence above the sea level system. This in the equatorial region, which create a moist and misty
allows the air to rise and create a low-pressure center. atmosphere.

154. Solution: (a) 156. Solution: (c)


Exp) Option a is the correct answer. Exp) Option c is the correct answer.
• Chaparrals are evergreen shrublands that are found in Advection of cold dry air in the lower troposphere and
regions with a Mediterranean climate, characterized by warm moist air in the upper troposphere: This is not a
hot dry summers and mild wet winters. Chaparrals are favorable condition for the occurrence of a thunderstorm.
the dominant vegetation of coastal and inland mountain Advection is the horizontal movement of air by wind. If cold
areas of southwestern North America, especially dry air moves into the lower troposphere and warm moist
California and northern Mexico. air moves into the upper troposphere, this would create a
stable atmosphere, where the air resists vertical motion.
• Maquis are dense thickets of evergreen shrubs and
A stable atmosphere inhibits thunderstorm development,
small trees that are found in regions with a Mediterranean
as it prevents the formation of cumulonimbus clouds and
climate, characterized by hot dry summers and mild wet
winters. Maquis are the dominant vegetation of southern precipitation.
Europe, especially around the Mediterranean Sea. Important Tips
• Fynbos are heathlands of low-growing shrubs and Favorable condition for occurrence of a thunderstorm:
herbs that are found in regions with a Mediterranean
• Atmospheric instability: This means that the air near
climate, characterized by hot dry summers and mild wet
the surface is warm and moist, and the air above it is
winters. Fynbos are the dominant vegetation of the Cape
cooler and drier. This creates a steep lapse rate, which
Floristic Region in South Africa, which is one of the
means that the temperature decreases rapidly with
world’s biodiversity hotspots
height.
• Malle scrubs are woodlands of multi-stemmed
• Lifting of potentially unstable air: This means that
eucalyptus trees that are found in regions with a semi-arid
there is some mechanism that triggers the upward
climate, characterized by low and erratic rainfall. Malle
movement of the warm and moist air near the surface.
scrubs are the dominant vegetation of southwestern
Australia, especially in the Mallee region . • Large supply of warm and moist air: This means that
there is enough water vapor in the lower atmosphere
155. Solution: (a) to fuel the thunderstorm. The water vapor condenses
Exp) Option a is the correct answer. into cloud droplets as the air rises and cools, releasing
latent heat.
The equatorial region is a zone of the Earth that lies between
5° to 10° north and south of the equator. It includes parts of • Upper level divergence: This means that there is a
Central and South America, Central Africa, Southeast Asia, horizontal outflow of air at high altitudes above the
and some islands in the Caribbean Sea. thunderstorm. This helps to create a low-pressure
Statement 1 is correct: It is a megathermal region, meaning center at the top of the storm, which enhances the
that it has a tropical climate with high temperatures upward motion of the air.

129 PYQ Workbook


GENERAL GEOGRAPHY

157. Solution: (a) the earth’s surface due to various factors like cloud cover,
Exp) Option a is the correct answer. mountains, valleys, water bodies, vegetation, and desert
lands, plays a role in the creation of wind patterns. The earth’s
Tropical wet evergreen forests are characterized by high
rotation generates the Coriolis force. The Coriolis force
levels of rainfall, typically exceeding 250 cm. They have a
deflects the winds to the right in the Northern Hemisphere
high average humidity, often exceeding 75%. The annual
and to the left in the Southern Hemisphere. This is why the
temperature range in these forests is typically within 25-
trade winds blow from east to west in the tropics, and the
27°C.
westerlies blow from west to east in the mid-latitudes.
One of the distinguishing features of tropical wet
evergreen forests is that they have a continuous leaf canopy Important Tips
throughout the year. This means that trees in these forests There are three categories of Planetary Winds:
do not shed their leaves seasonally, unlike deciduous forests.
Trade winds: Steady prevailing winds that blow from
Important Tips the northeast in the Northern Hemisphere and from the
Tropical Wet Evergreen: southeast in the Southern Hemisphere. They are located
between the equator and the subtropical high-pressure
• In India, tropical wet evergreen forests are found in
zones.
the Andaman and Nicobar Islands, the Western Ghats,
the coastline of peninsular India, and the greater Westerlies: Prevailing winds that blow from the
Assam region in the north-east. southwest in the Northern Hemisphere and from the
northwest in the Southern Hemisphere. They are located
• Some of the dominant tree species in these forests
between the subtropical high-pressure zones and the
are ebony, mahogany, rosewood, rubber, chinchona,
subpolar low-pressure zones.
hollong, sal, dipterocarpus, and terminalia.
Polar winds: Cold winds that blow from the polar regions
158. Solution: (d) toward lower latitudes. In the Northern Hemisphere,
Exp) Option d is the correct answer. they are known as polar easterlies, and in the Southern
Statement 1 is incorrect- A stationary front is a front that is Hemisphere, they are known as polar westerlies. These
not moving, so it cannot create an occluded front. winds are associated with the subpolar low-pressure zones
Statement 2 is incorrect- A cold air mass moving to a warm and the polar high-pressure zones.
air mass can create a cold front, but not an occluded front.
160. Solution: (d)
Statement 3 is incorrect- A warm air mass moving to a cold
Exp) Option d is the correct answer.
air mass can create a warm front, but not an occluded front.
Buttress roots are large, wide, and above-ground roots
Statement 4 is correct- An occluded front is a weather
that extend from the base of the trunk of certain tree species
front that forms when a cold air mass overtakes a warm air
mass. The cold front moves faster than the warm front, so in tropical rainforests. These roots are specifically adapted
it eventually catches up to and lifts the warm air mass up to provide additional stability and support to the tall and
and over the cold air mass. This creates a complex weather heavy trees in the rainforest ecosystem. The buttresses
system with a variety of precipitation types. have to bear the mechanical load of the hardwoods, which
helps prevent the trees from toppling over in the dense and
unstable rainforest soil.

Important Tips
Tropical Rainforests:
Tropical rainforests are located near the equator between
the Tropic of Cancer (23°27’N) and the Tropic of
Capricorn (23°27’S).
• They experience consistently high temperatures
throughout the year.
• Tropical rainforests receive very high annual rainfall.
• The soils in tropical rainforests are nutrient-poor due
to the leaching of nutrients caused by heavy rains.
• The heavy rainfall washes away organic material
from the soil.
159. Solution: (c) • Tropical rainforests exhibit extremely high
Exp) Option c is the correct answer. biodiversity compared to other ecosystems.
The formation of the planetary wind system is influenced by • Some of the most common trees found in tropical
factors such as the earth’s rotation on its axis and the sun’s rainforest include- Mahogany, Teak, Rubber,
heating power. The sun’s radiation, absorbed differently on Rosewood, Rubber, and Balsa.

PYQ Workbook 130


GENERAL GEOGRAPHY

161. Solution: (a) between the latitudes of 40° and 50°. These winds are caused
Exp) Option a is the correct answer. by the combination of air being displaced from the Equator
towards the South Pole and the Earth’s rotation. The Earth’s
Statement 1 is incorrect- The wind direction in cyclones of
rotation deflects the winds to the west, creating the strong
the Northern Hemisphere is actually counter-clockwise, not
westerly winds.
clockwise.
Statement II is false- The strong east to west air currents are
Statement 2 is incorrect- Trade winds blow from the
caused by the combination of air being displaced from the
Northeast to the Southwest in the Northern Hemisphere
Equator towards the South Pole and the Earth’s rotation.
and from the Southeast to the Northwest in the Southern
The fact that there are few landmasses to serve as wind
Hemisphere.
breaks does contribute to the strength of the winds, but it is
Statement 3 is correct- In the anti-cyclone of the Southern not the primary cause
Hemisphere, the wind direction is counter-clockwise.
Anti-cyclones are high-pressure systems, and the air flows 164. Solution: (d)
outward in a counter-clockwise direction in the Southern Exp) Option d is the correct answer.
Hemisphere.
The Kuroshio Current begins off the coast of the Philippines
Statement 4 is correct- Both westerlies and trade winds and flows northeastward past Taiwan and Japan. The
do originate from sub-tropical high-pressure systems. Kuroshio is a warm ocean current that is often referred to as
Westerlies blow from the Southwest in the Northern the “Black Stream” due to its deep blue color.
Hemisphere and from the Northwest in the Southern
Hemisphere, while trade winds blow from the Northeast in Important Tips
the Northern Hemisphere and from the Southeast in the Ocean Currents across the world:
Southern Hemisphere.

162. Solution: (a)


Exp) Option a is the correct answer.
Statement I is correct- The Doldrums, also known as the
Intertropical Convergence Zone (ITCZ), is a region near
the Equator where the trade winds from the Northern and
Southern Hemispheres converge. Due to the convergence
of trade winds, the air becomes relatively calm, resulting in
light and variable winds.
Statement II is correct- The Equator receives direct and
intense sunlight, which heats the surface and causes the air
to rise. As the air rises, it creates a region of low pressure.
The rising air then moves towards the poles, resulting in the
northward and southward travel.

165. Solution: (a)


Exp) Option a is the correct answer.
The innermost region of a hurricane is known as the
hurricane eye. The eye is a relatively calm and clear area at
the center of the storm, surrounded by the eyewall where
the strongest winds and heaviest rainfall occur. Within the
eye, the temperature is generally the highest compared to
the rest of the storm, as the sinking air in this region leads to
warming. Additionally, the pressure in the eye is the lowest,
as the air is descending and creating a relatively calm and
stable environment.

166. Solution: (d)


Exp) Option d is the correct answer.
Option 1 is correct- The North Atlantic Current is a
163. Solution: (c)
warm current that originates in the Gulf Stream and
Exp) Option c is the correct answer. flows northward along the east coast of North America. It
Statement I is true- The Roaring Forties are strong westerly then turns eastward and flows across the Atlantic Ocean,
winds found in the Southern Hemisphere, generally eventually reaching the Sargasso Sea.

131 PYQ Workbook


GENERAL GEOGRAPHY

Option 2 is correct- The Canary Current is a cold current average annual temperature is around -49°C (-56°F) and
that originates in the north Atlantic Ocean and flows can drop to -89°C (-128°F) at the Vostok research station,
southward along the coast of Africa. It then turns westward which is the lowest temperature ever recorded on Earth.
and flows across the Atlantic Ocean, eventually reaching the C. Cherrapunji: Cherrapunji is a town in Meghalaya, India,
Sargasso Sea. that is one of the wettest places in the world. It has a
Option 3 is correct- The North Atlantic Equatorial Current subtropical highland climate with heavy monsoonal
is a warm current that originates in the Gulf of Guinea and rainfall. The average annual rainfall is around 11,777
flows eastward across the Atlantic Ocean. It then turns mm (463 in) and can reach up to 26,000 mm (1,024 in)
northward and flows across the Atlantic Ocean, eventually in some years. The town holds the record for the greatest
reaching the Sargasso Sea. rainfall in 48 hours, with 2,493 mm (98 in) of rain on 15-
16 June 1995.
Important Tips
D. Chile: Chile is a country in South America that lies along
The Sargasso Sea is surrounded by several currents that
the Pacific Ocean. It has a variety of climates, ranging
define its boundaries. It is bounded on the west by the
from desert to Mediterranean to temperate. The Atacama
Gulf Stream, on the north by the North Atlantic Current,
Desert in northern Chile is considered to be the driest
on the east by the Canary Current, and on the south by
place on Earth, with some weather stations that have
the North Atlantic Equatorial Current. These currents
never recorded any rainfall. The average annual rainfall
create a circular motion, forming a gyre, and contribute
in the Atacama Desert is less than 15 mm (0.6 in)
to the unique characteristics of the Sargasso Sea. A gyre
is a large system of circulating currents, and the Sargasso 169. Solution: (a)
Sea serves as an example of this phenomenon.
Exp) Option a is the correct answer.
167. Solution: (a) Dense forests on Earth are mostly found near the equator
Exp) Option a is the correct answer. because the equatorial region receives the most sunlight and
rainfall. Tropical rainforests appear dense because the tree
Capillaries are tiny pores in the soil that allow water to
grows at different heights i.e. in four distinct layers, each
move upwards against gravity. Capillarity is an important
with unique characteristics that interact with one another:
process for plant growth and soil water retention. The smaller
the soil particles, the smaller the capillary pores, and the Emergent Layer: The uppermost layer, receives lots of
more effective the capillaries are at moving water upwards. sunlight and rain and is dominated by towering trees like
Clayey soil has the smallest soil particles, followed by silt, white-tailed hawks and harpy eagles, and inhabited by
loam, and sand. Therefore, capillaries are most effective flying creatures such as birds, bats, and butterflies.
in clayey soil. Canopy Layer: Located just below the Emergent Layer,
this layer forms a dense leafy roof, blocking sunlight and
creating a humid, still environment. Trees in the Canopy
have adapted with glossy leaves that repel water.
Understory Layer: Darker, more humid, and quieter than
the Canopy Layer, it hosts shorter plants with larger leaves
and noticeable, fragrant flowers. This layer receives less
sunlight.
Forest Floor Layer: The darkest layer where plant growth
is limited. This layer teems with decomposers like slugs,
scorpions, worms, and fungi, which break down organic
matter from above, enriching the soil with nutrients.

168. Solution: (b)


Exp) Option b is the correct answer.
A. Sahara: The Sahara is the largest hot desert in the world,
covering most of North Africa. It has an arid climate with
very low rainfall and high temperatures. The average
annual temperature is around 30°C (86°F) and can reach
up to 58°C (136°F) in some areas.
B. Antarctica: Antarctica is the southernmost continent
and the coldest place on Earth. It has a polar climate
with extremely low temperatures and strong winds. The

PYQ Workbook 132


GENERAL GEOGRAPHY

170. Solution: (b) especially in the afternoon when the Sun’s heat is at its peak.
Exp) Option b is the correct answer. • The Mediterranean climate is characterized by dry and
Radiation is the transfer of heat energy through a vacuum. hot summers and mild and wet winters. The rainfall is
On a clear night, there are no clouds to block the Earth’s mostly concentrated in the winter months, while the
radiation from escaping into space. As a result, the Earth’s summer months are usually dry.
surface cools down. • Monsoon climate is marked by seasonal changes in
Clouds, on the other hand, act like a blanket and trap the wind direction and precipitation. The monsoon zone
Earth’s radiation. This prevents the Earth’s surface from typically experiences a wet season and a dry season,
cooling down as much as it would on a clear night. Therefore, depending on the movement of the monsoon winds. The
clear sky nights are cooler than cloudy sky nights due to wet season usually lasts from June to September, while
radiation. the dry season lasts from October to May.
• Desert climate is defined by very low rainfall and high
Important Tips
evaporation rates. The desert zone receives less than 25
Condensation is the process of water vapor turning into cm of rainfall per year, and most of it falls in sporadic
liquid water. It does not affect the temperature of the air. and unpredictable events. The rainfall pattern in the
Induction is the process of heat transfer between two desert zone does not depend on the seasons, but rather
objects that are not touching. It is not a major factor in the on the occurrence of rare atmospheric disturbances.
cooling of the Earth’s surface.
172. Solution: (a)
Conduction is the transfer of heat energy through direct
contact between two objects. It is not a major factor in the Exp) Option a is the correct answer.
cooling of the Earth’s surface, as the Earth’s atmosphere is Terra Rossa, also known as “red soil,” primarily forms
a poor conductor of heat. on limestone bedrock in karst regions. This soil is
characterized by its reddish color due to the preferential
171. Solution: (b)
formation of hematite over goethite. Terra Rossa has good
Exp) Option b is the correct answer. drainage properties and a neutral pH, making it suitable for
The equatorial zone is a region near the equator that wine production. It is found in Mediterranean regions and
experiences high temperatures and heavy rainfall throughout karst areas worldwide, including wine-growing regions like
the year. The rainfall is often accompanied by thunderstorms, La Mancha in Spain and various parts of Australia.

133 PYQ Workbook


GENERAL GEOGRAPHY

GENERAL GEOGRAPHY
INDIA PHYSICAL ENVIRONMENT
*This unit consists of questions from Location and Physiography of India, Drainage system, Indian
Climate, Natural Vegation and Soils in India.

5.1. UPSC CSE Previous Years’ Questions 5. Consider the following pairs:

1. Consider the following statements: Peak Mountains


1. Jhelum River passes through Wular Lake. 1. Namcha Barwa Garhwal Himalaya
2. Krishna River directly feeds Kolleru Lake. 2. Nanda Devi Kumaon Himalaya
3. Meadering of Gandak River formed 3. Nokrek Sikkim Himalaya
Kanwar Lake.
Which of the pairs given above is/are correctly
How many of the statements given above are matched? [UPSC CSE Pre 2022]
correct? [UPSC CSE Pre 2023]
(a) 1 and 2
(a) Only one (b) 2 only
(b) Only two
(c) 1 and 3
(c) All three
(d) 3 only
(d) None
2. Which one of the following is the best 6. With reference to the Indus River system,
example of repeated falls in sea level, giving of the following four rivers, three of them
rise to present-day extensive marshland? pour into one of them which joins the Indus
[UPSC CSE Pre 2023] direct. Among the following which is one is
(a) Bhitarkanika Mangroves such river that joins the Indus direct?
(b) Marakkanam Salt Pans [UPSC CSE Pre 2021]
(c) Naupada Swamp (a) Chenab
(d) Rann of Kutch (b) Jhelum
3. Consider the following statements: (c) Ravi
(d) Sutlej
[UPSC CSE Pre 2023]
1. Amakantak Hills are at the confluence of 7. With reference to India, Didwana
Vindhya and Sahyadri Ranges. Kuchaman, Sargol and Khatu are the names
2. Biligirirangan Hills constitute the of [UPSC CSE Pre 2021]
easternmost part of Satpura Range. (a) glaciers
3. Seshachalam Hills constitute the (b) mangrove areas
southernmost part of Western Ghats. (c) Ramsar sites
How many of the statements given above are (d) saline lakes
correct?
8. Consider the following rivers:
(a) Only one
1. Brahmani
(b) Only two
(c) All three 2. Nagawali
(d) None 3. Subarnarekha
4. Vamsadhara
4. Gandikota canyon of South India was
created by which one of the following rivers? Which of the above rise from the Eastern
[UPSC CSE Pre 2022] Ghats? [UPSC CSE Pre 2021]
(a) Manjira (a) 1 and 2
(b) Pennar (b) 2 and 4
(c) Cauvery (c) 3 and 4
(d) Tungabhadra (d) 1 and 3

PYQ Workbook 134


GENERAL GEOGRAPHY

9. The black cotton soil of India has been 14. Consider the following pairs:
formed due to the weathering of [UPSC CSE Pre. 2019]
[UPSC CSE Pre 2021]
Glacier River
(a) brown forest soil
(b) fissure volcanic rock 1. Bandarpunch Yamuna
(c) granite and schist 2. Bara Shigri Chenab
(d) shale and limestone 3. Milam Mandakini
10. Siachen Glacier is situated to the: 4. Siachen Nubra
[UPSC CSE Pre 2020] 5. Zemu Manas
(a) East of Aksai Chin Which of the pairs given above are correctly
(b) East of Leh matched?
(c) North of Gilgit (a) 1, 2 and 4 only
(d) North of Nubra Valley (b) 1, 3 and 4 only
11. With reference to Ocean Mean Temperature (c) 2 and 5 only
(OMT), which of the following statements (d) 3 and 5 only
is/are correct? [UPSC CSE Pre. 2020] 15. Which one of the following is an artificial
1. OMT is measured up to a depth of 26°C lake? [UPSC CSE Pre. 2018]
isotherm which is 129 meters in the south (a) Kodaikanal (Tamil Nadu)
-western Indian Ocean during January–
(b) Kolleru (Andhra Pradesh)
March.
(c) Nainital (Uttarakhand)
2. OMT collected during January –March (d) Renuka (Himachal Pradesh)
can be used in assessing whether the
amount of rainfall in monsoon will be less 16. Consider the following statements:
or more than a certain long -term mean. [UPSC CSE Pre. 2018]
Select the correct using the code given below: 1. The Barren Island volcano is an active
(a) 1 only volcano located in the Indian territory.
(b) 2 only 2. Barren Island lies about 140 km east of
(c) Both 1 and 2 Great Nicobar.
(d) Neither 1 nor 2 3. The last time the Barren Island volcano
12. Consider the following pairs: erupted was in 1991 and it has remained
inactive since then.
[UPSC CSE Pre. 2019]
Which of the statements given above is/are
Famous Place River correct?
1. Pandharpur Chandrabhaga (a) 1 only
2. Tiruchirappalli Cauvery (b) 2 and 3 only
3. Hampi Malaprabha (c) 3 only
(d) 1 and 3 only
Which of the pairs given above are correctly
matched? 17. Among the following cities, which one lies
(a) 1 and 2 only on a longitude closest to that of Delhi?
(b) 2 and 3 only [UPSC CSE Pre. 2018]
(c) 1 and 3 only (a) Bengaluru
(d) 1, 2 and 3 (b) Hyderabad
13. What is common to the places known as (c) Nagpur
Aliyar, Isapur and Kangsabati? (d) Pune
[UPSC CSE Pre. 2019] 18. At one of the places in India, if you stand
(a) Recently discovered uranium deposits on the seashore and watch the sea, you
(b) Tropical rain forests will find that the sea water recedes from
(c) Underground cave systems the shoreline a few kilometres and comes
(d) Water reservoirs back to the shore, twice a day, and you can

135 PYQ Workbook


GENERAL GEOGRAPHY

actually walk on the sea floor when the between tropical Western Indian Ocean
water recedes. This unique phenomenon is and tropical Eastern Pacific Ocean.
seen at- [UPSC CSE Pre 2017] 2. An IOD phenomenon can influence an El
(a) Bhavnagar Nino’s impact on the monsoon.
(b) Bheemunipatnam Select the correct answer using the code given
(c) Chandipur below:
(d) Nagapattinam (a) 1 only
19. Consider the following statements: (b) 2 only
[UPSC CSE Pre. 2017] (c) Both 1 and 2
1. In India, the Himalayas are spread over (d) Neither 1 nor 2
five states only. 23. In which of the following regions of India
2. Western Ghats are spread over five states are shale gas resources found?
only. [UPSC CSE Pre. 2016]
3. Pulicat Lake is spread over two States 1. Cambay Basin
only.
2. Cauvery Basin
Which of the statements given above is/are
3. Krishna-Godavari Basin
correct?
(a) 1 and 2 only Select the correct answer using the code
(b) 3 only given below.
(c) 2 and 3 only (a) 1 and 2 only
(d) 1 and 3 only (b) 3 only
(c) 2 and 3 only
20. If you travel by road from Kohima to (d) 1, 2 and 3
Kottayam, what is the minimum number
of States within India through which you 24. Recently, which of the following States has
can travel, including the origin and the explored the possibility of constructing an
destination? [UPSC CSE Pre. 2017] artificial inland port to be connected to sea
(a) 6 by a long navigational channel?
(b) 7 [UPSC CSE Pre. 2016]
(c) 8 (a) Andhra Pradesh
(d) 9 (b) Chhattisgarh
21. With reference to river Teesta, consider the (c) Karnataka
following statements:[UPSC CSE Pre. 2017] (d) Rajasthan
1. The source of river Teesta is the same as 25. Recently, linking of which of the following
that of Brahmaputra but it flows through rivers was undertaken?
Sikkim. [UPSC CSE Pre. 2016]
2. River Rangeet originates in Sikkim and it (a) Cauvery and Tungabhadra
is a tributary of river Teesta.
(b) Godavari and Krishna
3. River Teesta flows into Bay of Bengal on (c) Mahanadi and Son
the border of India and Bangladesh. (d) Narmada and Tapti
Which of the statements given above is/are
correct? 26. Which of the following is/are tributary/
(a) 1 and 3 only tributaries of Brahmaputra?
(b) 2 only [UPSC CSE Pre. 2016]
(c) 2 and 3 only 1. Dibang
(d) 1, 2 and 3 2. Kameng
22. With reference to ‘Indian Ocean Dipole 3. Lohit
(IOD)’ sometimes mentioned in the news Select the correct answer using the code
while forecasting Indian monsoon, which given below.
of the following statements is/are correct? (a) 1 only
[UPSC CSE Pre. 2017] (b) 2 and 3 only
1. IOD phenomenon is characterised by (c) 1 and 3 only
a difference in sea surface temperature (d) 1, 2 and 3

PYQ Workbook 136


GENERAL GEOGRAPHY

27. In India, in which one of the following types 32. In a particular region in India, the local
of forests is teak a dominant tree species? people train the roots of living trees into
[UPSC CSE Pre. 2015] robust bridges across the streams. As the
(a) Tropical moist deciduous forest time passes, these bridges become stronger.
(b) Tropical rain forest These unique ‘living root bridges’ are found
(c) Tropical thorn scrub forest in [UPSC CSE Pre. 2015]
(d) Temperate forest with grasslands (a) Meghalaya
28. Consider the following rivers: (b) Himachal Pradesh
(c) Jharkhand
[UPSC CSE Pre. 2015]
(d) Tamil Nadu
1. Vamsadhara
2. Indravati 33. Consider the following pairs:
3. Pranahita Hills Region
4. Pennar 1. Cardamom Hills Coromandel Coast
Which of the above are tributaries of
2. Kaimur Hills Konkan Coast
Godavari?
(a) 1, 2 and 3 only 3. Mahadeo Hills Central India
(b) 2, 3 and 4 only 4. Mikir Hills North-East India
(c) 1, 2 and 4 only Which of the above pairs are correctly
(d) 2 and 3 only matched? [UPSC CSE Pre 2014]
29. Consider the following States: (a) 1 and 2
[UPSC CSE Pre. 2015] (b) 2 and 3
1. Arunachal Pradesh (c) 3 and 4
2. Himachal Pradesh (d) 2 and 4
3. Mizoram 34. Which one of the following pairs of islands
In which of the above States do ‘Tropical Wet is separated from each other by the ‘Ten
Evergreen Forests’ occur? Degree Channel’? [UPSC CSE Pre. 2014]
(a) 1 only (a) Andaman and Nicobar
(b) 2 and 3 only (b) Nicobar and Sumatra
(c) 1 and 3 only (c) Maldives and Lakshadweep
(d) 1, 2 and 3 (d) Sumatra and Java
30. Which one of the following pairs of States 35. The seasonal reversal of winds is the typical
of India indicates the easternmost and characteristic of [UPSC CSE Pre. 2014]
westernmost State [UPSC CSE Pre. 2015] (a) Equatorial climate
(a) Assam and Rajasthan (b) Mediterranean climate
(b) Arunachal Pradesh and Rajasthan (c) Monsoon climate
(c) Assam and Gujarat
(d) All of the above climates
(d) Arunachal Pradesh and Gujarat
36. If you travel through the Himalayas, you are
31. Consider the following pairs:
likely to see which of the following plants
[UPSC CSE Pre. 2015]
naturally growing there?
Place of Pilgrimage Location [UPSC CSE Pre. 2014]
1. Srisailam Nallamala Hills 1. Oak
2. Omakareshwar Satmala Hills 2. Rhododendron
3. Pushkar Mahadeo Hills 3. Sandalwood
Which of the above pairs is/are correctly Select the correct answer using the code
matched ? given below.
(a) 1 only (a) 1 and 2 only
(b) 2 and 3 only (b) 3 only
(c) 1 and 3 only (c) 1 and 3 only
(d) 1, 2 and 3 (d) 1, 2 and 3

137 PYQ Workbook


GENERAL GEOGRAPHY

37. Consider the following rivers: (c) 1, 3 and 4 only


[UPSC CSE Pre. 2014] (d) 1, 2, 3 and 4
1. Barak 41. Which of the following statements regarding
2. Lohit laterite soils of India are correct?
3. Subansiri [UPSC CSE Pre. 2013]
Which of the above flows/flow through 1. They are generally red in colour.
Arunachal Pradesh? 2. They are rich in nitrogen and potash.
(a) 1 only 3. They are well-developed in Rajasthan and
(b) 2 and 3 only UP.
(c) 1 and 3 only
4. Tapioca and cashew nuts grow well on
(d) 1, 2 and 3
these soils.
38. With reference to ‘Changpa’ community of Select the correct answer using the codes
India, consider the following statements: given below.
[UPSC CSE Pre. 2014] (a) 1, 2 and 3
1. They live mainly in the State of (b) 2, 3 and 4
Uttarakhand (c) 1 and 4
2. They rear the Pashmina goats that yield a (d) 2 and 3 only
fine wool.
3. They are kept in the category of Scheduled 42. Consider the following statements:
Tribes. [UPSC CSE Pre. 2012]
Which of the statements given above is/are 1. The duration of the monsoon decreases
correct? from southern India to northern India.
(a) 1 only 2. The amount of annual rainfall in the
(b) 2 and 3 only northern plains of India decreases from
(c) 3 only east to west.
(d) 1, 2 and 3 Which of the statements given above is/are
39. The Narmada river flows to the west, while correct?
most other large peninsular rivers flow to (a) 1 only
the east. Why? [UPSC CSE Pre. 2013] (b) 2 only
1. It occupies a linear rift valley. (c) Both 1 and 2
2. It flows between the Vindhyas and the (d) Neither 1 nor 2
Satpuras. 43. When you travel in Himalayas, you will see
3. The land slopes to the west from Central the following: [UPSC CSE Pre. 2012]
India. 1. Deep gorges
Select the correct answer using the codes 2. U-turn river courses
given below. 3. Parallel mountain ranges
(a) 1 only 4. Steep gradients causing land-sliding
(b) 2 and 3
(c) 1 and 3 Which of the above can be said to be the
(d) None evidences for the Himalayas being young fold
mountains?
40. Consider the following pairs: (a) 1 and 2 only
[UPSC CSE Pre. 2013] (b) 1, 2 and 4 only
Tribe State (c) 3 and 4 only
(d) 1, 2, 3 and 4
1. Limboo (Limbu) Sikkim
44. A particular State in India has the following
2. Karbi Himachal Pradesh
characteristics: [UPSC CSE Pre. 2012]
3. Dongaria Odisha 1. It is located on the same latitude which
4. Bonda Tamil Nadu passes through northern Rajasthan.
Which of the above pairs are correctly 2. It has over 80% of its area under forest
matched? cover.
(a) 1 and 3 only 3. Over 12% of the forest over constitutes
(b) 2 and 4 only Protected Area Network in this State.

PYQ Workbook 138


GENERAL GEOGRAPHY

Which one among the following States has all 49. If there were no Himalayan ranges,
the above characteristics? what would have been the most likely
(a) Arunachal Pradesh geographical impact on India?
(b) Assam 1. Much of the country would experience
(c) Himachal Pradesh the cold waves from Siberia.
(d) Uttarakhand 2. Indo-Gangetic plain would be devoid of
45. The Brahmaputra, Irrawady and Mekong such extensive alluvial soils.
rivers originate in Tibet and flow through 3. The pattern of monsoons would be
narrow and parallel mountain ranges different from what it is at present.
in their upper reaches. Of these rivers, Which of the statements given above is/are
Brahmaputra makes a “U” turn in its course correct? [UPSC CSE Pre 2010]
to flow into India. This “U” turn is due to
(a) 1 only
[UPSC CSE Pre. 2011]
(b) 1 and 3 only
(a) Uplift of folded Himalayan series (c) 2 and 3 only
(b) Syntaxial bending of geologically young (d) 1, 2 and 3
Himalayas
(c) Geo-Tectonic disturbance in the tertiary 50. With reference to soil conservation,
folded mountain chains consider the following practices:
(d) Both (a) and (b) above [UPSC CSE Pre 2010]
46. A state in India has the following 1. Crop rotation
characteristics: [UPSC CSE Pre. 2011] 2. Sand fences
1. Its northern part is arid and semi-arid. 3. Terracing
2. Its central part produces cotton. 4. Windbreaks
3. Cultivation of cash crops is predominant Which of the above are considered appropriate
over food crops. methods for soil conservation in India?
Which one of the following states has all of (a) 1, 2 and 3
the above characteristics? (b) 2 and 4
(a) Andhra Pradesh (c) 1, 3 and 4
(b) Gujarat (d) 1, 2, 3 and 4
(c) Karnataka
(d) Tamil Nadu 51. When you travel in certain parts of India,
you will notice red soil. What is the main
47. Two important rivers- one with its source in reason for this colour?
Jharkhand (and known by a different name
[UPSC CSE Pre 2010]
in Odisha), and another, with its source
in Odisha-merge at a place only a short (a) Abundance of magnesium
distance from the coast of Bay of Bengal (b) Accumulated humus
before flowing into the sea. This is an (c) Presence of ferric oxides
important site of wildlife and biodiversity (d) Abundance of phosphates
and a protected area. Which one of the 52. In India, which type of forest among the
following could be this site? following occupies the largest area?
[UPSC CSE Pre. 2011] [UPSC CSE Pre 2010]
(a) Bhitarkanika (a) Montane Wet Temperate Forest
(b) Chandipur-on-sea (b) Sub-tropical Dry Evergreen Forest
(c) Gopalpur-on-sea
(c) Tropical Moist Deciduous Forest
(d) Simlipal
(d) Tropical Wet Evergreen Forest
48. Among the following States, which one has
the most suitable climatic conditions for 53. With reference to the river Luni, which one
the cultivation of a large variety of orchids of the following statements is correct?
with minimum cost of production, and can [UPSC CSE Pre 2010]
develop an export-oriented industry in this (a) It flows into the Gulf of Khambhat
field? [UPSC CSE Pre. 2011] (b) It flows into the Gulf of Kutch
(a) Andhra Pradesh (c) It flows into Pakistan and merges with a
(b) Arunachal Pradesh tributary of Indus
(c) Madhya Pradesh (d) It is lost in the marshy land of the Rann
(d) Uttar Pradesh of Kutch

139 PYQ Workbook


GENERAL GEOGRAPHY

54. Which one of the following rivers does not (c) 9


originate in India? [UPSC CSE Pre 2009] (d) 10
(a) Beas 60. In which state is the Guru Shikhar Peak
(b) Chenab located? [UPSC CSE Pre 2007]
(c) Ravi (a) Rajasthan
(d) Sutlej (b) Gujarat
55. At which one of the following places do (c) Madhya Pradesh
two important rivers of India originate; (d) Maharashtra
while one of them flows towards north 61. Which one among the following major
and merges with another important river Indian cities is most eastward located?
flowing towards the Bay of Bengal, the [UPSC CSE Pre 2007]
other one flows towards the Arabian Sea?
(a) Hyderabad
[UPSC CSE Pre 2009] (b) Bhopal
(a) Amarkantak (c) Lucknow
(b) Badrinath (d) Bangalore
(c) Mahabaleshwar
(d) Nasik 62. Consider the following statements:
Assertion (A): River Kalinadi is an east-
56. Consider the following statements: flowing river in the southern part of India.
1. There are no east flowing rivers in Kerala.
Reason (R): The Deccan Plateau is higher
2. There is no west flowing river in Madhya along its western edge and gently slopes
Pradesh. towards the Bay of Bengal in the east.
Which of the statement(s) given above is/are Code: [UPSC CSE Pre 2007]
correct? [UPSC CSE Pre 2009] (a) Both (A) and (R) are individually true,
(a) 1 only and (R) is the correct explanation of (A)
(b) 2 only (b) Both (A) and (R) are individually true,
(c) Both 1 and 2 but (R) is not the correct explanation of
(d) Neither 1 nor 2 (A)
57. Consider the following pairs: (c) (A) is true, but (R) is false
(d) (A) is false, but (R) is true
Tributary River Main River
63. Match list-I with list-II and select the
1. Chambal Narmada correct answer using the codes given below
2. Son Yamuna the lists:

3. Manas Brahmaputra List-I (Town) List-II (River nearby)


A. Betul 1. Indravati
Which of the pairs given above is/are correctly
matched? [UPSC CSE Pre 2008] B. Jagdalpur 2. Narmada
(a) 1, 2 and 3 C. Jabalpur 3. Kshipra
(b) 1 and 2 only D. Ujjain 4. Tapti
(c) 2 and 3 only Code: [UPSC CSE Pre 2007]
(d) 3 only
A B C D
58. Which of the following hills are found (a) 1 4 2 3
where the Eastern Ghats and the Western (b) 4 1 2 3
Ghats meet? [UPSC CSE Pre 2008] (c) 4 1 3 2
(a) Nilgiri Hills (d) 1 4 3 2
(b) Cardamom Hills
(c) Annamalai Hills 64. Which one of the following statements is
not correct? [UPSC CSE Pre 2006]
(d) Sahyadri Hills
(a) Mahanadi River rises from the
59. In India, how many States share the Chhattisgarh plateau.
coastline? [UPSC CSE Pre 2008] (b) Godavari River rises in Maharashtra.
(a) 7 (c) Cauvery River rises in Andhra Pradesh.
(b) 8 (d) Tapti River rises in Madhya Pradesh.

PYQ Workbook 140


GENERAL GEOGRAPHY

65. Consider the following statements: (a) Kochi – Kolkata – Delhi – Patna
Assertion (A): The percentage of net sown (b) Kolkata – Kochi – Patna – Delhi
area in the total area of Andhra Pradesh is (c) Kochi – Kolkata – Patna – Delhi
less as compared to that of West Bengal. (d) Kolkata – Kochi – Delhi – Patna
Reason (R): The soil of most of Andhra 70. Which one of the following statements is
Pradesh is laterite. not correct? [UPSC CSE Pre 2005]
Code: [UPSC CSE Pre 2006] (a) The Western Ghats are relatively high in
(a) Both (A) and (R) are individually true, their Northern region.
and (R) is the correct explanation of (A). (b) The Anamudi is the highest peak in the
(b) Both (A) and (R) are individually true. Western Ghats.
but (R) is not the correct explanation of (c) Tapti river lies to the south of Satpura.
(A). (d) The Narmada and the Tapti River valleys
(c) (A) is true, but (R) is false. are said to be old rift valleys.
(d) (A) is false, but (R) is true.
71. Which one of the following is the correct
66. Consider the following statements: sequence of the given hills starting from the
1. Assam shares border with Bhutan and north and going towards the south?
Bangladesh.
[UPSC CSE Pre 2005]
2. West Bengal shares border with Bhutan
and Nepal. (a) Nallamalai Hills – Nilgiri Hills – Javadi
Hills – Anaimalai Hills
3. Mizoram shares border with Bangladesh
and Myanmar. (b) Anaimalai Hills – Javadi Hills – Nilgiri
Hills – Nallamalai Hills
Which of the statements given above are (c) Nallamalai Hills – Javadi Hills – Nilgiri
correct? [UPSC CSE Pre 2006] Hills – Anaimalai Hills
(a) 1, 2 and 3 (d) Anaimalai Hills – Nilgiri Hills – Javadi
(b) 1 and 2 only Hills – Nallamalai Hills
(c) 2 and 3 only
(d) 1 and 3 only 72. Consider the following statements:
67. From north towards south, which one of 1. Silent Valley National Park is in the
the following is the correct sequence of the Nallamalla range.
given rivers in India? [UPSC CSE Pre 2006] 2. Pathrakkadavu Hydroelectric project is
(a) Shyok – Spiti – Zaskar – Satluj proposed to be built near the Silent Valley
(b) Shyok – Zaskar – Spiti – Satluj National Park.
(c) Zaskar – Shyok – Satluj – Spiti 3. The Kunthi river originates in Silent
(d) Zaskar – Satluj – Shyok – Spiti Valley’s rainforests.
68. Match list-I with list-II and select the Which of the statements given above is/are
correct answer using the codes given below correct? [UPSC CSE Pre 2005]
the lists: (a) 1 and 3 only
(b) 2 only
List-I (Valley) List-II (State) (c) 2 and 3 only
A. Markha Valley Sikkim (d) 1, 2 and 3
B. Dzukou Valley Himachal Pradesh 73. Nanda Devi peak forms a part of:
C. Sangla Valley Jammu and Kashmir [UPSC CSE Pre 2003]
D. Yumthang Valley Nagaland (a) Assam Himalayas
Code: [UPSC CSE Pre 2006] (b) Garhwal Himalayas
(c) Nepal Himalayas
A B C D (d) Punjab Himalayas
(a) 2 4 3 1
(b) 3 1 2 4 74. Consider the following statements
(c) 2 1 3 4 1. Longitude of Jabalpur’s location is
(d) 3 4 2 1 between those of Indore and Bhopal.
69. Which one of the following is the correct 2. Latitude of Aurangabad’s location is
sequence of the given Indian cities in the between those of Vadodara and Pune.
decreasing order of their normal annual 3. Bengaluru is situated more southward
rainfall? [UPSC CSE Pre 2005] than Chennai.

141 PYQ Workbook


GENERAL GEOGRAPHY

Which of these statements is/are correct?: (c) 2 5 3 4


[UPSC CSE Pre 2003] (d) 4 1 5 2
(a) 1 and 3 80. Identify the correct order of the process of
(b) Only 2 soil-erosion form the following:
(c) 2 and 3 [UPSC CSE Pre 2001]
(d) 1, 2 and 3
(a) Splash erosion, Sheet erosion, Rill
75. Among the following cities, which one is erosion, Gully erosion
nearest to the Tropic of Cancer? (b) Sheet erosion, Splash erosion, Gully
[UPSC CSE Pre 2003] erosion, Rill erosion
(a) Delhi (c) Rill erosion, Gully erosion, Sheet erosion,
(b) Kolkata Splash erosion
(c) Jodhpur (d) Gully erosion, Rill erosion, Sheet erosion,
(d) Nagpur Splash erosion
76. Which one of the following is not a lagoon? 81. Consider the climate diagram given below:
[UPSC CSE Pre 2002] [UPSC CSE Pre 2001]
(a) Ashtamudi lake
(b) Chilika lake
(c) Periyar lake
(d) Pulicat lake
77. The average annual temperature of a
meteorological station is 26 0C, its average
annual rainfall is 63 cm, and the annual
range temperature is 9 0C. The station in
question is: [UPSC CSE Pre 2002]
(a) Allahabad
(b) Chennai
(c) Cherrapunji
(d) Kolkata
78. Open stunted forests with bushes and small In the shaded area of the above map, the
trees having long roots and sharp thorns mean temperature for the month of July
repeated spines are commonly found in: varies between:
[UPSC CSE Pre 2002] (a) 22.50C - 25.00C
(a) Eastern Odisha (b) 25.00C - 27.50C
(b) North-Eastern Tamil Nadu (c) 27.50C - 30.00C
(c) Siwaliks and Terai region (d) 30.00C - 32.50C
(d) Western Andhra Pradesh 82. Which one of the following east flowing
79. Match list-I with list-II and select the rivers of India has rift valley due to down
correct answer using the codes given below warping? [UPSC CSE Pre 1998]
the lists: (a) Damodar
(b) Mahanadi
List-I (Mangroves) List-II (State) (c) Son
A. Achra Ratnagiri 1. Karnataka (d) Yamuna
B. Coondapur 2. Kerala 83. Some people in Manipur live in houses built
C. Pichavaram 3. Andhra Pradesh on floating islands of weeds and decaying
D. Vembanad 4. Maharashtra vegetation held together by suspended silt.
These islands are called-
5. Tamil Nadu [UPSC CSE Pre 1998]
Code: [UPSC CSE Pre 2002] (a) Tipis
A B C D (b) Burkhans
(a) 2 1 5 4 (c) Phumdi
(b) 4 5 3 2 (d) Izba

PYQ Workbook 142


GENERAL GEOGRAPHY

84. In the map shown in the given figure, rivers List-I List-II
labelled 1, 2, 3 and 4 respectively are:
A. Deccan Traps 1. Late Cenozoic
[UPSC CSE Pre 1997]
B. Western Ghats 2. Pre-Cambrian
C. Aravalli 3. Cretaceous
Eocene
D. Narmada-Tapti 4. Cambrian
alluvial deposits
5. Pleistocene
Code: [UPSC CSE Pre 1997]
A B C D
(a) 3 5 1 4
(b) 3 1 2 5
(a) Kosi, Gomati, Ghaghara and Gandak (c) 2 1 3 4
(b) Kosi, Ganga, Gomati and Ghaghara (d) 1 4 2 5
(c) Gandak, Ganga, Gomati and Ghaghara 87. Match list-I with list-II and select the
(d) Teesta, Gomati, Ghaghara and Kosi correct answer using the codes given below
85. Consider the map given below: the lists:
List-I (Climatic List-II (Reasons)
condition)
A. Chennai is 1. North-East
warmer than Monsoon
Kolkata
B. Snowfall in 2. Altitude
Himalaya
C. Rainfall 3. Western
decreases from depressions
West Bengal to
Punjab
D. Sutlej-Ganga 4. Distance from sea
plain gets some
rain in winter
5. Latitude
Code: [UPSC CSE Pre 1997]
The place marked A, B, C and D in the map A B C D
are respectively: [UPSC CSE Pre 1997]
(a) 1 2 4 5
(a) Rift valley region, Chhattisgarh plain, (b) 4 5 1 3
Rain shadow region and Chhota Nagpur (c) 5 2 4 3
plateau (d) 5 1 3 4
(b) Chhattisgarh plain, Chhota Nagpur
plateau, Rift valley region and Rain 88. “You might see a few curious Danes around,
shadow region but that is because ……. used to be Danish
(c) Rift valley region, Chhattisgarh plain, out post. This quaint town with its fort
Chhota Nagpur plateau and Rain shadow and a beautiful church, the New Jerusalem,
empty streets and deserted beach front is a
region
quaint gem.” The place referred to in this
(d) Chhattisgarh plain, Rain shadow region, quotation lies on the–
Chhota Nagpur plateau and Rift valley
[UPSC CSE Pre 1996]
region
(a) Tamil Nadu Coast
86. Match list-I with list-II and select the (b) Kerala Coast
correct answer using the codes given below (c) Karnataka Coast
the lists: (d) Goa Coast

143 PYQ Workbook


GENERAL GEOGRAPHY

89. “Saddle peak” the highest peak of Andaman 94. Examine the map of Jammu and Kashmir
& Nicobar is located in- given below: [UPSC CSE Pre 1995]
[UPSC CSE Pre 1996]
(a) Great Nicobar
(b) Middle Andaman
(c) Little Andaman
(d) North Andaman
90. Which one of the following rivers thrice
forks into two streams and reunites a few
miles farther on, thus forming the islands
of Srirangappattanam, Sivasamudram, and
Srirangam? [UPSC CSE Pre 1996]
(a) Cauvery The mountains ranges marked 1, 2, 3 and 4
(b) Tungabhadra respectively:
(c) Krishna (a) Ladakh, Zanskar, Karakoram and Pir
(d) Godavari Panjal
91. Consider the following rivers: (b) Karakoram, Pir Panjal, Zanskar and
1. Kishan Ganga Ladakh
2. Ganga (c) Karakoram, Ladakh, Zanskar and Pir
3. Wain Ganga Panjal
4. Pen Ganga (d) Ladakh, Pir Panjal, Karakoram and
The correct sequence of these rivers when Zanskar
arranged in the north-south direction is- 95. During ancient Indian historical geography,
[UPSC CSE Pre 1996] the term ‘Ratnakara’ denoted–
(a) 1, 2, 3, 4 [UPSC CSE Pre 1994]
(b) 2, 1, 3, 4 (a) The Arabian Sea
(c) 2, 1, 4, 3
(d) 1, 2, 4, 3 (b) The Bay of Bengal
(c) The Indian Ocean
92. High temperature and low pressure over the (d) The Confluence of the Ganga, the Jamuna
Indian Subcontinent during the summer and the mythical Saraswati in Prayag
season draws air from the Indian Ocean
leading to the in-blowing of the: 96. The Himalayas are formed of parallel fold
[UPSC CSE Pre 1996] ranges, of which the oldest range is–
(a) South-East monsoon [UPSC CSE Pre 1994]
(b) South-West monsoon (a) The Shiwalik Range
(c) Trade winds (b) The Lesser Himalayas
(d) Westerlies (c) The Great Himalayan Range
93. Consider the map given below: (d) The Dhauladhar Range
97. Consider the following statements:
[UPSC CSE Pre 1994]
Assertion (A): The Monsoonal rainfall
decreases as one goes toward the West and
North-West in the Ganga plain.
Reason (R): The moisture bearing Monsoonal
winds go higher as one moves up in the Ganga
plain.
In the context of the above two statements,
which one of the following is correct?
The divisions along India’s coastal region (a) Both (A) and (R) are true, and (R) is
indicate? [UPSC CSE Pre 1995] correct explanation of (A).
(a) Coastal pollution zones (b) Both (A) and (R) are true, but (R) is not
(b) Salinity density isopleths the correct explanation of (A).
(c) The areas upto which sovereignty extends (c) (A) is true, but (R) is false.
(d) Underwater relief contours (d) (A) is false, but (R) is true.

PYQ Workbook 144


GENERAL GEOGRAPHY

98. Consider the map of India given below: 103. Consider the climate diagram given below:
[UPSC CSE Pre 1994] [UPSC CSE Pre 1993]

The above diagram is related to the climate of


which of the following regions?
(a) North-West region of India
(b) South region of India
The map shows: (c) Central region of India
(a) An Isohyet (d) North-East region of India
(b) All India water divide
(c) A combination of national highways 104. Which among the following pairs of
(d) A combination of major highway route places have most marked differences in
total annual rainfalls even though located
99. Which one of the following soil types of approximately along the same latitude?
India is rendered infertile by the presence [UPSC CSE Pre 1993]
of excess iron? [UPSC CSE Pre 1994] (a) Bengaluru and Chennai
(a) Desert sand (b) Mumbai and Vishakhapatnam
(b) Alluvial (c) Ajmer and Shillong
(c) Podzolic (d) Nagpur and Kolkata
(d) Lateritic
105. A rainy day as defined by the Indian
100. Which one of the following crops enriches meteorological department is a day when
the Nitrogen content in soil? the rainfall at a point received is:
[UPSC CSE Pre 1994] [UPSC CSE Pre 1993]
(a) Potato (a) 0.5 mm to 1 mm in 24 hours.
(b) Sorghum (b) 1.1 mm to 1.5 mm in 24 hours.
(c) Sunflower (c) 1.6 mm to 2 mm in 24 hours.
(d) Pea (d) Above 2.5 mm in 24 hours.
101. Which one of the following types of erosion 106. Soils of western Rajasthan have a high
is responsible for the formation of Chambal content of: [UPSC CSE Pre 1993]
Ravines? [UPSC CSE Pre 1994] (a) Aluminum
(a) Splash (b) Calcium
(b) Sheet (c) Nitrogen
(c) Rill (d) Phosphorus
(d) Gully 107. A fertile soil, suitable for growing common
102. Farmland tends to erode most rapidly when crops is likely to have a pH value of:
planted with: [UPSC CSE Pre 1994] [UPSC CSE Pre 1993]
(a) Sorghum (a) Three
(b) Potato (b) Four
(c) Wheat (c) Six to seven
(d) Clover (d) Nine to ten

145 PYQ Workbook


GENERAL GEOGRAPHY

5.2. Other Examination Previous Years’ (b) Both (A) and (R) are true but (R) is not
the correct explanation of (A)
Questions
(c) (A) is false but (R) is true
108. With reference to India, which of the (d) (A) is true but (R) is false
following statements is/are correct?
111. Match list-I with list-II and select the
1. India is the sixth largest country of the correct answer using the codes given below
world. the lists:
2. India occupies about 2.4% of the total
area of the world. List-I (State of India) List-II (Highest
3. The Tropic of Cancer passes through the Peak)
middle of the country dividing into two A. Tamil Nadu 1. Dhupgarh Peak
latitudinal halves. B. Rajasthan 2. Saramati Peak
4. India lies completely in tropic zone. C. Nagaland 3. Gurushikhar
Select the correct answer from the code given Peak
below: [U.P.P.C.S. (Pre) 2022]
D. Madhya Pradesh 4. Doddabetta
(a) 2 and 4 Peak
(b) 1 and 2 Code: [UPPCS (Pre) 2021]
(c) 2 and 3
(d) 3 and 4 A B C D
(a) 3 2 1 4
109. Match list-I with list-II and select the (b) 1 4 3 2
correct answer using the codes given below (c) 4 2 3 1
the lists: (d) 4 3 2 1
List-I List-II 112. Which one of the following rivers is not the
(Waterfalls of India) (Location) part of Indian Ganga River basin?
A. Dudhsagar 1. Karnataka [U.P.P.C.S. (Pre) 2021]
B. Barkana 2. Odisha (a) Punpun river
(b) Ajoy river
C. Khandadhar 3. Himachal (c) Jalangi river
Pradesh (d) Jonk river
D. Palani 4. Goa
113. Largest saline water lake in India is:
Code: [UPPCS (Pre) 2022] [U.P.P.C.S. (Pre) 2021]
A B C D (a) Chilika
(a) 4 3 2 1 (b) Sambhar
(b) 4 1 2 3 (c) Lonary
(c) 3 2 4 1 (d) Wullar
(d) 1 2 3 4
114. Given below are two statements, one is
110. Given below are two statements, one is labelled as Assertion (A) and other as
labelled as Assertion (A) and the other as Reason (R):
Reason (R). Assertion (A): Teesta River was earlier a
Assertion (A): Agra and Darjeeling are located tributary of Ganga now it is a tributary of
on the same latitude, but the temperature in Brahmaputra.
January in Agra is 160 Centigrade whereas it Reason (R): River capturing is a major feature
is only 40 Centigrade in Darjeeling. of Himalayan rivers.
Reason (R): Temperature decreases with Select the correct answer using the codes
height and due to thin air, places in the given below. [U.P.P.C.S. (Pre) 2021]
mountains are cooler than places in the (a) Both (A) and (R) are true and (R) is
plains. correct explanation of (A).
Select the correct answer from the code given (b) Both (A) and (R) are true, but (R) is not
below. [UPPCS (Pre) 2022] the correct explanation of (A).
(a) Both (A) and (R) are true and (R) is the (c) (A) is true, but (R) is false.
correct explanation of (A) (d) (A) is false, but (R) is true.

PYQ Workbook 146


GENERAL GEOGRAPHY

115. Match list-I with list-II and select the (c) Glacial plain
correct answer using the codes given below (d) Alluvial plain
the lists: 119. With reference to the Himalayan range,
List-I (Lake) List-II (Location) which of the statements is/are correct?
A. Sala Lake 1. Arunachal Pradesh 1. The sedimentary rocks of the greater
Himalayas were fossil less.
B. Badkhal Lake 2. Haryana
2. Marine livings fossils are found in the
C. Loktak Lake 3. Manipur sedimentary rocks of lesser Himalayas.
D. Kaliveli Lake 4. Tamil Nadu 3. Remains of human civilization are found
Code: [UPPCS (Pre) 2020] in the outer or Shivalik Himalayas.
A B C D Select the correct answer using the codes
(a) 1 2 3 4 given below: [U.P.P.C.S. (Pre) 2019]
(b) 2 1 3 4 (a) 1 and 2 only
(c) 1 3 2 4 (b) 2 and 3 only
(c) 1 and 3 only
(d) 1 4 2 3
(d) 1, 2 and 3 are correct
116. Consider the following tributaries of River 120. Which one of the following is the correct
Yamuna and arrange them from West to sequence of the hills of Central India
East: located from West to East?
1. Betwa [U.P.P.C.S. (Pre) 2019]
2. Ken (a) Maikal, Satpura, Mahadeo and
3. Sindh Chhotanagpur
4. Chambal (b) Satpura, Mahadeo, Maikal and
Chhotanagpur
Select the correct answer from the codes
(c) Maikal, Mahadeo, Satpura and
given below. [U.P.P.C.S. (Pre) 2020]
Chhotanagpur
(a) 4, 3, 1 and 2 (d) Satpura, Mahadeo, Chhotanagpur and
(b) 1, 2, 3 and 4 Maikal
(c) 3, 2, 1 and 4
121. In which of the following rivers, the upper
(d) 2, 3, 1 and 4
course contains freshwater but saline water
117. Given below are two statements, one is flowing at the lower part?
labelled as Assertion (A) and the other as [U.P.P.C.S. (Pre) 2019]
Reason (R). (a) Barak river
Assertion (A): The highest concentration of (b) Luni river
Seismic zones lies in the Himalayan region in (c) Ghaggar river
India. (d) None of the above
Reason (R): There are many longitudinal 122. The winter rains caused by Western
thrust zones in the Himalayas. disturbance in North-Western Plain of
India gradually decreases from:
Select the correct answer from the codes
[U.P.P.C.S. (Pre) 2019]
given below: [U.P.P.C.S. (Pre) 2019]
(a) East to West
(a) Both (A) and (R) are true and (R) is the
(b) West to East
correct explanation of (A). (c) North to South
(b) Both (A) and (R) are true, but (R) is not (d) South to North
the correct explanation of (A).
(c) (A) is true, but (R) is false 123. Which of the following States of India has
(d) (A) is false, but (R) is true. the longest coastline?
[U.P.P.C.S. (Pre) 2018]
118. ‘Imphal Basin’ surrounded by Manipur hills (a) Maharashtra
is a fine example of- [UPPCS (Pre) 2019] (b) Andhra Pradesh
(a) Lacustrine plain (c) Kerala
(b) Loess plain (d) Gujarat

147 PYQ Workbook


GENERAL GEOGRAPHY

124. Match list-I with list-II and select the (c) 2 4 1 3


correct answer using the codes given below (d) 2 3 4 1
the lists: 128. In which state is the Nelang Valley located?
List-I (States) List-II (Highest Peak) [U.P.P.C.S. (Pre) 2016]
A. Kerala 1. Doddabetta (a) Himachal Pradesh
(b) Sikkim
B. Nagaland 2. Nanda Devi
(c) Jammu and Kashmir
C. Uttarakhand 3. Anamudi (d) Uttarakhand
D. Tamil Nadu 4. Saramati 129. River Ganga, after entering Bangladesh, is
Code: [UPPCS (Pre) 2018] known by which of the following names?
A B C D [U.P.P.C.S. (Mains) 2016]
(a) 1 3 4 2 (a) Lohit
(b) 2 3 4 1 (b) Padma
(c) 3 4 2 1 (c) Kaliganga
(d) 1 2 3 4 (d) Nabaganga
125. Which of the following statements is not 130. Which one of the following is not correctly
correct? [U.P.P.C.S. (Pre) 2018] matched? [U.P.P.C.S. (Mains) 2015]
(a) Black soil is locally called ‘regur’ (a) Chang La - Jammu and Kashmir
(b) According to Krabs, the regur soil is (b) Rohtang - Himachal Pradesh
essentially a mature soil (c) Bomdi La - Arunachal Pradesh
(c) Black soils are highly retentive to moisture (d) Se La - Uttarakhand
(d) Black soil is found in the Himalayan 131. Kuttanad (or Kuttanadu) of Kerala is
region famous for: [U.P.P.C.S. (Mains) 2015]
126. The Tropic of cancer passes through which (a) A freshwater lake
of the following states? (b) The region with the lowest altitude in
1. Gujarat India
(c) A coral island
2. Chhattisgarh
(d) The westernmost point of India
3. Uttar Pradesh
4. Jharkhand 132. Which one of the following rivers is called
the ‘Ganga of the South’?
Select the correct answer from the code given
[U.P.P.C.S. (Pre) 2015]
below: [U.P.P.C.S. (Pre) 2017]
(a) Cauvery
Code: (b) Krishna
(a) 1, 2 and 4 (c) Godavari
(b) 1, 2, 3 and 4 (d) Narmada
(c) 1, 3 and 4
(d) 2, 3 and 4 133. Match list-I with list-II and select the
correct answer using the codes given below
127. Match list-I with list-II and select the the lists:
correct answer using the codes given below
the lists: List-I (Lakes) List-II (Location)
A. Ashtamudi 1. Haryana
List-I (Waterfall) List-II (Rivers)
B. Pulicat 2. Kerala
A. Dudhsagar 1. Ghatprabha
C. Roopkund 3. Tamil Nadu
B. Duduma 2. Machhkund
D. Surajkund 4. Uttarakhand
C. Gokak 3. Sharavati
Code: [UPPCS (Pre) (Re-Exam) 2015]
D. Jog 4. Mandavi
A B C D
Code: [UPPCS (Pre) 2017] (a) 3 2 1 4
A B C D (b) 2 3 4 1
(a) 4 2 1 3 (c) 4 2 3 1
(b) 4 2 3 1 (d) 1 4 2 3

PYQ Workbook 148


GENERAL GEOGRAPHY

134. Which of the following soils of India has Code: [U.P.P.C.S. (Mains) 2011]
been formed due to weathering of basaltic (a) Both (A) and (R) are true, and (R) is the
lava? [U.P.P.C.S. (Pre) (Re-Exam) 2015] correct explanation of (A)
(a) Alluvial Soils (b) Both (A) and (R) are true, but (R) is not
(b) Laterite Soils the correct explanation of (A)
(c) Red Soils (c) (A) is true, but (R) is false
(d) Regur Soils (d) (A) is false, but (R) is true
140. The Indian Standard Time Meridian does
135. Consider the following statements:
not pass through- [U.P.P.C.S. (Pre) 2010]
Assertion (A): The rivers of Western Ghats (a) Andhra Pradesh
do not form the delta. (b) Chhattisgarh
Reason (R): They flow through short courses (c) Maharashtra
with high speed over hard rocks. (d) Uttar Pradesh
Select the correct answer from the codes 141. Which soil needs little irrigation as it
given below: [U.P.P.C.S. (Pre) 2014] retains soil water? [U.P.P.C.S. (Pre) 2010]
(a) Both (A) and (R) are correct, and (R) is (a) Red
the correct explanation of (A) (b) Black
(b) Both (A) and (R) are correct, but (R) is (c) Laterite
not the correct explanation of (A) (d) Alluvial
(c) (A) is true, but (R) is false 142. Which of the following is the highest
(d) (A) is false, but (R) is true mountain peak in India?
136. The rivers of South India mainly have which [U.P.P.C.S. (Mains) 2009]
of the following drainage patterns? (a) Everest
(b) Siachen
[U.P.P.C.S. (Pre) 2014] (c) K2
(a) Radial (d) Kargil
(b) Intermittent
143. The Hagari river is a tributary of:
(c) Dendritic
(d) Trellis [U.P.P.C.S. (Mains) 2009]
(a) Bhima
137. Which of the following is correctly matched? (b) Godavari
[UPPCS (Pre) 2012] (c) Krishna
Lakes – Locations (d) Tungabhadra
(a) Lonar – Madhya Pradesh 144. Which one of the following states of India
(b) Nakki – Gujarat has an international boundary on its three
(c) Kolleru – Andhra Pradesh sides? [U.P.P.C.S. (Spl.) (Mains) 2008]
(d) Pulicat – Kerala (a) Assam
(b) Nagaland
138. Kori Creek lies in– (c) Tripura
[U.P.P.C.S. (Mains) 2011] (d) West Bengal
(a) Gulf of Kutch 145. Which one of the following gives correct
(b) Gulf of Khambhat sequences of Himalayan peaks in the
(c) Little Rann of Kutch direction east to west?
(d) Rann of Kutch [U.P.P.C.S. (Spl) (Mains) 2008]
139. Consider the following statements and (a) Dhaulagiri, Kanchenjunga, Makalu,
select the correct answer from the code Mount Everest
given below: (b) Namcha Barwa, Kanchenjunga, Nanda
Devi, Mount Everest
Assertion (A): All rivers originating from the
(c) Makalu, Dhaulagiri, Kumaun, Namcha
Himalayas are perennial. Barwa
Reason (R): Himalayas receive much of their (d) Namcha Barwa, Kanchenjunga, Mount
precipitation from South-Western monsoon. Everest, Nanda Devi

149 PYQ Workbook


GENERAL GEOGRAPHY

146. Match list-I with list-II and select the the Narmada and the Tapi flow into the
correct answer using the codes given below Arabian sea.
the lists: Reason (R): The Narmada and the Tapi rivers
List-I (Sea Beach) List-II (State) flow in rift valley.
A. Digha 1. Tamil Nadu Select the correct answer using the codes
given below: [U.P.P.C.S. (Mains) 2002]
B. Gopalpur 2. West Bengal
(a) Both (A) and (R) are true, and (R) is the
C. Calangute 3. Orissa correct explanation of (A).
D. Marina 4. Goa (b) Both (A) and (R) are true, but (R) is not
Code: [UPPCS (Spl) (Mains) 2008] the correct explanation of (A).
A B C D (c) (A) is true, but (R) is false.
(a) 1 2 4 3 (d) (A) is false, but (R) is true.
(b) 2 3 4 1 153. Which of the following soil is most suited
(c) 2 1 3 4 for tea plantation? [U.P.P.C.S. (Pre) 2002]
(d) 4 3 2 1
(a) Acidic
147. Which of the following lake is shared by two (b) Alkaline
Indian States? [U.P.P.C.S. (Mains) 2008] (c) Alluvial
(a) Chilika (d) Regur
(b) Kolleru
(c) Lonar 154. Match list-I with list-II and select the
(d) Pulicat correct answer using the codes given below
the lists:
148. Which of the following rivers is an example
of the superimposed river? List-I (Forest) List-II (Region)
[U.P.P.C.S. (Mains) 2007] A. Tropical moist 1. Arunachal
(a) Alaknanda deciduous Pradesh
(b) Kosi B. Tropical dry 2. Sahyadris
(c) Chambal deciduous
(d) Godavari
C. Alpine 3. Middle Ganga
149. Which of the following is the highest peak Plain
in South India? [U.P.P.C.S. (Pre) 2005] D. Tropical 4. Tarai
(a) Anamudi evergreen
(b) Doddabetta
Code: [UPPCS (Pre) 2001]
(c) Amarkantak
(d) Mahendragiri A B C D
(a) 4 3 1 2
150. Chaurabari Glacier is located towards-
(b) 4 2 1 3
[U.P.P.C.S. (Mains) 2004]
(c) 1 3 2 4
(a) South of Kedarnath temple (d) 3 1 4 2
(b) West of Kedarnath temple
(c) North of Kedarnath temple 155. Consider the following statements:
(d) East of Kedarnath temple [U.P.P.C.S. (Pre) 1999]
151. When there is midnight at IST meridian, Assertion (A): The Regur soils of the Deccan
a place observes 6 A.M. The meridian on Trap are black in colour.
which the said place is located is: Reason (R): They are rich in Humus.
[U.P.P.C.S. (Mains) 2002] Select the correct answer from the codes
(a) 7° 31’ E given below:
(b) 172° 30’ E
(a) Both (A) and (R) are true, and (R) is the
(c) 7° 30’ E
(d) 127° 30’ E correct explanation of (A).
(b) Both (A) and (R) are true, but (R) is not
152. Consider the following statements: the correct explanation of (A).
Assertion (A): While all major rivers of (c) (A) is true, but (R) is false.
peninsular India flow into the Bay of Bengal, (d) (A) is false, but (R) is true.

PYQ Workbook 150


GENERAL GEOGRAPHY

156. Which one of the following towns is nearest 163. Examine the following statements and
to the Tropic of Cancer: select the correct answer by using codes
[U.P.P.C.S. (Pre) 1999] given below:
(a) Agartala 1. Devprayag is located at the confluence of
(b) Gandhinagar the Bhagirathi and Alaknanda rivers.
(c) Jabalpur 2. Rudraprayag is located at the confluence
(d) Ujjain of the Mandakini and Alaknanda rivers.
157. The paleomagnetic results obtained from 3. Alaknanda flows from Badrinath.
India indicate that in the past, the Indian 4. Kedarnath is considered the biggest seat
land mass has moved: established by Adi Shankaracharya.
[U.P.P.C.S. (Pre) 1998] Code: [U.P. Lower Sub. (Pre) 2002]
(a) Northward (a) 1, 2 and 3
(b) Southward
(b) 2, 3 and 4
(c) Eastward
(d) Westward (c) 1, 2 and 4
(d) 1, 2, 3 and 4
158. Vembanad lake is in: [U.P.P.C.S. (Pre) 1997]
(a) Andhra Pradesh 164. Compare the Himalayan Rivers with the
(b) Kerala Peninsular Rivers based on the following
(c) Orissa comparisons:
(d) Tamil Nadu 1. Most of the Himalayan rivers are
perennial, whereas most of the Peninsular
159. Which of the following option represents rivers are rain fed.
correct order of mountain Passes from west
to east? 2. The gradient of the Himalayan River is
steeper than the Peninsular River.
I. Shipki La
3. The Peninsular River causes more
II. Nathu La erosion on its way in comparison to the
III. Bomdi La Himalayan River.
IV. Lipulekh Choose the correct answer from the options
Code: [U.P. Lower Sub. (Pre) 2009] given below. [67th BPSC (Pre) 2022]
(a) I, II, III, IV (a) 1 and 2 only
(b) II, III, IV, I (b) 2 and 3 only
(c) I, IV, II, III (c) 1 and 3 only
(d) III, II, I, IV
(d) 1, 2 and 3
160. Which of the following is known as ‘self- (e) None of the above/More than one of the
ploughed’ soil of the country? above
[U.P. Lower Sub. (Pre) 2009]
165. The rivers Damodar, Koel and Subarnarekha
(a) Alluvial soil
originate from which of the following?
(b) Lateritic soil
(c) Black cotton soils [67th BPSC (Pre) 2022]
(d) Desert soils (a) Deccan Plateau
(b) Central Highland
161. Identify the only tributary of River Ganges
(c) Chhota Nagpur Plateau
which rises in the plains.
(d) Meghalaya Plateau
[U.P. Lower Sub. (Pre) 2008]
(e) None of the above/More than one of the
(a) Son above
(b) Sharda or Saryu
(c) Gomti 166. Which is Asia’s largest and freshwater
(d) Ram Ganga Oxbow Lake in Bihar?
162. The Kullu Valley is situated between the [67th BPSC (Pre) (Re-Exam) 2022]
mountain ranges of: (a) Ghogha Lake
[U.P. Lower Sub. (Spl.) (Pre) 2002] (b) Kanwar Lake
(a) Dhauladhar and Pir Panjal (c) Anupam Lake
(b) Ranjoti and Nagtibba (d) Kusheshwar Lake
(c) Ladakh and Pir Panjal (e) None of the above/More than one of the
(d) Middle Himalayas and Siwalik above

151 PYQ Workbook


GENERAL GEOGRAPHY

167. Rajasthan receives very little rain because: (c) Snowfall in Shimla
[67th BPSC (Pre) (Re-Exam) 2022] (d) Pollution in Amritsar
(a) the monsoon fails to reach this area 173. Which soil particles are present in loamy
(b) it is too hot soils? [53rd to 55th B.P.S.C. (Pre) 2011]
(c) there is no water available and thus the (a) Sand particles
winds remain dry (b) Clay particles
(d) the winds do not come across any barriers (c) Silt particles
to cause the necessary uplift to cool the (d) All types of particles
air
(e) None of the above/More than one of the 174. Consider the following statements
above regarding formation of landforms in India:
1. Structurally, the Meghalaya plateau is an
168. For which cultivation Karewas are famous? extended part of the Deccan plateau.
[67th BPSC (Pre) (Re-Exam) 2022] 2. The Valley of Kashmir was formed in a
(a) Banana synclinorium.
(b) Saffron 3. The Gangetic plain was formed in a fore
(c) Mango deep.
(d) Grapes 4. The Himalayas originated as a result of
(e) None of the above/More than one of the triangular convergence of the Indian
above plate, the European plate and the Chinese
169. Among the following tributaries, which one plate.
is part of the Ganga River basin? Which of these statements are correct?
[65th B.P.S.C. (Pre) 2019] [47th B.P.S.C. (Pre) 2005]
(a) Sankh (a) I, II and III
(b) North Koel (b) I, III and IV
(c) South Koel (c) I and III
(d) Barakar (d) II and IV
(e) None of the above/More than one of the 175. Extra peninsular mountains of India were
above formed during- [41st B.P.S.C. (Pre) 1996]
170. Coral reefs are not found in which one of (a) Eozoic Era
the following regions? (b) Palaeozoic Era
[65th B.P.S.C. (Pre) 2019] (c) Mesozoic Era
(a) Gulf of Cambay (d) Cenozoic Era
(b) Gulf of Mannar 176. Old Kachhari clay of Gangetic plain is
(c) Gulf of Kutch called- [41st B.P.S.C. (Pre) 1996]
(d) Lakshadweep and Minicoy Island (a) Bhabar
(e) None of the above/More than one of the (b) Bhangar
above (c) Khadar
171. Which one of the following is not a part of (d) Khondolyte
the Meghalaya Plateau? 177. Chotanagpur Plateau–
[64th B.P.S.C. (Pre) 2018] [40th B.P.S.C. (Pre) 1995]
(a) Bhuban Hills (a) Is a front sloping
(b) Garo Hills (b) Is a pitfall
(c) Khasi Hills (c) Is a foothill
(d) Jaintia Hills (d) Is a plain subland
(e) None of the above/More than one of the
178. Near Manasarovar Lake in Tibet, there is
above the source of the river-
172. Amritsar and Shimla are almost on the [39th B.P.S.C. (Pre) 1994]
same latitude, but their climate difference (a) Brahmaputra
is due to- [53rd to 55th B.P.S.C. (Pre) 2011] (b) Satluj
(a) The difference in their altitudes (c) Indus
(b) Their distance from sea (d) All of the above

PYQ Workbook 152


GENERAL GEOGRAPHY

179. The Indian monsoon is indicated by (a) Bhander and Maikal


seasonal displacement because of- (b) Satpura and Aravalli
[39th B.P.S.C. (Pre) 1994] (c) Satpura and Vindhyachal
(a) Differential temperature of land and sea (d) Vindhyachal and Aravalli
(b) Cold wind of middle Asia 186. Which one of the following Indian states
(c) Excess similarity of temperature does not share border with Bhutan?
(d) None of the above
[M.P.P.C.S. (Pre) 2012]
180. Match list-I with list-II and select the (a) Sikkim
correct answer using the codes given below (b) Meghalaya
the lists: (c) Arunachal Pradesh
List-I (Tributaries) List-II (Rivers) (d) West Bengal
A. Betwa 1. Chambal 187. The general direction of flow of summer
B. Kshipra 2. Yamuna monsoon in India is- [M.P.P.C.S. (Pre) 2012]
(a) from South to North
C. Wain Ganga 3. Narmada
(b) from South-West to South-East
D. Tawa 4. Godavari (c) from South-East to South-West
Code: [MPPCS (Pre) 2022] (d) from South-West to North-East
A B C D 188. The westernmost point of India is:
(a) 3 4 2 1 [M.P.P.C.S. (Pre) 2008]
(b) 2 1 4 3
(a) 68° 7’ W, Gujarat
(c) 3 4 1 2
(d) 1 3 2 4 (b) 68° 7’ W, Rajasthan
(c) 68° 7’ E, Gujarat
181. Which valley of the following rivers is (d) 68° 7’ E, Rajasthan
known to have deep Ravines?
189. The Southernmost point of India “Indira
[M.P.P.C.S. (Pre) 2019]
Point” is in: [M.P.P.C.S. (Pre) 2006]
(a) Narmada
(b) Son (a) Tamil Nadu
(c) Chambal (b) Little Nicobar
(d) Tapti (c) Great Nicobar
(d) Car Nicobar Island
182. The boundary line between India and
Pakistan is an example of- 190. Which one of the following can be used to
[M.P.P.C.S. (Pre) 2016] make acidic soil cultivable?
(a) Superimposed boundary [M.P.P.C.S. (Pre) 2006]
(b) Antecedent boundary (a) Lime
(c) Relict boundary (b) Gypsum
(d) Subsequent boundary (c) Calcium
(d) Vegetable compost
183. Which range acts as a water divide between
Yamuna and Son? [M.P.P.C.S. (Pre) 2016] 191. Which of the following separates India and
(a) Bhander Sri Lanka? [M.P.P.C.S. (Pre) 1990]
(b) Kaimur (a) Suez Canal
(c) Maikal (b) Palk Strait Bay
(d) Mukundwara (c) Gulf of Khambhat
184. Sundarbans Delta is formed by the rivers- (d) Gulf of Kutch
[M.P.P.C.S. (Pre) 2014] 192. The dust storms originating in Rajasthan in
(a) Ganga and Brahmaputra May and June months are caused due to-
(b) Ganga and Jhelum 1. Origin of convectional currents at few
(c) Sindhu and Jhelum places
(d) Ganga and Sindhu 2. Aravalli hills are parallel to south-western
185. The Narmada Valley lies between which of winds
the following mountain ranges? 3. Origin of fast speedy easterly winds
[M.P.P.C.S. (Pre) 2014] Code: [R.A.S/R.T.S. (Pre) 2018]

153 PYQ Workbook


GENERAL GEOGRAPHY

(a) 1 and 3 only iii. Godavari


(b) 1, 2 and 3 iv. Narmada
(c) 1 and 2 only Code: [Chhattisgarh P.C.S. (Pre) 2020]
(d) 1 only
(a) (iv), (i), (ii), (iii)
193. Palghat is situated between: (b) (ii), (iii), (i), (iv)
[R.A.S./R.T.S. (Pre) 2013] (c) (i), (ii), (iv), (iii)
(a) The Nilgiris and the Cardamom Hills (d) (iii), (ii), (iv), (i)
(b) The Nilgiris and the Annamalai Hills 199. Choose the correct statement out of the
(c) The Annamalai Hills and the Cardamom four statements given regarding alluvial
Hills soil: [Chhattisgarh P.C.S. (Pre) 2020]
(d) The Cardamom Hills and Palani Hills (a) This soil is found in about 14% portion of
194. Winter rainfall in Tamil Nadu is mostly due India’s land surface.
to- [R.A.S./R.T.S. (Pre) 2010] (b) Phosphoric acid is found in rich quantities
(a) Western disturbances in this soil.
(b) South-West monsoon (c) This soil is also known by names such as
(c) North-East monsoon Reh, Thur and Chopan.
(d) South-East monsoon (d) This soil is not fertile.
195. Which one of the following is called ‘flame 200. Which religious place is situated at the
of the forest’? [R.A.S./R.T.S. (Pre) 2010] confluence of Mand and Mahanadi?
(a) Bouhinia variegata [Chhattisgarh PCS (Pre) 2019]
(b) Jacaranda mimosafolia (a) Didineshwari Devi
(c) Butea monosperma (b) Angar Moti
(d) Tectona grandis (c) Chandrahasini Devi
196. Which of the following Indian state shares (d) Mahamaya
maximum boundary with Myanmar? 201. Which of the following regions is not
[R.A.S./R.T.S. (Pre) 2007] affected much by the Arabian Sea branch of
(a) Manipur Monsoon? [Chhattisgarh P.C.S. (Pre) 2016]
(b) Arunachal Pradesh (a) The Western Ghat
(c) Mizoram (b) Deccan Plateau
(d) Nagaland (c) Madhya Pradesh
197. Match the following river given in list-I (d) Chhattisgarh Basin
with the place of origin given in list-II: (e) None of these
202. Which of the following is the correct order
List-I (River) List-II (Place of Origin)
of the Himalayan Ranges from south to
A. Mand 1. Ludeng hills north in the western section?
B. Banas 2. Sonbera plateau [Chhattisgarh P.C.S. (Pre) 2015]
C. Jonk 3. Devgarh hills (a) Great Himalaya - Lesser Himalaya -
D. Kelo 4. Mainpat Shiwalik
(b) Shiwalik - Lesser Himalaya - Great
Code: [Chhattisgarh PCS (Pre) 2021]
Himalaya
A B C D (c) Lesser Himalaya - Great Himalaya -
(a) 3 1 4 2 Shiwalik
(b) 4 3 2 1 (d) Shiwalik - Great Himalaya - Lesser
(c) 2 3 1 4 Himalaya
(d) 1 2 4 3 (e) None of these
198. Four rivers have been mentioned below. 203. Sriharikota island is located in–
What will be the decreasing order of these [Chhattisgarh P.C.S. (Pre) 2011]
rivers in length (starting from the longest (a) Near Chilka Lake
to the shortest)? (b) Near the mouth of Mahanadi River
i. Mahanadi (c) Near Pulicat Lake
ii. Krishna (d) Near the mouth of Godavari River

PYQ Workbook 154


GENERAL GEOGRAPHY

204. Which of the following characteristic 210. Which of the following rivers does not make
features is appropriate for Narmada River? estuary? [Jharkhand P.C.S. (Pre) 2003]
[Jharkhand P.C.S. (Pre) 2021] (a) Narmada
(a) Volcanic origin (b) Tapti
(b) Wind origin (c) Mandavi
(d) Mahanadi
(c) Structural origin
(d) Glacial origin 211. Which of the following Island is not a part
of Laccadive Island?
205. In which of the following parts of India [Uttarakhand PCS (Pre) 2021]
‘Atolls’ are commonly found? (a) Tilanchong
[Jharkhand P.C.S. (Pre) 2021] (b) Agathi
(a) Near Sunderbans (c) Kavaratti
(b) Near Khambhat Gulf (d) Kalpeni
(c) Near Lakshadweep 212. Koppen has used which of the following
(d) Near Rameshwaram words for the climate of Great Plains of
206. Which of the following statements is true India? [Uttarakhand PCS (Pre) 2021]
regarding Laterite soil in India (a) Amw
1. Laterite soil develops in areas with high (b) As
(c) Cwg
temperatures and heavy rainfall. (d) Aw
2. Laterite soil is rich in humus and found in
West Bengal, Assam and Odisha. 213. Out of the following rivers which is not the
tributary of river Sutlej?
Code: [Jharkhand P.C.S. (Pre) 2021]
[Uttarakhand PCS (Pre) 2021]
(a) Only 2 is true (a) Baspa
(b) Both 1 and 2 are true (b) Spiti
(c) Both 1 and 2 are false (c) Ravi
(d) Only 1 is true (d) Beas
207. Which of the following pairs is not correctly 214. ‘Khair’ (Catechu) is a tree of which type of
matched? [Jharkhand P.C.S. (Pre) 2016] forest? [Uttarakhand PCS (Pre) 2021]
(a) Total area of India: 3.28 million sq. km (a) Sub-Alpine Forests
(b) Latitudinal extent: 8°4’ N - 37°6’ N (b) Cool-Temperate Forests
(c) Longitudinal extent: 68°7’ E - 97°25’ E (c) Sub-Tropical Forests
(d) Number of states in India: 26 (d) Temperate Forests
208. Name the place in India where Bay of 215. Which plateau lies between the Aravalli and
Bengal, Arabian Sea and Indian Ocean the Vindhya ranges?
meet. [Jharkhand P.C.S. (Pre) 2016] [Uttarakhand PCS (Pre) 2021]
(a) Malwa Plateau
(a) Kanyakumari
(b) Chhota Nagpur Plateau
(b) Indira Point (c) Deccan Plateau
(c) Nagercoil (d) Peninsular Plateau
(d) Rameswaram
216. Match list-I with list-II and select the
209. Consider the following relief features: correct answer using the codes given below
1. Zaskar Range the lists:
2. Dhauladhar Range List-I (Doab) List-II (Rivers)
3. Ladakh Range
A. Bist Doab 1. Between Ravi and
4. Karakoram Range Chenab
The correct south to the northward sequence B. Bari Doab 2. Between Ravi and
of the above relief features is- Beas
[Jharkhand P.C.S. (Pre) 2013]
C. Rachna Doab 3. Between Beas and
(a) 2, 1, 3, 4 Satluj
(b) 2, 3, 4, 1
(c) 4, 3, 2, 1 D. Chaj Doab 4. Between Chenab and
(d) 4, 2, 1, 3 Jhelum

155 PYQ Workbook


GENERAL GEOGRAPHY

Code: [Uttarakhand PCS (Pre) 2016] (a) Bhagirathi


A B C D (b) Alaknanda
(a) 3 2 1 4 (c) Saryu
(b) 1 2 3 4 (d) Mandakini
(c) 4 3 2 1 222. The birch tree is found in-
(d) 1 4 2 3 [Uttarakhand P.C.S. (Pre) 2006]
217. Which one of the following groups is the (a) Aravallis
correct sequence of mountain peaks from (b) Himalayas
east to west? (c) Nilgiris
[Uttarakhand P.C.S. (Pre) 2012] (d) Vindhyas
(a) Everest, Kanchenjunga, Annapurna, 223. Consider the following clues about a State
Dhaulagiri of India:
(b) Kanchenjunga, Everest, Annapurna, 1. It is known as land of red River and blue
Dhaulagiri hills’.
(c) Kanchenjunga, Dhaulagiri, Annapurna, 2. It contains three of the six physiographic
Everest divisions of India.
(d) Everest, Kanchenjunga, Dhaulagiri, 3. It covers about 2.39 percent of total areas
Annapurna of the country.
218. Match list-I with list-II and select the Identify the State on the basis of the given
correct answer by using the codes given clues. [CDS-2023(I)]
below the lists: (a) Karnataka
List-I (Source Region) List-II (River) (b) Andhra Pradesh
(c) Assam
A. Brahmagiri Hills 1. Cauvery (d) Chhattisgarh
B. Verinag Spring 2. Jhelum
224. Which of the following sets of States shares
C. Mahabaleshwar 3. Krishna boundary with Telangana? [CDS-2023(I)]
D. Chhota Nagpur 4. Subarnarekha (a) Odisha, Andhra Pradesh, Karnataka,
Plateau Maharashtra
Code: [Uttarakhand PCS (Pre) 2016] (b) Chhattisgarh, Andhra Pradesh,
A B C D Karnataka, Madhya Pradesh
(a) 4 3 2 1 (c) Odisha, Andhra Pradesh, Karnataka,
(b) 1 2 3 4 Madhya Pradesh
(c) 2 1 4 3 (d) Chhattisgarh, Andhra Pradesh,
(d) 3 2 1 4 Karnataka, Maharashtra

219. Which among the following places will have 225. Rani told her friends that last year she did
maximum length of day in the month of boating in four different lakes, namely,
June? [Uttarakhand PCS (Mains) 2006] Loktak, Barapani, Kolleru and Pulicat.
Which of the following States did she visit?
(a) Hyderabad
(b) Chennai [CDS-2023(I)]
(c) Bhopal (a) Manipur, Tamil Nadu, Meghalaya, Kerala
(d) Delhi (b) Manipur, Meghalaya, Andhra Pradesh,
Tamil Nadu
220. In which part of Uttarakhand are artesian (c) Mizoram, Tamil Nadu, Assam, Kerala
wells found? (d) Mizoram, Assam, Andhra Pradesh, Tamil
[Uttarakhand P.C.S. (Mains) 2006] Nadu
(a) Bhabhar
226. The Tropic of Cancer passes through how
(b) Tarai
many coastal and landlocked States of
(c) Shivalik Hills
India, respectively? [CDS-2023(I)]
(d) None of the above
(a) 1 and 7
221. Name the river which flows between (b) 2 and 6
Kedarnath and Rudra Prayag? (c) 2 and 5
[Uttarakhand P.C.S. (Pre) 2006] (d) 3 and 5

PYQ Workbook 156


GENERAL GEOGRAPHY

227. Which one among the following States 233. Manikaran in Himachal Pradesh is known
has the longest extension in north- south for: [CDS-2022(I)]
direction? [CDS-2023(I)] (a) Solar energy
(a) Jharkhand (b) Geothermal energy
(b) Telangana (c) Bioenergy
(c) Odisha (d) Wind energy
(d) Chhattisgarh
234. Match List I witth List II and select the
228. Consider the following characteristics of a correct answer using the code given below
soil type:
the lists:
1. These soils consist of various proportions
of sand, silt and clay. List I (Soil List II (Salient
2. These soils contain adequate proportions Type) Characteristic)
of potash, phosphoric acid and lime. A. Entisols 1. Soils having weakly
3. These soils can be further classified on or quickly developed
the basis of their age. horizons
Identify the type of soll on the basis of the B. Inceptisols 2. Weakly developed soils
given characteristics. [CDS-2023(I)] with no horizons
(a) Alluvial
C. Histosols 3. These soils are very old
(b) Laterite
and highly weathered
(c) Arid
(d) Black D. Oxisols 4. These soils have
very high quantity of
229. Indus river basin is extended up to organic matter in the
[CDS-2023(I)] upper layers
(a) Ladakh, Jammu and Kashmir, Himachal Code: [CDS-2022(I)]
Pradesh, Punjab
(b) Ladakh, Jammu and Kashmir, Punjab, A B C D
Haryana (a) 2 1 4 3
(c) Ladakh, Jammu and Kashmir, Punjab, (b) 2 4 1 3
Haryana, Rajasthan (c) 3 4 1 2
(d) Ladakh, Jammu and Kashmir, Himachal (d) 3 1 4 2
Pradesh, Punjab, Haryana, Rajasthan
235. Ten Degree Channel is found between:
230. Which one of the following Indian States [CDS-2022(I)]
has the longest coastline? [CDS-2023(I)] (a) North Andaman and Middle Andaman.
(a) Tamil Nadu (b) South Andaman and Little Andaman.
(b) Gujarat (c) Little Andaman and Car Nicobar.
(c) Maharashtra
(d) Car Nicobar and Great Nicobar.
(d) Andhra Pradesh
236. Match List I with List II and select the
231. Which one of the following climatic
types (Köppen’s Scheme) is best suited to correct answer using the code given below
describe the climate of the western fringe of the lists:
Rajasthan? [CDS-2022(I)]
List I (Waterfall) List II (State)
(a) Amw
(b) Aw A. Barkana 1. Tamil Nadu
(c) Bwhw B. Dudhsagar 2. Odisha
(d) Cwg
C. Duduma 3. Karnataka
232. Which one of the following States/Union D. Kiliyur 4. Goa
Territories has the highest percentage of
forest area to the total geographical area? Code: [CDS-2022(I)]
[CDS-2022(I)] A B C D
(a) Arunachal Pradesh (a) 3 2 4 1
(b) Lakshadweep (b) 3 4 2 1
(c) Nagaland (c) 1 4 2 3
(d) Mizoram (d) 1 2 4 3

157 PYQ Workbook


GENERAL GEOGRAPHY

237. Match List I with List II and select the 242. Consider the following statements about an
correct answer using the code given below Indian State:
the lists: 1. It is well known globally for its variety of
List I (Lake) List.II (State) rocks and minerals.
2. It has the largest deposits of Chrysotile
A. Kaliveli 1. Sikkim
Asbestos in the country.
B. Khajjiar 2. Kerala 3. The Neelam Sanjiva Reddy Sagar Hydel
C. Khecheopalri 3. Himachal Pradesh Project is located in this State.
D. Vembanad 4. Tamil Nadu Identify the State from the following:
Code : [CDS-2022(I)] [CDS-2022(II)]
A B C D (a) Jharkhand
(a) 2 1 3 4 (b) Rajasthan
(b) 2 3 1 4 (c) Andhra Pradesh
(c) 4 1 3 2 (d) Chhattisgarh
(d) 4 3 1 2
243. Which of the following statements about
238. Mount Thullier (642 m) is an important the coasts of India is/are correct?
mountain peak of Andaman and Nicobar [CDS-2022(II)]
Islands. It is located in: [CDS-2022(I)]
(a) North Andaman 1. The West Coast of India is a high rocky,
(b) Middle Andaman retreating coast.
(c) South Andaman 2. The West Coast of India is dominated by
(d) Great Nicobar erosional landforms.
239. Which one of the following is a tributary of 3. The East Coast of India, however, is a low
Barak River in Manipur? [CDS-2022(II)] sedimentary coast exhibiting depositional
(a) Dhaleswari forms.
(b) Subansiri Select the correct answer using the code given
(c) Dhansiri below:
(d) Manas (a) 1 and 2 only
240. Which types of vegetation will you find (b) 2 and 3 only
on the way as you travel from Bikaner to (c) 1 only
Konark by road by shortest distance? (d) 1, 2 and 3
[CDS-2022(II)] 244. Which one of the following States/UTs has
(a) Tropical thorny, Desert, Moist deciduous, the largest area under Coral Reefs in India?
Mangrove
(b) Desert, Tropical thorny, Moist deciduous, [CDS-2022(II)]
Mangrove (a) Gujarat
(c) Moist deciduous, Tropical thorny, (b) Tamil Nadu
Mangrove, Desert (c) Andaman and Nicobar Islands
(d) Desert, Tropical thorny, Mangrove, Moist (d) Lakshadweep
deciduous
245. In which one of the following highlands of
241. Consider the following statements about India can you find temperate forests called
Karewas: ‘Sholas’? [CDS-2022(II)]
1. These are the lacustrine deposits of (a) Nilgiris
Pleistocene period. (b) Vindhyas
2. These are found along the lower slopes of (c) Satpuras
Pir Panjal. (d) Himalayas
3. Karewas are well known for the cultivation
of saffron, almond and walnut. 246. The Indian desert also known as Marusthali
How many of the above statements is/are is believed to be under the sea during which
correct? [CDS-2022(II)] one of the following eras? [CDS-2022(II)]
(a) 1 (a) Cenozoic era
(b) 2 (b) Palaeozoic era
(c) 3 (c) Mesozoic era
(d) None (d) Archaean and Pre-Cambrian era

PYQ Workbook 158


GENERAL GEOGRAPHY

247. Identify the type of soil on the basis of the 1. There is one standard time for the whole
given characteristics: country.
1. They are rich in lime, iron, magnesia and 2. Andaman and Nicobar Islands and
alumina. Lakshadweep Islands have different Time
2. They are generally clayey, deep and Zones.
impermeable. 3. Indian Standard Time (IST) is five and
3. They are mainly found in Maharashtra, half hours behind GMT.
Madhya Pradesh and Gujarat. Select the correct answer using the codes
Select the correct answer using the codes given below. [CDS-2021(II)]
given below: [CDS 2021 (I)] (a) Only 1
(a) Laterite soil (b) Only 2
(b) Red and yellow soil (c) 1 and 2
(c) Saline soil (d) 1 and 3
(d) Black soil
254. D had recently visited Khardung La, Nubra
248. Alluvial soils vary in nature from sandy Valley and several Buddhist monasteries.
loam to clay. They are generally Which one of the following States / UT had
[CDS 2021 (I)] she visited? [CDS-2021(II)]
(a) poor in potash and rich in phosphorus. (a) Himachal Pradesh
(b) poor in both potash and phosphorus. (b) Uttarakhand
(c) rich in both potash and phosphorus. (c) Sikkim
(d) rich in potash and poor in phosphorus. (d) Ladakh
249. Which one of the following water bodies 255. Which one of the following states receives
separates the Andaman and the Nicobar rainfall from both the Arabian Sea branch
islands? [CDS 2021 (I)] and the Bay of Bengal branch of Monsoon?
(a) Gulf of Mannar [CDS-2021(II)]
(b) Ten Degree Channel (a) Punjab
(c) Eleven Degree Channel (b) Maharashtra
(d) Palk Strait (c) Meghalaya
250. Which one of the following longitudes is the (d) Tamil Nadu
standard meridian of India? [CDS 2021 (I)] 256. S had witnessed contour bunding and
(a) 83°30’ E contour ploughing while visiting one of the
(b) 82°30’ E states in India. Identify the state from the
(c) 82°30’ W options given below. [CDS-2021(II)]
(d) 83°30’ W (a) Punjab
251. Four persons are travelling to different (b) Haryana
States of India. To whom will you advise to (c) Himachal Pradesh
protect oneself from blizzards? (d) Rajasthan
[CDS 2021 (II)] 257. D performs her train journey by the shortest
(a) The person travelling to Arunachal route from Bengaluru to New Delhi. Which
Pradesh one of the following rivers will she not cross
(b) The person travelling to Assam while performing the journey?
(c) The person travelling to Tripura [CDS-2021(II)]
(d) The person travelling to Odisha (a) Narmada
252. Which one of the following biosphere (b) Yamuna
reserves comprises islands with estuaries, (c) Godavari
beaches, coral reefs, salt marshes and (d) Krishna
mangroves? [CDS 2021 (II)] 258. Identify the State on the basis of the
(a) Sunderban biosphere reserve following characteristics.
(b) Gulf of Mannar biosphere reserve 1. Tropic of Cancer passes through the state.
(c) Nilgiri biosphere reserve 2. The state has more North-South
(d) Nandadevi biosphere reserve extension.
253. Which of the following statements is/are 3. The state has international border with
correct with respect to Time Zone in India? Bangladesh and Myanmar.

159 PYQ Workbook


GENERAL GEOGRAPHY

Select the correct using the codes given D. Guru Shikhar 4. Annamalai
below: [CDS-2021(II)]
Codes: [CDS 2019 (II)]
(a) Tripura
(b) Mizoram A B C D
(c) Nagaland (a) 3 2 1 4
(d) Manipur (b) 3 1 2 4
(c) 4 1 2 3
259. M wants to visit a place in a Union Territory,
(d) 4 2 1 3
which is located at 34° N and 77° E. Which
one of the following Union Territories must 265. Where are the largest quantity of cichlids
he have planned to visit? [CDS-2021(II)] found in India? [CDS 2019 (II)]
(a) Andaman and Nicobar Islands (a) Backwaters of Kerala
(b) Lakshadweep (b) Sunderbans
(c) Puducherry (c) Narmada
(d) Ladakh (d) Godavari
260. Which one of the following rivers joins 266. The term ‘soil impoverishment’ relates to
Ganga directly? [CDS 2020 (I)] which one of the following? [CDS 2019 (II)]
(a) Chambal (a) Soil erosion
(b) Son (b) Soil deposition
(c) Betwa (c) Soil getting very deficient in plant
(d) Ken nutrients
261. Which one of the following Indian places (d) Soil getting enriched with plant nutrients
receives minimum rainfall in a year?
267. Rains caused by thunderstorms during the
[CDS 2020 (II)] hot weather season (mid-March to mid-
(a) Jodhpur June) in Karnataka are called
(b) Leh
[CDS 2019 (II)]
(c) New Delhi
(d) Bengaluru (a) Kalbaisakhi
(b) Mango showers
262. Which one of the following is not correct (c) Loo
regarding South India? [CDS 2019 (II)] (d) Cherry blossoms
(a) Diurnal range of temperature is less
(b) Annual range of temperature is less 268. Which one of the following river valleys of
(c) Temperature is high throughout the year India is under the influence of intensive
(d) Extreme climatic conditions are found gully erosion? [CDS 2019 (I)]
(a) Kosi
263. Which one of the following statements
(b) Chambal
regarding temperate coniferous forest
biome is not correct? [CDS 2019 (II)] (c) Damodar
(d) Brahmaputra
(a) They are characterised by very little
undergrowth. 269. Match List I with List II and select the
(b) They have a growing period of 50. to 100 correct answer using the codes given below
days in a year. the lists.
(c) There is low variation in annual
temperature. List I (Type of Lake) List II (Example)
(d) There is high range in spatial distribution A. Tectonic 1. Lonar Lake
of annual precipitation. B. Crater 2. Gangabal Lake
264. Match List I with List II and select the C. Glacial 3. Purbasthali Lake
correct answer using the codes given below
the lists. D. Fluvial 4. Bhimtal Lake
Codes: [CDS 2019 (I)]
List I (Peak) List II (Name of Hill) A B C D
A. Anamudi 1. Nilgiri (a) 4 1 2 3
B. Doddabetta 2. Satpura (b) 4 2 1 3
(c) 3 1 2 4
C. Dhupgarh 3. Aravalli (d) 3 2 1 4

PYQ Workbook 160


GENERAL GEOGRAPHY

270. South Arcot and Ramanathapuram receive (c) 1, 2, 3, 4


over 50 percent of their annual rainfall (d) 1, 3, 2, 4
from which one of the following?
276. The following consist of two statements,
[CDS 2018 (II)] statement I and statement II. Examine these
(a) South-West monsoon two statements carefully and select the
(b) North-East monsoon correct answer using the codes given below.
(c) Bay of Bengal branch of summer monsoon
(d) Western disturbances Statement I : The interior part of Maharashtra
does not receive adequate rain in the summer
271. Which one among the following is not a season.
tributary of river Luni? [CDS 2018 (II)] Statement II: The interior part of Maharashtra
(a) Khari lies in the rain shadow area of the Western
(b) Sukri Ghats.
(c) Jawai
Codes: [CDS 2017 (II)]
(d) Banas
(a) Both the statements are individually true
272. Which one among the following passes and statement II is the correct explanation
links Lhasa with Ladakh? [CDS 2018 (II)] of statement I
(a) Lanak La (b) Both the statements are individually
(b) Babusar true, but statement II is not the correct
(c) Burzil explanation of statement I
(d) Khyber (c) Statement I is true, but statement II is
273. Which one of the following places of India false
experiences highest atmospheric pressure (d) Statement I is false, but statement II is
during winter? true
(a) Jaisalmer 277. Match List I with List II and select the
(b) Leh correct answer using the codes given below
(c) Chennai the lists.
(d) Guwahati
List I (Pass) List II (Place)
274. Consider the following tributaries of river
A. Zoji La 1. Himachal Pradesh
Ganga
1. Gandak B. Shipki La 2. Uttarakhand
2. Kosi C. Lipulekh 3. Jammu and Kashmir
3. Ghaghara D. Nathu La 4. Sikkim
4. Gomti Code: [CDS 2017 (II)]
Which one of the following is the correct A B C D
order of the above rivers from East to West? (a) 4 2 1 3
[CDS 2018 (I)] (b) 4 1 2 3
(a) 3, 4, 1, 2 (c) 3 1 2 4
(b) 2, 1, 3, 4 (d) 3 2 1 4
(c) 2, 3, 1, 4
(d) 1, 2, 4, 3 278. Which of the following statements with
respect to the Indian Peninsular Plateau are
275. Arrange the following Tiger Reserves of correct
India from North to South 1. The Deccan Plateau gradually rises from
1. Dudhwa North to South.
2. Panna 2. The Malwa Plateau dominates the
3. Pench Vindhyan scraps, forming the Eastern
4. Indravati flank of the plateau.
Select the correct answer using the codes 3. The Western Satpuras separate the
given below: [CDS 2017 (II)] Narmada and Tapi rivers.
(a) 4, 3, 2, 1 4. The Chota Nagpur Plateau is composed
(b) 2, 1, 4, 3 of Archaean granite and gneiss rocks.

161 PYQ Workbook


GENERAL GEOGRAPHY

Select the correct answer using the codes 1. Himalayan rivers have their origin in
given below? [CDS 2017 (II)] the snow-covered areas, hence are dry in
(a) 1, 2, 3 and 4 winter season.
(b) 1, 3 and 4 2. Rivers of the Peninsular Plateau have
(c) 2 and 3 reached maturity.
(d) 1 and 3 3. Himalayan rivers depict all the three
279. Which of the following lakes is/are situated stages of normal cycle of erosion.
in Ladakh? Select the correct answer using the codes
1. Tso Kar given below: [CDS 2017 (II)]
2. Pangong Tso (a) Only 2
(b) 2 and 3
3. Tsomgo
(c) 1 and 3
4. Tso Moriri (d) 1, 2 and 3
Select the correct answer using the codes
given below: [CDS 2017 (II)] 283. Which one of the following is the correct
sequence of occurrences of rivers from
(a) Only 1
Chennai to Kolkata when travelled by road
(b) 2 and 3
(shortest distance)? [CDS 2017 (I)]
(c) 1, 2 and 4
(d) 2 and 4 (a) Krishna, Kaveri, Godavari, Mahanadi,
Subarnarekha
280. Which of the following statement related to (b) Krishna, Periyar, Godavari, Mahanadi,
latitude are true? Subarnarekha
1. Rainfall, temperature and vegetation vary (c) Penneru, Krishna, Godavari, Mahanadi,
with latitude. Subarnarekha
2. The difference between the longest day (d) Penneru, Mahanadi, Subarnarekha,
and the shortest day increases with Godavari, Krishna
latitude. 284. Which one of the following with regard
3. Indira Point is located approximately at to Aridisol, one of the soil orders, is not
6°45’N latitude. correct? [CDS 2017 (I)]
Select the correct answer using the codes (a) Lack of water for plants during most part
given below: [CDS 2017 (II)] of the year
(a) 1 and 2 (b) High organic matter
(b) 1 and 3 (c) Large accumulation of carbonates at
(c) 2 and 3 depth
(d) 1, 2 and 3 (d) Absence of deep wide cracks
281. Match List I with List II and select the 285. Which one of the following statements
correct answer using the codes given below concerning the natural vegetation of India
the lists. is not correct? [CDS 2016 (II)]
(a) Sal is found in moist deciduous forests
List I (Lake) List II (Type of Lake) (b) Casuarina is largely found along the
A. Bhimtal 1. Lagoon coastal region of Tamil Nadu
B. Ashtamudi 2. Landslide (c) Deodar is a coniferous species of tree
(d) Shola forests are found mainly on the
C. Gohna 3. Tectonic upper reaches of Himalaya
D. Lonar 4. Crater
286. Consider the following statements
Code: [CDS 2017 (II)] regarding laterite soils of India
A B C D 1. Laterite soils are generally red in colour.
(a) 3 1 2 4 2. Laterite soils are rich in nitrogen and
(b) 3 2 1 4 potash.
(c) 4 2 1 3 3. Laterite soils are well developed in
(d) 4 1 2 3 Rajasthan and Uttar Pradesh.
282. Which of the following statements is/are 4. Tapioca and cashew nuts grow well in this
correct? soil.

PYQ Workbook 162


GENERAL GEOGRAPHY

Which of the statements given above is/are 3. Upper air circulation


correct? [CDS 2016 (I)] 4. The Himalayan Mountains
(a) Only 1 Select the correct answer using the codes
(b) 2, 3 and 4 given below: [CDS 2015 (II)]
(c) 1 and 4
(a) 1 and 4
(d) 1, 2 and 4
(b) 1, 2, 3 and 4
287. Consider the following tributaries of river (c) 2, 3 and 4
Brahmaputra: (d) 1, 2 and 3
1. Lohit
291. If a ship has to go from Chennai to Kochi,
2. Tista
it has to go around Sri Lanka rather than
3. Subansiri crossing through the Palk Strait. Why?
4. Sankosh
[CDS 2015 (II)]
Arrange the above rivers from West to East: (a) The Palk Strait has disputed islands and
[CDS 2016 (I)] the Sri Lankan Navy does not allow the
(a) 2, 4, 3, 1 ships to cross through
(b) 2, 3, 4, 1 (b) It is too shallow for ships to cross
(c) 4, 2, 3, 1 (c) Shipping is prohibited through the strait
(d) 3, 1, 2, 4 due to its religious significance connected
288. Match the following: with the epic Ramayana
(d) The around Sri Lanka route is actually
List I (Regions) List II (State) shorter than crossing through the Palk
A. Baghelkhand 1. West Bengal Strait
B. Kuttanad 2. Madhya Pradesh 292. Consider the following statements with
C. Saurashtra 3. Kerala regard to cold waves in winter season in
D. Rarh 4. Gujrat Northern India:
Codes: [CDS 2015 (II)] 1. There is lack of maritime influence.
A B C D 2. Northern India is nearer to the Himalayan
(a) 2 3 4 1 region.
(b) 2 4 3 1 3. Air mass comes from polar regions of
(c) 1 3 4 2 Northern India.
(d) 1 4 3 2 Which of the statement(s) given above is/are
289. Match the following: correct? [CDS 2015 (I)]
(a) Only 1
List I (lake) List II (Type) (b) 1 and 3
A. Ashtamundi 1. Residual (Sweet (c) 2 and 3
Kayal Water) (d) 1 and 2
B. Himayat Sagar 2. Lagoon 293. Match List I with List II and select the
C. Dal Lake 3. Glacial correct answer using the codes given below
the lists:
D. Tsomgo Lake 4. Artificial (Sweet
Water) List I (Sanctuary) List II (State)
Codes: [CDS 2015 (II)] A. Sharavath Valley 1. Gujarat
A B C D B. Satkosia Gorge 2. Goa
(a) 2 4 3 1
(b) 2 1 4 3 C. Pirotan Island 3. Odisha
(c) 3 1 4 2 D. Bhagwan Mahaveer 4. Karnataka
(d) 3 4 1 2 Codes: [CDS 2015 (I)]
290. Which of the following are the major factors A B C D
responsible for the monsoon type of climate (a) 2 1 3 4
in India? (b) 4 1 3 2
1. Location (c) 4 3 1 2
2. Thermal contrast (d) 2 3 1 4

163 PYQ Workbook


GENERAL GEOGRAPHY

294. The following item consist of two 1. High and dry winds would blow outward
statements, Statement I and Statement II. from this high pressure area.
You have to examine these two statements 2. The Northern plain would become cold.
carefully and select the answers from these 3. Scorching winds (locally called loo)
items using the codes given below: would blow during the day of time.
Statement I: India’s offshore and deep sea 4. There would be torrential rains brought
fish catch is very poor considering the marine by thunderstorms.
potential. Select the correct answer using the codes
Statement II: Indian coast does not have given below: [CDS 2014 (I)]
many gulfs, bays, estuaries and backwaters. (a) 1 and 2
(b) 2 and 3
Codes: [CDS 2015 (I)]
(c) 3 and 4
(a) Both the statements are individually true (d) All of these
and statement II is the correct explanation
of statement I 298. Veliconda group of low hills is a structural
(b) Both the statements are individually part of which of the following region?
true, but statement II is not the correct [CDS 2014 (I)]
explanation of statement I (a) Nilgiri Hills
(c) Statement I is true, but statement II is (b) Western Ghats
(c) Eastern Ghats
false
(d) Cardamom Hills
(d) Statement I is false, but statement II is
true 299. Ganga water is being shared at Farakka with
which one of the following countries?
295. Which of the following would have
[CAPF 2022]
happened if the Himalaya did not exist?
(a) Nepal
[CDS 2014 (II)] (b) Bhutan
(a) Monsoon rains would have taken place in (c) Bangladesh
winter months. (d) Myanmar
(b) Coastal India would have experienced
300. Which of the following sets of Indian
Mediterranean climate.
States are landlocked and do not have an
(c) North Indian plain would have been international boundary? [CAPF 2022]
much cooler in winter.
(a) Andhra Pradesh. Pradesh. Chhattisgarh,
(d) North-Western part of India would have Haryana, Jharkhand and Madhya Pradesh
experienced humid condition. (b) Chhattisgarh, Haryana, Jharkhand,
296. Which of the following method(s) is/ Madhya Pradesh and Telangana
are suitable for soil conservation in hilly (c) Chhattisgarh, Haryana, Himachal
region? Pradesh, Madhya Pradesh and Telangana
(d) Andhra Pradesh, Chhattisgarh, Haryana,
1. Terracing and contour bunding
Himachal Pradesh and Madhya Pradesh
2. Shifting cultivation
3. Contour ploughing 301. Arrange the following passes from West to
East direction:
Select the correct answer using the codes
1. Bara Lacha La
given below: [CDS 2014 (I)]
2. Shipki La
(a) 1 and 3 3. Jelep La
(b) Only 2
4. Bomdi La
(c) Only 3
(d) All of the above Select the correct answer using the code given
below: [CAPF 2022]
297. What would be the influence on the weather (a) 1-2-3-4
conditions when in mid-winter a feeble high (b) 1-3-2-4
pressure develops over the North-Western (c) 2-3-4-1
part of India? (d) 2-4-3-1

PYQ Workbook 164


GENERAL GEOGRAPHY

302. Match List-I with List-II and select the (a) Only 1
correct answer using the code given below (b) 1 and 2
the Lists: (c) 2 and 3
(d) Only 2
List-I (Type of List-II (Characteristic
plain) feature) 307. Which one among the following stations
A. Bhabar 1. Marsh receives maximum rainfall? [CAPF 2020]
(a) Hyderabad
B. Bhangar 2. Silt
(b) Bengaluru
C. Khadar 3. Gravels (c) Delhi
D. Delta 4. Kankar (d) Raipur
Code: [CAPF 2022] 308. Which one of the following latitudes passes
A B C D through maximum Indian states?
(a) 3 2 4 1 [CAPF 2019]
(b) 3 4 2 1 (a) 20o N latitude
(c) 1 4 2 3 (b) 22o N latitude
(d) 1 2 4 3 (c) 24o N latitude
303. In which one of the following soils, the (d) 26o N latitude
salt content is so high that common salt is
309. Gersoppa Falls is formed by which one of
obtained by evaporating the saline water in
some areas? [CAPF 2021] the following rivers? [CAPF 2018]
(a) Peaty soil (a) Lohit
(b) Alluvial soil (b) Tungabhadra
(c) Laterite soil (c) Sharavati
(d) Arid soil (d) Krishna
304. Match List-I with List-II and select the 310. Which one of the following major river
correct answer using the codes given below basins of India is the shortest in length and
the lists: smaller in terms of catchment area?
[CAPF 2018]
List-1 (Tributary) List-II (River)
(a) Cauvery
A. Sutlej 1. Yamuna (b) Mahi
B. Kosi 2. Brahmaputra (c) Penner
C. Teesta 3. Ganga (d) Sabarmati
D. Chambal 4. Indus 311. Arabian Sea records higher salinity than
Codes: [CAPF 2021] the Bay of Bengal because [CAPF 2018]
A B C D (a) Arabian Sea exhibits high rate of
(a) 1 3 2 4 evaporation and low influx of freshwater
(b) 1 2 3 4 (b) Arabian Sea has shallow water
(c) 4 3 2 1 (c) Arabian Sea has more enclosed land
(d) 4 2 3 1 (d) Arabian Sea receives more rainfall
305. Which one of the following is not a desert 312. Which one of the following sites does not
river? [CAPF 2021] have coral reef formation? [CAPF 2018]
(a) Barak (a) Gulf of Cambay/Khambhat
(b) Rupen (b) Gulf of Mannar
(c) Banas (c) Lakshadweep
(d) Machchu (d) Andaman and Nicobar Islands
306. Consider the following statements: 313. Which one of the following does not pass
1. River Kaveri has well developed valley. through the Kaziranga National Park?
2. Valley of River Alakananda is still growing [CAPF 2017]
3. River Krishna is largest east flowing (a) The Brahmaputra River
peninsular river. (b) The Diphlu river
Which of the statements given above is/are (c) National Highway No. 37
correct? [CAPF 2020] (d) Railway Track of Indian Railway

165 PYQ Workbook


GENERAL GEOGRAPHY

314. Which one of the following statements 318. Which one among the following is the most
about the Himalayas is not correct? plausible explanation for the location of the
[CAPF 2017] Thar Desert in Western India?[CAPF 2014]
(a) They are young fold mountains. (a) Prevalence of sand dunes
(b) They have geosynclinal rocks. (b) The evaporation of moisture by heat
(c) Himalayan Frontal Faults (HFF) separates (c) The absence of mountains to the North
Himalaya from Tibet. of Rajasthan to cause orographic rainfall
(d) Indus and Sutlej rivers form antecedent in it.
drainage in Himalaya (d) The Moisture carried by the South-West
Monsoon is driven away by the dry upper
315. The Bum-La Pass is located in [CAPF 2017] at current.
(a) Sikkim 319. Which of the following statements with
(b) Arunachal Pradesh regard to the Indian Peninsular Plateau is/
(c) Himachal Pradesh are correct?
(d) Uttarakhand 1. The Southern plateau block is formed
316. Which one of the following rivers flows mainly of granite and gneiss.
between Satpura and Vindhya ranges? 2. The Deccan lava plateau is an elevated
[CAPF 2016] tableland consisting of horizontally
(a) Tapi arranged lava sheets.
(b) Sabarmati 3. The Malwa plateau dominates the
(c) Narmada Vindhyan scraps, forming the Eastern
(d) Mahi flank of the plateau.
4. The trough of the Narmada and Tapti is
317. Which of the following statements relating interposed between the Vindhyan and the
to Dharwar geological system are correct? Satpura ranges.
1. They belong to Archaean geological Select the correct answer using the codes
period given below. [CAPF 2014]
2. They are metamorphosed sedimentary (a) 1, 2 and 3
rocks. (b) 1, 2 and 4
3. They are of great economic importance (c) 1 and 2 only
for its mineral resources. (d) 4 only
4. They are found predominantly in 320. Three important rivers of the Indian
Karnataka, Tamil Nadu, Odisha and sub-continent have their source near the
Jharkhand. Manasarovar Lake in the Great Himalayas.
Select the correct answer using the code given Which among the following are the rivers?
below. [CAPF 2016] [CAPF 2014]
(a) 1, 2, 3 and 4 (a) Indus, Jhelum and Sutlej
(b) 2, 3 and 4 only (b) Sutlej, Yamuna and Brahmaputra
(c) 3 and 4 only (c) Brahmaputra, Indus and Sutlej
(d) 1 and 2 only (d) Sutlej, Jhelum and Yamuna

PYQ Workbook 166


GENERAL GEOGRAPHY

SOLUTIONS
3. Solution: (d)
5.1. UPSC CSE Previous Years’ Questions
Exp) Option d is the correct answer
1. Solution: (b) Statement 1 is incorrect: Amarkantak is a pilgrim town in
Exp) Option b is the correct answer. Anuppur, Madhya Pradesh, India. The Amarkantak region
Statement 1 is correct: The Jhelum, an important tributary is a unique natural heritage area and is the meeting point of
of the Indus, rises from a spring at Verinag situated at the the Vindhya and the Satpura Ranges, with the Maikal Hills
foot of the Pir Panjal in the south-eastern part of the valley being the fulcrum.
of Kashmir. It flows through Srinagar and the Wular lake Statement 2 is incorrect: The Billigirirangan hills, also
before entering Pakistan through a deep narrow gorge. It known as Biligiriranga Hills or BR Hills, are not a part of
joins the Chenab near Jhang in Pakistan. the Satpura range. The Biligirirangana Hills or Biligirirangan
Statement 2 is incorrect: The Kolleru lake serves as a Hills is a hill range situated in south-western Karnataka,
natural flood-balancing reservoir for two rivers. The lake at its border with Tamil Nadu (Erode District) in South
is fed directly by water from the seasonal Budameru and India. The area is called Biligiri Ranganatha Swamy Temple
Tammileru streams. It is connected to the Krishna (but not Wildlife Sanctuary or simply BRT Wildlife Sanctuary.
directly feeds) and Godavari systems by over 68 inflowing Statement 3 is incorrect: The Sesachalam hills, also known
drains and channels. as Tirumala hills, are not a part of the Western Ghats.
Statement 3 is correct: Once a haven for migratory birds, Seshachalam Hills are hilly ranges part of the Eastern Ghats
the Kanwar lake in Bihar, Asia’s largest freshwater oxbow in southern Andhra Pradesh state, in southeastern India.
lake, is today a dying wetland ecosystem. It is a residual The Seshachalam hill ranges are predominantly present
oxbow lake, formed due to meandering of Gandak river, a in Tirupati district of the Rayalaseema region in Andhra
tributary of Ganga. Pradesh, India.
Thus None of the Statements are correct.
2. Solution: (d)
Exp) Option d is the correct answer 4. Solution: (c)
Option a is incorrect: Bhitarkanika mangroves, located Exp) Option c is the correct answer.
in Odisha, India, is a significant mangrove ecosystem. Gorges are formed when rivers erode through gigantic rock
While it experiences tidal fluctuations, it is not primarily formations over thousands of years.
characterized by repeated falls in sea level. Rather, it is Gandikota is a small village in the Kadapa district of Andhra
influenced by the riverine system and is considered an Pradesh.
important breeding ground for various species, including the
The swift flowing waters of River Pennar cut through the
endangered saltwater crocodile.
granite rocks near the village of Gandikota, naturally eroding
Option b is incorrect: Marakkanam salt pans, situated in the rock and forming a deep valley. Gandikota got its name
Tamil Nadu, India, are salt marshes used for salt production. from ‘gandi’ the Telugu word for ‘gorge’.
They are formed in low-lying coastal areas where seawater is
The spectacular gorge is famously adjudged as the Grand
allowed to evaporate, leaving behind salt crystals. However,
Canyon of India.
these salt pans do not result from repeated falls in sea level
but are created artificially by utilizing the existing seawater. 5. Solution: (b)
Option c is incorrect: Naupada swamp, also known as Exp) Option b is the correct answer.
Naupada Wetland, is located in Odisha, India. It is a
Pair 1 is incorrectly matched. Namcha Barwa is not situated
freshwater swamp and is not primarily influenced by
in Garhwal Himalaya.
repeated falls in sea level. Instead, it is formed by the inflow
of freshwater from rivers and receives seasonal rainwater. Namcha Barwa is considered as the Eastern point of the
Himalayas. It is located on the extreme east of the Indian
Option d is correct: The Rann of Kutch, situated in Gujarat,
state of Arunachal Pradesh. The height of Namcha Barwa is
India, is the best example among the options provided. It
about 7782 m. Brahmaputra river takes a U turn on reaching
is a vast seasonal salt marsh that experiences significant
Namcha Barwa.
fluctuations in sea level due to its geological history. The
Rann of Kutch was once an arm of the Arabian Sea, and as Pair 2 is correctly matched. The part of the Himalayas
the sea level fell over time, it transformed into a marshy land. lying between Satluj and Kali rivers is known as Kumaon
The region is known for its unique ecosystem, supporting a Himalayas.
diverse range of flora and fauna, including migratory birds Nanda Devi is the second highest mountain in India and the
and endangered species. The Rann of Kutch is also famous highest entirely within the country (Kangchenjunga being
for the annual Rann Utsav, a cultural festival that celebrates on the border of India and Nepal). It is part of the Kumaon
the vibrant traditions of the region. Himalayas, and is located in the state of Uttarakhand,

167 PYQ Workbook


GENERAL GEOGRAPHY

between the Rishiganga valley on the west and the Goriganga 9. Solution: (b)
valley on the east. The surrounding Nanda Devi National Exp) Option b is the correct answer.
Park was declared a UNESCO World Heritage Site in 1988.
Black soil in India is formed by the weathering of fissure
volcanic rock.
Climatic condition along with the parent rock material is
the important factors for the formation of black soil. Black
soils are spread mostly across interior Gujarat, Maharashtra,
Karnataka, and Madhya Pradesh on the Deccan lava plateau
and the Malwa Plateau, where there is both moderate rainfall
and underlying basaltic rock.

10. Solution: (d)


Exp) Option d is the correct answer.
The Siachen Glacier is situated to the north of the Nubra
Pair 3 is incorrectly matched. Nokrek is not situated in Valley. It is located in the eastern Karakoram range in the
Himalayas, just northeast of the point NJ9842 where the
Sikkim Himalaya. Nokrek is the highest peak in West Garo
Line of Control between India and Pakistan ends. The
Hills of Meghalaya.
glacier is about 76 km long and falls from an altitude of 5,753
6. Solution: (d) m at its head to 3,620 m at its terminus.
Exp) Option d is the correct answer.
Option a is incorrect. The Chenab joins Satluj in Pakistan.
Option b is incorrect. The Jhelumjoins the Chenab near
Jhang in Pakistan.
Option c is incorrect. The Ravi joins the Chenab near Sarai
Sidhu.
Option d is correct. Satluj is joined by the Chenab in
Pakistan. Thus, Satluj receives the collective drainage of
the Ravi, Chenab and Jhelum rivers. It joins the Indus a few
kilometres above Mithankot.
Important Tips
7. Solution: (d)
Additional Facts on Siachen Glacier:
Exp) Option d is the correct answer.
• The entire Siachen Glacier has been under the
Didwana, Kuchaman, Sargol and Khatuare saline lakes of administration of India since 1984, but Pakistan
Rajasthan.‘ maintains a territorial claim over it and controls the
Didwana is a town in the Nagaur district of Rajasthan. It has region west of the Saltoro Ridge.
a famous Didwana salt-water lake of Rajasthan. • The glacier is of strategic importance as it lies
Kuchaman lake is located in Nagaur district of Rajasthan. immediately south of the drainage divide that
separates the Eurasian Plate from the Indian
8. Solution: (b) subcontinent.
Exp) Option b is the correct answer. • It is often referred to as the “Third Pole” due to its
Statement 1 is incorrect. The Brahmani, known as South extensive glaciated area.
Koel in its upper reaches, rises near Nagri village in Ranchi • The Siachen Glacier system covers about 700 km², and
district of Jharkhand at an elevation of about 600 m. the region experiences heavy snowfall and extreme cold
Statement 2 is correct. The Nagavali river originate in the temperatures, with average winter snowfall exceeding
EasternGhats near Lakhbahal, located at an altitude of 1,300 1000 cm and temperatures dropping as low as -50 °C.
metres in the Kalahandi district of the Indian state Odisha.
11. Solution: (b)
Langulya is another name for Nagavali River.
Exp) Option b is the correct answer.
Statement 3 is incorrect. Subernarekha River originate in
Statement 1 is incorrect. Ocean Mean Temperature is
the Chhotnagpur Plateau near Nagri village in the Ranchi
measured up to a depth of 26 degree C isotherm. The 26
District of Jharkhand at an elevation of 600 m.
degree C isotherm is seen at depths varying from 50–100
Statement 4 is correct. Vamsadhara river rises in the metres. During January–March, the mean 26 degree C
Eastern Ghats of Orissa State and enters Srikakulam District isotherm depth in the Southwestern Indian Ocean is 59
in Bhamini Mandal and finally falls into the Bay of Bengal metres. It is more stable and consistent, and the spatial
near Kalingapatnam. spread is also less.

PYQ Workbook 168


GENERAL GEOGRAPHY

Statement 2 is correct. Ocean mean temperature (OMT) Pair 4 is correctly matched. The Siachen in Jammu and
can be used for predicting whether the total amount of Kashmir (18,000ft.) near the Karakoram Pass is the
rainfall that India receives during the monsoon season longest glacier in the Indian Himalayas. It is 78km long
will be less or more than the long-term mean of 887.5 mm. and the source for the Nubra River that waters the Nubra
Using OMT, the information on whether the amount of Valley near Leh in the Trans Himalayas. The waters of the
monsoon rainfall will be more or less than the long-term Nubra drain into the Shyok River and then into the Indus.
mean will be available by beginning of April, two months Pair 5 is incorrectly matched. The Zemu glacier is the
before the southwest monsoon can set in. This is because largest in the Eastern Himalayas in Sikkim. It is at the base
OMT is analysed by measuring the ocean thermal energy of the Kanchendzonga and is one of the sources for the
during the period from January to March. Southwest Teesta.
monsoon sets in around June 1 each year in Kerala. 15. Solution: (a)
12. Solution: (a) Exp) Option a is the correct answer.
Exp) Option a is the correct answer. Kodaikanal Lake is a manmade lake located in the Kodaikanal
city in Dindigul district in Tamil Nadu, India. Sir Vere Henry
All the sites given above are famous for temple structures
Levinge, the then Collector of Madurai, was instrumental in
and being pilgrimage towns.
creating the lake in 1863, amidst the Kodaikanal town which
Pair 1 is correctly matched. Pandharpur is a pilgrimage was developed by the British and early missionaries from
town in Solapur district of Maharashtra. The famous Vitobha USA.
temple is located here.
Option b is incorrect. Kolleru Lake located in Andhra
Pair 2 is correctly matched. Tiruchirappalli is situated at Pradeshis a Ramsar site. It is located between Krishna and
the head of Cauvery delta, where the river seperates into two Godavari river systems.
streams. Tiruchirappalli or Trichy is home to many famous Option c is incorrect. Nainital lake is a natural fresh water
temples such as Ranganathaswamy temple, Erumbeeswarar lake which is tectonic in origin.
Temple and the Jambukeswarar temple.
Option d is incorrect. Renuka Lake is a designated Ramsar
Pair 3 is incorrectly matched. Classified as a World Heritage site. It is the largest fresh water lake of Himachal Pradesh.
Site by UNESCO, Hampi (Karnataka) situated along the
16. Solution: (a)
banks of the Tungabhadra River, was the capital city of
the Vijayanagara Empire. It is a famous pilgrimage place. Exp) Option a is the correct answer.
Vijaya Vittala Temple and Virupaksha Temple are situated Statement 1 is correct. Barren Island is an active volcano
in Hampi, The Famous stone chariot dedicated to Gadura located in the Andaman Sea in the Union territory of
is also located here. Both Malaprabha and Tungabhadra are Andaman and Nicobar Islands. It is the only confirmed
tributaries of Krisha river. active volcano of India and South Asia. It is a stratovolcano
composed of lava, rock fragments, and volcanic ash. It lies
13. Solution: (d) over the fault whose movement caused the tsunami on
Exp) Option d is the correct answer answer. December 26, 2004.
All the three are water reservoirs. Statement 2 is incorrect. It lies about 140km northeast of
Port Blair, the capital of Andaman and Nicobar Islands. It
Aliyar reservoir (Aliyar river, Tamil Nadu)
lies north of Great Nicobar. It is a tourist destination and
Isapur dam (Penganga river, Maharashtra) surrounded by waters ideal for scuba diving and is home to a
Kangsabati reservoir (Kangsabati river, West Bengal) wide variety of aquatic life.
Statement 3 is incorrect. Historically, the first record of the
14. Solution: (a)
volcano’s eruption dates back to 1787. It was known to have
Exp) Option a is the correct answer. erupted at least five times over the next 100 years. Then there
Pair 1 is correctly matched. Banderpoonch Glacier perched was silence for a century. In 1991, it spewed so massively that
in the Garhwal division of the Himalayas. The Banderpoonch smoke billowed out for about six months. Ever since, there
Glacier is highly significant as it is the prime source of fresh have been eruptions every two-three year. In the recent
water for the Yamuna basin. past, it erupted in February 2016 and in January 2017.
Pair 2 is correctly matched. The Bhara Shigiri glacier in 17. Solution: (a)
the Chandra Valley of Lahaul in Himachal Pradesh feeds the Exp) Option a is the correct answer.
Chenab river.
Longitude is measured by imaginary lines that run around
Pair 3 is incorrectly matched. Milam is a major glacier in the Earth vertically (up and down) and meet at the North
the state of Uttarakhand. Milam is situated in the Kumaon and South Poles. These lines are known as meridians. They
Himalayas, originating from the eastern slopes of the are used to define the East-West position of a location on
Trishuli. The river Gori Ganga has its source here. the planet.

169 PYQ Workbook


GENERAL GEOGRAPHY

Longitude of Delhi- 77.1025° E


Longitude of Bengaluru- 77.5946°E
Longitude of Hyderabad- 78.4867° E
Longitude of Nagpur- 79.0882° E
Longitude of Pune- 73.8567° E

Fig. Route from Kohima (Nagaland) to Kottayam (Kerala).

18. Solution: (c) 21. Solution: (b)

Exp) Option c is the correct answer. Exp) Option b is the correct answer.
Statement 1 is incorrect. Teesta River originates from the
The unique phenomenon can be seen at Chandipur Beach,
Pahunri (or Teesta Kangse) glacier and flows southward
Odisha. It is also known as the “hide-and-seek” beach
through the Sikkim Himalaya. where the Rangpo River
because of this unique phenomenon. The seawater recedes up joins, and where it forms the border between Sikkim
to 5 kilometerstwice a day during low tide, leaving behind and West Bengal. Whereas, The Brahmaputra river rises
a vast expanse of wet sand. This is caused by a combination from Chemayundung glacier of the Kailash range near
of factors, including the gentle slope of the beach, the tidal the Mansarovar Lake to the north of the Himalayas in the
range, and the shape of the coastline. During low tide, the southwest Tibet region.
water is pulled back by the gravitational force of the moon Statement 2 is correct. The Rangeet is a tributary of the
Teesta river, which forms a boundary between Sikkim and
and the sun. The gentle slope of the beach and the shape of
Darjeeling. The Rangeet river originates in the Himalayan
the coastline allow the water to recede a long distance.
mountains in West Sikkim district.
19. Solution: (b) Statement 3 is incorrect. Teesta flows through Sikkim, West
Bengal, and joins Jamuna river at Fulhhari in Bangladesh.
Exp) Option b is the correct answer.
Statement 1 is incorrect. In India, the Himalayas are spread 22. Solution: (b)
over Jammu & Kashmir, Himachal Pradesh, Uttarakhand, Exp) Option b is the correct answer.
Sikkim, Assam, West Bengal and Arunachal Pradesh. Statement 1 is incorrect. The Indian Ocean Dipole -
often called the “Indian Niño” because of its similarity
Statement 2 is incorrect. Western Ghats are spread over
to its Pacific equivalent - refers to the difference in sea-
Gujarat, Maharashtra, Goa, Karnataka, Tamil Nadu, Kerala.
surface temperatures in opposite parts of the Indian Ocean.
Statement 3 is correct. Pulicat Lake is located at border of Temperatures in the eastern part of the Indian Ocean oscillate
Andhra Pradesh and Tamil Nadu. between warm and cold compared with the western part,
cycling through phases referred to as “positive”, “neutral”
20. Solution: (b) and “negative”.
Exp) Option b is the correct answer. The dipole’s positive phase this year means warmer sea
If we follow the shortest route, the minimum number of temperatures in the western Indian Ocean region, with the
opposite in the east. The result of this may be higher-than-
states through which we travel from Kohima (Nagaland) and
average rainfall and floods in eastern Africa and droughts in
Kottayam (Kerala): south-east Asia and Australia.
Nagaland – Assam – West Bengal – Odisha – Andhra Pradesh During a negative IOD, the opposite happens, that is, the
– Tamil Nadu or Karnataka – Kerala. western Indian Ocean becomes cooler with higher air

PYQ Workbook 170


GENERAL GEOGRAPHY

pressures resulting in westerly winds blowing toward the Option 2 is correct. Kameng river also called as Bharali is
Indian subcontinent. a right bank tributrary of Brahmaputra. It originates near
Statement 2 is correct. The Indian Ocean Dipole (IOD) is Indo-Tibet border in Tawang and meets Brahmaputra near
the Indian Ocean counterpart of the Pacific El Niño and La Tezpur in Assam.
Niña. 27. Solution: (a)
Like ENSO, IOD is a coupled ocean-atmosphere phenomenon. Exp) Option a is the correct answer.
The shifting pools of warm/cool water contribute to
variations in rainfall and storm activities of many countries Teak is the most dominant species of Tropical Moist
surrounding the Indian Ocean. Deciduous Forest. Tropical Deciduous Forest are the
most widespread forests of India. They are also called
Positive IODs are often associated with El Niño and negative the monsoon forests and spread over the region receiving
IODs with La Niña. A positive IOD leads to greater monsoon rainfall between 200 cm and 70 cm. Trees of this forest type
rainfall and more active (above normal rainfall) monsoon
shed their leaves for about six to eight weeks in dry summer.
days while negative IOD leads to less rainfall and more
On the basis of the availability of water, these forests are
monsoon break days (no rainfall).
further divided into moist and dry deciduous. The former
is found in areas receiving rainfall between 200 and 100 cm.
These forests exist, therefore, mostly in the eastern part of
23. Solution: (d)
the country — northeastern states, along the foothills of
Exp) Option d is the correct answer. the Himalayas, Jharkhand, West Odisha and Chhattisgarh,
India has identified six basins as areas for shale gas exploration: and on the eastern slopes of the Western Ghats. Teak is the
Cambay (Gujarat), Assam-Arakan (North East), most dominant species of this forest. Bamboos, sal, shisham,
Gondwana (Central India), Krishna Godavari onshore sandalwood, khair, kusum, arjun and mulberry are other
(East Coast), Cauvery onshore, and Indo-Gangetic basin. commercially important species.
As per the US EIA 2015 report, India has got technically
recoverable shale gas of 96 trillion cubic feet. The 28. Solution: (d)
recoverable reserves are identified in Cambay, Krishna Exp) Option d is the correct answer.
– Godavari, Cauvery, Damodar Valley, Upper Assam, Option 1 is incorrect. Vamsadhara originates in Kalahandi
Pranahita – Godavari, Rajasthan and Vindhya Basins. district of Odisha. It finally joins the Bay of Bengal at
Kalingapatnam, Andhra Pradesh. Water sharing of this river
24. Solution: (d)
is a source of dispute between Andhra Pradesh and Odisha.
Exp) Option d is the correct answer.
Option 2 and 3 are correct. The river Godavari is the largest
To develop inland shipping port, Rajasthan is developing of the peninsular rivers. Pranhita is the largest tributary of
a channel along the Kutch Creek at Jalore which would be Godavari.
connected to the Arabian sea. If implemented, Rajasthan
Left Bank Tributaries of Godavari:
will not remain a landlocked state.
Banganga, Kadva, Shivana, Purna, Kadam, Pranahita,
25. Solution: (b) Indravati, Taliperu, Sabari, Dharna
Exp) Option b is the correct answer. Right Bank Tributaries:
The Pattiseema lift irrigation project is a river interlinking Nasardi, Pravara, Sindphana, Manjira, Manair, Kinnerasani
project which connects Godavari river to Krishna river.
Option 4 is incorrect. The Pennar river rises in the Chenna
Pattiseema scheme will help revive Krishna delta.
Kasava hill of the Nandidurg range, in Chikkaballapura
The first pump of this Godavari-Krishna linking project was district of Karnataka and flows towards east eventually
installed and switched on in September 2015. It will benefit draining into the Bay of Bengal.
the people especially farmers in the Krishna delta in terms of
timely availability and adequate quantities of water. 29. Solution: (c)
Under the National Perspective Plan (NPP) prepared by Exp) Option c is the correct answer.
Ministry of Water Resources, NWDA has already identified The tropical wet evergreen forest in India is usually found
14 links under Himalayan Rivers Component and 16 links in areas receiving more than 200 cm of rainfall and having
under Peninsular Rivers Component for inter basin transfer a temperature of 15-30 degrees Celsius. These forests are
of water found in the plains, on low hills and gentle slopes of the
foothills of the mountains, at elevations ranging from almost
26. Solution: (d)
sea level up to about 1000–1200 m. They occur in the states
Exp) Option d is the correct answer. of Maharashtra, Karnataka, Tamil Nadu, Kerala, Andaman
All of the given rivers are tributaries of Brahmaputra. and Nicobar islands in the south and in the entire north-
Option 1 and 3 are correct. Lohit originates in Tibet, flows eastern region including the sub- mountain division of West
through Arunachal Pradesh, before it is joined by Dibang Bengal.
in Assam where it flows into Brahmaputra. The Bhupen The forest of Himachal Pradesh can be classifies into
Hazarika Setu is constructed on River Lohit. nine types: Dry alpine forests, Moist alpine scrub forests,

171 PYQ Workbook


GENERAL GEOGRAPHY

Subalpine forests, Temperate forests, Wet temperate forest, Ten degrees Channel which lies between Little Andaman and
Subtropical Pine forest, Tropical dry dry deciduous forests Nicobar Islands.
and Tropical thorny forests. Thus. it is clear that tropic wet The Channel is approximately 150 km wide, running
evergreen forests are not found in Himachal Pradesh. essentially along an east-west orientation. It is so named as
30. Solution: (d) it lies on the 10-degree line of latitude, north of the equator.
Exp) Option d is the correct answer.
Arunachal Pradesh and Gujarat are the easternmost and
westernmost States of India. Kibithu in Arunachal Pradesh
is the Easternmost point of India. Ghuar Mota in the Kutch
region of Gujarat is the Westernmost point of India. It is
located near the disputed Sir Creek and the Koteshwar
temple, at 23.713°N 68.032°E.

31. Solution: (a)


Exp) Option a is the correct answer.
Pair 1 is correctly matched. Nallamala Hills located in eastern
Andhra Pradesh are a part of Eastern Ghats. Srisailam on the
River Krishna is an ancient temple dedicated to Lord Shiva.
Pair 2 is incorrectly matched. Omkareshwar is a temple
dedicated to Lord Shiva located in Madhya Pradesh, on
Madhata island in Narmada river. Whereas, Satmala is a
mountain range in Nashik District of Maharashtra.
Pair 3 is incorrectly matched. Pushkar in Ajmer district of
Rajasthan has a temple dedicated to Lord Brahma. Pushkar
town is situated in Aravalli hills. Mahadeo hills form the
northern section of Satpura ranges in Madhya Pradesh. 35. Solution: (c)
Exp) Option c is the correct answer
32. Solution: (a)
A monsoon is a seasonal reversal in wind patterns over a
Exp) Option a is the correct answer.
region. The seasonal wind shift is usually accompanied by a
The jing kieng jri or living root bridges — aerial bridges built dramatic change in precipitation. The phenomenon is best
by weaving and manipulating the roots of the Indian rubber developed over the Indian subcontinent.
tree — have been serving as connectors for generations in
During winter, there is a high-pressure area north of the
Meghalaya. Spanning between 15 and 250 feet and built over Himalayas. Cold dry winds blow from this region to the
centuries, the bridges, primarily a means to cross streams and low-pressure areas over the oceans to the south. In summer,
rivers, have also become world-famous tourist attractions. a low-pressure area develops over interior Asia, as well as,
33. Solution: (c) over north-western India. Thus, wind in India blows from
the northeast during cooler months and reverses direction
Exp) Option c is the correct answer.
to blow from the southwest during the warmest months of
Pair 1 is incorrectly matched: The Cardamom Hills or Yela the year.
Mala are mountain range of southern India and part of the
southern Western Ghats located in Idukki district, Kerala, 36. Solution: (a)
India. Exp) Option a is the correct answer.
Pair 2 is incorrectly matched: Kaimur Range is the eastern Option 1 and 2 are correct. The Eastern Himalayan
portion of the Vindhya Range, about 483 kilometres (300 broadleaf forests are diverse and species-rich, with a
mi) long, extending from around Katangi in Jabalpur district great diversity of oaks and rhododendrons in particular.
of Madhya Pradesh to around Sasaram in Rohtas district of The ecoregion has two broad forest types: evergreen and
Bihar. deciduous. Evergreen forests are characterized by oaks
Pair 3 is correctly matched: The Mahadeo Hills are a range together with Rhododendrons. In Western Himalayas, the
of hills in Madhya Pradesh state of central India. The hills evergreen broadleaf forests is dominated by oaks.
are situated in the northern section of the Satpura Range. Option 3 is incorrect. Sandalwood is an evergreen tree
Pair 4 is correctly matched: Karbi plateau or Mikir Hills which generally grows in the tropical dry, deciduous forests
is known oldest landform in Assam, North-east India. It is of Karnataka, Tamil Nadu, Kerala and Andhra Pradesh.
pear-shaped and has an area of about 7000 sq. km. It is part 37. Solution: (b)
of the Karbi-Plateau. Its highest peak is Dambuchko.
Exp) Option b is the correct answer.
34. Solution: (a) Option 1 is incorrect. Barak originates in Manipur, flows
Exp) Option a is the correct answer. through Mizoram and Assam and drains into Bangladesh.

PYQ Workbook 172


GENERAL GEOGRAPHY

Option 2 and 3 are correct. Lohit and Subansiri are a 41. Solution: (c)
tributarries of the Brahmaputra River. They originate in Exp) Option c is the correct answer.
Tibet and flow through Arunachal Pradesh, before joining
Statement 1 is correct. Laterite soil are leached soils which
Brahmaputra in Assam.
develops in areas with high temperature and heavy rainfall.
38. Solution: (b) These Soils are acidic in nature and coarse and crumbly in
Exp) Option b is the correct answer. texture. The high iron oxide content gives it a red-rusty
colour.
Statement 1 is incorrect. The Changpa are a semi-nomadic
Tibetan people. They are mainly found in the Changtang, a Statement 2 is incorrect. Laterite soils lack nitrogen,
high plateau that stretches across the cold desert of Ladakh, potassium and organic elements, which makes these soils
Jammu and Kashmir. unsuitable for cultivation.
Statement 2 is correct. They usually stay in one place for a Statement 3 is incorrect. The laterite soils are commonly
few months in a row, near pastures where their sheep, yaks found in Karnataka, Kerala, Tamil Nadu, Madhya Pradesh
and Pashmina goats can graze. and the hilly areas of Odisha and Assam.
Statement 3 is correct. In 1989 the Changpa were granted Statement 4 is correct. With proper irrigation and use of
official status in India as a Scheduled Tribe. fertilizers make it suitable for growing crops, such as cashew,
tea, coffee, rubber, cinchona, coconut, and Tapioca.
39. Solution: (a)
Exp) Option a is the correct answer 42. Solution: (c)

Statement 1 is correct. The Narmada and the Tapi flow Exp) Option c is the correct answer.
through the rift valley. The Narmada and The Tapti flow in Statement 1 is correct. The word monsoon is derived from
trough faults. They along with many small rivers discharge the Arabic word ‘Mausim’ which means season. Monsoon
their waters in the Arabian Sea. refers to the seasonal reversal in the wind direction during
Statement 2 is incorrect. Narmada flows between Vindhyas a year. The Southern part of India, being near to the sea,
in north and the Satpuras in south but this is not the reason gets rainfall early and for a longer duration than in the
for flowing towards West. northern part. As rain-bearing winds cross western ghats
humidity decreases and so does rainfall in the northern
Statement 3 is incorrect. The Peninsular plateau consists of
part of India. The Southern part of India receives rainfall
two broad divisions, namely, the Central Highlands and the
from the southwest monsoon and the retreating southwest
Deccan Plateau. The part of the peninsular plateau lying to
monsoon (northeast monsoon).
the north of the Narmada river, covering a major area of the
Malwa plateau, is known as the Central Highlands. The flow Statement 2 is correct. The Bay of Bengal is the main branch
of the rivers draining this region, namely the Chambal, the of the monsoon winds which moves from northeast and then
Sind, the Betwa and the Ken is from southwest to northeast, hits Himalayas to return westwards by covering the northern
thus indicating the slope. Thus, the slope is towards plains. As the winds move westward, their moisture contains
northeast. tends to reduce as they cause rains along Indian subcontinent.
So that the rainfall decreases from east to west in northern
40. Solution: (a) India. The Arabian Sea branch does not contribute much and
Exp) Option a is the correct answer. exhaust most of its moisture when it hit Western Ghats.
Pair 1 is correctly matched. Limboo tribe occupy
43. Solution: (d)
the region in eastern Nepal and western Sikkim.
Pair 2 is incorrectly matched. Karbi people are the main Exp) Option d is the correct answer.
tribe of Karbi Anglong and West Karbi Anglong districts of The Himalayas began forming in the Upper Cretaceous
Assam. period (60 million years ago). Young fold mountains have
The scheduled tribes in Himachal Pradesh includes Gaddis, not yet undergone extensive erosional process of exogenic
Gujjars, Kinnara or Kinnauras, Lahaulas and Pangwalas forces, thus they have deep gorges, U-turn river courses,
apart from some other smaller tribes groups like Bhot/Bodh, Parallel mountain ranges and steep gradients.
Beda, Jad/Lamba/ Khampa, and Swangla. These features are generally absent in old fold mountains
Pair 3 is correctly matched. The Niyamgiri hill range in Odisha which are characterized by lower altitude, rounded peaks,
state, eastern India, is home to the Dongria Kondh tribe. The gentle slopes and rolling hills.
Dongria have repeatedly expressed opposition to efforts
44. Solution: (a)
to mine in Niyamgiri, recently against Vedanta Resources.
Pair 4 is incorrectly matched. Bondas are a Particularly Exp) Option a is the correct answer.
Vulnerable Tribal Groups (PVTGs) found in Odisha. They Arunachal Pradesh and Northern Rajasthan lie on same
live in the isolated hill regions of the Malkangiri district latitude at around 28°N. Approximately, 82 percent area of
of southwestern Odisha near the junction of the three states Arunachal Pradesh is under forest cover and over 12% of the
of Odisha, Chhattisgarh, and Andhra Pradesh. forest over constitutes Protected Area Network.

173 PYQ Workbook


GENERAL GEOGRAPHY

45. Solution: (b) 50. Solution: (d)


Exp) Option b is the correct answer. Exp) Option d is the correct answer
Syntaxial bending is the primary reason for the sudden “U” All of the methods mentioned are appropriate methods for
turn of Brahamaputra when it enters India. soil conservation in India.
46. Solution: (b) • Crop rotation is a method of planting different crops
in the same field in a planned sequence. This helps to
Exp) Option b is the correct answer.
prevent soil depletion and improve soil fertility.
The climate of North of Gujarat is arid and semi-arid
whereas the southern Gujarat comes under the influence of • Sand fences are barriers made of sand, straw, or other
monsoon winds. Gujarat is the leading producer of cotton. materials that are used to trap wind-blown sand. They
Gujarat’s agricultural economy mainly produces cash crops are used to prevent desertification and to protect crops
like peanut, cotton, tobacco. and land from wind erosion.
• Terracing is a method of farming that involves building
47. Solution: (a)
steps on the side of a hill. This helps to level the land and
Exp) Option a is the correct answer. prevent the soil from eroding.
Bhitarkanika Wildlife Sanctuary is located in the estuarial • Windbreaks are rows of trees or shrubs that are planted
region of Bramhani-Baitarani, in the north-eastern place of
to protect crops and land from wind erosion.
Kendrapara district of Odisha (Orissa).
Important Tips
48. Solution: (b)
Other methods for soil conservation in India:
Exp) Option b is the correct answer.
• Contour farming: Plowing and planting crops along
Orchids require cold temperature and moist environment the contours of the land to help slow down the flow
with indirect sunlight to grow. In Arunachal Pradesh, the of water.
richest concentration of orchids is believed to be in Ziro
• Strip cropping: Planting different crops in strips next
Valley. Arunachal Pradesh alone accounts for 52% of total to each other to help break up the wind and water.
species of Orchids in the country.
• Mulching: Covering the soil with organic material,
Government of India has recognized orchids as the prime such as straw or leaves, to help protect the soil from
plant group for promoting export oriented activity in NE the wind and water.
region.
• Afforestation: Planting trees and shrubs to help hold
49. Solution: (d) the soil in place.
Exp) Option d is the correct answer. 51. Solution: (c)
Statement 1 is correct: The Himalayan Mountain range acts Exp) Option c is the correct answer
as a natural barrier, shielding northern India from the cold
Option c is correct. The main reason for the red color of soil
winds and cold waves that come from Siberia. If there were
in certain parts of India is the presence of ferric oxides. Ferric
no Himalayas, the Siberian winds would have swept across
India, making the winters much colder. oxides are iron oxides that give the soil its characteristic red
color.
Statement 2 is correct: The Himalayas are responsible for the
formation of the Indo-Gangetic plain’s rich alluvial soils. The Option a is incorrect. Magnesium is not found in high
mountains have been eroding over millions of years, and the quantities in red soil.
sediment carried by rivers like the Ganges and Brahmaputra Option b is incorrect. Humus is a dark brown or black
has been deposited in the plains, creating highly fertile soils. substance that is formed from the decomposition of organic
Without the Himalayas as a source of sediment, the Indo- matter. It is not found in high quantities in red soil.
Gangetic plain would not have the same extent of fertile
Option d is incorrect. Phosphates are found in low quantities
alluvial soils.
in red soil.
Statement 3 is correct: The Himalayas also trap the monsoon
rains. The monsoon rains are caused by the difference in Important Tips
temperature between the land and the sea. The land heats Here are some other characteristics of red soil:
up faster than the sea, and this creates a low-pressure area • It is a loamy soil that is low in organic matter and
over the land. The monsoon winds blow from the sea to nutrients.
the land to fill this low-pressure area. The Himalayas force • It is not as fertile as other types of soil, but can be used
the monsoon winds to rise, which cools them and causes for growing a variety of crops.
them to drop their moisture. If there were no Himalayas,
• It is prone to erosion.
the monsoon rains would have been much less, and India
would have been a much drier country. • It can be improved by adding lime and fertilizers.

PYQ Workbook 174


GENERAL GEOGRAPHY

52. Solution: (c) Pair 2 is incorrectly matched: Son is a tributary of the


Exp) Option c is the correct answer. Ganges, not the Yamuna.
In India, the largest area is occupied by Tropical Moist Pair 3 is correctly matched: Manas is a tributary of the
Deciduous Forest. Around 65 % forest area of India is Brahmaputra.
covered by deciduous forests. Moist Deciduous Forests 58. Solution: (a)
cover approximately 37% of the total Forest area. Tropical
Exp) Option a is the correct answer.
Deciduous Forests, also called the Monsoon forests occupies
highest percentage of area in India. They grow in the areas The Nilgiri Mountains form part of the Western Ghats in
with annual rainfall of 70-200 cm. northwestern Tamil Nadu, Southern Karnataka, and eastern
Kerala in India. They are located at the trijunction of
53. Solution: (d) the three states and connect the Western Ghats with the
Exp) Option d is the correct answer. Eastern Ghats. They are known for their scenic beauty and
The Luni River is known for being lost in the marshy are home to several national parks, including Mudumalai,
land of the Rann of Kutch. It originates from the western Mukurthi, and Silent Valley. The Nilgiri Hills are part of
slopes of the Aravalli ranges near Ajmer, flows for about 511 the Nilgiri Biosphere Reserve (itself part of the UNESCO
kilometers in Rajasthan, and ultimately dissipates into the World Network of Biosphere Reserves) and form a part of
marshy expanses of the Rann of Kutch, where it does not the protected bio-reserves in India.
have a significant outlet, causing it to be lost in the marshy 59. Solution: (c)
terrain.
Exp) Option c is the correct answer.
54. Solution: (d) The Indian coastline is shared by nine states and four union
Exp) Option d is the correct answer. territories. These states include Gujarat, Maharashtra,
The Sutlej River does not originate in India. It originates Goa, Karnataka, Kerala, Tamil Nadu, Andhra Pradesh,
close to Lake Rakhastal in Tibet in the vicinity of the Odisha, and West Bengal. Additionally, the four union
Kailash Mountain Range and then flows through the regions territories with coastlines are Daman and Diu, Andaman &
of Himachal Pradesh and Punjab in India before entering Nicobar Islands, Lakshadweep Islands and Puducherry.
Pakistan and eventually joining the Indus River. In contrast, 60. Solution: (a)
the Beas, Chenab, and Ravi rivers all have their origins in
Exp) Option a is the correct answer.
India’s Himalayan region.
Guru Shikhar, a peak in the Arbuda Mountains of Sirohi
55. Solution: (a) district in Rajasthan, is the highest point of the Aravalli
Exp) Option a is the correct answer. Range, Rajasthan, and Western India. It rises to an elevation
Amarkantak, located in Anuppur, Madhya Pradesh, is a of 1,722 metres (5,650 ft). It is named Guru-Shikhar or ‘the
significant geographical point where two important rivers peak of the guru’ after Dattatreya, an incarnation of Vishnu,
of India originate. The Narmada River begins its journey and hosts a temple dedicated to him. Additionally, the Mt.
towards the Arabian Sea, while the Son River and its Abu Observatory nearby conducts astronomy research.
tributary, the Johilla River, flow northward towards the
Ganges, eventually merging with the Ganges River as
it flows towards the Bay of Bengal. This unique location
marks the confluence of these river systems.

56. Solution: (d)


Exp) Option d is the correct answer.
Statement 1 is incorrect: There are indeed east-flowing
rivers in Kerala. The state has three major east-flowing rivers:
the Kabini, Bhavani, and Pambar. These rivers eventually
join the Kaveri River.
Statement 2 is incorrect: Madhya Pradesh does have west-
flowing rivers. The Narmada and Tapi rivers, two of the
prominent west-flowing rivers in Peninsular India, flow
through Madhya Pradesh before emptying into the Arabian
Sea. 61. Solution: (c)
Exp) Option c is the correct answer.
57. Solution: (d)
Option a is incorrect: The longitude of Hyderabad is 78.476°
Exp) Option d is the correct answer. E
Pair 1 is incorrectly matched: Chambal is a tributary of the Option b is incorrect: The longitude of Bhopal is 77°
Yamuna, not the Narmada. 24’45.4068’’ E.

175 PYQ Workbook


GENERAL GEOGRAPHY

Option c is correct: From the given options, Lucknow is Important Tips


the most eastward located. Lucknow is the capital of Uttar
Here are some other reasons why the percentage of net
Pradesh. It is located at 80°58’59.8764’’ E.
sown area in Andhra Pradesh is less than that of West
Option d is incorrect: The longitude of Bangalore is 77°
Bengal:
35’40.4376’’ E.
• The terrain of Andhra Pradesh is more hilly and
62. Solution: (d) mountainous than that of West Bengal.
Exp) Option d is the correct answer. • Andhra Pradesh has a longer coastline, which is less
Assertion (A) is False: The Kali River is a west-flowing river suitable for cultivation.
in the southern part of India. It rises in the Western Ghats • Andhra Pradesh has a more humid climate than
and flows into the Arabian Sea near the town of Karwar. West Bengal, which can lead to waterlogging and soil
Reason (R) is true: The Deccan Plateau exhibits an elevation erosion.
gradient from west to east. Its western edge, marked by • These factors, along with the poor quality of the
the Western Ghats Mountain range, is higher in altitude, laterite soil, have contributed to the lower percentage
while the terrain gradually descends towards the east, of net sown area in Andhra Pradesh.
culminating in the coastal plains along the Bay of Bengal.
This topographical variation influences regional climate, 66. Solution: (a)
vegetation, and river systems.
Exp) Option a is the correct answer.
63. Solution: (b) Statement 1 is Correct:Assam Shares Inter-national
Exp) Option b is the correct answer. boundary with Bangladesh and Bhutan.
The correct matching of the Towns (List-I) with the Statement 2 is Correct: West Bengal shares Inter-national
respective nearby rivers (List-II) is as follows:
boundary with Nepal, Bhutan, and Bangladesh.
A. Betul - 4. Tapti
Statement 3 is Correct: Mizoram shares Inter-national
B. Jagdalpur - 1. Indravati
boundary with Myanmar and Bangladesh.
C. Jabalpur - 2. Narmada
D. Ujjain - 3. Kshipra

Important Tips
• Betul district is a district of Madhya Pradesh state in
central India. The Tapti River originates from Multai
in the Betul district.
• Jagdalpur is a city in Bastar district in the Indian state
of Chhattisgarh. It is located on the banks of the
Indravati River.

64. Solution: (c)


Exp) Option c is the correct answer.
The river Cauvery originates at Talakaveri in Coorg
District of Karnataka in Brahmagiri Range of hills in
the Western ghats at an elevation of 1341 m. (above MSL)
and then flows through Karnataka and Tamil Nadu before
emptying into the Bay of Bengal. It does not originate in
Andhra Pradesh.

65. Solution: (c)


Exp) Option c is the correct answer
Assertion (A) is correct. The percentage of net sown area
in the total area of Andhra Pradesh is 40.7%, while the
percentage of net sown area in the total area of West Bengal 67. Solution: (b)
is 60.4%.
Exp) Option b is the correct answer.
Reason (R) is incorrect. Most of the soil in Andhra Pradesh
is red soil and black soil. Also, Andhra Pradesh has deltaic The correct sequence of the rivers from north to south is
alluvial soil and laterite soil. Shyok – Zaskar – Spiti – Satluj.

PYQ Workbook 176


GENERAL GEOGRAPHY

• Dzüko Valley, located between Manipur’s Senapati


district and Nagaland’s Kohima district in
Northeast India, is renowned for its stunning natural
surroundings, seasonal flowers, and diverse flora and
fauna. Notably, it hosts the famous “Dzükou Lily”
(Lilium chitrangadae) and stands at an altitude of
2,452 meters (8,045 feet) above sea level.
• Sangla is a town in the Baspa Valley, also referred
to as the Sangla valley, in the Kinnaur District
of Himachal Pradesh, India, close to the Tibetan
border. It serves as a hub for exploration and trekking,
featuring attractions like Kamru Fort and Kupa Lake
Important Tips
and Dam.
Shyok River:
• The Yumthang Valley or Sikkim Valley of Flowers
• Rising from the Karakoram Range, it flows through sanctuary, is a nature sanctuary with river, hot
the Northern Ladakh region in J&K. springs, yaks and grazing pasture on rolling meadows
• It has a length of about 550km. surrounded by the Himalayan mountains in the North
• A tributary of the Indus River, it originates from the Sikkim district of Sikkim state in India.
Rimo Glacier. The river widens at the confluence with
the Nubra River. 69. Solution: (c)
• Shyok River marks the south-eastern fringe of the Exp) Option c is the correct answer.
Karakoram ranges by forming a V-shaped bend around The correct sequence of the given Indian cities in the
it. decreasing order of their normal annual rainfall is Kochi
Spiti River: (3228.3 mm) – Kolkata (1,582 mm) – Patna (1000 –1237
• Spiti river is a 225 kilometres (140 mi) long tributary mm) – Delhi (707 mm).
of Sutlej River. • Kochi: Kochi has a tropical wet and dry climate, which
• The Spiti River nurtures the beautiful valleys of means it has hot and humid summers and mild winters.
Lahaul-Spiti district of Himachal Pradesh. The average annual temperature is 27.3 °C, and the
average annual rainfall is 3228.3 mm. Kochi receives
• It originates from the Kunzum Range of the
Himalayas in a way that divides Lahaul and Spiti. most of its rainfall from the southwest monsoon, which
lasts from June to September.
Zanskar River:
• Kolkata: Kolkata also has a tropical wet and dry climate,
• The Zanskar River is the first major tributary of the
but it is slightly cooler and drier than Kochi. The average
Indus River, equal or greater in volume than the main
annual temperature is 26.8 °C, and the average annual
river, which flows entirely within Ladakh.
rainfall is 1582 mm. Kolkata receives rainfall from both
• It originates northeast of the Great Himalayan range
the southwest and northeast monsoons, which occur
and drains both the Himalayas and the Zanskar
from June to September and from October to December
Range within the region of Zanskar.
respectively.
• It flows northeast to join the Indus River near Nimo.
• Patna: Patna has a humid subtropical climate, which
68. Solution: (d) means it has hot summers and cool winters. The average
Exp) Option d is the correct answer. annual temperature is 26.4 °C, and the average annual
rainfall is 1116 mm. Patna receives rainfall from both
The correct matching of the valleys with their respective
the southwest and northeast monsoons, which occur
states is as follows:
from June to September and from October to December
• A. Markha Valley - 3. Jammu and Kashmir respectively.
• B. Dzukou Valley - 4. Nagaland • Delhi: Delhi has a semi-arid climate, which means it has
• C. Sangla Valley - 2. Himachal Pradesh very hot summers and cold winters. The average annual
• D. Yumthang Valley - 1. Sikkim temperature is 25.1 °C, and the average annual rainfall
is 707 mm. Delhi receives most of its rainfall from the
Important Tips southwest monsoon, which lasts from July to September.
• Markha Valley, situated in Ladakh’s arid cold desert
70. Solution: (a)
within Hemis National Park, offers a breathtaking
trekking experience. Starting from Ganda La pass Exp) Option a is the correct answer.
near Spituk in the west and concluding near Hemis at The Western Ghats are relatively high in their southern
Gongmaru La pass, it’s a renowned trekking route in region, not the northern region. The elevation of the
Ladakh. Western Ghats gradually decreases as one moves northward.

177 PYQ Workbook


GENERAL GEOGRAPHY

Important Tips Park in Palakkad district, Kerala. The project, sanctioned in


2007, aims to build a 64.5-meter-high dam on the Kunthi
• The Anamudi peak is the highest peak in the Western
river, with a storage capacity of 0.872 million cubic meters,
Ghats, with an elevation of 2,695 meters (8,842 feet).
to generate 70 MW of power.
It is the highest point in India south of Himalayas.
Thus, it is known as “Everest of South India”. Statement 3 is correct: The Kunthi river is a waterway
located in the state of Kerala, India. It flows through the
• Tapti river lies to the south of Satpura and North of
Silent Valley National Park. This river is also called as
Ajanta ranges.
Thuthapuzha by the local population. Kunthi river is one of
• The Narmada and Tapi are two old rift valleys formed the main tributaries of the Bharathapuzha River, the second-
due to faulting of the land and drained to the west to
longest river in Kerala.
join in Arabian Sea.
Important Tips
Additional Facts on Silent Valley National Park:
• Lion tailed macaques is the flagship species of the
Park.
• The indigenous tribal groups that live within park
boundaries include Irulas, Kurumbas, Mudugas
and Kattunaikkars, the ethnic heritage of these
communities is well protected.
• Karimpuzha Wildlife Sanctuary, New Amarambalam
Reserved Forest, and Nedumkayam Rainforest in
71. Solution: (c) Nilambur Taluk of Malappuram district, Attappadi
Reserved Forest in Mannarkkad Taluk of Palakkad
Exp) Option c is the correct answer.
district, and Mukurthi National Park of Nilgiris
The correct sequence of the given hills, starting from the district, are located around Silent Valley National
north and going towards the south, is: Park.
• Nallamalai Hills
73. Solution: (b)
• Javadi Hills
Exp) Option b is the correct answer.
• Nilgiri Hills
Nanda Devi peak forms a part ofGarhwal Himalayas.
• Anaimalai Hills
Nanda Devi is the third-highest mountain in India, after
Kangchenjunga and K2, and the highest located entirely
within the country. Nanda Devi is the 23rd-highest peak in
the world. It is located in Chamoli district of Uttarakhand,
between the Rishiganga valley on the west and the
Goriganga valley on the east. Nanda Devi, meaning “Bliss-
Giving Goddess,” is revered as the patron goddess of the
Garhwal and Kumaon Himalayas. To safeguard its fragile
ecosystem and religious importance, India prohibited access
in 1983. Nanda Devi National Park has been a UNESCO
World Heritage Site since 1988.

72. Solution: (c)


Exp) Option c is the correct answer.
Statement 1 is incorrect: Silent Valley National Park,
situated in Kerala’s Nilgiri hills, spans 89.52 km². It lies
at the border of Mannarkkad Taluk in Palakkad district,
Nilambur Taluk in Malappuram district, Kerala, and Nilgiris
district, Tamil Nadu. Whereas Nallamalla Range are a section
of the Eastern Ghats which forms the eastern boundary
of Rayalaseema region of the state of Andhra Pradesh and 74. Solution: (c)
Nagarkurnool district of the state of Telangana, in India. Exp) Option c is the correct answer.
Statement 2 is correct: The Pathrakkadavu Hydroelectric Statement 1 is incorrect: Indore is located at a longitude of
project (PHEP) is proposed near the Silent Valley National approximately 75.8577° East. Bhopal is located at a longitude

PYQ Workbook 178


GENERAL GEOGRAPHY

of approximately 77.4126° East. The longitude of Jabalpur temperature is 18.1°C.Allahabad has a warm temperate
is approximately 79.9414° East. Thus, the longitude of climate. The Köppen-Geiger system classifies it as the
Jabalpur’s location is to the east of Bhopal. It is not located Csa type.
between Indore and Bhopal. • Cherrapunji has an average temperature of 17.3°C and
Statement 2 is correct: Vadodara is located at a latitude of gets an annual rainfall of 463.7 in. Its annual range of
approximately 22.3072° North. Pune is located at a latitude of temperature is around 8°C.Cherrapunji has a warm
approximately 18.5204° North. The latitude of Aurangabad temperate climate. It is classified as the Cwb by the
is approximately 19.8762° North. Thus, the latitude of Köppen-Geiger system. Being very close to Mawsynram,
Aurangabad lies between the latitude of Vadodara and Pune. it also receives heavy rainfall.
Statement 3 is correct:Bengaluru, is located at a latitude • Kolkata has an average temperature of 26.2°C and gets an
of approximately 12.9716° North.Chennai is located at a annual rainfall of 68.3 in. It’s annual range of temperature
latitude of approximately 13.0827° North. Thus, Bangalore is 10.9°C.Kolkata’s climate is of tropical climate type. It is
lies on a more southern latitude than Chennai. classified as the Aw type by the Köppen-Geiger climate
classification system.
75. Solution: (b)
Exp) Option b is the correct answer. 78. Solution: (d)
Option a is incorrect: The distance between Delhi and the Exp) Option d is the correct answer.
Tropic of Cancer is 580 km. Open stunted forests with bushes and having long roots
Option b is correct: The distance between Kolkata and the and sharp thorns or spines are commonly found in Western
Tropic of Cancer is 97 km. From the given option it is closest Andhra Pradesh. Such types of forests are commonly found
to Tropic of Cancer. in areas where the rainfall is less than 80 cm. Such areas are
found in Rajasthan, Gujarat, and western Andhra Pradesh.
Option c is incorrect: The distance between Jodhpur and
Western Andhra Pradesh is the rain shadow region of the
the Tropic of Cancer is 315 km.
Western Ghats.
Option d is incorrect: The distance between Nagpur and the
Tropic of Cancer is 255 km. 79. Solution: (d)
Exp) Option d is the correct answer.
The mangroves and their locations:
• Achra Ratnagiri mangroves are found in Maharashtra
• Coondapur mangroves are found in Karnataka
• Pichavaram mangroves are found in Tamil Nadu
• Vembanad mangroves are found in Kerala
Important Tips
Mangroves:
• Mangroves are a unique type of coastal ecosystem
found in tropical and subtropical regions. They are
dense forests of salt-tolerant trees and shrubs that
76. Solution: (c)
thrive in intertidal zones, where land meets the sea.
Exp) Option c is the correct answer.
• Mangroves exhibit Viviparity mode of reproduction,
The Periyar Lake, also known as Thekkady Lake, is not a where seeds germinate within the tree before falling to
lagoon. It was created artificially when the Mullaperiyar the ground.
Dam was constructed in 1895, forming a reservoir within
• Mangrove plants have special roots like prop roots and
the Periyar National Park & Wildlife Sanctuary. While
pneumatophores, which help impede water flow and
it is a significant water body for wildlife conservation and
provide support in the challenging tidal environment.
boating, it is distinct from natural lagoons like Ashtamudi,
• According to the Indian State Forest Report 2021,
Chilika, and Pulicat lakes.
Mangrove cover in India is 4992 sq. Km which is 0.15%
77. Solution: (b) of the country’s total geographical area.
Exp) Option b is the correct answer. • Sunderban in West Bengal is the largest mangrove
Chennai has an average temperature of 26°C and gets an forest region in the world.
annual rainfall of 63 cm. It’s annual range of temperature • West Bengal has approximately 40% of India’s
is 9°C.Chennai’s climate is tropical. It is classified by the mangrove forests.
Köppen-Geiger system as the Aw type of climate. • The second largest mangrove forest in India is
• Allahabad has an average temperature of 25.7 °C and Bhitarkanika in Odisha created by the two river deltas
gets an annual rainfall of 38.6 in. It’s annual range of of River Brahmani and Baitarani.

179 PYQ Workbook


GENERAL GEOGRAPHY

• Top states/UTs having maximum mangrove forest Some people in Manipur live in houses built on floating
area are - West Bengal, Gujarat, A&N Islands, islands of weeds and decaying vegetationheld together by
Andhra Pradesh suspended silt. These islands are called Phumdi. A Phumdi
is found in Loktak Lake of Manipur in the world.Loktak
• Government Initiatives Related to Mangrove
Lake is the freshwater lake of Manipur.
Conservation:
• MISHTI (Mangrove Initiative for Shoreline 84. Solution: (a)
Habitats & Tangible Incomes) initiative Exp) Option a is the correct answer.
• SAIME (Sustainable Aquaculture In Mangrove • 1 represents Kosi River
Ecosystem) initiative • 2 represents Gomati River
80. Solution: (a) • 3 represents Ghaghara River
Exp) Option a is the correct answer • 4 represents Gandak River
The correct order of the process of soil erosion is:
Splash erosion --> Sheet erosion --> Rill erosion --> Gully
erosion.
• Splash erosion is the smallest and most initial form of
soil erosion. It occurs when raindrops hit the soil surface
and dislodge small particles of soil. These particles are
then carried away by wind or water.
• Sheet erosion is a more advanced form of soil erosion. It
occurs when the top layer of soil is removed by water or
wind. Sheet erosion can be prevented by planting cover
crops and using conservation practices, such as contour
farming. Important Tips

• Rill erosion is a more serious form of soil erosion. It Ghaghara River:


occurs when small channels are formed in the soil by • The Ghaghara River, alternatively known as
water runoff. Rill erosion can be prevented by using Karnali or Kauriala, originates from the glaciers of
conservation practices, such as terracing and strip Mapchachungo on the Tibetan plateau near Lake
cropping. Mansarovar.
• Gully erosion is the most serious form of soil erosion. • It traverses through the Himalayas in Nepal and
It occurs when large channels are formed in the soil enters India, where it is joined by the Sharda River
by water runoff. Gully erosion can be very difficult to at Brahmaghat.
control and can lead to the complete loss of soil. • The Ghaghara is a major left-bank tributary of the
81. Solution: (b) Ganga River and eventually joins it at Chhapra in
Bihar.
Exp) Option b is the correct answer.
• The total length of the Ghaghara River is approximately
The shaded area of the map corresponds to the southern
1080 kilometers.
peninsula of India, which has a tropical humid and dry
climate. The mean temperature for the month of July varies • It serves as the primary source of water for the Bara-
between 25.00C and 27.50C in this region because it is Banki District of Uttar Pradesh.
influenced by the southwest monsoon winds that bring moist • Key tributaries of the Ghaghara include Rapti, Chhoti
air from the Arabian Sea and the Bay of Bengal. Gandak, Sharda, and Sarju.
Kosi River:
82. Solution: (a)
• Also known as Saptakoshi, the Kosi River is an
Exp) Option a is the correct answer.
antecedent transboundary river that flows through
The Damodar River, which flows in a south-easterly Nepal and India.
direction, has a rift valley due to down warping. The
• Several of its tributaries, such as the Arun, Sun Kosi,
Damodar River Valley is a rift valley that was formed due to
and Bhote Koshi, originate in Tibet.
down warping of the Earth’s crust. The down warping was
caused by the movement of the Indian Plate towards the • This 729-kilometer-long river is one of the largest
Eurasian Plate. It originates in the Palamau hills of Chota tributaries of the Ganga and joins it at Kursela in
Nagpur and traverses through this rift valley, eventually Kathiar district.
joining the Hooghly River. • Mt. Everest and Kanchenjunga, the world’s highest
peaks, are situated in the Kosi catchment area.
83. Solution: (c)
• Bagmati is a significant tributary.
Exp) Option c is the correct answer.

PYQ Workbook 180


GENERAL GEOGRAPHY

• The Kosi River has shifted its course more than 120 Important Tips
kilometers from east to west in the last 250 years, • Deccan Traps: The Deccan Traps, a massive volcanic
earning it the nickname “The Sorrow of Bihar” due feature in west-central India, formed during the Late
to its unstable nature and heavy monsoon silt. Cretaceous to Early Eocene era, approximately 67
Gandak River: to 63 million years ago. It comprises extensive layers
of solidified flood basalt, covering around 500,000
• It is formed by the union of the Kali and Trisuli rivers,
square kilometers and with a volume of approximately
which rise in the Great Himalayan Range in Nepal.
1,000,000 cubic kilometers.
From this junction to the Indian border the river is
called the Narayani. • Western Ghats: The Western Ghats are a mountain
range that runs along the western coast of India. The
• It enters the Ganga River opposite Patna in a place Western Ghats, the mountainous edge of the Deccan
called Sonepur after a winding course of 765 km. Plateau, took shape during the Late Cenozoic era,
• The Burhi Gandak flows parallel to and east of the around 150 million years ago, as a result of the
Gandak River. Gondwana supercontinent breakup. This geologically
• The upper catchment area of the river is bleak and rich region features predominately basalt rocks, along
desolate, lying in the rain shadow area of the Himalayan with various other rock types and valuable mineral
range. deposits.
• Aravalli Mountains: The Aravalli Mountains are
Rain shadow region, Chhota Nagpur plateau and Rift valley
a mountain range that runs through the states of
region
Rajasthan, Haryana, Gujarat and Delhi. They are
85. Solution: (c) the oldest mountain range in India, and they were
Exp) Option c is the correct answer. formed in the Pre-Cambrian era, about 500 million
years ago. The Aravalli Mountains are made up of a
A. The rift valley region: A rift valley is a linear shaped variety of rocks, including sandstone, limestone, and
lowland between several highlands or mountain ranges quartzite.
produced by the action of a geologic rift. Rifts are formed
• Narmada-Tapti alluvial deposits: The Narmada-Tapti
as a result of the pulling apart of the lithosphere due to
alluvial deposits are a group of sedimentary rocks that
extensional tectonics. Region A is a rift valley between
were formed in the Pleistocene era, about 2 million
the Satpura in the south and the Vindhyan range in the
years ago. They are made up of sand, gravel, and clay.
north. Narmada river flows in this rift valley.
The Narmada-Tapti alluvial deposits are found in the
B. The Chhattisgarh plain: The Chhattisgarh Plain is a states of Madhya Pradesh, Gujarat, and Maharashtra.
wide plain in central India. About 100 miles (160 km)
wide, it is bounded by the Chota Nagpur plateau to the 87. Solution: (c)
north, the Raigarh hills to the northeast, the Raipur Exp) Option c is the correct answer.
Upland to the southeast, the Bastar plateau to the south, A. Chennai is warmer than Kolkata because it is closer to
and the Maikala Range to the west. the equator and has a lower latitude. Kolkata is farther
C. The Chhota Nagpur plateau: The Chota Nagpur Plateau from the equator and has a higher latitude, which means
is a plateau in eastern India, which covers much of it receives less solar radiation and has a lower average
Jharkhand state as well as adjacent parts of Chhattisgarh, temperature.
Odisha, West Bengal and Bihar. B. Snowfall in the Himalayas occurs because of the high
D. The rain shadow region:A rain shadow is a dry region altitude of the mountain range. The temperature
on the leeward side of a mountain range where moist decreases with increasing altitude, which means that
air is blocked by the mountains, resulting in limited the air is colder and can hold less moisture. When the
precipitation. The windward side receives rain, while the moist air from the south-west monsoon or the western
leeward side remains dry. Region D is the eastern side disturbances encounters the Himalayas, it rises and
of the Sahyadri mountain range on the Deccan plateau cools, condensing into clouds and snow.
which is a rain shadow zone. It includes Northern C. Rainfall decreases from West Bengal to Punjab because
Karnataka, Solapur Plateau, Beed, Osmanabad, and of the distance from the sea. West Bengal is near the Bay
Vidarbha. of Bengal, which provides a source of moisture for the
south-west monsoon winds. Punjab is far from the sea,
86. Solution: (b)
which means that the monsoon winds lose their moisture
Exp) Option b is the correct answer. as they travel inland and become dry.
The correct match is: D. Sutlej-Ganga plain gets some rain in winter because
• Deccan Traps - Cretaceous Eocene of the western disturbances. These are low-pressure
• Western Ghats - Late Cenozoic systems that originate over the Mediterranean Sea and
move eastward along the Himalayas. They bring cold
• Aravalli - Pre-Cambrian and moist air, which causes rainfall and snowfall in the
• Narmada-Tapti alluvial deposits – Pleistocene north-western parts of India.

181 PYQ Workbook


GENERAL GEOGRAPHY

88. Solution: (a)


Exp) Option a is the correct answer.
The quote refers to Tharangambadi, a coastal town on the
Tamil Nadu Coast, which was once a Danish outpost. The
town features attractions such as the New Jerusalem Church,
a fort, quiet streets, and a deserted beachfront, making
it a charming destination with historical significance.
The New Jerusalem Church was built by Royal Danish
missionary Bartholomaus Ziegenbalg in a coastal town of
Tharangambadi, Tamil Nadu in 1718.

89. Solution: (d)


Exp) Option d is the correct answer.
Saddle Peak, the highest peak of the Andaman & Nicobar
Islands, is located on North Andaman Island. It stands at
an elevation of 732 meters and is situated close to the east
coast of the island. The peak is surrounded by the Saddle
Peak National Park and is positioned south of Diglipur and 92. Solution: (b)
southwest of Kalipur Beach. There’s a scenic viewpoint Exp) Option b is the correct answer.
called Mehendi Tikrey View Point to the east of the peak, The south-west monsoon winds originate from the Indian
and the Kalpong Dam lies to its west. Ocean and blow towards the Indian subcontinent during
90. Solution: (a) the summer season. They bring moisture-laden air that
causes heavy rainfall in most parts of India. The south-west
Exp) Option a is the correct answer.
monsoon is also known as the summer monsoon or the rainy
The Cauvery River thrice forks into two streams and
season in India.
reunites a few miles farther on, forming the islands of
Srirangappattanam, Sivasamudram, and Srirangam. Important Tips
Originating at Talakaveri in Karnataka’s Kodagu district, • Trade winds: These are permanent winds that blow
the Cauvery River meanders through diverse landscapes, from the subtropical high-pressure belts to the
creating natural features such as islands and waterfalls like equatorial low-pressure belts in both hemispheres.
Shivanasamudra, before finally reaching its destination in They are called northeast trade winds in the northern
the Bay of Bengal after covering a distance of 765 km. The hemisphere and southeast trade winds in the southern
river’s unique branching and rejoining patterns contribute hemisphere. They are important for sailing and
to the formation of these islands along its course. navigation.
91. Solution: (a) • Westerlies: These are permanent winds that blow from
the subtropical high-pressure belts to the subpolar low-
Exp) Option a is the correct answer.
pressure belts in both hemispheres. They are called
The correct sequence of the rivers in the north-south southwest westerlies in the northern hemisphere and
direction is: northwest westerlies in the southern hemisphere. They
• Kishan Ganga are responsible for bringing rainfall to the western
• Ganga coasts of continents in the mid-latitudes
• Wain Ganga 93. Solution: (d)
• Pen Ganga Exp) Option d is the correct answer.
The divisions along India’s coastal region indicate
underwater relief contours. The above figure shows a
bathymetric chart of the coasts of India. A bathymetric
chart is a specialized map that illustrates the submerged
terrain of oceans and seas. It focuses on depicting the
depth contours, underwater features, and physiographic
characteristics of the seafloor. These charts employ contour
lines and points to represent variations in depth or elevation.
Closed shapes with nested patterns can signify ocean trenches
or seamounts, depending on whether depth increases or
decreases toward the center, providing essential information
for marine navigation and scientific research.

PYQ Workbook 182


GENERAL GEOGRAPHY

94. Solution: (c) The oldest range in the Himalayas is the Great Himalayan
Exp) Option c is the correct answer. Range. Its formation resulted from the collision between
• Region 1 is Karakoram Range: The Karakoram Range the Indian and Eurasian tectonic plates, around 50 million
stretches across Pakistan, China, India, Afghanistan, and years ago. This collision led to the uplift of sedimentary
Tajikistan. It includes K2, the world’s second-highest rocks and the creation of parallel fold ranges. Various
peak. This 500 km long range is the most glaciated place geological mechanisms, such as subduction, extrusion, and
outside polar regions and is surrounded by various rivers thrusting, played roles in shaping the Himalayas, ultimately
and plateaus, holding geopolitical significance. resulting in the impressive topographic relief we see today.
• Region 2 is Ladakh Range: The Ladakh Range, a Important Tips
southern extension of the Karakoram Range, stretches
Chronology of Formation of Himalayan Ranges:
230 miles from the Indus and Shyok rivers’ confluence
in Baltistan to Ladakh’s Tibetan border. It forms the • The Great Himalayan Range is the oldest among
northeast bank of the Indus River and the west bank of the Himalayan ranges. It began forming around
the Shyok River, with an average height of around 6,000 50 million years ago when the Indian tectonic plate
meters and several significant mountain passes. collided with the Eurasian plate. This collision resulted
in the initial uplift of sediments and the formation of
• Region 3 is Zanskar Range: The Zaskar Range, in the
the Great Himalayan Range.
Himalayas of northern India and western Tibet, spans
about 400 miles from the Karcha River to the upper • Following the initial collision, the Lesser Himalayas
Karnali River. Kamet Peak (25,446 feet) is the highest began to form as a result of continued tectonic
point, and notable passes include Shipki, Lipu Lekh pressure and the folding of sediments. This process
(Lipulieke), and Mana. started shortly after the Great Himalayan Range began
• Region 4 is Pir Panjal Range: The Pir Panjal Range is to rise, roughly 20 to 50 million years ago.
a significant mountain chain in the Lower Himalayas of • The Dhauladhar Range is a subrange of the Lesser
the Western Himalayas, spanning between the Beas and Himalayas. It formed concurrently with the Lesser
Neelam/Kishanganga rivers in Himachal Pradesh and Himalayas, approximately 20 to 50 million years ago,
Jammu and Kashmir, India, with a portion extending as a result of the same tectonic forces and geological
into Pakistan. It separates the Kashmir Valley from the processes.
Jammu region’s hills. • The Shiwalik Range is the youngest of the Himalayan
ranges. It began forming relatively more recently,
around 2 to 20 million years ago, due to the ongoing
collision between the Indian and Eurasian plates. This
range primarily consists of unconsolidated sediments
and foothills.

97. Solution: (c)


Exp) Option c is the correct answer.
Assertion (A) is true: The monsoonal rainfall decreases as
one goes towards the west and northwest in the Ganga plain.
This is because the moisture-bearing monsoonal winds lose
their moisture as they travel inland from the Bay of Bengal
Note: Now the state of Jammu Kashmir is divided into UTs Reason (R) is false: The moisture-bearing monsoonal
of Jammu and Kashmir and Ladakh and one of the regions winds do not go higher as one moves up in the Ganga
marked in the question is part of UT of Ladakh. plain. In fact, the Ganga plain has a very low relief and does
95. Solution: (c) not offer any significant barrier to the monsoon winds. The
elevation of the plain drops only about 210 meters over a
Exp) Option c is the correct answer.
distance of 1,600 km from Delhi to the Bay of Bengal.
During ancient Indian historical geography, the term
‘Ratnakara’ denoted the “Indian Ocean.” It was referred 98. Solution: (b)
to as the “storehouse of great gems and jewels” in ancient Exp) Option b is the correct answer.
texts, such as the Śivapurāṇa, symbolizing its vastness and
An all-India water divide is a line on a map that separates
the treasures it held. This term highlights the importance of
the drainage basins of different rivers. The map shows
the Indian Ocean in historical Indian geography and trade,
as it played a significant role in the exchange of valuable the Western Ghats, which form the main water divide
commodities. in Peninsular India. The map also shows the Himalayan
ranges, which form the water divide between the Indus and
96. Solution: (c) the Ganga-Brahmaputra River systems. Therefore, the map
Exp) Option c is the correct answer. shows an all-India water divide.

183 PYQ Workbook


GENERAL GEOGRAPHY

Important Tips 101. Solution: (d)

• An isohyet is a line on a map that connects points with Exp) Option d is the correct answer
the same amount of rainfall in a given period or for a Option d is correct: Gully erosion is the most serious form
specific storm. It is also called an isohyetal line. of soil erosion. It occurs when large channels are formed in
the soil by water runoff. Gully erosion can be very difficult
• Isohyets are used to estimate the average rainfall
to control and can lead to the complete loss of soil. The
over an area and to show the spatial distribution of
Chambal Ravines are a series of deep gullies that are found
precipitation.
in the Chambal River basin in India. The ravines are caused
• Isohyets are based on interpolation between rain gauge by a combination of factors, including:
stations and may be affected by topography and other
• Heavy rainfall: The Chambal River basin receives an
factors.
average of 1,000 mm of rainfall per year.
99. Solution: (d) • Steep slopes: The ravines are found on steep slopes,
Exp) Option d is the correct answer which makes them more susceptible to erosion.
Option d is correct. Lateritic soil is a type of soil that is • Deforestation: The Chambal River basin has been
formed in tropical and subtropical regions with high rainfall deforested over the years, which has left the soil more
and temperatures. The high temperatures and rainfall cause exposed to erosion.
the rocks to weather and the minerals to leach out. This • Poor farming practices: Poor farming practices, such
leaves behind a soil that is rich in iron and aluminum oxides,
as plowing along the contours of the land, can also
but low in humus and other nutrients. The excess iron in
contribute to gully erosion.
laterite soil can make it infertile and difficult to cultivate.
Option a, b, and c are incorrect: The other types of erosion
Option a is incorrect Desert sand is a type of soil that is
mentioned in the options are not as serious as gully erosion.
found in deserts. It is made up of sand, silt, and clay. Desert
Splash erosion is the smallest and most initial form of soil
sand is low in organic matter and nutrients, but it is not
erosion. Sheet erosion is a more advanced form of soil
rendered infertile by the presence of excess iron.
erosion than splash erosion. Rill erosion is a more serious
Option b is incorrect Alluvial soil is a type of soil that is form of soil erosion than sheet erosion, but it is not as serious
formed from the deposition of river sediments. It is a fertile
as gully erosion.
soil that is rich in organic matter and nutrients. Alluvial soil
is not rendered infertile by the presence of excess iron.
Option c is incorrect Podzolic soil is a type of soil that is
found in cool, humid regions. It is low in organic matter and
nutrients, but it is not rendered infertile by the presence of
excess iron.

100. Solution: (d)


Exp) Option d is the correct answer
Option d is correct Pea is a leguminous crop. Leguminous
crops have the ability to fix atmospheric nitrogen in the soil
through a symbiotic relationship with bacteria. This means 102. Solution: (a)
that they can enrich the nitrogen content in the soil. Exp) Option a is the correct answer
Option a is incorrect. Potato is a non-leguminous crop. To prevent erosion, it is important to rotate crops, plant
Option b is incorrect. Sorghum is a non-leguminous crop. cover crops, and use conservation practices, such as contour
Option c is incorrect. Sunflower is a non-leguminous crop. farming and terracing.

Important Tips Statement 1 is correct:Sorghum is a tall, grassy plant that


is grown for its grain and forage. It is a deep-rooted crop,
Here are some other crops that enrich the nitrogen content which means that it can tap into deep layers of soil for water
in soil: and nutrients. However, sorghum is also a thirsty crop,
• Beans which means that it can deplete the soil of moisture. When
• Clover sorghum is planted continuously in the same field, the soil
• Lentils can become depleted of moisture and nutrients. This makes
the soil more susceptible to erosion, especially by wind.
• Lupins
Statement 2, 3, and 4 are incorrect: Potato, wheat, and
• Soybeans clover are not as susceptible to erosion as sorghum. Potato
These crops are all legumes and have the ability to fix is a shallow-rooted crop, which means that it does not tap
atmospheric nitrogen in the soil. This is a valuable process into deep layers of soil for water and nutrients. Wheat is a
as it helps to improve the fertility of the soil and can reduce moderate-rooted crop, clover is a shallow-rooted crop that
the need for fertilizer. also fixes nitrogen in the soil.

PYQ Workbook 184


GENERAL GEOGRAPHY

Important Tips 105. Solution: (d)

Other crops that are susceptible to erosion: Exp) Option d is the correct answer.

• Cotton: Cotton plants have relatively shallow root A rainy day as defined by the Indian Meteorological
systems, and their fields are often tilled between Department is a day when the rainfall at a point received
planting seasons, leaving the soil vulnerable to erosion. is 2.5 mm or more in 24 hours. IMD further defines that
rainfall for a station is called heavy if it is greater than 650
• Sugarcane: Sugarcane is grown in tropical regions and
mm and very heavy if it is greater than 1300 mm.
often on slopes. The steep terrain and shallow root
system can make it prone to soil erosion. 106. Solution: (b)
• Vegetable Crops: Some vegetable crops, like lettuce Exp) Option b is the correct answer
and carrots, have shallow roots and may be planted in
Soils of western Rajasthan typically have a high content of
rows with exposed soil in between.
Calcium. This is because the region has a semi-arid to arid
• Orchards and Vineyards: Young orchards and climate, which can lead to the accumulation of calcium in
vineyards may have limited ground cover and shallow the soil over time due to low leaching and high evaporation
roots, increasing the risk of erosion. rates.
103. Solution: (d)
107. Solution: (c)
Exp) Option d is the correct answer.
Exp) Option c is the correct answer
The above diagram is related to the climate of North-East
The pH value of a soil is a measure of its acidity or alkalinity.
region of India. The climate diagram given above shows the
A pH value of 7 is neutral, while a pH value below 7 is acidic
following characteristics:
and a pH value above 7 is alkaline.
• The humid temperature is high throughout the year,
A fertile soil, suitable for growing common crops is likely to
ranging from 25°C to 35°C. This indicates a tropical
have a pH value of 6 to 7. This is because most plants prefer
climate zone.
a slightly acidic or neutral soil.
• The relative humidity is also high throughout the year,
ranging from 70% to 90%. This indicates a humid climate Important Tips
condition. Here are some other things to keep in mind about soil pH:
• The rainfall is very high, especially from June to • Acidic soils have a pH value below 6.5.
September, when it exceeds 500 mm per month. This • Alkaline soils have a pH value above 7.5.
indicates a monsoon climate regime. • Neutral soils have a pH value of 7.
These characteristics match the climate of the north- • The pH of a soil can be affected by a number of factors,
east region of India, which includes the states of Assam, including the type of rock from which it was formed,
Arunachal Pradesh, Meghalaya, Manipur, Mizoram, the amount of organic matter in the soil, and the
Nagaland, and Tripura. This region has a tropical humid and amount of rainfall.
wet climate, influenced by the south-west monsoon winds
If your soil is too acidic or alkaline, you can adjust the pH
that bring heavy rains from the Bay of Bengal. The region
by adding lime or sulfur.
is also known as one of the wettest places on Earth, with
Cherrapunji and Mawsynram receiving the highest annual Here are some tips for improving the pH of your soil:
rainfall in the world. • Add lime to acidic soils.
• Add sulfur to alkaline soils.
104. Solution: (c)
• Add organic matter to the soil.
Exp) Option c is the correct answer.
• Mulch around your plants.
The pair of places that have the most marked differences
in total annual rainfall even though located approximately • Water your plants regularly.
along the same latitude is Ajmer and Shillong. The average By following these tips, you can improve the pH of your
annual rainfall of Ajmer is 53.8 cm, while that of Shillong is soil and create a more fertile environment for your plants
223.5 cm. This means that Shillong receives about four times to grow.
more rainfall than Ajmer. The reason for this huge difference
is the influence of the monsoon winds and the topography
of the regions. Ajmer is located in the arid western part of 5.2. Other Examination Previous Years’
India, where the monsoon winds are weak and dry. Shillong Questions
is located in the humid eastern part of India, where the
108. Solution: (c)
monsoon winds are strong and moist. Shillong also lies in
the Meghalaya plateau, which is one of the wettest places on Exp) Option c is the correct answer.
earth due to the orographic effect of the hills that intercept Statement 1 is incorrect: India is the seventh-largest country
the monsoon clouds. by area in the World. Russia, Canada, the United States,

185 PYQ Workbook


GENERAL GEOGRAPHY

China, Brazil, Australia are the top six largest countries • The Barkana Falls, formed by Seetha River, is a
respectively. It is also the most populous country with waterfall located near Agumbe in Shimoga district
over 1.406 billion people in 2023, and the most populous of state of Karnataka and the waterfalls is among the
democracy in the world ten highest waterfalls in India with a height of 250 m.
Statement 2 is correct: India occupies 3.28 million km2 • The Khandadhar Falls is located at Nandapani,
geographical area which forms 2.4% of the world’s land area. Bonaigarh in Sundargarh district in the Indian state
Statement 3 is correct: The Tropic of Cancer passes through of Odisha. It is the 9th highest waterfall in India with a
the middle of the country from the Rann of Kachchh in the height of 244 m and second highest waterfall in Odisha
west to Mizoram in the east. It divides the country into two after Barehipani Falls in Mayurbhanj.
halves. The states through which Tropic of Cancer passes • Palani Waterfall is located near Kullu in the district
are Rajasthan, Gujarat, Madhya Pradesh, Chhattisgarh, of Kullu in Himachal Pradesh. This waterfall is one
Jharkhand, West Bengal, Tripura, and Mizoram. of the highest surge waterfalls in India with a height
Statement 4 is incorrect: The part of India which is south of 150 m.
of the Tropic of Cancer is called the tropical area and the 110. Solution: (a)
remaining area to the north of the Tropic is called sub-
Exp) Option a is the correct answer.
tropics. Thus, only southern part of India lies in tropical
region. Assertion (A) is true: Agra and Darjeeling lie on nearly the
same latitude - close to 27-degree North. However, Darjeeling
109. Solution: (b) is at a higher altitude compared to Agra. Darjeeling is present
Exp) Option b is the correct answer. at an altitude of 2100m whereas Agra is present at 170m
The correct matching of the Waterfalls (List-I) with their
Locations (List-II) is as follows:
A. Dudhsagar - 4. Goa
B. Barkana - 1. Karnataka
C. Khandadhar - 2. Odisha
D. Palani - 3. Himachal Pradesh

Reason (R) is true and correctly explains Assertion (A): The


reason (R) provides a correct explanation for why Agra and
Darjeeling, despite being on the same latitude, have different
temperatures in January. Agra is located in the plains, while
Darjeeling is in a hilly/mountainous region. This difference
in elevation leads to variations in temperature. As we
ascend in altitude, the temperature tends to decrease. This
Important Tips
is why mountainous regions, like Darjeeling, which is in the
• Dudhsagar Falls is a four-tiered waterfall located Himalayan foothills, tend to be cooler compared to plains
on the Mandovi River in the Indian state of Goa. like Agra.
Dudhsagar Falls is amongst India’s tallest waterfalls
with a height of 310 m and an average width of 30 111. Solution: (d)
meters (100 feet). The falls are located in the Bhagwan Exp) Option d is the correct answer.
Mahaveer Sanctuary and Mollem National Park among The correct match for the highest peaks in different states
the Western Ghats. of India:

PYQ Workbook 186


GENERAL GEOGRAPHY

A. Tamil Nadu - 4. Doddabetta Peak 113. Solution: (a)


B. Rajasthan - 3. Gurushikhar Peak Exp) Option a is the correct answer.
C. Nagaland - 2. Saramati Peak Chilika Lake is the largest saline water lake in India. It
is also the largest brackish water lagoon in Asia and the
D. Madhya Pradesh - 1. Dhupgarh Peak
second-largest coastal lagoon globally. The lake covers an
Important Tips area of over 1,100 square kilometers and spans across the
Puri, Khordha, and Ganjam districts of Odisha, situated
• Doddabetta Peak (2,637 m) is the highest mountain
on the east coast of India at the mouth of the Daya River,
in the Nilgiri Mountains, located in the Nilgiris
flowing into the Bay of Bengal.
District of Tamil Nadu, India. Doddabetta is a popular
tourist destination, and it features an observatory at Important Tips
the summit equipped with telescopes for public use. Additional facts on Chilika Lake:
The area surrounding Doddabetta is rich in flora, with
• Chilika Lake is the largest lagoon in Asia and the
Sholas covering the slopes and a variety of sub-alpine
second largest in the world.
vegetation.
• In 1981, it gained recognition as India’s first wetland
• Saramati is a peak rising above the surrounding of international importance under the Ramsar
peaks at the mountainous border of the Indian state Convention.
of Nagaland and the Naga Self-Administered Zone
• The Irrawaddy dolphins, frequently seen near
of the Sagaing Region of Myanmar. It has a height of
Satapada Island, are a major attraction.
3,826 metres (12,552 ft).
• Nalabana Island, covering around 16 sq km within the
• Mount Dhupgarh, is the highest peak in the lagoon, was designated as a bird sanctuary in 1987.
Mahadeo Hills, which is a part of the Satpura Range
• Kalijai Temple is situated on an island in Chilika
in Madhya Pradesh, India. Situated near Pachmarhi
Lake.
in Hoshangabad district, this peak stands at an
elevation of 1,352 meters (4,429 feet) above sea level. It • The lake is a haven for migratory birds from various
is a popular location for watching sunsets, and the hill regions, including the Caspian Sea, Lake Baikal, and
station of Pachmarhi is located nearby. the Himalayas.

114. Solution: (a)


112. Solution: (d)
Exp) Option a is the correct answer.
Exp) Option d is the correct answer.
Assertion (A) is true: The Teesta River was earlier a
The Jonk River is not part of the Indian Ganga River basin. tributary of the Ganga River. However, in the 18th century,
It is a tributary of the Mahanadi River and flows through the river changed its course and now it is a tributary of
the states of Odisha and Chhattisgarh.The River starts from the Brahmaputra River. This was due to a process called
the Sunabeda plateau and enters Maraguda valley where it is river capturing, which is a common phenomenon in the
joined by a stream called Gaidhas-nala near Patora village. Himalayas.
The river forms Beniadhas fall (80 feet) and Kharaldhas Reason (R) is true and correct explanation of (A): River
Fall (150 feet) before entering the valley. It joins Mahanadi capturing is a process where a river captures the course
of another river, which is a common phenomenon in the
at Shivrinarayan. The other three rivers, Punpun, Ajoy,
Himalayas. This can happen due to a number of factors,
and Jalangi, are all tributaries of the Ganga River.
such as a change in the course of the river, a landslide, or
Important Tips an earthquake. In the case of the Teesta River, the river
changed its course due to a landslide. The landslide
• The Punpun River is a tributary of the Ganges. The
blocked the original course of the river and forced it to
Punpun originates on the Chota Nagpur Plateau, at an
change course and join the Brahmaputra River.
elevation of 300 m, the river mostly flows in a north-
east direction and joins the Ganges at Fatuha, 25 km Important Tips
downstream of Patna. Teesta River:
• The Ajay River is a tributary of the Ganges. The • Teesta River, a Brahmaputra tributary (known as
Ajay River originates near Munger in Bihar, flows Jamuna in Bangladesh), flows through India and
through Jharkhand, and enters West Bengal at Simjuri. Bangladesh.
It forms borders between Bardhaman and Jharkhand, • Originating in the Himalayas near Chunthang,
Bardhaman, and Birbhum, finally joining the Ganga Sikkim, it heads southward through West Bengal
River at Katwa Town. before entering Bangladesh.
• Jalangi River is a branch of the Ganges River in • The Teesta Barrage dam aids irrigation in the plains
Murshidabad and Nadia districts in the Indian state between the upper Padma and Jamuna rivers.
of West Bengal. It flows into the Ganga River and • Teesta river water conflict is one of the most contentious
strengthens its lower channel, the Hooghly. issues between India and Bangladesh.

187 PYQ Workbook


GENERAL GEOGRAPHY

115. Solution: (a) 117. Solution: (a)


Exp) Option a is the correct answer. Exp) Option a is the correct answer.
The correct matching of lakes with their locations is as Assertion (A) is true: The Himalayan region in India has the
follows: highest concentration of seismic zones due to the ongoing
• A. Sala Lake - 1. Arunachal Pradesh tectonic collision between the Indian and Eurasian plates.
• B. Badkhal Lake - 2. Haryana This collision results in intense geological activity, including
frequent earthquakes, making it a highly seismically active
• C. Loktak Lake - 3. Manipur
area in the country.
• D. Kaliveli Lake - 4. Tamil Nadu
Reason (R) is true and is the explanation of Assertion (A):
Important Tips The Himalayas feature numerous longitudinal thrust zones,
Sala Lake: where tectonic plates converge. These zones are characterized
• Sala Lake is a high-altitude lake located in the East by rocks being pushed over each other, resulting in intense
Kameng district of Arunachal Pradesh, India. geological pressure and causing earthquakes. The complex
• The lake is surrounded by snow-capped mountains and interaction of these thrusts contributes to the Himalayas’
is a popular destination for trekking and camping. seismic activity.
• The lake is fed by the Sala River, which originates 118. Solution: (a)
from the Sela Pass. Exp) Option a is the correct answer.
Badkhal Lake:
The Imphal Basin, surrounded by the Manipur hills in
• Badkhal Lake is a natural lake situated in Badkhal Northeast India, is a prime example of a lacustrine plain.
village near Faridabad, in the Indian state of This type of plain forms due to sediment deposition in
Haryana. an area where a lake once existed. Over time, as the lake
• It is a biodiversity area within the Northern Aravalli drains or evaporates, the sediments are left behind, creating
leopard wildlife corridor stretching from Sariska Tiger a level and often fertile plain. The Imphal Basin’s formation
Reserve to Delhi. is attributed to past tectonic activity and the presence of
Kaliveli Lake: rivers and streams that drained the area.
• Kaliveli Lake is a coastal lake and lagoon with
119. Solution: (b)
wetlands in the Viluppuram District of Tamil Nadu
state. Exp) Option b is the correct answer.
• Kaliveli Lake is a seasonal wetland, with a gradient Statement 1 is incorrect: The sedimentary rocks of the
from freshwater to brackish water, and is an important greater Himalayas are not fossil less. While these rocks
feeding and breeding ground on migratory bird flyway. have undergone significant metamorphism due to the
intense geological processes associated with mountain
116. Solution: (a)
building, they were originally formed in ancient marine
Exp) Option a is the correct answer. environments. As a result, they can contain fossils, although
The tributaries of River Yamuna from west to east are: fossils found in these rocks may be relatively rare and
• Chambal poorly preserved compared to those in less metamorphosed
• Sindh sedimentary rocks. On a few peaks, such as Mount Everest,
the crystalline rocks carried old fossil-bearing Tethys
• Betwa
sediments from the north piggyback to the summits.
• Ken
Statement 2 is correct: The sedimentary rocks of the lesser
Himalayas do indeed contain marine fossils. These rocks
were once part of ancient marine environments and
have preserved fossils of marine organisms. While fossils
may not be as abundant or diverse as those in the Tethyan
Himalayas, they are still found in sedimentary rocks of the
lesser Himalayas.
Statement 3 is correct: It is true that remains of ancient
human civilizations have been found in the outer or Shivalik
Himalayas. These areas have been important archaeological
sites, yielding tools, artifacts, and fossils that provide
insights into early human habitation in the region.

120. Solution: (b)


Exp) Option b is the correct answer.
The correct sequence of the hills from west to east is:

PYQ Workbook 188


GENERAL GEOGRAPHY

• Satpura Range: It is a mountain range that runs through • The peculiarity of this river is that it tends to increase
the states of Gujrat, Madhya Pradesh, Chhattisgarh and its width rather than deepening the bed because the
Maharashtra. The highest peak in the range is Dhupgarh, banks are of soil, which is easily erodible whereas
which is 1,350 meters (4,430 feet) high. beds are of sand. The floods develop and disappear so
• Mahadeo Range: It is a mountain range in the state of rapidly that they have no time to scour the bed.
Madhya Pradesh. The hills are situated in the northern
section of the Satpura Range. 122. Solution: (b)
• Maikal Range: It is a mountain range that runs through Exp) Option b is the correct answer.
the states of Chhattisgarh and Madhya Pradesh.
Western disturbances are extratropical storms that originate
Amarkantak is the highest peak of Maikal Hills.
in the Mediterranean region and bring sudden winter rain
• Chhotanagpur Plateau: It is a plateau that covers the
states of Jharkhand, Chhattisgarh, Odisha, and West to the northwestern parts of the Indian subcontinent. The
Bengal. The plateau is known for its mineral wealth, winter rains caused by western disturbances in North-
including coal, iron ore, and mica. Western Plain of India gradually decrease from West to
East as they gradually lose their effectiveness. Western
disturbances are vital for the growth of Rabi crops, such
as wheat, in India. They also contribute to soil erosion and
landslides in some regions.

123. Solution: (d)


Exp) Option d is the correct answer.
Gujarat, holds the distinction of having the longest
coastline among Indian states. This vast coastal stretch is
121. Solution: (b) home to numerous ports, including one major port and forty
Exp) Option b is the correct answer. intermediate or minor ports. Gujarat’s coastline features a
The Luni River in northwest India exhibits the characteristic range of attractions, including picturesque beaches like Diu,
of having freshwater in its upper course but becoming Dwarka, and Porbandar, making it a significant maritime
saline as it flows towards the lower part. The upper
state in India.
course of the Luni River has freshwater primarily because
it receives its water from relatively fresh sources such Length of the Coastline of India
as rainfall, seasonal streams, and rivers originating in
the Aravalli Range. During the monsoon season, these State/UT Length (in Km)
sources contribute freshwater to the river, giving it its
Gujarat 1214.7
initial freshwater character in the upper reaches. However,
as the river flows through the arid Thar Desert with high Andhra Pradesh 973.7
evaporation rates and encounters naturally occurring salts
in the soil and rocks, the water gradually becomes more Tamil Nadu 906.9
saline in its lower course. Maharashtra 652.6
Important Tips Kerala 569.7
Additional facts on Luni River
Odisha 476.4
• The Luni or the Salt River (Lonari or Lavanavari in
Sanskrit) is named so because its water is brackish Karnataka 280
below Balotra.
West Bengal 157.5
• Luni is the only river basin of any significance in
Western Rajasthan, which forms the bulk of the arid Goa 101
zone.
Andaman & Nicobar 1962
• Luni originates from the western slopes of the
Islands
Aravalli ranges at an elevation of 772 m near Ajmer
flowing in the southwest direction and traversing a Lakshadweep 132
course of 511 km in Rajasthan, it finally flows into the
Rann of Kachchh (it gets lost in the marsh). Puducherry 47.6
• Most of its tributaries drain the steep northwest Daman & Diu 42.5
of Aravalli hills and join it on the left side. Its total
catchment area falls in Rajasthan. Total 7,516.6

189 PYQ Workbook


GENERAL GEOGRAPHY

124. Solution: (c) Important Tips


Exp) Option c is the correct answer. Map of Tropic of Cancer:
The correct matching of the states with their highest peaks
is as follows:
• Anamudi: Anamudi is a mountain located in Ernakulam
district and Idukki district of the Indian state of
Kerala. It is the highest peak in the Western Ghats and
in South India, at an elevation of 2,695 metres (8,842 ft).
• Saramati: Saramati is a peak rising above the surrounding
peaks at the mountainous border of the Indian state of
Nagaland and the Naga Self-Administered Zone of the
Sagaing Region of Myanmar. It has a height of 3,826
metres (12,552 ft).
• Nanda Devi: Nanda Devi is the second-highest mountain
in India, after Kangchenjunga, and the highest located
entirely within the country. It is located in Chamoli
district of Uttarakhand, between the Rishiganga valley
on the west and the Goriganga valley on the east.
• Doddabetta: Doddabetta is the highest mountain in the
Nilgiri Mountains at 2,637 metres (8,652 feet). There is
a reserved forest area around the peak. It is 9 km from
Ooty, on the Ooty-Kotagiri Road in the Nilgiris District
of Tamil Nadu, India. 127. Solution: (a)
Exp) Option a is the correct answer.
125. Solution: (d)
The correct matching of the Waterfalls (List-I) with the
Exp) Option d is the correct answer Rivers (List-II) is as follows:
Black soil is not found in the Himalayan region. It is found in A. Dudhsagar - 4. Mandavi
the Deccan plateau of India, the central part of the country. B. Duduma - 2. Machhkund
The other statements are all correct. C. Gokak - 1. Ghatprabha
• Black soil is locally called ‘regur’. D. Jog - 3. Sharavati
• The regur soil is essentially mature soil. Important Tips
• Black soils are highly retentive to moisture. • Duduma Waterfall is situated in the border of
The Himalayan region is covered with mountain soil and Koraput (Odisha) districts of India. This horsetail
alluvial soil. It is a loose and sandy soil that has a low water- type waterfall is 175 metres (574 ft) in height and is
retention capacity. Therefore, black soil is not found in the formed by the Machkund River.
Himalayan region. • The Gokak Falls is a waterfall with a height of 52m,
located on the Ghataprabha River in Belagavi district
126. Solution: (a) of Karnataka, India.
Exp) Option a is the correct answer. • Jog Falls is a waterfall on the Sharavati River located
The Tropic of Cancer divides the country into almost two in Siddapura taluk, Uttara Kannada District and its
equal parts. It lies across the latitude approximately 23°27′ viewpoint located in Shimoga district of Karnataka,
N. It is the only latitude that passes through India. It passes India. It the third-highest waterfall in India with a
through 8 Indian states, and they are as follows: height of 253 m.
• Gujarat. 128. Solution: (d)
• Rajasthan. Exp) Option d is the correct answer.
• Madhya Pradesh. The Nelang Valley is located in the state of Uttarakhand,
India. It is situated in the Uttarkashi District and is known
• Chhattisgarh.
for its picturesque landscapes, steep gorges, and the Jadh
• Jharkhand. River, an important tributary of the Bhagirathi River,
• West Bengal. flowing through it.

• Tripura. 129. Solution: (b)


• Mizoram Exp) Option b is the correct answer.

PYQ Workbook 190


GENERAL GEOGRAPHY

After entering Bangladesh, the River Ganga is known as the • The Bom di La Pass is a border pass between China’s
Padma. The Padma is a significant river in Bangladesh and Tsona County in Tibet and India’s Tawang district
serves as the primary distributary of the Ganges. It flows in Arunachal Pradesh. The pass currently serves as a
southeast for approximately 356 kilometers before merging trading point between Arunachal Pradesh and Tibet. It
with the Meghna River near the Bay of Bengal. Rajshahi is is also an agreed Border Personnel Meeting point for
a major city situated along the banks of the Padma River. the security forces of China and India.
• Chang La is a high mountain pass in Ladakh, situated
at an elevation of 5,391 meters within the Ladakh
Range. It connects Leh to Pangong Lake via a motorable
road, passing through Karu and other towns, linking
Leh to the rest of India.

131. Solution: (b)


Exp) Option b is the correct answer.
Kuttanad, also known as Kuttanadu, in Kerala is famous
for being the region with the lowest altitude in India,
situated at 2.2 meters below sea level. It is renowned as
the “Rice Bowl of Kerala” due to extensive rice farming
practices in the area. Kuttanad is unique as it is one of the few
places globally where farming is conducted below sea level.
Kuttanad farmers are noted for Biosaline Farming (growth
of plants in saline groundwater or soil). In 2013, the Food
and Agriculture Organization (FAO) declared this farming
style as a Globally Important Agricultural Heritage System
(GIAHS). It is also famous for Vallamkali boat race in the
Punnamada Backwaters.
Important Tips Important Tips
• The Padma enters Bangladesh from India near Globally Important Agricultural Heritage Systems
Nawabganj and meets the Jamuna near Aricha and (GIAHS):
retains its name, but finally meets with the Meghna GIAHS are distinctive land use systems worldwide,
near Chandpur and adopts the name “Meghna” before characterized by rich biodiversity, resilient ecosystems,
flowing into the Bay of Bengal. and cultural heritage. These systems are managed by
• The Jamuna River is the lower stream of the local communities, including farmers, herders, fisherfolk,
Brahmaputra River. and tribals. The United Nations Food and Agriculture
Organization (UN FAO) leads the GIAHS program.
• The Meghna is formed due to the joining of the Surma
and Kushiyara rivers (together known as Barak River Designated Sites in India (as of September 2023):
in India) originating from the hilly regions of eastern • Pampore Saffron Heritage, Jammu & Kashmir
India. (2011):
• Known for saffron cultivation, which is integral
130. Solution: (d) to Kashmiri cuisine, medicinal traditions, and
Exp) Option d is the correct answer. cultural heritage.
The Sela Pass is a high-altitude mountain pass located on • Rice, maize, millets, pulses, almonds, apples,
the border between the Tawang and West Kameng districts cherries, pears, peaches, and walnuts.
in the Indian state of Arunachal Pradesh. Itconnects • Koraput Traditional Agriculture, Odisha (2012):
the town of Tawang to Dirang and Guwahati, carrying • Located in the Eastern Ghats, this region practices
National Highway 13. The pass, at 4170 m (13,700 ft), is subsistence paddy cultivation on highland slopes.
known for its challenging terrain and is open year-round • Strong ties with tribal communities, sacred groves
except for temporary closures due to landslides or snowfall. for plant genetic resource preservation.
Important Tips • Kuttanad Below Sea Level Farming System, Kerala
• Rohtang Pass is a high mountain pass (elevation 3,980 (2013):
m (13,058 ft)) on the eastern end of the Pir Panjal • It comprises wetlands for paddy cultivation and
Range of the Himalayas around 51 km from Manali fishing, garden lands for coconut and food crops,
in the Indian state of Himachal Pradesh. It connects and water areas for inland fishing.
the Kullu Valley with the Lahaul and Spiti Valleys of • Rice cultivation below sea level in land created by
Himachal Pradesh, India. draining delta swamps in brackish waters.

191 PYQ Workbook


GENERAL GEOGRAPHY

132. Solution: (a) Important Tips


Exp) Option a is the correct answer. • The black soil is generally clayey.
The river called the “Ganga of the South” is the Kaveri • They swell greatly and become sticky when wet in
River, also spelled Cauvery. It originates on Brahmagiri Hill the rainy season. In the dry season, the moisture
in Karnataka, flows for 475 miles through Karnataka and evaporates, the soil shrinks and develops wide cracks.
Tamil Nadu, and eventually empties into the Bay of Bengal.
Known as Daksina Ganga or the Ganges of the South • Black soil is rich in iron, lime, aluminium, magnesium
among devout Hindus, the Kaveri River holds significance and also contains potassium. However, these soils are
for its sacredness, and its course is celebrated in Tamil deficient in nitrogen, phosphorus and organic matter.
literature. • Cotton, pulses, millets, castor, tobacco, sugarcane,
citrus fruits, linseed, etc. are mainly cultivated in
133. Solution: (b) black soil.
Exp) Option b is the correct answer.
135. Solution: (a)
The correct matching of lakes with their locations is as
follows: Exp) Option a is the correct answer.
A. Ashtamudi - 2. Kerala Assertion (A): It is true that the rivers of the Western Ghats
do not form deltas. Deltas are typically formed along the
B. Pulicat - 3. Tamil Nadu
eastern coast of India.
C. Roopkund - 4. Uttarakhand
Reason (R) is true and (R) is the correct explanation of
D. Surajkund - 1. Haryana
(A): Rivers in the Western Ghats region typically do not form
Important Tips deltas. This is because they have shorter courses and flow
Ashtamudi Lake: over hard rocks, which limits the deposition of sediments
necessary for delta formation. Deltas are more commonly
• Ashtamudi Lake (Ashtamudi Kayal), in the Kollam
associated with rivers that have longer courses and flow
District of the Indian state of Kerala.
over softer sedimentary terrain.
• It possesses a unique wetland ecosystem and a large
palm-shaped (also described as octopus-shaped) Important Tips
water body, second only in size to the Vembanad West flowing rivers of peninsular India don’t form deltas
estuary ecosystem of the state. because of following reasons:
• The lake is also called the gateway to the backwaters • Short Course: Many of the rivers in the Western Ghats
of Kerala and is well known for its houseboat and region have relatively short courses before they reach
backwater resorts. the Arabian Sea. These shorter courses do not allow
• Ashtamudi Wetland was included in the list of for the extensive deposition of sediments that are
wetlands of international importance, as defined necessary for delta formation.
by the Ramsar Convention for the conservation and • Hard Rocks: The Western Ghats are characterized by
sustainable utilization of wetlands. hard rocks, including basaltic formations. These rocks
Surajkund: do not erode easily and do not contribute significant
amounts of sediment to the rivers.
• Surajkund is an ancient reservoir of the 10th century
located on Southern Delhi Ridge of Aravalli range in • High Gradient: The Western Ghats have a steep
Faridabad city of Haryana state. gradient, which means that the rivers flow rapidly.
Fast-flowing rivers have less time to deposit sediment,
• It is said to have been built by the king Surajpal of
which is essential for delta formation.
the Tomara Rajputs in the 10th century.
• Lack of Alluvial Plains: Unlike the rivers originating
• Surajkund is known for its annual fair “Surajkund
in the Himalayas, which have vast alluvial plains, the
International Craft Mela”.
Western Ghats rivers do not have extensive low-lying
134. Solution: (d) areas where sediment can accumulate to form deltas.
Exp) Option d is the correct answer • Direct Entry into the Sea: Instead of forming deltas,
Black soil or Regur soil covers most of the Deccan Plateau many Western Ghats rivers enter the Arabian Sea
– parts of Maharashtra, Madhya Pradesh, Gujarat, Andhra directly. They do not meander and deposit sediments
Pradesh and some parts of Tamil Nadu. It is formed due to in the way that rivers with longer courses and lower
weathering of basaltic lava. Most of the black soils in India gradients do.
are derived from two types of rocks, the Deccan and the
136. Solution: (c)
Rajmahal trap. It is a rich and fertile soil that is well-suited
for agriculture. Black soil is typically dark in color and has Exp) Option c is the correct answer.
a high clay content. It is also high in organic matter and The rivers of South India mainly have a dendritic drainage
nutrients. pattern. This pattern resembles the irregular branches

PYQ Workbook 192


GENERAL GEOGRAPHY

of a tree and forms in regions with uniform lithology is a part of the Indus River Delta, which lies across Gujarat
and minimal faulting or jointing. Dendritic patterns are state in India and Sindh in Pakistan.
prevalent when the underlying rock or material is consistent
and erodes uniformly in all directions. Examples of rivers
with dendritic patterns in South India include the Godavari,
Mahanadi, Cauvery, and Krishna.

137. Solution: (c)


Exp) Option c is the correct answer.
Kolleru Lake is one of India’s largest freshwater lakes,
located between the deltas of the Krishna and Godavari
rivers in Andhra Pradesh. It covers a significant area and
serves as a vital habitat for resident and migratory birds. 139. Solution: (b)
It has been declared a wildlife sanctuary and designated as Exp) Option b is the correct answer.
a wetland of international importance under the Ramsar Statement (A) is true: Many rivers originating from the
Convention.The other options are not correctly matched as Himalayas are perennial, meaning they flow throughout
Lonar Lake is located in the state of Maharashtra, Nakki Lake the year. This is because they receive water not only from
is located in the state of Rajasthan, Pulicat Lake is located in rainfall but also from the melting snow and glaciers in the
the states of Andhra Pradesh and Tamil Nadu. Himalayan region, ensuring a consistent flow of water.
Important Tips Statement (R) is true: The Himalayas receive a significant
Lonar Lake: portion of their precipitation from the South-Western
monsoon, which brings heavy rainfall to the region during
• Lonar Lake, also called Lonar crater, is a saline and
the monsoon season. This monsoonal rainfall contributes to
alkaline lake in Maharashtra’s Buldhana district.
the perennial nature of the rivers.
• Located within the Deccan Plateau, which is a vast area
(R) is not the correct explanation of (A): The perennial
of volcanic basalt rock formed by volcanic eruptions.
nature of the rivers in the Himalayas is primarily due to the
• The lake is oval-shaped and has a mean diameter of continuous melting of snow and glaciers. This meltwater
1.2 km. provides a constant source of water throughout the year,
• Designated as a National Geo-heritage Monument. ensuring that these rivers flow consistently, even during
• Believed to have formed over 52,000 years ago due to the dry seasons. Additionally, other factors like rainfall and
a meteorite impact on Earth. groundwater contribute to the perennial flow, but the snow
Nakki Lake: and glacier melt are particularly significant in maintaining
the rivers’ continuous flow.
• Nakki Lake is a lake situated in the Indian hill station
of Mount Abu in Aravalli range in state of Rajasthan. 140. Solution: (c)
• The lake stretches for approximately 800 meters in Exp) Option c is the correct answer.
length and spans about 400 meters in width, with Due to Earth’s rotation and revolution around the Sun,
depths ranging from 20 to 30 feet.
various nations worldwide observe various time zones.
• Mahatma Gandhi’s ashes were immersed in this The result is that the world is split into 24 time zones, each
holy lake on 12 February 1948 and Gandhi Ghat was offset by one hour. India’s standard meridian is 82°30’E.
constructed. The standard meridian of India passes through five states
in India which are Orissa, Chhattisgarh, Uttar Pradesh,
138. Solution: (d)
Andhra Pradesh, and Madhya Pradesh. Thus, it does not pass
Exp) Option d is the correct answer. through Maharashtra.
The Kori Creek is a tidal creek in the Kutch region of the
Indian state of Gujarat. It lies just to the west of the Great 141. Solution: (b)
Rann of Kutch area of India. This region belonging to India Exp) Option b is the correct answer

193 PYQ Workbook


GENERAL GEOGRAPHY

Option b is correct. Black soil is a clayey soil that is rich in the Western Ghats and flows through Karnataka and Andhra
organic matter and minerals. It has a high water-retention Pradesh. It eventually joins the Tungabhadra River, which
capacity and is therefore less prone to erosion. It is also a is a significant river in southern India. On the banks of
fertile soil and is suitable for growing a variety of crops, the Vedavathi, there is a famous temple devoted to Shri
including cotton, wheat, and rice. Anjaneya at Kellodu, Hosadurga Taluk, Karnataka.
Option a is incorrect. Red soil is a sandy loam soil that is low Important Tips
in organic matter and minerals. It has a low water-retention Tungabhadra River:
capacity and is therefore more prone to erosion.
• The Tungabhadra River was known as Pampa in
Option c is incorrect. Laterite soil is an acidic soil that ancient times.
is formed from the weathering of rocks in hot and humid
• It is formed by the confluence of two rivers, the Tunga
climates. It has a low water-retention capacity and is therefore River and the Bhadra River, which originate from the
not suitable for growing a variety of crops. eastern slopes of the Western Ghats in Karnataka.
Option d is incorrect Alluvial soil is a soil that is formed • The Tungabhadra River meanders through the plains
from the deposition of river sediments. It is a loose and for a distance of 531 km before joining the Krishna
sandy soil that has a low water-retention capacity. River at Gondimalla, near the famous Alampur
Junction in Mahaboobnagar District of Andhra
142. Solution: (c)
Pradesh.
Exp) Option c is the correct answer. • The primary tributaries of the Tungabhadra include
K2, at 8,611 metres (28,251 ft) above sea level, is the Varada, Hagari, and Handri.
second-highest mountain on Earth, after Mount Everest at • The wedge of land located north of the Tungabhadra
8,849 metres (29,032 ft) and the highest mountain peak of River, between the Tungabhadra and the Krishna, is
India. It lies in the Karakoram range, partially in the Gilgit- referred to as the Raichur Doab.
Baltistan region of Kashmir and partially in the China- • Important urban centers situated along the
administered Trans-Karakoram Tract in the Taxkorgan Tajik Tungabhadra River include Harihar, Hospet, Hampi,
Autonomous County of Xinjiang. Mantralayam, and Kurnool.

144. Solution: (c)


Exp) Option c is the correct answer.
Tripura has an international boundary on its three side with
Bangladesh, the fourth side is bordered by Indian state of
Assam and Mizoram.
Nagaland has an international boundary on its eastern side
with Myanmar.
Assam has an international boundary with Bhutan on its
Important Tips northeastern side and with Bangladesh on southwestern
K2 Mountain: side.
• Although the summit of Everest is at a higher altitude, West Bengal has an international boundary with Nepal on
K2 is a more difficult and dangerous climb, due in its northeastern side and with Bhutan on its northern side
part to its more northern location, where inclement and with Bangladesh on Eastern side.
weather is more common.
145. Solution: (d)
• Of the five highest mountains in the world, K2 is the
Exp) Option d is the correct answer.
deadliest; approximately one person dies on the
mountain for every four who reach the summit. Also The correct sequence of Himalayan peaks in the direction
occasionally known as Mount Godwin-Austen, other east to west is:
nicknames for K2 are The King of Mountains and The • Namcha Barwa (7,782 m): Namcha Barwa is considered
Mountaineers’ Mountain. as the Eastern point of the Himalayas. It is located on the
• The summit was reached for the first time by the Italian extreme east of the Indian state of Arunachal Pradesh.
climbers Lino Lacedelli and Achille Compagnoni, on • Kanchenjunga (8,586 m): Kangchenjunga is the third-
the 1954 Italian expedition led by Ardito Desio. highest mountain in the world. It lies in the border
region between Nepal and Mangan district, Sikkim state
143. Solution: (d) of India.
Exp) Option d is the correct answer. • Mount Everest (8,848 m): Mount Everest is Earth’s
The Hagari River is a tributary of the Tungabhadra River. highest mountain above sea level, located in the
The Hagari River is also known as the Vedavathi in some Mahalangur Himal sub-range of the Himalayas. The
regions. Itoriginates from the Bababudanagiri Mountains in China–Nepal border runs across its summit point.

PYQ Workbook 194


GENERAL GEOGRAPHY

• Nanda Devi (7,816 m): Nanda Devi is the third-highest


mountain in India. It is located in Chamoli district of
Uttarakhand, between the Rishiganga valley on the west
and the Goriganga valley on the east.

146. Solution: (b)


Exp) Option b is the correct answer.
The correct match of sea beaches with their respective states
is as follows:
A. Digha - 2. West Bengal
Important Tips
B. Gopalpur - 3. Orissa Additional facts on Pulicat Lake:
C. Calangute - 4. Goa • Second largest brackish water lagoon in India, after
D. Marina - 1. Tamil Nadu Chilika Lake.
• Located in the Coromandal Region, bordering
Andhra Pradesh and Tamil Nadu. Predominantly
(96%) in Andhra Pradesh, with a small portion (3%)
in Tamil Nadu.
• Separated from the Bay of Bengal by the barrier
island of Sriharikota, housing the Satish Dhawan
Space Centre.
• Home to the Pulicat Lake Bird Sanctuary and hosts
the annual Flamingo Festival.
• Main inflows: Arani, Kalangi, and Swarnamukhi
rivers.
• Features the Buckingham Canal, a navigation channel,
on its western side.

148. Solution: (c)


Exp) Option c is the correct answer.
Chambal River is an example of the superimposed river.
147. Solution: (d) Superimposed drainage, also known as epigenetic or
superinduced drainage, occurs when a river maintains
Exp) Option d is the correct answer.
its course and pattern on a landscape that has undergone
Pulicat Lake is shared by two Indian states, Andhra significant changes due to geological processes. In this
Pradesh and Tamil Nadu. It is the second-largest brackish type of drainage, the river’s current path doesn’t align
water lagoon in India, covering 759 square kilometers, with the underlying rock structure or geological features
with a major portion falling within the Tirupati district because it was established on a previous layer of rocks
that have since eroded away. The river retains its original
of Andhra Pradesh. The lagoon comprises the following
course unaffected by the newly exposed geological features.
regions: Pulicat Lake (Andhra Pradesh and Tamil Nadu), Examples of superimposed rivers include the Damodar,
Marshy/Wetland Land Region (AP), Venadu Reserve Forest Subarnarekha, Chambal, Banas, and several rivers in the
(AP), and Pernadu Reserve Forest (AP). Rewa Plateau. These rivers flow independently of the

195 PYQ Workbook


GENERAL GEOGRAPHY

current topography, following their initial paths set before of 2,695 meters (8,842 feet) above sea level, making it the
significant geological changes occurred. The Alaknanda tallest peak in the region. It is the highest point in India
River, Kosi River, and Godavari River all flow through south of Himalayas. Thus, it is known as “Everest of South
regions with a single geological history. Their drainage India”. The name “Anamudi” translates to “elephant’s head”
patterns are consistent with the underlying rock structure. due to its resemblance to the shape of an elephant’s head.
This peak is a prominent natural landmark and is located
Important Tips
within the Eravikulam National Park, known for its diverse
Discordant drainage patterns: wildlife and unique flora.
Discordant drainage patterns are characterized by rivers
Important Tips
that do not align with the surrounding topography
and geology. They are classified into two main types: • Mahendragiri:
antecedent and superimposed. • Peak of the Eastern Ghats located near Behrampur
• Antecedent Drainage (Inconsequent Drainage): in the district of Gajapati, Orissa (Odisha).
• It is the highest peak of Orissa and is situated near
• Occurs when a part of the river slope and the
the border with Andhra Pradesh.
surrounding area undergo uplift.
• Doddabetta:
• The river maintains its original course, cutting
through the uplifted portion. • Highest peak of the Nilgiri Hills in the Nilgiri
district of Tamil Nadu.
• This cutting action results in the formation of deep
gorges. • Located 35 kilometers north of Ooty.
• Covered with Shola forests on its slopes,
• It is characterized by vertical erosion or vertical
contributing to its natural beauty and ecological
downcutting.
significance.
• Antecedent rivers are typically older than the
• Amarkantak:
geological features they traverse.
• The Amarkantak region is a unique natural heritage
• Examples include the Indus, Sutlej, and
area and is the meeting point of the Vindhya and
Brahmaputra rivers, which are older than the the Satpura Ranges, with the Maikal Hills being
Himalayan Mountain range. the fulcrum.
149. Solution: (a) • This is where the Narmada River, the Son River
Exp) Option a is the correct answer. and Johilla River (tributary of Son) originate.

150. Solution: (c)


Exp) Option c is the correct answer.
Chorabari Glacier, also known as Chorabari Bamak
Glacier, is located to the north of Kedarnath temple in
Uttarakhand, India. Spanning over 15 square kilometers
and approximately 7 kilometers in length, this glacier is
situated in the Garhwal Himalayan region.It serves as the
source of the Mandakini River and Chorabari Tal. The trek
to Chorabari Glacier, starting from Sonprayag, is popular
among trekkers and offers stunning views of the Himalayan
landscape. This glacier is best observed while en route to the
Kedarnath shrine, which is in close proximity.

The highest peak in South India is Anamudi. Anamudi


is situated in the Indian state of Kerala and is part of the
Western Ghats Mountain range. It stands at an elevation

PYQ Workbook 196


GENERAL GEOGRAPHY

151. Solution: (b) retain moisture well. Regur soils are also not suitable for tea
Exp) Option b is the correct answer. plantation because they are too clayey and do not allow the
roots of the plants to breathe.
When it is Midnight i.e., 12.00 AM at Indian Standard Time
(IST), the local time at another place is 6 AM. Thus, the Important Tips
local place is ahead by 6 hours than the IST. A difference of Here are some other characteristics of soil suitable for tea
1 degree longitude makes a difference of 4 minutes in local plantation:
time. Therefore, a difference of 6 hours makes a difference of • The soil should be well-drained.
90-degree longitude. IST meridian is 82-degree 30-minute E
• The soil should be rich in organic matter.
longitude so the place will be 90-degree E of IST. Therefore,
the place is located at 172-degree 30-minute E. • The soil should be free from pests and diseases.
• The soil should be at an altitude of 600 to 2000 meters
152. Solution: (a) above sea level.
Exp) Option a is the correct answer.
154. Solution: (a)
Assertion (A) is true: Major rivers in peninsular India,
like the Godavari, Krishna, and the rivers of the Ganges- Exp) Option a is the correct answer.
Brahmaputra system, flow into the Bay of Bengal. However, • Tropical moist deciduous: They are found in the eastern
the Narmada and the Tapi rivers are the two major rivers that part of the country such as northeastern states along the
flow into the Arabian Sea. foothills of the Himalayas, especially in Tarai region.
Along with that they are found in some part of Jharkhand,
Reason (R) is true: The Narmada and the Tapi rivers flow in
West Orissa and Chhattisgarh and on the eastern slopes
rift valleys. A rift valley is a lowland region that forms where
of the Western Ghats.
Earth’s tectonic plates move apart, or rift.
• Tropical dry deciduous: They are found in Middle
(R) is the correct explanation of (A): The fact that the Ganga Plain along with Madhya Pradesh, Chhattisgarh,
Narmada and the Tapi rivers flow through rift valleys is the Maharashtra, Telangana and Andhra Pradesh.
reason why they flow into the Arabian Sea instead of the Bay
• Alpine: Alpine forests are mostly found in mountainous
of Bengal. The rift valleys provide a natural path for the
areas at elevations ranging from 2,900 to to 3800 m
rivers to flow to the Arabian sea.
above sea level, based on location and plant variety. They
are found in Arunachal Pradesh, Himachal Pradesh,
Uttrakhand, Jammu and Kashimir, etc. Alpine forests are
characterised by silver fir, junipers, pines and birches
trees.
• Tropical evergreen: They are found in the Andaman and
Nicobar Islands , the Western Ghats (Sahyadris), the
coastline of peninsular India, and the Assam region in
the north-east.
Important Tips
The Tarai region is a low-lying strip of land that lies in
the southern part of Nepal. It is a belt of flat land that
stretches from the foothills of the Himalayas in the north
to the Indo-Gangetic plain in the south.
Ganga Plains can be subdivided into the following sub-
regions:
The Upper Ganga Plain:
• It covers portions of the Agra Division, the
Rohilkhand Division, and the Ganga-Yamuna
153. Solution: (a) Doab.
Exp) Option a is the correct answer • It is one of India’s most productive and fertile
The soil that is most suited for tea plantation is acidic. plains, and the Green Revolution has had great
Tea plants prefer a soil with a pH value of 4.5 to 5.5. This success here.
is because the roots of tea plants are sensitive to alkaline The Middle Ganga Plain:
conditions. Acidic soils are also good at retaining moisture, • It covers portions of the central and eastern Uttar
which is essential for tea plants. Pradesh and the Bihar Plains up to Muzaffarpur
Option b, c and d are incorrect. Alkaline soils are not and Patna.
suitable for tea plantation because they can cause the roots • The Kosi is one of the dominant rivers in this plain
of the plants to burn. Alluvial soils are also not suitable which frequently shift their courses in this area due
for tea plantation because they are too sandy and do not to the low gradient of the plain.

197 PYQ Workbook


GENERAL GEOGRAPHY

The Lower Ganga Plain: • Paleomagnetism helps detect the polarity of the
• It covers portions from Patna in the west, the foot of Earth’s magnetic field and identifies magnetic field
Darjeeling Himalaya north, and the Bay of Bengal reversals over time. This is possible by studying rocks
south. of different ages.
• Basaltic rocks, which make up a significant portion
155. Solution: (c)
of the ocean floor, contain magnetic minerals. As
Exp) Option c is the correct answer these rocks solidify, these minerals align with the
Assertion A is correct. Regur or black cotton soil is found prevailing magnetic field, preserving its direction.
mostly in deccan trap region. The colour of these soils varies • Paleomagnetic studies have shown that the orientation
from deep black to grey. of the Earth’s magnetic field has alternated over
Reason R is incorrect. Black soil is rich in iron, lime, geological time, a phenomenon known as geomagnetic
aluminium, magnesium and also contains potassium. reversal.
However, these soils are deficient in nitrogen, phosphorus • Paleomagnetism played a crucial role in supporting the
and organic matter or humus. continental drift hypothesis and the development of
theories like Sea Floor Spreading and Plate Tectonics.
156. Solution: (b)
Exp) Option b is the correct answer. 158. Solution: (b)
Option a is incorrect: The distance between Agartala and Exp) Option b is the correct answer.
the Tropic of Cancer is 44 km. Vembanad Lake, also known by various names including
Option b is correct: The distance between Gandhinagar and Vembanad Kayal, Vembanad Kol, Punnamada Lake (in
the Tropic of Cancer is 25 Km. Therefore, from the options Kuttanad), and Kochi Lake (in Kochi), is located in
given, the town of Gandhinagar is nearest to the Tropic of Kerala, spanning multiple districts and covering a vast
Cancer. area of over 2033.02 square kilometers. It is fed by four
Option c is incorrect: The distance between Jabalpur and rivers, Meenachil, Achankovil, Pampa and Manimala and is
the Tropic of Cancer is 30 km. separated from the Arabian Sea by a narrow barrier island,
Option d is incorrect: The distance between Ujjain and the making it a renowned backwater destination in Kerala.
Tropic of Cancer is 28 km. Important Tips
157. Solution: (a) Additional Facts on Vembanad Lake:
Exp) Option a is the correct answer. • Vallam Kali, also known as the Nehru Trophy Boat
Race, is an annual Snake Boat Raceheld in August on
The paleomagnetic results obtained from India indicate
Vembanad Lake.
that in the past, the Indian landmass has moved northward.
This is because the magnetic field of the Earth has reversed • In 2002, it was designated a wetland of international
its polarity several times in the past, and the rocks in India importance under the Ramsar Convention.
record these reversals. By studying the orientation of the • It is the second largest Ramsar site in India after the
magnetic minerals in these rocks, scientists can determine Sundarbans in West Bengal.
the direction in which the Indian landmass was moving at • Vembanad wetland is identified under the National
the time the rocks were formed. Wetlands Conservation Programme by the Government
The Indian landmass was once part of the Gondwana of India.
supercontinent, which also included Africa, South America, • The Kumarakom Bird Sanctuary is situated on the
Australia, and Antarctica. About 180 million years ago, lake’s eastern coast.
Gondwana began to break apart, and the Indian landmass • In 2019, Willingdon Island, a seaport in Kochi, was
began to drift northward. It collided with the Eurasian created within Vembanad Lake.
landmass about 40 million years ago, and this collision
• The lake features a 1252-meter-long saltwater barrier
caused the formation of the Himalayas.
called Thanneermukkom, constructed to prevent
The northward movement of the Indian landmass is still saltwater intrusion into Kuttanad.
ongoing, and it is estimated that Indian landmass is moving
about 5 centimeters per year. This slow movement is causing 159. Solution: (c)
the Himalayas to rise at a rate of about 1 centimeter per year. Exp) Option c is the correct answer.
Important Tips The correct order of mountain passes from west to east is as
Paleomagnetism: follows:
• Paleomagnetism is a field of study that investigates the I. Shipki La
Earth’s magnetic field history using records found in IV. Lipulekh
rocks, sediments, and archaeological materials. II. Nathu La

PYQ Workbook 198


GENERAL GEOGRAPHY

III. Bomdi La • In Jaunpur, the river divides the city into two equal
halves and widens significantly.

162. Solution: (a)


Exp) Option a is the correct answer.
Kullu Valley is located between the mountain ranges
of Dhauladhar and Pir Panjal. This picturesque valley in
Himachal Pradesh is known for its temples, lush forests, and
apple orchards. The Beas River flows through it, bordered
by the majestic Dhauladhar and Pir Panjal ranges, creating a
stunning natural landscape.

160. Solution: (c) 163. Solution: (a)


Exp) Option c is the correct answer Exp) Option a is the correct answer.
Option c is correct. Black cotton soils are known as self- Statement 1 is correct: Devprayag is a town in Uttarakhand,
ploughed soils. They are clayey in nature and have a high India, where the Alaknanda and Bhagirathi rivers converge to
water-retention capacity. In the rainy season, they swell up
form the Ganges River, known as the Ganga. This confluence
and become sticky, making it difficult to plough. However, in
is one of the Panch Prayag (five confluences) along the
the dry season, they crack and become loose, making them
easy to plough. Ganges’s course. The meeting point of the Bhagirathi and
Alaknanda rivers, where they converge to form the Ganges
Option a is incorrect. Alluvial soils are formed from the
deposition of river sediments. They are loose and sandy in River (Ganga), is Devprayag. Located near New Tehri city in
nature, and do not have a high water-retention capacity. Tehri Garhwal District in Uttarakhand, Devprayag is one of
Therefore, they are not self-ploughed soils. the five sacred confluences known as the Panch Prayag.
Option b is incorrect. Lateritic soils are formed from the Statement 2 is correct: Rudraprayag, situated in Uttarakhand,
weathering of rocks in hot and humid climates. They are India, is where the Mandakini and Alaknanda rivers converge
acidic in nature and have a low water-retention capacity. to form the Alaknanda River. This confluence marks one of
Therefore, they are not self-ploughed soils.
the Panch Prayag (five confluences) along the course of the
Option d is incorrect. Desert soils are sandy and saline in Alaknanda River.
nature. They have a low water-retention capacity and are not
self-ploughed soils. Statement 3 is correct: The Alaknanda originates from
Badrinath in Uttarakhand and is one of the two primary
161. Solution: (c) tributaries of the Ganges, along with the Bhagirathi. Together,
Exp) Option c is the correct answer. they form the sacred Ganges River in Northern India.
The Gomti River is the only tributary of the River Ganges Statement 4 is incorrect: Shankaracharya was an early 8th
that originates in the plains, specifically from Gomat Taal
century Indian philosopher and theologian who consolidated
near Madho Tanda, Pilibhit, in Uttar Pradesh. Unlike many
the doctrine of Advaita Vedanta. He established four major
other tributaries that start in the hilly regions, the Gomti is
primarily fed by monsoon rains and groundwater sources, mathas in different regions of India - Badrinath, Dwarka,
making it unique among the Ganges’ tributaries. Puri, and Sringeri. These mathas were founded to promote
Advaita Vedanta and Sanatana Dharma. The Sringeri matha
Important Tips
is considered to be the biggest seat established by Adi
Gomti River:
Shankaracharya. It is also the oldest of the four mathas.
• Originating from Gomat Taal, also known as Fulhaar
Jheel, near Madho Tanda in Pilibhit, Uttar Pradesh. Important Tips
• It flows for approximately 900 kilometers through • The Alaknanda rises at the confluence and feet of the
the state of Uttar Pradesh before merging with the Satopanth and Bhagirath Kharak glacier in Uttarakhand
Ganges River in Ghazipur. near the border with Tibet.
• The confluence of the Gomti and Ganga rivers is • The headwaters of the Bhagirathi are formed at
marked by the famous Markandey Mahadeo temple. Gaumukh, at the foot of the Gangotri glacier and
• A significant tributary, the Sai River, joins the Gomti Khatling glaciers in the Garhwal Himalaya. These
near Jaunpur. two sacred rivers join to form the Ganges (Ganga) in
Devprayag.
• Several cities, including Lucknow, Lakhimpur Kheri,
Sultanpur, and Jaunpur, are situated along the banks • Devprayag is 70 km from Rishikesh. Devprayag has an
of the Gomti. average elevation of 830 metres (2,723 feet).

199 PYQ Workbook


GENERAL GEOGRAPHY

Important Tips
The difference between Himalayan and Peninsular
Rivers:

Himalayan Rivers Peninsular Rivers

• These rivers originate • These rivers originate


from the Himalayan from the peninsular
Mountain ranges. plateaus.

• They are
• They are longer
comparatively smaller
and larger than the
than the Himalayan
peninsular rivers.
Rivers.

• They have larger • They have smaller


basins and catchment basins and catchment
Important Tips areas. areas.
The Five Prayags: • The bedrocks of
• Devaprayag : the place of confluence of Bhagirathi • The bedrocks of these
these rivers are soft,
River and Alaknanda River. rivers are hard and not
sedimentary and
easily erodible.
• Rudraprayag : the place of confluence of Mandakini easily erodible.
River and Alaknanda River.
• They are perennial in • They are seasonal and
• Nandaprayag : the place of confluence of Nandakini nature. non-perennial.
River and Alaknanda River.
• They are fed by the
• Karnaprayag : the place of confluence of Pindar River • They are fed only by
meltwater from
and Alaknanda River. rain.
glaciers/ rains.
• Vishnuprayag : the place of confluence of Dhauliganga
River and Alaknanda River. • They form V-shaped • They form U-shaped
valleys. valleys.
164. Solution: (a)
• They may not form
Exp) Option a is the correct answer. • They form meanders.
meanders.
Statement 1 is correct: Most of the rivers originating from
the Himalayas are perennial, meaning they flow throughout • They form big deltas. • They form estuaries.
the year. This is because they are primarily fed by snowmelt • They are antecedent • They are consequent
from the Himalayan glaciers and receive consistent water rivers. rivers.
supply from the melting snow even during dry seasons. In
contrast, many peninsular rivers are rain-fed, which means
165. Solution: (c)
their flow is highly dependent on seasonal rainfall. During
the monsoon season, these rivers can swell significantly, but Exp) Option c is the correct answer.
they may experience reduced flow or even dry up during the The rivers Damodar, Koel, and Subarnarekha originate
dry season when there is less rainfall. from the Chhota Nagpur Plateau.
Statement 2 is correct: The Himalayan rivers have steeper • The 563-km-long Damodar originates near Chandwa
gradients or slopes because they originate from the high village in the Chhotanagpur hills in Bihar’s Palamau
mountain ranges of the Himalayas. The rapid descent from district. It flows through one of the richest mineral belts
the mountains results in fast-flowing and turbulent rivers. in the world before draining into the Hooghly, about 50
Whereas peninsular rivers generally have gentler gradients km south of Calcutta.
as they flow through the relatively flat Deccan Plateau. They
tend to have slower and less turbulent flows compared to the • Koel River originates on the Lawapani Waterfalls,
Himalayan rivers. near Lohardaga, Chota Nagpur Plateau a Lawapani
Waterfalls, near Lohardaga, Chota Nagpur Plateau
Statement 3 is incorrect: The Himalayan River, with its steep
82.5 kilometres (51 mi) from Ranchi and conjoins the
gradients and powerful flow, causes more erosion compared
Belsiangar and Singbhum Rivers. The Koel is fed by three
to Peninsular Rivers. This is due to the rugged terrain it
traverses, carrying large amounts of sediment eroded from streams in Jharkhand, namely the North Karo, South
the mountains. In contrast, Peninsular Rivers flow over Karo and Koina.
relatively stable landforms, resulting in lower erosion rates • The Subarnarekha River originates from Piska/Nagri
and sediment transport. (Rani Chuan) near Ranchi, on the Chota Nagpur

PYQ Workbook 200


GENERAL GEOGRAPHY

Plateau. This river flows through the states of Jharkhand, orchards.


West Bengal and Odisha before eventually emptying into
169. Solution: (b)
the Bay of Bengal.
Exp) Option b is the correct answer.
166. Solution: (b)
• Sankh: The Sankh River flows through Jharkhand,
Exp) Option b is the correct answer. Chhattisgarh, and Odisha states in India, covering a
Kanwar Lake is Asia’s largest freshwater oxbow lake located total distance of 240 kilometers. It originates in Gumla
in the Begusarai district of Bihar. It is the only Ramsar site district, Jharkhand, and merges with the Koel River in
in Bihar. Oxbow lakes form in flat, low plains where the river Odisha after passing through these states. It is not part
drains into another water body. It is a residual oxbow lake of Ganga River basin.
that was created by the geological meandering of the Burhi • North Koel: The North Koel rises on the Chhota Nagpur
Gandak River, a tributary of the Ganga. plateau and enters Latehar district, below Netarhat near
Important Tips Rud. After flowing nearly due west for about 32 Km, it
turns north at an almost complete right angle through
Additional facts on Kanwar Lake:
a gorge at Kutku, and flows through the centre of the
• Kabartal Wetland, also known as Kanwar Jheel, is district until it falls into the Son a few miles north-west
situated in the Begusarai district of Bihar, covering of Haidarnagar. It is a part of Ganga River basin.
2,620 hectares in the Indo-Gangetic plains.
• South Koel: South Koel River is a 285 Km long river
• It serves as a crucial flood buffer for the region and which runs across Jharkhand and Odisha states in India.
offers livelihood opportunities for local communities. It originates on the Lawapani Waterfalls, near Lohardaga,
• The wetland is home to significant biodiversity, Chota Nagpur Plateau and conjoins the Belsiangar and
including 165 plant species and 394 animal species, Singbhum Rivers.The South Koel enters Odisha and
with more than 50 fish species documented. joins with Sankh River at Vedavyas near Rourkela from
• As an essential stopover along the Central Asian where it is named Brahmani. It is not part of Ganga
Flyway, it hosts 58 migratory waterbird species for River basin.
resting and refueling. • Barakar: The Barakar River is the main tributary of the
• Five critically endangered species inhabit the site, Damodar River in eastern India. Originating near Padma
including three vultures (red-headed vulture, white- in Hazaribagh district of Jharkhand it flows for 225 Km
rumped vulture, Indian vulture) and two waterbirds across the northern part of the Chota Nagpur Plateau,
(sociable lapwing and Baer’s pochard). mostly in a west to east direction, before joining the
Damodar near Dishergarh in Asansol, Bardhaman
• Major threats to the site include water management
district of West Bengal. It cannot be called directly a
activities like drainage, water abstraction, damming,
part of Ganga River basin.
and canalization.

167. Solution: (d)


Exp) Option d is the correct answer.
Rajasthan receives very little rain because of its geographical
location and the orientation of the Aravalli range. The
Arabian Sea branch of the south-west monsoon that causes
rains in western and south-western India runs parallel to the
Aravalli range and thus hardly causes any rainfall as the
winds do not get interrupted (winds do not come across
any barriers to cause the necessary uplift to cool the air).
The Bay of Bengal branch of the monsoon also fails to reach
Rajasthan as it is blocked by the Himalayas and the Vindhya
range. Therefore, the winds that reach Rajasthan are dry and
hot and do not bring any moisture.

168. Solution: (b)


Exp) Option b is the correct answer 170. Solution: (a)
Karewas are the thick deposits of glacial clay and other Exp) Option a is the correct answer.
materials embedded with moraines. These are unconsolidated Coral reefs are not found in the Gulf of Cambay. However,
lacustrine deposits. they are present in other regions of India, including the
Karewa formations are useful for the cultivation of Zafran is Gulf of Kutch, Gulf of Mannar, Andaman & Nicobar
a local variety of Saffron in Kashmir valley. These are also Islands, and Lakshadweep Islands. India’s coastline of 7,517
important for the cultivation of almond, walnut, apple, and km hosts various types of coral reefs, such as atoll, fringing,

201 PYQ Workbook


GENERAL GEOGRAPHY

and barrier reefs, with each region having its unique coral • The Meghalaya Plateau spans from the river Dhansiri
ecosystem. in the east to the Singimari River in the west, covering
approximately 35,291 square kilometers.
• It is characterized by an average length of about
400 kilometers and an average width of around 40
kilometers.
• The Karbi Plateau, on the other hand, is pear-shaped
and covers an area of about 7,000 square kilometers.
• The link between the Karbi Plateau and the Meghalaya
Plateau proper is located to the south and involves a
patch of highly denuded and subdued senile terrain.
• This geological formation is believed to have been
caused by the northeastward movement of the Indian
plate during the Himalayan origin, resulting in a
significant fault between the Rajmahal hills and the
Karbi-Meghalaya plateau.
• Over time, this depression was filled up by the
depositional activity of numerous rivers.

172. Solution: (a)


Exp) Option a is the correct answer.
171. Solution: (a)
Amritsar is located in a relatively flat and low-lying area,
Exp) Option a is the correct answer.
with an elevation that is not significantly higher above sea
The Bhuban Hills are not a part of the Meghalaya Plateau. level. Shimla is situated in the Himalayan region. Shimla is
While the Meghalaya Plateau includes the Garo Hills, Khasi at a much higher altitude compared to Amritsar. Because of
Hills, and Jaintia Hills, the Bhuban Hills are located in this significant difference in altitude, Shimla experiences a
Assam’s Cachar district, situated near the Assam-Manipur much cooler and temperate climate compared to Amritsar.
border in the east and adjacent to the state of Mizoram in The higher elevation of Shimla leads to lower temperatures,
the south. especially during the summer months, making it a popular
hill station and summer retreat. In contrast, Amritsar, being
in a lower-lying region, has a warmer and more arid climate.

173. Solution: (d)


Exp) Option d is the correct answer
Loamy soil is a mixture of sand, silt, and clay particles.
The sand particles are the largest, the silt particles are
in the middle, and the clay particles are the smallest. The
proportions of these particles vary depending on the location
of the soil. Sand particles are good for drainage, silt particles
help to retain water, and clay particles help to bind the soil
together. Loamy soil is a good soil for growing crops because
it has a good balance of these properties.
Important Tips
Here are some other characteristics of loamy soil:
• It is a fertile soil that is suitable for growing a variety
of crops.
Important Tips
• It is easy to work with and does not become too
Karbi-Meghalaya plateau: compacted.
• The Karbi-Meghalaya plateau is an extension of • It has a moderate water-retention capacity.
the main Indian peninsular plateau and originally
• It is a good soil for drainage.
consists of two distinct plateaus: the Karbi Anglong
Plateau and the Meghalaya Plateau. 174. Solution: (a)
• The Meghalaya Plateau is traditionally divided into Exp) Option a is the correct answer.
three regions: Garo Hills, Khasi Hills, and Jaintia
Statement 1 is correct: The Meghalaya Plateau is an extension
Hills. of the Deccan Plateau towards the east. It stretches from the

PYQ Workbook 202


GENERAL GEOGRAPHY

Rajmahal Hills and is separated by the Garo-Rajmahal Gap.


This gap was created by a normal fault, leading to a downward
movement of a block of Earth. Sediments deposited by the
Ganga and Brahmaputra rivers later filled this gap. The
plateau comprises Archaean quartzites, shales, schists and
slopes towards the Brahmaputra valley in the north and the
Surma and Meghna valleys in the south.
Statement 2 is correct: A synclinorium is a type of fold
where the layers of rock are bent downwards. The Kashmir
Valley was formed when the Indian Plate collided with the
Eurasian Plate. The collision caused the Earth’s crust to
buckle and fold, creating the Pir Panjal and Zanskar ranges.
The Kashmir Valley is located in the trough between these
two mountain ranges.
Statement 3 is correct: The Gangetic Plain, a vast alluvial
plain in northern India, was created within a geological
feature known as a foredeep. A foredeep is a depression
created by the subsidence of the Earth’s crust adjacent to a
mountain range. It formed as a result of the collision between
the Indian Plate and the Eurasian Plate, with the Indian Plate
sliding beneath the Eurasian Plate, causing the Eurasian Plate
to push up and form the Himalayan Mountains while the
foredeep region sank, accumulating sediments over millions
of years, shaping the fertile Gangetic Plain.
Statement 4 is incorrect: The Himalayas originated as a
result of the collision of the Indian Plate and the Eurasian
Plate.

175. Solution: (d)


Exp) Option d is the correct answer.
The extra-peninsular mountains of India were formed
during the Cenozoic Era. The Cenozoic Era is the most
recent era in the geological timescale. It began about 66
million years ago and continues today. The extra-peninsular
mountains of India are the Himalayas, the Karakoram
Range, and the Hindu Kush. These mountains were formed
176. Solution: (b)
by the collision of the Indian Plate with the Eurasian Plate.
The collision began about 50 million years ago and is still Exp) Option b is the correct answer
ongoing. Bhangar is the older alluvial soil of the Gangetic plain. It is
found on the uplands and is characterized by its compactness
and high clay content. Bhangar is a fertile soil that is suitable
for growing a variety of crops, including wheat, rice, and
sugarcane. Old Kachhari clay is a type of Bhangar soil that
is found in the upper Gangetic plain. It is formed from the
weathering of older rocks and is rich in nutrients.

177. Solution: (a)


Exp) Option a is the correct answer.
The Chota Nagpur Plateau is characterized as a front-
sloping plateau because of its formation through
continental uplift due to tectonic forces. This geological
process pushed the region’s landmass above the surrounding
areas, resulting in an elevated plateau with a sloping front.
Important Tips
Chota Nagpur Plateau:
• Chota Nagpur is a vast plateau in eastern India,
spanning parts of Chhattisgarh and Jharkhand states.

203 PYQ Workbook


GENERAL GEOGRAPHY

• Comprising the Ranchi, Hazaribagh, and Kodarma The correct matching of the tributaries (List-I) with the
plateaus, it covers 25,293 square miles. The Ranchi respective rivers (List-II) is as follows:
Plateau is its largest section, with an average elevation A. Betwa – 2. Yamuna
of 2,300 feet. B. Kshipra – 1. Chambal
• Situated between the Ganges and Son rivers to the C. Wain Ganga – 4. Godavari
north and the Mahanadi River to the south, it is crossed
D. Tawa – 3. Narmada
by the coal-rich Damodar River valley.
• The region’s rich mineral resources include coal, mica, 181. Solution: (c)
copper, limestone, iron ore, and more. Bokaro hosts a Exp) Option c is the correct answer.
significant thermal plant and a large steel mill. The Chambal River Valley is known for its deep ravines,
making it a distinctive geographical feature. The Chambal
River and its tributaries have carved out these ravines
through constant vertical erosion over semi-arid and arid
regions. This type of terrain is characterized by undulating
floodplains, gullies, and deep ravines. The Chambal
ravines are characterized by their irregular topography,
steep ridges, low hills, deep trenches, and broad incised
meanders. This unique landscape poses challenges to
agriculture and livelihoods in the region due to soil erosion
and land degradation.

178. Solution: (d)


Exp) Option d is the correct answer.
The source of multiple rivers, namely Brahmaputra, Satluj, 182. Solution: (a)
and Indus, is found near Manasarovar Lake in Tibet. Exp) Option a is the correct answer.
• Brahmaputra: It originates at the Angsi Glacier, A superimposed boundary is a boundary that has been
southeast of Mount Kailash and Lake Manasarovar. imposed on an area by an outside or conquering power.
• Sutlej: The Sutlej’s source is located west of Lake The Border between India and Pakistan is an example of
Rakshastal, which is just west of Lake Manasarovar and Superimposed boundary as partition was enforced on them
south of Mount Kailash. by the British.
• Indus: The glaciers in the Kailash range near Mansarovar Important Tips
Lake serve as the source of the Indus River. • Subsequent Boundary are boundaries which are
established after the settlement in an area. It changes
179. Solution: (a)
as the cultural landscape changes and is drawn to
Exp) Option a is the correct answer. accommodate developments due to a certain event
The Indian monsoon is caused by the difference in heating such as war. The best example is Yugoslavia. It was
and cooling rates of the land and the ocean. During the formed following World War I by the merger of several
summer, the land gets hotter than the ocean, creating a low- territories.
pressure area over India. This attracts moist winds from the • Antecedent Boundary are boundaries that exists
ocean, which bring rainfall to the region. During the winter, before the present settlement and was often based on
the land cools faster than the ocean, creating a high-pressure landforms. One of the examples is of Andes Mountains
area over India. This pushes dry winds from the land to the which separates Chile, Argentina, and Bolivia.
ocean, resulting in dry weather.
• Relict Boundary is one that no longer functions but
180. Solution: (b) can still be detected on the cultural landscape. One of
Exp) Option b is the correct answer. the examples is of the Berlin Wall built in 1961.

PYQ Workbook 204


GENERAL GEOGRAPHY

183. Solution: (b) • Over 320 species of inland and migratory birds, with
Exp) Option b is the correct answer. waterfowl being predominant.
The Kaimur Range serves as the water divide between the • Common avian species: egrets, storks, herons,
Yamuna and Son rivers. This range is the eastern part of the sandpipers, curlews.
Vindhya Range and extends for about 483 kilometers from • The largest reptile is the Estuarine crocodile (up to 7
Katangi in Madhya Pradesh to Sasaram in Bihar. It forms the meters in length).
watershed or divide between these two major rivers, with the
• Supports nearly 400 fish species in its aquatic
Son flowing to the south and the Tons (tributary of Yamuna) ecosystems, encompassing both freshwater and
to the north. saltwater fish.
184. Solution: (a) 185. Solution: (c)
Exp) Option a is the correct answer. Exp) Option c is the correct answer.
The Sundarbans Delta is formed by the confluence of three The Narmada Valley lies between the Satpura and
major rivers: the Ganga, Brahmaputra, and Meghna. These Vindhyachal mountain ranges. The Narmada River flows
rivers deposit sediments over time, creating a unique and through this rift valley, which is bordered by these two
dynamic mangrove ecosystem. The Sundarbans is renowned mountain ranges. It is one of the unique features of the
for its Sundari trees and is divided into various protected
Narmada River, as it runs from east to west and eventually
areas, including Sundarbans West, Sundarbans South,
drains into the Arabian Sea.
Sundarbans East, and Sundarbans National Park.These
areas are designated as UNESCO World Heritage Sites due Important Tips
to their ecological significance. Narmada River:
• Narmada is the largest west-flowing river in
peninsular India.
• It flows westwards through a rift valley between the
Vindhyan Range to the north and the Satpura Range
to the south.
• Originating from the Maikala range near Amarkantak
in Madhya Pradesh at an elevation of about 1057 m.
• The Narmada basin spans states such as Madhya
Pradesh, Gujarat, Maharashtra, and Chhattisgarh,
covering an area of approximately 1 lakh sq. km.
• Bounded by the Vindhyas (north), Maikala range
(east), Satpuras (south), and the Arabian Sea (west).
• Total length from source to estuary in the Gulf of
Khambhat is 1,310 km.
Important Tips • The upper basin is hilly, while the lower-middle reaches
Vegetation of the Sundarbans: are fertile for cultivation.
• Predominantly mangrove vegetation, including trees, • Jabalpur is the primary urban center in the basin.
shrubs, grasses, epiphytes, and lianas. • Notable natural features include DhuanDhar Falls
• Most plants are evergreen with similar physiological (Marble Rocks), Mandhar and Dardi waterfalls, and
and structural adaptations. Sahasradhara Falls.
• Many trees have pneumatophores for aerial • Right bank Tributaries: Barna, Hiran River, Tendoni
respiration. River, Choral River, Kolar River, Man River, Uri River,
• Sundari (Heritiera fomes) and Gewa (Excoecaria Hatni River, Orsang River.
agallocha) are prominent species. • Left bank Tributaries: Burhner River, Banjar River,
• Contains 35 out of 50 known mangrove plant species. Sher River, Shakkar River, Dudhi River, Tawa River,
Fauna of the Sundarbans: Ganjal River, Chhota Tawa River, Kaveri River, Kundi
River, Goi River, Karjan River.
• Approximately 50 mammal species, including the
endangered Bengal Tiger. More than three fourths of • Major Hydro Power projects in the basin: Indira
the Royal Bengal Tigers in eastern India can be found Sagar, Sardar Sarovar, Omkareshwar, Bargi,
in the marshy delta of the Sunderbans. Maheshwar.

• Other notable mammals: Spotted deer, Barking deer, 186. Solution: (b)
Rhesus macaque, Leopard cat, and wild boar. Exp) Option b is the correct answer.

205 PYQ Workbook


GENERAL GEOGRAPHY

The Indian states that share international boundary with 190. Solution: (a)
Bhutan are West Bengal, Arunachal Pradesh, Sikkim, and Exp) Option a is the correct answer
Assam. It does not share its border with Meghalaya. Bhutan
Lime is a basic substance that can be used to neutralize the
Shares 699 Km long Border with India.
acidity of soil. It is added to the soil in the form of ground
Important Tips limestone or dolomite. Lime helps to improve the water-
holding capacity of the soil and makes it more suitable for
Bhutan
plant growth.
• It is a landlocked country and is bordered by China in
Gypsum is also a basic substance, but it is not as effective
the north and India in the south.
as lime in neutralizing acidity. Calcium is a nutrient that is
• The capital of Bhutan is Thimphu. essential for plant growth, but it does not help to neutralize
• Buddhism is the state religion of Bhutan. acidity. Vegetable compost is a good source of organic matter,
• The official language of Bhutan is Dzongkha. It is a but it does not have any alkaline properties.
Sino-Tibetan language. Therefore, lime is the best option for making acidic soil
cultivable.
Important Tips
Here are some other ways to make acidic soil cultivable:
• Add organic matter to the soil. Organic matter helps to
improve the soil structure and make it more alkaline.
• Use a cover crop. A cover crop is a plant that is grown
to cover the soil and prevent erosion. Cover crops also
help to add organic matter to the soil.
• Irrigate the soil regularly. Irrigation helps to flush out
the acidic compounds from the soil.

191. Solution: (b)


187. Solution: (d) Exp) Option b is the correct answer.
Exp) Option d is the correct answer. Palk Strait is a strait between southeastern India and northern
The general direction of flow of summer monsoon in India Sri Lanka. It connects the Bay of Bengal in the northeast with
is from South-West to North-East. This is because the the Palk Bay to Gulf of Mannar in the southwest.It ismade
summer monsoon in India is caused by the low-pressure of a chain of low islands and reef shoals that are collectively
area over the Thar Desert, which attracts moist winds from called Adam’s Bridge/Ram Setu.
the Indian Ocean. These winds while crossing the Equator Important Tips
get deflected towards their right because of the Coriolis
Suez Canal: It is a man-made waterway connecting the
Effect and reach the western coast of India as South-West
Mediterranean Sea to the Indian Ocean via the Red Sea.
Monsoon winds. The direction of these winds reverses from
the winter monsoon, which blows from the northeast during Gulf of Khambat: It is also known as Gulf of Cambay. It is
cooler months. an 80 miles long gulf that divides the Kathiawar peninsula
to the west from the eastern part of Gujarat. The Gulf of
188. Solution: (c) Cambay is home to Alang shipyard.
Exp) Option c is the correct answer. Gulf of Kutch: The Gulf of Kutch divides the Kutch and
Ghuar Mota in the Kutch region of Gujarat is the westernmost the Kathiawar peninsula regions in the state of Gujarat.
region of India. It is located at 68° 7’E in Gujarat. It has several ports including Mandvi, Bedi, and Kandla.

Important Tips 192. Solution: (d)


• Kibithu is the easternmost place in the country, which Exp) Option d is the correct answer.
is in Arunachal Pradesh at 23.3°N and 97˚25’E. Statement 1 is correct- Dust storms that occur in Rajasthan
• The northernmost point of India, Indira Col, is in the during May and June are primarily driven by the formation
union territory of Ladakh. of convectional currents at specific locations in the region.
Convectional currents involve the circular movement of air
189. Solution: (c) due to temperature differences, with warmer, less dense air
Exp) Option c is the correct answer. rising while cooler air descends. In Rajasthan, the intense
Indira Point is the southernmost point of India’s territory. It heating of the desert air during the scorching summer months
is located in Great Nicobar in Nicobar district of Andaman creates an unstable lower atmosphere, leading to the mixing
and Nicobar Islands of India. It was formerly known as of strong winds from the middle troposphere downward to
Pygmalion Point and Parsons Point. the surface and resulting in stronger surface winds. These

PYQ Workbook 206


GENERAL GEOGRAPHY

powerful winds, often associated with thunderstorms, give Important Tips


rise to dust storms.
• Western disturbances are low-pressure systems that
Statement 2 is incorrect- The Aravalli Mountain range runs originate from the Mediterranean region and affect
parallel to the southwest monsoonwinds originating from northwestern India, especially during winter. They
the Arabian Sea. Consequently, these mountains do not bring snowfall to the Himalayan regions and rainfall
effectively block these winds, resulting in western Rajasthan to the plains.
being characterized as a low-rainfall or arid region.
• South-West monsoon is the main source of rainfall for
Statement 3 is incorrect-Easterly Winds do not play a role most parts of India, except for Tamil Nadu and some
in the origination of Dust Storms in Rajasthan. Easterly parts of Andhra Pradesh and Karnataka. It occurs
winds are generally weak and gentle winds, and they do not from June to September and is caused by the southwest
tend to create the strong, gusty winds that are necessary for winds blowing from the Arabian Sea and the Bay of
dust storms to form. However, it helps to transport dust Bengal
storms from Rajasthan to other parts of India.
195. Solution: (c)
193. Solution: (b)
Exp) Option c is the correct answer.
Exp) Option b is the correct answer. Butea monosperma is the scientific name of Palash. They
Palghat, also known as the Palakkad Gap, is situated are bright red and yellow in color. During the spring, they
between the Nilgiris and the Annamalai Hills. This low seem like a brightly lit torch. These flowers are visible from
mountain pass in the Western Ghats connects Coimbatore distance like a flame and hence known as the flame of the
in Tamil Nadu and Palakkad in Kerala, playing a Forest.
significant role in weather patterns, transportation,
trade, and wind energy generation in the region. It allows 196. Solution: (b)
moisture-laden Southwest monsoon winds into western Exp) Option b is the correct answer.
Tamil Nadu, affecting rainfall and moderating temperatures. The Indian state which shares international boundary with
The gap serves as a vital trade route and transport corridor, Myanmar are Arunachal Pradesh, Nagaland, Mizoram and
with highways and railways passing through it, and is a Manipur. The Indian state which shares maximum boundary
major hub for wind power generation due to the funneling with Myanmar is Arunachal Pradesh with long border of 520
effect on winds. km. The other states bordering Myanmar with their length
are Mizoram (510 km), Manipur (398 km), and Nagaland
(215 km).
Important Tips
Myanmar
Myanmar was formerly known as Burma.
It gained independence from Britain in 1948.
Th Capital of Myanmar is Nay Pyi Taw.

197. Solution: (b)


Exp) Option b is the correct answer.
The correct matching of rivers (List-I) with their places of
origin (List-II) is as follows:
A. Mand - 4. Mainpat
B. Banas - 3. Devgarh hills
C. Jonk - 2. Sonbera plateau
D. Kelo - 1. Ludeng hills
Important Tips
• The Mand River is a tributary of the Mahanadi in
194. Solution: (c) India. The river originates from the northern part
of the plateau Mainpat of Surguja district. It joins
Exp) Option c is the correct answer.
the Mahanadi in Chandarpur, in Chhattisgarh, 28 km
Tamil Nadu coast receives winter rainfall mostly due to from the Odisha border and before the river reaches
the North-East monsoon, which occurs from October to the Hirakud Dam.
December. The North-East monsoon is caused by the reversal
of winds from the northeast over the Bay of Bengal, which 198. Solution: (d)
bring moisture and rainfall to the eastern coast of India. Exp) Option d is the correct answer.

207 PYQ Workbook


GENERAL GEOGRAPHY

The correct decreasing order of the lengths of the rivers • Extends for about 2,400 km from Potwar Plateau to
Mahanadi, Krishna, Godavari and Narmada is: Brahmaputra valley.
• Godavari (1465 km) • Southern slopes are steep, while northern slopes
• Krishna (1400 km) are gentle.
• Narmada (1312 km) • Width varies from 50 km in Himachal Pradesh to
• Mahanadi (858 km) <15 km in Arunachal Pradesh.
• Valleys form synclines, and hills form anticlines.
199. Solution: (b)
• Middle or Lesser Himalayas (Himachal or Lower
Exp) Option b is the correct answer
Himalaya):
Option a is incorrect. Alluvial soil is found in about 40%
• Located between Shiwaliks (south) and Greater
portion of India’s land surface.
Himalayas (north).
Option b is correct. Phosphoric acid is found in rich
• Runs parallel to both ranges.
quantities in alluvial soil.
• 60-80 km wide, 2400 km long.
Option c is incorrect. Alluvial soil is also known by names
such as Khadar and Bhangar. Reh, Thur and Chopan are the • Elevations from 3,500 to 4,500 m.
names of desert soils. • Steep, bare southern slopes; forested, gentle
Option d is incorrect. Alluvial soil is fertile soil. It is the northern slopes.
most fertile soil in India and is suitable for growing a variety • Notable ranges include Mussoorie, Nag Tibba, and
of crops. Mahabharat Lekh.

200. Solution: (c) • Extends into Sapt Kosi, Sikkim, Bhutan, and other
regions.
Exp) Option c is the correct answer.
• Greater Himalayas (Inner Himalaya or Himadri):
The religious place situated at the confluence of Mand
and Mahanadi rivers is the Chandrahasini Devi temple. • Average elevation of 6,100 m, width around 25 km.
This temple is located in Chhattisgarh’s Janjgir district, 30 • Composed of central crystalline rocks overlain by
km off the Raigarh City, specifically in the area known as metamorphosed sediments.
Chandrapur. It is a revered site where devotees worship Maa • Asymmetrical folds with steep southern slopes and
Chandrahasini Devi, and it is known for its significance as gentle northern slopes.
one of the 52 Shaktipeeths. • Terminates at syntaxial bends, like Nanga Parbat
201. Solution: (d) (NW) and Namcha Barwa (NE).
Exp) Option d is the correct answer. • Home to the world’s tallest peaks, perpetually
snow-covered.
The Arabian Sea branch of the monsoon affects the western
and central parts of India, but not the eastern parts. The 203. Solution: (c)
Chhattisgarh Basin lies in the eastern part of Madhya Exp) Option c is the correct answer.
Pradesh and is mainly influenced by the Bay of Bengal
branch of the monsoon. Sriharikota Island, located off the Bay of Bengal coast,
is situated near Pulicat Lake. It is a barrier island and is
The other regions mentioned in the options are all affected part of the Shar Project settlement in Tirupati district,
by the Arabian Sea branch to varying degrees. The Western Andhra Pradesh, India. The island is notable for housing
Ghats receive very heavy rainfall from this branch, while the Satish Dhawan Space Centre, which is one of India’s
the Deccan Plateau and Madhya Pradesh receive moderate primary satellite launch centers. The Indian Space Research
rainfall. Organisation (ISRO) utilizes Sriharikota for launching
202. Solution: (b) satellites, taking advantage of its proximity to the equator for
added launch efficiency.
Exp) Option b is the correct answer.
The correct order of the Himalayan ranges from south to Important Tips
north in the western section is: Pulicat Lake:
• Shiwalik Range • The second-largest brackish water ecosystem in
• Lesser Himalaya India, following Chilika Lake in Odisha.
• Great Himalaya • Situated on the border of Andhra Pradesh and Tamil
Nadu, with the majority of its expanse in Andhra
Important Tips Pradesh.
• Shiwalik Range (Outer Himalayas): • An annual haven for diverse bird species, including
grey pelicans and painted storks.
• Located between the Great Plains and Lesser
Himalayas. • Both grey pelicans and painted storks are categorized
as near-threatened species on the IUCN Red List of
• The altitude ranges from 600 to 1500 meters. Threatened Species.

PYQ Workbook 208


GENERAL GEOGRAPHY

204. Solution: (c) Statement 1 is correct. Laterite soil develops in areas with
Exp) Option c is the correct answer. high temperatures and heavy rainfall. The high temperatures
and rainfall cause the rocks to weather and the minerals to
The characteristic feature appropriate for the Narmada
leach out. This leaves behind a soil that is rich in iron and
River is “Structural origin”. The Narmada River flows
aluminum oxides, but low in humus and other nutrients.
through a rift valley, which is a geological formation resulting
from the Earth’s crust being stretched and pulled apart. This Statement 2 is incorrect. Laterite soil is not rich in humus
rift valley was formed due to tectonic forces and structural and is not found in West Bengal, Assam and Odisha. It is
processes, making it a river with a structural origin. The found in the Eastern Ghats and the Western Ghats, and in the
river flows between the Vindhyan Range on the north and southern part of Maharashtra, Karnataka, Andhra Pradesh,
the Satpura Range on the south, shaped by the geological Telangana, Kerala, and Tamil Nadu.
structures of the region. Important Tips
205. Solution: (c) Here are some other characteristics of laterite soil:
Exp) Option c is the correct answer. • It is a reddish-brown color due to the high iron oxide
Atolls are commonly found near Lakshadweep in India. content.
Lakshadweep consists of twelve atolls, three reefs, and five • It is porous and has a low water-retention capacity.
submerged banks, making it a region with numerous atolls. • It is acidic in nature.
These atolls have a typical northeast-southwest orientation,
• It is not suitable for growing a variety of crops, but can
with the islands located on the eastern rim and submerged
be used for growing plantation crops like tea, coffee,
reefs on the western rim, forming lagoons within.
rubber, and cashew nuts.
• Laterite soil can be improved by adding lime and
fertilizers. It can also be used to make bricks and tiles

207. Solution: (d)


Exp) Option d is the correct answer.
Pair 1 is correct: India occupies 3.28 million km2
geographical area which forms 2.4% of the world’s land area.
Pairs 2 and 3 are correct: India is entirely in the Northern
hemisphere. The mainland of India extends between latitudes
8°4’N and 37°6’N and longitudes 68°7’E and 97°25’E.
Pair 4 is incorrect: There are 28 states and 8 Union territories
in the country.

Important Tips Important Tips


Different types of coral reef formations: INDIA
• Fringing Reefs: These are coral reefs that develop in
close proximity to the shore and extend outward into
the sea, forming a submerged platform. For example,
the Ningaloo Reef in Australia.
• Barrier Reefs: Barrier reefs are located at a considerable
distance from the shoreline, separated by wide expanses
of water, and they run parallel to the coastline. The
Great Barrier Reef in Australia is a famous example.
• Atolls: Atolls are circular formations of reefs that
encircle a central lagoon, often surrounding a low-
lying island. They are commonly found in the Indian
and South Pacific Oceans, such as the Atolls of
Lakshadweep.
• Patch Reefs: These are small, isolated coral reefs that
emerge from the open seabed of the continental shelf
or island platform. They are typically situated between
fringing reefs and barrier reefs, varying in size and
rarely reaching the water’s surface.

206. Solution: (d)


Exp) Option d is the correct answer

209 PYQ Workbook


GENERAL GEOGRAPHY

• India has a land boundary of about 15,200 km and the • Velocity contrast: East-flowing rivers, with gentler
total length of the coastline of the mainland, including gradients, have slower flows allowing sediment
Andaman and Nicobar and Lakshadweep, is 7,516.6 transport. West-flowing rivers maintain high speeds
km. due to their steep gradients, impeding sediment
• India is bounded by the young fold mountains in the accumulation.
northwest, north and northeast. • Elevation difference: The eastward-sloping Deccan
• South of about 22° north latitude, young fold mountains Plateau elevates the Western side, limiting low-lying
begin to taper, and extends towards the Indian Ocean, areas for sediment deposition on the West coast.
dividing it into two seas, the Arabian Sea on the west • Estuaries vs. Deltas: East-flowing rivers, with their
and the Bay of Bengal on its east ability to deposit sediments, often form deltas near
• India is a southward extension of the Asian continent their mouths. Deltas are characterized by their fan-like
landforms created by the accumulation of sediments
208. Solution: (a) carried by the river. West-flowing rivers, due to their
Exp) Option a is the correct answer. higher speeds and reduced sediment-carrying capacity,
Kanyakumari is a coastal town in the state of Tamil Nadu. It tend to form estuaries instead. Estuaries are transitional
is the meeting point of the Bay of Bengal, Indian Ocean, and zones where freshwater meets seawater, but they lack
Arabian sea. This place is also known as Cape Comorin. the characteristic deltaic landforms.

Important Tips 211. Solution: (a)


• Nagercoil: It is a city and administrative headquarters Exp) Option a is the correct answer.
of Kanyakumari District in Tamil Nadu. The island that is not a part of the Laccadive Islands is
• Rameswaram: It is a town on Pamban Island, in the Tillanchong. Tillangchong, also known as Tillanchang, is
southeast Indian state of Tamil Nadu. Rameshwaram an island and a village in the Nicobar district of Andaman
Temple is renowned for being a part of one of the and Nicobar Islands, India. The other three islands, Agatti,
twelve Jyotirlingas of Lord Shiva in India. Kavaratti, and Kalpeni, are part of the Union Territory of
Lakshadweep in the Arabian Sea.
209. Solution: (a)
Exp) Option a is the correct answer.
The correct south to northward sequence of the mentioned
relief features is:
• Dhauladhar Range
• Ladakh Range
• Zaskar Range
• Karakoram Range

210. Solution: (d)


Exp) Option d is the correct answer.
The Mahanadi River does not form an estuary. An estuary
is a partially enclosed coastal body of brackish water formed
where freshwater from rivers and streams meets and mixes
with saltwater from the ocean or sea. The Narmada, Tapti,
and Mandavi rivers do form estuaries as they meet the
Arabian Sea. In the case of the Mahanadi River, it forms Important Tips
a delta as it empties into the Bay of Bengal. A delta is a • Agatti Island is a 7.6 km long island, situated on a
landform created by the deposition of sediment carried by coral atoll called Agatti atoll in the Union Territory
the river, typically resulting in a network of distributaries of Lakshadweep, India. It is 459 km west of the city of
or channels as the river divides into smaller streams before Kochi.
reaching the sea.
• Kavaratti is the capital of the Union Territory of
Important Tips Lakshadweep in India. Kavaratti is a census town
Deltas are common in the East-flowing rivers of India, as well as the name of the atoll upon which the town
while they are nearly absent on the West coast: stands. It is located 332 km (206 mi) west to the city
• Geological variation: East-flowing rivers pass through of Kannur.
regions with broken hills and plateaus, promoting • Kalpeni is an inhabited Atoll in the Union Territory
sediment deposition. West-flowing rivers face steep, of Lakshadweep, India. It has a distance of 287 km
rocky Western Ghats, hampering sediment collection. west of the city of Kochi.

PYQ Workbook 210


GENERAL GEOGRAPHY

212. Solution: (c) 213. Solution: (c)


Exp) Option c is the correct answer. Exp) Option c is the correct answer.
The Great Plains of India have a humid sub-tropical climate Ravi is not a tributary of the Sutlej River; it is actually
(Cwg) with dry winter. This climate type is characterized a tributary of the Chenab River. The Sutlej River’s main
by hot summers, mild winters, and a pronounced monsoon tributaries include Baspa, Spiti, Beas, Nogli Khad, and
season. The Great Plains of India cover the states of Punjab, Soan River.
Haryana, Uttar Pradesh, Bihar, and parts of Rajasthan and
Madhya Pradesh. Important Tips
Important Tips • Beas River: Beas River, an important tributary of
Sutlej, emerges from Rohtang pass in Himachal
Koppen climate classification system in India:
Pradesh. The river before entering Pakistan merges
According to the Koppen climate classification system, with the Sutlej River at Hari-Ke-Pattan in Punjab.
India has six major climatic subtypes: Af (tropical wet), The tourist resorts of Manali are situated on the right
Am (tropical monsoon), Aw (tropical wet and dry), BSh
banks of the river Beas.
(subtropical steppe), BWh (subtropical desert) and Cwa
(humid subtropical). Some of the characteristics of these • Baspa River: The Baspa River originates in the Baspa
climatic subtypes are: Hills and eventually joins the Sutlej River in the
Kinnaur district. Before merging with the Sutlej, it
• Af: This subtype is found in the Andaman and
receives contributions from several smaller rivers that
Nicobar Islands, Lakshadweep Islands and some
parts of Kerala and Tamil Nadu. It has no dry season are fed by melting snow water.
and receives abundant rainfall throughout the year. • Spiti River: The Spiti River has two tributaries, the
The average temperature of the coldest month is above Tegpo and Kabzian streams, which originate from
18°C. the Kunzum Range. This river is primarily sourced by
• Am: This subtype is found in most of the northeastern water from the Pin Valley and experiences its highest
states, some parts of West Bengal, Odisha, Andhra flow during the summer months when glacier melting
Pradesh and Karnataka. It has a short dry season is extensive. The Spiti River flows through the Spiti
and a long monsoon season. The rainfall is very high Valley and ultimately joins the Sutlej River in the
and varies with the monsoon winds. The average Kinnaur district as well.
temperature of the coldest month is above 18°C.
214. Solution: (c)
• Aw: This subtype is found in most of the central
and northern parts of India, including Delhi, Exp) Option c is the correct answer.
Uttar Pradesh, Bihar, Madhya Pradesh, Rajasthan, ‘Khair’ (Catechu) is a tree of Sub-Tropical Forests. The
Gujarat, Maharashtra and Telangana. It has a winter dominant species of subtropical forests are Teak, Sal,
dry season and a summer wet season. The rainfall is Shisham, Pipal, Mohowa, Khair, etc.These species of tree
moderate to low and depends on the summer monsoon. are mainly concentrated in Uttar Pradesh, Madhya Pradesh,
The average temperature of the coldest month is above Bihar, Rajasthan, Gujarat and Himachal Pradesh. These
18°C.
States mainly come under sub-tropical region.
• BSh: This subtype is found in some parts of Gujarat,
Rajasthan, Punjab, Haryana and Jammu and 215. Solution: (a)
Kashmir. It is a semi-arid or dry climate with low Exp) Option a is the correct answer.
rainfall and high evaporation. The summers are very
Malwa Plateau lies between the Aravalli and Vindhya
hot, and the winters are mild to cool. The average
temperature of the coldest month is above 18°C. ranges. This plateau is of volcanic origin and is located in
west-central India. It encompasses parts of western Madhya
• BWh: This subtype is found in some parts of Rajasthan,
Pradesh and southeastern Rajasthan. The Malwa region has
Gujarat and Ladakh. It is an arid or desert climate
with very low rainfall and very high evaporation. The historical and cultural significance and was once a princely
summers are extremely hot, and the winters are cold. state known as Madhya Bharat before being merged into
The average temperature of the coldest month is below Madhya Pradesh.
18°C.
216. Solution: (a)
• Cwa: This subtype is found in some parts of Punjab,
Haryana, Himachal Pradesh, Uttarakhand, Jammu Exp) Option a is the correct answer.
and Kashmir, Sikkim and Arunachal Pradesh. It is The correct match between List-I (Doab) and List-II (Rivers)
a humid subtropical climate with a winter dry season is as follows:
and a summer wet season. The rainfall is high and A. Bist Doab - 3. Between Beas and Satluj
varies with the monsoon winds. The summers are hot
and humid, and the winters go from cool to cold. The B. Bari Doab - 2. Between Ravi and Beas
average temperature of the coldest month is below C. Rachna Doab - 1. Between Ravi and Chenab
18°C but above -3°C. D. Chaj Doab - 4. Between Chenab and Jhelum

211 PYQ Workbook


GENERAL GEOGRAPHY

217. Solution: (b) The correct matching of the source regions (List-I) with the
Exp) Option b is the correct answer. respective rivers (List-II) is as follows:
The correct sequence of mountain peaks from east to west in A. Brahmagiri Hills – 1. Cauvery
the Himalayas is: B. Verinag Spring – 2. Jhelum
• Kanchenjunga
C. Mahabaleshwar – 3. Krishna
• Mount Everest
D. Chhota Nagpur Plateau – 4. Subarnarekha
• Annapurna
• Dhaulagiri Important Tips
• The river Cauvery originates at Talakaveri in Coorg
District of Karnataka in Brahmagiri Range of hills
in the Western ghats at an elevation of 1341 m. (above
MSL) and drains a total area of 81,155 Sq. Kms.
• The Jhelum rises from a deep spring at Verinag, in
western Jammu and Kashmir union territory.
• The Krishna River originates in the Western Ghats
near Mahabaleshwar at an elevation of about 1,300
metres (4,300 ft), in the state of Maharashtra in central
India.
• The Subarnarekha River originates from Piska/Nagri
(Rani Chuan) near Ranchi, on the Chota Nagpur
Plateau.

219. Solution: (d)


Exp) Option d is the correct answer.
The length of the day depends upon the geographic latitude
of a place and upon the inclination of the sun. It is due to
this inclination that as we go northwards in summer in the
northern hemisphere, the hours of daylight increase steadily.
This means that the places located at higher latitudes will
have longer daytime. Hence, Delhi will have maximum
length in the month of June, as Hyderabad, Chennai and
Bhopal are south of Delhi.
Important Tips
220. Solution: (b)
Doabs of Punjab:
Exp) Option b is the correct answer.
• Bist Doab, located between the Beas and Sutlej
rivers, is a highly fertile region known as Doaba. Key Artesian wells are found in the Tarai region of Uttarakhand.
cities in this region include Jalandhar, Hoshiarpur, These flowing artesian wells are a unique feature of the
and Kapurthala. Tarai zone, distinguished by their natural discharge of
groundwater without the need for pumps. The Bhabar
• Bari Doab, situated between the Beas and Ravi rivers,
zone is characterized by porous and permeable sediments,
is another fertile area often referred to as Majha. It
forming the recharge area for the artesian aquifer system
encompasses major cities like Amritsar, Lahore, Tarn
Taran, Gurdaspur, and Pathankot. in the Tarai region. The elevation difference between the
Bhabar zone and the Tarai zone creates the artesian head
• Rachna Doab, between the Ravi and Chenab rivers,
in the aquifer, allowing groundwater to naturally rise to
is known for its fertility and includes cities such as
the surface in the form of flowing wells. However, over-
Gujranwala and Sheikhupura in Pakistan.
exploitation and environmental factors are causing a decline
• Chaj Doab, lying between the Chenab and Jhelum in the water flow from these wells, posing challenges for
rivers, is another fertile region with significant cities sustainable agricultural practices in the region.
like Gujarat, Bhera, and Shahpur in Pakistan.
Important Tips
• Sindh Sagar Doab, located between the Indus River
and the Jhelum, experiences lower rainfall and is Artesian Wells:
less fertile. It encompasses areas around Rawalpindi, • Artesian wells are groundwater sources characterized
Pakistan. by natural flow without mechanical pumping.
218. Solution: (b) • They are typically found in confined aquifers,
Exp) Option b is the correct answer. sandwiched between impermeable layers.

PYQ Workbook 212


GENERAL GEOGRAPHY

• Water in artesian wells is under positive pressure, The four states that share a boundary with Telangana are:
causing it to rise above the aquifer’s top. • Chhattisgarh (north)
• These wells are prevalent in regions like the Tarai • Andhra Pradesh (south and east)
zone of Uttarakhand, India. • Karnataka (west)
• Water discharge in artesian wells is influenced by the • Maharashtra (north-west)
elevation difference between the aquifer and the
ground surface. 225. Solution: (b)

• Overexploitation, deforestation, and climate change Exp) Option b is the correct answer
can impact artesian wells’ water flow. • Loktak Lake is located in Manipur.
• Measures like installing control valves and conserving • Barapani Lake, also known as Umiam Lake, is located in
groundwater are essential to sustain these unique Meghalaya.
water sources. • Kolleru Lake is located in Andhra Pradesh.
221. Solution: (d) • Pulicat Lake is located in Tamil Nadu.
Exp) Option d is the correct answer. 226. Solution: (b)
The river that flows between Kedarnath and Rudraprayag Exp) Option b is the correct answer
is the Mandakini. It originates from the Chorabari glacier The Tropic of Cancer passes through 8 states in India: Gujarat,
near the ancient temple of Kedarnath and is a tributary of Rajasthan, Madhya Pradesh, Chhattisgarh, Jharkhand, West
the Alaknanda River. The Mandakini flows gracefully past Bengal, Tripura, and Mizoram. Of these, only Gujarat and
the Kedarnath temple and eventually meets the Alaknanda West Bengal are coastal states. The remaining 6 states are
River at Rudraprayag, forming a significant confluence point landlocked. Therefore, the Tropic of Cancer passes through
in the Panch Prayag region. 2 coastal states and 6 landlocked states of India.
Important Tips
227. Solution: (d)
Mandakini River: Exp) Option d is the correct answer
• The Mandakini River, a tributary of the Alaknanda, Chhattisgarh is the state with the longest extension in
originates from the Chorabari glacier near the north-south direction. Chhattisgarh has a north-south
Kedarnath in Uttarakhand. extension of about 416 kilometers. Jharkhand has a north-
• It flows past Kedarnath temple and Madhyamaheshwar south extension of about 388 kilometers, Telangana has a
temple at Ukhimath and merges with the Alaknanda north-south extension of about 376 kilometers, and Odisha
at Rudraprayag, forming a holy confluence known as has a north-south extension of about 360 kilometers.
Panch Prayag.
228. Solution: (a)
• The river follows NH 107, providing a scenic driving
Exp) Option a is the correct answer
experience in the hilly terrain.
Alluvial soils are formed by the deposition of river sediments.
• Eventually, the Alaknanda River flows to Devprayag,
They are typically found in floodplains and deltas. Alluvial
where it meets the Bhagirathi, forming the sacred
soils are rich in nutrients, such as potash, phosphoric acid,
Ganga River.
and lime. They are also well-drained and easy to work with.
• The Mandakini River holds religious significance and Alluvial soils can be further classified on the basis of their
is mentioned in ancient Hindu texts like the Srimad age.
Bhagavad and Ramayana.
Important Tips
222. Solution: (b)
• Black soil
Exp) Option b is the correct answer.
• Also known as regur soil, it is found in the Deccan
The birch tree is found inthe Himalayas.Birch trees are Plateau and the valleys of the Narmada and Tapi
usually found in montane forests having altitude more than rivers.
3500 meters above sea level. In India, they can only be seen
• It is a very fertile soil, rich in organic matter,
in the Himalayan region.
nitrogen, and phosphorus.
223. Solution: (c) • It is well-drained and has a high water holding
Exp) Option c is the correct answer capacity.
Assam is known as ‘land of red River and blue hills,’ • It is suitable for growing a variety of crops, including
contains three of India’s physiographic divisions, and cotton, wheat, and millet.
covers about 2.39% of the country’s total area. • Laterite soil
224. Solution: (d) • It is found in tropical and subtropical regions with
Exp) Option d is the correct answer high rainfall and high temperatures.

213 PYQ Workbook


GENERAL GEOGRAPHY

• It is a reddish-brown soil that is rich in iron and months, while the h indicates hot desert conditions.
aluminium. 232. Solution: (b)
• It is not very fertile, as it is low in organic matter Exp) Option b is the correct answer
and nitrogen.
Acoording to Indian Forest survey report, 2021
• It is often used for growing crops that are tolerant Lakshadweep has the highest percentage of forest area to
of poor soil conditions, such as cashew nuts and
the total geographical area, at 90.33%. Forest area to the
rubber.
total geographical area of Arunachal Pradesh is 79.33%, of
• Arid soil Nagaland is 73.90% and of Mizoram is 84.53%.
• It is found in hot, dry regions with low rainfall.
Important Tips
• It is a sandy or loamy soil that is low in organic
Key findings ofIndian Forest survey report, 2021
matter and nutrients.
• The total forest and tree cover is 24.62% of the
• It is often used for grazing or for growing drought-
country’s geographical area, showing an increase of
resistant crops, such as wheat and barley.
0.28% compared to the previous assessment in 2019.
229. Solution: (d) • Forest cover increased by 0.22%, while tree cover
Exp) Option d is the correct answer increased by 0.76% at the national level.
In India, the Indus River basin covers the states of Ladakh, • Within the Recorded Forest Area/Green Wash (RFA/
Jammu and Kashmir, Himachal Pradesh, Punjab, Haryana, GW), there was a slight increase of 31 sq km in forest
and Rajasthan. cover, and outside the RFA/GW, an increase of 1,509
sq km.
Important Tips
• The top five states with increased forest cover are
• Indus basin covers an area of about 1,165,500 square Andhra Pradesh, Telangana, Odisha, Karnataka, and
kilometers, and includes parts of China, India, Jharkhand.
Pakistan, and Afghanistan.
• Forest cover in hill districts decreased by 902 sq
• In India, the Indus basin spreads over the states of km (0.32%), while forest cover in tribal districts
Jammu & Kashmir, Himachal Pradesh, Punjab and a decreased by 655 sq km inside the RFA/GW and
part of Rajasthan, Haryana, and UT of Chandigarh increased by 600 sq km outside.
and Ladakh having an area of 3,21,289 sq. km, which
• The North-Eastern region experienced a decrease of
is nearly 9.8% of the total geographical area of the
1,020 sq km (0.60%) in forest cover.
country.
• Mangrove cover increased by 0.34%.
• The culturable area of the basin is 9.6 Mha (Million
Hectare Area), which is about 4.9 percent of the total • The total carbon stock in forests increased by 79.4
culturable area of the country. million tonnes since 2019, reaching 7,204.0 million
tonnes.
• The major part of the basin received an average annual
rainfall of over 620.96 mm. • Approximately 22.27% of the forest cover is prone to
forest fires.
• The major part of basin is covered with agricultural
land and accounting to 35.8 percent of the total area. • Bamboo-bearing areas decreased by 10,594 sq km
1.85 percent of the basin is covered by water bodies. compared to the estimate in 2019.
The snowmelt makes a significant contribution to this
233. Solution: (b)
huge water flow.
Exp) Option b is the correct answer
230. Solution: (b) Manikaran in Himachal Pradesh is known for its
Exp) Option b is the correct answer geothermal energy. Geothermal energy is a form of
Gujarat has the longest coastline in India, with a total renewable energy that is generated from the heat within the
length of 1,600 kilometers. It is followed by Maharashtra Earth. Manikaran is located in a region where there is a high
with 720 kilometers, Andhra Pradesh with 600 kilometers, concentration of geothermal energy, and this energy is used
and Tamil Nadu with 572 kilometers. to generate electricity and heat water.

231. Solution: (c) 234. Solution: (a)


Exp) Option c is the correct answer Exp) Option a is the correct answer
The western fringe of Rajasthan is best described by the Soil Type Salient Characteristic
climatic type Bwhw in Köppen’s scheme. The climatic type
A. Entisols Weakly developed soils with no horizons
B in Köppen’s scheme represents a dry climate with a
potential evapotranspiration that exceeds precipitation. The B Inceptisols Soils having weakly or quickly developed
w indicates that the dry season occurs during the winter horizons

PYQ Workbook 214


GENERAL GEOGRAPHY

C. Histosols These soils have very high quantity of vegetation. In Chhattisgarh, you will see mostly moist
organic matter in the upper layers deciduous vegetation. In Odisha, you will see a mix of
moist deciduous and mangrove vegetation.
D. Oxisols These soils are very old and highly
weathered 241. Solution: (c)

235. Solution: (c) Exp) Option c is the correct answer

Exp) Option c is the correct answer Statement 1 is correct:Karewas are the thick deposits of
glacial clay and other materials embedded with moraines.
Ten Degree Channel is found between Little Andaman and
These are unconsolidated lacustrine deposits meaning they
Car Nicobar. It is a 150 km wide channel that separates the
were formed in a lake. They were formed in Pleistocene
two islands. The channel is named after its location on the
period (1 Million years ago) when the entire Kashmir valley
10th degree of latitude north of the equator.
was under water.
236. Solution: (b) Statement 2 is correct:Due to the rise of Pir Panjal, the
Exp) Option b is the correct answer drainage was impounded and a lake of about 5000 sq. km
• Barkana Falls is located in the Shimoga district of area was developed, thus a basin was formed. Subsequently
Karnataka. the lake was drained through Bramulla gorge. The deposits
left in the process are known as Karewas.
• Dudhsagar Falls is located in the Bhagwan Mahavir
Wildlife Sanctuary in Goa. Statement 3 is correct: The fertile soil deposited by the
ancient lakes in the Karewa formations is highly suitable for
• Duduma Falls is located in the Koraput district of Odisha.
agriculture. The Kashmir Valley, with its Karewa deposits, is
• Kiliyur Falls is located in the Villupuram district of Tamil renowned for growing saffron, almonds, and walnuts.
Nadu.
242. Solution: (c)
237. Solution: (d)
Exp) Option c is the correct answer
Exp) Option d is the correct answer
Andhra Pradesh is well known globally for its variety of
• Kaliveli Lake is a saltwater lake located in the Viluppuram rocks and minerals. It has the largest deposits of Chrysotile
district of Tamil Nadu. Asbestos in the country. The Neelam Sanjiva Reddy Sagar
• Khajjiar Lake is a freshwater lake located in the Chamba Hydel Project is also located in this state.
district of Himachal Pradesh.
243. Solution: (d)
• Khecheopalri Lake is a glacial lake located in the West
Sikkim district of Sikkim. Exp) Option d is the correct answer

• Vembanad Lake is a brackish water lake located in the Statement 1 is correct: The Western Coast of India is
Alappuzha district of Kerala. characterized by a high rocky coastline. It is influenced by
strong wave action from the Arabian Sea, The coast is also
238. Solution: (d) experiencing long-term coastal retreat due to wave erosion.
Exp) Option d is the correct answer Statement 2 is correct: The high-energy waves along
Mount Thullier is located on the island of Great Nicobar, the West Coast of India have resulted in the formation of
which is the largest and southernmost island of the erosional landforms such as cliffs, sea caves, sea stacks, and
Andaman and Nicobar Islands. wave-cut platforms. These features indicate the dominant
erosional processes occurring along the coast.
239. Solution: (a)
Statement 3 is correct: The Eastern Coast of India, also
Exp) Option a is the correct answer known as the Coromandel Coast, is characterized by a
The Dhaleswari River is a tributary of the Barak River low-lying, sedimentary coast. It is influenced by sediment
in Manipur. It originates in the Manipur Hills and flows deposition from rivers and longshore drift. Depositional
through the Imphal Valley before joining the Barak River landforms such as beaches, spits, and estuaries are commonly
near Bhanga Bazar. Subansiri, Dhansiri and Manas are found along the East Coast of India.
tributaries of Brahmaputra river.
244. Solution: (c)
240. Solution: (b) Exp) Option c is the correct answer
Exp) Option b is the correct answer Andaman and Nicobar Islands has the largest area under
The shortest distance between Bikaner and Konark is via the coral reefs in India. The islands have a total of 1,021 square
NH52 highway. This highway passes through the states of kilometers of coral reefs, which is about 80% of the total
Rajasthan, Madhya Pradesh, Chhattisgarh, and Odisha. In coral reefs in India.
Rajasthan, you will see mostly desert vegetation, such as
cacti, thorny bushes, and scrubland. In Madhya Pradesh, 245. Solution: (a)
you will see a mix of tropical thorny and moist deciduous Exp) Option a is the correct answer

215 PYQ Workbook


GENERAL GEOGRAPHY

Sholas are temperate forests that are found in the higher The standard meridian of India, which is used as a reference
elevations of the Nilgiris, Anaimalai, and Palani hills for calculating time and determining the standard time of
in South India. These forests are characterized by short, the country, is located at longitude 82°30’ E. This imaginary
stunted trees that are evergreen. The trees in shola forests are line passes through the city of Allahabad (now Prayagraj)
adapted to the cool, moist conditions of the high altitudes. in Uttar Pradesh. It is commonly referred to as the “Indian
Standard Time (IST) meridian” and serves as the baseline
246. Solution: (c) for India’s time zone. By establishing a standard meridian,
Exp) Option c is the correct answer India ensures uniform timekeeping and coordination across
The Mesozoic era is the middle era of the Phanerozoic eon, the country.
lasting from 252 to 66 million years ago. It is also known as
251. Solution: (a)
the “Age of Reptiles” because reptiles were the dominant land
animals during this time. The Indian desert is believed to Exp) Option a is the correct answer.
have been under the sea during the Mesozoic era, when the Arunachal Pradesh, with its highest point reaching 7,060
Indian plate was part of Gondwanaland, a supercontinent meters, is more likely to experience blizzards. Blizzards
that also included Africa, South America, Antarctica, are cold, violent, powdery polar winds that are prevalent
Australia, and Madagascar. in high and mid-latitudes. Arunachal Pradesh’s elevation
and geographical location make it susceptible to blizzard
247. Solution: (d) conditions, making it important for individuals traveling to
Exp) Option d is the correct answer. this state to protect themselves from blizzards.
Black soil, also known as regur or black cotton soil, exhibits
252. Solution: (b)
the following characteristics:
Exp) Option b is the correct answer.
• It is rich in lime, iron, magnesia, and alumina, which
contribute to its fertility. The Gulf of Mannar biosphere reserve is known for its rich
biodiversity and diverse ecosystems, including islands
• It is clayey in texture, deep, and impermeable, which
with estuaries, beaches, coral reefs, salt marshes, and
means it has a high water-holding capacity.
mangroves. Located along the southeastern coast of India,
• It is mainly found in Maharashtra, Madhya Pradesh, the Gulf of Mannar separates the Tamil Nadu state from Sri
and Gujarat regions of India. Lanka.
248. Solution: (d) 253. Solution: (a)
Exp) Option d is the correct answer. Exp) Option a is the correct answer
Alluvial soils, formed through the deposition of sediment by Statement 1 is correct: India follows Indian Standard Time
rivers and streams in floodplains and river valleys, exhibit (IST), which is the standard time used across the country.
a range of particle sizes and soil textures, from sandy loam
Statement 2 is incorrect: Andaman and Nicobar Islands and
to clay. These soils are characterized by their richness in
Lakshadweep Islands do not have different time zones. They
potash (potassium), an essential nutrient for plant growth.
also follow Indian Standard Time (IST).
However, they tend to be deficient in phosphorus, which is
crucial for energy transfer and various metabolic processes Statement 3 is incorrect: Indian Standard Time (IST) is five
in plants. and a half hours ahead of GMT (Greenwich Mean Time),
not behind it. GMT is the primary time standard used as a
249. Solution: (b) reference point for time zones around the world, and IST is
Exp) Option b is the correct answer. ahead of it by five and a half hours.
The Ten Degree Channel is the water body that separates 254. Solution: (d)
the Andaman and Nicobar Islands. It is a strait located
Exp) Option d is the correct answer
in the Bay of Bengal, running along the approximate 10th
parallel north of the equator. This channel serves as a crucial D had recently visited Khardung La, Nubra Valley, and
maritime route, allowing ships and vessels to navigate several Buddhist monasteries which are located in Ladakh.
between the Andaman Sea and the Bay of Bengal, providing 255. Solution: (a)
a connection between the Andaman and Nicobar Islands.
Exp) Option a is the correct answer
Important Tips Punjab is the state that receives rainfall from both the
The Eight Degree Channel separates Minicoy Island and Arabian Sea branch and the Bay of Bengal branch of
the Maldives, while the Nine Degree Channel separates Monsoon. The Arabian Sea branch of the Monsoon brings
Kalpeni, Suheli Par, and Minicoy Island from Maliku rain to the western part of Punjab, while the Bay of Bengal
Atoll in the Indian Ocean. The Eleven degree channel branch brings rain to the eastern part of Punjab.
divides Amindivi and Cannanore.
Important Tips
250. Solution: (b) Punjab receives rainfall from both branches of the
Exp) Option b is the correct answer. Monsoon:

PYQ Workbook 216


GENERAL GEOGRAPHY

• The Arabian Sea branch of the Monsoon brings rain to The shortest train route from Bengaluru to New Delhi is 2276
the western part of Punjab because it blows from the km, via Dharmavaram Junction, Kurnool Town, Kacheguda,
Arabian Sea towards the Himalayas.As the winds rise Balharshah, Nagpur, Bhopal, and Jhansi. The Yamuna river
up the Himalayas, they cool and condense, forming flows alongside the New Delhi Railway station, but it
clouds. These clouds then release their moisture as does not pass through it. Therefore, D will not cross the
rain on the western side of the Himalayas, including Yamuna river while performing her train journey. However,
the state of Punjab. D will cross the Narmada, Godavari, and Krishna rivers
while traveling through Karnataka and Telangana..
• The Bay of Bengal branch of the Monsoon brings rain
to the eastern part of Punjab because it blows from the 258. Solution: (b)
Bay of Bengal towards the Thar Desert.As the winds
Exp) Option b is the correct answer
cross the Ganges Plain, they pick up moisture from
the rivers and the soil. This moisture is then released Mizoram is the state that passes through The Tropic of
as rain on the eastern side of the Himalayas, including Cancer, which has more north-south extension than east-
the state of Punjab. west extension and shares an international border with
Bangladesh and Myanmar.
256. Solution: (c)
Exp) Option c is the correct answer 259. Solution: (d)

Himachal Pradesh is the state where S had witnessed contour Exp) Option d is the correct answer
bunding and contour ploughing. Due to the undulating Ladakh is the Union Territory located at 34° N and 77° E.
terrain of Himachal Pradesh, contour bunding and contour The other Union Territories mentioned in the question are
ploughing are commonly employed techniques to prevent located at different latitudes and longitudes.
soil erosion and maintain soil fertility in the agricultural
• Andaman and Nicobar Islands: 12° N and 92° E
fields.
• Lakshadweep: 10° N and 74° E
Important Tips
• Puducherry: 12° N and 79° E
• Contour bunding is a technique used to prevent soil
erosion in hilly areas. It involves building small dams 260. Solution: (b)
or walls across the contours of the land to slow down Exp) Option b is the correct answer.
the flow of water.
The river that directly joins the Ganga is the Son. It is a
• Contour ploughing is a technique used to reduce major tributary originating in Madhya Pradesh and flowing
soil erosion and improve crop yields in hilly areas. It through Chhattisgarh, Jharkhand, and Bihar before
involves ploughing the land along the contours of joining the Ganga near Patna. Chambal, Betwa, and Ken are
the land instead of up and down the slopes. tributaries of the Yamuna River, not the Ganga.

261. Solution: (b)


Exp) Option b is the correct answer.
Among the given options, Leh receives the minimum
rainfall in a year. Leh is located in the mountainous region
of Ladakh in Jammu and Kashmir, India. Due to its high
altitude and location in the rain shadow of the Himalayas,
Leh experiences a cold desert climate with very low
precipitation.

Important Tips
A rain shadow area refers to a region of arid land situated
on the side of a mountain that is sheltered from prevailing
winds. Tall mountains act as barriers to the movement of
moist air, resulting in reduced rainfall and dry conditions
on the leeward side. However, if the mountains are
sufficiently high and intercept rain-bearing winds, they
can induce precipitation on the windward side while
creating a rain shadow effect on the opposite side.

262. Solution: (d)


Contour Bunding/Contour ploughing
Exp) Option d is the correct answer.
257. Solution: (b) South India is not characterized by extreme climatic
Exp) Option b is the correct answer conditions. South India generally experiences a tropical

217 PYQ Workbook


GENERAL GEOGRAPHY

monsoon climate, which is characterized by relatively high of interconnected lakes, lagoons, and canals, are known for
temperatures throughout the year and distinct wet and their rich biodiversity, including a significant population of
dry seasons. However, extreme climatic conditions such as cichlids.
severe cold, heavy snowfall, or arid desert-like conditions are
Important Tips
not typically found in South India.
• Cichlids are a diverse family of freshwater fish known
Statement a is correct: In South India, the diurnal (daily)
for their vibrant colors and varied species.
range of temperature is generally less compared to regions
• Relatively few cichlids occur in Asia. The areas of
with continental climates. The temperature remains relatively
the Middle East, India and Madagascar were closely
consistent throughout the day.
positioned along with what is now the eastern coast of
Statement b is correct: South India has a relatively narrow Africa.
annual temperature range. The temperatures do not vary
• It is because of this, that a small number of cichlids
significantly from season to season.
migrated to India.
Statement c is correct: South India experiences high
temperatures throughout the year due to its proximity to the 266. Solution: (c)
equator and the influence of the monsoon climate. Exp) Option c is the correct answer.

263. Solution: (c) The term ‘soil impoverishment’ refers to the condition
where the soil becomes very deficient in essential plant
Exp) Option c is the correct answer.
nutrients. This can happen due to various reasons, including
Temperate coniferous forests, also known as taiga or boreal excessive farming practices, improper land management,
forests, are characterized by long, cold winters and short, and insufficient replenishment of nutrients.
cool summers. Therefore, there is a considerable variation
in temperature throughout the year, with distinct seasonal Important Tips:
changes. Summers tend to be relatively short and cool, while • Soil erosion: Soil erosion refers to the process of the
winters are long and cold. removal of the topsoil by wind or water, leading to the
loss of soil fertility.
Important Tips
• Soil deposition: Soil deposition refers to the process
• Temperate coniferous forests have a sparse of sediment or soil particles being deposited in a
undergrowth due to the limited amount of sunlight new location, often as a result of erosion or sediment
reaching the forest floor, as the tall coniferous trees transport.
form a dense canopy. • Soil getting enriched with plant nutrients: It refers to
• Due to the short summer season, the growing period the process of adding or replenishing nutrients in the
in temperate coniferous forests is relatively brief, soil, leading to increased soil fertility.
typically ranging from 50 to 100 days.
267. Solution: (d)
• The temperate coniferous forest biome exhibits a
wide range in the spatial distribution of annual Exp) Option d is the correct answer.
precipitation, varying from relatively dry areas to Rains caused by thunderstorms during the hot weather
regions with higher levels of precipitation. season (mid-March to mid-June) in Karnataka are called
“Cherry blossoms.” These showers occur during the pre-
264. Solution: (c) monsoon period when the temperatures rise, and the hot and
Exp) Option c is the correct answer. humid conditions lead to the formation of thunderstorms.
• Annamudi is the highest peak in the Western Ghats Important Tips
Mountain range and is located in the Annamalai Hills
• Mango Showers are local winds in Kerala, Konkan, Goa
in Kerala, India.
that help in the ripening of mangoes.
• Doddabetta is the highest peak in the Nilgiri Hills, • Kalbaisakhi or Norwesters are the local winds in
which are located in the Western Ghats of Tamil Nadu, Bengal. It is known as Bardoichila in Assam.
India.
• Loo is the local wind flowing in the summers in the
• Dhupgarh is the highest peak in the Satpura Range, Indo-Gangetic plains.
which is a range of hills in central India spanning
multiple states. 268. Solution: (b)
• Guru Shikhar is the highest peak in the Aravalli Range, Exp) Option b is the correct answer.
a range of hills in western India. The river valley in India that is under the influence of
intensive gully erosion is the Chambal Valley.The running
265. Solution: (a)
water cuts through the clayey soils and makes deep channels
Exp) Option a is the correct answer. which are known as gullies. The soil is removed by water
The largest quantity of cichlids in India is found in the flowing along definite paths downs the slope or in channels,
backwaters of Kerala. The backwaters of Kerala, a network it is called gully erosion.

PYQ Workbook 218


GENERAL GEOGRAPHY

Important Tips 271. Solution: (d)

• The Chambal River flows through the states of Exp) Option d is the correct answer.
Rajasthan, Madhya Pradesh, and Uttar Pradesh in Banas is a tributary of the Chambal River. Chambal is itself a
central India. tributary of the Yamuna river.
• The Chambal Valley is known for its significant Important Tips
problem of gully erosion, which refers to the formation • Luni is a west-flowing river located entirely in
and development of deep and narrow channels (gullies) Rajasthan.
due to erosion caused by water runoff.
• It is also known as the Sagarmati River.
• The loose soil composition and steep slopes in the
• It rises near Ajmer and flows southwest into the Rann
Chambal Valley contribute to the occurrence of
of Kutch.
intensive gully erosion.
• The major left bank tributaries of the Luni River
269. Solution: (a) include Jawai, Khari, Sukri, Sugai, Mithri, and Bandi.
Exp) Option a is the correct answer. • The Right Bank tributaries of the Luni River is Jojri.
The correct match is as follows: 272. Solution: (a)
(Lake) - (Type of Lake) Exp) Option a is the correct answer.
• Bhimtal (Uttrakhand) -Tectonic Lanak La pass is located in the Himalayas and it links Lhasa
• Purbasthali (West Bengal) - Fluvial (the capital city of Tibet) with Ladakh (a region in Jammu
and Kashmir, India).It is located at an elevation of 5466
• Gangabal (Jammu and kashmir) - Glacial
meters in Aksai-China (Ladakh).
• Lonar (Maharashtra)- Crater
Important Tips
Important Tips Other Important Passes:
• Lonar Lake: Lonar Lake is a tectonic lake located in • Mintaka Pass- Kashmir and China
Maharashtra, India. It was formed around 50,000 years
• Bara Lacha La- Himachal Pradesh and Jammu &
ago when a meteorite impact created a crater. The lake
Kashmir
is known for its unique saline and alkaline water and
its rich biodiversity. • Banihal Pass- Kashmir and China

• Gangabal Lake: Gangabal Lake is a high-altitude • Shipki La- Himachal Pradesh and Tibet
glacial lake situated in the Kashmir Valley, India. It is • Aghil Pass- Ladakh region and Xinjiang
located near the base of Mount Haramukh, a popular • Chang-La- Ladakh and Tibet
trekking destination. • Lanak La- India and China
• Purbasthali Lake: Purbasthali Lake is a fluvial lake
located in West Bengal, India. It is formed by the 273. Solution: (b)
damming of the Bhagirathi River. The lake serves Exp) Option b is the correct answer.
as a reservoir and plays a vital role in irrigation and Leh is located in the high-altitude region of Ladakh, in
agriculture in the region. the northern part of India. During winter, the region
• Bhimtal Lake: Bhimtal Lake is a picturesque lake experiences extremely cold temperatures and clear skies.
situated in the town of Bhimtal, Uttarakhand, India. The high elevation and cold temperatures result in a higher
It is a freshwater lake formed in a natural depression. atmospheric pressure compared to other places in India
during the winter season.
270. Solution: (b)
274. Solution: (b)
Exp) Option b is the correct answer.
Exp) Option b is the correct answer.
South Arcot and Ramanathapuram receive over 50 percent
of their annual rainfall from the North-East monsoon. The The correct order of the above rivers from East to West is
as follows:
North-East monsoon, also known as the winter monsoon
or the retreating monsoon, occurs during the months of • The Kosi River is located in the eastern part of Nepal and
October to December in the southern peninsular region of flows into the Ganga in Bihar.
India. • The Gandak River, also known as the Narayani River,
The regions of South Arcot and Ramanathapuram, located originates in Nepal and joins the Ganga in Bihar.
along the east coast of Tamil Nadu, are influenced by the • The Ghaghara River, also known as the Karnali River,
North-East monsoon. These areas receive a significant originates in Nepal and flows through Uttar Pradesh
portion of their annual rainfall during this season, making before joining the Ganga.
the North-East monsoon the primary source of precipitation • Finally, the Gomti River originates in Uttar Pradesh and
for these regions. merges with the Ganga near Varanasi.

219 PYQ Workbook


GENERAL GEOGRAPHY

275. Solution: (c) A. Zoji La: Zoji La is a mountain pass located in Jammu and
Exp) Option c is the correct answer. Kashmir, India.
The correct order of Tiger Reserves of India from North to B. Shipki La: Shipki La is a mountain pass located in
South is as follows: Himachal Pradesh, India.
• Dudhwa Tiger Reserve is located in Uttar Pradesh. C. Lipulekh: Lipulekh is a mountain pass located in
• Panna Tiger Reserve is located in Madhya Pradesh Uttarakhand, India.
• Pench Tiger Reserve is located in Madhya Pradesh and D. Nathu La: Nathu La is a mountain pass located in Sikkim,
Maharashtra India.
• Indravati Tiger Reserve is located in Chhattisgarh. Important Tips
Important Tips Some other mountain passes in India:
• Dudhwa Tiger Reserve: It is situated on the Indo- • Rohtang Pass: Situated in the Pir Panjal Range of
Nepal border and covers an area of approximately the Himalayas, Rohtang Pass in Himachal Pradesh
1,284 square kilometers. The reserve is home to several provides a vital link between the Kullu Valley and
endangered species, including the Bengal tiger, Indian Lahaul-Spiti Valley.
rhinoceros, swamp deer, and the Gangetic dolphin. • Bomdila Pass: Situated in the eastern Himalayas in
• Panna Tiger Reserve: Situated in Madhya Pradesh, Arunachal Pradesh, Bomdila Pass is a mountain pass
Panna Tiger Reserve is renowned for its scenic beauty connecting the Tawang district with the rest of India.
and wildlife. It spans an area of around 542 square • Chang La Pass: Situated in the Ladakh region of
kilometers and is characterized by its deep gorges, Jammu and Kashmir, Chang La Pass is one of the
plateaus, and teak forests. The reserve is known for its highest motorable mountain passes in the world.
successful tiger conservation efforts and is also home
• Karakoram Pass: Located in the Karakoram Range,
to species like leopard, sloth bear, and various species
the Karakoram Pass is a high mountain pass connecting
of deer and antelope.
India with China’s Xinjiang region.
• Pench Tiger Reserve: Straddling the border of Madhya
Pradesh and Maharashtra, Pench Tiger Reserve is 278. Solution: (b)
named after the Pench River that flows through it. It Exp) Option b is the correct answer.
covers an area of approximately 758 square kilometers
Statement 1 is correct- The Deccan Plateau, located in the
and is famous for its rich biodiversity. The reserve is
southern part of the Indian Peninsula, gradually rises from
known for its thriving tiger population, along with
north to south, forming an inclined plane.
other wildlife species such as Indian leopard, Indian
bison (gaur), wild dog, and various bird species. Statement 2 is incorrect- The Malwa Plateau is not part of
the Vindhyan ranges. It is a separate plateau located in the
• Indravati Tiger Reserve: Located in the Bastar district
western part of central India, spanning across the states of
of Chhattisgarh, Indravati Tiger Reserve is spread
Madhya Pradesh and Rajasthan. The Vindhyan ranges, on
across an area of about 1,445 square kilometers. It is
recognized as one of the finest and most significant the other hand, lie to the south of the Malwa Plateau.
tiger habitats in India. The reserve is characterized by Statement 3 is correct- TheWestern Satpura Range acts as a
hilly terrains, dense forests, and the scenic Indravati barrier and separates the Narmada and Tapi rivers. It runs
River. Apart from tigers, it is home to leopards, wild roughly parallel to the Vindhyan ranges.
dogs, sloth bears, and a wide range of bird species. Statement 4 is correct- The Chota Nagpur Plateau, located
276. Solution: (a) in the northeastern part of the Indian Peninsula, is indeed
composed of Archaean granite and gneiss rocks.
Exp) Option a is the correct answer.
Both the statements are individually true and statement II is 279. Solution: (c)
the correct explanation of statement I. Exp) Option c is the correct answer.
Statement I is correct: The interior regions of Maharashtra, Option 1 is correct- Tso Kar is a lake situated in Ladakh.
particularly in the central and eastern parts, experience
Option 2 is correct- Pangong Tso is a well-known lake
relatively lower rainfall during the summer months.
located in Ladakh and extends into Tibet.
Statement II is correct: The Western Ghats act as a barrier
Option 3 is incorrect- Tsomgo also known as Changu Lake,
and cause the moisture-laden winds coming from the
is a glacial lake located in Sikkim.
Arabian Sea to rise and release rainfall on the windward side
(coastal areas), resulting in reduced rainfall in the leeward Option 4 is correct- Tso Moriri is a high-altitude lake
side (interior areas). located in Ladakh.

277. Solution: (c) 280. Solution: (d)


Exp) Option c is the correct answer. Exp) Option d is the correct answer.

PYQ Workbook 220


GENERAL GEOGRAPHY

Statement 1 is correct-Rainfall, temperature, and When traveling by road from Chennai to Kolkata, the correct
vegetation patterns are influenced by latitude. As one moves sequence of occurrences of rivers is-
from the equator towards the poles, there are variations in Penneru: It is a river that lies between Tamil Nadu and
climate, resulting in changes in rainfall, temperature, and Andhra Pradesh. It is located in the southern part of the
types of vegetation. As we move from the equator towards journey.
the poles, insolation (incoming solar radiation) decreases
Krishna: The Krishna River is one of the major rivers in
due to the increasing angle at which the Sun’s rays strike
South India. It flows through the states of Maharashtra,
the Earth’s surface.
Karnataka, and Andhra Pradesh. It is encountered after
Statement 2 is correct- The difference between the longest crossing Penneru.
day (summer solstice) and the shortest day (winter Godavari: The Godavari River is the second longest river
solstice) increases as one moves away from the equator in India and flows through the states of Maharashtra,
towards higher latitudes. This is because the tilt of the Earth’s Telangana, and Andhra Pradesh. It is encountered after
axis causes variations in the amount of daylight received at Krishna.
different latitudes.
Mahanadi: The Mahanadi River flows through the states of
Statement 3 is correct- Indira Point, the southernmost Chhattisgarh and Odisha. It is encountered after Godavari.
point of the Great Nicobar Island in India, is approximately
Subarnarekha: The Subarnarekha River is a river that flows
located at 6°45’N latitude. This latitude places it in the
through the states of Jharkhand, Odisha, and West Bengal.
tropical region.
It is encountered after Mahanadi and before reaching
281. Solution: (a) Kolkata.
Exp) Option a is the correct answer.
A. Bhimtal: Bhimtal is a lake located in the state of
Uttarakhand, India. It is a tectonic lake, meaning it was
formed due to the movement of tectonic plates.
B. Ashtamudi: Ashtamudi Lake is a scenic lake located in
Kerala, India. It is known for its unique ecosystem and
is classified as a lagoon, which is a shallow coastal lake
separated from the sea by sandbars or barrier islands.
C. Gohna: Gohna Lake is a beautiful lake situated in
the hills of Uttarakhand, India. It is formed due to a
landslide, which is a mass movement of rocks, soil, and
debris down a slope.
D. Lonar: Lonar Lake is a crater lake lake located in
Maharashtra, India. It is a unique lake as it was formed
around 50,000 years ago when a meteorite struck the
Earth, creating a crater.

282. Solution: (b)


Exp) Option b is the correct answer.
Statement 1 is incorrect- Himalayan rivers do not become 284. Solution: (b)
dry in the winter season. In fact, these rivers are fed by Exp) Option b is the correct answer.
snowmelt from the himalayas, which ensures a continuous Aridisols are generally low in organic matter content. The
flow throughout the year. arid conditions, combined with limited plant growth and
Statement 2 is correct- Rivers of the Peninsular Plateau, decomposition, result in a scarcity of organic materials in
such as the Godavari, Krishna, and Kaveri, have reached the soil. Aridisol is a soil order that is typically found in
maturity. This means they have completed the stages of arid or semi-arid regions characterized by limited rainfall
youth and middle age and have developed wide valleys and and high evaporation rates. It is associated with desert-like
meandering courses. conditions where water availability for plants is limited for
Statement 3 is correct- Himalayan rivers, like the Ganga, most of the year.
Brahmaputra, and Yamuna, depict all the three stages
Important Tips
of the normal cycle of erosion. They exhibityouthful
characteristicsin their upper reaches with steep gradients Apart from Aridisols, there are several other soil orders,
and V-shaped valleys, middle-aged features in the form of including:
meanders and broader valleys, and old-age characteristics • Alfisols: Moderately weathered soils with a clay-rich
near their delta regions. subsurface horizon.
283. Solution: (c) • Andisols: Soils formed from volcanic ash or other
volcanic materials.
Exp) Option c is the correct answer.

221 PYQ Workbook


GENERAL GEOGRAPHY

288. Solution: (a)


• Entisols: Recently formed soils with minimal
development of horizons. Exp) Option a is the correct answer
• Gelisols: Soils that are permanently or seasonally • Baghelkhand - 2. Madhya Pradesh
frozen for significant portions of the year. • Kuttanad - 3. Kerala
• Histosols: Organic soils characterized by a high • Saurashtra - 4. Gujarat
content of partially decomposed plant material (peat • Rarh - 1. West Bengal
or muck).
• Inceptisols: Young soils with only moderate 289. Solution: (b)
development of horizons. Exp) Option b is the correct answer
• Mollisols: Soils with a dark, fertile surface horizon • Location: India is located in the tropical region, which
rich in organic matter. allows it to receive the seasonal monsoon winds.
• Oxisols: Highly weathered tropical soils with a reddish • Thermal contrast: The differential heating and cooling
color and low fertility. of land and water during different seasons create a
• Spodosols: Acidic forest soils with a subsurface pressure gradient, leading to the monsoon winds.
accumulation of organic matter and aluminum and • Upper air circulation: The presence of upper air
iron compounds. anticyclone during the summer season over the Indian
• Ultisols: Highly weathered soils with a clay-rich subcontinent helps in the establishment of the southwest
subsurface horizon and low natural fertility. monsoon.
• The Himalayan Mountains: The Himalayan mountain
285. Solution: (d) range acts as a barrier to the cold northerly winds during
Exp) Option d is the correct answer the winter season and influences the direction of the
Shola forests are a type of tropical montane forest found monsoon winds.
in the upper reaches of the Western Ghats, in the Nilgiris, 290. Solution: (b)
Anaimalai hills and Palani hills. There are no shola forests
Exp) Option b is the correct answer
in the Himalayas.
The Palk Strait is a shallow strait that separates India from
286. Solution: (c) Sri Lanka. The average depth of the Palk Strait is only
Exp) Option c is the correct answer about 30 meters, which is not deep enough for large ships
Statement 1 is correct: Laterite soils are generally red in to cross.
colour due to the presence of iron oxides. 291. Solution: (d)
Statement 2 is incorrect: Laterite soils are deficient in Exp) Option d is the correct answer.
nitrogen and potash.
Statement 1 is correct: Cold waves in Northern India during
Statement 3 is incorrect: Laterite soils are well developed the winter season are primarily influenced by continental air
in the Deccan Plateau, the Western Ghats, and the Eastern masses. Northern India is located far away from the coastal
Ghats and not in Rajasthan and Uttar Pradesh. areas, and therefore, the maritime influence is limited,
Statement 4 is correct: Tapioca and cashew nuts are drought- resulting in colder temperatures.
resistant crops that can grow well in laterite soils. Statement 2 is correct: Northern India is close to the
Himalayan mountain range, which acts as a barrier to the
287. Solution: (a)
warm and moist air from the south. The Himalayas also
Exp) Option a is the correct answer receive heavy snowfall during winter, which lowers the
Tista is the westernmost tributary, followed by Sankosh, temperature of the surrounding areas
Subansiri, and finally Lohit, which is the easternmost Statement 3 is incorrect: The air masses that influence
tributary. Northern India during the winter season primarily originate
from the continental regions, such as Central Asia and
Siberia. These air masses are cold and dry, contributing to
the cold waves in Northern India. The polar regions, which
are located much farther north, have a relatively minimal
direct influence on the weather patterns in Northern India.

292. Solution: (c)


Exp) Option c is the correct answer.
• The Sharavathi Valley sanctuary is located in the state
of Karnataka. It is situated near the Sharavathi River and
is known for its diverse wildlife and lush greenery.

PYQ Workbook 222


GENERAL GEOGRAPHY

• The Satkosia Gorge sanctuary is located in the state of • Contour bunding involves constructing barriers along
Odisha. It is situated along the Mahanadi River and is
the contours of the land to control water flow and
known for its scenic beauty and rich biodiversity.
prevent erosion.
• Pirotan Island is located in the state of Gujarat. It is an
• Contour ploughing is a method where furrows or
uninhabited island located in the Arabian Sea near the
ridges are created along the contour lines of a slope.
coast of Gujarat. The island is known for its coral reefs
and rich marine life, including various species of fish, • Shifting cultivation, also known as slash-and-burn
crabs, and other marine organisms. agriculture, involves clearing land, burning vegetation,
• The Bhagwan Mahaveer sanctuary, also known as the and cultivating crops for a few years before moving to
Mollem National Park, is located in the state of Goa. It a new plot.
is a popular wildlife sanctuary and national park in Goa,
296. Solution: (a)
known for its diverse flora and fauna.
Exp) Option a is the correct answer.
293. Solution: (a)
Statement 1 is correct: When a high-pressure system
Exp) Option a is the correct answer.
develops, it is characterized by sinking air. As the air sinks,
Both the statements are individually true, and statement it tends to spread outwards from the center of high pressure.
II is the correct explanation of statement I.
This leads to the development of high and dry winds that
Statement I is correct: India’s offshore and deep-sea fish blow away from the high-pressure area. These winds are
catch is poor considering its marine potential of 20-25
typically dry because the sinking air suppresses the formation
million tonnes.
of clouds and precipitation.
Statement II is correct: India’s offshore and deep-sea fish
catch is very poor considering the marine potential of 20- Statement 2 is correct: In the case of the North-Western
25 million tonnes because Indian coast does not have many part of India, when a feeble high-pressure system develops
gulfs, bays, estuaries and backwaters as is the case with in mid-winter, it would bring colder air masses into the
Norway. As such it lacks good fishing grounds. region. The sinking air associated with the high pressure
prevents warm air from rising, leading to cooler temperatures
294. Solution: (c)
in the Northern plain.
Exp) Option c is the correct answer.
Statement 3 is incorrect: Scorching winds, commonly
If the Himalayas did not exist, the absence of this massive
known as “loo,” are hot and dry winds that typically occur
mountain range would have significant effects on the climate
of the surrounding regions. in the summer months, particularly in the North-Western
region of India. They are associated with the intense heating
• The Himalayas act as a barrier that prevents the cold,
dry winds from the north, such as the Siberian High, of the land, which creates a low-pressure system and draws
from directly reaching the northern plains of India. in hot winds from the surrounding areas.
• As a result, the northern plains, including the North Statement 4 is incorrect: High-pressure systems are
Indian plain, receive a relatively milder flow of air generally associated with stable atmospheric conditions,
during the winter months. where sinking air suppresses the formation of clouds and
• Without the presence of the Himalayas, the barrier precipitation.
effect would be absent, and the cold, dry winds from
the north would be able to reach the North Indian 297. Solution: (c)
plain more easily. Exp) Option c is the correct answer.
295. Solution: (a) The Veliconda group of low hills is a structural part of
Exp) Option a is the correct answer. the Eastern Ghats. The Veliconda Hills, also known as the
Velikonda Range, are a prominent part of the Eastern Ghats.
Statement 1 is correct: Both terracing and contour bunding
They are located in the southern part of the range, primarily
are effective methods for soil conservation in hilly regions
as they help reduce soil erosion by controlling water runoff. in the state of Andhra Pradesh.
Statement 2 is incorrect: While shifting cultivation may Important Tips
have certain cultural and historical significance, it is not • Veliconda Hills support a variety of vegetation,
considered a suitable method for soil conservation. including dry deciduous forests, scrublands, and
Statement 3 is correct: Contour ploughing is an effective grasslands.
technique for soil conservation in hilly regions, as it helps
• The region experiences a tropical monsoon climate,
to reduce soil erosion caused by runoff.
with hot summers and moderate to heavy rainfall
Important Tips during the monsoon season.
• Terracing involves creating flat or nearly flat areas • The Veliconda Hills have significant geological and
on slopes to slow down water runoff and prevent soil mineralogical importance, with deposits of limestone,
erosion. barytes, and other minerals found in the region.

223 PYQ Workbook


GENERAL GEOGRAPHY

298. Solution: (c) borders with Odisha, Telangana, Karnataka, Tamil Nadu,
Exp) Option c is the correct answer. and the Union Territory of Puducherry. Himachal Pradesh
shares borders with Jammu and Kashmir, Punjab, Haryana,
Ganga water is being shared at Farakka with Bangladesh.
Uttarakhand, and the Union Territory of Chandigarh.
The Ganges Water Sharing Treaty, signed in 1996, ensures
Telangana shares borders with Andhra Pradesh, Maharashtra,
the minimum level of water flow to be shared between India
Karnataka, and Chhattisgarh.
and Bangladesh during the dry season from January to May.
301. Solution: (a)
299. Solution: (b)
Exp) Option a is the correct answer.
Exp) Option b is the correct answer.
The Karbi Anglong Hills are located in the northeastern
part of the Deccan Plateau. They are a part of the Meghalaya
Plateau, which is a hilly region that is home to many different
ethnic groups.

302. Solution: (b)


Exp) Option b is the correct answer.
• Bhabar: Characterized by a belt of gravels.
• Bhangar: Characterized by coarse-grained sands,
gravels, and kankar.
• Khadar: Characterized by fine-grained silt and clay
deposits.
• Delta: Characterized by fertile, marshy lands.

300. Solution: (b)


Exp) Option b is the correct answer.

303. Solution: (d)


Exp) Option d is the correct answer.
Arid soils are characterized by their high salt content.
This is because the evaporation rate in arid regions is
very high, which causes the salts to accumulate in the soil.
In some areas, the salt content of arid soils is so high that
common salt is obtained by evaporating the saline water in
those areas.

304. Solution: (c)


Exp) Option c is the correct answer.
A. Sutlej is a tributary of the Indus River 4).
B. Kosi is a tributary of the Ganga River 3).
C. Teesta is a tributary of the Brahmaputra River 2).
The states of Chhattisgarh, Haryana, Jharkhand, Madhya
D. Chambal is a tributary of the Yamuna River 1).
Pradesh, and Telangana are all landlocked and do not
have an international boundary. Andhra Pradesh shares 305. Solution: (a)

PYQ Workbook 224


GENERAL GEOGRAPHY

Exp) Option a is the correct answer. The Gersoppa Falls, also known as the Jog Falls, are a
Rupen, Banas, and Machchu, are all desert rivers. They cataract of the Sharavati River in western Karnataka state,
flow through arid regions and do not have a perennial flow. southwestern India. They are one of the most spectacular
The Barak River, on the other hand, flows through the waterfalls in India and the second-highest plunge waterfall
northeastern states of India and Bangladesh. It is a major in the country after the Nohkalikai Falls in Meghalaya.
river in the region and has a perennial flow.
Important Tips
Important Tips • The Gersoppa Falls are located 18 miles (29 km)
• The Rupen River is a desert river that flows through upstream from Honavar at the river’s mouth on the
the Thar Desert in the northwestern part of India. It Arabian Sea.
originates in the Aravalli Range and flows through the • The river cuts through the Western Ghats to fall in four
arid regions of Rajasthan. cascades, the highest of which is about 830 feet (250
• The Banas River is another desert river located in the metres).
arid regions of Rajasthan, India. It originates from • The four cascades are known as the Raja, or
the Khamnor Hills in the Aravalli Range and flows Horseshoe; Roarer; Rocket; and Ranee (“Queen”), or
through the Thar Desert. La Dame Blanche (“The White Lady”).
• The Machchu River is a desert river that flows through
the arid regions of Gujarat, India. It originates from 310. Solution: (d)
the Girnar Hills in the Saurashtra Peninsula and runs Exp) Option d is the correct answer.
through the Kathiawar region. Sabarmati has a length of about 370 kilometers and a
306. Solution: (b) catchment area of about 0.2 lakh square kilometers.
Exp) Option b is the correct answer. • Cauvery has a length of about 800 kilometers and a
Statement 1 is correct: It originates from Talakaveri in the catchment area of about 0.8 lakh square kilometers.
Brahmagiri range of the Western Ghats and flows through • Mahi has a length of about 580 kilometers and a
the states of Karnataka, Tamil Nadu, and Kerala. The valley catchment area of about 0.3 lakh square kilometers.
of the Kaveri River is home to a number of important cultural • Penner has a length of about 560 kilometers and a
and religious sites, including the temples of Srirangam and
catchment area of about 0.6 lakh square kilometers
Madurai.
Statement 2 is correct: Alakananda River is a tributary of 311. Solution: (a)
the Ganges River and originates in the Garhwal Himalayas. Exp) Option a is the correct answer.
The valley of the Alakananda River is still growing due to
The Arabian Sea has higher salinity than the Bay of Bengal
the ongoing erosion of the mountains.
because:
Statement 3 is incorrect: The Godavari River is the largest
east flowing peninsular river in India. The Arabian Sea exhibits high rate of evaporation and
low influx of freshwater. The Arabian Sea is located in a
307. Solution: (d) subtropical region with high solar radiation and low rainfall.
Exp) Option d is the correct answer. It also receives dry and hot winds from the surrounding
Raipur, the capital of Chhattisgarh, receives the maximum deserts, which increase the evaporation rate. The Arabian
rainfall. The average annual rainfall in Raipur is 1,498 Sea has few major rivers that discharge freshwater into it,
millimeters (58.9 inches). The city receives most of its except for the Indus River in Pakistan. The freshwater input
rainfall during the monsoon season, which runs from June from the Red Sea and the Persian Gulf is also negligible
to September. Hyderabad receives average annual rainfall compared to the evaporation loss.
of 1,250 millimeters (49.2 inches) compared to Bengaluru’s
1,175 millimeters (46.2 inches) and Delhi’s 642 millimeters 312. Solution: (a)
(25.2 inches). Exp) Option a is the correct answer.

308. Solution: (c) Gulf of Cambay/Khambhat is a shallow inlet of the Arabian


Sea along the coast of Gujarat. It does not have coral reef
Exp) Option c is the correct answer.
formation, as it is influenced by high sedimentation from
The 24° N latitude passes through the maximum number rivers such as Narmada and Tapti, as well as industrial
of Indian states. This latitude line runs through the central
pollution and urbanization.
part of India, traversing various states such as Rajasthan,
Madhya Pradesh, Uttar Pradesh, Jharkhand, West Bengal, Important Tips
and parts of Chhattisgarh, Manipur and Mizoram. It is an • The Gulf of Kutch in Gujarat features some of the
important reference line for understanding the geographical world’s northernmost reefs, which are adapted to
distribution and climatic patterns of these states.
high salinity and temperature variations. These are
309. Solution: (c) fringing reefs that are relatively less developed and
Exp) Option c is the correct answer. diverse than other reefs in India.

225 PYQ Workbook


GENERAL GEOGRAPHY

• Palk Bay is a shallow sea between India and Sri Important Tips
Lanka, which contains fringing reefs along the coasts • The Himalayas are young fold mountains, meaning
of Tamil Nadu and Sri Lanka. These reefs are rich in that they are formed by the folding and uplifting of the
biodiversity and support many livelihoods such as Earth’s crust due to tectonic forces.
fishing and seaweed farming.
• The Himalayas have geosynclinal rocks, meaning that
• The Andaman and Nicobar Islands are a group of they have rocks that were deposited in a geosyncline,
more than 500 islands in the Bay of Bengal, which which is a long and narrow trough of sedimentary rock
have fringing reefs around most of them. These reefs between two tectonic plates
are among the most pristine and diverse in India, with
• The Indus and Sutlej rivers form antecedent drainage
more than 200 species of coral and 1,000 species of
in Himalaya, meaning that they have maintained their
fish.
courses across the rising mountain ranges. Antecedent
• The Gulf of Mannar is a narrow sea between India drainage is a type of drainage pattern in which a river
and Sri Lanka, which contains 21 islands with fringing or stream cuts through a mountain barrier that is
reefs along the coasts of Tamil Nadu and Sri Lanka. uplifted due to tectonic forces.
• The Lakshadweep Islands are a group of 36 coral
atolls in the Arabian Sea, which have ring-shaped reefs 315. Solution: (b)
around them. Exp) Option b is the correct answer.

313. Solution: (d) Bum La Pass is a border pass between India and China,
Exp) Option d is the correct answer. located at an elevation of 5,000 m above sea level. It is
situated in the Tawang district of Arunachal Pradesh, a
There is no railway track of Indian Railway that passes
northeastern state of India that shares borders with China,
through Kaziranga National Park. The nearest railway
station to the park is Furkating Junction, which is about Bhutan, and Myanmar. Bum La Pass is about 37 km away
75 km away from the park. The station is well connected to from the town of Tawang and 43 km from the town of Tsona
some major stations like Guwahati, Kolkata, and Delhi. Dzong in China’s Tsona County.
• Kaziranga National Park is a wildlife sanctuary in Assam, Important Tips
India, that is famous for its one-horned rhinos, tigers, Important border pass of India:
elephants, and other animals. The park is located on the
• Nathu La Pass: It is in the state of Sikkim. This famous
banks of the Brahmaputra River, which flows through
pass is in the India- China border and was reopened
the park and provides water and habitat for many species.
in 2006. It forms a part of an offshoot of the ancient
• The Diphlu River is a tributary of the Brahmaputra silk route. It is one of the trading border posts between
River that forms the southern boundary of the park. India and China.
The river also serves as a natural barrier between the • Shipki La Pass: It is located through Sutlej Gorge. It
park and the human settlements. The river is home to connects Himachal Pradesh with Tibet. It is India’s
many aquatic animals, such as otters, dolphins, turtles, third border post for trade with China after Lipu Lekh
and fish. and Nathula Pass.
• National Highway No. 37 is a major road that connects • Lipulekh Pass: It is situated in the Pithoragarh district
Guwahati and Dibrugarh in Assam. The highway passes of Uttarakhand. It lies near the trijunction of India,
through the northern edge of the park and provides China, and Nepal. It connects the Kumaon region of
access to the various tourist attractions and entry gates of Uttarakhand with Tibet. It is India’s important border
the park. The highway also poses a threat to the wildlife, post for trade with China. The pilgrims to Kailas and
as many animals are killed or injured by speeding vehicles Manasarovar Lake travel through this pass.
• Zoji La Pass: It is in the Zaskar range of Jammu &
314. Solution: (c) Kashmir. The road route from Srinagar to Leh goes
Exp) Option c is the correct answer. through this pass. It was created by the Indus River.
The Himalayan Frontal Faults (HFF) do not separate • Banihal Pass: It is a mountain pass across the Pir
Himalaya from Tibet, meaning that they are not the Panjal Range at a maximum elevation of 2,832 m. It
boundary between the two regions. The HFF are a series connects the Kashmir Valley in the union territory of
of reverse faults that demarcate the boundary of the Siwalik Jammu and Kashmir to the outer Himalayas and the
Range, which is the southernmost range of the Himalayas, plains to the south. Jawahar tunnel was constructed
under this pass.
from the alluvial expanse of the Indo-Gangetic Plain, which
is a large plain in northern India and Pakistan. The HFF are • Jelep La Pass: This pass passes through the Chumbi
also known as the Main Frontal Thrust or the Himalayan valley. It connects Sikkim with Lhasa, the capital of
Frontal Thrust. Tibet.

PYQ Workbook 226


GENERAL GEOGRAPHY

• Mana Pass: The pass is one of the highest vehicle-


accessible passes in the world, containing a road
constructed in the 2005-2010 period for the Indian
military by the Border Roads Organisation. Mana Pass
is the last point between India and China Border.
• Niti Pass: The Niti Pass located at 5800 meters connects
India with Tibet. The pass is located in Uttarakhand.

316. Solution: (c) Important Tips

Exp) Option c is the correct answer. Different types of rainfall in India:

The Narmada River flows between the Satpura and Vindhya • Convectional rainfall: It is caused by the sun heating
ranges. It originates in the Amarkantak hills of Madhya the ground, which causes the air above it to warm
Pradesh and flows west for about 1,300 kilometers before up and rise. As the air rises, it cools and condenses,
emptying into the Gulf of Khambhat. forming clouds. When the water droplets in the clouds
become too large, they fall as rain. Convectional
Important Tips rainfall is common in tropical regions, where the sun
• The Tapi River flows between the Satpura and is strong, and the air is warm.
Sahyadri ranges. • Frontal rainfall: It is caused by the meeting of two air
• The Sabarmati River flows through the Aravalli masses with different temperatures and humidities.
Range. When the air masses meet, they force each other up,
which causes the air to cool and condense. When the
• The Mahi River flows between the Aravalli and
water droplets in the clouds become too large, they
Vindhya ranges.
fall as rain. Frontal rainfall is common in temperate
317. Solution: (a) regions, where the weather is more variable.
Exp) Option a is correct answer.
319. Solution: (b)
Statement 1 is correct- The Dharwar geological system
belong to the Archaean geological period. The Archaean Exp) Option b is the correct answer.
period is the oldest known geological period, dating back Statement 1 is correct- The Southern plateau block, which
to around 4 to 2.5 billion years ago. includes the Nilgiri Hills, Anaimalai Hills, and Cardamom
Statement 2 is correct- The rocks in the Dharwar system Hills, is predominantly composed of granite and gneiss
are metamorphosed sedimentary rocks. Metamorphism rocks.
refers to the process of changes in the mineralogy, texture, Statement 2 is correct- The Deccan lava plateau, also
and composition of rocks due to high pressures and known as the Deccan Trap, is a vast plateau region in India
temperatures. consisting of horizontally arranged layers of solidified lava
Statement 3 is correct- The Dharwar system is of great flows.
economic importance due to its mineral resources. It
Statement 3 is incorrect- The Malwa plateau is a part of
contains significant deposits of minerals like gold, copper,
the Indian Peninsular Plateau, but it is not located in the
iron ore, manganese, and others, making it an important
Eastern flank. It is situated in the northwest region of the
region for mining activities.
plateau.
Statement 4 is correct- The Dharwar system is predominantly
Statement 4 is correct- The Narmada and Tapti rivers have
found in the states of Karnataka, Tamil Nadu, Odisha, and
carved out a deep trough that lies between the Vindhyan
Jharkhand in India.
range and the Satpura range.
318. Solution: (c)
320. Solution: (c)
Exp) Option c is the correct answer.
Exp) Option c is the correct answer.
The most plausible explanation for the location of the Thar
Desert in Western India is the absence of mountains to the Three important rivers of the Indian sub-continent that
north of Rajasthan that could cause orographic rainfall in have their source near the Manasarovar Lake in the Great
the region. Unlike other areas where mountains can trigger Himalayas are the Brahmaputra, the Indus, and the Sutlej.
rainfall by forcing moist air to rise, the Thar Desert lacks The Brahmaputra River flows through Tibet, India, and
significant mountain ranges in its northern vicinity. The Bangladesh, while the Indus River passes through Tibet,
Aravalli Range in southern Rajasthan acts as a barrier, India, and Pakistan. The Sutlej River, a tributary of the
preventing moisture-laden winds from the Arabian Sea from Indus, also originates near the Manasarovar Lake and
reaching the northern parts of the region. As a result, the flows through Tibet, India, and Pakistan. These rivers play
area experiences arid or semi-arid conditions, leading to significant roles in the hydrology, culture, and economy of
the formation of the Thar Desert. the regions they traverse.

227 PYQ Workbook


GENERAL GEOGRAPHY

GEOGRAPHY
ECONOMIC & HUMAN GEOGRAPHY (INDIA AND THE WORLD)
*This unit consists of questions from Natural & Mineral Resources and their distribution across the
world, Industry and Location Factors and Population.

6.1. UPSC CSE Previous Years’ Questions 4. About three-fourths of world’s cobalt, a
metal required for the manufacture of
1. Consider the following statements: batteries for electric motor vehicles, is
Statement-I : India, despite having Uranium produced by [UPSC CSE Pre 2023]
deposits, depends on coal for most of its (a) Argentina
electricity production. (b) Botswana
Statement-II: Uranium, enriched to the (c) The Democratic Republic of the Congo
extent of at least 60%, is required for the (d) Kazakhstan
production of electricity.
5. With reference to India’s projects on
Which one of the following is correct in connectivity, consider the following
respect of the above statements? statements:
UPSC CSE Pre 2023]
1. East-West Corridor under Golden
(a) Both Statement-I and Statement – II are Quadrilateral Project connects Dibrugarh
correct and Statement- II is the correct
and Surat.
explanation for Statement- I.
(b) Both Statement I and Statement II are 2. Trilateral Highway connects Moreh in
correct and Statement-II is not the correct Manipur and Chiang Mai in Thailand via
explanation for Statement-I. Myanmar.
(c) Statement- I is correct but Statement-II is 3. Bangladesh- China- India- Myanmar
incorrect. Economic Corridor connects Varanasi in
(d) Statement-I incorrect but Statement-II is Uttar Pradesh with Kunming in China.
correct. How many of the above statements are
2. Consider the following pairs : correct? [UPSC CSE Pre 2023]
Port Well known as (a) Only one
(b) Only two
1. Kamarajar Port First major port in (c) All three
India registered as a (d) None
company.
6. Consider the following statements:
2. Mundra Port Largest privately
owned port in India. Statement-I: Switzerland is one of the leading
exporters of gold in terms of value.
3. Visakhapatnam Largest container
port in India. Statement-II: Switzerland has the second
How many of the above pairs are correctly largest gold reserves in the world.
matched? [UPSC CSE Pre 2023] Which one of the following is correct in
(a) Only one pair respect of the above statements?
(b) Only two pairs [UPSC CSE Pre 2023]
(c) All three pairs (a) Both Statement-I and Statement-II are
(d) None of the pairs correct and Statement-II is the correct
3. Ilmenite and rutile, abundantly available explanation for Statement-I.
in certain coastal tracts of India, are rich (b) Both Statement-1 and Statement-II are
sources of which one of the following? correct and Statement-II is not the correct
[UPSC CSE Pre 2023] explanation for Statement-I.
(a) Aluminium (c) Statement-l is correct but Statement-II is
(b) Copper incorrect.
(c) Iron (d) Statement-I is incorrect but Statement-II
(d) Titanium is correct.

PYQ Workbook 228


GENERAL GEOGRAPHY

7. With reference to coal-based thermal power rainfall for its growth. A light well-drained
plants in India, following statements: soil capable of retaining moisture is ideally
1. None of them uses seawater. suited for the cultivation of the crop.”
2. None of them is set up in water-stressed Which one of the following is that crop?
district. [UPSC CSE Pre. 2020]
3. None of them is privately owned. (a) Cotton
How many of the above statements are (b) Jute
correct? [UPSC CSE Pre 2023] (c) Sugarcane
(a) Only one (d) Tea
(b) Only two 12. With reference to the management of minor
(c) All three minerals in India, consider the following
(d) None statements:[UPSC CSE Pre. 2019]
8. With reference to India, consider the 1. Sand is a ‘minor mineral’ according to the
following statements: prevailing law in the country.
1. Monazite is a source of rare earths. 2. State Governments have the power to
2. Monazite contains thorium. grant mining leases of minor minerals,
3. Monazite occurs naturally in the entire but the powers regarding the formation
Indian coastal sands in India. of rules related to the grant of minor
4. In India, Government bodies only can minerals lie with the Central Government.
process or export monazite. 3. State Government have the power to
Which of the statements given above are frame rules to prevent illegal mining of
correct? [UPSC CSE Pre 2022] minor minerals.
(a) 1, 2 and 3 only Which of the statements given above is/are
(b) 1, 2 and 4 only correct?
(c) 3 and 4 only (a) 1 and 3 only
(d) 1, 2, 3 and 4 (b) 2 and 3 only
9. Consider the following statements: (c) 3 only
(d) 1, 2 and 3
1. Gujarat has the largest solar park in India.
2. Kerala has a fully solar powered 13. Consider the following statements:
International Airport. [UPSC CSE Pre. 2018]
3. Goa has the largest floating solar 1. In India, State Governments do not have
photovoltaic project in India. the power to auction non-coal mines.
Which of the statements given above is/are 2. Andhra Pradesh and Jharkhand do not
correct? [UPSC CSE Pre 2022] have gold mines.
(a) 1 and 2 3. Rajasthan has iron ore mines.
(b) 2 only Which of the statements given above is/are
(c) 1 and 3 correct?
(d) 3 only (a) 1 and 2 only
10. Consider the following minerals: (b) 2 only
[UPSC CSE Pre. 2020] (c) 1 and 3 only
1. Bentonite (d) 3 only
2. Chromite 14. In which of the following regions of India
3. Kyanite are shale gas resources found?
4. Sillimanite [UPSC CSE Pre. 2016]
In India, which of the above is/are officially 1. Cambay Basin
designated as major minerals? 2. Cauvery Basin
(a) 1 and 2 only 3. Krishna-Godavari Basin
(b) 4 only Select the correct answer using the code given
(c) 1 and 3 only below:
(d) 2, 3 and 4 only (a) 1 and 2 only
11. “The crop is subtropical in nature. A hard (b) 3 only
frost is injurious to it. It requires at least 210 (c) 2 and 3 only
frost -free days and 50 to 100 centimeters of (d) 1, 2 and 3

229 PYQ Workbook


GENERAL GEOGRAPHY

15. In which of the following activities are Which of the above pairs is/are correctly
Indian Remote Sensing (IRS) satellites matched?
used? [UPSC CSE Pre. 2015] (a) 1 and 2 only
1. Assessment of crop productivity (b) 3 only
2. Locating groundwater resources (c) 1, 2 and 3
(d) None
3. Mineral exploration
4. Telecommunications 19. Consider the following pairs:
5. Traffic studies [UPSC CSE Pre. 2014]
Select the correct answer using the code given Programme/Project Ministry
below. 1. Drought-Prone Ministry of
(a) 1, 2 and 3 only Area Programme Agriculture
(b) 4 and 5 only 2. Desert Ministry of
(c) 1 and 2 only Development Environment and
(d) 1, 2, 3, 4 and 5 Programme Forests
16. Consider the following pairs: 3. National Ministry of Rural
[UPSC CSE Pre. 2014] Watershed Development
Development
Region Well-known for the Project for
production of Rainfed Areas
1. Kinnaur Areca nut Which of the above pairs is/are correctly
2. Mewat Mango matched?
3. Coromandel Soya bean (a) 1 and 2 only
(b) 3 only
Which of the above pairs is/are correctly (c) 1, 2 and 3
matched? (d) None
(a) 1 and 2 only
20. What are the benefits of implementing
(b) 3 only the ‘Integrated Watershed Development
(c) 1, 2 and 3 Programme? [UPSC CSE Pre. 2014]
(d) None 1. Prevention of soil runoff
17. Consider the following towns of India: 2. Linking the country’s perennial rivers
[UPSC CSE Pre. 2014] with seasonal rivers
1. Bhadrachalam 3. Rainwater harvesting and recharge of
groundwater table
2. Chanderi
4. Regeneration of natural vegetation
3. Kancheepuram
Select the correct answer using the code given
4. Karnal below:
Which of the above are famous for the (a) 1 and 2 only
production of traditional sarees/fabric? (b) 2, 3 and 4 only
(a) 1 and 2 only (c) 1, 3 and 4 only
(b) 2 and 3 only (d) 1, 2, 3 and 4
(c) 1, 2 and 3 only 21. Which of the following is / are the
(d) 1, 3 and 4 only characteristic/ characteristics of Indian
18. Consider the following pairs: coal? [UPSC CSE Pre. 2013]
1. High ash content
[UPSC CSE Pre. 2014]
2. Low sulphur content
National Highway Cities connected 3. Low ash fusion temperature
1. NH 4 Chennai and Select the correct answer using the codes
Hyderabad given below.
2. NH 6 Mumbai and Kolkata (a) 1 and 2 only
(b) 2 only
3. NH 15 Ahmedabad and (c) 1 and3 only
Jodhpur (d) 1, 2 and 3

PYQ Workbook 230


GENERAL GEOGRAPHY

22. Consider the following statements: List-I List-II


[UPSC CSE Pre. 2013]
(Coal Producing (Coalfields)
1. Natural gas occurs in the Gondwana beds. Areas)
2. Mica occurs in abundance in Kodarma.
3. Dharwars are famous for petroleum. A. Damodar Valley 1. Talcher
Which of the statements given above is/are B. Son Valley 2. Barakar
correct? C. Godavari Valley 3. Umaria
(a) 1 and 2 only D. Mahanadi Valley 4. Singareni
(b) 2 only
Code: [UPSC CSE Pre 2010]
(c) 2 and 3 only
(d) None A B C D
23. Despite having large reserves of coal, why (a) 2 3 4 1
does India import millions of tonnes of (b) 3 2 1 4
coal? [UPSC CSE Pre. 2012] (c) 1 3 4 2
1. It is the policy of India to save its own (d) 4 1 2 3
coal reserves for future and import it 27. Tamil Nadu is a leading producer of mill-
from other countries for the present use. made cotton yarn in the country. What
2. Most of the power plants in India are could be the reason? [UPSC CSE Pre 2010]
coal based and they are not able to get
sufficient supplies of coal from within the 1. Black Cotton soil is the predominant type
country. of soil in the state.
3. Steel companies need large quantity of 2. Rich pool of skilled labour is available.
coking coal which has to be imported Which of the above is/are the correct reasons?
Which of the statements given above is/are (a) 1 only
correct? (b) 2 only
(a) 1 only (c) Both 1 and 2
(b) 2 and 3 only (d) Neither 1 nor 2
(c) 1 and 3 only
(d) 1, 2 and 3 28. Consider the following statements:
[UPSC CSE Pre 2009]
24. Which of the following is the chief
characteristic of ‘mixed farming’? 1. India does not have any deposits of
[UPSC CSE Pre. 2012] Thorium.
(a) Cultivation of both cash crops and food 2. Kerala’s monazite sands contain Uranium.
crops Which of the statements given above is/are
(b) Cultivation of two or more crops in the correct?
same field (a) 1 only
(c) Rearing of animals and cultivation of (b) 2 only
crops together
(c) Both 1 and 2
(d) None of the above.
(d) Neither 1 nor 2
25. Which one of the following is the appropriate
reason for considering the Gondwana 29. Which of the following minerals are found
rocks as most important of rock systems of in a natural way in the State of Chhattisgarh?
India? [UPSC CSE Pre 2010] [UPSC CSE Pre 2008]
(a) More than 90% of limestone reserves of 1. Bauxite
India are found in them. 2. Dolomite
(b) More than 90% of India’s coal reserves are
3. Iron ore
found in them.
(c) More than 90% of fertile black cotton 4. Tin
soils are spread over them. Select the correct answer using the code given
(d) None of the reasons given above is below:
appropriate in this context. (a) 1, 2 and 3
26. Match list-I with list-II and select the (b) 2, 3 and 4
correct answer using the codes given below (c) 1 and 4
the lists: (d) 1, 2, 3 and 4

231 PYQ Workbook


GENERAL GEOGRAPHY

30. Which one of the following statements is (c) (A) is true, but (R) is false.
not correct? [UPSC CSE Pre 2005] (d) (A) is false, but (R) is true.
(a) Rourkela Steel Plant, the first integrated 34. Which one of the following does not contain
steel plant in the Public Sector of India silver? [UPSC CSE Pre 2007]
was set up with the Soviet Union
(a) Horn Silver
collaboration.
(b) Salem Steel Plant is a premier producer of (b) German Silver
stainless steel in India. (c) Ruby Silver
(c) Maharashtra Elektrosmelt Ltd. is a (d) Lunar Caustic
subsidiary of the Steel Authority of India 35. Which of the following substances are
Ltd. found in the beach sands of many parts of
(d) Visakhapatnam Steel Plant is a unit of the Kerala? [UPSC CSE Pre 2006]
Rashtriya Ispat Nigam Ltd. 1. Ilmenite
31. Consider the following statements: 2. Zircon
1. Balaghat is known for its diamond mines. 3. Sillimanite
2. Majhgawan is known for its manganese 4. Tungsten
deposits. Select the correct answer using the codes
Which of the statements given above is/are given below:
correct? [UPSC CSE Pre 2007] (a) 1, 2, 3 and 4
(a) 1 only (b) 1, 2 and 3 only
(b) 2 only (c) 3 and 4 only
(c) Both 1 and 2 (d) 1 and 2 only
(d) Neither 1 nor 2
36. Which one of the following is a prominent
32. Match list-I with list-II and select the Uranium producer? [UPSC CSE Pre 2006]
correct answer using the codes given below (a) U.S.A.
the lists: (b) Canada
List-I List-II (c) Germany
(Aluminium (Location) (d) Zambia
Company) 37. Consider the following image:
A. BALCO 1. Hirakud [UPSC CSE Pre 2001]
B. HINDALCO 2. Korba
C. Indian Aluminium 3. Koraput
D. NALCO 4. Renukoot
Code: [UPSC CSE Pre 2007]
A B C D
(a) 3 1 4 2
(b) 2 4 1 3
(c) 3 4 1 2
(d) 2 1 4 3
33. Consider the following statements:
Assertion (A): There are no tea plantations
in any African country.
Reason (R): Tea Plants need fertile soil with
high humus.
Code: [UPSC CSE Pre 2007] In the above map, the black marks show the
(a) Both (A) and (R) are individually true, distribution of:
and (R) is the correct explanation of (A). (a) Asbestos
(b) Both (A) and (R) are individually true, (b) Gypsum
but (R) is not the correct explanation of (c) Limestone
(A). (d) Mica

PYQ Workbook 232


GENERAL GEOGRAPHY

38. Quartzite is metamorphosed from: 42. Match list-I with list-II and select the
[UPSC CSE Pre 2001] correct answer using the codes given below
(a) Limestone the lists:
(b) Obsidian List-I List-II
(c) Sandstone (Ores) (State where they are
(d) Shale mined)
39. Consider the following statements: A. Manganese 1. Madhya Pradesh
Assertion (A): In Australia, cattle rearing is B. Nickel 2. Odisha
done more for meat than for milk. C. Lead-zinc 3. Rajasthan
Reason (R): Australian are traditionally non- D. Asbestos 4. Andhra Pradesh
vegetarians. Code: [UPSC CSE Pre 1996]
Code: [UPSC CSE Pre 2000] A B C D
(a) Both (A) and (R) are true, and (R) is the (a) 1 3 2 4
correct explanation of (A). (b) 4 3 2 1
(c) 1 2 3 4
(b) Both (A) and (R) are true, but (R) is not
(d) 4 2 3 1
the correct explanation of (A).
(c) (A) is true, but (R) is false. 43. Which one of the following was part of the
(d) (A) is false, but (R) is true. erstwhile Soviet Union over which Kurdish
population is spread? [UPSC CSE Pre 1993]
40. Match list-I with list-II and select the (a) Armenia
correct answer using the codes given below (b) Azerbaijan
the lists: (c) Georgia
List-I List-II (d) Turkmenistan
(Minerals) (Major Producer) 6.2. Other Examination Previous Years’
A. Minerals Oil 1. Zambia Questions
B. Copper 2. Guyana 44. Which among the following is an important
C. Manganese 3. Venezuela mineral of Pegu Yoma region of Myanmar?
[UPPCS (Pre) 2022]
D. Bauxite 4. Gabon
(a) Tin
Code: [UPSC CSE Pre 2000] (b) Silver
A B C D (c) Mineral Oil
(a) 3 1 4 2 (d) Copper
(b) 3 1 2 4 45. Match list-I with list-II and select the
(c) 1 3 2 4 correct answer using the codes given below
(d) 1 3 4 2 the lists:
41. Consider the following statements: List-I List-II
Statement (A): Chile is still a prominent (Centre) (Industry)
copper producer in the world. A. Osaka 1. Cigar
Reason (R): Chile is endowed with the largest B. Detroit 2. Ship building
deposits of porphyry copper. C. Cuba 3. Cotton textile
Code: [UPSC CSE Pre 1999] D. St. Petersburg 4. Automobile
(a) Both (A) and (R) are true, and (R) is Code: [UPPCS (Pre) 2022]
correct explanation of (A). A B C D
(b) Both (A) and (R) are true, but (R) is not (a) 4 3 2 1
the correct explanation of (A). (b) 3 4 1 2
(c) (A) is true, but (R) is false. (c) 2 1 4 3
(d) (A) is false, but (R) is true. (d) 1 2 3 4

233 PYQ Workbook


GENERAL GEOGRAPHY

46. Match list-I with list-II and select the 50. Given below are two statements, one is
correct answer using the codes given below labelled as Assertion and other as Reason
the lists: (R): [U.P.P.C.S. (Pre) 2020]
List-I List-II Assertion (A): Ahmedabad is the largest
(Mineral Exploration (Headquarters) center of cotton textile industry in India.
Agencies) Reason (R): Ahmedabad is located in major
A. Oil and Natural Gas 1. Bhubaneswar cotton growing region of India, so it has no
Commission problem of raw materials.
B. Mineral Exploration 2. Hyderabad Select the correct answer from the codes
Corporation Ltd.
given below.
C. National Mineral 3. Nagpur
(a) Both (A) and (R) are true and (R) is the
Development
Corporation correct explanation of (A).
(b) Both (A) and (R) are true but (R) is not
D. National Aluminium 4. New Delhi
the correct explanation of (A).
Company Limited
Code: [UPPCS (Pre) 2021] (c) (A) is true but (R) is false.
A B C D (d) (A) is false but (R) is true.
(a) 4 2 3 1 51. Match list-I with list-II and select the
(b) 1 2 4 3 correct answer using the codes given below
(c) 4 3 2 1
(d) 2 3 4 1 the lists:

47. Which of the following pair is not correctly List-I List-II


matched? [UPPCS (Pre) 2021] (Ethnic Group) (Country)
Place Tribe A. Jews 1. Egypt
(a)Alaska Koryak B. Teda 2. Iran
(b) Borneo Punan C. Beja 3. Libya
(c) Arab Desert Ruwala D. Lur 4. Israel
(d)Sweden and Lapps Code: [U.P.P.C.S. (Pre) 2020]
Finland A B C D
48. Which one of the following is not a major (a) 1 2 3 4
Coco producer country? (b) 4 1 2 3
[U.P.P.C.S. (Pre) 2021] (c) 4 1 3 2
(a) Latvia (d) 4 3 1 2
(b) Cameroon
(c) Ghana 52. Which one of the following is not correctly
(d) Ivory Coast matched? [UPPCS (Pre) 2020]
49. Which of the following are not correctly Country Iron-Ore Producing
matched? Area
(Minerals) (Mining City) (a)Kazakhstan Karaganda
1. Copper Cigar (b) Ukraine Krivoy Rog
2. Iron Ore Ballari (c) Germany Normandy
3. Manganese Bhilwara (d)France Pyrenees
4. Bauxite Katni 53. In Malaysia, the ‘Kinta Valley’ is famous
Select the correct answer using the codes for: [U.P.P.C.S. (Pre) 2020]
given below. [UPPCS (Pre) 2020]
(a) Rubber Production
(a) 1 and 2
(b) 2 and 3 (b) Tea Production
(c) 1 and 3 (c) Tin Production
(d) 2 and 4 (d) Coffee Production

PYQ Workbook 234


GENERAL GEOGRAPHY

54. Postmasburg and its adjacent areas of South 58. Match list-I with list-II and select the
Africa is major producer of which of the correct answer using the codes given below
following minerals? [U.P.P.C.S. (Pre) 2020] the lists:
(a) Uranium List-I List-II
(b) Bauxite (Mineral) (Country)
(c) Manganese A. Silver 1. Chile
(d) Mica
B. Mica 2. Russian Federation
55. Match list-I with list-II and select the C. Iron Ore 3. Mexico
correct answer using the codes given below
D. Copper 4. India
the lists:
Code: [UPPCS (Mains) 2017]
List-I List-II A B C D
(Coal Fields) (Located) (a) 3 4 2 1
(b) 2 3 4 1
A. Talcher 1. Damodar Valley (c) 3 2 1 4
B. Karnapura 2. Son Valley (d) 3 1 4 2
C. Singroli 3. Godavari Valley 59. Match list-I with list-II and select the
D. Singreni 4. Mahanadi Valley correct answer using the codes given below
Code: [UPPCS (Pre) 2019] the lists:
A B C D List-I List-II
(a) 4 1 2 3 (Coalfields) (States)
(b) 3 2 1 4 A. Rajmahal 1. West Bengal
(c) 2 4 3 1 B. Sohagpur 2. Chhattisgarh
(d) 1 3 2 4
C. Bisrampur 3. Madhya Pradesh
56. Which of the following is not correctly D. Raniganj 4. Jharkhand
matched? [UPPCS (Pre) 2019] Code: [UPPCS (Mains) 2017]
Tribe Place
(a)Pygmy Congo basin A B C D
(a) 1 2 3 4
(b) Angami Nagaland
(b) 4 3 2 1
(c) Ainu Japan (c) 1 4 3 2
(d)Khirghiz Sudan (d) 3 2 4 1

57. Given below are two statements. One 60. What is meant by the term ‘Umland’?
labelled as Assertion and the other as [U.P.P.C.S. (Mains) 2017]
Reason (R): [U.P. P.C.S. (Mains) 2017] (a) The heart area of a town
(b) A satellite town
Assertion (A): Gondwana System of Rocks (c) Area surrounding a town
provides about 95% of the coal of India. (d) Residential area of a town
Reason (R): Most of the ferrous and non- 61. Match list-I with list-II and select the
ferrous minerals are associated with the correct answer using the codes given below
Dharwar system of rocks. the lists:
Select the correct answer from the codes List-I List-II
given below: ( I n d u s t r i a l (Country)
(a) Both (A) and (R) are correct, and (R) is Region)
the correct explanation of (A). A. Kinki 1. China
(b) Both (A) and (R) are correct, but (R) is B. Canton 2. Japan
not the correct explanation of (A).
C. Lorraine 3. Brazil
(c) (A) is true, but (R) is false.
(d) (A) is false, but (R) is true. D. Belo Horizonte 4. France

235 PYQ Workbook


GENERAL GEOGRAPHY

Code: [UPPCS (Pre) 2016] 68. Which of the following is the correct
A B C D sequence of States of India in descending
(a) 2 1 4 3 order in terms of reserves of coal?
(b) 2 1 3 4 [U.P.P.C.S. (Mains) 2014]
(c) 3 2 1 4 (a) Odisha, Chhattisgarh, Jharkhand,
(d) 4 3 2 1 Madhya Pradesh
(b) Chhattisgarh, Odisha, Madhya Pradesh,
62. Which of the following states has immense Jharkhand
Tin Ore reserves? (c) Jharkhand, Odisha, Chhattisgarh,
[U.P.P.C.S. (Pre) (Re-Exam) 2015] Madhya Pradesh
(a) Assam (d) Madhya Pradesh, Chhattisgarh,
(b) Jammu and Kashmir Jharkhand, Odisha
(c) Chhattisgarh 69. Rajasthan has near monopoly in-
(d) West Bengal [U.P.P.C.S. (Mains) 2013]
63. Which of the following states is the largest (a) Copper
producer of asbestos in India? (b) Mica
[U.P. Lower Sub. (Pre) 2015] (c) Zinc
(a) Madhya Pradesh (d) Dolomite
(b) Bihar 70. Match list-I with list-II and select the
(c) Karnataka correct answer using the codes given below
(d) Rajasthan the lists:
64. The ‘Rice Gene Bank’ of the world is situated List-I List-II
in– [U.P. Lower Sub. (Pre) 2015] (Industry) (Place)
(a) China A. Paper 1. Ambala Mukul
(b) Philippines
B. Cement 2. Bhilai
(c) Japan
(d) India C. Iron & Steel 3. Titagarh
65. For the location of which one of the D. Mineral Oil 4. Lakheri
Refinery
following industries, the availability of raw
material is not the primary consideration? Code: [UPPCS (Mains) 2013]
[U.P.P.C.S. (Pre) 2015] A B C D
(a) Iron and Steel (a) 2 4 3 1
(b) Sugar (b) 3 4 2 1
(c) Electronics (c) 4 2 1 3
(d) Cement (d) 2 3 1 4
71. Match list-I with list-II and select the
66. Which one of the following is not correctly
correct answer using the codes given below
matched? [UPPCS (Pre) (Re-Exam) 2015]
the lists:
Industry Location List-I List-II
(a)Paper Ontario (Industry) (Centre)
(b) Cotton Textiles Detroit A. Aluminium 1. Malanjkhand
(c) Chemical Texas B. Copper 2. Tundoo
(d)Motor Car Nagoya C. Zinc 3. J.K. Nagar
67. To which of the following racial groups, the D. Jute 4. Bhatpara
natives of North America belong? Code: [UPPCS (Pre) 2013]
[U.P.P.C.S. (Pre) 2015] A B C D
(a) Austric (a) 3 1 4 2
(b) Caucasoid (b) 3 1 2 4
(c) Mongoloid (c) 1 4 2 3
(d) Negroid (d) 1 2 3 4

PYQ Workbook 236


GENERAL GEOGRAPHY

72. Match list-I with list-II and select the List-I List-II
correct answer using the codes given below
the lists: (Coal fields) (Location)
A. Kuzbas 1. United Kingdom
List-I List-II
B. Red Basin 2. Russia
A. Cocoa 1. Ivory Coast
C. Bristol 3. Australia
B. Coffee 2. Kenya
D. New South 4. China
C. Tea 3. South Africa
Wales
D. Sugarcane 4. Ghana Code: [UPPCS (Mains) 2013]
Code: [UP Lower Sub. (Pre) 2013]
A B C D
A B C D
(a) 1 3 2 4
(a) 3 2 1 4 (b) 2 4 1 3
(b) 4 1 2 3
(c) 3 2 4 1
(c) 1 4 3 2
(d) 4 3 1 2
(d) 2 3 4 1
73. Match list-I with list-II and select the 76. Which one of the following pairs is not
correct answer using the codes given below correctly matched? [U.P.P.C.S. (Mains)
the lists: 2013]
(a) Cleveland - Iron and Steel
List-I List-II
(b) Detroit - Motor Vehicle
(Crop) (Region/Country) (c) Mesabi Range - Coal field
A. Rubber 1. Mauritius (d) Philadelphia – Ship Building
B. Coffee 2. Italy 77. Which of the following is correctly
C. Olive 3. Columbia matched? [U.P.P.C.S. (Mains) 2012]
D. Sugarcane 4. Malaysia (a) Eskimo - Amazon Basin
Code: [UPPCS (Pre) 2013] (b) Pigmy - Irrawadi Basin
(c) Bundu - Sahara
A B C D
(d) Bushman - Kalahari
(a) 1 2 3 4
(b) 4 3 2 1 78. Match list-I with list-II and select the
(c) 4 3 1 2 correct answer using the codes given below
(d) 3 4 2 1 the lists:
74. Match list-I with list-II and select the List-I List-II
correct answer using the codes given below (Mining Areas) (Minerals)
the lists:
A. Gurumahisani 1. Lead
List-I List-II
B. Talcher 2. Uranium
(Crop fields) (Country)
C. Jaduguda 3. Iron Ore
A. Donetsk 1. Germany
D. Zawar 4. Coal
B. Kuznetsk 2. UK
Code: [UPPCS (Pre) 2012]
C. Lancashire 3. Russia
A B C D
D. Saar 4. Ukraine (a) 3 4 2 1
Code: [UPPCS (Pre) 2013] (b) 3 2 1 4
A B C D (c) 2 4 3 1
(a) 1 2 3 4 (d) 1 2 3 4
(b) 4 3 2 1
(c) 3 4 1 2 79. Which of the following pairs is not correctly
(d) 1 3 2 4 matched?[U.P.P.C.S. (Pre) 2012]
(a) Fulani - West Africa
75. Match list-I with list-II and select the (b) Bantu - Sahara
correct answer using the codes given below (c) Masai - East Africa
the lists: (d) Nuba - Sudan

237 PYQ Workbook


GENERAL GEOGRAPHY

80. Match list-I with list-II and select the 83. Match list-I with list-II and select the
correct answer using the codes given below correct answer using the codes given below
the lists: the lists:
List-I List-II List-I List-II
(Copper Feilds) (State) A. Mineral Oil 1. Odisha
A. Chandrapur 1. Maharashtra B. Gypsum 2. Karnataka
B. Hasan 2. Andhra Pradesh C. Gold 3. Gujarat
C. Khamman 3. Rajasthan D. Bauxite 4. Rajasthan
D. Khetri 4. Karnataka Code: [UP Lower Sub. (Spl) (Pre) 2010]
Code: [UPPCS (Mains) 2011] A B C D
(a) 3 2 1 4
A B C D
(b) 2 1 4 3
(a) 1 4 2 3 (c) 4 3 1 2
(b) 2 3 4 1 (d) 3 4 2 1
(c) 4 2 3 1
84. Which one of the following pairs is not
(d) 3 1 2 4 correctly matched?
81. Match list-I with list-II and select the [U.P.P.C.S. (Mains) 2010]
correct answer using the codes given below (a) Berber - Morocco
the lists: (b) Inuits - Canada
(c) Semangs - Indonesia
List-I List-II (d) Veddas - Sri Lanka
(Iron-Steel Centre) (Country)
85. The Ruhr basin is the famous industrial
A. Hamilton 1. China region of– [U.P.P.C.S (Pre) 2010]
B. Birmingham 2. Canada (a) China
C. Essen 3. UK (b) Germany
(c) Japan
D. Anshan 4. Germany
(d) United Kingdom
Code: [UPPCS (Pre) 2011]
86. Match list-I with list-II and select the
A B C D correct answer using the codes given below
(a) 4 3 1 2 the lists:
(b) 2 1 4 3
Minerals Important Centers
(c) 2 3 4 1
A. Copper 1. Butte
(d) 3 4 2 1
B. Diamond 2. Katanga
82. Match list-I with list-II and select the
C. Gold 3. Kimberley
correct answer using the codes given below
the lists: D. Silver 4. Witwatersrand
Code: [UPPCS (Mains) 2010]
List-I List-II
A B C D
A. Alupuram 1. Chhattisgarh (a) 2 4 3 1
B. Angul 2. Kerala (b) 2 3 4 1
C. Belgaum 3. Odisha (c) 1 3 2 4
(d) 3 1 4 2
D. Korba 4. Karnataka
Code: [UPPCS (Mains) 2010] 87. In which of the following districts in India
the largest ‘Mica Belt’ is found?
A B C D
[U.P. Lower Sub. (Pre) 2009]
(a) 4 2 3 1 (a) Balaghat and Chhindwada
(b) 2 3 4 1 (b) Udaipur, Ajmer and Alwar
(c) 1 3 4 2 (c) Hazaribagh, Gaya and Munger
(d) 2 1 3 4 (d) Salem and Dharampuri

PYQ Workbook 238


GENERAL GEOGRAPHY

88. Match list-I with list-II and select the 93. Heroin is obtained from:
correct answer using the codes given below [U.P.P.C.S. (Mains) 2007]
the lists: (a) Hemp
List-I List-II (b) Opium
(Tribe) (Country) (c) Tobacco
(d) Betelnut
A. Eskimo 1. Botswana
94. Which of the following countries are
B. Maasai 2. Saudi Arabia
included in the ‘Golden Crescent’?
C. Bedouins 3. Canada [U.P.P.C.S. (Mains) 2007]
D. Bushman 4. Kenya 1. Afghanistan
Code: [UPPCS (Pre) 2009] 2. Iran
A B C D 3. Iraq
(a) 1 2 3 4 4. Pakistan
(b) 4 3 2 1
(c) 3 4 2 1 Select the correct answer from the codes
(d) 2 4 1 3 given below:
(a) 1 and 2
89. Which of the following pairs is not correctly (b) 3 and 4
matched? [U.P. Lower Sub. (Pre) 2009]
(c) 1, 2 and 3
(a) Maori - New Zealand (d) 1, 2 and 4
(b) Maasai - Zambia
(c) Mayas - Guatemala 95. Which one of the following pairs is correctly
(d) Bushman - Kalahari matched? [UPPCS (Mains) 2007]
90. The Inuit people are not found in– Mineral Producer
[U.P.P.C.S. (Mains) 2009]
(a)Copper Arizona
(a) Alaska
(b) Greenland (b) Coal Karatzas
(c) Canada (c) Iron Ore Rasatnoora
(d) Sweden
(d)Petroleum Donbas
91. Match list-I with list-II and select the
correct answer using the codes given below 96. The most mineralized rock system of India
the lists: is- [U.P.P.C.S. (Pre) 2005]
(a) Cuddapah System
List-I List-II (b) Dharwar System
(Region) (Mineral) (c) Gondwana System
(d) Vindhyan System
A. Badam Pahar 1. Copper
B. Kodarma 2. Iron Ore 97. Match list-I with list-II and select the
correct answer using the codes given below
C. Mosabani 3. Petroleum
the lists:
D. Rava 4. Mica
List-I List-II
Code: [UPPCS (Pre) 2008]
(Oil Refinery) (States)
A B C D A. Haldia 1. Assam
(a) 4 3 1 2 B. Jamnagar 2. Kerala
(b) 2 4 3 1 C. Kochi 3. Gujarat
(c) 1 2 4 3
(d) 2 4 1 3 D. Numaligarh 4. West Bengal
Code: [UPPCS (Pre) 2005]
92. The largest producer of coconut in the
world is: [U.P.P.C.S. (Spl) (Mains) 2008] A B C D
(a) Brazil (a) 4 3 1 2
(b) India (b) 4 3 2 1
(c) Indonesia (c) 1 2 3 4
(d) Philippines (d) 3 4 2 1

239 PYQ Workbook


GENERAL GEOGRAPHY

98. Rourkela Steel Plant gets its supplies of iron A B C D


ore from- [U.P.P.C.S. (Mains) 2005] (a) 1 2 3 4
(a) Keonjhar (b) 4 3 2 1
(b) Dalli-Rajhara (c) 3 4 1 2
(c) Kamangundi (d) 2 3 1 4
(d) Mayurbhanj 102. A huge amount of sources found in the
Vindhya rocks are of- [U.P. Lower Sub.
99. Consider the following statements:
(Pre) 2004]
[U.P.P.C.S. (Mains) 2005] (a) Limestone
Assertion (A): Commercial Viticulture is (b) Basalt
specific to the Mediterranean Region of (c) Lignite
Europe. (d) Iron-ore
Reason (R): 85% of its grapes go to wine 103. Which one of the following is not correctly
making. matched? [U.P.P.C.S. (Spl) (Mains) 2004]
Select the correct answer from the code given (a) Bhilai : Madhya Pradesh
below: (b) Durgapur: West Bengal
(a) Both (A) and (R) are true, and (R) is the (c) Jamshedpur: Jharkhand
(d) Rourkela: Odisha
correct explanation of (A).
(b) Both (A) and (R) are true, but (R) is not 104. Which one of the following is not correctly
the correct explanation of (A). matched? [U.P.P.C.S. (Mains) 2004]
(c) (A) is true, but (R) is false. (a) Ramagundam – Fertilizer
(d) (A) is false, but (R) is true. (b) Chittaranjan – Locomotive
(c) Korba – Aluminium
100. Match list-I with list-II and select the (d) Pipri – Pesticides
correct answer using the codes given below
105. Which one of the following is not correctly
the lists:
matched? [U.P.P.C.S. (Spl) (Mains) 2004]
List-I List-II (a) Central Asia - Alpine
(Centre) (Industry) (b) Congo basin - Negrito
(c) Kalahari - Bushmen
A. Pittsburg 1. Ship Building
(d) Scandinavia - Nordic
B. Shanghai 2. Iron & Steel
106. Which one of the following is not correctly
C. Dundee 3. Cotton Textile matched? [U.P.P.C.S. (Mains) 2004]
D. Leningrad 4. Jute Textile (a) Donbas Basin - Coal
Code: [UPPCS (Mains) 2005, UPPCS (b) Mesabi Range - Copper
(Pre) 2005] (c) Mosul - Petroleum
(d) Transvaal - Gold
A B C D
(a) 1 2 3 4 107. Match list-I with list-II and select the
(b) 4 3 2 1 correct answer using the codes given below
(c) 2 3 4 1 the lists:
(d) 4 3 1 2 List-I List-II
101. Match list-I with list-II and select the (Crop) (Production zone)
correct answer using the codes given below A. Coffee 1. Prairies Plains
the lists: B. Jute 2. Yangtze Plains
List-I List-II C. Rice 3. Sao Paulo Plateau
(Centre) (Industry) D. Wheat 4. Ganges Delta
A. Anshan 1. Cutlery Code: [UPPCS (Mains) 2003]
B. Glasgow 2. Paper A B C D
C. Ottawa 3. Ship building (a) 3 1 4 2
(b) 2 4 1 3
D. Sheffield 4. Iron and Steel (c) 3 4 2 1
Code: [UPPCS (Mains) 2005] (d) 4 3 2 1

PYQ Workbook 240


GENERAL GEOGRAPHY

108. Match list-I with list-II and select the 112. Which of the following is not correctly
correct answer using the codes given below matched? [U.P.P.C.S. (Pre) 1998]
the lists: (a) Detroit - Automobiles
(b) Magnitogorsk - Iron and Steel
List-I List-II (c) Johannesburg - Gold Mining
A. Copper 1. Kochi (d) Birmingham - Shipbuilding
B. Gas Plant 2. Auraiya 113. Bauxite is an ore of-
C. Aluminium 3. Korba [U.P.P.C.S. (Pre) 1997]
(a) Lead
D. Petroleum 4. Khetri (b) Aluminium
Code: [UPPCS (Pre) 2002] (c) Zinc
A B C D (d) Copper
(a) 4 2 3 1 114. The largest producer of Silver in the world
(b) 1 2 3 4 is: [U.P.P.C.S. (Pre) 1993]
(c) 4 3 2 1 (a) Chile
(d) 3 4 1 2 (b) China
109. Match list-I with list-II and select the (c) Mexico
correct answer using the codes given below (d) Peru
the lists: [UP Lower (Sub.) (Pre) 2002] 115. ‘Kalgoorlie’ situated in Australia is famous
List-I List-II for– [U.P.P.C.S. (Pre) 1991]
(a) Gold production
(Cities) (Industry) (b) Good Climate
A. Anshan 1. Cotton Textile (c) Education
B. Detroit 2. Cigar (d) Poultry
C. Hawana 3. Automobile 116. Which fruit is not Tropical?
[U.P.P.C.S. (Pre) 1990]
D. Nagoya 4. Iron and Steel (a) Walnut
Code: (b) Coconut
A B C D (c) Apple
(a) 1 2 3 4 (d) Cashewnut
(b) 4 3 2 1 117. Combustion of underground coal occurs in
(c) 3 4 1 2 the State of: [67th BPSC (Pre) 2022]
(d) 2 4 1 3 (a) West Bengal
110. Which of the following is correctly (b) Bihar
matched? [U.P.P.C.S. (Pre) 2001] (c) Jharkhand
(d) Odisha
(a) Eskimo - Canada (e) None of the above/More than one of the
(b) Oraon - Japan above
(c) Lapps - India
(d) Gonds - Africa 118. Which State of India is the largest producer
of copper?
111. Match list-I with list-II and select the [66th B.P.S.C. (Pre) (Re-Exam) 2020]
correct answer using the codes given below
(a) Madhya Pradesh
the lists: [UPPCS (Pre) 1999] (b) Rajasthan
List-I List-II (c) Jharkhand
(d) Chhattisgarh
A. Iron Ore 1. Port Radium (e) None of the above/More than one of the
B. Petroleum 2. Binghama above
C. Copper 3. Baku 119. Among the following countries, which one
D. Uranium 4. Mesabi is the largest producer of saffron in the
world? [65th B.P.S.C. (Pre) 2019]
Code:
(a) Spain
A B C D (b) Greece
(a) 4 3 2 1 (c) New Zealand
(b) 3 4 1 2 (d) Iran
(c) 1 2 3 4 (e) None of the above/More than one of the
(d) 1 3 2 4 above

241 PYQ Workbook


GENERAL GEOGRAPHY

120. Kaimur Plateau is famous for: 126. Most attractive, warmest and lightest wool
[63rd BPSC (Pre) 2017] of the world ‘Shahtoosh’ is produced in–
(a) Copper [39th B.P.S.C. (Pre) 1994]
(b) Limestone (a) Nepal
(c) Lithium (b) Uzbekistan
(d) Bauxite (c) China
(e) None of the above/More than one of the (d) Bangladesh
above
127. Tin is found- [39th B.P.S.C. (Pre) 1994]
121. ‘Rust Bowl’ of the USA is associated with
which one of the following regions? (a) in Placer deposits
[63rd B.P.C.S. (Pre) 2017] (b) in Metamorphic rocks
(c) in little silica Igneous rocks
(a) Great Lakes region
(d) in all these
(b) Alabama region
(c) California region 128. Radium is extracted from the mines of–
(d) Pittsburg region [39th B.P.S.C. (Pre) 1994]
(e) None of the above/More than one of the (a) Limestone
above
(b) Pitchblende
122. Match list-I with list-II and select the (c) Rutile
correct answer using the codes given below (d) Hematite
the lists:
129. Consider the following statements about
List-I List-II the seaports in India:
(Place) (Industry) 1. There are 4 major and 25 minor seaports
A. Bengaluru 1. Iron & Steel in India.
2. The major ports are under the
B. Korba 2. Copper
administrative control of the Ministry of
C. Jamshedpur 3. Aircraft Ports, Shipping and Waterways.
D. Malajkhand 4. Aluminium 3. The minor ports are under the jurisdiction
Code: [60th to 62nd BPSC (Pre) 2016] of respective State Maritime Board/State
A B C D Government.
(a) 1 2 3 4 Which of the statements given above is/are
(b) 2 1 4 3 correct? [CAPF 2022]
(c) 4 3 2 1 (a) 1, 2 and 3
(d) 3 4 1 2 (b) 3 only
(e) None of the above/More than one of the (c) 1 and 2 only
above (d) 2 and 3 only
123. The oldest oil field in India is- 130. Which of the following main types of iron
[56th to 59th B.P.S.C. (Pre) 2015] ores are found in India?
(a) Bombay High, Maharashtra 1. Haematite
(b) Ankleshwar, Gujarat 2. Magnetite
(c) Navgam, Gujarat
(d) Digboi, Assam 3. Limonite
4. Siderite
124. In which of the following states of India,
silver is not found? Select the correct answer using the codes
[43rd B.P.S.C. (Pre) 1999] given below. [CAPF 2021]
(a) Odisha (a) 1 and 2
(b) Andhra Pradesh (b) 1, 2 and 4
(c) Gujarat (c) 2 and 4
(d) Jharkhand (d) 1 and 3
125. Johannesburg is famous for– 131. Which one of the following is the only
[42nd B.P.S.C. (Pre) 1997] riverine major port in India? [CAPF 2021]
(a) Gold mining (a) Mormugao
(b) Tin mining (b) Deendayal
(c) Mica mining (c) Kolkata
(d) Iron-ore mining (d) Paradip

PYQ Workbook 242


GENERAL GEOGRAPHY

132. Natural Population Change is calculated by 137. Which of the following statements related
[CAPF 2020] to tribes in India are correct?
(a) subtracting the death rate from the birth 1. Tharu tribes are found in Uttar Pradesh
rate and Uttarkhand.
(b) multiplying the death rate with the birth 2. Irula, Chanchu and Sumali tribes are
rate found in Kerala.
(c) adding the death rate with the birth rate 3. Garasia tribes are found in Goa.
(d) subtracting the birth rate from the death
4. Gaddi tribes are nomadic herders of
rate
Jammu & Kashmir and Himachal Pradesh
133. 2011 Census has divided the working Select the correct answer using the code given
population of India into four major below: [CAPF 2017]
categories. Which one of the following is
(a) 1 and 3
NOT one of them? [CAPF 2020]
(b) 2 and 4
(a) Cultivators (c) 1, 2 and 4
(b) Domestic workers
(d) 1, 2, 3 and 4
(c) Household industrial workers
(d) Agricultural labourers 138. Which of the following statements are
134. Which one of the following is the correct correct? [CAPF 2016]
descending order of Indian States in terms 1. Kolkata port is the only riverine major
of their density of population (as per 2011 port of India.
Census)? [CAPF 2018] 2. The port of Cochin is located on
(a) Bihar-Kerala-Uttar Pradesh-West Bengal Willington Island.
(b) West Bengal-Bihar-Kerala-Uttar Pradesh 3. Maharashtra has three major ports.
(c) Bihar-West Bengal-Kerala-Uttar Pradesh 4. Mundra port is India’s major port in
(d) Kerala-West Bengal-Bihar-Uttar Pradesh private sector.
135. Match List I with List II and select the Select the correct answer using the code given
correct answer using the code given below below.
the Lists: (a) 1 and 2 only
List I (Mineral) List II (Mine) (b) 3 and 4 only
(c) 2, 3 and 4
A. Zinc 1. Amjhore (d) 1, 2 and 4
B. Gold 2. Sukinda 139. Which one among the following States is
C. Chromite 3. Zawar the leading producer of salt in India?
D. Pyrite 4. Hutti [CAPF 2015]
Codes: [CAPF 2017] (a) Rajasthan
A B C D (b) Gujarat
(a) 1 2 4 3 (c) Tamil Nadu
(b) 3 2 4 1 (d) Andhra Pradesh
(c) 3 4 2 1 140. According to the Census 2011, which of the
(d) 1 4 2 3 following statements is/are correct?
136. Which of the following are the necessary [CDS-2023(I)]
conditions for the growth of coral reefs? 1. A person who can only read but cannot
1. Photic conditions write, is not literate.
2. Clean and sediment-free water 2. A person aged seven years and above,
3. SEA salinity of 6% who can both read and write with
4. Tropical Sea water with temperature of understanding in any language is treated
200C 210 C as literate.
Select the correct answer using the code given Select the correct answer using the code given
below: [CAPF 2017] below.
(a) 1, 2 and 4 (a) 1 only
(b) 2 and 4 (b) 2 only
(c) 1 and 3 (c) Both 1 and 2
(d) 1, 2, 3 and 4 (d) Neither 1 nor 2

243 PYQ Workbook


GENERAL GEOGRAPHY

141. Consider the following statements and 146. Match List I with List II and select the
identify the correct answer using the code correct answer using the codes given below
given below: the lists.
1. In a thermal power station, fuels such
as oil, coal or natural gas are used to List I List II
generate electricity. (Ethnic Territorial (Related
2. Fuels are burned to heat water and turn it Segment) Occupational
into steam, which goes through a turbine, Pattern)
which spins and turns, generating A. Maruta Makkal 1. Pastoralists
electricity.
Code: [CDS-2023(I)] B. Kuravan Makkal 2. Fishing people
(a) Statement 1 is correct but statement 2 is C. Mullai Makkal 3. Ploughmen
not correct. D. Neytal Makkal 4. Hill people
(b) Statement 2 is correct but statement 1 is Codes [CDS 2021 (I)]
not correct.
(c) Both the statements 1 and 2 are correct A B C D
and statement 2 explains statement 1. (a) 3 1 4 2
(d) Both the statements 1 and 2 are correct but (b) 2 1 4 3
statement 2 does not explain statement 1. (c) 3 4 1 2
(d) 2 4 1 3
142. Which of the following statements is/are
correct? [CDS-2023(I)] 147. Match List I with List II and select the
1. Visakhapatnam Port is a land- locked correct answer using the codes given below
harbour. the lists.
2. Deendayal Port is a tidal port. List I (City) List II (Product)
Select the correct answer using the code given A. Detroit (USA) 1. Motarcar
below.
(a) 1 only B. Antwerp 2. D i a m o n d
(b) 2 only (Belgium) Cutting
(c) Both 1 and 2 C. Tokyo (Japan) 3. Steel
(d) Neither 1 nor 2 D. Harbin (China) 4. Ship building
143. Which of the following variable(s) is/are Codes [CDS 2019 (I)]
displayed in population pyramids? A B C D
[CDS-2022(I)] (a) 3 4 2 1
1. Age-groups (b) 3 2 4 1
2. Sex indicators (male and female) (c) 1 4 2 3
Select the correct answer using the code given (d) 1 2 4 3
below:
148. Match List I with List II and select the
(a) 1 only correct answer using the codes given below
(b) 2 only the lists
(c) Both 1 and 2
(d) Neither 1 nor 2 List I (Classification List II (Example)
144. Which one of the following is an exhaustible of town)
but renewable natural resource? A. Industrial Town 1. Vishakhapatam
[CDS-2022(I)] B. Transport Town 2. Bhilai
(a) Solar energy
C. Mining Town 3. Singrauli
(b) Water in usable condition
(c) Soil D. Garrison 4. Ambala
(d) Landscape in its natural condition Cantonment Town
145. Which one of the following is the largest Code: [CDS 2018 (II)]
linguistic group of India? [CDS 2021 (I)] A B C D
(a) Sino-Tibetan (a) 2 1 3 4
(b) Austric (b) 2 3 1 4
(c) Indo-Aryan (c) 4 3 1 2
(d) Dravidian (d) 4 1 3 2

PYQ Workbook 244


GENERAL GEOGRAPHY

149. Which one of the following Indian States 154. Which one of the following statements
has the highest percentage of Scheduled concerning research centres in India is not
Tribe population to its total population? correct? [CDS 2016 (II)]
[CDS 2018 (I)] (a) Rajiv Gandhi Centre for Biotechnology is
(a) Mizoram located at Kolkata
(b) Nagaland (b) Central Arid Zone Research Institute is
(c) Meghalaya located at Jodhpur
(d) Arunachal Pradesh (c) Centre for Social Forestry and Eco-
Rehabilitation is located at Allahabad
150. Which one of the following tribal groups is (d) Institute of Forest Productivity is located
dominantly found in the ‘blue mountains’? at Ranchi
[CDS 2017 (II)]
155. With regard to water transportation in
(a) Lambadas India, which of the following statements are
(b) Gonds correct? [CDS 2016 (II)]
(c) Jarawas 1. Headquarters of Central Inland Water
(d) Todas Transport Corporation is located at Delhi.
151. Match List I with List II and select the 2. Headquarters of Inland Waterways
correct answer using the codes given below Authority of India is located at Noida.
the lists 3. National Inland Navigation Institute is
located at Kolkata.
List I (Industrial List II (Country)
Region) 4. First National Inland Waterway was
between Haldia and Allahabad.
A. Leipzig 1. USA
Select the correct answer using the codes
B. Detroit 2. UK given below.
C. Lorraine 3. France (a) 1 and 2
D. Cumberland 4. Germany (b) 2 and 3
(c) 2 and 4
Code: [CDS 2017 (II)]
(d) 1, 2 and 4
A B C D
(a) 2 1 3 4 156. Consider the following statements:
(b) 4 3 1 2 [CDS 2016 (I)]
(c) 4 1 3 2 1. Most of the coal and the ferrous group of
(d) 2 3 1 4 minerals in India occur in the peninsular
India, South of the Vindhyas.
152. Which one of the following statements with 2. The peninsular India once formed part
regard to the Indian Railways is not correct? of the super-continent which included
[CDS 2017 (I)] Australia, Antarctica, Africa and South
(a) The Indian rail network have been America.
developed throughout the Konkan coast. Which of the statement(s) given above is/are
(b) There is very low density of railway lines correct?
in the Himalayan region. (a) Only 1
(c) The North Indian Plain has a dense (b) Only 2
network of railway. (c) Both 1 and 2
(d) At present, India has the largest railway (d) Neither 1 nor 2
network in the world.
157. Which of the following statement(s)
153. Which one of the following about sugar regarding chemical industry in India is/are
industry of Peninsular India is not correct? correct? [CDS 2016 (I)]
[CDS 2017 (I)] 1. Chemical industry is one of the oldest
(a) High yield per hectare of sugarcane. industries in India.
(b) Higher sucrose content. 2. Dyestuff sector is one of the important
(c) Long crushing season. segments of chemical industry.
(d) Most of the mills in the Peninsula are 3. Textile industry accounts for the largest
located mainly along the East coast. consumption of dyestuffs.

245 PYQ Workbook


GENERAL GEOGRAPHY

Select the correct answer using the codes 162. Which of the following is/are the stage(s) of
given below: demographic transition? [CDS 2014 (I)]
(a) Only 1 1. High death rate and birthrate, low growth
(b) 2 and 3 rate.
(c) 1 and 3 2. Rapid decline in death rate, continued
(d) 1, 2 and 3 low birthrate, very low growth rate.
158. Match the following. [CDS 2016 (II)] 3. Rapid decline in birthrate, continued
decline in death rate.
List I (Places) List II (Industries) 4. Low death rate and birthrate, low growth
A. Bongaigaon 1. Paper rate.
B. Koraput 2. Machine Select the correct answer using the codes
C. Pinjore 3. Aircraft given below:
(a) Only 1
D. Sirpur 4. Petrochemicals
(b) 1, 2 and 3
Codes (c) 3 and 4
A B C D (d) 1 and 4
(a) 4 2 3 1 163. Which one of the following is a land-locked
(b) 4 3 2 1 harbour? [CDS 2014 (I)]
(c) 1 2 3 4
(d) 1 3 2 4 (a) Vishakhapatnam
(b) Ennore
159. Match the following: (c) Mumbai
List I (Steel mill) List II (State) (d) Haldia
A. Kalinganagar 1. West Bengal 164. Choose the correct option representing
the following pipelines Transports in
B. Vijaynagar 2. Tamil Nadu decreasing order of their lengths?
C. Salem 3. Odisha [Chhattisgarh P.C.S. (Pre) 2020]
D. Durgapur 4. Karnataka 1. Salaya-Koyali-Mathura pipeline
Codes [CDS 2015 (II)] 2. Hajira-Bijapur-Jagdishpur gas pipeline
A B C D 3. Jamnagar-Loni (LPG) pipeline
(a) 1 4 2 3 4. Kandala-Bhatinda pipeline
(b) 1 2 4 3 Code:
(c) 3 4 2 1 (a) 1 > 2 > 3 > 4
(d) 3 2 4 1 (b) 2 > 4 > 1 > 3
160. Which of the following is/are the reason (s) (c) 2 > 4 > 3 > 1
behind Gujarat being the leading producer (d) 2 > 3 > 4 > 1
of Salt in India? [CDS 2015 (I)] 165. Which one of the following river valleys is
1. The long length of coastline. most important in respect of coal reserves
2. Long duration of hot and dry conditions. of the Gondwana period? [Chhattisgarh
3. Presence of gulf areas. P.C.S. (Pre) 2017]
Select the correct answer using the codes (a) The Son River valley
given below: (b) Mahanadi river valley
(a) 1 and 2 (c) Damodar river valley
(b) Only 2 (d) Godavari river valley
(c) 1, 2 and 3 (e) None of the above
(d) 1 and 3 166. Out of the following types of iron ore, a
161. Arrange the locations of four oil refineries reserve of which type is largest in India?
of India from West to an East. [Chhattisgarh P.C.S. (Pre) 2017]
[CDS 2014 (I)] (a) Hematite
(a) Koyali, Kochi, Panipat, Mathura (b) Magnetite
(b) Kochi, Koyali, Panipat, Mathura (c) Siderite
(c) Koyali, Panipat, Kochi, Mathura (d) Limonite
(d) Koyali, Panipat, Mathura, Kochi (e) None of these

PYQ Workbook 246


GENERAL GEOGRAPHY

167. The first cotton mill of India was established C. Kolar 3. Bauxite
at- [Jharkhand P.C.S. (Pre) 2013]
D. Mosabani 4. Zinc and Lead
(a) Bombay
(b) Ahmedabad Code: [MPPCS (Pre) 2019]
(c) Baroda A B C D
(d) Calcutta (a) 1 2 3 4
(b) 1 4 3 2
168. ‘Transhumance’ refers to– (c) 3 4 1 2
[Jharkhand P.C.S. (Pre) 2013] (d) 3 2 4 1
(a) Migration of animals in ranches 172. Which of the following is different from the
(b) Human migration in search of job remaining three– [M.P.P.C.S. (Pre) 2008]
(c) Seasonal movement of people and their (a) Hematite
herds from the valley to the mountain (b) Magnetite
and vice versa (c) Limonite
(d) Migration of nomads (d) Bauxite
169. Match list-I with list-II and select the 173. Which of the following Indian states is the
correct answer using the codes given below biggest producer of Mica?
the lists: [MPPCS (Pre) 2021] [M.P.P.C.S. (Pre) 1990]
List-I List-II (a) Andhra Pradesh
(b) Bihar
(Economic Activity/ (Country)
(c) Jharkhand
Agricultural Region) (d) Rajasthan
A. Commercial dairy 1. Argentina
farming 174. The Wood of which tree is used for catechu
production? [M.P.P.C.S. (Pre) 1990]
B. Commercial grain 2. France (a) Saal
farming (b) Khair
C. Commercial 3. Denmark (c) Babool
plantation farming (d) Saaja
D. Commercial fruits 4. Malaysia 175. Match list-I with list-II and select the
production correct answer using the codes given below
Code: the lists:
A B C D List-I List-II
(a) 3 1 4 2 (Industrial Regions) (Country)
(b) 1 2 3 4
(c) 4 3 2 1 A. Lancashire Region 1. United States of
(d) 2 4 1 3 America
B. Ruhr Region 2. Germany
170. ‘Marquette range’ in United States of
America is famous for which mineral? C. Keihin Region 3. United Kingdom
[MPPCS (Pre) 2021] D. Southern 4. Japan
(a) Uranium Appalachian
(b) Copper Code: [RAS/RTS (Pre) 2016]
(c) Zinc A B C D
(d) Iron ore (a) 3 2 1 4
171. Match list-I with list-II and select the (b) 3 4 2 1
correct answer using the codes given below (c) 1 2 3 4
(d) 3 2 4 1
the lists:
176. The Lorraine Region of France is famous
List-I (Mining Area) List-II for: [RAS/RTS (Pre) (Re-Exam) 2013]
(Mineral Wealth) (a) Chemical Industry
A. Kalahandi 1. Gold (b) Iron and Steel Industry
(c) Cotton and Textile Industry
B. Zawar 2. Copper (d) Paper Industry

247 PYQ Workbook


GENERAL GEOGRAPHY

177. Which group of minerals represents non- 183. African country Zambia has prolific copper
ferrous metals? reserves, but economic progress has not
[R.A.S./R.T.S. (Re. Exam) (Pre) 2013] been made in the country, because of–
(a) Nickel, Zinc, Copper, Aluminum [R.A.S./R.T.S. (Pre) 1992]
(b) Nickel, Aluminum, Iron ore, Zinc (a) Dense Forests
(c) Copper, Pig iron, Nickel, Steel (b) Lack of means of transportation
(d) Nickel, Carbon Steel, Aluminum, Zinc (c) No seaport
178. Which wood raw material is used for making (d) Less population
paper pulp? [R.A.S./R.T.S. (Pre) 2013] 184. Kodarma is famous for which mineral?
(a) Peparin [Uttarakhand PCS (Pre) 2021]
(b) Poplar (a) Iron-Ore
(c) Bagasse (b) Mica
(d) Rice straw (c) Gypsum
179. Which continent is known as ‘the Cradle of (d) Bauxite
Mankind’? [R.A.S./R.T.S. (Pre) 2012] 185. Pittsburg of India is:
(a) Asia [Uttarakhand PCS (Pre) 2021]
(b) Africa
(c) Europe (a) Bhagalpur
(d) South America (b) Varanasi
(c) Sindri
180. The area which is the ‘storehouse of (d) Jamshedpur
minerals’ in South America is:
186. Marble is- [Uttarakhand P.C.S. (Pre) 2006]
[R.A.S./R.T.S. (Pre) 2010]
(a) Brazilian Plateau (a) Re-crystallized limestone
(b) Bolivian Plateau (b) An igneous rock
(c) Patagonia Plateau (c) Sandstone
(d) Orinoco basin (d) Formed from changes of organic matters
to inorganic one
181. The largest producer of bananas in the
world is: [R.A.S./R.T.S.(Pre) 2003] 187. Coal is an example of–
(a) Columbia [Uttarakhand P.C.S. (Pre) 2006]
(b) Zimbabwe (a) Igneous rock
(c) Malaysia (b) Metamorphic rock
(d) India (c) Sedimentary rock
(d) All of the above
182. Which pair is correctly matched?
[R.A.S./R.T.S. (Pre) 1992] 188. Coal, Crude Oil and Natural gas are called–
(a) Assam - Barauni oil refinery [Uttarakhand P.C.S. (Mains) 2002]
(b) Andhra Pradesh - Visakhapatnam oil (a) Crude Fuel
refinery (b) Traditional Fuel
(c) Maharashtra - Koyna oil refinery (c) Natural Fuel
(d) Gujarat - Mathura oil refinery (d) Fossil Fuel

PYQ Workbook 248


GENERAL GEOGRAPHY

SOLUTIONS
and rutile are processed to obtain titanium dioxide (TiO2),
6.1. UPSC Previous Years’ Questions
which is then further refined to produce metallic titanium.
1. Solution: (c) Therefore, ilmenite and rutile, found abundantly in certain
coastal tracts of India, are valuable sources of titanium.
Exp) Option c is the correct answer.
Statement I is correct. Both conventional (thermal, 4. Solution: (c)
nuclear, and hydro) and renewable (wind, solar, biomass, Exp) Option c is the correct answer
etc.) energy sources are used to produce electricity in
The DRC (The Democratic Republic of the Congo) is the
India. However, the majority of electricity is produced by
world’s largest producer of cobalt, accounting for about
coal-fired thermal power plants, which provide about 58%
three-fourths (75%) of the global production. The country
of the total electricity. In the year 2021–2022, approximately
has been the top producer of the metal for some time, and
3.15 per cent of the nation’s electricity was generated using
reported output of 130,000 MT in 2022. Cobalt is a crucial
nuclear energy. component in the manufacture of batteries for electric
Statement II is incorrect. Uranium enrichment of U 235 vehicles, as well as for various electronic devices such as
isotope 3 to 5 percent is required for electricity generation. smartphones and laptops.
Research reactors use uranium that has been highly enriched There are several reasons why the DRC has become the
to a concentration of at least 20%. At least 90% enriched primary source of cobalt production:
Uranium is considered as weapons grade uranium.
• Abundance of Cobalt Reserves: The DRC is home to vast
2. Solution: (b) reserves of cobalt. It is estimated to have approximately
Exp) Option b is the correct answer. 3.4 million metric tons of cobalt reserves, which is the
largest in the world. This abundance of cobalt resources
Pair 1 is correct: Kamarajar Port, located on the Coromandel
has made the DRC a significant player in the global
Coast about 24 km north of Chennai Port, Chennai, it is
cobalt market.
the 12th major port of India, and the first port in India
which is a public company. The Kamarajar Port is the only • Artisanal Mining: A significant portion of cobalt
corporatized major port and is registered as a company. production in the DRC comes from artisanal and small-
Today, the landlord port is the dominant port model in scale mining operations. These operations involve
larger and medium sized ports. individual miners or small groups extracting cobalt
using basic tools and equipment. However, due to the
Pair 2 is correct: Mundra port is the largest private port in
informal nature of this mining sector, it can be associated
India. The port of Mundra is located on the north shores of
with challenges such as poor working conditions,
the Gulf of Kutch near Mundra, Kutch district, in the state
environmental concerns, and potential human rights
of Gujarat. Mundra is a major hub for containers and bulk
issues.
cargo. It is run by Adani Ports and SEZ Limited (APSEZ) and
began operations in 2001. Mundra Airport is undergoing • Industrial Mining Operations: Alongside artisanal
mining, the DRC also has large-scale industrial mining
major developments to convert it into an international
operations run by multinational mining companies. These
airport for air cargo.
operations use modern machinery and technologies to
Pair 3 is incorrect: Jawaharlal Nehru Port Trust – Nhava extract cobalt from mines. The industrial mining sector
Sheva (JNPT) is the largest container port in India and one plays a crucial role in meeting the global demand for
of the most essential subcontinents harbours on the Western cobalt.
coast.
5. Solution: (d)
3. Solution: (d)
Exp) Option d is the correct answer
Exp) Option d is the correct answer
Statement 1 is incorrect: The East-West Corridor is
Ilmenite (FeTiO3) and rutile (TiO2) are both minerals a part of the National Highways Development Project
that contain high concentrations of titanium. Titanium (NHDP) in India, which aims to connect Silchar in Assam
is a strong and lightweight metal known for its excellent to Porbandar in Gujarat. It does not connect Dibrugarh
corrosion resistance and high strength-to-weight ratio. It in Assam and Surat in Gujarat. East-West Corridor
is widely used in various industries, including aerospace, is a 3,300 kilometres (2,100 mi) corridor via NH 27
automotive, medical, and construction. (Porbandar–Rajkot-Samakhiali–Radhanpur–Kota–Jhansi–
In India, certain coastal tracts are known for their significant Kanpu r- Lu ck n ow – Ayo d hy a - G or a k hpu r- Mu z af f ar pu r–
ilmenite and rutile deposits. These minerals are commonly Darbhanga-Supaul-Purnia– Dalkhola-Kishanganj, Islampur–
found in beach sand deposits and are extracted through Sonapur- Ghoshpukur-Jalpaiguri Alipurduar- Bongaigaon
beach sand mining operations. The extracted ilmenite - Nalbari Bijni–Guwahati–Nagaon–Dabaka–Silchar).

249 PYQ Workbook


GENERAL GEOGRAPHY

Statement 2 is incorrect: The Trilateral Highway is Monazite usually occurs in small, isolated crystals or grains
a connectivity project that aims to connect Moreh in that are resistant to weathering and become concentrated in
Manipur, India, with Mae Sot in Thailand via Myanmar. soils.
It is an important infrastructure project to enhance trade, Statement 1 and 2 are correct. Monazite is a major source of
connectivity, and people-to-people ties among the three rare earths and it contains thorium.
countries.
It yields a number of rare-earth elements, such as
Statement 3 is incorrect: The Bangladesh-China-India- neodymium and praseodymium. Both of these are in
Myanmar Economic Corridor (BCIM) is a proposed demand internationally for making high-performance rare-
economic corridor that aims to connect Kunming in China earth magnets (components of power wind turbines, electric
with Kolkata in India, passing through Myanmar and vehicles and robotics)
Bangladesh. Varanasi in Uttar Pradesh is not directly
Statement 3 is incorrect. Though monazite is found in
connected to this corridor.
most coastal areas of India. It is questionable to be found
Thus none of the statements are correct. along entire coast.
6. Solution: (c) The main mines are found along the coasts of southern India
Exp) Option c is the correct answer in Kerala, Tamil Nadu and in Orissa.
Statement 1 is correct: It is true that Switzerland is the Statement 4 is correct. Export of monazite without a
leading exporter of gold in the world. In 2021, Switzerland license from AERB is a violation of the Atomic Energy
exported $86.7B in Gold, making it the 1st largest exporter (Radiation Protection) Rules 2004. Indian Rare Earths
of Gold in the world. The main destinations of Gold exports Limited (IREL), a wholly owned Public Sector Undertaking
from Switzerland are: India ($29.3B), China ($16B), United of the Government of India (GOI) under DAE, is the only
States ($8.13B), Germany ($5.8B), and Hong Kong ($4.67B). entity which has been permitted to produce and process
monazite, and handle it for domestic use as well as for export.
Statement 2 is incorrect: Switzerland does not have the
second largest gold reserves in the world, rather it has the 9. Solution: (b)
seventh largest gold reserve in the world. The top three
Exp) Option b is the correct answer.
countries with largest gold reserves in the world are the
United States of America (USA) followed by Germany, Statement 1 is incorrect: Rajasthan tops the list of solar
Italy. park installations in the country followed by Karnataka and
then Andhra Pradesh. Bhadla Solar Park in Rajasthan, with
7. Solution: (d) a capacity of 2245 MW, is the world’s largest solar park.
Exp) Option d is the correct answer. Statement 2 is correct: Cochin International Airport, India’s
Statement 1 is incorrect- The Mundra Thermal Power Plant first airport built under a public-private-partnership (PPP)
employs a closed-cycle induced draft circulating cooling model, becomes the first airport in the world that operates
water system that utilizes seawater. Seawater is drawn from completely on solar power. This plant is the first Megawatt
the Gulf of Kutch through robust glass reinforced pipes of scale installation of a Solar PV system in the State of Kerala
significant diameter. In addition, purified seawater from a Statement 3 is incorrect: Andhra Pradesh is now home to
reverse osmosis plant is utilized by various supplementary India’s largest floating solar power plant. State-run NTPC
systems. started operations at India’s largest floating solar PV project
Statement 2 is incorrect- According to recent research by at its Simhadri thermalstation in Visakhapatnam.
WRI (World Resources Institute), 40 percent of India’s
10. Solution: (d)
thermal power plants are situated in regions experiencing
significant water stress. This poses a challenge as these Exp) Option d is the correct answer.
plants rely on water for cooling purposes. The scarcity of In India, the minerals are classified as minor minerals and
water is already causing disruptions in electricity generation major minerals.
in these areas, with 14 out of India’s 20 largest thermal Major minerals are those specified in the first schedule
utilities having experienced at least one shutdown between appended in the Mines and Minerals (Development
2013 and 2016 due to water shortages. These shutdowns have and Regulation) Act, 1957 (MMDR Act 1957) and the
resulted in significant financial losses, amounting to $1.4 common major minerals are Lignite, Chromite, Kyanite,
billion for the affected companies. Sillimanite, Coal, Uranium, iron ore, gold etc. It may be
Statement 3 is incorrect- India has a total of 269 Thermal noted that there is no official definition for “major minerals”
Power Plants, with 138 of them being owned by the public in the MMDR Act. Hence, whatever is not declared as a
sector and the remaining 131 owned by the private sector. “minor mineral” may be treated as the major mineral.
The list of minerals to be notified as minor minerals
8. Solution: (b)
include: Agate, Ball Clay, Barytes, Calcareous Sand, Calcite,
Exp) Option b is the correct answer. Chalk, China Clay, Clay (Others), Diaspore, Dolomite,
Monazite is a brown crystalline mineral consisting of Felsite, Fireclay, Fuschite Quartzite, Gypsum, Jasper, Kaolin,
cerium, lanthanum, other rare earth elements, and thorium. Laterite, Limekankar, Mica, Quartzite, Sand (Others), Shale,

PYQ Workbook 250


GENERAL GEOGRAPHY

Silica Sand, Slate, and Steatite/Talc/Soapstone, Barytes, iron is coming from Hematite reserves, Magnetite reserves
Bentonite, Corundum and Sapphire, Dunite & Pyroxinite, are not being exploited as they are located in fragile zones
Felspar, Fuller’s Earth, Granite and Selenite, Ball Clay, like Western Ghats. Around 96% of hematite resources are
Other Clays and Shale, Marble, Ochre and Other Calcareous confined to Odisha, Jharkhand, Chhattisgarh, Karnataka and
Materials, Pyrophyllite, Quartz & Other Silica Minerals, Goa. The remaining resources are spread in Maharashtra,
Slate, Sandstone & Other Dimension Stones, Talc, Soapstone Madhya Pradesh, Rajasthan, Assam etc. Both Hematite and
and Steatite. Magnetite ores occur in Rajasthan.

11. Solution: (a) 14. Solution: (d)


Exp) Option a is the correct answer. Exp) Option d is the correct answer.
Cotton is the crop of tropical and sub-tropical areas and Oil Shales are usually fine-grained sedimentary rocks
requires uniformly high temperature varying between containing relatively large amounts of organic matter from
21°C and 30°C. The growth of cotton is retarded when the which significant quantities of shale oil and combustible
temperature falls below 20°C. Frost is enemy number one of gas can be extracted by destructive distillation.
the cotton plant and it is grown in areas having at least 210 Shale gas can emerge as an important new source of energy in
frost free days in a year. The modest requirement of water the country. India has several Shale Formations which seem
can be met by an average annual rainfall of 50- 100 cm. to hold shale gas. The Shale Gas Formations are spread over
However, it is successfully grown in areas of lesser rainfall several sedimentary basins, such as, Gangetic plain, Gujarat,
with the help of irrigation. Rajasthan, Andhra Pradesh and other coastal areas in the
country including hydrocarbon-bearing ones - Cambay,
12. Solution: (a)
Assam Arkan & Damodar Basins have large shale deposits.
Exp) Option a is the correct answer. Various developmental activities are going on in Gandhar
Statement 1 is correct. As defined under the Mines and area of Cambay Basin, Krishna-Godavari Basin, Cauvery
Minerals (Development and Regulation) Act, 1957 (MMDR Basin and Assam & Assam Arkan Basin.
Act) Sand is a minor mineral. ONGC is presently carrying out shale gas and oil exploration
Statement 2 is incorrect. The administration of the mining in four onland basins, namely, Cambay, KG, Cauvery, Assam
of minor minerals is a subject that is, completely within and Assam-Arakan. A total of twelve exploratory locations
the domain of the respective State Governments. This act for shale gas and oil are available for drilling in 2016-17 and
empowers State Government to grant mining leases of minor subsequent year. In addition, coring and other data collection
minerals. Also, it empowers State governments to frame are planned in suitable exploratory wells in identified blocks.
rules for regulating the grant of mineral concessions in ONGC has drilled 18 wells so far and one well is under
respect of minor minerals. drilling. So far 69 cores have been collected in 17 wells.
Statement 3 is correct. Control of illegal mining is under 15. Solution: (a)
the legislative and administrative jurisdiction of State
Exp) Option a is the correct answer.
Governments. Section 23C of the MMDR Act, 1957 empowers
India’s remote sensing program was developed with the idea
State Governments to frame rules to prevent illegal mining
of applying space technologies for the benefit of humankind
and the state governments may, by notification in the Official
and the development of the country.
Gazette, make such rules for preventing illegal mining,
transportation and storage of minerals (both major minerals India’s remote sensing programme under the ISRO started off
and minor minerals). in 1988 with the IRS-1A, the first of the series of indigenous
state-of-art operating remote sensing satellites, which was
13. Solution: (d) successfully launched into a polar sun-synchronous orbit on
Exp) Option d is the correct answer. March 17, 1988 from the Soviet Cosmodrome at Baikonur.
Statement 1 is incorrect. As per the new mining law, Mines Applications:
and Minerals (Development and Regulation) Amendment Space Based Inputs for Decentralized Planning (SIS-DP)
Act, 2015, the non-coal mines have to be auctioned by the
National Urban Information System (NUIS)
respective state governments.
ISRO Disaster Management Support Programme (ISRO-
Statement 2 is incorrect. Largest resources in terms of DMSP)
gold ore (primary) are located in Bihar (44%) followed
Preharvest crop area and production estimation of major
by Rajasthan (25%), Karnataka (21%),West Bengal (3%),
crops.
Andhra Pradesh (3% ), Jharkhand (2 %). Karnataka was the
leading producer of primary gold accounting for 99% of the Drought monitoring and assessment based on vegetation
total production. The remaining production was reported condition.
from Jharkhand. Flood risk zone mapping and flood damage assessment.
Statement 3 is correct. Hematite and Magnetite are the most Hydro-geomorphological maps for locating underground
prominent of the iron ores found in India. Almost all the water resources for drilling well.

251 PYQ Workbook


GENERAL GEOGRAPHY

Irrigation command area status monitoring 19. Solution: (d)


Snow-melt run-off estimates for planning water use in down Exp) Option d is the correct answer.
stream projects Pair 1 is incorrectly matched. Drought prone area
• Land use and land cover mapping programme was initiated during the Fourth Five Year Plan
• Urban planning in 1973, with the objectives of providing employment to
• Forest survey the people in drought-prone areas and creating productive
assets. It is an initiative of Ministry of Rural Development.
• Wetland mapping
Pair 2 is incorrectly matched. The basic object of the
• Environmental impact analysis
programme is to minimise the adverse effect of drought
• Mineral Prospecting and control desertification through rejuvenation of
• Coastal studies natural resource base of the identified desert areas. It was
Indian Remote Sensing satellite is not used for introduced as a centrally sponsored scheme in 1977- 78
telecommunications or in traffic studies and for these, geo- under the Ministry of Rural Development.
stationary satellite is utilised. Pair 3 is incorrectly matched. The scheme of National
Watershed Development Project for Rainfed Areas
16. Solution: (d)
(NWDPRA) was launched in 1990-91. It is under the aegis
Exp) Option d is the correct answer. of Ministry of Agriculture. The main objective of the
Pair 1 is incorrectly matched. Kinnaur Valley in Himachal scheme are sustainable management of natural resources,
Pradesh is well known for Apples and orchards. India is enhancement of agricultural production, restoration of
the highest producer of areca nut with Karnataka and Kerala ecological balance, reduction in regional disparity and
accounting for nearly 72 per cent of the total production. creation of sustained employment opportunities.
Pair 2 is incorrectly matched: The major mango-growing
20. Solution: (c)
states are Andhra Pradesh, Uttar Pradesh, Karnataka, Bihar,
Gujarat and Tamil Nadu. Whereas, Mewat lies on the border Exp) Option c is the correct answer.
of Haryana and Rajasthan. Statement 1, 3 and 4 are correct.The main aims of this
Pair 3 is incorrectly matched. Production of soybean in programme are as follows:
India is dominated by Maharashtra and Madhya Pradesh To restore the ecological balance by harnessing, conserving
which contribute 89 per cent of the total production. and developing degraded natural resources such as soil,
Whereas, Coromandel lies in the eastern and southern part vegetative cover and water.
of Tamil Nadu. • Prevention of soil run-off, soil loss.
17. Solution: (b) • Regeneration of natural vegetation.
Exp) Option b is the correct answer. • Rain water harvesting and recharging of the ground
Option 1 is incorrect. Bhadrachalam in Telangana is a hindu Water table.
pilgrimage site situated in Telangana. A 17th-century Enabling multi-cropping and the introduction of diverse
Bhadrachalam Temple complex, dedicated to Lord Rama is agro-based activities, which help to provide sustainable
situated here. livelihoods to the people residing in the watershed area.
Option 2 and 3 are correct. Chanderi Sarees emanating Statement 2 is incorrect. The National River Linking
from Chanderi in Madhya Pradesh and Kancheepuram Project (NRLP) formally known as the National Perspective
Sarees produced in Kancheepuram town in Tamil Nadu, are Plan, envisages the transfer of water from water ‘surplus’
famous varieties of traditional saress. Both these varieties basins to water ‘deficit’ regions. Thus, Inter-Linking the
have earned Geographical Indication Tag in 2005. country’s perennial rivers with seasonal rivers is not a benefit
Option 4 is correct. Karnal is a town located in Haryana, it of implementing the ‘Integrated Watershed Development
is known for production of rice and wheat. However, it is Programme.
not a place of origin for any traditional saree or fabric.
21. Solution: (a)
18. Solution: (d) Exp) Option a is the correct answer.
Exp) Option d is the correct answer. Statement 1 is correct. Due to its drift origin, Indian Coal
Pair 1 is incorrectly matched. NH-4 links four of the 10 has comparatively high ash content (35 – 45%), compared to
most populous Indian cities - Mumbai, Pune, Bangalore, and 10-20% present in imported coal. Coal seams formed due
Chennai. NH 4 is 1,235 km in length and passes through the to drift origin contains higher ash as compared to in-situ
states of Maharashtra, Karnataka and Tamil Nadu. origin of formation. Higher ash content coal burn slowly
Pair 2 is incorrectly matched. NH-6 runs over 1,949 km and it is costly to clean.
from Surat to Kolkata Statement 2 is correct. Indian coal has low sulphur
Pair 3 is incorrectly matched. NH-15 connects Gujarat and content(0.5-1%). Low sulphur content means lower SOX
Punjab. emissions.

PYQ Workbook 252


GENERAL GEOGRAPHY

Statement 3 is incorrect. The coal has high ash fusion Gondwana rocks are the biggest source of coal deposits in
temperature (AFT) of about 1,500°C. low-rank coals, such India. Gondwana system is the major coal deposit of India,
as lignite the operating temperature of the gasifier must be containing up to 90 percent of our coal deposits. Coal is
lower than the AFT to prevent slagging. For gasification of a fossil fuel formed from plant material that accumulated
high-rank coals in entrained beds, however, the operating and underwent transformation over millions of years. The
temperature of the gasifier should be greater than the AFT Gondwana rocks contain layers of coal-bearing sediments
so that the molten ash (slag) could flow down the walls that were deposited during the Permian and Mesozoic eras.
and drain easily from the gasifier in order to reduce the
undesirable slag related problems 26. Solution: (a)
Exp) Option a is the correct answer.
22. Solution: (b)
• The Damodar Valley coal producing area is associated
Exp) Option b is the correct answer.
with the Barakar coalfield, which is the largest and most
Statement 1 is incorrect. Gondwana beds are known for important coalfield in the region.
coal deposits and more recently for shale gas deposits but
• The Son Valley coal producing area is associated with
not natural gas.
the Umaria coalfield, which is located in the Shahdol
Statement 2 is correct. Kodarma district in Jharkhand has district of Madhya Pradesh.
been famous worldwide for mica mining, especially for ruby
• The Godavari Valley coal producing area is associated
mica.
with the Singareni coalfield, which spans across
Statement 3 is incorrect. Dharwars located in Karnataka Telangana and Andhra Pradesh.
possess minerals such as iron and manganese but does not
• The Mahanadi Valley coal producing area is associated
possess any petroleum reserves.
with the Talcher coalfield, which is the largest coalfield
[According to NCERT, the “Dharwar system of rocks” possess in Odisha and one of the major sources of power
petroleum and uranium reserves. However, according to generation in the state.
UPSC’s official answer key, the answer is “b”, thus here we
assume that UPSC is asking about Dharwars in Karnataka 27. Solution: (b)
only] Exp) Option b is the correct answer.
23. Solution: (b) Statement 1 is incorrect: Red soil (not Black Cotton) soil is
Exp) Option b is the correct answer. the predominant type of soil in the state.

Statement 1 is incorrect. The imports are mainly to Statement 2 is correct: Rich pool of skilled labour is
compensate the lack of good quality coal, especially coking available in Tamil Nadu. It is the largest producer of mill-
coal from the mining sources in the country. Thus India made cotton yarns (approx. 44%) in the country and has
imports huge quantity of coal out of compulsion and not the largest number of textile mills. Coimbatore is the largest
as a policy matter. cotton producing district in Tamil Nadu.
Statement 2 is correct. Coal meets around 52% of primary 28. Solution: (b)
commercial energy needs in India and around 66% of India’s Exp) Option b is the correct answer.
power generation is coal based. The domestic production is
able to meet only approximately 80% of domestic demand Statement 1 is incorrect: India has one of the largest reserves
for the rest, coal needs to be imported. of thorium in the world, which can be used as a source of
clean energy.
Statement 3 is correct. In India, about 97 percent coal is
of Gondwana type and remaining is the Tertiary type. The Statement 2 is correct: Thorium is mainly derived from
coal obtained from the Gondwana formations is mainly monazite, a mineral that contains about 8-10% thorium
bituminous and needs to be converted into Coke before and 0.3% uranium. The beach sands of Kerala and Orissa
it can be used in the iron and steel industry. Thus, the have rich reserves of monazite, which is the main source of
imports compensate for the lack of good quality coal. thorium in India.

24. Solution: (c) 29. Solution: (d)


Exp) Option c is the correct answer. Exp) Option d is the correct answer.
The main characteristic of the mixed farming is that farms Each of the mentioned minerals is naturally found in the
pro duce both crops and livestock and the two enterprises are state of Chhattisgarh:
interwoven and integrated. • Chhattisgarh is rich in bauxite deposits. Bauxite is the
Mixed farming reduces dependence on external sources like primary source of aluminium and is extensively used
fertilizers because the crop and animal components in the in various industries, particularly in the production of
farm support each other aluminium metal.
• Dolomite is another important mineral found in
25. Solution: (b)
Chhattisgarh. It is a type of carbonate mineral that
Exp) Option b is the correct answer. is used in a variety of applications, including in the

253 PYQ Workbook


GENERAL GEOGRAPHY

production of cement, as a flux in iron and steel making, • Bharat Aluminium Company Ltd. (BALCO) is located
and in the chemical industry. at Korba in Chhattisgarh. It was incorporated in the year
• Chhattisgarh is known for its substantial reserves of 1965. In 2001 Govt. of India disinvested 51% shares of
iron ore. Iron ore is a crucial raw material for the iron Bharat Aluminium Company Limited (BALCO) to Sterlite
and steel industry, and its availability plays a significant Industries Limited, a subsidiary of Vedanta Limited.
role in the economic development of the region. • Hindustan Aluminium Corporation Limited
• Tin deposits are also found in Chhattisgarh. Tin is used (HINDALCO) was established in 1958 by the Aditya
in various industries, including electronics, packaging, Birla Group. GD Birla set up India›s first integrated
Aluminium facility at Renukoot, in the eastern fringe
and soldering. It is also used to make alloys, such as
of Uttar Pradesh, India. It has the alumina refineries at
bronze.
Belagavi in Karnataka, Alupuram in Kerala and Muri in
30. Solution: (a) Jharkhand.
Exp) Option a is the correct answer. • The Indian Aluminium Company Ltd. (INDAL) started
Option 1 is incorrect: Rourkela Steel Plant of Odisha was production in 1938 as a private company and was converted
the first integrated steel plant in the Public Sector of India. into a public company in 1944. Its three smelting units
It was set up with the collaboration of Germany (not Soviet are located at Alupuram (Kerala), Hirakud (Orissa),
Union). It is the first plant in Asia to adopt the energy- and Belgaum (Karnataka). HINDALCO Industries, a
efficient LD (Linz-Donawitz) process of steel making. company of the Aditya Birla group, mergered the Indian
Aluminium Company (INDAL).
Option 2 is correct: Salem Steel Plant is a premier producer
• National Aluminium Company Limited (NALCO)
of stainless steel in India. It is a unit of Steel Authority
is an Indian public sector company having integrated
of India Limited (SAIL).
and diversified operations in mining, metal and power.
Option 3 is correct: Maharashtra Elektrosmelt Ltd. is Presently, the Government of India holds a 51.5%
99.12% subsidiary of the Steel Authority of India Ltd. It equity stake in NALCO, while the Ministry of Mines
has been merged with SAIL. The process of merger of MEL has administrative control over the company. NALCO is
with SAIL was started in April 2006 and culminated with headquartered in Bhubaneswar, Odisha. The Company
the receipt of the final order from the Ministry of Corporate has Alumina refinery at Damanjodi in the District of
Affairs on June 14, 2011. The merger meets the requirement Koraput in Odisha and Aluminium Smelter & Captive
of manganese based ferro-alloys, a key input for iron & steel Power Plant at Angul.
making.
33. Solution: (d)
Option 4 is correct: Visakhapatnam Steel Plant is a unit
of the Navaratna Rashtriya Ispat Nigam Ltd. Visakhapatnam Exp) Option d is the correct answer.
Steel Plant in Andhra Pradesh is India›s first shore-based (A) is false because there are many tea plantations in
integrated steel plant built with state-of-the-art technology. several African countries, such as Kenya, Uganda, Tanzania,
Malawi, South Africa, and Zambia. Kenya is the highest
31. Solution: (d) producer of tea in Africa and the third-largest producer in
Exp) Option d is the correct answer. the world. Tea is one of the major export crops and sources
Statement 1 is incorrect: Balaghat of Madhya Pradesh is of income for many African countries.
known for its manganese mines, not diamond mines. (R) is true because tea plants do need fertile soil with high
Statement 2 is incorrect: Majhgawan of Madhya Pradesh is humus content. Humus is the organic matter that improves
famous for its diamond mines, not manganese deposits. the soil structure, water retention, nutrient availability, and
microbial activity. Tea plants also prefer acidic soil with a pH
Important Tips between 4.5 and 5.5, and a medium to high level of organic
• Balaghat has the largest and deepest underground matter. However, this is not the reason why there are no tea
manganese mine in Asia. plantations in some African countries, as there are other
• Majhgawan is the site of the only active diamond factors such as climate, rainfall, elevation, pests, diseases,
mine in India, operated by the National Mineral and market demand that affect tea production.
Development Corporation. Important Tips
• India is the third-largest producer of manganese ore in Some of the conditions for tea plantation are:
the world, after China and South Africa.
• Temperature: Tea plants prefer cool to warm
• India is also one of the major producers and exporters temperatures, with a mean annual temperature of 18°C
of diamonds, mainly from the Panna and Chhatarpur to 21°C during the growing season.
districts of Madhya Pradesh.
• Rainfall: Tea plants need heavy rainfall, ranging from
32. Solution: (b) 150 cm to 250 cm per year, with a well-distributed
Exp) Option b is the correct answer. pattern throughout the year.

PYQ Workbook 254


GENERAL GEOGRAPHY

• Humidity: Tea plants thrive in high humidity, with a Canada was the third largest producer of uranium in
relative humidity of 70% to 90% during the growing 2021, with a production volume of 4,693 tonnes, which
season. accounted for 9.1% of the world’s production. Canada has
• Soil: Tea plants require fertile soil with high humus a long history of uranium mining and exploration, dating
content and good drainage. back to the late 1930s. Canada’s uranium reserves are mainly
• Altitude: Tea plants can grow from sea level to located in the Athabasca Basin in northern Saskatchewan,
high altitudes, but the quality and flavor of tea vary which hosts some of the world’s highest-grade uranium
depending on the elevation.
deposits. Canada also has uranium mines and mills in
34. Solution: (b) Ontario and Nunavut. Some of the major uranium mining
Exp) Option b is the correct answer. companies operating in Canada are Cameco, Orano, and
German silver does not contain any silver but is a copper Denison Mines. The top two producers are Kazakhstan and
alloy with nickel and often zinc. It is named for its silvery Namibia.
appearance, but it is not a true silver.
Important Tips
Important Tips
Uranium Production in India:
• Horn silver is another name for silver chloride, which
is a white or grayish mineral that turns purple when • India produces about 2 percent of the world’s uranium.
exposed to light. The total reserves of uranium are estimated at 30,480
tonnes.
• Ruby silver is another name for pyrargyrite, which is a
silver sulfantimonite that has a deep red color and is an • Uranium deposits occur in Singhbhum and Hazaribagh
important source of silver. districts of Jharkhand, Gaya district of Bihar, and in
the sedimentary rocks in Saharanpur district of Uttar
• Lunar caustic is another name for silver nitrate,
Pradesh.
which is a silver salt that has many uses, such as in
photography and medicine. It was called lunar caustic • Some uranium is also found in the copper mines of
because ancient alchemists associated silver with the Udaipur in Rajasthan.
moon. • India ranks ninth in the world in terms of uranium
production
35. Solution: (b)
Exp) Option b is the correct answer. 37. Solution: (d)
The beach sands of many parts of Kerala contain an Exp) Option d is the correct answer.
assemblage of Ilmenite, Rutile, Leucoxene, Monazite,
Zircon and Sillimanite. Tungsten is not found in the beach Mica is widely distributed in India and the minerals
sands of Kerala. bearing mica are found in the states of Andhra Pradesh,
Important Tips Maharashtra, Jharkhand, Odisha, Rajasthan, etc. The map
• Ilmenite is a titanium-iron oxide mineral that is the shows the distribution of mica in these states, as well as in
most important ore of titanium and the main source of Bihar, Madhya Pradesh and Tamil Nadu. Mica is used for
titanium dioxide, which is used in paints, printing inks, various purposes such as electrical and electronic devices,
fabrics, plastics, paper, sunscreen, food and cosmetics. cosmetics, paints, etc.
Ilmenite is also used for processing of graded steel,
welding electrodes, manufacturing of titanium metal, Important Tips
coatings and lining for blast furnace hearth. • Asbestos: This is a mineral that is composed of thin
• Zircon is a zirconium silicate mineral that is mainly fibrous crystals. It has been used for insulation,
consumed as an opacifier and has been known to fireproofing, roofing, etc. However, it is also a
be used in the decorative ceramics industry and carcinogen and can cause serious health problems such
zirconium metal production. Zircon is also used in as lung cancer, mesothelioma, etc. India is the second
nuclear reactors, photovoltaic cells, nanotechnology, largest consumer of asbestos in the world, after China.
electronics, and many other cutting-edge applications. India does not produce much asbestos but imports it
• Sillimanite is an aluminium silicate mineral that is from countries like Russia and Brazil.
used as a refractory material for high temperature • Gypsum: This is a soft sulfate mineral that is widely
applications. Sillimanite is also used in the manufacture used as a fertilizer, plaster, wallboard, etc. It is also used
of high-quality photo grey sunglasses, commercial to make sculptures, ornaments, and alabaster. India is
aerospace, and many other cutting-edge applications one of the major producers and consumers of gypsum
in the world. India produces around 2.5% of the world’s
36. Solution: (b) natural gypsum, which is mainly concentrated in
Exp) Option b is the correct answer. Rajasthan.

255 PYQ Workbook


GENERAL GEOGRAPHY

• Limestone: This is a sedimentary rock that is mainly Both (A) and (R) are true, and (R) is the correct explanation
composed of calcium carbonate. It is used for building of (A).
materials, cement, lime, etc. It is also a source of fossils Assertion (A) is true: Chile is the top copper producer in
and minerals such as calcite, aragonite, dolomite, etc. the world with 28 percent of global copper production and
Karnataka is the leading state having 27% of the total the greatest concentration of the largest copper porphyry
resources followed by Andhra Pradesh and Rajasthan deposits.
(12% each), Gujarat (10%), Meghalaya (9%), Telangana Reason (R) is true and correctly explains Assertion (A):
(8%) and Chhattisgarh and Madhya Pradesh (5% each). Porphyry copper deposits are copper ore bodies that are
formed from hydrothermal fluids that originate from a
38. Solution: (c)
voluminous magma chamber several kilometers below the
Exp) Option c is the correct answer. deposit itself. Chile is endowed with the largest reserves of
Quartzite is a metamorphic rock that forms when quartz- porphyry copper in the world, which account for most of its
rich sandstone is changed by heat, pressure, and chemical copper output.
activity of metamorphism. The sand grains and the silica
42. Solution: (c)
cement that bind them together are recrystallized into a
network of interlocking quartz grains of incredible strength. Exp) Option c is the correct answer.
Quartzite is usually white to grey in color, but it can also be • Madhya Pradesh has the second largest reserves/
pink, red, purple, yellow, green, blue, or brown depending resources of manganese ore in the country, with 12%
on the impurities present. Quartzite has a hardness of about share. The total reserves/resources of manganese ore
seven on the Mohs scale and is very resistant to weathering in Madhya Pradesh as on 1.04.2015 were 59.77 million
and erosion. Quartzite is used as a building material, a tonnes.
decorative stone, an abrasive, and a gemstone. • Nickel ore is found in Sukinda Valley in Odisha, which
has the largest resources of nickel ore in the country
39. Solution: (b)
at 140 million tonnes (74%). Jharkhand has 9 million
Exp) Option b is the correct answer. tonnes (5%) resources, most of which are in Singhbhum
Assertion (A) is correct: Australia is known for its extensive (East) district.
beef cattle industry, and a significant portion of cattle • Lead-zinc ore is mostly found in Rajasthan, which has
rearing is for meat production. the largest resources of lead-zinc ore in the country at
Reason (R) is correct but does not correctly explain 607.53 million tonnes (89%). Andhra Pradesh has 22.69
Assertion (A): Most of the Australian people are non- million tonnes (3%) resources, followed by Madhya
vegetarians. But this is not the reason for the growth of Pradesh with 14.84 million tonnes (2%).
cattle rearing industry for meat in Australia. The main • Asbestos is mainly produced in Andhra Pradesh, which
factors that influence the cattle industry are the availability has the largest resources of asbestos in the country at 12.6
of land, water, feed, climate, market demand (exports), and million tonnes (55%). Rajasthan ranks second with 8.02
animal health and welfare. million tonnes (35%) resources, followed by Karnataka
with 1.73 million tonnes (8%).
40. Solution: (a)
Exp) Option a is the correct answer. 43. Solution: (a)
• Venezuela is the major producer of mineral oil. Exp) Option a is the correct answer.
Venezuela has the largest proven oil reserves in the Kurdish population is spread in the western parts of
world, with 302.8 billion barrels as of 2020. Armenia, which was once part of the erstwhile Soviet
• Zambia is the seventh largest copper producer in the Union. Most Kurdish Armenians are Yazidis, with a strong
world, with 861,000 metric tons of copper output in Kurdish identity. The 2011 census recorded 35,308 Yazidis
2020. It also holds about 6 percent of the world’s known and 2,162 Kurdish individuals in Armenia. While Yazidis
copper reserves. form a distinct ethnic group, they share cultural ties with the
broader Kurdish community.
• Gabon is the second largest manganese producer in the
world, with 2.5 million metric tons of manganese output
in 2020. It also has the fourth largest manganese reserves
6.2.. Other Examination Previous Years’
in the world, with 200 million metric tons. Questions
• Guyana is the fifth largest bauxite producer in the 44. Solution: (c)
world, with 1.8 million metric tons of bauxite output in
Exp) Option c is the correct answer.
2020. It also has significant bauxite reserves, estimated at
350 million metric tons Pegu Yoma region of Myanmar is rich in minerals, including
metal ores, petroleum, and natural gas, mineral oil and also
41. Solution: (a) has significant deposits of precious and semiprecious stones.
Exp) Option a is the correct answer. Pegu Yoma region is a range of low mountains or hills and

PYQ Workbook 256


GENERAL GEOGRAPHY

uplands that stretches from Mount Popa in the north to the Ministry of Steel. Its Registered Head office is in
Singuttara Hill in the south. The region is bounded by the Hyderabad. It is the single largest producer of iron
Irrawaddy River to the west and the Sittaung River to the ore in India. NMDC is considered to be one of the low-
east. cost producers of iron ore in the world. It also operates
the only mechanized diamond mine in India at Panna,
45. Solution: (b) Madhya Pradesh.
Exp) Option b is the correct answer. • National Aluminium Company Limited (NALCO)
• Osaka is a city in Japan that is known for its cotton is a Navratna CPSE established on 7th January 1981
textile industry, among other sectors. It has a long having its registered office at Bhubaneswar. At present,
history of producing and trading cotton fabrics, especially Government of India holds 51.28% of paid-up equity
in the 19th and 20th centuries, when it attracted foreign capital.
investment and commerce. Osaka’s cotton mills employed
millions of workers and produced a variety of textiles and 47. Solution: (a)
garments. Exp) Option a is the correct answer.
• Detroit is a city in the United States that is famous for The Koryaks are an indigenous people of the Russian Far
its automobile industry. It is the largest city in the state East, residing north of the Kamchatka Peninsula and not
of Michigan, which is located in the Great Lakes region. Alaska. They live along the Bering Sea coast and share
Detroit was the home of the “Big Three” American car cultural and linguistic ties with the Chukchis.
manufacturers: Ford, General Motors, and Chrysler. It
Important Tips
was also known as the “Motor City” or “Motown” for its
influence on the car culture and music industry. • Punan Bah or Punan is an ethnic group found in
Sarawak, Malaysia and in Kalimantan (Indonesian
• Cuba is a country in the Caribbean that is one of the portion of the island of Borneo), Indonesia. The
world’s leading producers of cigars, as well as other Punan Bah people are distinct and unrelated to the
tobacco products. Cuba has a rich tradition of growing semi-nomadic Penan people. Their name stems from
and curing tobacco, dating back to the colonial times. two rivers along the banks of which they have been
Cuban cigars are renowned for their quality, flavor, and living since time immemorial.
aroma, and are exported to many countries around the
• The Ruwallah is a prominent Arab tribe historically
world.
located in northern Arabia and the Syrian Desert,
• St. Petersburg is a city in Russia that has a strong including modern-day Jordan. They emerged around
tradition of shipbuilding. It is located on the Baltic Sea the 16th century, belonging to the Dhana Maslam
and has access to several rivers and lakes. St. Petersburg branch of the ‘Anizzah tribal confederation. While
was the main base of the Russian Navy and a major center once a warrior tribe, many members have now adopted
of naval engineering and construction. It also produced sedentary or urban lifestyles in Saudi Arabia, Jordan,
civilian ships, such as icebreakers and cargo vessels. and Syria.
46. Solution: (c) 48. Solution: (a)
Exp) Option c is the correct answer. Exp) Option a is the correct answer.
• Oil and Natural Gas Commission has its headquarters Latvia is not a major cocoa producer country, as it is
in New Delhi. ONGC was founded on 14 August 1956 by located in Northern Europe, where the climate is not
the Government of India. It is the largest government- suitable for growing cocoa trees. According to the Food and
owned-oil and gas explorer and producer in the country Agriculture Organization of the United Nations, Latvia did
and produces around 70 percent of India’s domestic not produce any cocoa beans in 2019. The other options are
production of crude oil and around 84 percent of all major cocoa producer countries, as they are located in the
natural gas. In November 2010, the Government of India tropical and subtropical regions of Africa, where the climate
conferred the Maharatna status to ONGC. is hot and humid.
• Mineral Exploration and Consultancy Limited (MECL) According to the International Cocoa Organization,
(Formerly Mineral Exploration Corporation Ltd.) was Cameroon produced 290,000 tonnes of cocoa beans in
established as an autonomous Public Sector Company 2019/2020, Ghana produced 800,000 tonnes, and Ivory Coast
in October 1972, under the administrative control produced 2,225,000 tonnes. These three countries accounted
of Ministry of Mines, Government of India for systematic for about 66% of the world’s total cocoa production in that
exploration of minerals, to bridge the gap between year.
the initial discovery of a prospect and its eventual
exploitation. The Company has its corporate office 49. Solution: (c)
at Nagpur, Maharashtra. Exp) Option c is the correct answer.
• The National Mineral Development Corporation Pair 1 is incorrect: Copper is mainly mined in Khetri,
(NMDC) is a Navratna Public Sector Enterprise under Rajasthan, which is the largest copper producing area in

257 PYQ Workbook


GENERAL GEOGRAPHY

India. Chitradurga, Karnataka, on the other hand, is known 52. Solution: (c)
for its iron ore deposits. Therefore, the first pair of copper Exp) Option c is the correct answer.
and Chitradurga is not correctly matched.
Normandy is a region of northern France, not Germany.
Pair 3 is incorrect: Manganese is mainly mined in Normandy was the site of the Allied invasion of Nazi-
Balaghat, Madhya Pradesh, which is the largest producer occupied France during World War II, known as D-Day.
of manganese ore in India. Bhilwara, Rajasthan, is famous
Important Tips
for its mica and textile industries, but not for manganese
mining. Therefore, the third pair of manganese and Bhilwara • Karaganda is a region in Kazakhstan that has
is also not correctly matched. abundant coal and iron ore resources. It is home to the
ArcelorMittal Temirtau company, which operates an
Pairs 2 and 4 are correct: Iron ore is mined in Ballari, integrated steel plant, coal mines and iron ore mines
Karnataka, which is one of the major iron ore producing in the area.
districts in India. Bauxite is mined in Katni, Madhya Pradesh,
• Krivoy Rog is a city in southern Ukraine that is located
which has rich deposits of bauxite and alumina.
at the confluence of two rivers. It is one of the largest
50. Solution: (d) iron ore deposits in the world, with an estimated reserve
of over 20 billion tons3. The iron ore is mainly hosted
Exp) Option d is the correct answer.
by ferruginous quartzite with martite and jaspilite.
Statement 1 is incorrect: Mumbai (not Ahmedabad) is the • Pyrenees are a mountain range that forms the natural
largest cotton textile center in the country. It is known as the border between France and Spain. The region has a
cottonpolis of India due to Manufactuing hub. The city has centuries-old tradition of working iron and steel, as it
a humid, warm climate which supports the growth of cotton. is rich in iron ore and has abundant forests and water
Statement 2 is correct: Ahmedabad is located in major resources.
cotton growing region of India, so it has no problem of raw
53. Solution: (c)
materials. Black soil is favorable for the cotton production
which is present in the state of Gujarat. There is no problem Exp) Option c is the correct answer.
getting raw materials. The climate is humid and is suitable The Kinta Valley is historically very rich in tin, and their
for this industry. mines have been among the most productive in the world.
The valley is formed by the Kinta River, which flows between
51. Solution: (d) the Titiwangsa Mountains and the Kledang Range. It forms
Exp) Option d is the correct answer. the largest tin field along the western Malayan tin belt and
The correct matching of ethnic groups with their respective has been mined since the 1880s.
countries is as follows: Important Tips
A. Jews 4. Israe Tin Ore in Inida:
B. Teda 3. Libya • Chhattisgarh is the only tin producing State in India.
C. Beja 1. Egypt • Tin ore is known as cassiterite, which was reported
in Dantewada district by the Directorate of Geology
D. Lur 2. Iran
and Mining and was found to be associated with the
Important Tips lepidolite-bearing pegmatites.

• Jews are an ethnoreligious group practicing Judaism, • In India tin ore is found associated with granite,
tracing their heritage to the ancient Hebrews or pegmatites and quartz veins and also in placer deposits.
Israelites. Their identity is defined by both religious 54. Solution: (c)
faith and shared ancestry.
Exp) Option c is the correct answer.
• The Teda language, also known as Tedaga, is a Saharan
language spoken by the Teda, a northern subgroup of Postmasburg and its adjacent areas of South Africa are
the Toubou people that inhabits southern Libya, major producers of manganese ore, which is used in the
northern Chad and eastern Niger. production of steel, batteries, and chemicals. Postmasburg
has two belts of manganese ore deposits: the Western Belt of
• The Beja people are an ethnic group native to the
ferruginous manganese ores and the Eastern Belt of siliceous
Eastern Desert, inhabiting a coastal area from
manganese ores. The ores are composed mainly of braunite
southeastern Egypt through eastern Sudan and into
group minerals, including braunite, partridgeite and bixbyite.
northwestern Eritrea.
The formation of these ores is related to karstification of
• The Lurs are an Iranian people living in western Iran.
manganese-rich dolomites and hydrothermal activity.
The four Luri branches are the Bakhtiari, Mamasani,
Kohgiluyeh and Lur proper, who are principally linked 55. Solution: (a)
by the Luri language. Exp) Option a is the correct answer.

PYQ Workbook 258


GENERAL GEOGRAPHY

• Talcher coal field is located in the Mahanadi Valley in Important Tips


Odisha. It is the second largest coal reserve in India and
Coal Formation:
produces non-coking coal of various grades and qualities.
Coal is a sedimentary rock that forms from the
• Karnapura coal field is located in the Damodar Valley accumulation and compression of plant matter over
in Jharkhand. It is one of the most important coalfields millions of years. The Gondwana System of Rocks dates
in India and has a total geological coal reserve of 9,074.51 back to the Carboniferous and Permian periods, when
million tonnes. most of the landmasses were united in a supercontinent
• Singroli coal field is located in the Son Valley in Madhya called Gondwana. During this time, vast swamps and
Pradesh. It is the largest coalfield in India in terms of forests covered the land, providing abundant organic
reserves and production. It produces both coking and material for coal formation.
non-coking coal of different grades and qualities. 58. Solution: (a)
• Singreni coal field is located in the Godavari Valley in Exp) Option a is the correct answer.
Telangana and Andhra Pradesh. It is the only coalfield • Mexico is the world’s largest silver producer, accounting
in South India and spans across four districts. It produces for about 23% of the global output in 2022. Mexico has
non-coking coal of various grades and qualities rich silver deposits in several states, such as Zacatecas,
Chihuahua, Durango, and Sonora. Some of the major
56. Solution: (d)
silver mines in Mexico are Fresnillo, Saucito, Peñasquito,
Exp) Option d is the correct answer. and San Dimas.
The Khirghiz people are traditionally associated with • India is one of the world’s largest producers and
Central Asia, particularly countries like Kyrgyzstan, exporters of mica. India’s mica industry plays an
Kazakhstan, and Uzbekistan. They are not from Sudan. important role in the country’s overall mining activity,
They speak the Kyrgyz language and have a rich history, contributing significantly to the economic growth of
with roots dating back to ancient Central Asian tribes and India. India has abundant mica resources in several states,
empires. such as Andhra Pradesh, Bihar, Jharkhand, Rajasthan,
and Telangana.
Important Tips
• Russia is one of the world’s leading producers and
• The Ainu are indigenous people in Northern Japan exporters of iron ore. According to GlobalData, Russia
and nearby regions, including Hokkaido Island, was the fifth-largest producer of iron ore in 2022, with
Sakhalin Island, and the Kuril Islands. They have output down by 16.3% on 2021.
occupied these areas since before the arrival of modern
• Chile is the world’s largest copper producer, accounting
Japanese and Russians. Official estimates put the Ainu
for about 28% of the global output in 2022 . Chile has
population in Japan at 25,000, but unofficial estimates
rich copper deposits in several regions, such as Atacama
suggest it could be much higher due to assimilation
Desert, Andes Mountains, and Central Valley. Some of
into Japanese society.
the major copper mines in Chile are Escondida (the
• The Angamis are a major Naga ethnic group world’s largest copper mine), Chuquicamata (the
predominantly inhabiting the Kohima District, world’s largest open-pit mine), Collahuasi (the world’s
Chümoukedima District and Dimapur District in the second-largest copper mine), and El Teniente (the
Northeast Indian state of Nagaland. world’s largest underground mine).
57. Solution: (b) 59. Solution: (b)
Exp) Option b is the correct answer. Exp) Option b is the correct answer.
Assertion (A) is true: Gondwana system of rocks contains The coalfields and the states they are located in are as
nearly 98% of India’s coal reserves1. The Gondwana system follows:
is also called the Carboniferous system because it was • Rajmahal coalfield is located in the east of India in
formed during the Carboniferous period, about 300 million Jharkhand.
years ago. The Gondwana rocks are mainly composed of • Sohagpur coalfield is located in Shahdol district in
sandstones, shales, and coal seams that were deposited in Madhya Pradesh in the basin of the Son River.
large basins by rivers and lakes.
• Bisrampur coalfield is located in the east of India in
Reason (R) is correct but do not correctly explain Assertion Chhattisgarh
(A): The Dharwar system of rocks is a highly metamorphosed • Raniganj coalfield is primarily located in the Asansol and
sedimentary rock system that formed between 2.5 billion to Durgapur subdivisions of Paschim Bardhaman district
1.8 billion years ago. The Dharwar rocks are mainly schists, of West Bengal. It spreads over to the neighbouring
phyllites, and slates that contain valuable minerals like iron districts of Birbhum, Bankura, Purulia and to Dhanbad
ore, manganese, copper, lead, gold, etc district of Jharkhand

259 PYQ Workbook


GENERAL GEOGRAPHY

60. Solution: (c) Important Tips


Exp) Option c is the correct answer. Tin Ore:
The term ‘Umland’ denotes the surrounding area of a town • In India, tin ore (cassiterite) is found associated with
or city. It encompasses the region adjacent to an urban granite, pegmatites and quartz veins and also in placer
center, characterized by functional interconnections deposits. Resources are spread over in Bastar and
with the town or city. This includes economic, social, and Dantewada districts of Chhattisgarh, Tosham deposit
infrastructure ties, as well as the movement of people who in Bhiwani district of Haryana and Malkangiri district
often travel from the Umland to the urban area for services, of Odisha.
employment, and the acquisition of goods. It signifies the • Tin is now used mostly for tin plating, soldering special
broader urban influence sphere beyond the city limits. alloys and in making bronze.
• China has the largest tin reserves globally, estimated at
61. Solution: (a)
1.1 mn metric tons, accounting for 23% of the global
Exp) Option a is the correct answer. tin reserves
• Kinki is an industrial region in Japan, located in
63. Solution: (d)
the southern-central part of the island of Honshu. It
Exp) Option d is the correct answer.
includes the prefectures of Kyoto, Osaka, Hyogo, Nara,
Wakayama, and Mie. It is one of the most densely Rajasthan is the largest producer of asbestos in India,
populated and economically developed regions in Japan, accounting for 59% of the total reserves/resources of
with a large concentration of manufacturing, service, and asbestos in the country. The important occurrences of
asbestos in Rajasthan are known in Udaipur, Dungarpur,
high-tech industries.
Alwar, Ajmer and Pali districts. Andhra Pradesh is the
• Canton is an industrial region in China, also known second-largest producer of asbestos in India, followed by
as Guangdong province. It is located in the south of Karnataka. Madhya Pradesh and Bihar do not have significant
China, bordering the South China Sea. It is the most deposits of asbestos.
populous and richest province in China, with a vibrant
and diversified economy. It is a major hub of trade, 64. Solution: (b)
commerce, manufacturing, and innovation, especially Exp) Option b is the correct answer.
in sectors such as electronics, telecommunications, The world’s largest rice gene bank is situated in the
automobiles, petrochemicals, and biotechnology. International Rice Research Institute (IRRI) in Los Baños,
• Lorraine is an industrial region in France, located the Philippines. The IRRI gene bank, which was established
in 1962, holds more than 132,000 available accessions of
in the northeast of the country. It borders Germany,
rice and its wild relatives as of December 2019. The gene
Luxembourg, and Belgium. It is a historical and cultural
bank is the biggest collection of rice genetic diversity in the
region that has been influenced by various European
world and aims to develop crops that can survive climate
powers over the centuries. It has a rich natural and
change and other challenges. The gene bank also received
mineral heritage, especially in iron ore and coal. It is a
permanent funding from the Crop Trust, a non-governmental
major center of steel production, metallurgy, automobile organization that supports food security and crop diversity,
manufacturing, and textiles. in 2018.
• Belo Horizonte is an industrial region in Brazil,
Important Tips
located in the southeast of the country. It is the capital
International Crops Research Institute for the Semi-
and largest city of Minas Gerais state. It is a planned
Arid Tropics (ICRISAT):
modern city that was founded in 1897 to replace Ouro
• It is a non-profit, non-political organization that
Preto as the state capital. It is a dynamic and diversified
conducts agricultural research for development in
urban center that hosts various sectors of industry,
the drylands of Asia and sub-Saharan Africa.
commerce, services, and technology. It is known for its
• It is headquartered in Hyderabad, Telangana
steel, automotive, textile, mining, and biotechnology
State, in India, with two regional hubs (Nairobi,
industries.
Kenya and Bamako, Mali) and country offices in
62. Solution: (c) Niger, Nigeria, Zimbabwe, Malawi, Ethiopia and
Mozambique.
Exp) Option c is the correct answer.
• ICRISAT conducts research on five highly nutritious
As per data from Indian Bureau of Mines, Chhattisgarh
drought-tolerant crops: chickpea, pigeonpea, pearl
has the largest reserves of tin ore in India, with about millet, sorghum, and groundnut.
4,419 tonnes of ore containing 154 tonnes of metal. Tin
ore is mainly found in Bastar and Dantewada districts of 65. Solution: (c)
Chhattisgarh. Exp) Option c is the correct answer.

PYQ Workbook 260


GENERAL GEOGRAPHY

The electronics industry is not dependent on the Rajasthan is the sole producer of zinc ores in India
availability of raw materials as the primary factor affecting and accounts for 100% of the country’s zinc production.
its location. The electronics industry mainly relies on the Rajasthan also has deposits of lead ores, which are often
availability of skilled labor, capital, technology, and market found along with zinc ores. Rajasthan has a near monopoly
access. The other industries, such as iron and steel, sugar, in the production of both these minerals. Copper, mica and
and cement, are more influenced by the availability of raw dolomite are not produced exclusively by Rajasthan and are
materials such as iron ore, coal, sugarcane, and limestone found in other states as well.
respectively.
Important Tips
66. Solution: (b) • Copper: India is the ninth largest producer of copper
Exp) Option b is the correct answer. in the world, with an annual production of about
Detroit is not a major center of cotton textile production, 700,000 tonnes. The major copper-producing states
but rather of automobile manufacturing. Detroit is known are Rajasthan, Madhya Pradesh, Jharkhand, Gujarat
as the Motor City because of its long history and prominent and Maharashtra.
role in the U.S. automotive industry. Detroit is home to the • Mica: India is one of the top mica producers in the
headquarters of the Big Three automakers: General Motors, world, with an annual production of about 16,000
Ford, and Chrysler. tonnes in 2022. The major mica-producing states are
Andhra Pradesh, Rajasthan, Bihar, Jharkhand and
Important Tips
Maharashtra.
• Ontario is Canada’s largest producer of paper
• Dolomite: Dolomite is a sedimentary rock
and pulp products, accounting for about 40% of
composed mainly of calcium magnesium carbonate
the country’s output. Ontario has abundant forest
(CaMg(CO3)2). India is one of the leading producers
resources and access to waterways for transportation
of dolomite in the world, with an annual production of
and power generation.
about 7.3 million tonnes in 2014. The major dolomite-
• Texas is the leading state in the U.S. chemical producing states are Madhya Pradesh, Andhra Pradesh,
industry, producing about 28% of the nation’s Chhattisgarh, Odisha, Karnataka and Rajasthan.
chemicals by value. Texas has a large supply of
natural gas and oil, which are the main feedstocks for 70. Solution: (b)
chemical production. Texas also has a well-developed Exp) Option b is the correct answer.
infrastructure of pipelines, ports, railroads, and • Titagarh, in West Bengal, is famous for its paper
highways for transporting chemicals. industry and freight wagons of Indian Railway. The first
• Nagoya is Japan’s third-largest city and a major hub of paper mill was set up in the Titagarh in 1882, which was
the automotive industry. Nagoya is the headquarters known as the Titagarh Paper Mill. Till independence,
of Toyota Motor Corporation, the world’s largest the Titagarh Paper Mills was one of the country’s leading
automaker by sales. Nagoya also hosts several other manufacturers of specialty papers such as currency note
automotive companies and suppliers, such as Denso, papers and newsprint.
Aisin Seiki, Mitsubishi Motors, Suzuki, and Yamaha. • Lakheri, in Rajasthan, is a cement manufacturing
67. Solution: (c) unit of Associated Cement Companies Ltd. (ACC). This
plant is the longest-running cement plant in Asia having
Exp) Option c is the correct answer. opened in 1912–1913.
The natives of North America belong to the racial • Bhilai, in Chhattisgarh, is famous for the Iron &
group known as Mongoloid. According to the Meyers Steel industry. Bhilai Steel Plant is one of the largest
Konversations-Lexikon (1885–90), peoples included in the in India. The Bhilai Steel Plant is renowned as the sole
Mongoloid race are North Mongol, Chinese & Indochinese, manufacturer of rails in India that are used by the
Japanese & Korean, Tibetan & Burmese, Malay, Polynesian, Indian Railways.
Maori, Micronesian, Eskimo, and Native American. This
racial group is characterized by features such as yellowish • Ambala Mukul, in Kerala, is famous for its Mineral
or light wheatish skin, straight black hair, small almond- Oil Refinery. The BPCL Kochi Refinery embarked on
shaped eyes, and a lean physique. its journey in 1966. It is located at Ambala Mukul or
Ambalamugal, near Kochi in Kerala. It is one of the three
68. Solution: (c) Refineries of BPCL.
Exp) Option c is the correct answer. 71. Solution: (b)
The correct sequence of states of India in descending order in Exp) Option b is the correct answer.
terms of reserves of coal (measured) is Jharkhand, Odisha,
Chhattisgarh, Madhya Pradesh. • J.K. Nagar is situated in Bankura, West Bengal. Lakshmipat
Singhania set up the Aluminium Corporation in J.K.
69. Solution: (c) Nagar. This unit was later merged with BALCO in 1984
Exp) Option c is the correct answer. and has since been known as BALCO Bidhanbag unit.

261 PYQ Workbook


GENERAL GEOGRAPHY

• Malanjkhand is an open-pit copper mine in India. It is grown in tropical and subtropical regions, where the
is located near the town of Malanjkhand in the district climate is hot and moist.
Balaghat of Madhya Pradesh. It is the single largest
copper deposit of India with nearly 70% of the country’s 74. Solution: (b)
reserve and contributing around 80% to Hindustan Exp) Option b is the correct answer.
Copper Limited (HCL) total copper production. • Donetsk coal field is located in eastern Ukraine, near
• The Tundoo near Dhanbad (Jharkhand) is famous for the border with Russia.
its first lead smelting plant. The lead smelting plants are • Kuznetsk coal field is located in southwestern Siberia,
used in Zinc production. Commercial production by Russia, and is one of the largest coal mining areas in the
this plant commenced in 1945. It was later on taken over country.
by Hindustan Zinc Ltd.
• Lancashire coal field is located in Northwest England
• The Reliance Jute Mills Company Limited was and was an important British coalfield in the 19th and
incorporated on 25th October 1906 and owned the Jute
20th centuries.
Mill situated at Bhatpara in West Bengal. North 24
Parganas district, in which Bhatpara and Kankinara fall, • Saar coal field is located in the Saarland region of
is a prominent jute belt of West Bengal. Germany, near the border with France, and was a
disputed territory between the two countries in the 20th
72. Solution: (b) century.
Exp) Option b is the correct answer.
75. Solution: (b)
• Cocoa is mainly grown in the Ghana. Ghana is the
Exp) Option b is the correct answer.
worlds second largest Cocoa produder.
• Kuzbas is a coalfield in Siberia, Russia, that is one of
• Coffee is predominantly cultivated in Ivory coast,
the largest coal mining areas in the world. It covers
which is the Africa largest coffee producer.
an area of about 70,000 square kilometers and contains
• Tea is a major crop in Kenya, which is the world’s
about 200 billion tonnes of coal reserves.
largest exporter and third-largest producer of tea. Kenya
produces about 10% of the world’s tea and exports about • Red Basin is a coalfield in Sichuan province, China,
20% of the global share. that is also known as the Sichuan Coalfield. It covers
an area of about 6,000 square kilometers and contains
• Sugarcane is a significant crop in South Africa, which
about 60 billion tonnes of coal reserves. It is the largest
is the world’s tenth-largest producer and second-largest
anthracite coalfield in China and produces high-quality
exporter of sugar. South Africa produces about 2% of the
coking coal.
world’s sugar and exports about 10% of the global share.
• Bristol is a coalfield in the west of England, that
73. Solution: (b) comprises the coal-bearing rocks arranged around
Exp) Option b is the correct answer. the Coalpit Heath Syncline and Kingsdown Anticline. It
• Rubber is mainly grown in Malaysia, which is the extends beneath the eastern parts of the city of Bristol
world’s largest producer and exporter of natural rubber. and northwards through southern Gloucestershire. It has
Malaysia accounts for about 30% of the global rubber several satellite coalfields, such as the Severn Coalfield
production and exports. Rubber is cultivated in tropical and the Nailsea Basin.
and subtropical regions, where the climate is warm and • New South Wales is a state in Australia that has several
humid. coalfields, such as the Sydney-Gunnedah Basin, the
• Coffee is predominantly cultivated in Colombia, Gloucester Basin, and the Oaklands Basin. It has more
which is the world’s third-largest producer and exporter than 15 billion tonnes of recoverable coal reserves and is
of coffee. Colombia produces about 10% of the world’s one of the world’s major exporters of coal. Coal is NSW’s
coffee and exports about 15% of the global share. Coffee largest export earner in value terms, worth around $13.2
is grown in warm and wet climates, where the soil is rich billion in 2015-16.
and well-drained.
76. Solution: (c)
• Olive is a major crop in Italy, which is the world’s
second-largest producer and consumer of olive oil. Italy Exp) Option c is the correct answer.
produces about 15% of the world’s olive oil and consumes The Mesabi Range is not a coal field, but an iron ore
about 30% of the global share. Olives are grown in mining district in northeastern Minnesota, U.S. It is the
Mediterranean regions, where the climate is mild and largest of four major iron ranges in the region collectively
sunny. known as the Iron Range of Minnesota. Cleveland is a major
• Sugarcane is a significant crop in Mauritius, which center of iron and steel production in the U.S., Detroit is
is the world’s tenth-largest producer and exporter of the headquarters of the American automobile industry, and
sugar. Mauritius produces about 4% of the world’s sugar Philadelphia is one of the oldest and largest shipbuilding
and exports about 20% of the global share. Sugarcane cities in USA.

PYQ Workbook 262


GENERAL GEOGRAPHY

77. Solution: (d) • Khamman is a district in Andhra Pradesh that has


Exp) Option d is the correct answer. copper ore reserves. The district has abundant mineral
resources like coal, iron ore, limestone, quartz, dolomite,
The San people, also known as Bushmen, are indigenous
copper and manganese.
hunter-gatherer cultures in southern Africa. They
historically inhabited the Kalahari region, which spans • Khetri is a town in Rajasthan that is known for its
across Botswana, Namibia, Angola, Zambia, Zimbabwe, copper belt. Khetri is situated at the foothills of the
Lesotho, and South Africa. The Bushmen are among the Aravalli Range, which hosts copper mineralization,
oldest surviving cultures in this area and speak languages of giving rise to an 80 km long metallogenetic province
the Khoe, Tuu, and Kxa language families. from Singhana in the north to Raghunathgarh in the
south, popularly known as “Khetri Copper Belt.”
Important Tips
• Bundu, also known as Sande or zadεgi, is a women’s 81. Solution: (c)
initiation society in Liberia, Sierra Leone, Guinea, Exp) Option c is the correct answer.
and the Ivory Coast. It conducts rituals, including
• Hamilton is a city in Ontario, Canada, that has a long
female genital mutilation, to initiate girls into
history of iron and steel production. It is located near the
adulthood. Supporters claim it fosters fertility, moral
Great Lakes, which provide water and transportation for
values, proper sexual behavior, and lifelong support for
industry. Hamilton is home to several steel mills, such as
its members.
ArcelorMittal Dofasco and Stelco.
• Birmingham is a city in England, UK, that was once a
78. Solution: (a)
major centre of iron and steel manufacturing during the
Exp) Option a is the correct answer. Industrial Revolution. It was known as the “workshop
• Gurumahisani is a mining town in Odisha, India, where of the world” for its diverse and innovative industries.
iron ore is extracted. It is India’s first iron ore mine and Birmingham’s iron and steel sector declined in the 20th
has been operating since 1910. century due to competition and deindustrialization.
• Talcher is a coalfield in Angul district of Odisha, India, • Essen is a city in North Rhine-Westphalia, Germany,
where coal is mined. It has reserves of 38.65 billion that was the headquarters of the Krupp family, one of
tonnes, the highest in India. The coal is of lower grade the largest industrial dynasties in Europe. The Krupp
and has high ash content. company produced iron and steel, as well as weapons,
• Jaduguda is a uranium mine in Purbi Singhbhum machinery, and vehicles. Essen was heavily bombed
district of Jharkhand, India, where uranium ore is during World War II but rebuilt its iron and steel industry
processed. It commenced operation in 1967 and was the after the war.
first uranium mine in India. The deposits at this mine • Anshan is a city in Liaoning province, China, that is
were discovered in 1951. one of the largest iron and steel producers in the country.
• Zawar is a mining area in Rajasthan, India, where Lead It has rich deposits of iron ore and coal, as well as
and Zinc is produced. limestone and manganese. Anshan Iron and Steel Group
Corporation (Angang) is the main company in the city,
79. Solution: (b)
which also produces other metals and chemicals.
Exp) Option b is the correct answer.
The Bantu people are a vast ethnolinguistic group 82. Solution: (b)
comprising around 400 distinct African ethnic groups Exp) Option b is the correct answer.
speaking Bantu languages. These languages are primarily • HINDALCO Alupuram Works was set up in 1938 in
native to Central, Southeast, and Southern Africa, totaling Kerala. It was here that the country’s first aluminium
between 440 to 680 languages and 350 million speakers. They ingot was produced at the Alupuram smelter in 1943.
are not associated with the Sahara but are concentrated in
• Korba is a city and an industrial area in the Indian state
the regions south of it.
of Chhattisgarh. It is famous for Korba Super Thermal
80. Solution: (a) Power Station and Bharat Aluminium Company Ltd.
Exp) Option a is the correct answer. (BALCO).
• Chandrapur is a district in Maharashtra that has copper • Angul is a town and a municipality in the state of
mines. Primary explorations have revealed vast deposits Odisha. The important PSUs of the district are National
of copper ore in four blocks: Thanewasna, Dubarpeth, Aluminium Company (NALCO), National Thermal
Lavari, and Adegaon-Motegaon. The Chandrapur Power Corporation (NTPC) and Mahanadi Coal Field
district has large deposits of coal. The district also has Ltd. (MCL), etc.
limestone mines for the manufacturing of cement. • Belgaum (or Belagavi) is a city in the Indian state of
• Hasan is a district in Karnataka that has copper Karnataka. Rich deposits of Bauxite are found in Belgaum
deposits. The district is also rich in iron ore, manganese district and have led to the creation of the HINDALCO
and granite. Industries of the Aditya Birla Group.

263 PYQ Workbook


GENERAL GEOGRAPHY

Important Tips Dortmund, and Bochum, that have grown together into
a densely populated urban area with a population of over
Aluminium Production: 5 million people. The Ruhr area was one of the European
• India is the fourth largest producer of aluminium in Capitals of Culture in 2010.
the world with a share of around 5.3% of the global
aluminium output. 86. Solution: (b)
• It has nearly 10% of the world’s bauxite reserves. Exp) Option b is the correct answer.

• Odisha is the largest aluminium producing state. It • Copper is an important mineral produced in Katanga,
produces more than half of India’s aluminium. a province of the Democratic Republic of Congo (DRC).
Katanga is home to some of the world’s largest copper
83. Solution: (d) deposits and hosts several mining companies.
Exp) Option d is the correct answer. • Diamond is an important mineral produced in
The following are the states that produce the most of each Kimberley, a city in South Africa. Kimberley is famous
mineral in India: for its historical diamond mines that were the site
of a massive diamond rush in the late 19th century.
• Mineral oil: Gujarat produces the most mineral oil in
Kimberley is also the origin of the Kimberley Process, an
India, accounting for about 42% of the total production
international certification scheme to prevent the trade of
in 2019-20.
conflict diamonds.
• Gypsum: Rajasthan produces the most gypsum in India,
• Gold is an important mineral produced in
accounting for about 81% of the total resources as of
Witwatersrand, a region in South Africa. Witwatersrand
2020.
is the largest gold-producing area in the world and
• Gold: Karnataka produces the most gold in India, has been mined since 1886. Witwatersrand hosts the
accounting for about 21% of the total production in 2020. Witwatersrand Basin, an underground geological
• Bauxite: Odisha produces the most bauxite in India, formation that contains about 22% of all the gold ever
accounting for about 49% of the total production in 2021. mined.
84. Solution: (c) • Silver is an important mineral produced in Butte,
a city in Montana, USA. Butte was once known as the
Exp) Option c is the correct answer.
“Richest Hill on Earth” for its abundant silver and copper
The Semang people are an ethnic-minority group residing deposits. Butte was also a major producer of zinc, lead,
in the Malay Peninsula, predominantly in Malaysia and manganese.
and southern Thailand, not in Indonesia. They inhabit
remote forested areas and are considered nomadic hunter- 87. Solution: (c)
gatherers. The Semang are among the different ethnic groups Exp) Option c is the correct answer.
of Southeast Asia who, based on their dark skin and other Hazaribagh, Gaya and Munger belt contains the richest
perceived physical similarities, are sometimes referred to by deposits of high-quality ruby mica in India. It extends for
the superficial term Negrito. about 150 km in length and 32 km in width from the Gaya
Important Tips district of Bihar to Hazaribagh and Koderma districts of
Jharkhand. The main centres of mica production in this belt
• Berbers, also known as Amazigh or Imazighen, are
are Kodarma, Dhorhakola, Domchanch, Dhab, Gawan, Tisri,
diverse indigenous ethnic groups in North Africa,
Chakai and Chakapathal.
predating Arab migrations. They speak various Berber
languages and are scattered across Morocco, Algeria, 88. Solution: (c)
Libya, Tunisia, Mauritania, Mali, Niger, Burkina Faso, Exp) Option c is the correct answer.
Egypt’s Siwa Oasis, and other regions in North Africa.
The correct matching is as follows:
• The Vedda, also known as Wanniyalaeto, are
indigenous people in Sri Lanka, including sub- A. Eskimo 3. Canada
communities like Coast Veddas, Anuradhapura B. Maasai 4. Kenya
Veddas, and Bintenne Veddas. They are accorded C. Bedouins 2. Saudi Arabia
indigenous status but face assimilation, with many
D. Bushman 1. Botswana
speaking Sinhala instead of their native languages,
which are at risk of extinction.
Important Tips
85. Solution: (b) • The Maasai are a Nilotic ethnic group residing in
Exp) Option b is the correct answer. Kenya and Tanzania. They are among the best-known
local populations internationally due to their residence
The Ruhr basin is a major industrial region along the
course of the Ruhr River in western Germany. It is also near the many game parks of the African Great Lakes
known as the Ruhrgebiet, Ruhrpott, or simply the Ruhr. and their distinctive customs and dress. They speak the
It consists of several large cities, such as Duisburg, Essen, Maa language, but many also use Swahili and English.

PYQ Workbook 264


GENERAL GEOGRAPHY

• The Bedouin, nomadic Arab tribes, historically • Badam Pahar is famous for iron ore. It is located in
inhabited desert regions across the Arabian the Mayurbhanj district of Odisha and is part of the
Peninsula, North Africa, the Levant, and Singhbhum-Orissa iron belt.
Mesopotamia. They originated in the Syrian and • Kodarma is famous for mica. It is located in the
Arabian Deserts, spreading throughout the Arab Hazaribagh district of Jharkhand and is known as the
world with the expansion of Islam. Bedouin culture mica capital of India.
revolves around herding camels and goats, and they are • Mosabani is famous for copper. It is located in the
organized into tribes or clans. Most Bedouins follow Purbi Singhbhum district of Jharkhand and is part of the
Islam, with a minority of Christian Bedouins in the Singhbhum copper belt.
Fertile Crescent. • Rava is famous for petroleum. It is located in the
89. Solution: (b) Krishna-Godavari basin of Andhra Pradesh and is one of
the major oil and gas fields in India.
Exp) Option b is the correct answer.
The Maasai are nomadic pastoralists found in East Africa, 92. Solution: (c)
particularly in Kenya and Tanzania, along the Great Rift Exp) Option c is the correct answer.
Valley, and not Zambia. They are known for their unique Indonesia is the world’s leading coconut producer in
cultural practices and are primarily speakers of the Eastern 2021, with about 17.16 million metric tons of coconuts
Sudanic language called Maa. The Maasai have a fully produced. India is the second largest coconut producer in
nomadic lifestyle, relying on their herds for sustenance, the world, with about 14.3 million metric tons of coconuts
including meat, blood, and milk. They live in kraals, which produced in 2021. Brazil and Philistine are not among the
are circular thornbush fences containing mud-dung houses top ten coconut-producing countries in the world.
and accommodating several families. Marriage customs
include polygyny and the payment of a bride-price in Important Tips
livestock. Coconut Production in India:
Important Tips • The major coconut-producing states in India are
Karnataka, Tamil Nadu, Kerala, Andhra Pradesh, West
• Māori are the indigenous Polynesian people of
Bengal, Odisha, Maharashtra, Gujarat, Assam and
mainland New Zealand, originating from East
Bihar.
Polynesian settlers who arrived in multiple waves
of canoe voyages between 1320 and 1350. Isolated • These states account for more than 95% of the total
for centuries, they developed a unique culture with coconut area and production in the country.
its language, mythology, crafts, and performing arts. • Karnataka is the leading state in terms of coconut
Some Māori migrated to the Chatham Islands, giving production, with about 4.2 million metric tons in
rise to the Moriori, another Polynesian ethnic group in 2021-222.
New Zealand. • Tamil Nadu and Kerala follow closely with about
• The Maya are indigenous people of Mesoamerica, 3.8 million metric tons and 3.3 million metric tons
known for the ancient Maya civilization. Modern respectively
Maya inhabit southern Mexico, Guatemala, Belize,
93. Solution: (b)
El Salvador, Nicaragua, and Honduras. The term
“Maya” is collective, not historically used by the Exp) Option b is the correct answer.
indigenous groups themselves, who had distinct Heroin is an opioid drug made from morphine, a natural
traditions and cultures. Estimated at seven million in substance taken from the seed pod of the various opium
the 21st century, they blend with mestizo cultures in poppy plants grown in Southeast and Southwest Asia,
some regions while preserving their heritage in others. Mexico, and Colombia. Opium is the dried latex of certain
poppy species (e.g. Papaver somniferum L.) that contains
90. Solution: (d) morphine and other alkaloids. Heroin is a semisynthetic
Exp) Option d is the correct answer. product obtained by acetylation of morphine, which involves
The Inuit people, also known as Eskimos, are not found several different types of chemicals and includes many steps.
in Sweden. They inhabit the Arctic and subarctic regions Heroin can be a white or brown powder, or a black sticky
of Greenland, Alaska, Canada, and parts of northern substance known as black tar heroin. It is mainly used as
countries. Their culture is deeply intertwined with the harsh a recreational drug for its euphoric effects, but it can also
northern environment, and they are renowned for their cause serious health problems such as addiction, overdose,
knowledge of surviving in extreme cold. Traditionally, they and infectious diseases.
rely on hunting and fishing for sustenance and have a rich
94. Solution: (d)
cultural heritage.
Exp) Option d is the correct answer.
91. Solution: (d) The Golden Crescent is the name given to one of Asia’s
Exp) Option d is the correct answer. two principal areas of illicit opium production (with the

265 PYQ Workbook


GENERAL GEOGRAPHY

other being the Golden Triangle), located at the crossroads • Gondwana System: This system comprises of
of Central, South, and Western Asia. This space overlaps
sedimentary rocks that were formed during the
three nations, Afghanistan, Iran, and Pakistan, whose
Carboniferous to Jurassic periods. The Gondwana
mountainous peripheries define the crescent. Iraq is not
System is widely distributed in the peninsular India,
part of the Golden Crescent, as it is located in the Middle
East and does not share a border with Afghanistan or especially in the Damodar, Mahanadi, Godavari and
Pakistan. Afghanistan is the world’s largest producer of Narmada valleys. The Gondwana System is known for
opium, accounting for over 90% of the global supply. Iran its coal deposits, which are the main source of energy
and Pakistan are mainly transit and consumer countries, in India. The Gondwana System also contains some
as they are situated along the major trafficking routes from other minerals like iron-ore, copper, uranium, etc.
Afghanistan to Europe and Asia. The Golden Crescent is a • Vindhyan System: This system consists of sedimentary
major source of instability, violence, and corruption in the rocks that are mainly composed of sandstones, shales
region, as well as a public health and security threat for the and limestones. The Vindhyan System is found in
rest of the world. the Vindhyan basin of Madhya Pradesh and Uttar
Important Tips Pradesh and also in some parts of Rajasthan, Bihar and
Jharkhand. The Vindhyan System is poor in mineral
Golden Triangle:
resources, but it has some deposits of glass sand,
• The Golden Triangle in Southeast Asia is the area potash, rock phosphate, etc.
where the borders of Myanmar, Thailand, and Laos
meet at the confluence of the Ruak and Mekong Rivers. 97. Solution: (b)
• Along with the Golden Crescent in Southwest Asia, it Exp) Option b is the correct answer.
has been one of the main sources of opium and heroin • The Haldia Refinery is located in Haldia, West Bengal.
for the global market since the 1950s. It is operated by the Indian Oil Corporation (IOC) and is
• The name “Golden Triangle” was coined by the CIA to one of the largest refineries in Eastern India.
describe the lucrative drug trade in this region. • The Jamnagar Refinery complex, often referred to as
the Reliance Industries Limited (RIL) Jamnagar Refinery,
95. Solution: (a)
is located in Jamnagar, Gujarat. It is one of the largest
Exp) Option a is the correct answer.
refining facilities in the world.
Arizona is a major producer of copper in the United States
• The Kochi Refinery, also known as Bharat Petroleum
and the world. The other pairs are not correctly matched.
Kochi Refinery, is situated in Kochi, Kerala. It is operated
• Karatzas is not a coal producer, but a company that by Bharat Petroleum Corporation Limited (BPCL).
provides consulting services for the shipping industry.
• The Numaligarh Refinery is located in Golaghat,
• Donbas is not a petroleum producer, but a historical,
Assam. It is operated by Numaligarh Refinery Limited
cultural, and economic region in eastern Ukraine that
(NRL), a joint venture between Bharat Petroleum, Oil
has been affected by the Russo-Ukrainian war.
India, and the Government of Assam.
• Iron ore is not produced by Ras Tanura. It is a major oil
terminal in Saudi Arabia. 98. Solution: (a)
96. Solution: (b) Exp) Option a is the correct answer.
Exp) Option b is the correct answer. The Rourkela Steel Plant was set-up in 1959 in the Sundargarh
district of Odisha in collaboration with Germany. The plant
The most mineralized rock system of India is the Dharwar
System. This system is composed of highly metamorphosed receives coal from Bokaro & Jharia coalfields of Jharkhand
sedimentary rocks that are rich in valuable minerals like and iron-ore from Sundargarh and Keonjhar of Odisha.
high grade iron-ore, manganese, copper, lead, gold, etc. Important Tips
The Dharwar System is found in abundance in the Dharwar
• Bhilai Steel Plant was established in Russian
district of Karnataka and also in some parts of Tamil Nadu,
Andhra Pradesh, Madhya Pradesh and Rajasthan. collaboration in Durg district of Chhattisgarh in
1959. The iron ore comes from Dalli- Rajhara mine,
Important Tips Coal comes from Kobra and Kargali Coal field.
Rock system of India: • Visvesvaraya Iron and Steel Plant (VISL) of
• Cuddapah System: This system consists of sedimentary Bhadravati gets its iron ore from the Kamangundi
rocks that are mainly composed of quartzites, shales, or kemmangundi mines. Kemmanagundi is located
limestones and dolomites. The Cuddapah System in Baba Budan hills of Chikmagalur district of
is found in the Cuddapah basin of Andhra Pradesh Karnataka.
and also in some parts of Karnataka, Tamil Nadu and • Tata Steel plant obtains its requirements of iron ore
Chhattisgarh. The Cuddapah System is not very rich from Gurumahisani mines in Mayurbhanj district of
in minerals, but it contains some deposits of barite, Odisha and from the Noamundi mines in Singhbhum
asbestos, corundum, steatite, etc. district of Jharkhand.

PYQ Workbook 266


GENERAL GEOGRAPHY

99. Solution: (b) 20th centuries. It produced many innovative and famous
Exp) Option b is the correct answer. vessels, such as the RMS Queen Elizabeth and the HMS
Hood.
Both (A) and (R) are true, but (R) is not the correct
explanation of (A). C. Ottawa is the capital city of Canada and has a diverse
Assertion (A) is true: Commercial viticulture, or the economy that includes sectors such as banking,
large-scale cultivation of grapes for wine production, is insurance, technology, and public administration. One
specific to regions with Mediterranean climates, which are of its industries is paper production, which dates back
characterized by warm, dry summers and mild, wet winters. to the 19th century when several mills were established
along the Ottawa River.
Reason (R) is true but not the correct explanation of
Assertion (A): The majority of grapes grown in regions with D. Sheffield is a city in England that has a long history of
commercial viticulture are indeed used for wine production. metallurgy and steel-making. It was the second centre
This is a significant portion of the total grape production of cutlery production in England after London, and
in such areas. The Mediterranean region of Europe has a it invented or developed many types of steel, such as
favorable climate for viticulture, with long growing seasons stainless steel, crucible steel, and Sheffield plate.
of moderate to warm temperatures and little seasonal change.
102. Solution: (a)
The region also has a rich diversity of landscapes and soils
that influence the quality and character of the wines. The Exp) Option a is the correct answer.
Mediterranean region of Europe produces about 60% of the The Vindhya rocks are famous sources of red sandstone and
global wine production and consumes about 50% of it. other building material, but they also contain a huge amount
100. Solution: (c) of limestone. Limestone is a sedimentary rock composed
mainly of calcium carbonate, which is used for cement,
Exp) Option c is the correct answer.
lime, and other purposes. The Vindhya rocks have several
A. Pittsburg is known for its iron and steel industry, which limestone formations, such as the Kajrahat Limestone,
developed in the late 19th and early 20th centuries. It was the Fawn Limestone, and the Rewa Limestone. The well-
the leading producer of steel in the United States until known Panna and Golconda diamonds are also found in this
the mid-20th century, when it faced competition from formation.
other regions and countries.
B. Shanghai is one of the largest and most prosperous 103. Solution: (a)
cities in China, with a long history of cotton textile Exp) Option a is the correct answer.
production. It was the center of the Chinese textile The Bhilai Steel Plant is located in Durg district of the
industry in the late 19th and early 20th centuries, when Indian state of Chhattisgarh (not Madhya Pradesh). It was
it attracted foreign investment and trade. Shanghai’s set up with the help of the USSR in 1955.
cotton mills employed millions of workers and produced
a variety of fabrics and garments. Important Tips
C. Dundee is a city in Scotland that was once famous for • The Durgapur Iron and Steel Plant was set up in 1956
its jute textile industry. Jute is a plant fiber that can be with the help of British Companies. It is located in the
used to make coarse fabrics, such as sacks and carpets. Bardhaman district of West Bengal, along the Damodar
Dundee became the global center of the jute trade in River. Production was started here in 1962. It obtains
the 19th century, when it imported raw jute from India iron ore from Singhbum iron ore mines of Jharkhand
and processed it in its mills. Dundee was nicknamed and Keonjhar iron ore mines of Odisha. It obtains
“Juteopolis” for its dominance in the industry. coal from Jharia and Raniganj coalfields, manganese
D. Leningrad, now known as Saint Petersburg, is a city in from Balaghat in Madhya Pradesh, and water from the
Russia that has a strong tradition of shipbuilding. It is Damodar River.
located on the Baltic Sea and has access to several rivers • The Bokaro Steel Plant was established in 1964 with
and lakes. Leningrad was the main base of the Soviet the help of the Soviet collaboration in Jharkhand.
Baltic Fleet and a major center of naval engineering and Production was started here in 1972. It obtains iron ore
construction. It also produced civilian ships, such as from Kiriburu mines, coal from Bokaro, Jharia, and
icebreakers and cargo vessels Kargali coalfield, limestone from the Palamu district,
and water from the Tenu dam across the Damodar
101. Solution: (b) River.
Exp) Option b is the correct answer.
104. Solution: (d)
A. Anshan is a city in China that is known for its iron and
steel industry. It has one of the largest steel producers in Exp) Option d is the correct answer.
China, the Anshan Iron and Steel Group. Pair 1 is correct: The Ramagundam Fertilizers and
B. Glasgow is a city in Scotland that was a major centre of Chemicals Limited (RFCL) was incorporated in 2015 to set
shipbuilding and marine engineering in the 19th and up a natural gas-based ammonia urea complex along with

267 PYQ Workbook


GENERAL GEOGRAPHY

offsite & utility facilities at Ramagundam, Telangana. It is a 107. Solution: (c)


Joint Venture Company of National Fertilizers Limited (NFL), Exp) Option c is the correct answer.
Engineers India Limited (EIL) and Fertilizer Corporation of
A. Coffee is a tropical crop that requires a warm and humid
India Limited (FCIL) (Promoters) and Govt. of Telangana.
climate, rich and well-drained soil, and moderate rainfall.
Pair 2 is correct: Chittaranjan Locomotive Works (CLW) is The Sao Paulo Plateau in Brazil has all these conditions,
the major Electric Locomotive manufacturer in the country. making it ideal for coffee cultivation. Brazil accounts
It is located in West Bengal. Production of steam loco for about a third of the world’s coffee production and
commenced on 26th January 1950. exports.
Pair 3 is correct: Major operations of Bharat B. Jute is a fibre crop that grows well in hot and moist
Aluminium Company Ltd (BALCO) are in the town climates, fertile alluvial soil, and abundant water supply.
of Korba (Chhattisgarh), whereas its mines supplying high The Ganges Delta in India and Bangladesh provides
grade Bauxite are situated at Kawardha and Mainpat of these favourable conditions for jute farming. India is the
Chhattisgarh. world’s largest producer of jute, followed by Bangladesh.
Pair 4 is incorrect: Pipri is not famous for Pesticides. Rihand Jute is used for making ropes, mats, bags, carpets, and
Dam, also known as Govind Ballabh Pant Sagar, is a concrete other products.
gravity dam located at Pipri in Sonbhadra District in Uttar C. Rice is a cereal crop that thrives in warm and wet
Pradesh. climates, low-lying and flat lands, and waterlogged soil.
105. Solution: (b) The Yangtze Plains in China have these characteristics,
making them suitable for rice cultivation. China is the
Exp) Option b is the correct answer.
world’s largest producer and consumer of rice, with about
Negrito refers to a diverse range of indigenous ethnic 30% of the global output. Rice is a staple food for more
groups inhabiting isolated regions of Southeast Asia than half of the world’s population.
and the Andaman Islands, not in the Congo basin. These
D. Wheat is another cereal crop that grows well in cool and
populations, such as the Andamanese peoples, Semang
dry climates, fertile and well-drained soil, and moderate
peoples of Malaysia, and Aeta in the Philippines, are often
rainfall. The Prairies Plains in Canada and the United
characterized by their smaller stature and distinct cultural
States have these features, making them conducive for
practices. Historically, they were hunter-gatherers, but many
wheat farming. Canada and the United States are among
have integrated into mainstream societies, facing challenges
the top wheat exporters in the world, with about 20% of
related to discrimination and socioeconomic disadvantages.
the global share. Wheat is used for making bread, pasta,
Important Tips noodles, cakes, and other products.
• The Norsemen, or Norse people, were an Early
108. Solution: (a)
Middle Ages North Germanic group who spoke
Old Norse. They played a significant role in the Exp) Option a is the correct answer.
Viking Age, with Norse Vikings from Norway mainly A. Copper is mainly produced in Khetri, Rajasthan, which
raiding the northwestern British Isles, Ireland, and is one of the largest copper mines in India.
western Britain, while Danish Vikings focused on B. Gas Plant refers to the Auraiya Thermal Power Station,
eastern Britain. Modern descendants include Danes, which is a gas-based power plant of NTPC in Uttar
Icelanders, Faroe Islanders, Norwegians, and Swedes, Pradesh.
collectively referred to as “Scandinavians” today.
C. Aluminium is produced by BALCO, which operates a
• The Raeti were Alpine tribes whose culture was 0.57 MTPA aluminium smelter in Korba, Chhattisgarh.
linked to the Etruscans. They inhabited Tyrol in
D. Petroleum is refined by BPCL Kochi Refinery, which is
Austria, eastern Switzerland, and the Alpine regions
the largest public sector refinery in India with a capacity
of northeastern Italy before the Roman conquest.
of 15.5 MTPA
The Roman province of Raetia was named after them.
109. Solution: (b)
106. Solution: (b)
Exp) Option b is the correct answer.
Exp) Option b is the correct answer.
A. Anshan is a city in China that is known for its iron and
The Mesabi Range is a mining district in northeastern
steel industry. It is home to the Anshan Iron and Steel
Minnesota that contains large deposits of iron ore, not
Group, one of the largest steel producers in China.
copper. Copper is mainly found in the Keweenaw Peninsula
of Michigan, which is part of the same geological formation as B. Detroit is a city in the United States that is the epicenter
the Mesabi Range. The other options are correctly matched: of the global automotive industry. It is home to the
Donbas Basin is a coal-rich region in Ukraine and Russia, headquarters of General Motors, Ford Motor Company,
Mosul is a city in Iraq that lies near major oil fields, and and Stellantis, among others.
Transvaal is a province in South Africa that has produced C. Hawana is a city in Cuba that is famous for its cigar
most of the world’s gold. industry. It produces some of the finest cigars in the

PYQ Workbook 268


GENERAL GEOGRAPHY

world, using tobacco grown in the nearby Vuelta Abajo oldest and most prolific oil-producing regions is Baku,
region. the capital of Azerbaijan. Baku lies on the Caspian Sea
D. Nagoya is a city in Japan that has a rich history and and has been producing oil since the 7th century BC. It
culture of textile production. Nagoya yuzen is a type of was the world’s first oil boomtown in the late 19th and
early 20th centuries, when it supplied half of the world’s
dyed and painted cloth that is unique to the city and has
oil.
a minimal palette of simple colors.
C. Copper is a metal that is used for electrical wiring,
110. Solution: (a) plumbing, coins, and other purposes. It is found in
Exp) Option a is the correct answer. various types of ore deposits, such as porphyry, skarn,
hydrothermal, and sedimentary. One of the largest and
Eskimo is a term used to refer to two closely related
most impressive copper mines in the world is Bingham
Indigenous peoples: the Inuit and the Yupik, who
Canyon Mine in Utah, USA. Bingham Canyon Mine is
traditionally inhabit Arctic and subarctic regions,
an open pit mine that has been operating since 1906 and
including Canada. The Inuit are part of the Indigenous has produced more than 19 million tons of copper. It is
population in Canada’s northern sectors. While some also the largest man-made excavation and the deepest
individuals find the term Eskimo pejorative, Canada open pit mine in the world.
officially uses the term Inuit to describe these Indigenous
D. Uranium is a radioactive metal that is used for nuclear
people.
power, weapons, and medical applications. It is found in
Important Tips various types of rocks, such as granite, sandstone, shale,
• The Oraon, also known as Kurukh or Dhangar in and conglomerate. One of the first and most important
some regions, are a Dravidian-speaking ethnic uranium mines in the world was Port Radium Mine in
group living primarily in the Chhotanagpur Plateau Northwest Territories, Canada. Port Radium Mine was
and neighboring areas, including Jharkhand, Odisha, discovered in 1930 and produced radium and uranium
Chhattisgarh, and West Bengal in India. Their native for medical research and military purposes. It was also
language is Kurukh, which is part of the Dravidian the source of uranium for the Manhattan Project, which
language family. developed the first atomic bombs during World War II

• The Lapps, historically known as Laplanders, are 112. Solution: (d)


the indigenous Sámi people of Sápmi, encompassing Exp) Option d is the correct answer.
northern parts of Norway, Sweden, Finland, and
Birmingham is a city in England that is known for its
the Kola Peninsula in Russia. They speak the Sámi
industrial heritage, especially in metalworking and
languages, a branch of the Uralic language family. The
manufacturing. It is not a major shipbuilding center,
term “Lapps” is considered offensive, and the Sámi
unlike some other British cities such as Glasgow, Belfast, or
prefer to use their own languages, such as Northern Portsmouth.
Sámi and Sápmi.
• The Gonds, also known as Koitur, are an Important Tips
ethnolinguistic group in India. They speak the Gondi • Detroit is a city in the United States that is famous
language, which belongs to the Dravidian family. for its automobile industry. It is the largest city in
Spread across several Indian states, including Madhya the state of Michigan, which is located in the Great
Pradesh, Maharashtra, Odisha, Chhattisgarh, and Lakes region. Detroit was the home of the “Big Three”
others, they are recognized as a Scheduled Tribe under American car manufacturers: Ford, General Motors,
India’s reservation system. and Chrysler.
• Magnitogorsk is a city in Russia that is one of the
111. Solution: (a) largest producers of iron and steel in the world. It is
Exp) Option a is the correct answer. located in the Ural Mountains, near the border with
Kazakhstan. Magnitogorsk was founded in 1929 as a
A. Iron ore is a metal that is used to make steel and other planned industrial city, based on the model of Gary,
products. It is found in many places around the world, Indiana.
but one of the largest and most famous iron ore deposits
• Johannesburg is a city in South Africa that is the
is the Mesabi Range in Minnesota, USA. The Mesabi
largest and most populous city in the country. It is
Range has been mined since the late 19th century and
also the economic and financial hub of South Africa
has produced over 3 billion tons of iron ore.
and Africa. Johannesburg was founded in 1886 as a
B. Petroleum is a liquid fossil fuel that is used for energy, gold mining town, after the discovery of gold on the
transportation, and manufacturing. It is formed from Witwatersrand, a ridge of hills that runs through the
the remains of ancient plants and animals that were city.
buried under layers of sediment and subjected to heat and
pressure over millions of years. Petroleum is extracted 113. Solution: (b)
from underground reservoirs or wells, and one of the Exp) Option b is the correct answer.

269 PYQ Workbook


GENERAL GEOGRAPHY

Bauxite is a sedimentary rock with a relatively high 116. Solution: (c)


aluminium content. It is the world’s main source of Exp) Option c is the correct answer.
aluminium and gallium. Bauxite consists mostly of the
Apple is a temperate fruit that grows well in cool and
aluminium minerals like gibbsite, boehmite and diaspore,
mixed with iron oxides, clay minerals and other impurities. moderate climates, such as those found in Europe, North
America, and some parts of Asia. Apple trees require a
Bauxite is formed by lateritic weathering of various silicate
period of cold dormancy to produce fruit.
rocks or by dissolution of carbonate rocks. The lateritic
bauxites are found mostly in the tropics, while the carbonate Walnut, coconut, and cashew are all tropical fruits that
bauxites occur in Europe, Guyana, Suriname and Jamaica. grow in warm and humid climates, such as those found in
Africa, South America, and Southeast Asia. These fruits do
Important Tips
not need a cold period to produce fruit.
Bayer Process:
Important Tips
• Bayer process is a method of refining bauxite, which
is the most important ore of aluminium, into alumina, Apple production in India:
which is aluminium oxide. • Apple fruit production in India is mainly concentrated
• The Bayer process was developed by Carl Josef Bayer in in the states of Jammu & Kashmir, Himachal Pradesh,
1888 and is still used today in the aluminium industry. and Uttarakhand.
• The Bayer process involves four main steps: digestion, • These states account for more than 99% of the total
clarification, precipitation, and calcination. apple production in the country.
• The Bayer process is also a source of gallium, which is • In 2021-22, India produced 2.44 million metric tonnes
a rare metal used in electronics and optics. of apples, which was a slight increase from the previous
year.
114. Solution: (c) • India ranks sixth in the world in terms of apple
Exp) Option c is the correct answer. production, after China, USA, Turkey, Poland, and
According to the data from the US Geological Survey Italy.
(USGS), the largest producer of silver in the world in 2022
was Mexico with 6,300 metric tons of silver, accounting 117. Solution: (c)
for 24.2% of the global production. China was the second- Exp) Option c is the correct answer.
largest producer with 3,600 metric tons, followed by Peru Jharkhand is the state where combustion of underground
with 3,100 metric tons. Chile ranked fourth with 1,600 coal occurs in India. This phenomenon is also known as coal-
metric tons of silver. seam fire, which is a burning of an outcrop or underground
Important Tips coal seam. One of the most prominent examples of coal-seam
Silver Production: fire in India is the Jharia coalfield, which has been burning
continuously for over a century. The fire has caused severe
• India is not a major producer of silver in the world.
environmental and health problems for the residents of the
• The major production comes from Zawar mines in area, as well as economic losses for the coal industry. The
Udaipur district of Rajasthan, where silver is obtained fire is also a major source of greenhouse gas emissions and
as a by-product during the concentration and smelting air pollution.
of galena ore in Hindustan Zinc Smelter.
• India’s share in the global silver mine production was 118. Solution: (a)
only 0.1% in 20222 Exp) Option a is the correct answer.

115. Solution: (a) The largest producer of copper in India is Madhya Pradesh.
It accounts for 53 percent of copper production in India. The
Exp) Option a is the correct answer.
other states that produce copper in India are Andhra Pradesh,
Kalgoorlie is a city in the Goldfields–Esperance region of Gujarat, Haryana, Karnataka, Maharashtra, Meghalaya,
Western Australia, located 595 km east-northeast of Perth Orissa, Sikkim, Tamil Nadu, Uttarakhand and West Bengal.
at the end of the Great Eastern Highway. It is the principal
settlement of the East Coolgardie goldfield, on the western Important Tips
fringe of the Nullarbor Plain and the Great Victoria Desert. Some of the famous copper mines in India are:
Kalgoorlie was established in 1893 during the Western
• Malanjkhand Mine: The Malanjkhand Mine is a
Australian gold rushes and has been a major Centre of gold
surface and underground mine located in Madhya
mining ever since.
Pradesh. It is owned by Hindustan Copper
Kalgoorlie is home to the Super Pit, Australia’s largest open-
• Khetri Mine: Located in Rajasthan, the Khetri Mine
cut gold mine, which produces about 800,000 ounces of
is owned by Hindustan Copper. The underground
gold per year. Kalgoorlie also has a rich history and culture,
with many heritage buildings, museums, and festivals that mine produced an estimated 6.732 Thousand tonnes
celebrate its mining heritage. of copper in 2020.

PYQ Workbook 270


GENERAL GEOGRAPHY

• Banwas Mine: The Banwas Mine is located in • Korba in Chhattisgarh has importance for its Aluminium
industry. Bharat Aluminium Company Ltd. (BALCO) set
Rajasthan. It is owned by Hindustan Copper
up its plant at Korba (Chhattisgarh) in 1965. The high-
• Kendadih Mine: Owned by Hindustan Copper, the grade Bauxite is supplied from Kawardha and Mainpat
Kendadih Mine is an underground mine located in Bauxite mines of Chhattisgarh.
Jharkhand. • Jamshedpur is famous for Iron and steel industries.
119. Solution: (d) The first steel plant by TATA was set up in Jamshedpur. It
is known as the Pittsburg of India.
Exp) Option d is the correct answer.
• Malanjkhand is famous for copper mines. It is an open-
Iran is the world’s leading producer of saffron, producing pit copper mine in India. It is located near the town of
430 tons in 2019. That year, India was the second largest Malanjkhand in the district Balaghat of Madhya Pradesh.
saffron producer with only 22 tons of production. Saffron is It is the single largest copper deposit of India with nearly
a spice derived from the flower of Crocus sativus, commonly 70% of the country’s reserve and contributing around
known as the “saffron crocus”. It has a rich golden-yellow 80% to Hindustan Copper Limited (HCL) total copper
hue and a distinctive aroma and flavor. It is used for various production.
purposes such as food, medicine, dye, and perfume. Saffron
is also one of the most expensive spices in the world, as it 123. Solution: (d)
requires a lot of labor and land to produce. It takes about Exp) Option d is the correct answer.
150,000 flowers to produce 1 kg of saffron. Iran has a long Digboi is the oldest oil field in India, situated in the
history and tradition of saffron cultivation and trade, dating northeast of Tipam hills in the Dibrugarh district of Upper
back to ancient times. Iran produces about 90% of the global Assam. The oil-bearing strata cover an area of about 13 sq
saffron supply, mainly in the provinces of Khorasan, Fars, km where oil is available at 400 to 2,000-metre depth. Over
Isfahan, and Kerman. Iran exports saffron to more than 50 800 oil wells have been drilled in Digboi so far. The first oil
countries, including Spain, India, China, UAE, and France. production started in India in 1889 near Digboi. The first
refinery was also started at Digboi in 1893. Digboi is India’s
120. Solution: (b) oldest operating refinery and one of the oldest operating
Exp) Option b is the correct answer. refineries in the world. It has been termed as the “Gangotri
of the Indian Hydrocarbon sector”.
Kaimur Plateau is famous for limestone, a type of hard
white sedimentary rock that is used for building or for 124. Solution: (c)
making cement. The Kaimur Plateau is situated in Bihar and Exp) Option c is the correct answer.
extends in the eastern zone of the Vindhya Range. It is an
Silver is found in India mainly as a by-product of zinc and
undulating table land that has a height of 1,490 feet (450 m)
lead mining. It is not found in the state of Gujarat. The
above sea level at Rohtasgarh. The plateau also has many
total reserves/resources of silver in the country as of 1.4.2015
waterfalls along its edges, such as Devdari Falls, Telharkund in terms of metal content was estimated at 29,982 tonnes, of
Falls, and Kuaridah Falls. which 7,172 tonnes are under ‘Reserves’ and 22,810 tonnes
121. Solution: (d) are under the ‘Remaining Resources’. By States, Rajasthan
accounted for about 87% of reserves/ resources in terms of
Exp) Option d is the correct answer. ore, Jharkhand 5%, Andhra Pradesh 3% and Karnataka
The ‘Rust Bowl’ of the USA is associated with the Pittsburgh 2%. Madhya Pradesh, Uttarakhand, Odisha, Meghalaya,
region. This term refers to a region in the United States that Sikkim, Tamil Nadu and Maharashtra together shared 3%
was once a major industrial and manufacturing center but ore reserves/remaining resources.
experienced a decline in the late 20th century. The Pittsburgh
125. Solution: (a)
region was known for its steel production, which made it a
vital part of the American economy and defense. However, Exp) Option a is the correct answer.
due to various factors such as globalization, automation, Johannesburg is famous for gold mining. It is the largest
environmental regulations, and labor disputes, the steel city in South Africa and the centre of the country’s gold
industry collapsed and left many factories abandoned and industry. Johannesburg was founded in 1886, when a large
rusting. The term ‘Rust Bowl’ is a play on the term ‘Dust gold reef was discovered on a farm called Langlaagte. The
Bowl’, which was another name for the Great Plains region city soon attracted thousands of fortune seekers from around
that suffered from severe drought and soil erosion during the world, who established mines along the Witwatersrand
the 1930s. Main Reef.
Important Tips
122. Solution: (d)
Gold Mining in India:
Exp) Option d is the correct answer.
• India has a total of 501.83 million tonnes of gold ore
• Bengaluru is famous for Aircraft industry. The hub
(primary) estimated by the National Mineral Inventory,
of the nation’s aviation manufacturing industry is
out of which 17.22 million tonnes are classified as
at Bangalore which has had a 65% share in this sector
since 1924. Hindustan Aeronautics Limited (HAL) is reserves and 484.61 million tonnes as remaining
also headquartered in Bangalore. resources

271 PYQ Workbook


GENERAL GEOGRAPHY

• In the twentieth century, outside the city of Bangalore, Maritime Board or the State Government in which they are
the Kolar Gold Fields were the major gold mining located.
operations in the country. They produced more than
800 tonnes of gold during their 120-year history but
closed in 2001 due to declining grades and increasing
costs.
• According to the Indian Mineral Federation, only about
13% of India’s land with mineral resource potential has
been explored in detail.

126. Solution: (c)


Exp) Option c is the correct answer.
Shahtoosh is a type of wool obtained from the fur of the
chiru, also known as the Tibetan antelope, which lives in
the high-altitude plateaus of China, India, and Nepal.
However, shahtoosh shawls are mainly produced in the
Kashmir region of India and Pakistan, where the wool is
smuggled from China. Shahtoosh shawls are considered
to be the most attractive, warmest, and lightest woolen
products in the world, but they are also illegal and unethical,
as they involve the killing of an endangered species. The
trade and possession of shahtoosh shawls have been banned
130. Solution: (a)
by international law under CITES since 1979.
Exp) Option a is the correct answer.
127. Solution: (a)
Haematite and Magnetite are important iron ore minerals
Exp) Option a is the correct answer. abundant in India, while Limonite and Siderite exist in
Tin is found mostly in placer deposits. Placer deposits minor quantities. Haematite is widely distributed in states
are formed by the erosion and transport of primary tin like Jharkhand, Odisha, Chhattisgarh, Andhra Pradesh, and
deposits by water or ice. Tin is a relatively rare element Karnataka, and Magnetite is found in Karnataka, Andhra
in the Earth’s crust, with approximately 2 ppm (parts per Pradesh, Goa, Tamil Nadu, and Maharashtra.
million), compared to iron with 50,000 ppm. Tin is usually
concentrated in cassiterite (SnO2), which is a tin oxide 131. Solution: (c)
mineral that is resistant to weathering and can be found in Exp) Option c is the correct answer.
riverbeds, beaches, or offshore areas. Primary tin deposits Kolkata is the only riverine major port in India. It is
are associated with intrusive granite rocks, which are formed located on the Hooghly River, which is a distributary of
when magma cools and solidifies beneath the Earth’s surface. the Ganges River.
128. Solution: (b) Important Tips
Exp) Option b is the correct answer. Some Key facts about Ports in India:
Pitchblende is a mineral that contains uranium and • Total no. of Major Seaports in India – 13.
radium. Radium is extracted from pitchblende by dissolving • Largest Port in India – Mumbai Port, also Busiest
the mineral in nitric acid, precipitating the sulphates of Port in India and largest Natural Port and Harbor
radium and other metals, and separating radium from barium In India.
by fractional crystallization. This method was developed
• Oldest Port of India – Lothal Port (Gujarat).
by Marie and Pierre Curie, who discovered radium in 1898
from ore mined at Jáchymov. Pitchblende is also known as • First Corporate Port in India – Ennore Port.
uraninite and is the most important source of uranium. • First Private Port in India – Port Pipavav
(Gujarat).
129. Solution: (d)
• Only major Riverine Port in India – Kolkata Port.
Exp) Option d is the correct answer.
• Iron Ore Exporting Port in India – Mormugoa Port.
Statement 1 is incorrect: India has 13 major seaports and
around 200 minor seaports along its vast coastline. • Largest Container Port in India – Jawaharlal Nehru
Port.
Statement 2 is correct: The major ports in India are indeed
under the administrative control of the Ministry of Ports, • Largest Artificial Port in India – Jawaharlal
Shipping and Waterways. Nehru Port.
Statement 3 is correct: Minor ports in India are generally • Only Natural Seaport in the shores of Bay of Bengal
under the jurisdiction and control of the respective State – Vizag Port.

PYQ Workbook 272


GENERAL GEOGRAPHY

• First Major Port commissioned after Independence • Sukinda: It is located in Jajpur district, Odisha.
– Paradip Port. Chromium mineral is found in Sukinda Mines. Odisha
accounts for 98% of the Chromium of the total Chromite
• Biggest port of India based on the volume of
reserves of the country. National Highway NH-200 passes
freight handled – Kandla Port.
through Sukinda.
• Indian State with the Largest no. of Major Ports –
• Zawar: It is located in Udaipur, Rajasthan. Zinc and
Tamil Nadu.
Lead are found in the mines of Zawar Zawar township
132. Solution: (a) created by mining company Hindustan Zinc Limited.
Exp) Option a is the correct answer. • Hutti: It is a Gold mine. It is located in Raichur,
Karnataka. This mine is probably one of the most ancient
The Natural Population Change is calculated by subtracting
metal mines in the world, dating to the Pre-Ashokan
the death rate from the birth rate. This calculation helps
period, the ancient miners having worked down to a
determine the net increase or decrease in the population
depth of over 2300 feet.
due to natural factors (births and deaths) without
considering migration. 136. Solution: (a)
133. Solution: (b) Exp) Option a is the correct answer.
Exp) Option b is the correct answer. Coral reefs are underwater structures made of calcium
carbonate secreted by corals, which are marine animals
The 2011 Census of India divided the working population
that live in symbiosis with algae called zooxanthellae. The
into four major categories: Cultivators, Agricultural
necessary conditions for the growth of coral reefs are:
labourers, Household industrial workers and Other
workers. Domestic workers are not included in any of these • Photic conditions: Corals need to grow in shallow water
categories. They are considered to be a part of the informal where sunlight can reach them. Sunlight is essential for
sector, and they are not covered by the same labor laws as the photosynthesis of the zooxanthellae, which provide
other workers. nutrients and oxygen to the corals.
• Clean and sediment-free water: Corals need clear water
Important Tips
that lets sunlight through. Sediment and pollution can
• Cultivators: People engaged in agricultural activities, reduce the light penetration, smother the corals, and
including farming and cultivation of crops. interfere with their feeding and reproduction.
• Household industrial workers: Individuals involved • Sea salinity of 6%: This is not a necessary condition for
in small-scale household industries, such as handloom the growth of coral reefs. Sea salinity is the amount of
weaving, handicrafts, and cottage industries. salt dissolved in seawater, which is usually around 3.5%.
• Agricultural laborers: Workers employed in Corals can tolerate a range of salinity levels, but they
agricultural activities but not involved in cultivation prefer stable conditions.
or owning land.
• Tropical sea water with temperature of 20°C - 21°C:
• Other workers: This category includes individuals This is a necessary condition for the growth of coral reefs.
engaged in various non-agricultural activities, such as Most corals are found in tropical and subtropical regions,
manufacturing, construction, trade, services, etc. where the water temperature is between 20°C and 32°C.
134. Solution: (c) 137. Solution: (c)
Exp) Option c is the correct answer. Exp) Option c is the correct answer.
According to the 2011 Census of India, the average density Statement 1 is correct: Tharu tribes are found in Uttar
of population for the whole country was 382 persons per Pradesh and Uttarakhand. The Tharu are an ethnic group
square kilometer, which was an increase from 324 persons indigenous to the Terai region, which is a lowland belt along
per square kilometer in 2001. the southern border of Nepal and northern border of India.
According to the 2011 Census of India, the states with the Statement 2 is correct: Irula, Chanchu and Sumali tribes
highest density of population were: are found in Kerala. The Irula are a Dravidian-speaking
1. Bihar: 1106 persons per square kilometer tribe who live mainly in the states of Tamil Nadu and Kerala.
2. West Bengal: 1028 persons per square kilometer They are traditionally hunter-gatherers, snake-catchers, and
3. Kerala: 860 persons per square kilometer rat-catchers.
4. Uttar Pradesh: 828 persons per square kilometer Statement 3 is incorrect: Garasia are an ethnic group who
live mainly in the states of Rajasthan, Gujarat, Madhya
135. Solution: (c) Pradesh, and Maharashtra. They speak various Bhil
Exp) Option c is the correct answer. languages and follow Hinduism or animism.
• Amjhore: It is located in Bihar. Pyrite mineral is found Statement 4 is correct: Gaddi tribes are nomadic herders of
in the mines of Amjhore. Jammu & Kashmir and Himachal Pradesh. The Gaddi are

273 PYQ Workbook


GENERAL GEOGRAPHY

an ethnic group who inhabit the western Himalayan regions This is because a person who can only read is not able to
of Jammu & Kashmir, Himachal Pradesh, and Uttarakhand. communicate effectively, and is therefore not considered
literate.
138. Solution: (d)
Statement 2 is correct: As per census 2011, a person
Exp) Option d is the correct answer.
aged seven and above, who can both read and write with
Statement 1 is correct - Kolkata Port is the only riverine understanding in any language, is treated as literate.
major port of India, also known as Syama Prasad Mukherjee
Port. 141. Solution: (c)
Statement 2 is correct - The port of Cochin is located on Exp) Option c is the correct answer
Willingdon Island in Cochin, Kerala. It is connected to Statement 1 is correct: Thermal power stations typically use
Cochin by Venduruthy Bridge. fossil fuels as their primary energy source.
Statement 3 is incorrect - Maharashtra has two major ports, Statement 2 is correct: It explains the process of how
Mumbai Port and Jawaharlal Nehru Port, both located in electricity is generated in a thermal power station. It
Mumbai harbour. mentions that the fuels are burned to heat water and turn
Statement 4 is correct - Mundra Port is India’s largest it into steam. The steam then goes through a turbine, which
private port, located in Kutch district, Gujarat, on the spins and generates electricity. This is known as the steam
north shores of the Gulf of Kutch. turbine power generation process, which is widely used in
Important Tips thermal power plants.

Major Ports: Therefore, statement 2 provides an explanation for statement


1, as it describes the specific mechanism by which the
• India has a total of 13 major ports located along
fuels are utilized to generate electricity in a thermal power
its coastline.
station.
• These major ports are under the jurisdiction of
the central government and are managed by the 142. Solution: (c)
respective port trusts or authorities. Exp) Option c is the correct answer
• Some of the major ports in India include Mumbai Statement 1 is correct: Vishakhapatnam,AP is a landlocked
Port Trust, Jawaharlal Nehru Port Trust (Navi port in India.The port which is encircled by land from all
Mumbai), Chennai Port Trust, Kolkata Port sides with a water passage towards sea or ocean is termed as
Trust, Paradip Port Trust, and Visakhapatnam land-locked port.
Port Trust. Statement 2 is correct: Deendayal Port, formerly known as
Minor Ports: Kandla Port, is located in Kandla, Gujarat. It is a tidal port,
• India also has numerous minor ports that are meaning it experiences the rise and fall of tides. This tidal
smaller in size and handling capacity compared to movement allows ships to enter and exit the port based on
major ports. the tidal cycle.

• These minor ports are usually managed by 143. Solution: (c)


state governments, private operators, or local Exp) Option c is the correct answer
authorities.
By displaying both age-groups and sex indicators,
• Minor ports are often located in natural harbors, population pyramids provide a comprehensive overview
estuaries, and coastal areas, serving as important of the age and sex composition of a population, facilitating
regional trade hubs. the analysis of demographic trends and patterns.
• Some notable minor ports in India include Kandla
Important Tips
Port (Gujarat), Mormugao Port (Goa), Mangalore
Port (Karnataka), Tuticorin Port (Tamil Nadu), Population Pyramid:
and Haldia Port (West Bengal). • A population pyramid, also known as an age-sex
pyramid, is a graphical illustration of the distribution
139. Solution: (b) of a population (typically that of a country or region of
Exp) Option b is the correct answer. the world) by age groups and sex.
Gujarat is the leading producer of salt in India. The state • It typically takes the shape of a pyramid when the
accounts for about 45% of India’s salt production. The other population is growing. The population pyramid is
major producers of salt in India are Rajasthan, Tamil Nadu, divided into two halves, with the male population on
and Andhra Pradesh. the left and the female population on the right.
140. Solution: (c) • The age groups are typically divided into five-year
Exp) Option c is the correct answer intervals, starting with the youngest age group at the
bottom and the oldest age group at the top.
Statement 1 is correct: According to the Census 2011, a
person who can only read but cannot write, is not literate.

PYQ Workbook 274


GENERAL GEOGRAPHY

• The width of each bar in the population pyramid • Andamanese: Languages spoken by indigenous
represents the number of people in that age group and communities in the Andaman and Nicobar Islands,
sex. The height of the pyramid represents the total such as Great Andamanese and Jarawa.
population.
146. Solution: (c)
• A population pyramid with a broad base and a
narrow top indicates a growing population, while a Exp) Option c is the correct answer.
population pyramid with a narrow base and a broad According to Tamil Literature
top indicates a declining population. • Maruta Makkal - Ploughmen living in villages and
inhabiting fertile tracts
• Kuravan Makkal - Hill people who told people their
fortunes
• Mullai Makkal - Pastoralist, Shepherds.
• Neytal Makkal - Fishing people living in coastal villages
called pattinam.
• Palai Makkal - People of the dry plains.

147. Solution: (d)


Exp) Option d is the correct answer.
• Detroit, also known as the Motor City is the largest
city in the midwestern state of Michigan in the US
144. Solution: (b) and is recognized as the historic heart of the American
automotive industry. Detroit is famous for its thriving
Exp) Option b is the correct answer car industries located there.
Water in usable condition is considered an exhaustible • Antwerp is a port city of Belgium which is well known for
but renewable natural resource, as it has the potential for its diamond industry, cutting and trading of diamonds.
replenishment but can also be depleted and degraded
• Tokyo, the capital city of Japan is famous for its
through human activities and environmental factors.
Shipbuilding market.
145. Solution: (c) • Harbin, the capital of Heilongjiang, China’s
Exp) Option c is the correct answer. northernmost province is the country’s gateway to trade
with Russia and hosts the China-Russia Expo every year.
The Indo-Aryan group is the largest linguistic group in
Since the 1950s it has become a hub for mining and steel
India. Indo-Aryan languages, which are part of the broader
production.
Indo-European language family, are spoken by a substantial
portion of the Indian population. These languages are 148. Solution: (a)
predominantly used in the northern and central regions Exp) Option a is the correct answer.
of the country, encompassing major languages like Hindi,
• Vishakhapatnam is known for its major port and
Bengali, Punjabi, Gujarati, and Marathi, among others.
transportation activities, making it a suitable example of
With their widespread usage, these languages are spoken by a transport town.
millions of people residing in various states and regions of
• Bhilai is famous for its steel plant and industrial
India.
activities, making it an industrial town.
Important Tips • Singrauli is an important mining area in India, known
Other linguistic groups in India: for its coal reserves and mining operations, hence it is
• Dravidian: Spoken mainly in South India, including classified as a mining town.
languages like Tamil, Telugu, Kannada, and • Ambala is a well-known garrison town and cantonment
Malayalam. in India, housing various military establishments, making
• Austroasiatic: Languages spoken by tribal it an example of a garrison cantonment town.
communities in Central and Eastern India, such as 149. Solution: (a)
Santali, Ho, and Mundari.
Exp) Option a is the correct answer.
• Tibeto-Burman: Languages spoken in Northeast
India, including Assamese, Manipuri, and Bodo. The proportion of the Scheduled Tribes to the total
population of the States/Union Territories is highest in
• Sino-Tibetan: Languages spoken in parts of Northeast
Mizoram (94.4%) and Lakshadweep (94.8%) followed
India and the Himalayan region, like Tibetan and
by Nagaland (86.5%), Meghalaya (86.1%), Chhattisgarh
Sherpa.
(30.6%), Jharkhand (26.2%) and Odisha (22.8%).

275 PYQ Workbook


GENERAL GEOGRAPHY

Population of ST : Madhya Pradesh (1,53,16,784); Assam


• Manchester - Located in Greater Manchester, United
(38,84,371), Bihar (13,36,573), Odisha (95,90,756) as per Kingdom.
2011 census.
• Ulsan - Located in Ulsan Metropolitan City, South
150. Solution: (d) Korea.
Exp) Option d is the correct answer. • Milan - Located in Lombardy, Italy.

The ‘blue mountains’ refer to the Nilgiri Hills, which are 152. Solution: (d)
located in the Western Ghats of southern India. The Toda
Exp) Option d is the correct answer.
tribe is an indigenous tribe that inhabits the Nilgiri Hills,
Indian Railways is not the largest railway network in the
particularly the Nilgiri District of Tamil Nadu.
world. While it manages a significant railway network, it
Important Tips ranks as the fourth-largest in terms of size, following the
• Lambadas: The Lambadas, also known as Banjaras or railway networks of the USA, China, and Russia. The Indian
Gypsies, are a nomadic tribe found in various parts railway network is well-connected throughout the country,
of India. Lambadas were known for their itinerant except in the Himalayan regions and north-east India.
lifestyle and were involved in the transportation of
153. Solution: (d)
goods using bullock carts.
Exp) Option d is the correct answer.
• Gonds: The Gonds are one of the largest tribal
communities in India, primarily found in central and The tropical climate in peninsular India contributes to its
southern regions of the country. suitability for sugar production, resulting in higher yields
• Jarawas: The Jarawas are an indigenous tribe residing per hectare of land compared to other regions. Peninsular
in the Andaman Islands of India. They are one of the India also exhibits a higher content of sucrose, making it
indigenous tribes that have had minimal contact with conducive to sugar production. Additionally, the crushing
the outside world. season in peninsular India is almost twice as long as that
in Northern India. As a result, the majority of sugar mills
151. Solution: (c) in peninsular India are concentrated in Maharashtra and
Exp) Option c is the correct answer. Tamil Nadu.
The correct match is (c) 4, 1, 3, 2. 154. Solution: (a)
A. Leipzig: Leipzig is an industrial region located in Exp) Option a is the correct answer
Germany. It is known for its manufacturing and trade The Rajiv Gandhi Centre for Biotechnology is located in
activities. Thiruvananthapuram, Kerala, not Kolkata.
B. Detroit: Detroit is an industrial region located in the
155. Solution: (c)
United States. It is famously known as the center of the
American automobile industry. Exp) Option c is the correct answer
C. Lorraine: Lorraine is an industrial region located in Statement 1 is incorrect: The headquarters of the Central
France. It has a rich history of steel production and is Inland Water Transport Corporation is located in Kolkata,
known for its metallurgical industries. not Delhi.

D. Cumberland: Cumberland is an industrial region Statement 2 is correct: The headquarters of the Inland
Waterways Authority of India (IWAI) is located in Noida,
associated with United Kingdom.
Uttar Pradesh.
Important Tips Statement 3 is incorrect: The National Inland Navigation
Some of the most famous industrial regions across the Institute (NINI) is located in Patna, Bihar not Kolkata.
world: Statement 4 is correct: The first national inland waterway
• Silicon Valley - Located in California, United States. in India was between Haldia (West Bengal) and Prayagraj
• Ruhr Valley - Located in North Rhine-Westphalia, (earlier called as Allahabad).
Germany.
156. Solution: (c)
• Pearl River Delta - Located in Guangdong Province,
Exp) Option c is the correct answer
China.
• Kansai Industrial Region - Located in the Kansai area Statement 1 is correct: The peninsular India is made up of
of Japan. old rocks that have been folded and uplifted, which created
conditions favorable for the formation of these minerals.
• Detroit - Located in Michigan, United States.
Statement 2 is correct: The statement is supported by
• Bangalore - Located in Karnataka, India.
geological evidence, such as the similar rock formations
• Sao Paulo - Located in Sao Paulo State, Brazil. and fossils found in these continents.

PYQ Workbook 276


GENERAL GEOGRAPHY

157. Solution: (d) 1. Koyali: Koyali refinery is located in Gujarat, which is in


Exp) Option d is the correct answer the western part of India. Therefore, it is the westernmost
refinery among the given options.
Statement 1 is correct: The chemical industry has a long
history in India and is considered one of the oldest industries 2. Kochi: Kochi refinery is located in Kerala, which is
in the country. towards the southwestern part of India. While it is not
as westward as Koyali, it is still more westward than the
Statement 2 is correct: The dyestuff sector is a significant
segment of the chemical industry in India. It is responsible remaining options.
for the production of dyes and pigments used in various 3. Panipat: Panipat refinery is located in Haryana, which is
industries, including textiles, paints, and printing inks. in the northern part of India. It is situated more eastward
Statement 3 is correct: Dyestuffs are used extensively in the compared to Koyali and Kochi.
textile sector for dyeing and coloring fabrics. 4. Mathura: Mathura refinery is located in Uttar Pradesh,
which is in the northern part of India. It is the easternmost
158. Solution: (b)
refinery among the given options.
Exp) Option b is the correct answer.
162. Solution: (d)
• Bongaigaon is a place known for its petrochemical
industry. It is located in the state of Assam, India. Exp) Option d is the correct answer.
• Koraput is associated with the manufacturing of aircraft Statement 1 is correct: High death rate and birthrate,
engines. It is a district in the state of Odisha, India. low growth rate describes the initial stage of demographic
transition when both birth rates and death rates are high,
• Pinjore is famous for machine tools factory. It is located
in the Panchkula district of Haryana, India. resulting in a relatively low growth rate.

• Sirpur is known for its paper industry. It is a small town Statement 4 is correct: Low death rate and birthrate,
located in the Mahasamund district of Chhattisgarh, low growth rate describes the fourth and final stage of
India. demographic transition. In this stage, both the death rate
and birth rate are low, resulting in a low growth rate or even
159. Solution: (c) population decline.
Exp) Option c is the correct answer Important Tips
• Kalinganagar Steel Plant is located in Jajpur district of • Demographic transition (DT) refers to the transition
Odisha. from high birth and death rates to lower birth and
• Vijaynagar Steel Plant is located in Bellary district of death rates as a country or region develops from a pre-
Karnataka. industrial to an industrialized economic system.
• Salem Steel Plant is located in Salem district of Tamil • The theory was proposed in 1929 by the American
Nadu. demographer Warren Thompson, who observed
• Durgapur Steel Plant is located in Durgapur district of changes, or transitions, in birth and death rates in
West Bengal. industrialized societies over the previous 200 years.
• Most developed countries have completed the
160. Solution: (b) demographic transition and have low birth rates;
Exp) Option b is the correct answer. most developing countries are in the process of this
The reason behind Gujarat being the leading producer of transition.
salt in India is the long duration of hot and dry conditions.
Gujarat experiences a prolonged period of high temperatures 163. Solution: (a)
and low humidity, which is favourable for the evaporation Exp) Option a is the correct answer.
of seawater and the crystallization of salt. This climatic
Vishakhapatnam, also known as Visakhapatnam or Vizag,
condition creates an ideal environment for salt production.
is a major port city located in Andhra Pradesh, India. It is
• While Gujarat does have a long coastline and presence of situated along the coast of the Bay of Bengal. Although it is
gulf areas, these factors alone do not directly contribute a harbor, it is considered a land-locked harbor because it is
to its leading position in salt production. protected by the natural features of the coastline and does
• The crucial factor is the hot and dry climate, which not have a direct open access to the open sea.
allows for the efficient evaporation of seawater and the
Important Tips
formation of salt pans.
• Ennore: Ennore Port, officially known as Kamarajar
161. Solution: (a) Port Limited, is located in Ennore near Chennai in the
Exp) Option a is the correct answer. state of Tamil Nadu, India. It is a relatively newer port
The correct arrangement of the four oil refineries of India that was developed to cater to the growing demand for
from west to east is: handling coal, petroleum, and other bulk cargo.

277 PYQ Workbook


GENERAL GEOGRAPHY

• Mumbai Port, also known as Jawaharlal Nehru Port, Hematite is the most common and abundant type of iron
is situated on the western coast of India in Mumbai, ore found in India. It accounts for about 80 per cent of the
total iron ore reserves in the country. Hematite is reddish
Maharashtra. It is the largest port in India in terms of
in colour and has a high metallic content of 65-70 per
container traffic and is a vital gateway for international
cent. It is mainly found in the states of Odisha, Jharkhand,
trade.
Chhattisgarh, Andhra Pradesh, Karnataka, and Goa.
• Haldia: Haldia Port is located on the Haldia River in Hematite is also known as the best quality iron ore and is
the state of West Bengal, India. It is an inland port that used for steel production.
is connected to the Bay of Bengal through the Hooghly
River. Haldia Port serves as a major trade gateway for Important Tips
the eastern and northeastern regions of India. • Magnetite: It is the most valuable type of iron ore in
India. It has a high metallic content of more than 70
164. Solution: (c)
per cent. It is mainly found in the states of Karnataka,
Exp) Option c is the correct answer. Andhra Pradesh, Rajasthan, Tamil Nadu and Kerala.
The lengths of the pipelines are as follows: • Limonite: It is an inferior type of iron ore with a low
• Hajira-Bijapur-Jagdishpur gas pipeline: This pipeline is metallic content of less than 50 per cent. It has many
also known as the HVJ pipeline or the Integrated Hazira- impurities and is a low-grade iron ore. It is mainly
Vijaipur-Jagdishpur pipeline. It has a length of 4222 km. found in the states of West Bengal, Uttarakhand, Uttar
• Kandala-Bhatinda pipeline: This pipeline is also known Pradesh, and Himachal Pradesh
as the Mundra-Bhatinda pipeline or the MDPL pipeline. • Siderite: It is another inferior type of iron ore with a
It has a length of 1894 km. low metallic content of 40 to 50 per cent. It contains
• Jamnagar-Loni (LPG) pipeline: This pipeline is also many impurities and is not economically viable for
known as the JNPL pipeline. It has a length of 1,414km mining. However, it is self-fluxing due to the presence
• Salaya-Koyali-Mathura pipeline: This pipeline is also of lime. It is mainly found in the states of Assam, Bihar,
known as the SKM pipeline. It has a length of 1326 km Goa, Jharkhand, Kerala, Maharashtra, Meghalaya and
Nagaland
Important Tips
Crude Oil, Petroleum Product and Gas Pipelines: 167. Solution: (d)
• India has a network of more than 15,000 km long Exp) Option d is the correct answer.
crude oil, petroleum product and gas pipelines with a The first cotton textile mill in India was established at Fort
throughput capacity of 94.56 million metric tonnes per Gloster near Calcutta in 1818. Large scale production of
annum of oil and 21.69 million metric standard cubic cotton started in Bombay in 1854.
meters per day of gas
Important Tips
• The longest pipeline in India is the Hajira-Bijapur-
Cotton Production in India:
Jagdishpur gas pipeline, also known as the HVJ pipeline
or the Integrated Hazira-Vijaipur-Jagdishpur pipeline. • India is 2nd largest producer in the world after
It has a length of 4222 km and connects the states of China.
Gujarat, Madhya Pradesh, Rajasthan, Uttar Pradesh, • India is the world’s third-largest exporter of cotton
Haryana, Delhi, Punjab and Uttarakhand and the second-largest exporter of textiles.
• The first transnational pipeline of India is the Motihari- • India has the largest area (36% of world area) under
Amlekhganj Pipeline, which was commissioned in cotton cultivation in the world.
July 2019. It is a 69 km long pipeline that transports • Gujarat is the largest producer of cotton in India.
petroleum products from India to Nepal. • Around 67% of India’s cotton is grown on rain-fed
165. Solution: (c) areas and 33% on irrigated area.
Exp) Option c is the correct answer. • India is on 38th rank with yield of 510 kg/ha.
Damodar river valley is one of the main coal mining 168. Solution: (c)
centres of India, accounting for 80 per cent of the total
Exp) Option c is the correct answer.
coal production in the country. The Damuda series of the
lower Gondwana age contains the best worked coalfields in Transhumance refers to the seasonal movement of people
India. The Damodar River valley lies in Jharkhand and West and their herds from the valley to the mountain and vice
Bengal, where most of the coal fields are located in a narrow versa. It is a traditional practice commonly observed in
belt running in the east-west direction. The Damodar River mountainous regions. During the warmer seasons, herders
valley also has rich deposits of iron ore, copper, uranium and and their livestock migrate to higher-altitude pastures
antimony. for grazing, and in the colder seasons, they move back to
lower altitudes. This practice allows livestock to access fresh
166. Solution: (a) pastures while adapting to seasonal changes in climate and
Exp) Option a is the correct answer. vegetation.

PYQ Workbook 278


GENERAL GEOGRAPHY

Important Tips B. Zawar is a township in the Udaipur district of Rajasthan,


India. It is famous for its ancient zinc and lead mines,
Differences between transhumance and nomadic
which are the oldest in the world. Zinc and lead were
pastoralism:
extracted from Zawar mines since the Mauryan period
Transhumance Nomadic Pastoralism (322 BCE - 185 BCE).

Transhumance is the regular Nomadic Pastoralism is C. Kolar is a city in the state of Karnataka, India. It was the
movement of herders with the irregular movement site of the Kolar Gold Fields (KGF), which were among
their livestock for herding of the herds to seek fresh the deepest gold mines in the world. Gold mining in Kolar
and grazing. pasture. started in 1880 under the British Raj and continued until
2001, when the mines were closed due to low production
It is mainly practised in It is practised in regions and high costs.
mountainous highlands with arable lands.
D. Mosabani is a town in the East Singhbhum district of
and valleys.
Jharkhand, India. It is located near the Singhbhum
Regular and predictable Irregular and Copper Belt, which is one of the major copper producing
movement. unpredictable movement. regions in India. Copper mining in Mosabani began in
1904 by the British Indian Copper Corporation and later
169. Solution: (a) by Hindustan Copper Limited.
Exp) Option a is the correct answer. Important Tips
A. Denmark is one of the leading countries in commercial Bauxite Production:
dairy farming, with a high level of productivity, quality, • India is the sixth-largest producer of bauxite in
and innovation. Denmark produces about 5.5 million the world, accounting for about 6% of the global
tonnes of milk annually, of which 90% is exported to production in 2022
other countries.
• Bauxite is mainly found in tertiary deposits and is
B. Argentina is one of the major countries in commercial associated with laterite rocks occurring on the plateaus
grain farming, with a large production of wheat and and hill ranges of peninsular India, as well as along the
maize. Argentina produces about 19 million tonnes of
coasts.
wheat and 50 million tonnes of maize annually, of which
most are exported to other countries. • The major bauxite producing states in India are Orissa,
Gujarat, Jharkhand, Maharashtra, Chhattisgarh, Tamil
C. Malaysia is one of the leading countries in commercial
Nadu and Madhya Pradesh.
plantation farming, with a large production of rubber
and oil palm. Malaysia produces about 800,000 tonnes • Orissa is the largest producer of bauxite in India,
of rubber and 19 million tonnes of palm oil annually, of contributing about 50% of the total production. The
which most are exported to other countries. main bauxite belt in Orissa is in Kalahandi, Koraput
and Baragarh districts.
D. France is one of the major countries in commercial
fruits production, with a large production of apples, • Gujarat is the second-largest producer of bauxite in
grapes, peaches, pears, and apricots. France produces India, with the most important deposits occurring in
about 1.8 million tonnes of apples and 5.6 million tonnes a belt between the Gulf of Kachchh and the Arabian
of grapes annually, of which some are exported to other Sea through Bhavnagar, Junagadh and Amreli districts
countries.
172. Solution: (d)
170. Solution: (d) Exp) Option d is the correct answer.
Exp) Option d is the correct answer. Bauxite is different from the other three minerals because
The Marquette Iron Range is a deposit of iron ore located it is not an iron oxide, but a mixture of hydrated aluminum
in Marquette County, Michigan in the United States. The oxides. Bauxite is the main source of aluminum, while
towns of Ishpeming and Negaunee developed as a result of hematite, magnetite, and limonite are iron ores.
mining this deposit. A smaller counterpart of Minnesota’s
Mesabi Range, this is one of two iron ranges in the Lake Important Tips
Superior basin that are in active production as of 2018. Bauxite Ore:
• Bauxite is a mixture of hydrated aluminum oxides,
171. Solution: (c)
which means it contains aluminum, oxygen, and water
Exp) Option c is the correct answer. in various proportions.
A. Kalahandi is a district in the state of Odisha, India. It • Bauxite is formed by the weathering of rocks that
is known for its rich deposits of bauxite, which is the contain aluminum minerals, such as feldspar or clay.
main source of aluminium. Bauxite mining in Kalahandi
started in 1980s by the public sector National Aluminium • Bauxite is usually reddish-brown, yellow, or white in
Company (NALCO). color, and has a pisolitic or earthy texture.

279 PYQ Workbook


GENERAL GEOGRAPHY

• Bauxite is the main source of aluminum, which is used diverse range of industries, such as textiles, furniture,
for making various products, such as cans, foil, aircraft, paper, and electronics.
and alloys. 176. Solution: (b)
173. Solution: (a) Exp) Option b is the correct answer.
Exp) Option a is the correct answer. The Lorraine region of France, historically known for its
rich deposits of iron ore and coal, has been a significant
The biggest producer of mica in India is Andhra Pradesh.
center for the iron and steel industry. This area, particularly
It has 41% share in the country’s total resources of mica
around the cities of Metz and Nancy, has been a hub for steel
and produces both muscovite (potash or white mica) and
production for many years.
phlogopite (magnesium or amber mica).
Over time, the Lorraine region has been associated with
Important Tips various steel production facilities, including blast furnaces,
Facts About Mica: steel mills, and related industries. It has been a cornerstone
• Jharkhand has the richest deposits of high-quality of France’s industrial base, supplying materials for
ruby mica, which is found in a belt extending from construction, manufacturing, and other sectors.
Gaya district of Bihar to Hazaribagh and Koderma
177. Solution: (a)
districts of Jharkhand
Exp) Option a is the correct answer.
• Rajasthan has a long mica belt that extends from Jaipur
to Udaipur along the Aravalli range Nickel, Zinc, Copper and Aluminum are all examples
of non-ferrous metals, which are metals or alloys that do
• Mica is exported mainly to Japan, the USA, and the not contain iron in appreciable amounts. Some other non-
UK. India has a near monopoly in the production of ferrous metals are lead, tin, titanium, gold, silver, and
mica, accounting for 60% of the world’s total. platinum. Non-ferrous metals have some advantages over
• Mica is also used for making jewellery, leather ferrous metals, such as higher conductivity, lower weight,
accessories, and insulation products. non-magnetic properties, or resistance to corrosion. Non-
ferrous metals are also widely used in various applications,
174. Solution: (b)
such as electrical and electronic devices, pipelines, and
Exp) Option b is the correct answer. industrial processes.
Khair, also known as Acacia catechu, is a tree that belongs
178. Solution: (b)
to the family Fabaceae. It is widely distributed in India,
especially in the drier and deciduous regions. The heartwood Exp) Option b is the correct answer.
of Khair is used for catechu production, which is an extract Poplar is a type of hardwood tree that is used for making
of acacia trees used for various purposes such as food paper pulp. Poplar has short fibers that are suitable for
additive, astringent, tannin, and dye. Catechu is obtained by producing smooth and fine paper. Poplar is also a fast-
boiling the wood chips in water and evaporating the resulting growing and widely available tree that can be harvested
brew1. Catechu is also known as cutch, black cutch, cachou, sustainably. Other types of hardwood trees that are used for
or katha. paper pulp include birch, eucalyptus, and aspen.

175. Solution: (d) Important Tips


Exp) Option d is the correct answer. Non-wood sources for making paper pulp:
A. The Lancashire region is an industrial region in the • Peparin is a type of grass that grows in wetlands and
United Kingdom, located in the northwest of England. can be used for making paper pulp.
It is known for its textile industry, especially cotton and • Bagasse is the fibrous residue left after sugarcane is
wool, as well as its coal mining and engineering sectors. crushed to extract its juice and can be used for making
B. The Ruhr region is an industrial region in Germany, paper pulp.
located in the state of North Rhine-Westphalia. It is one • Rice straw is the stalks of rice plants that are left after
of the largest urban areas in Europe and a major center of harvesting the grains and can be used for making paper
coal, steel, and chemical production. pulp.
C. The Keihin region is an industrial region in Japan, • Non-wood pulp has some advantages over wood pulp,
consisting of the cities of Tokyo, Kawasaki, and Yokohama. such as lower cost, less environmental impact, and
It is one of the most densely populated and economically higher yield.
dynamic regions in the world, with a large concentration • However, non-wood pulp also has some disadvantages,
of manufacturing, trade, and service industries. such as lower quality, shorter fibers, and higher
D. The Southern Appalachian region is an industrial chemical consumption.
region in the United States of America, spanning parts
of several states from Virginia to Alabama. It is rich in 179. Solution: (b)
natural resources, especially coal and timber, and has a Exp) Option b is the correct answer.

PYQ Workbook 280


GENERAL GEOGRAPHY

Africa is known as ‘the Cradle of Mankind’ because it is Visakhapatnam refinery is one of the oil refineries of
where human life originated. The term specifically refers HPCL in India, located in Visakhapatnam city in Andhra
to a region in South Africa, which has yielded fossil remains Pradesh. The refinery was commissioned in 1957 by Caltex
of several human ancestors dating back millions of years. Oil Refining India and has a capacity of 8.3 million tonnes
This region, including sites like Sterkfontein, Swartkrans, per year.
Kromdraai, Drimolen, Bolt’s Farm, and Gondolin holds • Assam does not have a Barauni oil refinery, but a Digboi
crucial evidence for the study of human evolution, and it was refinery.
designated a UNESCO World Heritage site in 1999.
• Maharashtra does not have a Koyna oil refinery, but a
180. Solution: (a) Mumbai refinery.
Exp) Option a is the correct answer. • Gujarat does not have a Mathura oil refinery, but a
The area which is the ‘storehouse of minerals’ in South Jamnagar refinery.
America is Brazilian Plateau. It covers most of the eastern, 183. Solution: (c)
southern and central portions of Brazil. One of the most
Exp) Option c is the correct answer.
important aspects of this plateau is its vast natural resources.
This includes mineral deposits such as gold, iron, uranium, Zambia is a landlocked country in Southern Africa,
nickel, copper, tin, and manganese. Other than that, surrounded by eight neighboring countries. It has no direct
agricultural produce such as coffee beans, soybeans, and access to the sea, which makes it difficult and costly to
cotton are also produced. export its copper and other minerals to the global market.
According to the World Bank, Zambia’s transport costs
Important Tips are among the highest in Sub-Saharan Africa, accounting
• Bolivia Plateau: It is also known as Altiplano region of for 30-40% of the total value of exports. This reduces the
southeastern Peru and western Bolivia. competitiveness and profitability of Zambia’s mining sector,
• Patagonia Plateau: It is the southernmost part of which is the main source of its foreign exchange earnings
South America. It covers part of Argentina and Chile. and economic growth. Moreover, Zambia faces challenges
The region is known for dramatic mountain peaks, an such as inadequate infrastructure, unreliable power supply,
abundance of glaciers and an array of unique wildlife. policy uncertainty, and corruption that hamper its mining
development and diversification. Therefore, the lack of a
• Orinoco Basin: It is the part of South America drained
seaport is one of the major reasons why Zambia has not
by the Orinoco River and its tributaries. The Orinoco
made much economic progress despite its prolific copper
watershed covers most of Venezuela and eastern part
reserves.
of Colombia.
184. Solution: (b)
181. Solution: (d)
Exp) Option b is the correct answer.
Exp) Option d is the correct answer.
Kodarma is a district in Jharkhand that is famous for its
India is the largest banana producer in the world with
mica mines. It is also known as the Mica Capital of India
30,460,000 tonnes production per year. India produces
or Abrakh-Nagri. Kodarma produces the largest quantity of
nearly 20 million more tonnes of bananas per year than the
muscovite, a type of mica, in the world. Mica is a mineral
second-largest producer, China, which produces 11,998,329
that has many uses in the electrical and electronics industry,
tonnes per year. Colombia, Zimbabwe and Malaysia are not
as well as in cosmetics and toothpaste. It is also valued for
among the top ten banana-producing countries in the world.
its glittery appearance and unique properties of elasticity,
Important Tips toughness, flexibility, and transparency.
Banana Production: 185. Solution: (d)
• The major banana producing states in India are Exp) Option d is the correct answer.
Andhra Pradesh, Maharashtra, Gujarat, Tamil Nadu,
Jamshedpur is known as the Pittsburg of India. Just like
and Karnataka.
Pittsburg, Jamshedpur is famous for steel production in
• These five states account for more than 80% of the total India. The first steel plant by TATA was set up in Jamshedpur.
banana production in India. Pittsburg is an important steel city in the United States of
• Banana is grown in different agro-climatic zones and America.
under different farming systems in India.
186. Solution: (a)
• It is a rich source of carbohydrates, vitamins, minerals,
and dietary fiber. It is also used for making various Exp) Option a is the correct answer.
products such as chips, puree, powder, wine, vinegar, Marble is a metamorphic rock primarily composed of re-
etc. crystallized calcium carbonate minerals, typically calcite
or dolomite. Marble is derived from limestone, which is a
182. Solution: (b) sedimentary rock composed of calcium carbonate (CaCO3).
Exp) Option b is the correct answer. Through a process of metamorphism, which involves high

281 PYQ Workbook


GENERAL GEOGRAPHY

heat and pressure beneath the Earth’s surface, limestone • The major coalfields in India are located in the eastern
undergoes profound physical and chemical changes. and central regions of the country, and the coal found
During this metamorphic process, the calcite or dolomite in India is primarily bituminous and sub-bituminous
crystals in limestone recrystallize and grow larger, resulting in nature.
in the formation of marble. The recrystallization process can • Coal in India has been mined since 1774, and India is
also lead to the development of a wide range of colours and the second largest producer and consumer of coal after
unique veining patterns in the marble. China, mining 893.19 million tonnes in FY 2022-23.
187. Solution: (c) • Coking coal is being imported by steel sector mainly to
Exp) Option c is the correct answer. bridge the gap between the requirement and indigenous
availability and to improve the quality.
Coal is a sedimentary rock formed from the accumulation
and preservation of plant materials in swamp environments. 188. Solution: (d)
Coal is composed of organic matter, mainly carbon,
Exp) Option d is the correct answer.
hydrogen, oxygen, nitrogen, and sulfur. The grade or rank of
coal depends on the percentage of carbon present. Coal, crude oil, and natural gas are all considered fossil
fuels because they were formed from the fossilized, buried
Important Tips
remains of plants and animals that lived millions of years
Coal Reserves in India: ago. Fossil fuels are hydrocarbon-containing materials that
• India has around 319 billion tonnes of coal reserves are extracted and burned as a source of energy. They are
and is one of the largest coal-producing countries in nonrenewable resources, which means that they cannot be
the world. replenished once they are used up.

PYQ Workbook 282


GENERAL GEOGRAPHY

GENERAL GEOGRAPHY
MISCELLANEOUS
*This unit consists of questions from Location based and other Miscellaneous geography concepts.

7.1. UPSC CSE Previous Years’ Questions (c) 3, 4 and 5 only


(d) 1, 2, 3, 4 and 5
1. Which one of the following is a part of the
Congo Basin? [UPSC CSE Pre 2023] 5. Consider the following pairs:
(a) Cameroon Region often mentioned in Country
(b) Nigeria the news
(c) South Sudan
(d) Uganda 1. Anatolia Turkey
2. Amhara Ethiopia
2. Consider the following pairs:
3. Cabo Delgado Spain
Area of conflict Country where it
4. Catalonia Italy
mentioned in news is located
How many of the above pairs are correctly
1. Donbas Syria
matched? [UPSC CSE Pre 2022]
2. Kachin Ethiopia (a) Only one pair
3. Tigray North Yemen (b) Only two pairs
How many of the above pairs are correctly (c) Only three pairs
matched? [UPSC CSE Pre 2023] (d) All four pairs
(a) Only one
(b) Only two 6. Consider the following pairs:
(c) All three [UPSC CSE Pre. 2020]
(d) None
River Flows into
3. Consider the following countries
1. Mekong Andaman Sea
1. Bulgaria
2. Czech Republic 2. Thames Irish Sea
3. Hungary 3. Volga Caspian Sea
4. Latvia 4. Zambezi Indian Ocean
5. Lithuania Which of the pairs given above is/are correctly
6. Romania
matched?
How many of the above countries share a land
(a) 1 and 2 only
border with Ukraine? [UPSC CSE Pre 2023]
(b) 3 only
(a) Only two
(b) Only three (c) 3 and 4 only
(c) Only four (d) 1, 2 and 4 only
(d) Only five 7. Consider the following pairs:
4. Consider the following countries: [UPSC CSE Pre. 2019]
1. Azerbaijan
2. Kyrgyzstan Sea Bordering
3. Tajikistan country
4. Turkmenistan 1. Adriatic Sea Albania
5. Uzbekstan 2. Black Sea Croatia
Which of the above have borders with 3. Caspian Sea Kazakhstan
Afghanistan? [UPSC CSE Pre 2022]
4. Mediterranean Sea Morocco
(a) 1, 2 and 5 only
(b) 1, 2, 3 and 4 only 5. Red Sea Syria

283 PYQ Workbook


GENERAL GEOGRAPHY

Which of the pairs given above are correctly Select the correct answer using the code given
matched? below:
(a) 1, 2 and 4 only (a) 1, 2 and 3
(b) 1, 3 and 4 only (b) 2 and 3 only
(c) 2 and 5 only (c) 3 and 4 only
(d) 1, 2, 3, 4 and 5 (d) 1, 3 and 4 only
8. Consider the following pairs: 12. If you travel by road from Kohima to
[UPSC CSE Pre. 2018] Kottayam, what is the minimum number
of States within India through which you
Regions sometimes Country can travel, including the origin and the
mentioned in news destination? [UPSC CSE Pre 2017]
1. Catalonia Spain (a) 6
2. Crimea Hungary (b) 7
3. Mindanao Philippines (c) 8
(d) 9
4. Oromia Nigeria
Which of the pair given above are correctly 13. Which one of the following countries of
matched? South-West Asia does not open out to the
(a) 1, 2 and 3 Mediterranean Sea? [UPSC CSE Pre. 2015]
(b) 3 and 4 only (a) Syria
(c) 1 and 3 only (b) Jordan
(d) 2 and 4 only (c) Lebanon
(d) Israel
9. Consider the following pairs:
[UPSC CSE Pre. 2018] 14. Turkey is located between
[UPSC CSE Pre. 2014]
Towns sometimes Country mentioned in (a) Black Sea and Caspian Sea
news (b) Black Sea and Mediterranean Sea
1. Aleppo Arabia (c) Gulf of Suez and Mediterranean Sea
2. Kirkuk Yemen (d) Gulf of Aqaba and Dead Sea
3. Mosul Palestine 15. What is the correct sequence of occurrence
4. Mazar-i-sharif Afghanistan of the following cities in South-East Asia as
one proceeds from south to north?
Which of the pairs given above are correctly
matched? [UPSC CSE Pre. 2014]
(a) 1 and 2 1. Bangkok
(b) 1 and 4 2. Hanoi
(c) 2 and 3 3. Jakarta
(d) 3 and 4 4. Singapore
10. Which of the following is geographically Select the correct answer using the code given
closest to Great Nicobar? below:
[UPSC CSE Pre. 2017] (a) 4-2-1-3
(a) Sumatra (b) 3-2-4-1
(b) Borneo (c) 3-4-1-2
(c) Java (d) 4-3-2-1
(d) Sri Lanka 16. Consider the following pairs:
11. Mediterranean Sea is a border of which of National Highway Cities Connected
the following countries?
1. NH 4 Chennai and
[UPSC CSE Pre. 2017]
Hyderabad
1. Jordan
2. Iraq 2. NH 6 Mumbai and Kolkata
3. Lebanon 3. NH 15 Ahmedabad and
4. Syria Jodhpur

PYQ Workbook 284


GENERAL GEOGRAPHY

Which of the above pairs is/are correctly (a) Australia


matched? [UPSC CSE Pre 2014] (b) Cuba
(a) 1 and 2 only (c) Ghana
(b) 3 only (d) Philippines
(c) 1, 2 and 3
23. Where is Copacabana Beach located?
(d) None of the above
[UPSC CSE Pre 2007]
17. Match list-I with list-II and select the (a) Buenos Aires
correct answer using the codes given below (b) Hawaiian Islands
the lists: (c) Rio de Janeiro
List-I List-II (d) Valletta
(Geographic Feature) (Country) 24. Through which one of the following straits
A. Great Victoria Desert 1. Australia does a tunnel connect the United Kingdom
and France? [UPSC CSE Pre 2007]
B. Grand Canyon 2. Canada
(a) Davis Strait
C. Lake Winnipeg 3. New Zealand (b) Denmark Strait
D. Southern Alps 4. USA (c) Strait of Dover
Code: [UPSC CSE Pre 2009] (d) Strait of Gibraltar
A B C D 25. In which one of the following oceans
(a) 1 2 4 3 Diamantina Trench is situated?
(b) 1 4 2 3 [UPSC CSE Pre 2006]
(c) 3 2 4 1 (a) Pacific Ocean
(d) 3 4 2 1
(b) Atlantic Ocean
18. In India, the ports are categorized as major (c) Indian Ocean
and non-major ports. Which one of the (d) Arctic Ocean
following is a non-major port?
26. Through which one of the following groups
[UPSC CSE Pre 2009] of countries does the Equator pass?
(a) Kochi (Cochin)
[UPSC CSE Pre 2006]
(b) Dahej
(c) Paradip (a) Brazil, Zambia and Malaysia
(d) New Mangalore (b) Colombia, Kenya and Indonesia
(c) Brazil, Sudan and Malaysia
19. In which one of the following is Malta (d) Venezuela, Ethiopia and Indonesia
located? [UPSC CSE Pre 2008]
(a) Baltic Sea 27. Other than India and China, which one of
(b) Mediterranean Sea the following groups of countries border
(c) Black Sea with Myanmar? [UPSC CSE Pre 2006]
(d) North Sea (a) Bangladesh, Thailand and Vietnam
(b) Cambodia, Laos and Malaysia
20. Which one of the following cities is nearest (c) Thailand, Vietnam and Malaysia
to the equator? [UPSC CSE Pre 2008]
(d) Thailand, Laos and Bangladesh
(a) Colombo
(b) Jakarta 28. Huangpu River flows through which one of
(c) Manila the following cities? [UPSC CSE Pre 2006]
(d) Singapore (a) Beijing
21. Which one of the following Straits is nearest (b) Ho Chi Minh City
to the International Date Line? (c) Shanghai
[UPSC CSE Pre 2008] (d) Manila
(a) Malacca Strait 29. Consider the following statements:
(b) Bering Strait 1. Length of a terrestrial mile is lesser than
(c) Strait of Florida that of a nautical mile.
(d) Strait of Gibraltar 2. Harmattan is a dusty land-wind of the
22. The largest Coral reef in the world is found East African Coast.
near the coast of which one of the following 3. Greece and Albania form a part of the
Countries? [UPSC CSE Pre 2007] Iberian Peninsula.

285 PYQ Workbook


GENERAL GEOGRAPHY

Which of the statements given above is/are 34. Which one of the following countries does
correct? [UPSC CSE Pre 2006] not border with Lithuania?
(a) 1, 2 and 3 [UPSC CSE Pre 2005]
(b) 2 and 3 only (a) Poland
(c) 3 only (b) Ukraine
(d) 1 only (c) Belarus
30. Bermuda Triangle extends up to which of (d) Latvia
the following places?[UPSC CSE Pre 2006] 35. Consider the following statements:
1. Southern Florida [UPSC CSE Pre 2005]
2. Puerto Rico 1. Great Britain comprises England, Wales,
3. Hawaii Islands Scotland and Northern Ireland.
Select the correct answer using the codes 2. England covers less than 60% of the total
given below: area of the United Kingdom.
(a) 1, 2 and 3 Which of these statements given above is/are
(b) 1 and 2 only correct?
(c) 2 and 3 only (a) 1 only
(d) 1 and 3 only (b) 2 only
(c) Both 1 and 2
31. Which one of the following pairs is not (d) Neither 1 nor 2
correctly matched? [UPSC CSE Pre 2006]
(a) Slovenia: Bratislava 36. Which one of the following pairs is not
(b) Seychelles: Victoria correctly matched? [UPSC CSE Pre 2005]
(c) Sierra Leone: Freetown (a) Bahamas: Nassau
(d) Uzbekistan: Tashkent (b) Costa Rica: San Jose
(c) Nicaragua: Belmopan
32. Consider the following statements: (d) Dominican Republic: Santo Domingo
1. Champaner-Pavagarh Archaeological
37. Which one of the following pairs is not
Park
correctly matched?[UPSC CSE Pre 2005]
2. Chhatrapati Shivaji Railway Station,
(a) Seikan Rail Tunnel: China
Mumbai
(b) Petronas Towers: Malaysia
3. Mamallapuram (c) Appalachian Trail: USA
4. Sun Temple (Konark Temple) (d) Rogun Dam: Tajikistan
Which of the above are included in the World 38. Match list-I with list-II and select the
Heritage List of UNESCO? correct answer using the codes given below
[UPSC CSE Pre 2005] the lists:
(a) 1, 2 and 3 List-I List-II
(b) 1, 3 and 4
(Atomic Power Plants/ (State)
(c) 2 and 4
Heavy Water Plant)
(d) 1, 2, 3 and 4
A. Thal 1. Andhra
33. Which one of the following is the correct Pradesh
sequence of the given continents in the B. Manuguru 2. Gujarat
descending order of their percentage of
Earth’s land? [UPSC CSE Pre 2005] C. Kakrapara 3. Maharashtra
(a) North America–Africa–South America– D. Kaiga 4. Rajasthan
Europe 5. Karnataka
(b) Africa–North America–South America– Code: [UPSC CSE Pre 2005]
Europe A B C D
(c) North America–Africa–Europe–South (a) 2 1 4 5
America (b) 3 5 2 1
(d) Africa–North America–Europe–South (c) 2 5 4 1
America (d) 3 1 2 5

PYQ Workbook 286


GENERAL GEOGRAPHY

39. Which one of the following countries does D. S. Sandwich South Atlantic
not share a border with the Caspian Sea? Ocean
[UPSC CSE Pre 2003]
Code: [UPSC CSE Pre 2000]
(a) Armenia
(b) Azerbaijan A B C D
(c) Kazakhstan (a) 2 4 1 3
(d) Turkmenistan (b) 2 3 1 4
40. Consider the following countries: (c) 1 3 2 4
1. Albania (d) 1 4 2 3
2. Bosnia Herzegovina 43. Consider the following statements:
3. Croatia 1. Tides are of great help in navigation and
4. Macedonia fishing.
Which of these countries has/have the 2. High tide enables big ships to enter or
Adriatic Sea as a boundary? leave the harbour safely.
[UPSC CSE Pre 2002] 3. Tide prevents siltation in the harbours.
(a) 1 and 2 only 4. Kandla and Diamond Harbour are tidal
(b) 1, 2 and 3 only ports.
(c) 4 only Which of these statements are correct?
(d) 3 and 4 only [UPSC CSE Pre 2000]
41. In the map given below, four islands of (a) 1 and 4
Indian Ocean region i.e., (A) Seychelles, (b) 2, 3 and 4
(B) Chagos, (C) Mauritius and (D) Socotra (c) 1, 2 and 3
are marked as 1, 2, 3 and 4. Match them and (d) 1, 2, 3 and 4
select the correct answer from the codes
given below: 44. Match the locations of ports labelled as A,
B, C and D in the given map with the names
of those ports and select the correct answer
using the codes given below the names of
the ports:

Code: [UPSC CSE Pre 2002]


A B C D
(a) 1 3 4 2
(b) 3 1 2 4
(c) 1 3 2 4 Names of Ports:
(d) 3 1 4 2 1. Kakinada
2. Karwar
42. Match list-I with list-II and select the
correct answer using the codes given below 3. Mangalore
the lists: 4. Tuticorin
5. Veraval
List-I List-II Code: [UPSC CSE Pre 2000]
(Oceanic Trench) (Location) A B C D
A. Aleutian Indian Ocean (a) 4 2 3 5
B. Kermadec North Pacific (b) 5 2 4 1
(c) 1 3 4 2
C. Sunda South Pacific (d) 5 3 2 1

287 PYQ Workbook


GENERAL GEOGRAPHY

45. Match the different ports of Union Territory 49. Consider the map given below.
of Pondicherry labelled as A, B, C and D in [UPSC CSE Pre 1996]
the given map with their respective names
and select the correct answer using the
codes given below the list of ports:

The dotted (broken) line in the map is the-


(a) Durand line
List (Ports of Pondicherry): (b) Mac Mahon line
1. Karaikal (c) Line of demarcation between India and
Pakistan suggested by the Boundary
2. Mahe
Commission (1947)
3. Pondicherry (d) Route followed by the Younghusband
4. Yanam Expedition
Code: [UPSC CSE Pre 2000]
50. The Palk Bay lies between:
(a) A-2, B-1, C-3, D-4
[UPSC CSE Pre 1996]
(b) A-1, B-2, C-3, D-4
(c) A-2, B-1, C-4, D-3 (a) Gulf of Kachchh and Gulf of Khambhat
(d) A-1, B-2, C-4, D-3 (b) Gulf of Mannar and Bay of Bengal
(c) Lakshadweep and Maldives Islands
46. Which one of the following lakes forms as an (d) Andaman and Nicobar Islands
international boundary between Tanzania
and Uganda? [UPSC CSE Pre 2000] 51. Match list-I with list-II and select the
correct answer using the codes given below
(a) Chad
the lists:
(b) Malawi
(c) Victoria List-I List-II
(d) Zambezi A. European 1. Paris to Istanbul
47. Daily weather map showing isobars is an Tran-
example of: [UPSC CSE Pre 1997] Continental
(a) Choropleth map Railway
(b) Isopleth map B. Trans-Andean 2. Leningrad to
(c) Chorochromatic map Railway Vladivostok
(d) Choro schematic map C. Trans- 3. Leningrad to
Siberian Volgograd
48. “The distance from Aceh in the far North-
Railway
West to Torres Strait in the east is 5000
miles, almost equal to the distance London D. Orient 4. Buenos Aires to
and Baghdad. The archipelago has 14,000 Express Valparaiso
islands, some mere equatorial rocks, others 5. Paris to Warsaw
some of the largest in the world.” This Code: [UPSC CSE Pre 1995]
description best fits: [UPSC CSE Pre 1997] A B C D
(a) West Indies (a) 5 4 2 1
(b) Japan (b) 1 4 3 2
(c) Philippines (c) 5 1 2 3
(d) Indonesia (d) 1 2 3 4

PYQ Workbook 288


GENERAL GEOGRAPHY

52. Which one of the following is the continent 55. Consider the map given below: The numbers
with the highest mean elevation in the marked 1, 2, 3 and 4 respectively indicate
world? [UPSC CSE Pre 1995] the islands of: [UPSC CSE Pre 1994]
(a) Antarctica
(b) North America
(c) Asia
(d) South America
53. Given below is a map of some countries
which were parts of the erstwhile Soviet
Union, with water bodies shown by shaded
areas: [UPSC CSE Pre 1995]

The countries marked 1, 2, 3, 4, and 5 are


respectively:
(a) Tajikistan, Turkmenia, Uzbekistan, (a) The Bahamas, Azores, Falkland Cape
Kirgizia, Kazakhstan Verde
(b) Turkmenia, Kirgizia, Tajikistan, (b) The Bahamas, Azores, Cape Verde and
Uzbekistan, Kazakhstan Falkland
(c) Kazakhstan, Uzbekistan, Tajikistan,
(c) Azores, Cape Verde, the Bahamas and
Kirgizia, Turkmenia
(d) Kazakhstan, Turkmenia, Uzbekistan, Falkland
Kirgizia, Tajikistan (d) Cape Verde, the Bahamas, Falkland and
Azores
54. Even though the Railways are the most
pervasive mode of transport in India, it is 56. The shortest air-route from Perth to London
road transport which received the maximum is: [UPSC CSE Pre 1994]
impetus in most of the post-independence (a) Perth, Bombay, Rome, London
period. Which of the following are the (b) Perth, Ankara, Paris, London
reasons for this? (c) Perth, Aden, Paris, London
1. Railways are cheap to operate but the (d) Perth, Mombasa, Rome, London
associated capital costs are very high.
2. Given the geographic spread of human 7.2. Other Examination Previous Years’
settlements, it is unrealistic that the Questions
railways alone can serve the needs of
transport. 57. Which of the following coasts is the largest
3. The per unit (tonne/km) cost of transport tidal energy-producing area in India?
is cheaper in the road compared to rail [UPPCS (Pre) 2022]
transport. (a) Mannar Coast
4. Given the indivisible nature of the (b) Kerala Coast
Railways, it is not always convenient for (c) North Circars Coast
the population to avail it with the same (d) Khambhat Coast
ease as private cars, buses or twowheelers.
Select the correct answer by using codes given 58. Which one of the following is a land locked
below: [UPSC CSE Pre 1994] country? [UPPCS (Pre) 2022]
(a) 1, 2 and 3 (a) Suriname
(b) 1, 2 and 4 (b) Bolivia
(c) 1, 3 and 4 (c) Uruguay
(d) 2, 3 and 4 (d) Peru

289 PYQ Workbook


GENERAL GEOGRAPHY

59. With reference to Borneo Island, located 65. Darling range is situated along which of the
in Southeast Asia, which of the following following coasts of Australia?
statements is/are true? [UPPCS (Pre) 2022] [UPPCS (Pre) 2020]
1. It is divided into three countries. (a) North-Eastern Coast
2. It has been formed due to volcano (b) Southern Coast
deposition. (c) Eastern Coast
Select the correct answer using the codes (d) South-Western Coast
given below. 66. Which one of the following is not correctly
(a) Both 1 and 2 matched? [UPPCS (Pre) 2020]
(b) Only 1
(c) Neither 1 nor 2 (Country) (Capital)
(d) Only 2 (a)Morocco Rabat
60. Which of the following countries is not (b) Tunisia Tunis
located on the bank of the Black Sea?
(c) Algeria Algiers
[U.P.P.C.S. (Pre) 2021]
(a) Syria (d)Cape Verde Bamako
(b) Turkey 67. In the Suez Canal region, the correct order
(c) Georgia of lakes lying from North to South direction
(d) Bulgaria is: [U.P.P.C.S. (Pre) 2019]
61. Nobi and Kanto plains are located in which (a) Lake Timsah - Little Bitter Lake - Great
of the following countries? Bitter Lake - Lake Manzala
[U.P.P.C.S. (Pre) 2021] (b) Great Bitter Lake - Little Bitter lake - lake
(a) North Korea Timsah - Lake Manzala
(b) South Korea (c) Lake Manzala - Great Bitter lake - Little
(c) Japan Bitter Lake - Lake Timsah
(d) China (d) Lake Manzala - Lake Timsah - Great
62. With reference to Socotra Island, which of Bitter Lake - Little Bitter Lake
the following statements is/are correct? 68. At which of the following places is the
[U.P.P.C.S. (Pre) 2021] Secretariat of the International Solar
1. It is an Island of Oman, situated in the Alliance (ISA) located at present?
Arabian Sea. [UPPCS (Pre) 2019]
2. It was designated as a world natural (a) Paris
heritage site by UNESCO in 2008. (b) New Delhi
Select the correct answer using the codes (c) Gurugram
given below. (d) New York
(a) Only 1 69. Telegraphic Plateau is a part of:
(b) Only 2
[U.P.P.C.S. (Pre) 2019]
(c) Both 1 and 2
(d) Neither 1 nor 2 (a) North Atlantic Ridge
(b) South Atlantic Ridge
63. Which of the following is the correct (c) Indian Ocean Ridge
sequence of the Islands of Indonesia from (d) None of these
West to East? [U.P.P.C.S. (Pre) 2021]
(a) Java, Sumatra, Lombok, Bali 70. Match list-I with list-II and select the
(b) Sumatra, Java, Bali, Lombok correct answer using the codes given below
(c) Sumatra, Java, Lombok, Bali the lists:
(d) Bali, Sumatra, Java, Lombok (Grassland/Hill (Location/State)
64. Which of the following Islands is not Station)
located in the Caribbean Sea? A. Banni 1. Uttarakhand
[U.P.P.C.S. (Pre) 2021] Grassland
(a) Grenada B. Bugyal 2. Jammu and Kashmir
(b) Montserrat Grassland
(c) Madeira
(d) Anguilla C. Khajjiar 3. Gujarat

PYQ Workbook 290


GENERAL GEOGRAPHY

D. Pahalgam 4. Himachal Pradesh 77. Which one of the following continents has
the largest number of countries?
Code: [UPPCS (Mains) 2017]
[U.P.P.C.S. (Mains) 2015]
A B C D (a) Africa
(a) 1 3 2 4 (b) Europe
(b) 3 1 4 2 (c) Asia
(c) 3 1 2 4 (d) South America
(d) 4 2 1 3
78. Which one of the following is known as the
71. Which one amongst the following is the ‘Sea of Mountains’?
largest plateau? [U.P.P.C.S. (Pre) 2017] [U.P.P.C.S. (Mains) 2015]
(a) Coconino (a) Japan
(b) Aquarius (b) British Columbia
(c) Colorado (c) West Coast of South America
(d) Columbia (d) South-East Asia

72. Israel has common borders with: 79. Dasht-e-Lut is located in–
[U.P.P.C.S. (Mains) 2015]
[U.P.P.C.S. (Pre) 2017]
(a) Iran
(a) Lebanon, Syria, Jordan and Egypt (b) Libya
(b) Lebanon, Syria, Turkey and Jordan (c) Kenya
(c) Cyprus, Turkey, Jordan and Egypt (d) Nigeria
(d) Turkey, Syria, Iraq and Yemen
80. Which one of the following countries has
73. Red Sea is an example of- the longest coastal line?
[U.P.P.C.S. (Pre) 2017] [U.P.P.C.S. (Pre) 2015]
(a) Volcanic Valley (a) India
(b) Eroded valley (b) Canada
(c) Axial trough (c) Australia
(d) Brazil
(d) U-shaped valley
81. Match list-I with list-II and select the
74. Which of the following is not correctly correct answer using the codes given below
matched? [UPPCS (Mains) 2016] the lists:
(Desert) (Country) List-I List-II
(a)Sonoran United States of A. Golden 1. Myanmar
America Triangle
(b) Taklamakan China B. Bermuda 2. Opium producing
(c) Karakum Turkmenistan Triangle area of Southeast
(d)Gibson Brazil Asia
C. Rice Bowl of 3. China
75. Which of the following is not correctly
Far East
matched? [UPPCS (Pre) 2016]
D. Red Basin 4. North Atlantic
(Ports) (Country) Ocean
(a) Rotterdam The Netherland Code: [UPPCS (Pre) (Re-Exam) 2015]
(b) Igarka China A B C D
(a) 3 2 4 1
(c) Montevideo Uruguay (b) 2 4 1 3
(d)Jakarta Indonesia (c) 4 3 2 1
(d) 1 4 3 2
76. Where is the Central Research Institute for
82. The name of which of the following
Dryland Agriculture (CRIDA) located? countries closely means ‘country of 40
[U.P.P.C.S. (Mains) 2015] tribes? [U.P.P.C.S. (Pre) (Re-Exam) 2015]
(a) Hyderabad (a) Kazakhstan
(b) Hissar (b) Kyrgyzstan
(c) New Delhi (c) Tajikistan
(d) Rajkot (d) Uzbekistan

291 PYQ Workbook


GENERAL GEOGRAPHY

83. Which pair amongst the following is not 88. The most populated desert in the world is–
correctly matched? [U.P.P.C.S. (Pre) 2015] [U.P.P.C.S. (Pre) 2012]
(a) The queen of Adriatic Sea - Rome (a) Sahara
(b) Lorraine Coal Fields - France (b) Gobi
(c) West Midlands - Birmingham (c) Thar
(d) Tula - Russia (d) Kalahari
84. Match list-I with list-II and select the 89. Which of the following pairs of countries
correct answer using the codes given below are around the Aral Sea?
the lists: [U.P.P.C.S. (Pre) 2012]
List-I (World List-II (Owner (a) Kazakhstan - Uzbekistan
Islands) Country) (b) Kazakhstan - Turkmenistan
A. Aleutian 1. Russia (c) Azerbaijan - Uzbekistan
Islands (d) Kazakhstan - Russia
B. Bear Islands 2. Denmark 90. The highest share in power generation in
C. Greenland 3. Norway India is of: [U.P.P.C.S. (Mains) 2011]
D. Franz Josef 4. USA (a) Atomic power
Island (b) Hydropower
Code: [UPPCS (Mains) 2014] (c) Thermal power
(d) All above have equal shares
A B C D
(a) 4 3 2 1 91. Which one of the following is a temperate
(b) 1 2 3 4 desert? [U.P.P.C.S. (Mains) 2011]
(c) 3 1 4 2 (a) Arabian Desert
(d) 2 4 1 3 (b) Atacama Desert
85. Which of the following does not have the (c) Kalahari Desert
‘National Museum of Natural History’? (d) Patagonian Desert
[U.P.P.C.S. (Mains) 2013] 92. Match list-I with list-II and select the
(a) Mysore correct answer using the codes given below
(b) Hyderabad the lists:
(c) Bhuvaneshwar
List-I List-II (Country)
(d) Bhopal
(Mountain)
86. Which of the following are the natural ports A. Allegheny 1. Canada
of India?
B. Cantabrian 2. Iran
1. Chennai
C. Elburz 3. Spain
2. Cochin
3. Tuticorin D. Mackenzie 4. USA
4. Vishakhapatnam Code: [UPPCS (Mains) 2010]
Select the correct answer from the code given A B C D
below: [U.P.P.C.S. (Pre) 2013] (a) 4 3 2 1
(a) 1 and 2 (b) 4 1 3 2
(b) 1 and 3 (c) 2 3 4 1
(c) 2 and 3 (d) 1 4 2 3
(d) 2 and 4 93. Which one of the following African countries
87. Which of the following is a specific example does not border with the Mediterranean
of ‘Out Port’? [U.P.P.C.S. (Mains) 2012] Sea? [U.P.P.C.S. (Mains) 2010]
(a) Porbandar (a) Algeria
(b) Haldia (b) Chad
(c) Panaji (c) Morocco
(d) Vishakhapatnam (d) Tunisia

PYQ Workbook 292


GENERAL GEOGRAPHY

94. Which one of the following rivers of Asia 100. Which of the following is not coastline
flows towards the south? country of Persian Gulf?
[U.P.P.C.S. (Mains) 2010] [U.P.P.C.S. (Mains) 2008]
(a) Amur (a) Kuwait
(b) Lena (b) Oman
(c) Ob (c) Qatar
(d) Salween (d) United Arab Emirates
95. The Prime Meridian passes through- 101. Which one of the following statements is
1. Algeria not true regarding the Atacama?
2. France [U.P.P.C.S. (Mains) 2006]
3. Nigeria (a) It is a desert in South America.
4. Portugal (b) It is World’s driest desert.
(c) It lies in Southern Chile.
Use the code below to select the correct
(d) It has rich reserves of nitrates.
answer. [U.P.P.C.S. (Spl) (Mains) 2008]
(a) 1 and 2 102. The Tropic of Cancer does not pass through-
(b) 2 and 3 [U.P.P.C.S. (Pre) 2005]
(c) 3 and 4 (a) Egypt
(d) 1 and 3 (b) India
96. The Kalahari Desert lies in- (c) Iran
(d) Myanmar
[U.P.P.C.S. (Mains) 2008]
1. Botswana 103. Which of the following countries is not a
2. Namibia land-locked country?
3. Zaire [U.P.P.C.S. (Pre) 2005]
4. Zambia (a) Afghanistan
(b) Liberia
Select the correct answer from the code given
(c) Laos
below.
(d) Luxembourg
(a) 1 only
(b) 2 only 104. Which one of the following continents has
(c) 1 and 2 only no landlocked country?
(d) 3 and 4 only [U.P.P.C.S. (Mains) 2005]
97. Rio Grande River makes boundary (a) Africa
between– [U.P.P.C.S. (Spl) (Mains) 2008] (b) Asia
(c) Europe
(a) Canada and USA
(d) North America
(b) Mexico and USA
(c) Mexico and Guatemala 105. Consider the following statements:
(d) Guatemala and Honduras [U.P.P.C.S. (Mains) 2005]
98. The country which provides the best Assertion (A): Egypt is the Gift of the Nile.
example of radial drainage is- Reason (R): It is an oasis of the Sahara Desert.
[U.P.P.C.S. (Spl) (Mains) 2008] Select the correct answer from the codes
(a) Australia given below:
(b) France (a) Both (A) and (R) are true, and (R) is the
(c) Sri Lanka correct explanation of (A)
(d) Zaire (b) Both (A) and (R) are true, but (R) is not
99. Which of the following pair is not correctly the correct explanation of (A)
matched? [UPPCS (Spl) (Mains) 2008] (c) (A) is true, but (R) is false
(d) (A) is false, but (R) is true
(River) (Country)
106. Which one of the following pairs is correctly
(a) Dnieper Ukraine matched? [U.P.P.C.S. (Mains) 2003]
(b) Po Italy (a) Gulf of Bothnia - France
(c) Rhine Germany (b) Baffin Bay - Argentina
(c) Gulf of Carpentaria - Canada
(d) Rhone Spain (d) Gulf of Tonkin - Vietnam

293 PYQ Workbook


GENERAL GEOGRAPHY

107. Consider the following statements: 111. ‘Horn of Africa’ consists of–
Assertion (A): North Atlantic Sea route is the [U.P.P.C.S. (Pre) 1999]
busiest sea route in the world. (a) Algeria, Morocco and West Sahara
(b) Libya, Sudan and Egypt
Reason (R): It connects two major industrial
(c) Somalia, Ethiopia and Djibouti
zones of the world. (d) Zimbabwe, Botswana and Angola
In the context of the above statements which
112. Which of the following rivers flows through
of the following is correct? a rift valley? [U.P.P.C.S. (Pre) 1998]
[U.P.P.C.S. (Pre) 2003] (a) Amazon
(a) (A) and (R) both are true, and (R) is (b) Indus
correct explanation of (A). (c) Volga
(b) (A) and (R) both are true, but (R) is not (d) Rhine
the correct explanation of (A). 113. Datum line: [U.P.P.C.S. (Pre) 1991]
(c) (A) is true, but (R) is false. (a) Is a horizontal line to the sea level from
(d) (R) is true, but (A) is false. where heights and depths are measured
108. Think about the following countries: (b) Is the mean of primary and secondary
data
1. Australia
(c) International date line
2. Namibia (d) Is an imaginary line which passes through
3. Brazil zero-degree meridian
4. Chile 114. Match list-I with list-II and select the
The Tropic of Capricorn passes through correct answer using the codes given below
which of the following countries. the lists:
[U.P.P.C.S. (Pre) 2003] List-I List-II
(a) Only 1 A. Rail Coach 1. Bangalore
(b) 3, and 4 Factory
(c) 1, 2 and 3
B. Wheel and 2. Perambur
(d) 1, 2, 3 and 4
Axle Plant
109. Which one of the following is correctly C. Diesel 3. Kapurthala
matched? [U.P.P.C.S. (Mains) 2002] Locomotive
(a) Paraguay - Pampas Works
(b) U.S.A. – Manitoba D. Integral Coach 4. Varanasi
(c) Morocco - Atlas Mountains Factory
(d) Australia - Kimberley Code: [U.P.P.C.S. (Pre) 1991]
110. Match list-I with list-II and select the A B C D
correct answer using the codes given below (a) 1 2 3 4
the lists: (b) 4 3 2 1
(c) 1 3 4 2
List-I List-II (d) 3 1 4 2
(Country) (Dependent Territory) 115. Where are the Bermuda Islands located?
A. Australia 1. Martinique [CDS 2022(I)]
B. Denmark 2. Santa Cruz (a) Caribbean Sea
C. France 3. Greenland (b) North Atlantic Ocean
(c) Gulf of Mexico
D. Spain 4. Christmas Island (d) Mediterranean Sea
Code: [UPPCS (Pre) 2000]
116. An avalanche is a type of which one of the
A B C D following disasters? [CDS 2021 (II)]
(a) 1 2 3 4 (a) Atmospheric
(b) 4 3 1 2 (b) Terrestrial
(c) 1 3 2 4 (c) Aquatic
(d) 2 1 4 3 (d) Biological

PYQ Workbook 294


GENERAL GEOGRAPHY

117. Which one of the following rivers does not Code: [CDS 2018 (II)]
drain into Black Sea? [CDS 2020 (I)] A B C D
(a) Volga
(b) Dnieper (a) 2 1 3 4
(c) Don (b) 2 3 1 4
(d) Danube (c) 4 3 1 2
(d) 4 1 3 2
118. Which one of the following cities is closest
to the Equator? [CDS 2020 (II)] 124. Match List I with List II and select the
(a) Mogadishu correct answer using the codes given below
(b) Singapore the lists
(c) Colombo
(d) Manila List I (Strait) List II (Countries)
119. The largest Barrier Reef System in the world A. Bass Strait 1. UK and France
is found at [CDS 2020 (II)] B. Davis Strait 2. Australia and
(a) East Australian Coast Tasmania
(b) West Australian Coast
(c) North Australian Coast C. Dover Strait 3. USA and Cuba
(d) South Australian Coast D. Florida Strait 4. Canada and
120. Which one of the following countries is not Greenland
located on the Tropic of Capricorn? Code: [CDS 2017 (II)]
[CDS 2020 (II)] A B C D
(a) Chile (a) 2 4 1 3
(b) Brazil
(c) Paraguay (b) 2 1 4 3
(d) Uruguay (c) 3 4 1 2
(d) 3 1 4 2
121. Which one of the following mountains
separates Black Sea and Caspian Sea? 125. Which one of the following island is the
[CDS 2019 (II)] largest? [CDS 2017 (II)]
(a) Urals (a) Borneo
(b) Caucasus (b) Madagascar
(c) Carpathians
(c) New Guinea
(d) Balkan mountains
(d) Sumatra
122. Which one of the following statements
with regard to growth of coral reefs is not 126. The following consist of two statements,
correct? [CDS 2018 (II)] statement I and statement II. Examine these
(a) Coral can grow abundantly in fresh water. two statements carefully and select the
(b) It requires warm water between correct answer using the codes given below.
23°C-25°C.
Statement I: Global warming signifies the
(c) It requires shallow saltwater, not deeper
than 50 m. rise in global surface temperature.
(d) It requires plenty of sunlight to aid Statement II: The increase of concentration
photosynthesis. of greenhouse gases in the atmosphere causes
123. Match List I with List II and select the the rise in global surface temperature.
correct answer using the codes given below Codes [CDS 2017 (II)]
the lists (a) Both the statements are individually true
List I (Major Dam) List II (State) and statement II is the correct explanation
of statement I
A. Cheruthoni 1. Madhya Pradesh (b) Both the statements are individually
Dam
true, but statement II is not the correct
B. Indira Sagar 2. Tamil Nadu explanation of statement I
Dam
(c) Statement I is true, but statement II is
C. Krishnaraja 3. Karnataka false
Sagar
(d) Statement I is false, but statement II is
D. Mettur Dam 4. Kerala true

295 PYQ Workbook


GENERAL GEOGRAPHY

127. The boundary layer of the forest at which 133. Which of the following cities experience the
the energy exchange occurs, and some warm temperate Mediterranean climate?
insolation is returned directly to space is 1. Cape Town
[CAPF 2022] 2. Los Angeles
(a) forest floor 3. Adelaide
(b) forest soil
(c) forest canopy 4. Santiago
(d) forest litter Select the correct answer using the code given
below. [CAPF 2018]
128. Which of the following factors are needed
for the growth of corals? (a) 1 and 2 only
1. Freshwater (b) 3 and 4 only
2. Clear shallow salt water (c) 1, 2 and 3 only
(d) 1, 2, 3 and 4
3. Plenty of sunlight for photosynthesis
4. Warm water between 23°C and 25°C 134. Which one of the following pairs of Island
Select the correct answer using the codes and Sea/Ocean is not correctly matched?
given below: [CAPF 2021] [CAPF 2017]
(a) 1, 3 and 4 only (a) Cyprus: Mediterranean Sea
(b) 2, 3 and 4 only (b) Falkland: Atlantic Ocean
(c) 1 and 2 only (c) Chagos: North Pacific Ocean
(d) 1, 2, 3 and 4 (d) Islas Cocos: Indian Ocean
129. Which one of the following does NOT 135. Which one of the following is located at the
illustrate the importance of the Coral Reef? southernmost latitude? [CAPF 2017]
[CAPF 2020] (a) South Africa
(a) They provide employment to the people (b) Australia
(b) They supply building materials (c) Chile
(c) They protect shoreline from storms
(d) Polynesia
(d) They protect communities from
earthquakes 136. Which one of the following cities has greater
130. Which one of the following is NOT annual range of temperature? [CAPF 2016]
Mediterranean country? [CAPF 2020] (a) Kuala Lumpur
(a) Tunisia (b) New Delhi
(b) Libya (c) Shanghai
(c) North Sudan (d) Ulaanbaatar
(d) Egypt 137. Match list I with list II and select the correct
131. Singapore is known as Port of Call because answer using the code given below the lists.
[CAPF 2020]
List I (Climate List II (Major area)
(a) it deals in the processing and shipping of type in letter
oil code)
(b) it is a deep-water port built away from the
actual port A. Aw 1. Central California
(c) it is on main sea route where ships use to B. Af 2. India Subcontinent
anchor for refuelling, watering and taking C. Cs 3. North and South of
food items Amazon Forests
(d) it serves the parent ports by receiving the
ships which are unable to approach due to D. Am 4. Western Equatorial
their large size Africa
Codes: [CAPF 2016]
132. The Tropic of Cancer passes through which
one of the following countries?[CAPF 2018] A B C D
(a) North Sudan (a) 3 1 4 2
(b) Chad (b) 2 4 1 3
(c) Mali (c) 3 4 1 2
(d) Yemen (d) 2 1 4 3

PYQ Workbook 296


GENERAL GEOGRAPHY

138. Which one of the following is the correct [Jharkhand P.C.S. (Pre) 2013]
order of formation of geological systems in (a) Contours
India in terms of their age? (Starting with (b) Hachures
the oldest) [CAPF 2016] (c) Layer colouring
(a) Dharwars -Aravallis – Vindhyans - (d) Hill shading
Cuddapahs 145. The Indian Institute of Remote Sensing is
(b) Aravallis – Dharwars – Cuddapahs - located at- [Chhattisgarh P.C.S. (Pre) 2018]
Vindhyans
(c) Vindhyans – Dharwars – Aravallis - (a) Bengaluru
Cuddapahs (b) Hyderabad
(d) Cuddapahs – Vindhyans – Dharwars - (c) Ahmedabad
Aravallis (d) Dehradun

139. Which one among the following countries 146. Which of the following is the National
of Asia is land-locked? [CAPF 2014] Water Way number 1?
(a) Vietnam [Chhattisgarh P.C.S. (Pre) 2017]
(b) Cambodia (a) Godavari–Krishna Basin Water Way
(c) Laos (b) Mahanadi–Brahmani River Water Way
(d) Thailand (c) Brahmaputra River Water Way
(d) Ganga–Bhagirathi–Hugli River Water
140. Which of the following Islands are made of Way
small coral atolls and reefs in the Arabian (e) None of these
Sea? [CAPF 2014]
147. ‘Jawahar Tunnel’ passes through which
(a) Maldives
place? [Chhattisgarh P.C.S. (Pre) 2014]
(b) Andaman
(c) Sri Lanka (a) Pir Panjal
(d) Lakshadweep (b) Banihal
(c) Burjila
141. Which meridian forms the boundary line (d) Jojila
between the Indian Ocean and the Pacific
Ocean? [CAPF 2014] 148. 38th Parallel border lines divide which of
the two following countries?
(a) The meridian of Cape of Tasmania
(b) The meridian of Cape Town [Chhattisgarh PCS (Pre) 2008]
(c) The meridian of Wellington (a) Poland and Germany
(d) The meridian of Perth (b) Pakistan and Afghanistan
(c) India and Tibet
142. Which one of the following pairs is not (d) North Korea and South Korea
correctly matched?
[Jharkhand PCS (Pre) 2016] 149. Isobaths are used to show–
[Chhattisgarh P.C.S. (Pre) 2003]
(Area) (Important Resources) (a) Rainfall
(b) Pressure
(a)Kolhan Mineral Resources
(c) Time
(b) Kutch Forest Resources (d) Depth
150. Which one of the following pairs is not
(c) Malabar Coast Water Resources correctly matched?
(d)Middle Ganga Soil Resources [Uttarakhand PCS (Pre) 2021]
Plain (a) Foehn - Alps Mountain
(b) Bora - Siberia
143. The power plant of Manikaran based on (c) Mistral - Rhone Valley
geothermal energy is in the State of- (d) Khamsin - Egypt
[Jharkhand P.C.S. (Pre) 2013] 151. What is Dakshin Gangotri?
(a) Arunachal Pradesh [Uttarakhand P.C.S. (Pre) 2016]
(b) Himachal Pradesh (a) River Valley in Andhra Pradesh
(c) Jammu and Kashmir (b) Indian Research Station in Antarctica
(d) Uttarakhand (c) Second source of river Ganga in the South
144. What is the most accurate method of of Gangotri
representing relief features? (d) Island in the Indian Ocean near Antarctica

297 PYQ Workbook


GENERAL GEOGRAPHY

152. Which of the following is not included in the (c) 4 2 3 1


geographical group of countries designated (d) 4 3 2 1
with the name Oceania?
[Uttarakhand P.C.S. (Pre) 2010] 157. Match list-I with list-II and select the
(a) Indonesia correct answer using the codes given below
(b) Melanesia the lists:
(c) Micronesia
(d) Australia List-I List-II
153. The largest part of our hydrosphere is– (Mountain Peak) (Location/
Country)
[Uttarakhand P.C.S. (Pre) 2010]
(a) Atlantic Ocean A. Elbrus 1. New Zealand
(b) Indian Ocean B. Kilimanjaro 2. USA
(c) Pacific Ocean C. Mt. Cook 3. Tanzania
(d) Antarctica Ocean
D. McKinley 4. Russia
154. Which one of the following has the largest
harbour in the world? Code: [RAS/RTS (Pre) 2018]
[Uttarakhand P.C.S. (Pre) 2005] A B C D
(a) London (a) 3 4 2 1
(b) Colombo (b) 4 2 3 1
(c) Rotterdam
(d) New York (c) 4 3 1 2
(d) 3 1 4 2
155. Match list-I with list-II and select the
correct answer using the codes given below 158. Match list-I with list-II and select the
the lists: correct answer using the codes given below
List-I List-II the lists:
(Site) (Energy) List-I List-II
A. Pavagada 1. Wind (Mountain Peak) (Continent)
B. Tattapani 2. Solar A. Kosciuszko 1. Europe
C. Muppandal 3. Geo-thermal B. McKinley 2. Africa
D. Kakrapar 4. Atomic C. Elbrus 3. Australia
Code: [RAS/RTS (Pre) 2021] D. Kilimanjaro 4. North America
A B C D Code: [RAS/RTS (Pre) 2016]
(a) 3 2 1 4
(b) 1 3 2 4 A B C D
(c) 1 2 3 4 (a) 3 4 1 2
(d) 2 3 1 4 (b) 2 4 3 1
156. Match list-I with list-II and select the (c) 4 3 2 1
correct answer using the codes given below (d) 3 1 2 4
the lists:
159. Match list-I with list-II and select the
List-I List-II correct answer using the codes given below
(Landform) (Continent) the lists:
A. Aconcagua 1. Australia A. Strait of Gibraltar 1. Between
B. Meseta 2. Africa Indonesia &
C. Serengeti Plains 3. Europe Malaysia
D. Gibson desert 4. South America B. Strait of Malacca 2. Between Gulf of
Code: [RAS/RTS (Pre) 2021] Persia and Gulf of
Oman
A B C D
(a) 3 2 4 1 C. Strait of Bering 3. Between Africa
(b) 2 4 1 3 and Europe

PYQ Workbook 298


GENERAL GEOGRAPHY

D. Strait of Hormuz 4. Between Asia and (c) Rann of Kutch


North America (d) Gulf of Kutch
Code: [RAS/RTS (Pre) (Re-Exam) 2013] 166. Where is Sargasso Sea situated?
A B C D [MPPCS (Pre) 2021]
(a) 4 2 3 1 (a) North Atlantic Ocean
(b) 3 4 1 2 (b) South Atlantic Ocean
(c) 3 1 4 2 (c) Indian Ocean
(d) 4 3 2 1 (d) North Pacific Ocean
160. Which of the railway connecting the 167. Out of the following which combination is
following two stations has been recognized incorrect? [M.P.P.C.S. (Pre) 2019]
as heritage railway line by UNESCO? (a) Telegraph Plateau - Indian Ocean
[R.A.S./R.T.S. (Pre) 2013] (b) Cocoa Ridge - Pacific Ocean
(a) Siliguri and Darjeeling (c) Walvis Ridge - Atlantic Ocean
(b) Allahabad and Varanasi (d) Agulhas Basin - Indian Ocean
(c) Mumbai and Thane 168. Golden Quadrilateral Project of India
(d) Amritsar and Ludhiana joins- [M.P.P.C.S. (Pre) 2016]
161. Arrange the following countries in (a) Delhi–Mumbai–Chennai–Kolkata
descending order of their area and select (b) Delhi–Jhansi–Bengaluru–Kanyakumari
the correct answer using codes given below- (c) Srinagar–Delhi–Kanpur–Kolkata
(i) Brazil (d) Porbandar–Bengaluru–Kolkata–Kanpur
(ii) Canada 169. Which is not a renewable resource?
(iii) China [M.P.P.C.S. (Pre) 2015]
(iv) U.S.A. (a) Energy of water
Code: [R.A.S./R.T.S. (Pre) 2012] (b) Energy of Sun
(a) iii, iv, i, ii (c) Energy of Earth
(b) ii, iii, iv, i (d) None of the above
(c) ii, iv, iii, i 170. Which one of the following is known as the
(d) i, iii, ii, iv ‘Coffee port’ of the world?
162. Generally, where are coral reefs found? [M.P.P.C.S. (Pre) 2015]
[R.A.S./R.T.S. (Pre) 2008] (a) Sao Paulo
(a) In temperate climate above 180C (b) Santos
(b) Between Tropic of Cancer and Tropic of (c) Rio de Janeiro
Capricorn coastal areas (d) Buenos Aires
(c) On both east and west coasts of continents 171. The line joining the areas having the same
and islands only rainfall is called- [M.P.P.C.S. (Pre) 2015]
(d) On colder seacoasts (a) Isobar
163. The largest river of the world in terms of (b) Isohyet
drainage area is- [R.A.S./R.T.S. (Pre) 1999] (c) Isotherm
(a) Nile (d) Isohaline
(b) Amazon 172. Which of these countries forms the longest
(c) Congo territorial border with India?
(d) Mississippi-Missouri [M.P.P.C.S. (Pre) 2013]
164. The inland sea is the: (a) China
[R.A.S./R.T.S. (Pre) 1999] (b) Pakistan
(a) White Sea (c) Bangladesh
(b) Black Sea (d) Nepal
(c) Caspian Sea 173. The biggest desert in the world is:
(d) Sea of Japan
[M.P.P.C.S. (Pre) 2010]
165. Kandla port is located on: (a) Kalahari
[R.A.S./R.T.S. (Pre) 1996] (b) Gobi
(a) Gulf of Khambhat (c) Sahara
(b) Tropic of Cancer (d) Thar

299 PYQ Workbook


GENERAL GEOGRAPHY

174. Arrange the following from west to east and (c) Russia
select the correct answer from the codes (d) Brazil
given below:
181. Isogonic lines are:
1. London
[U.P. Lower Sub. (Pre) 1998]
2. Lisbon
(a) Same atmospheric pressure lines
3. Frankfurt (b) Same temperature lines
4. Beirut (c) Same salinity lines
Code: [M.P.P.C.S. (Pre) 2008] (d) Same magnetic declination line
(a) 1, 2, 3, 4
(b) 2, 1, 3, 4 182. Three mountain ranges determining the
(c) 2, 1, 4, 3 boundary between India and Myanmar are-
(d) 1, 2, 4, 3 [U.P. Lower Sub. (Pre) 1998]
(a) Khasi, Patkoi and Arakan Yoma
175. White Mountains are founded in (b) Akai Mountain Range
[U.P.U.D.A./L.D.A. (Pre) 2001] (c) Great Dividing Range
(a) Canada (d) None of the above
(b) Norway
(c) Russia 183. Which of the following countries does not
(d) United States of America have a land border with the Dead Sea?
[67th BPSC (Pre) 2022]
176. Chisapani Gorge is located in– (a) Lebanon
[U.P. Lower Sub. (Pre) 2015] (b) Jordan
(a) India (c) Israel
(b) Nepal (d) Palestine
(c) Bangladesh (e) None of the above/More than one of the
(d) Pakistan above
177. Which of the following islands are known 184. Addu Atoll, is situated in which ocean?
as ‘Sheep Islands’? [66th B.P.C.S. (Pre) (Re-Exam) 2020]
[U.P. Lower Sub. (Pre) 2015] (a) Atlantic Ocean
(a) Canary Islands (b) Arctic Ocean
(b) Madeira Islands (c) Indian Ocean
(c) Faroe Islands (d) Pacific Ocean
(d) Wellington Islands (e) None of the above/More than one of the
178. Which of the following rivers crosses the above
Equator twice? [U.P. Lower Sub. (Pre) 2015] 185. The Great Victorian Desert is located at–
(a) Zaire [60th to 62nd B.P.S.C. (Pre) 2016]
(b) Niger (a) Australia
(c) Nile (b) India
(d) Amazon (c) Egypt
179. Which of the following countries is trans- (d) North Africa
Continental? [U.P. Lower Sub. (Pre) 2009] (e) None of the above/More than one of the
1. Laos above
2. Georgia 186. Which strait connects the Red Sea and the
3. Turkey Indian Ocean?
4. Tunisia [53rd to 55th B.P.S.C. (Pre) 2011]
Select the correct answer from the codes (a) Bab-El-Mandeb
given below: (b) Hormuz
(a) 1 and 2 (c) Bosphorus
(b) 2 and 3 (d) Malacca
(c) 1 and 4 187. Where is ‘Ninety East Ridge’ situated?
(d) 3 and 4
[53rd to 55th B.P.S.C. (Pre) 2011]
180. The Great Artesian Basin (GAB) is located (a) Pacific Ocean
in– [U.P. Lower Sub. (Spl 2003] (b) Indian Ocean
(a) U.S.A (c) Atlantic Ocean
(b) Australia (d) Arctic Ocean

PYQ Workbook 300


GENERAL GEOGRAPHY

188. Which is known as ‘Roof of the World’? (c) New York


[44th B.P.S.C. (Pre) 2000] (d) London
(a) Aravalli
(b) Satpura 190. On the sea level, nearest place to the center
(c) Pamir of earth? [38th B.P.S.C. (Pre) 1992]
(d) Myanmar
(a) North pole
189. Which one of the following is not a sea-port
city? [40th B.P.S.C. (Pre) 1995] (b) Tropic of Capricorn
(a) Tokyo (c) Tropic of Cancer
(b) Canberra (d) Tropic of equator

301 PYQ Workbook


GENERAL GEOGRAPHY

SOLUTIONS

7.1. UPSC CSE Previous Years’ Questions


1. Solution: (a)
Exp) Option a is the correct answer.
Option a is correct: The Congo Basin is a vast region
in Central Africa, encompassing multiple countries. It
is primarily characterized by the Congo Rainforest, the
second-largest rainforest in the world. The Congo Basin
spans across six countries—Cameroon, Central African
Republic, Democratic Republic of the Congo, Republic of
the Congo, Equatorial Guinea and Gabon.
Option b is incorrect: Nigeria is located in West Africa and
is not part of the Congo Basin. It is situated to the west of
Cameroon.
Option c is incorrect: South Sudan is located in East- Pair 3 is incorrectly matched: The Tigray region, also known
Central Africa, and while it is geographically closer to the as the Tigray State, is a region located in the northern part
Congo Basin compared to Nigeria, it does not fall within the of Ethiopia. It is one of the ten regional states that make
up the federal system of Ethiopia. The first formal African
basin itself. The Congo Basin is mainly found to the south of
Union-led peace talks between an Ethiopian government
South Sudan.
team and Tigray forces are happening in South Africa.
Option d is incorrect: Uganda is also located in East Africa,
north of Rwanda and Tanzania. While it is relatively close to
the eastern edge of the Congo Basin, it does not lie entirely
within the basin.

2. Solution: (d)
Exp) Option d is the correct answer.
Pair 1 is incorrectly matched: Donetsk and Luhansk are two
states located in eastern Ukraine, which share a border with
Russia. Within these two states are two separatist territories
known as the Donetsk People’s Republic (DPR) and Luhansk
People’s Republic (LPR), which are run by Russian and
Russian-backed separatists. This entire region, which 3. Solution: (a)
includes Donetsk, Luhansk and their respective separatist Exp) Option a is the correct answer.
territories, is generally referred to as the ‘Donbas’ region. Ukraine is bordered by Belarus to the north, Russia to the
east, the Sea of Azov and the Black Sea to the south, Moldova
and Romania to the southwest, and Hungary, Slovakia, and
Poland to the west.

Pair 2 is incorrectly matched: The Kachin region, also


known as Kachin State, is a state located in the northern
part of Myanmar (Burma). It shares borders with China
to the north and east, Shan State to the south, and Sagaing In the given countries only Hungary and Romania are
Region to the west. correct. Thus only two are correct.

PYQ Workbook 302


GENERAL GEOGRAPHY

4. Solution: (c)
Exp) Option c is the correct answer.
Afghanistan, officially the Islamic Republic of Afghanistan,
is a mountainous landlocked country in southern Central
Asia. It borders Tajikistan, Turkmenistan, and Uzbekistan
to the north, Iran to the west, Pakistan to the east and
south.

Pair 3 is incorrectly matched: Cabo Delgado is the


northernmost province of Mozambique (not Spain). The
region is an ethnic stronghold of the Makonde tribe, with
the Makua and Mwani as leading ethnic minorities.
5. Solution: (b)
Exp) Option b is the correct answer
Pair 1 is correctly matched: Anatolia, also called Asia
Minor, the peninsula of land that today constitutes the Asian
portion of Turkey. Because of its location at the point where
the continents of Asia and Europe meet, Anatolia was, from
the beginnings of civilization, a crossroads for numerous
peoples migrating or conquering from either continent.

Pair 4 is incorrectly matched: Catalonia occupies a


triangular area in the north-eastern corner of Spain (not
Italy). Catalonia was formerly a principality of the crown of
Aragon, and it has played an important role in the history
of the Iberian Peninsula. From the 17th century it was the
centre of a separatist movement that sometimes-dominated
Spanish affairs.

Pair 2 is correctly matched: The Amhara Region is a


regional state in northern Ethiopia and the homeland of the
Amhara people. Its capital is Bahir Dar which is the seat of
the Regional Government of Amhara. Amhara is bordered by
Sudan to the west and northwest and by other the regions of
Ethiopia: Tigray to the north, Afar to the east, Benishangul-
Gumuz to the west and southwest, and Oromia to the south.

303 PYQ Workbook


GENERAL GEOGRAPHY

6. Solution: (c) Pair 2 and 3 are incorrectly matched. Kirkuk and Mosul
Exp) Option c is the correct answer. are cities located in Iraq. Kikuk is a region in Iraq which is
claimed by the Kurds, who controlled it for a brief period
Option 1 is incorrectly matched. Mekong river originates
in 2017. Mosul, located in Iraq witnessed major battles to
in Tibet, flows through China, Myanmar, Laos, Thailand,
defeat ISIS. Mosul was retaken from ISIS in 2017.
Cambodia and Vietnam before draining into the
South China Sea. It does not reach the Andaman Sea. Pair 4 is correctly matched. Mazar-i-sharif is a city in
Option 2 is incorrectly matched. Thames is the longest Afghanistan, which witnessed a massive Taliban attack on
river of England. It flows through the Thames Estuary before Afghanistan Armed Forces in 2017. Also, Indian consulate
draining into the North Sea. in Mazar-i-Sharif was attacked in 2016.
Option 3 is correctly matched. Volga is the longest river of Thus, all the places mentioned in the options are conflict
Europe. It Rises in the Valdai Hills northwest of Moscow, and ridden areas.
discharges into the Caspian Sea
10. Solution: (a)
Option 4 is correctly matched. Zambezi is a major river
Exp) Option a is the correct answer.
of Africa. It originates in Zambia, flows through Angola,
Namibia, Botswana, Zimbabwe and Mozambique before Sumatra, and Java are islands located in Indonesia.
draining into the Indian Ocean. Option a is correct. With distance of 1192 kms Sumatra is
geographically closest to the southernmost and largest of
7. Solution: (b)
the Nicobar Islands i.e. Great Nicobar.
Exp) Option b is the correct answer.
Option b is incorrect. Distance between Great Nicobar and
Option 1 is correct. Adriatic Sea borders Albania. Borneo islands is 2398 kms. Borneo islands lies in Malay
Option 2 is incorrect. Black Sea borders Russia, Turkey, peninsula and it is shared by Malaysia, Indonesia and Brunie.
Georgia, Romania, Ukraine and Bulgaria. Adriatic Sea Option c is incorrect. Distance between Great Nicobar and
borders Croatia. Java islands is 1817 kms.
Option 3 is correct. Russia, Iran, Azerbaijan, Kazakhstan Option d is incorrect. Distance between Great Nicobar and
and Turkmenistan are countries bordering Caspian Sea. Sri Lanka is 1437 kms.
Option 4 is correct. Morocco which lies in North western
Africa borders the Mediterranean Sea near Strait of
Gibraltar.
Option 5 is incorrect. Red Sea borders Egypt, Sudan, Yemen,
Saudi Arabia, Eritrea and Djibouti. The Mediterranean Sea
borders Syria.

8. Solution: (c)
Exp) Option c is the correct answer.
All the regions mentioned have been in news for seeking
autonomy from the countries they are located in.
Pair 1 is correct. Catalonia is a autonomous region in Spain.
They are seeking complete independence. 11. Solution: (c)
Pair 2 is incorrect. Till 2014, Crimea was part of Ukraine. Exp) Option c is the correct answer.
Following pro-Russian demonstrations, it was annexed by
Option 1 is incorrect. Jordan is bounded to the north by
Russia.
Syria, to the east by Iraq, to the southeast and south by Saudi
Pair 3 is correct. Mindanao is an Island in Philippines. Arabia, and to the west by Israel and the West Bank. It opens
Centuries of conquest first by Spanish and American colonial into the Red Sea at its southern most point.
forces that had ruled the Philippine archipelago followed by
Option 2 is incorrect. Iraq is bordered to the north by
Filipino Christian settlers have gradually turned Muslims
Turkey, to the east by Iran, to the west by Syria and Jordan,
into a minority group in Mindanao, triggering conflict over
and to the south by Saudi Arabia and Kuwait. It has access to
land, resources and sharing of political power.
Persian Gulf in the South
Pair 4 is incorrect. Oromia is a state in Ethiopia, which has
Option 3 and 4 are correct. Following countries have a
seen struggle for autonomy in the past by the Oromo people.
shoreline at the Mediterranean Sea:
9. Solution: (b) European countries: Spain, France, Italy, Malta, Monaco,
Exp) Option b is the correct answer. Slovenia, Croatia, Bosnia and Herzegovina, Montenegro,
Pair 1 is correctly matched. Aleppo is a city in northern Albania, and Greece.
Syria. It used to be largest city of Syria before civil war broke Western Asia (Middle East):Turkey, Cyprus, Syria, Lebanon,
out in 2011. Israel, and the Palestine Gaza Strip.

PYQ Workbook 304


GENERAL GEOGRAPHY

Slovenia, Spain, Syria, Tunisia, and Turkey share their border


with Mediterranean Sea.

Fig. Countries surrounding Mediterranean Sea

12. Solution: (b)


Fig. Countries surrounding Mediterranean Sea
Exp) Option b is the correct answer.
14. Solution: (b)
The journey from Kohima in Nagaland to Kottayam in
Kerala involves travelling across the breadth of the country Exp) Option b is the correct answer.
from north-east to south-west. The route involves passing Turkey is located between Mediterranean Sea to the South-
through the following states:
West and Black Sea to the North.
• Nagaland (Origin)
• Assam
• West Bengal
• Odisha
• Andhra Pradesh
• Tamil Nadu or Karnataka
• Kerala (Destination)
Therefore, the minimum number of states that one would
have to travel through while going from Kohima to Kottayam
is 7. 15. Solution: (c)
Exp) Option c is the correct answer.
The sequence of the given cities from South to North is
Jakarta, Singapore, Bangkok and Hanoi.

16. Solution: (d)


Exp) Option d is the correct answer.
NH4: NH 4 is the main highway in the Andaman and
Nicobar Islands, spanning 230.7 km, connecting Port Blair
to Diglipur.

13. Solution: (b) NH6: NH 6 is the main highway in the Meghalaya, spanning
Exp) Option b is the correct answer. 667 km, running from Jorabat in the north to the India/
Myanmar border at Zokhawthar in the south.
21 countries including Albania, Algeria, Bosnia and
Herzegovina, Croatia, Cyprus, Egypt, France, Greece, Israel, NH15: National Highway 15, stretching 664 km, runs from
Italy, Lebanon, Libya, Malta, Monaco, Montenegro, Morocco, Baihata in Assam to Wakro in Arunachal Pradesh, India.

305 PYQ Workbook


GENERAL GEOGRAPHY

Important Tips
The Great Victoria Desert is a sparsely populated desert
ecoregion and interim Australian bioregion in Western
Australia and South Australia.
The Grand Canyon is a steep-sided canyon carved by the
Colorado River in Arizona, United States. The Grand
Canyon is 277 miles (446 km) long, up to 18 miles (29
km) wide and attains a depth of over a mile (6,093 feet or
1,857 meters).
Lake Winnipeg is an extremely large, relatively shallow
24,514-square-kilometre (9,465 sq mi) lake in North
America, in the province of Manitoba, Canada. Its
southern end is about 55 km (34 mi) north of the city of
Winnipeg.
The Southern Alps are a mountain range extending
along much of the length of New Zealand’s South Island,
reaching its greatest elevations near the range’s western
side.

18. Solution: (b)


Exp) Option b is the correct answer.
Dahej is a non-major port. It is a cargo port situated on
Important Tips
the South-west coast of Gujarat, India in Bharuch district.
Old NH4: Before the National Highway renumbering in
There is 17.5 million tonnes per year capacity LNG terminal
Western and Southern India, National Highway 4 (NH
operated by Petronet in Dahej. Kochi, Paradip and New
4) was a significant route spanning 1,235 km, connecting
four major Indian cities: Mumbai, Pune, Bangalore, and Mangalore Ports are the three major ports out of 13 major
Chennai. It traversed the states of Maharashtra, Karnataka, ports in India.
and Tamil Nadu. Today, it is designated as National
Important Tips
Highway 48 (India).
New Mangalore Port, situated in Panambur, Karnataka,
Old NH6: National Highway 6, also known as Economic
India, spans approximately 480 hectares. It’s the deepest
Corridor 1 (EC1), originally spanned 1,949 km from
inner harbor on the west coast, the sole major port in
Surat to Kolkata, and was referred to as Asian Highway
Karnataka, and ranks as India’s seventh-largest port. The
46 (AH46), Mumbai-Kolkata Highway, and Great
port is managed by the New Mangalore Port Authority
Eastern Highway. In a 2010 notification by the Ministry
(NMPA).
of Road Transport and Highways, it was renumbered as
follows: Hajira-Deogarh became part of NH 53, Deogarh- Cochin Port, located in Kochi, Kerala, is a major port
Kharagpur as NH 49, and Kharagpur-Kolaghat-Kolkata as along the Arabian Sea - Laccadive Sea - Indian Ocean
NH 16. route. It’s one of India’s largest ports and the nation’s first
transshipment port. The port lies on two islands in the Lake
Old NH15: In old NH numbering, National Highway 15
of Kochi: Willingdon Island and Vallarpadam, towards
(NH 15) connected Pathankot in Punjab to Samakhiali in
the Fort Kochi river mouth opening onto the Laccadive
Gujarat, spanning 1,526 km, passing through Rajasthan.
Sea. The International Container Transshipment Terminal
(ICTT) within Cochin Port is India’s largest container
17. Solution: (b)
transshipment facility.
Exp) Option b is the correct answer.
19. Solution: (b)
The correct match for each geographic feature with its Exp) Option b is the correct answer.
respective country is as follows: Malta is a small island country in the Mediterranean Sea,
located south of Sicily and north of Africa. Malta has a
A. Great Victoria Desert - 1. Australia
long and rich history, dating back to the Phoenicians in the
B. Grand Canyon - 4. USA 8th century BC. It has been ruled by a number of different
empires over the centuries, including the Romans, the
C. Lake Winnipeg - 2. Canada
Arabs, the Normans, the Spanish, and the British. Malta
D. Southern Alps - 3. New Zealand gained its independence from the British in 1964.

PYQ Workbook 306


GENERAL GEOGRAPHY

Important Tips
International Date Line (IDL):
• The International Date Line (IDL) is an imaginary line
running from the North to the South Pole.
• It separates two calendar days and is crucial for
measuring time, communication, politics, and
commerce.
• The IDL approximately follows the 180° line of
longitude and passes through the Pacific Ocean.
20. Solution: (d) • However, it is not a straight line and deviates from
the 180° meridian at certain points.
Exp) Option d is the correct answer.
• These deviations create zig-zag patterns, moving east
Singapore is the city nearest to the equator among the
or west, to allow places with the same affiliations to
options provided. Its coordinates are approximately
share the same date and time.
1.3521° North latitude, making it very close to the equator.
• Without these deviations, landmasses could be divided
The equator is at 0° latitude, so Singapore is just slightly north
into different dates on the same day.
of this line. In contrast, the other cities listed are located at
• Starting from the North Pole, the IDL bends eastward
higher latitudes, either north or south of the equator.
near the Chukchi Peninsula and Wrangel Island,
Cities Coordinates then moves through the Bering Strait.
Singapore: 1.3521° N, 103.8198° E • It then heads west between St. Matthew and St.
Manila: 14.5995° N, 120.9842° E Lawrence Islands.
Jakarta 6.2088° S, 106.8456° E • Continuing, it runs between the Commander Islands
and the Aleutian Islands, returning to the 180°
Colombo 6.9271° N, 79.8612° E
meridian until it crosses the equator.
21. Solution: (b) • South of the equator, it swings eastward near the
Exp) Option b is the correct answer. 150° meridian, encircling Kiribati.

The Bering Strait is the narrowest strait between Asia • Then, it shifts westward, remaining east of the 180°
longitude, passing through American Samoa and
and North America, separating the Chukchi Sea from the
Samoa.
Bering Sea. It is also the closest strait to the International
Date Line, which passes through the middle of the strait. • Finally, the line bends southwestward and follows the
180° meridian until it reaches Antarctica.

22. Solution: (a)


Exp) Option a is the correct answer.
The largest coral reef in the world is the Great Barrier Reef,
which is located off the coast of Queensland, Australia. The
Great Barrier Reef is composed of over 2,900 individual reefs
and 900 islands stretching for over 2,300 kilometres (1,400
mi) over an area of approximately 344,400 square kilometres
(133,000 sq mi). It is a UNESCO World Heritage Site and a
natural wonder that supports a rich diversity of marine life.

307 PYQ Workbook


GENERAL GEOGRAPHY

Important Tips South America: 1. Ecuador, 2. Colombia, 3. Brazil

Coral Triangle: Africa: 4. Gabon, 5. Congo, 6. Democratic Republic of


Congo, 7. Uganda, 8. Kenya, 9. Sao Tome and Principe, 10.
The Coral Triangle is a region of the western Pacific Ocean
Somalia.
that is known for its high biodiversity and richness of
coral reefs. It covers the waters of six countries: Indonesia, Asia: 11. Maldives, 12. Indonesia, 13. Kiribati (Oceania)
Malaysia, the Philippines, Papua New Guinea, Timor- Water Bodies: Atlantic Ocean, Pacific Ocean, Indian
Leste and the Solomon Islands. Ocean.
The Coral Triangle has been called the “Amazon of the
seas” because it contains more than 75% of the world’s
coral species, 37% of its reef fish species, and six out of
seven of its marine turtle species.
It also supports millions of people who depend on its
resources for food, income and protection from storms.

23. Solution: (c)


Exp) Option c is the correct answer.
Copacabana Beach is located in Rio de Janeiro, Brazil. It is
a 4 km (2.5 miles) balneario beach renowned for its vibrant
atmosphere, hotels, restaurants, and annual New Year’s Eve
celebrations. Copacabana is a neighborhood in the South
27. Solution: (d)
Zone of Rio de Janeiro, and the beach runs from Posto Dois
to Posto Seis, with historic forts at both ends. It is a popular Exp) Option d is the correct answer.
destination for tourists and hosts various events, such as Myanmar is bordered by Bangladesh and India to the north-
the FIFA Beach Soccer World Cup in 2007 and Olympic west, China to the northeast, Laos and Thailand to the east
matches of beach volleyball in 2016. and southeast, and the Andaman Sea and the Bay of Bengal
to the south and southwest.
24. Solution: (c)
28. Solution: (c)
Exp) Option c is the correct answer.
Exp) Option c is the correct answer.
The tunnel that connects the United Kingdom and France
is called the Channel Tunnel or the Chunnel, and it runs Huangpu River is a landmark of Shanghai, separating the
under the Strait of Dover, which is a narrow passage of city into two parts, Pudong and Puxi. It runs through over 10
water between the English Channel and the North Sea. The districts in the city and has a total length of 113 km (70 mi)
tunnel is about 50 km long and 115 m deep at its lowest and a width of 300-770 m (984-2,526 ft).
point. It carries both passenger and freight trains between Important Tips
Folkestone, Kent, in England and Coquelles, Pas-de-Calais,
• Beijing: This is the capital and largest city of China,
in France.
located in the north of the country. It is not crossed by
25. Solution: (b) the Huangpu River, but by several other rivers, such as
Exp) Option b is the correct answer. the Yongding, Chaobai, and Juma.
The Diamantina Trench is situated in the Indian Ocean. • Ho Chi Minh City: This is the largest and most
It is one of the deepest points in the Indian Ocean, with populous city in Vietnam, located in the south of the
a maximum depth of about 8,047 meters. The trench is country. It is not crossed by the Huangpu River, but by
approximately 520 kilometers long and 70 kilometers wide, the Saigon River and several canals.
running in a northeast-southwest direction. It is located to • Manila: This is the capital and largest city of the
the south of the mid-eastern Indian Ocean, west of Perth, Philippines, located on the island of Luzon. It is not
Australia, and is named after the Diamantina River in crossed by the Huangpu River, but by the Pasig River,
Australia. which flows into Manila Bay. Manila Bay is connected
to the South China Sea.
26. Solution: (b)
Exp) Option b is the correct answer. 29. Solution: (d)
The equator is a circle of latitude that divides a spheroid, Exp) Option d is the correct answer.
such as Earth, into the northern and southern hemispheres. Statement 1 is correct: A terrestrial mile, also known as
On Earth, it is an imaginary line located at 0 degrees latitude, a statute mile, is shorter than a nautical mile. A terrestrial
about 40,075 km (24,901 mi) in circumference, halfway mile is approximately 1,609.34 meters, while a nautical mile
between the North and South poles. is slightly longer, approximately 1,852 meters.
The Equator passes through 13 countries, 3 continents and Statement 2 is incorrect: The Harmattan is not a wind on
3 water bodies. the East African Coast but rather a dry and dusty trade wind

PYQ Workbook 308


GENERAL GEOGRAPHY

that blows over West Africa, particularly affecting countries 31. Solution: (a)
like Nigeria, Ghana, and Senegal. It is known for carrying Exp) Option a is the correct answer.
dust from the Sahara Desert towards the Gulf of Guinea.
Option a is incorrect- Bratislava is the capital of Slovakia,
Statement 3 is incorrect: Greece and Albania are not part of not Slovenia. The capital of Slovenia is Ljubljana. Slovenia
the Iberian Peninsula. Spain, Portugal, Andorra, Gibraltar, is a small country in Central Europe, bordered by Austria
and a small part of France are on the Iberian Peninsula, with to the north, Hungary to the northeast, Croatia to the
Spain and Portugal making up most of the area. Greece and southeast, and the Adriatic Sea to the southwest. It is mostly
mountainous and forested, with a population of just over 2
Albania are located in southeastern Europe on the Balkan
million people.
Peninsula.
Option b is correct- The Capital of Seychelles is Victoria.
30. Solution: (b) Seychelles is an island country in the Indian Ocean, off the
coast of East Africa. It is known for its beautiful beaches,
Exp) Option b is the correct answer.
crystal-clear waters, and lush rainforests. Seychelles is a
The Bermuda Triangle is an area in the North Atlantic popular tourist destination, and it is also a major financial
Ocean that roughly forms a triangular shape with its center.
vertices at Miami (Florida), Bermuda, and Puerto Rico. Option c is correct- The capital of Sierra Leone is Freetown.
Hawaii Islands are not considered part of the Bermuda Sierra Leone is a country in West Africa. It is bordered by
Triangle. Guinea to the north, Liberia to the southeast, and the Atlantic
Ocean to the southwest. Sierra Leone is a poor country, but
it is rich in natural resources, including diamonds, bauxite,
and iron ore.
Option d is correct- The capital of Uzbekistan is Tashkent.
Uzbekistan is a country in Central Asia. It is bordered by
Kazakhstan to the north, Kyrgyzstan to the northeast,
Tajikistan to the southeast, Afghanistan to the south, and
Turkmenistan to the southwest. Uzbekistan is a landlocked
country, and it is the most populous country in Central Asia.

32. Solution: (d)


Exp) Option d is the correct answer
Champaner-Pavagarh Archaeological Park: This park is
located in the Champaner-Pavagadh district of Gujarat. It is
a UNESCO World Heritage Site that includes the ruins of the
Champaner Fort, the Pavagarh Fort, and a number of other
historical monuments.
Important Tips
The Chhatrapati Shivaji Maharaj Terminus (CSMT) is a
Bermuda Triangle: Railway Station in Mumbai. It was included in the World
The Bermuda Triangle, also known as the “Devil’s Heritage List of UNESCO in 2004. It is a UNESCO World
Triangle,” is a mysterious area in the North Atlantic Heritage Site that is known for its unique blend of Gothic,
Ocean. Indian, and Mughal architectural styles.
It has gained notoriety due to numerous unexplained Mamallapuram: This town is located in the Kancheepuram
disappearances of ships and aircraft. Over 50 ships and district of Tamil Nadu. It is a UNESCO World Heritage Site
20 aircraft are said to have vanished without a trace within that is known for its rock-cut temples and sculptures.
this region. Sun Temple (Konark Temple): This temple is located in the
Puri district of Odisha. It is a UNESCO World Heritage Site
Various theories abound, including magnetic anomalies,
that is known for its intricate carvings and its architectural
rogue waves, and underwater methane hydrates.
beauty.
Some suggest navigational errors near the agonic line,
where magnetic compass variation is nil. 33. Solution: (b)
Storms and converging weather patterns can create Exp) Option b is the correct answer.
treacherous conditions. Option b is correct: The correct sequence of the given
The U.S. National Oceanic and Atmospheric continents in the descending order of their percentage of
Administration (NOAA) notes that there’s no statistical Earth’s land
evidence of unusual incidents in this area compared to Africa: 20.3%
other well-traveled regions. North America: 16.5%
Despite scientific explanations, the Bermuda Triangle South America: 12.6%
continues to captivate imaginations and spark speculation. Europe: 7.3%

309 PYQ Workbook


GENERAL GEOGRAPHY

34. Solution: (b) 36. Solution: (c)


Exp) Option b is the correct answer. Exp) Option c is the correct answer.
Ukraine does not border with Lithuania. The other three Option a is correct- The capital of Bahamas is Nassau. The
countries listed in the question, Poland, Belarus, and Latvia, Bahamas is an island country in the North Atlantic Ocean,
all border Lithuania. southeast of the U.S. state of Florida and north of Cuba. It
Lithuania is a country in the Baltic region of Europe. It is is an archipelago of over 700 islands and cays, and has a
bordered by Latvia to the north, Belarus to the east and population of about 390,000 people.
south, Poland to the south, and Russia to the southwest. Option b is correct- The Capital of Costa Rica is San Jose.
Costa Rica is a country in Central America, bordered by
Nicaragua to the north and Panama to the south. It has a
Caribbean coastline to the east and a Pacific coastline to the
west. Costa Rica is a mountainous country, with a population
of about 5 million people.
Option c is incorrect- The capital of Nicaragua is Managua,
not Belmopan. Belmopan is the capital of Belize. Nicaragua
is a country in Central America, bordered by Honduras to
the north and Costa Rica to the south. It has a Caribbean
coastline to the east and a Pacific coastline to the west.
Nicaragua is a mountainous country, with a population of
about 6.8 million people.
Option d is correct- The Dominican Republic is Santo
Domingo. The Dominican Republic is a country on the
island of Hispaniola in the Caribbean Sea. It is bordered by
Haiti to the west. The Dominican Republic has a population
of about 10 million people.

37. Solution: (a)


35. Solution: (b)
Exp) Option a is the correct answer.
Exp) Option b is the correct answer.
The Seikan Rail Tunnel is not in China; it is located in
Statement 1 is incorrect. Great Britain is a geographical
Japan. It is a 53.85 km dual-gauge railway tunnel in Japan,
term referring to the island of Great Britain, which includes
with a 23.3 km portion under the seabed of the Tsugaru Strait,
England, Wales, and Scotland. Northern Ireland is not part
which separates Aomori Prefecture on the main Japanese
of Great Britain, but it is part of the United Kingdom.
island of Honshu from the northern island of Hokkaido.
Statement 2 is correct. England covers about 58% of the
total area of the United Kingdom. The other three countries, Important Tips
Wales, Scotland, and Northern Ireland, make up the The Petronas Towers, also known as the Petronas Twin
remaining 42% of the UK’s area. Towers, are an interlinked pair of 88-storey supertall
skyscrapers in Kuala Lumpur, Malaysia, standing at
451.9 metres (1,483 feet).
The Appalachian Trail, also called the A.T., is a hiking
trail in the Eastern United States, extending almost 2,200
miles (3,540 km) between Springer Mountain in Georgia
and Mount Katahdin in Maine, and passing through 14
states.
The Roghun Dam is an embankment dam under
construction on the Vakhsh River in southern Tajikistan.
The dam is situated 110 km from Dushanbe.

38. Solution: (d)


Exp) Option d is the correct answer.
A. Thal - 3. Maharashtra
Thal Power Plant is a heavy water plant located in Thal-
Vaishet village in Raigad district of Maharashtra, India.
The site is also accessible from Gateway of India, Mumbai
by speedboats and catamaran services. Thal Power Plant
was commissioned in 1987 and is operated by the Heavy

PYQ Workbook 310


GENERAL GEOGRAPHY

Water Board (HWB), a public sector company under the


Department of Atomic Energy, Government of India. The
plant has a production capacity of 400 tonnes of heavy water
per year.

B. Manuguru - 1. Andhra Pradesh

Manuguru is a town in Bhadradri Kothagudem district of


Telangana, India. It is about 120 km from Khammam, the
district headquarters. Manuguru is home to a heavy water
plant, which is operated by the Heavy Water Board (HWB),
a public sector company under the Department of Atomic
Energy, Government of India. The plant has a production
capacity of 220 tonnes of heavy water per year

C. Kakrapara - 2. Gujarat

Kakrapar Atomic Power Station (KAPS) is a nuclear


power plant located in Kakrapar, Surat district, Gujarat,
India. It is operated by the Nuclear Power Corporation of
India Limited (NPCIL). It has four units, two of which are
pressurized heavy water reactors (PHWRs) with a capacity of
220 MW each, and two of which are indigenously developed
Important Tips
PHWRs with a capacity of 700 MW each. The first two units • The Caspian Sea is the largest lake in the world by
were commissioned in 1993 and 1995, respectively, and the surface area, covering an area of 371,000 square
kilometers.
third and fourth units were commissioned in 2023.
• The Caspian Sea is the deepest lake in the world, with
D. Kaiga - 5. Karnataka a maximum depth of 1,025 meters.
The Kaiga Generating Station (KGS) is a nuclear power • The Caspian Sea is home to over 1,000 species of plants
and animals, many of which are endemic to the sea.
plant located in Kaiga, Uttara Kannada district, Karnataka,
• The Caspian Sea is an important economic resource
India. It is operated by the Nuclear Power Corporation of
for the countries that border it. The sea is a source of
India Limited (NPCIL). KGS has six units, all of which oil, gas, and fish.
are pressurized heavy water reactors (PHWRs). The first
unit was commissioned in 2000. KGS has a total installed 40. Solution: (b)

capacity of 2.28 GW.


Exp) Option b is the correct answer.
39. Solution: (a)
The following countries have the Adriatic Sea as a boundary:
Exp) Option a is the correct answer.
• Albania
Armenia is a landlocked country in Western Asia. It does not
share a border with the Caspian Sea. It is bordered by Turkey • Bosnia and Herzegovina
to the west, Georgia to the north, Azerbaijan to the east, and
• Croatia
Iran to the south.

The other three countries listed in the question, Azerbaijan, The Adriatic Sea is a body of water that separates the Italian
Kazakhstan, and Turkmenistan, all share borders with the
Peninsula from the Balkan Peninsula. It is bordered by Italy
Caspian Sea. The Caspian Sea is the largest inland body
to the west, Slovenia, Croatia, Bosnia and Herzegovina,
of water in the world and is bordered by five countries:
Azerbaijan, Iran, Kazakhstan, Russia, and Turkmenistan. Montenegro, and Albania to the east.

311 PYQ Workbook


GENERAL GEOGRAPHY

42. Solution: (b)


Exp) Option b is the correct answer.
The correct matching of List-I (Oceanic Trench) with List-II
(Location) is as follows:
A. Aleutian - 2. North Pacific
B. Kermadec - 3. South Pacific
C. Sunda - 1. Indian Ocean
D. S. Sandwich - 4. South Atlantic Ocean

Important Tips
• The Aleutian Trench is an oceanic trench along a
convergent plate boundary which runs along the
southern coastline of Alaska and the Aleutian Islands
in the North Pacific Ocean. The trench extends for
Important Tips 3,400 Km from a triple junction in the west with the
• The Adriatic Sea is an arm of the Mediterranean Sea, Ulakhan Fault and the northern end of the Kuril–
located between the Italian Peninsula and the Balkan Kamchatka Trench, to a junction with the northern
Peninsula. end of the Queen Charlotte Fault system in the east.
• The Adriatic Sea is relatively shallow, with an average • The Kermadec Trench is a linear ocean trench in the
depth of about 250 meters. South Pacific Ocean. It stretches about 1,000 km from
the Louisville Seamount Chain in the north (26°S) to
• The Adriatic Sea is home to over 1,300 islands.
the Hikurangi Plateau in the south (37°S), north-east
• The largest island in the Adriatic Sea is Krk, which is of New Zealand’s North Island.
located in Croatia.
• The Sunda Trench (also known as Java Trench), is
• The Adriatic Sea is home to a number of historical and an oceanic trench located in the Indian Ocean near
cultural sites, including the ancient city of Dubrovnik Sumatra, formed where the Australian-Capricorn
in Croatia. plates subduct under a part of the Eurasian Plate. It is
3,200 km long with a maximum depth of 7,290 meters.
41. Solution: (d)
Its maximum depth is the deepest point in the Indian
Exp) Option d is the correct answer. Ocean.
(A) Seychelles is marked as 3 on the map. Seychelles is an • The South Sandwich Trench is a deep arcuate trench
archipelago of 115 islands in the Indian Ocean, off East in the South Atlantic Ocean lying 100 km to the east of
Africa. the South Sandwich Islands. It is the deepest trench of
(B) Chagos is marked as 1 on the map. Chagos is a group the Southern Atlantic Ocean, and the second deepest
of the Atlantic Ocean after the Puerto Rico Trench.
of seven atolls comprising more than 60 islands in the
Indian Ocean about 500 kilometers south of the Maldives 43. Solution: (d)
archipelago. Exp) Option d is the correct answer.
(C) Mauritius is marked as 4 on the map. Mauritius is an Statement 1 is correct: Ships can use tide tables to predict
island nation in the Indian Ocean, known for its beaches, the depth of the water in a particular location at a given time.
lagoons and reefs. This information can be used to plan a safe route for the ship.
(D) Socotra is marked as 2 on the map. Socotra is the Fishermen can use tide charts to predict when and where fish
largest of four islands in the Socotra archipelago, located are most likely to be found. For example, fish are often more
between the Guardafui Channel and the Arabian Sea. active during high tide because there is more food available.

PYQ Workbook 312


GENERAL GEOGRAPHY

Statement 2 is correct: High tides allow ships with deeper


drafts to enter or exit a harbor without running aground.
This is especially important for large vessels.
Statement 3 is correct: The ebb and flow of tides help in
preventing siltation in harbors by agitating the water and
preventing sediments from settling, keeping the channels
clear for ships.
Statement 4 is correct: Kandla and Diamond Harbour are
examples of ports that experience significant tidal variations,
making them tidal ports where the level of water rises and
falls with the tides, impacting shipping operations.

44. Solution: (b)


Exp) Option b is the correct answer.
Correct matching of the locations with ports are as follows:
A: Veraval Port
B: Karwar Port
C: Tuticorin Port Important Tips
D: Kakinada Port Pondicherry is situated on the eastern coastline, bordered
by the Bay of Bengal. It is enclosed by the Tamil Nadu
Important Tips
district of Arcot on its remaining three sides.
• The Port of Veraval lies on the southwestern coast of
Karaikal, located approximately 150 kilometers to the south
the Kathiawar Peninsula on the Arabian Sea in the
of Pondicherry along the eastern coast, is surrounded by
state of Gujarat in northwestern India. It is home to
the Bay of Bengal on the east and the Tamil Nadu district
one of India’s biggest fishing ports, accounting for 35
of Thanjavur on its other three sides.
per cent of the marine fish catch in the country. The
Port of Veraval also produces agricultural products Mahe town is positioned on the Malabar coast, nestled
and timber and is home to manufacturers of textile amidst the Western Ghats. It is located to the south of the
bobbins, matches, and bone fertilizers. Mahe river and is completely surrounded by the state of
Kerala.
• Karwar is located in Uttar Kannada district at the
Southern side of the Kali River. It is the only all-weather Yanam, a larger enclave compared to Mahe, is situated 200
natural port out of 10 minor ports of Karnataka. It kilometers away from Vishakapatnam in the East Godavari
is sandwiched between the Arabian sea on one side district of Andhra Pradesh.
and the Western Ghats on the other. It is known as
the Kashmir of Karnataka as it is blessed with a wide 46. Solution: (c)
variety of flora and fauna. Exp) Option c is the correct answer.
• V.O. Chidambaranar Port Authority is a port in
Lake Victoria is the largest lake in Africa and the second
Tuticorin, Tamil Nadu, and is one of the 13 major
ports in India. It is second largest port in Tamil Nadu largest freshwater lake in the world, after Lake Superior in
and third largest container terminal in India. V.O. North America. It is located on the border of Tanzania,
Chidambaranar Port is an artificial port.
Uganda, and Kenya.
• Kakinada Port, situated 170 km south of Visakhapatnam
Port, includes Kakinada Anchorage Port, Kakinada
Deep Water Port, Kakinada Fishing Harbour, and
a Ship-Breaking Unit. The deep-water port has a
12-meter channel, ensuring year-round accessibility.

45. Solution: (a)


Exp) Option a is the correct answer.
Correct matching of the different ports of Union Territory of
Pondicherry is as follows:
A: Mahe
B: Karaikal
C: Pondicherry
D: Yanam

313 PYQ Workbook


GENERAL GEOGRAPHY

Important Tips Important Tips


Lake Malawi is an African Great Lake and the The Younghusband Expedition was a British military
southernmost lake in the East African Rift system, located expedition to Tibet in 1903-1904. The expedition was
between Malawi, Tanzania and Mozambique. It is the led by Colonel Francis Younghusband, and its goal was to
ninth largest lake in the world by area and the third largest negotiate a trade agreement with the Tibetan government
and second deepest lake in Africa. and to secure British interests in Tibet. The British
Lake Chad is a freshwater lake located at the junction force reached Lhasa in August 1904, and Younghusband
of Chad, Niger, Nigeria, and Cameroon in central and negotiated a trade agreement with the Tibetan government.
western Africa. The agreement, known as the Lhasa Convention, gave the
Lake Zambezi is a man-made lake on the border between British the right to trade in Tibet and to have a permanent
Zambia and Zimbabwe. It was created by the construction presence in Lhasa.
of the Kariba Dam in 1959.
50. Solution: (b)
47. Solution: (b)
Exp) Option b is the correct answer.
Exp) Option b is the correct answer.
The Palk Bay lies between Gulf of Mannar and Bay of
The daily weather map showing isobars is an example of
an isopleth map. Isopleth maps use lines to connect places Bengal. The Palk Bay is a semi-enclosed shallow water body
with equal values of a particular variable, such as isobars between the southeast coast of India and Sri Lanka, with
indicating equal atmospheric pressure. Other examples of a water depth maximum of 13 m. The Palk Bay is located
isopleth maps include isotherms (equal temperature) and
between 8° 50 0 and 10° North latitudes and 78° 50 0 and 80°
isohyets (equal rainfall), where lines connect areas with the
same respective values. 30 0 East longitudes. The Palk Bay is bounded on the south by
Pamban Island (India), Adam’s (Rama’s) Bridge (a chain of
Important Tips
shoals), the Gulf of Mannar, and Mannar Island (Sri Lanka).
A choropleth map is a statistical thematic map utilizing
The Palk Bay has one big eastern opening into the Bay of
pseudo colors to represent aggregate geographic data
within spatial units, like population density or per-capita Bengal and two narrow openings into the Gulf of Mannar.
income. It employs color gradients to visually convey
variations and patterns in data across regions. 51. Solution: (a)
A Chorochromatic map, also referred to as an area-class Exp) Option a is the correct answer.
or mosaic map, displays categorical data in regions using
Here’s the matching of List-I with List-II:
different color symbols. These maps are effective for
representing discrete categories or nominal data across A. European Tran-Continental Railway - 5. Paris to Warsaw
geographic areas, providing visual insights into regional
B. Trans-Andean Railway - 4. Buenos Aires to Valparaiso
variations.
A choro schematic map employs abstract symbols and C. Trans-Siberian Railway - 2. Leningrad to Vladivostok
shapes to depict spatial data, focusing on thematic D. Orient Express - 1. Paris to Istanbul
information rather than geographic accuracy. It is
valuable for displaying locations of facilities, population Important Tips
density, and environmental features, utilizing symbols like Some other major Railway Routes of the World:
circles, dots, and triangles to convey different data points
Canadian Pacific Railway: It spans from St. John on the
effectively.
east coast to Vancouver on the west coast of Canada. It
48. Solution: (d) connects industrial regions of Quebec and Montreal,
softwood regions, and wheat-producing prairies.
Exp) Option d is the correct answer.
Canadian National Railway: Links Halifax in Nova Scotia
Indonesia is a country that consists of about 14,000
to Prince Rupert in British Columbia.
islands, some of which are among the largest in the world,
such as Sumatra, Java, Borneo, and New Guinea. Indonesia Trans-Australia Railway: Longest Australian route,
stretches from Aceh in the north-west to Torres Strait in connecting Sydney in the east to Perth in the west.
the east, covering a distance of about 5,000 miles. This is Trans Caucasian Railway: Runs from Batum in the former
almost as far as from London to Baghdad, which is about USSR to Farghana and Kusk.
5,200 miles. Northern Trans-continental Railway: Connects Seattle
49. Solution: (a) with New York.

Exp) Option a is the correct answer. Mid Trans-continental Railway: Links San Francisco
with New York.
The dotted line in the map shows the Durand line. The
Durand Line is the 2,640-kilometre (1,640 mi) international Southern Trans-continental Railway: Connects Los
border between Afghanistan and Pakistan. Angeles with New York and New Orleans.

PYQ Workbook 314


GENERAL GEOGRAPHY

52. Solution: (a)


• Tajikistan:
Exp) Option a is the correct answer.
• Capital: Dushanbe
Option a is correct: Antarctica is the highest continent on
• Currency: Somoni
Earth, with an average elevation of 2,500 meters (8,200 feet).
The highest point in Antarctica is Mount Vinson, which is • Tajikistan is a landlocked country with a rich culture
and history. It is home to the Pamir Mountains,
4,892 meters (16,050 feet) tall.
which are some of the highest mountains in the
Option b, c, and d are incorrect: The other continents world.
have much lower average elevations. North America has
• Turkmenistan:
an average elevation of 570 meters (1,870 feet), Asia has
an average elevation of 950 meters (3,117 feet), and South • Capital: Ashgabat
America has an average elevation of 660 meters (2,165 feet). • Currency: Manat

Important Tips • Turkmenistan is a desert country with a unique


culture. It is home to the Darvaza gas crater, which
Antarctica is a unique and fascinating continent. It is the is also known as the “Door to Hell.”
coldest, driest, and windiest continent on Earth. It is also
the only continent that is not inhabited by humans. • Uzbekistan:

The high elevation of Antarctica also contributes to • Capital: Tashkent


its extremely cold climate. The average temperature in • Currency: Som
Antarctica is -57 degrees Celsius (-71 degrees Fahrenheit). • Uzbekistan is a landlocked country with a rich
The coldest temperature ever recorded on Earth was -89.2 history and culture. It is home to the ancient cities
degrees Celsius (-128.6 degrees Fahrenheit) at Vostok of Samarkand and Bukhara, which were once
Station, Antarctica. important centers of trade and learning on the Silk
Road.
53. Solution: (d)
54. Solution: (b)
Exp) Option d is the correct answer.
Exp) Option b is the correct answer.
As can be seen from the map given below the countries
Statement 1 is correct: Railways require significant initial
marked from 1 to 5 are Kazakhstan, Turkmenia, Uzbekistan,
capital investment for infrastructure development, such as
Kirgizia and Tajikistan respectively.
laying tracks, building stations, and acquiring rolling stock.
This high capital cost limits the rapid expansion of the
railway network.
Statement 2 is correct: India has a vast and diverse
geographical landscape with dispersed population centers.
Road transport is more flexible and can reach remote areas
where rail connectivity may not be feasible.
Statement 3 is incorrect: Rail transport in India is
significantly more cost-effective than road transport. In
2021, the per-tonne-kilometer cost for rail was lower, at 1.6
Indian rupees per metric ton kilometer compared to road
transport’s 3.6 rupees per metric ton kilometer.
Statement 4 is correct: Railways have fixed routes and
schedules, which may not always align with the individual
Important Tips travel needs of the population. Road transport, on the other
Brief overview of each of the Central Asian countries: hand, offers greater flexibility and convenience for personal
and local travel.
• Kazakhstan:
• Capital: Nur-Sultan
• Currency: Tenge
• Kazakhstan is the largest landlocked country in
the world and is home to Baikonur Cosmodrome,
the world’s first and largest space launch facility.
• Kyrgyzstan:
• Capital: Bishkek
• Currency: Som
• Kyrgyzstan is a mountainous country with stunning
scenery and is home to Lake Issyk-Kul, the world’s
second largest alpine lake.

315 PYQ Workbook


GENERAL GEOGRAPHY

Per metric ton kilometer cost of different modes of The other three countries listed in the question, Surinam,
logistics in India in 2021(in Indian rupees) Uruguay, and Peru; all have coastlines.
55. Solution: (c) A landlocked country is a country that does not have a
Exp) Option c is the correct answer. coastline. Landlocked countries are often more economically
The correct answer is (1) Azores, (2) Cape Verde, (3) the disadvantaged than countries with coastlines, as they have to
Bahamas and (4) Falkland. These are four groups of islands rely on other countries for access to the sea.
located in different parts of the Atlantic Ocean. The Azores
are a Portuguese archipelago in the north Atlantic, near
Europe. Cape Verde is an African island nation in the central
Atlantic, off the coast of Senegal. The Bahamas is a Caribbean
country in the western Atlantic, near North America. The
Falkland Islands are a British overseas territory in the south
Atlantic, near South America

56. Solution: (a)


Exp) Option a is the correct answer.
The shortest air-route from Perth to London follows this
path:
Perth, Australia
Bombay (now known as Mumbai), India
Rome, Italy
London, United Kingdom

59. Solution: (b)


Exp) Option b is the correct answer.

7.2. Other UPSC CSE Previous Years’ Statement 1 is correct: Borneo Island is divided among
Questions three countries: Indonesia, Malaysia, and Brunei. Indonesia
occupies about 73% of the island, Malaysia occupies about
57. Solution: (d)
26%, and Brunei occupies about 1%.
Exp) Option d is the correct answer.
Statement 2 is incorrect: Borneo Island was not formed
The Gulf of Khambhat is the largest tidal energy-producing
area in India. It has a tidal range of over 10 meters, which due to volcano deposition, but rather due to a complex
is one of the highest in the world. The strong tidal currents geological history involving plate tectonics, continental
in the Gulf of Khambhat make it ideal for tidal power collisions, and subduction-accretion. Borneo Island has a
generation. The Government of India is planning to develop base of rocks that date back to the Paleozoic era, about 400
a number of tidal power projects in the Gulf of Khambhat. million years ago. The island has also experienced several
The total potential tidal energy capacity of the Gulf of mountain-building events and sea-level changes over time.
Khambhat is estimated to be around 7,000 MW.
60. Solution: (a)
58. Solution: (b)
Exp) Option a is the correct answer.
Exp) Option b is the correct answer
Bolivia is a landlocked country in South America. It is Syria is not located on the bank of the Black Sea. The
bordered by Brazil to the north and east, Paraguay and countries that do border the Black Sea are Turkey, Georgia,
Argentina to the south, and Chile and Peru to the west. Bulgaria, Romania, Ukraine, and Russia.

PYQ Workbook 316


GENERAL GEOGRAPHY

62. Solution: (b)


Exp) Option b is the correct answer.
Statement 1 is incorrect: Socotra Island is not an island of
Oman, but an island of the Republic of Yemen in the Indian
Ocean.
Statement 2 is correct: Socotra Island was designated as a
world natural heritage site by UNESCO in 2008. Socotra
Island is well known for its unique biodiversity and endemic
species, such as the dragon blood tree and the Socotra
starling.

63. Solution: (b)


Exp) Option b is the correct answer.
Sumatra, Java, Bali, Lombok is the sequence of the
Important Tips islands of Indonesia from west to east according to their
Black Sea: geographical location on the map. Sumatra is the westernmost
Located in southeastern Europe. island of Indonesia, followed by Java, which lies to the east of
Sumatra. Bali is an island to the east of Java, separated by the
Connected to the Mediterranean Sea through the
Bali Strait. Lombok is an island to the east of Bali, separated
Bosporus Strait, the Sea of Marmara, and the Dardanelles
Strait. by the Lombok Strait.

Covers an area of approximately 436,000 square kilometers 64. Solution: (c)


(168,000 square miles).
Exp) Option c is the correct answer.
Receives freshwater inflows from rivers like the Danube,
Madeira is not located in the Caribbean Sea, but in the
Dnieper, and Dniester, primarily in the northwestern
coastal waters. Atlantic Ocean, off the coast of Morocco and Portugal.
Grenada, Montserrat, and Anguilla are all islands in the
Deeper waters have lower oxygen levels, creating one of
the world’s largest anoxic basins. Caribbean Sea, belonging to the Lesser Antilles region

Contains several islands, including Snake Island Important Tips


(Ukraine), Giresun Island (Turkey), and St. Ivan Island Islands in the Caribbean Sea:
(Bulgaria).
• The Bahamas, a group of about 700 islands and cays in
Trick to remember border countries: the northern part of the Caribbean Sea. The Bahamas
Mnemonic: Tea & BURGeR are known for their coral reefs, white sand beaches,
Where the name is the order of appearance in the trick and turquoise waters.
are: • Barbados, an island in the eastern Caribbean Sea,
T- Turkey located about 100 miles (160 km) east of the Windward
Islands.
B-Bulgaria
• Cuba, the largest island in the Caribbean Sea, situated
U-Ukraine
at the entrance of the Gulf of Mexico.
R-Russia
• Hispaniola, the second largest island in the Caribbean
G-Georgia Sea, divided between two countries: Haiti and the
R-Romania Dominican Republic.
• Jamaica, an island in the central Caribbean Sea,
61. Solution: (c)
located south of Cuba and west of Hispaniola. Jamaica
Exp) Option c is the correct answer. is renowned for its reggae music, jerk cuisine, and
The Nobi and Kanto plains are both located in Japan, Rastafarian culture.
and they are the largest and second-largest plains in the • Puerto Rico, an island in the northeastern Caribbean
country respectively. The Nobi Plain is an alluvial plain Sea, east of Hispaniola and west of the Virgin Islands.
created by the Ibi, Kiso and Nagara rivers and has very fertile • The Virgin Islands, a group of about 90 small islands
soil. The Kanto Plain is a lowland facing the Pacific Ocean and cays in the eastern Caribbean Sea. The Virgin
and covers more than half of the Kanto region. Both plains Islands are divided into two political entities: the
are surrounded by mountain ranges and volcanoes and have British Virgin Islands (BVI) and the United States
Virgin Islands (USVI).
many rivers flowing through them.

317 PYQ Workbook


GENERAL GEOGRAPHY

65. Solution: (d) Acid, Phosphinates (hypophosphites) and phosphonates


Exp) Option d is the correct answer. (phosphites), and Scrap Aluminum to India.
The Darling Range is a mountain range that runs parallel
Option c is correct- The capital of Algeria is Algiers. Algeria
to the south-western coast of Australia. It is located in
is a country in North Africa, bordered by Morocco to the
the state of Western Australia, and it extends for over 320
kilometers. It lies in the east of Perth and runs from Moore northwest, Tunisia to the northeast, Libya to the east, Niger
River (North) to Bridgetown (South). The Darling Range and Mali to the south, and Mauritania to the southwest.
has some of the oldest rocks on earth- granite, gneiss, and
Algeria exports mainly Crude Petroleum, Petroleum Gas,
quartzite.
and Calcium Phosphates.

Option d is incorrect- The capital of Cape Verde is Praia.


Cape Verde is an island country in the Atlantic Ocean, off
the coast of West Africa.

Important Tips
Maghreb Region - Algeria, Libya, Mauritania, Morocco
and Tunisia are collectively called the Maghreb. The
Maghreb is a region in North Africa that is bordered by
the Mediterranean Sea to the north and the Sahara Desert
to the south. It also includes the Atlas Mountains, which
run along the coast of North Africa.
Important Tips
Sirocco: A hot, dry wind that blows from the Sahara
Some basic information related to the rocks found in Desert into the Maghreb region. It is especially common
Darling Range: in the summer months and can cause high temperatures
Darling Range includes expansive desert areas, such as the and dust storms.
Great Sandy, Gibson, and Great Victoria deserts.
Granite is a plutonic rock, meaning that it is formed from
molten magma that cooled and solidified slowly deep
beneath the Earth’s surface. Granite is a coarse-grained
rock that is composed primarily of quartz, feldspar, and
mica.
Gneiss is a foliated metamorphic rock, meaning that it has
a banded appearance caused by the alignment of mineral
grains. Gneiss is often formed from the metamorphism of
granite, shale, or sandstone.
Quartzite is a non-foliated metamorphic rock that is
composed primarily of quartz. Quartzite is formed from
the metamorphism of sandstone.

66. Solution: (d)


Exp) Option d is the correct answer.
Option a is correct- The capital city of Morocco is Rabat.
Morocco is a country in North Africa, bordered by Algeria to
the east, Mauritania to the south, and the Atlantic Ocean to 67. Solution: (d)
the west and north. Morocco’s main exports to India include
Calcium phosphates, phosphoric acid, and chemical Exp) Option d is the correct answer.
fertilizers. The correct order of lakes lying from North to South
Option b is correct- The capital of Tunisia is Tunis. Tunisia direction in the Suez Canal region is:
is a country in North Africa, bordered by Algeria to the
west, Libya to the southeast, and the Mediterranean Sea Lake Manzala - Lake Timsah - Great Bitter Lake - Little
to the north and east. Tunisia exports mainly Phosphoric Bitter Lake

PYQ Workbook 318


GENERAL GEOGRAPHY

Important Tips
The ISA is an intergovernmental organization of more
than 120 countries, most of which are located in the
sun-rich tropical and subtropical regions. The ISA aims
to promote the use of solar energy in these countries to
achieve sustainable development. The ISA was founded in
2015 by India and France.
The ISA’s mission is to promote the use of solar energy
to reduce dependence on fossil fuels and combat climate
change. The ISA aims to achieve this by:
Facilitating cooperation among member countries on
solar energy development and deployment
Providing technical assistance to member countries
Promoting investment in solar energy
Raising awareness about the benefits of solar energy
The ISA has a number of initiatives underway, including:
The Solar Technology Application Resource Centres
(STAR-Cs) program, which provides technical assistance
to member countries to develop and deploy solar energy
technologies
The Solar Training Network (SOLARNET) program,
Important Tips which provides training to professionals in the solar
energy sector
Lakes in the Suez Canal Region:
The Solar Rooftop Programme, which supports the
• Lake Manzala, also Manzaleh, is a brackish lake,
deployment of solar rooftop systems in member countries
sometimes called a lagoon, in northeastern Egypt on
the Nile Delta near Port Said and a few miles from the The ISA InnoSolar Challenge, which is a competition for
ancient ruins at Tanis. start-ups and SMEs developing innovative solar energy
• Lake Timsah, also known as Crocodile Lake; is a lake technologies
in Egypt on the Nile delta. It lies in a basin developed
69. Solution: (a)
along a fault extending from the Mediterranean Sea to
the Gulf of Suez through the Bitter Lakes region. Exp) Option a is the correct answer.

• The Great Bitter Lake is a large saltwater lake in The Telegraph Plateau is a part of the North Atlantic Ridge
Egypt that is part of the Suez Canal. Before the canal that is supposed to be relatively flat, and shallow compared
was built in 1869, the Great Bitter Lake was a dry salt to the rest of the ocean. The Mid-Atlantic Ridge in the North
valley or basin. Atlantic Ocean is a divergent or constructive plate boundary.
• The Bitter Lake, located in Egypt’s Suez Canal, is 70. Solution: (b)
a saltwater lake that separates the northern and
Exp) Option b is the correct answer
southern sections of the canal.
The correct match is
68. Solution: (c) (Grassland/Hill Station) (Location/State)
Exp) Option c is the correct answer A. Banni Grassland 3. Gujarat
The Secretariat of the International Solar Alliance (ISA) is B. Bugyal Grassland 1. Uttarakhand
located in Gurugram, India. It was inaugurated in January C. Khajjiar 4. Himachal Pradesh
D. Pahalgam 2. Jammu and Kashmir
2016 by the Prime Ministers of India and France. The
Banni Grassland: The Banni Grassland is a semi-arid
Secretariat is responsible for the day-to-day operations of the grassland located in the Kutch district of Gujarat. It is one
ISA. It provides technical and financial assistance to member of the largest grasslands in India and is home to a variety of
wildlife, including blackbuck, chinkara, and Indian gazelle.
countries, and it also promotes the exchange of knowledge
Bugyal Grassland: The Bugyal Grassland is a high-altitude
and best practices on solar energy. The Secretariat is headed grassland located in the Uttarakhand Himalayas. It is a
by a Director General, who is appointed by the ISA Assembly. popular destination for trekking and camping.

319 PYQ Workbook


GENERAL GEOGRAPHY

Khajjiar: Khajjiar is a hill station located in the Chamba


district of Himachal Pradesh. It is known for its scenic
beauty and its picturesque meadows.

Pahalgam: Pahalgam is a hill station located in the Anantnag


district of Jammu and Kashmir. It is known for its scenic
beauty and its opportunities for trekking and mountaineering.

71. Solution: (c)

Exp) Option c is the correct answer.

Coconino, Aquarius, Colorado, Columbia are the names


of plateaus in the USA. Colorado is the largest plateau in
the USA. Colorado has a diverse landscape of arid desert,
river canyons, and the snow-covered Rocky Mountains. It is
divided by the Colorado River and the Grand Canyon. This
plateau is an example of the intermontane plateau. 73. Solution: (c)
Exp) Option c is the correct answer.
Important Tips
Types of plateaus: An axial summit trough (AST) is a narrow trough or
• Dissected plateau: It forms as a result of upward volcanically modified graben that develops at the crest of
movement in the Earth’s crust. The uplift is caused by a midocean ridge and typically is the locus of volcanic and
the slow collision of tectonic plates. E.g., Colorado hydrothermal activity. The Red Sea is located along the Red
Plateau of USA.
Sea Rift, which is a divergent plate boundary. The Arabian
• Volcanic plateau: It is formed by numerous small
Plate is moving away from the African Plate, and the Red
volcanic eruptions that slowly build up over time,
forming a plateau from the resulting lava flows. E.g., Sea is forming in the space between the two plates.
Columbia Plateau of USA and Deccan Traps of India
• Intermontane plateau: A plateau which is enclosed
or surrounded by mountain ranges is known as an
intermontane plateau. E.g., Tibet Plateau, Mongolia
Plateau
• Continental plateau: The plateaus that rise abruptly
from either seas or lowlands are referred to as
continental plateaus. E.g., Potwar Plateau in Pakistan
and the Ranchi Plateau in India.

72. Solution: (a)

Exp) option a is the correct answer.

Israel has common borders with Lebanon, Syria, Jordan, and


Egypt.

The other options are incorrect:

Turkey does not share a border with Israel.

Cyprus is an island and does not share a border with any


country.

Iraq and Yemen do not share a border with Israel.

PYQ Workbook 320


GENERAL GEOGRAPHY

Important Tips 76. Solution: (a)

Red Sea: Exp) Option a is the correct answer


Red Sea lies between Asia and Africa and is a seawater The Central Research Institute for Dryland Agriculture
inlet of Indian Ocean. A total of 6 countries surround the
(CRIDA) is located in Hyderabad, Telangana. It is an
Red Sea. The 438,000 km sq. is the area of the Red Sea.
Indian research institute that conducts research on dryland
Trick to remember countries bordering the Red Sea is
agriculture. It was established in 1985 by the Indian Council
Y SEEDS
of Agricultural Research (ICAR).
Which stands for
Y: Yemen, Important Tips
S: Saudi Arabia, Important Institutes and their location:
E: Eritrea, Institute Location
E: Egypt, International Crop Research Patancheru, Hyderabad,
Institute for Semi-Arid Telangana.
D: Djibouti,
Tropics (ICRISAT)
S: Sudan.
Central Research Institute Hyderabad, Telangana.
74. Solution: (d) for Dryland Agriculture
(CRIDA)
Exp) Option d is the correct answer.
Central Institute of Arid Bikaner, Rajasthan.
The Sonoran Desert is the hottest in North America. It is
Horticulture (CIAH)
located at border of United States of America and Mexico.
National Institute of Jaipur, Rajasthan.
It is the only place in the world where Saguaro Cacti grows.
Agricultural Marketing
The Taklamakan Desert is a cold desert in Southwest (NIAM)
Xinjiang in Northwest China. This is one of the largest
Central Food Technological Mysore, Karnataka.
sandy deserts in the world. The area is characterized by a dry
Research Institute (CFTRI)
climate, low rainfall, and high evaporation.
National Research Centre for Jhansi, Uttar Pradesh.
The Karakum Desert is the hot desert in the great sandy Agroforestry
region in Central Asia. It covers over 70% of Turkmenistan’s
Central Institute of Bhopal, Madhya Pradesh.
land area.
Agricultural Engineering
The Gibson Desert is situated in Australia (not in Brazil). (CIAE)
It lies between the Great Sandy Desert and the Great Victoria National Dairy Research Karnal, Haryana.
Desert. Institute (NDRI)
75. Solution: (b) Indian Institute of Pulses Kanpur, Uttar Pradesh.
Research (IIPR)
Exp) Option b is the correct answer.
Indian Institute of Vegetable Varanasi, Uttar Pradesh.
Igarka is not located in China. Igarka is actually a Port
Research (IIVR)
town located in Turukhansky District of Krasnoyarsk
Krai, Russia. It is situated north of the Arctic Circle and Central Institute of Lucknow, Uttar Pradesh.
Subtropical Horticulture
historically served as a timber-exporting port along the
(CISH)
Yenisei River.
CSIR-Central Institute of Dhanbad, Jharkhand.
Important Tips Mining and Fuel Research
• The Port of Montevideo, in the northern part of the (CIMFR)
Old City of Montevideo, Uruguay, is one of the major Indian Grassland and Fodder Jhansi, Uttar Pradesh.
ports of South America and plays a very important role Research Institute (IGFRI)
in the economy of Uruguay. Notably the port includes
Dr. Yashwant Singh Parmar Solan, Himachal Pradesh.
a number of important facilities including one of the
University of Horticulture
country’s main tourism terminals and the La Teja
and Forestry
Refinery which processes the bulk of the country’s oil.
National Sugar Institute (NSI) Kanpur, Uttar Pradesh.
• Port of Tanjung Priok is the busiest and most advanced
Indonesian seaport, handling more than 50% of Mishra Dhatu Nigam Limited Hyderabad, Telangana.
Indonesia’s trans-shipment cargo traffic. The port (MIDHANI)
is located at Tanjung Priok, North Jakarta, which is Institute of Military Law Kamptee, Nagpur,
operated by Indonesian state-owned PT Pelindo. Maharashtra.

321 PYQ Workbook


GENERAL GEOGRAPHY

Institute of National Pune, Maharashtra. the Garibaldi Neve and the Chilcotin Icefield. The glaciers
Integration (INI) provide fresh water for the province and are also a popular
tourist destination.
Indian Institute of Sugarcane Lucknow, Uttar Pradesh.
Research (IISR)
National Atlas and Thematic Kolkata, West Bengal.
Mapping Organization
(NATMO)
Directorate of Plant Faridabad, Haryana.
Protection, Quarantine and
Storage (DPPQS)
National Centre of Organic Ghaziabad, Uttar Pradesh.
Farming (NCOF)
Central Institute of Indian Mysuru, Karnataka.
Languages (CIIL)
International Centre for Nairobi, Kenya. 79. Solution: (a)
Research on Agroforestry
Exp) Option a is the correct answer.
Indian Institute of Forest Bhopal, Madhya Pradesh.
The Dasht-e-Lut or Lut is a large salt desert located in the
Management
Iran. Dasht-e-Kevir is the largest desert in Iran.
Central Agroforestry Jhansi, Uttar Pradesh.
Research Institute 80. Solution: (b)
Tata Energy Research New Delhi. Exp) option b is the correct answer
Institute Canada has the longest coastal line in the world, with a
V.V. Giri National Labour Noida, Uttar Pradesh. total length of 243,047 kilometers. The other countries in the
Institute (VVGNLI) list have the following coastal line lengths:
National Research Centre on Domara, Ajmer, India: 7,517 kilometers
Seed Spices (NRCSS) Rajasthan.
Australia: 50,673 kilometers
77. Solution: (a) Brazil: 7,491 kilometers
Exp) Option a is the correct answer.
81. Solution: (b)
Africa is home to 54 countries, more than any other
Exp) Option b is the correct answer.
continent. Here is a list of the continents, in order of the
number of countries they contain: The correct match for each geographic feature with its
• Africa: 54 respective location is as follows:

• Europe: 50 A. Golden Triangle - 2. Opium producing area of Southeast


Asia
• Asia: 48
• South America: 12 B. Bermuda Triangle - 4. North Atlantic Ocean

• North America: 23 C. Rice Bowl of Far East - 1. Myanmar

• Oceania: 14 D. Red Basin - 3. China


• Antarctica: 0 Important Tips

78. Solution: (b) The Golden Triangle is located in the area where the
borders of Thailand, Myanmar and Laos meet at the
Exp) Option b is the correct answer.
confluence of the Ruak and Mekong Rivers. Along with
British Columbia, situated as Canada’s westernmost the Golden Crescent (Iran, Pakistan, and Afghanistan),
province, shares its border with the Pacific Ocean. It covers a it is regarded as one of the largest producers of opium in
substantial area of 944,735 square kilometers, ranking as the the world since the 1950s until it was overtaken by the
third-largest province in Canada. The landscape of British Golden Crescent in the early 21st century.
Columbia is characterized by prominent mountain ranges,
Myanmar is known as the Rice bowl of Far East. Thailand
including the well-known Canadian Rockies. However,
is known as the ‘Rice Bowl of South-East Asia’.
it is primarily defined by the dominating presence of the
Coast Mountains, Cassiar Mountains, and the Columbia The Sichuan Basin, sometimes called the Red Basin, is a
Mountains. Mountain Fairweather is the highest mountain lowland region in southwestern China. It is surrounded
in the Canadian Province of British Columbia. The Coast by mountains on all sides and is drained by the upper
Mountains are also home to a number of glaciers, including Yangtze River and its tributaries.

PYQ Workbook 322


GENERAL GEOGRAPHY

82. Solution: (b) Natural ports, also known as deep-water ports, are those
Exp) Option b is the correct answer. that are naturally formed and offer sufficient depth to
accommodate large vessels without extensive artificial
The name of the country that closely means ‘country of 40
dredging or construction. These ports typically have
tribes’ is Kyrgyzstan. The name “Kyrgyz” is derived from the naturally occurring deep channels, making them suitable for
Turkic word for “We are forty,” which is believed to refer to shipping.
the forty clans of Manas, a legendary hero who united forty
Artificial ports, on the other hand, are created or
regional clans. The “-stan” suffix in Persian means “place of,”
significantly modified through extensive engineering
so Kyrgyzstan means “the place of the forty tribes.”
and dredging to enable ships to access them. These ports
83. Solution: (a) often require ongoing maintenance to keep their channels
navigable.
Exp) Option a is the correct answer.
The queen of the Adriatic Sea is not Rome but Venice. 87. Solution: (b)
Venice, Italy, is often referred to as the “Queen of the Exp) Option b is the correct answer.
Adriatic” because of its location on the Adriatic coast Haldia is a specific example of an “out port.” An outport
and its historical significance as a major maritime and is typically a secondary or subsidiary port, often smaller in
trading city. Because of its location, it dominated the scale, built to support the commercial activities of a larger,
Eastern Mediterranean beginning in the twelfth century. main port. Haldia Port, officially known as Haldia Dock
Her extensive trade network linked Europe to Byzantium, Complex, is located at the confluence of the Haldi River
the Moslem world, and even the distant Asian civilizations and the Hooghly River and serves as a secondary port to
explored by Marco Polo. It is also known as “City of Water,” support the trade activities of Kolkata Port Trust.
“City of Masks,” “City of Bridges,” “The Floating City,” and
Important Tips
“City of Canals.”
Different types of ports in shipping include:
The Lorraine Coal Fields are a region in northeastern France
that is known for its coal reserves. Birmingham is a city in • Inland Port:
the West Midlands region of England. Tula is a city in central • Located in navigable lakes, rivers, or canals with
Russia that is known for its metalworking industry. access to the sea.
• Used for cargo stocking and dispatching.
84. Solution: (a)
• Occasionally open to passengers.
Exp) Option a is the correct answer
• Fishing Port:
Aleutian Islands - USA: The Aleutian Islands are a chain of
islands in the North Pacific Ocean that belong to the United • Primarily used for recreational or aesthetic
States. purposes.
• Can be either inland or seaports.
Bear Islands - Norway: The Bear Islands are a Norwegian
archipelago in the Arctic Ocean. • Associated with the commercial fishing sector.
Greenland - Denmark: Greenland is an autonomous • Dry Port:
constituent country within the Kingdom of Denmark. • Inland terminals connected to seaports via road or
Franz Josef Island - Russia: Franz Josef Land is a Russian rail.
archipelago in the Arctic Ocean. • Used for transshipment to inland destinations.
• Equipped for efficient cargo clearance.
85. Solution: (b)
• Sea Port:
Exp) Option b is the correct answer
• Common for commercial shipping worldwide.
The ‘National Museum of Natural History’ did not have a
branch in Hyderabad. Therefore, the correct answer is (b) • Accommodates small to large vessels.
Hyderabad. • Divided into:
The National Museum of Natural History is a premier • Cruise Ports: Specialize in cruise ship activities.
museum in India that exhibits a wide variety of natural history • Port of Call: Brief stops for maintenance or
specimens, including plants, animals, fossils, and minerals. passenger embarkation.
The museum was established in 1973 and is a constituent • Cargo Ports: Handle specific types of cargo with
organization of the National Council of Science Museums varying facilities.
(NCSM). It is located in Mysuru, Bhopal, Bhuvaneshwar,
Sawai Madhopur and Gangtok. 88. Solution: (c)
Exp) Option c is the correct answer.
86. Solution: (d)
Despite its extreme climate, the Thar Desert has a total
Exp) Option d is the correct answer. population of about 16.6 million people. It is also the
Cochin and Visakhapatnam are natural ports whereas most densely populated hot desert in the world, with a
Chennai and Tuticorin are artificial port. population density of 83 persons per square km.

323 PYQ Workbook


GENERAL GEOGRAPHY

89. Solution: (a) three types of thermal power plants, each with its installed
Exp) Option a is the correct answer. capacity as of January 2021:
The Aral Sea, straddling the boundaries of Kazakhstan Coal-based power plants: These have a total capacity of
to the north and Uzbekistan to the south, was once a 206,404.50 MW.
substantial saltwater lake in Central Asia. Historically, it Gas-based power plants: Their combined capacity stands at
ranked as the world’s fourth-largest inland water body. 24,956.51 MW.
However, over the latter half of the 20th century and into the
Liquid Fuel (Diesel) power plants: These contribute a
21st, it experienced a dramatic reduction in its size and
capacity of 509.71 MW.
volume. This decline was primarily due to the diversion
of river waters from the Syr Darya and Amu Darya, Important Tips
which previously flowed into the Aral Sea, for irrigation Hydropower and Nuclear Power:
purposes.
Hydroelectricity is a way of making electricity using the
power of falling water, like a waterfall. It doesn’t use up
the water itself. It’s a very common kind of clean energy,
making up about 3% of all the energy used worldwide. In
India, the Koyna Hydroelectric Project is the biggest of
its kind.
Nuclear power, on the other hand, is the fourth-largest
source of electricity in India. The country has 21 nuclear
reactors in 7 power plants, according to reports from
2013. India is also exploring using thorium and uranium
with lower levels of radiation to make atomic reactors,
which is a cool development in energy technology.

91. Solution: (d)


Exp) Option d is the correct answer.
Patagonian Desert is a temperate desert. The Patagonian
Desert is the largest desert in Argentina and is the 8th
largest desert in the world by area.

90. Solution: (c)


Exp) Option c is the correct answer.
Thermal Power Plants account for more than 65% of the total
electricity generation in India. About 85% of thermal power 92. Solution: (a)
generation is through coal. In India, there are primarily Exp) Option a is the correct answer.

PYQ Workbook 324


GENERAL GEOGRAPHY

A. Allegheny - 4. USA Important Tips


The Allegheny Mountains are a mountain range in the eastern The Amur River is the easternmost of the three great
United States, extending from Pennsylvania to northern Siberian rivers that flow into the Arctic Ocean. It forms at
Alabama. They are part of the Appalachian Mountains and the confluence of the Shilka and Argun rivers, which have
are the highest mountains east of the Mississippi River. The their origins in the Altai Mountains.
highest peak in the Allegheny Mountains is Mount Mitchell
The Lena River is the eleventh-longest river in the world,
in North Carolina, which has an elevation of 6,684 feet.
and the longest river entirely within Russia. It flows
B. Cantabrian - 3. Spain northward from its source in the Baikal Mountains, near
The Cantabrian Mountains are a mountain range in Lake Baikal, to its delta in the Laptev Sea, an arm of the
northern Spain, stretching along the Cantabrian Sea coast. Arctic Ocean.
They are a relatively low mountain range, with the highest The Ob River is the westernmost of the three great
peak, Torre de Cerredo, having an elevation of 8,730 feet. Siberian rivers that flow into the Arctic Ocean. It forms at
C. Elburz - 2. Iran the junction of the Biya and Katun rivers, which have their
The Elburz Mountains are a mountain range in northern origins in the Altai Mountains.
Iran, stretching along the southern coast of the Caspian Sea.
95. Solution: (a)
They are the highest mountains in Iran, with the highest
peak, Mount Damavand, having an elevation of 18,406 feet. Exp) Option a is the correct answer.
D. Mackenzie - 1. Canada The Prime Meridian passes through Algeria and France.
The Mackenzie Mountains are a mountain range in These two countries are among the eight that the Greenwich
northwestern Canada, stretching along the border between Meridian crosses from north to south. The others include
the Yukon Territory and the Northwest Territories. They are Spain, Mali, Burkina Faso, Togo, Ghana, and it also touches
the northernmost mountain range in the Rocky Mountains Antarctica in Queen Maud Land.
and are home to some of the highest peaks in Canada,
including Mount Logan, the highest peak in Canada with an
elevation of 19,555 feet.

93. Solution: (b)


Exp) Option b is the correct answer.
Algeria, Morocco, and Tunisia are the North African
countries that have coastlines along the Mediterranean Sea.
Chad, on the other hand, is a landlocked country located
in north-central Africa and does not have access to any
major body of water, including the Mediterranean Sea.
Important Tips 96. Solution: (c)
The trick to remembering countries surrounding the Exp) Option c is the correct answer.
Mediterranean Sea is:
Kalahari Desert is an interior desert of Southern Africa.
Sci-Fi GM4 CASTLE BLAST
It occupies the area of Botswana, Namibia, and the
• SCI-FI: Spain, Cyprus, Israel, France, Italy
northernmost part of South Africa.
• GM4: Greece, Morocco, and 4 Countries of Middle
east Israel, Lebanon, Syria, Turkey
• CASTLE BLAST: Croatia, Algeria, Slovenia, Tunisia,
Lebanon, Egypt, Bosnia and Herzegovina, Libya,
Albania, Syria, Tunisia

94. Solution: (d)


Exp) Option d is the correct answer.
The Salween River is one of the longest rivers in Asia,
flowing for about 1,749 miles (2,815 km) from the Tibetan
Plateau to the Andaman Sea. It flows southward through
China, Myanmar, and Thailand, forming part of the border
between Myanmar and Thailand. It is one of the few major
rivers in Southeast Asia that remains largely undammed and
free-flowing. It supports a rich biodiversity and provides
livelihoods for millions of people along its course.

325 PYQ Workbook


GENERAL GEOGRAPHY

97. Solution: (b) Important Tips


Exp) Option b is the correct answer. The Dnieper is one of the major transboundary rivers of
The Rio Grande is the fifth longest river in North America, Europe, rising in the Valdai Hills near Smolensk, Russia,
and it forms the border between Mexico and the United before flowing through Belarus and Ukraine to the Black
States for over 1,200 miles. The river is also known as the Sea. It is the longest river of Ukraine and Belarus and the
Río Bravo del Norte in Mexico. fourth-longest river in Europe, after the Volga, Danube,
and Ural rivers
The Po is the longest river in Italy. It flows eastward across
northern Italy starting from the Cottian Alps. It forms a
fertile plain called the Po Valley, which is one of the most
densely populated and industrialized areas in Europe
The Rhine is one of the major European rivers. It begins
in the Swiss canton of Graubünden in the southeastern
Swiss Alps. It forms part of the Swiss-Liechtenstein, Swiss-
Austrian, and Swiss-German borders.

100. Solution: (b)

Exp) Option b is the correct answer.

Oman is not a coastline country of the Persian Gulf, but


it is a coastline country of the Gulf of Oman, which is
connected to the Persian Gulf by the Strait of Hormuz. The
countries that border the Persian Gulf are Bahrain, Iran, Iraq,
98. Solution: (c)
Kuwait, Qatar, Saudi Arabia, and the United Arab Emirates.
Exp) Option c is the correct answer.
Sri Lanka is an island country in the Indian Ocean that has 101. Solution: (c)
a radial drainage pattern at the macro-level. The central
Exp) Option c is the correct answer.
highlands of Sri Lanka, which include the highest peak of
Pidurutalagala (2,524 metres), are the source of many rivers Option a is correct: The Atacama Desert is a desert plateau
that radiate in different directions and drain into the ocean. located on the Pacific coast of South America. It is stretched
Some of the major rivers of Sri Lanka are Mahaweli, Kelani,
along a long strip of land west of the Andes Mountains.
Kalu, Walawe, and Aruvi.
A radial drainage pattern is formed when streams flow Option b is correct: The Atacama Desert is one of the driest
outward from a central high point, such as a volcano, a places on Earth. The desert is located in the rain shadow
dome, or a mountain. The streams resemble the spokes of a
region of the Andes Mountains. The cold water and the dry
wheel and may flow in different directions depending on the
slope of the land. air that comes with the Humboldt Current are also partially
responsible for the formation of the Atacama Desert.
Important Tips
Some other examples of radial drainage patterns are: Option c is incorrect: The Atacama Desert lies in Northern
The Amarkantak Mountain in India, which is the source of Chile (not in Southern Chile) in South America. The Driest
the Narmada, Son, and Mahanadi rivers.
place on earth is Atacama Desert in northern Chile. It
The Mount Cameroon in Cameroon, which is an active
receives less than 1 mm of precipitation each year, and some
volcano with many streams flowing from its summit.
The Mount Kilimanjaro in Tanzania, which is the highest areas haven›t seen a drop of rain in more than 500 years.
mountain in Africa and has several rivers originating from It is situated between two mountain chains (the Andes and
its glaciers. the Chilean Coast Range) of sufficient height to prevent
99. Solution: (d) moisture advection from either the Pacific or the Atlantic
Exp) Option d is the correct answer. Oceans, a two-sided rain shadow region.
The Rhone is a major river in France and Switzerland, not
Option d is correct: Atacama Desert region is the largest
Spain. It rises in the Alps and flows west and south through
Lake Geneva and southeastern France before discharging natural supply of Sodium Nitrate, and it is known as the
into the Mediterranean Sea. name of ‘Chile saltpeter’.

PYQ Workbook 326


GENERAL GEOGRAPHY

102. Solution: (c)


Exp) Option c is the correct answer. 103. Solution: (b)
The Tropic of Cancer is a prominent line of latitude situated Exp) Option b is the correct answer.
about 23.5 degrees north of the Equator. It serves as the
Option a is incorrect: Afghanistan is a landlocked country. It
boundary for the tropical region in the Northern Hemisphere.
is located in Southern Asia, bordered by Pakistan to the east
Tropic of cancer passes through sixteen countries, three and south, Iran to the west, Turkmenistan to the northwest,
continents and six water bodies. Uzbekistan to the north, Tajikistan to the northeast, and
North America - Bahamas (Archipelago), Mexico India & China to the northeast and east.
Africa - Niger, Algeria, Egypt, Libya, Mali, Western Sahara, Option b is correct: Liberia is not a landlocked country. It is
Mauritania located on the West African coast and has a coastline of 563
Asia - Oman, United Arab Emirates, Saudi Arabia, India, kilometers (350 miles). It is bordered by Guinea to the north,
Bangladesh, Myanmar, Taiwan, China. Sierra Leone to the northeast, Ivory Coast to the east, and
the Atlantic Ocean to the south and west.
Water Bodies - Indian Ocean, Atlantic Ocean, Pacific
Option c is incorrect: Laos is a landlocked country. It is
Ocean, Taiwan Strait, Red Sea, Gulf of Mexico.
located in Southeast Asia, bordered by Myanmar and China
Therefore, Tropic of Cancer does not pass through Iran. to the northwest, Vietnam to the east, Cambodia to the
southeast, and Thailand to the west and southwest. Laos is
the only landlocked country in Southeast Asia.
Option d is incorrect: Luxembourg is a landlocked country.
It is located in Western Europe, bordered by Belgium, France,
and Germany.

104. Solution: (d)


Exp) Option d is the correct answer.
North America is the only continent that has no landlocked
countries. All of the other continents have at least one
landlocked country. North America is a relatively small
continent, and it is surrounded by water on all sides.
Option a, b, and c is incorrect: The other three options
Important Tips
have landlocked countries in that continent.
The Tropic of Cancer passes through eight states in
Africa has 16 landlocked countries,
India: Gujarat (Jasdan), Rajasthan (Kalinjarh), Madhya
Pradesh (Shajapur), Chhattisgarh (Sonhat), Jharkhand Asia has 12 landlocked countries,
(Lohardaga), West-Bengal (Krishnanagar), Tripura Europe has 14 landlocked countries,
(Udaipur) and Mizoram (Champhai). South America has 2 landlocked countries.

327 PYQ Workbook


GENERAL GEOGRAPHY

105. Solution: (a) 108. Solution: (d)


Exp) Option a is the correct answer. Exp) Option d is the correct answer.
Assertion (A) is correct: Egypt is often called the Gift of The Tropic of Capricorn, the southernmost point on Earth
the Nile because the Nile River has been the source of life where the sun can be exactly overhead at noon, is located
at 23.5 degrees south and is the southern boundary of the
and civilization for the ancient and modern Egyptians. The
tropics.
Nile provides water for irrigation, transportation, fishing,
There are 10 countries, 3 continents and 3 water bodies
and hydroelectric power. The annual flooding of the Nile
that pass through the Tropic of Capricorn.
also deposits fertile silt on the river banks, making the land
Continents: South America, Africa, Australia
suitable for agriculture.
Countries: Argentina, Brazil, Chile, Paraguay, Namibia,
Reason (R) is correct: Egypt is an oasis within the vast Botswana, South Africa, Mozambique, Madagascar and
Sahara Desert. The presence of the Nile River and its annual Australia
floods created a narrow, fertile strip of land on both sides Water Bodies: Indian Ocean, Atlantic Ocean and Pacific
of the river. Beyond this strip lies the arid Sahara Desert, Ocean.
which covers much of North Africa.

106. Solution: (d)


Exp) Option d is the correct answer.
The Gulf of Tonkin is a gulf at the northwestern portion
of the South China Sea, located off the coasts of Tonkin
(northern Vietnam) and South China.
The Gulf of Bothnia is a sea off the northern coast of Europe,
between Finland and Sweden.
The Gulf of Carpentaria is a sea off the northern coast of
Australia, enclosed on three sides by northern Australia
and bounded on the north by the eastern Arafura Sea. 109. Solution: (c)
Baffin Bay is an arm of the Atlantic Ocean bounded by Baffin Exp) Option c is the correct answer.
Island in the west, Greenland in the east, and Ellesmere The Atlas Mountains are a prominent mountain range
Island in the north. in North Africa, stretching around 2,500 km through
Morocco, Algeria, and Tunisia. They serve as a natural
107. Solution: (a) barrier between the Sahara Desert and the coastal regions.
Exp) Option a is the correct answer. The highest peak, Toubkal, is located in central Morocco,
reaching an elevation of 4,167 meters (13,671 feet).
(A) and (R) both are true, and (R) is correct explanation These mountains are predominantly inhabited by Berber
of (A): The North Atlantic Sea route connects North- populations.
eastern U.S.A. and North-western Europe, which are two
Important Tips
of the most industrially developed regions in the world. This
connection facilitates extensive trade and commerce between Manitoba is a Canadian province bordered by Ontario
to the east and Saskatchewan to the west. Its landscape of
these regions. As a result, a significant portion of the world’s
lakes and rivers, mountains, forests and prairies stretches
foreign trade flows through this route, accounting for about
from northern Arctic tundra to Hudson Bay in the east
one-fourth of the world’s foreign trade. It is often referred
and southern farmland.
to as the “Big Trunk Route” due to its immense importance
The Pampas, originating from the Quechua word
in global trade. The connection between major industrial
“pampa” for “plain,” encompass over 1,200,000 square
zones of the world leads to a high volume of trade and
kilometers in South America, including Argentina,
shipping activity along this route, making it the busiest.
Uruguay, and parts of Brazil. With a temperate climate
and evenly distributed rainfall of 600 to 1,200 mm annually,
the region boasts fertile soils ideal for agriculture.
Kimberley is the capital city of South Africa’s Northern
Cape Province. It’s known for its 19th-century diamond
mines, like the deep, hand-dug Big Hole.

110. Solution: (b)


Exp) Option b is the correct answer
Australia - Christmas Island: Christmas Island is an
external territory of Australia located in the Indian Ocean.

PYQ Workbook 328


GENERAL GEOGRAPHY

Denmark - Greenland: Greenland is an autonomous and is also known as the Upper Rhine Plain or the Upper
territory of Denmark located in the North Atlantic Ocean. Rhine Graben. The other rivers, Amazon, Indus, and Volga
France - Martinique: Martinique is a French overseas do not flow through rift valleys, but through different types
department located in the Caribbean Sea. of landforms.
Spain- Santa Cruz: Santa Cruz is a Danish overseas territory 113. Solution: (a)
located in the Caribbean Sea.
Exp) Option a is the correct answer.
111. Solution: (c) A datum line is a horizontal reference line used to measure
Exp) Option c is the correct answer. heights and depths from sea level. It serves as a starting
The Horn of Africa consists of the following countries: point for position measurements, especially in surveying and
• Djibouti geodesy. This reference line helps establish consistent and
standardized measurements for various locations on Earth’s
• Eritrea
surface, facilitating accurate mapping and positioning.
• Ethiopia
• Somalia 114. Solution: (d)
The other options are incorrect: Exp) Option d is the correct answer.
Algeria, Morocco, and West Sahara are located in North The correct match for the given railway production units is
Africa. as follows:
Libya, Sudan, and Egypt are located in North Africa and A. Rail Coach Factory - 3. Kapurthala
Northeast Africa. B. Wheel and Axle Plant - 1. Bangalore
Zimbabwe, Botswana, and Angola are located in Southern C. Diesel Locomotive Works - 4. Varanasi
Africa.
D. Integral Coach Factory - 2. Perambur

Important Tips
List of Production Units on Indian Railways:
S.No. Name of PSU Place where situated
1. Chittaranjan Chittaranjan
Locomotive Works
2. Diesel Locomotive Varanasi
Works
3. Integral Coach Factory Chennai
4. Rail Coach Factory Kapurthala
5. Rail Wheel Factory Bangalore
Important Tips
6. Diesel Loco Patiala
The Horn of Africa is a peninsula in Northeast Africa
Modernisation Works
that juts into the Gulf of Aden and the Arabian Sea. It is
bordered by the Red Sea to the north and the Gulf of Aden
115. Solution: (b)
to the east.
Exp) Option b is the correct answer.
The Horn of Africa is a region of great diversity, with a
variety of ethnic groups and languages. It is also a region Bermuda is located in the North Atlantic Ocean,
of great poverty and conflict. approximately 600 milesa (970 km) east of Cape Hatteras,
The Horn of Africa is home to some of the oldest North Carolina, United States. Bermuda is a British
civilizations in the world, including the Aksumite Empire overseas territory consisting of 7 main islands and about 170
and the Kingdom of Kush. islets and rocks.
The Horn of Africa is also a strategic region, due to its 116. Solution: (b)
location on the Red Sea and the Gulf of Aden. It is a major
Exp) Option b is the correct answer.
shipping route and is also a source of oil and gas.
An avalanche is a type of terrestrial disaster. It refers to
112. Solution: (d) the rapid flow or slide of snow, ice, and debris down a
Exp) Option d is the correct answer. mountainside or slope. Avalanches are triggered by various
The Rhine River flows through a rift valley that was formed factors such as snowfall, snowpack instability, slope angle,
by the extension and thinning of the Earth’s crust between and human activities. They can pose significant dangers
the Alps and the Central European Uplands. The rift valley to human settlements, infrastructure, and recreational
stretches from Basel in Switzerland to Frankfurt in Germany activities in mountainous regions.

329 PYQ Workbook


GENERAL GEOGRAPHY

117. Solution: (a) 121. Solution: (b)


Exp) Option a is the correct answer. Exp) Option b is the correct answer.
The Volga River is the longest river in Europe and it The Caucasus Mountains separate the Black Sea and the
primarily flows through Russia, but it does not drain into Caspian Sea. The Caucasus is a mountain range located in
the Black Sea. Instead, the Volga River discharges its waters the border region between Eastern Europe and Western Asia.
into the Caspian Sea. It runs approximately east-west between the Black Sea and
the Caspian Sea, forming a natural boundary between the
118. Solution: (b) two bodies of water.
Exp) Option b is the correct answer. Important Tips
Singapore is a city-state located at the southern tip of the • Urals: The Ural Mountains are located in Russia and
Malay Peninsula in Southeast Asia. It is situated very close separate the European and Asian parts of the country.
to the Equator, making it the city among the given options
• Carpathians: The Carpathian Mountains are a
that is closest to the Equator. Coordinates of cities:
mountain range located in Central and Eastern Europe.
• Mogadishu - 2.0371° N, 45.3438° E),
• Balkan Mountains: The Balkan Mountains, also known
• Singapore - 1.3521° N, 103.8198° E), as the Balkans or the Balkan Peninsula, are located in
• Colombo - 6.9271° N, 79.8612° E), and Southeastern Europe.
• Manila - 14.5995° N, 120.9842° E) 122. Solution: (a)
Important Tips Exp) Option a is the correct answer.
Some of the Countries lying close to the equator includes Coral reefs are typically found in marine environments and
Ecuador, Brazil, Colombia, Indonesia, Kenya, Democratic require saltwater to survive. They cannot grow abundantly
Republic of Congo, Somalia, Malaysia, Uganda, Gabon, in fresh water because they have specific adaptations to live
Republic of Congo, Sao Tome and Principe, Maldives, in the unique conditions of the marine ecosystem.
Kiribati, Papua New Guinea. Option b is correct: Coral reefs generally require warm
water temperatures ranging between 23°C-25°C for optimal
119. Solution: (a) growth. They are mostly found in tropical and subtropical
Exp) Option a is the correct answer. regions where the water temperatures fall within this range.
The Great Barrier Reef, located off the northeastern coast Option c is correct: Coral reefs prefer shallow waters where
of Australia, is the largest coral reef system in the world. It sunlight can penetrate and support photosynthesis by their
stretches over 2,300 kilometers (1,430 miles) and covers an symbiotic zooxanthellae algae. While some species of coral
area of approximately 344,400 square kilometers (133,000 can be found at depths greater than 50 meters, most thrive in
square miles). It is renowned for its exceptional beauty, rich shallower areas closer to the water’s surface.
biodiversity, and ecological importance Option d is correct: Coral reefs rely on photosynthesis
carried out by zooxanthellae algae living within their tissues.
120. Solution: (d) These algae require sunlight to produce energy through
Exp) Option d is the correct answer. photosynthesis, which is essential for the growth and survival
The Tropic of Capricorn is an imaginary line of latitude that of the coral.
runs approximately 23.5 degrees south of the Equator. It 123. Solution: (d)
passes through several countries, primarily in the Southern
Exp) Option d is the correct answer.
Hemisphere. Uruguay is a country in South America, but it
is entirely located to the north of the Tropic of Capricorn. • Cheruthoni Dam - This dam is located in the state of
Coordinates of countries: Kerala. It is a part of the Idukki Hydroelectric Project,
which is one of the largest hydroelectric projects in India.
Chile- 35.6751° S, 71.5430° W
• Indira Sagar Dam - This dam is located in the state of
Brazil- 14.2350° S, 51.9253° W Madhya Pradesh. It is built on the Narmada River and
Paraguay- 23.4425° S, 58.4438° W is one of the key multipurpose projects in the Narmada
Uruguay- 32.5228° S, 55.7658° W Valley.
• Krishnaraja Sagar - This dam is located in the state of
Important Tips Karnataka. It is built on the Cauvery River near Mysore
The Tropic of Capricorn, an imaginary line on the Earth’s and serves as a major water source for irrigation in the
surface, passes through a total of 10 countries, spanning region.
across 3 continents and intersecting 3 water bodies. • Mettur Dam - This dam is located in the state of Tamil
These countries include Argentina, Chile, Namibia, South Nadu. It is built on the Cauvery River near Salem and
Africa, Madagascar, Australia, Brazil, Paraguay, Botswana, plays a crucial role in supplying water for irrigation and
and Mozambique. power generation.

PYQ Workbook 330


GENERAL GEOGRAPHY

124. Solution: (a) 127. Solution: (c)


Exp) Option a is the correct answer. Exp) Option c is the correct answer.
A. Bass Strait: Bass Strait is a strait located between
The forest canopy, comprising trees’ branches, leaves, and
Australia and Tasmania.
stems, acts as a barrier between the lower atmosphere and
B. Davis Strait: Davis Strait is a strait located between
Canada and Greenland. the forest ecosystem. The canopy plays a crucial role in
C. Dover Strait: Dover Strait is a strait located between the regulating energy exchange between the atmosphere
United Kingdom and France. and the forest by intercepting solar radiation, absorbing
D. Florida Strait: Florida Strait is a strait located between a portion of it for photosynthesis, and reflecting or
the United States and Cuba. transmitting the rest. This direct reflection of insolation
Important Tips from the forest canopy contributes to the cooling of the
• Straits are narrow waterways connecting larger Earth’s surface.
bodies of water, serving as vital maritime routes for
trade and transportation. Important Tips
• Some of the most famous straits in the world: • Forest canopy intercepts solar radiation, provides
habitat for plants and animals, and regulates the flow
• Strait of Hormuz: Located between Oman and Iran,
of air and water.
it connects the Persian Gulf to the Arabian Sea.
• Forest floor accumulates organic matter, provides
• Strait of Gibraltar: Located between Spain and
habitat for decomposers, and supports the growth of
Morocco, it connects the Mediterranean Sea to the
understory plants.
Atlantic Ocean.
• Forest soil stores water and nutrients, provides
• Strait of Malacca: Located between the Malay
anchorage for trees, and supports the growth of roots.
Peninsula and the Indonesian island of Sumatra, it
connects the Indian Ocean to the South China Sea. • Forest litter accumulates dead leaves, twigs, and other
organic matter.
• Bering Strait: Located between Russia and Alaska
(United States), it connects the Arctic Ocean to the
128. Solution: (b)
Bering Sea and separates Asia from North America.
Exp) Option b is the correct answer.
125. Solution: (c)
Factor 1 is incorrect: Corals do not require freshwater for
Exp) Option c is the correct answer.
their growth. In fact, they are adapted to thrive in saltwater
New Guinea’ is the largest island from the given options
but Greenland is the world’s largest island. environments.
It is located in the southwestern Pacific Ocean and is Factor 2 is correct: Corals are found in clear, shallow
divided between two countries: Papua New Guinea to the saltwater environments, typically in tropical regions.
east and Indonesia to the west. New Guinea is known for its
vast tropical rainforests, diverse wildlife, and rich cultural Factor 3 is correct: Sunlight is essential for the growth of
heritage. corals as it provides energy for photosynthesis. Corals have

126. Solution: (a) a mutualistic relationship with photosynthetic algae called

Exp) Option a is the correct answer. zooxanthellae, which reside within their tissues and provide

Both the statements are individually true and statement II them with nutrients.
is the correct explanation of statement I. Factor 4 is correct: Corals require warm water within
Statement I is correct: Global warming refers to the long- a specific temperature range for optimal growth. The
term increase in Earth’s average surface temperature,
temperature range of 23°C to 25°C is favorable for the growth
primarily due to the buildup of greenhouse gases in the
atmosphere. of corals.

Statement II is correct: Greenhouse gases, such as carbon 129. Solution: (d)


dioxide (CO2), methane (CH4), and water vapor, trap heat
radiated from the Earth’s surface and prevent it from Exp) Option d is the correct answer.
escaping into space, leading to a warming effect known Coral reefs do not protect communities from earthquakes.
as the greenhouse effect. The increased concentration
Earthquakes are caused by the movement of tectonic plates,
of greenhouse gases, primarily from human activities
like burning fossil fuels and deforestation, intensifies the and coral reefs are not strong enough to withstand the forces
greenhouse effect, resulting in global warming. of an earthquake.

331 PYQ Workbook


GENERAL GEOGRAPHY

131. Solution: (c)


Exp) Option c is the correct answer.
Singapore is known as a “Port of Call” because it is located
on a main sea route where ships often anchor for refueling,
watering, and taking on food items. As a strategic location,
Singapore has become a significant maritime hub, serving as
a stopover for ships during their journeys.

132. Solution: (c)


Exp) Option c is the correct answer.
The Tropic of Cancer is an imaginary line of latitude that
marks the northernmost point of the Sun’s direct rays.
It is currently located at about 23.5 degrees north of the
equator. The Tropic of Cancer passes through 17 countries, 3
continents and 6 water bodies
The countries that the Tropic of Cancer passes through are:
Important Tips
Western Sahara, Mauritania, Mali, Algeria, Niger, Libya,
• Coral reefs are crucial due to their biodiversity, acting Egypt, Saudi Arabia, United Arab Emirates, Oman, India,
as habitat and nursery grounds for marine species. Bangladesh, Myanmar, China, Taiwan, Bahamas, Mexico.
• They protect coasts from erosion and storms, have 133. Solution: (d)
economic value through fisheries and tourism, and
Exp) Option d is the correct answer.
hold potential for medicine.
A warm temperate Mediterranean climate, also called a dry
• They also sequester carbon, have cultural importance, summer climate, is a type of climate that occurs in the lower
offer recreational opportunities, contribute to food mid-latitudes, between roughly 30 and 45 degrees north or
security, and aid scientific research. south of the equator. It is characterized by hot, dry summers
and cool, wet winters.
130. Solution: (c)
All four cities mentioned in the options - Cape Town, Los
Exp) Option c is the correct answer.
Angeles, Adelaide, and Santiago - experience the warm
North Sudan is not a Mediterranean country because it temperate Mediterranean climate.
does not have a coastline on the Mediterranean Sea. The
134. Solution: (c)
other three countries, Tunisia, Libya, and Egypt, all have
Exp) Option c is the correct answer.
coastlines on the Mediterranean Sea.
• Chagos is an archipelago of islands in the Indian Ocean,
south of the Maldives and east of the Seychelles.
• Cyprus is an island country in the eastern Mediterranean
Sea, south of Turkey and west of Syria and Lebanon.
• Falkland is an archipelago of islands in the South
Atlantic Ocean, east of Argentina and south of Chile.
• Islas Cocos is an island in the Indian Ocean, southwest
of Sri Lanka and part of the Keeling Islands or Cocos
(Keeling) Islands, a territory of Australia .

135. Solution: (c)


Important Tips Exp) Option c is the correct answer.
The trick to remembering countries surrounding the Chile is located at the southernmost latitude, meaning that
Mediterranean Sea is: it is the farthest south from the equator. Chile is a long
and narrow country that stretches along the western coast of
Sci-Fi GM4 CASTLE BLAST
South America, from 17°S to 56°S. It borders Peru, Bolivia,
• SCI-FI: Spain, Cyprus, Israel, France, Italy and Argentina, and has a coastline on the Pacific Ocean.
• GM4: Greece, Morocco, and 4 Countries of Middle Chile also claims sovereignty over a part of Antarctica,
east Israel, Lebanon, Syria, Turkey which extends its southernmost latitude to 90°S.
• CASTLE BLAST: Croatia, Algeria, Slovenia, Tunisia, 136. Solution: (d)
Lebanon, Egypt, Bosnia and Herzegovina, Libya, Exp) Option d is the correct answer.
Albania, Syria, Tunisia
Ulaanbaatar is the capital of Mongolia. It is located in the

PYQ Workbook 332


GENERAL GEOGRAPHY

center of the country, at an altitude of over 1,300 meters. It 139. Solution: (c)
is surrounded by mountains, which block the moderating Exp) Option c is the correct answer.
influence of the ocean. As a result, Ulaanbaatar has a very
Laos is a landlocked country in Southeast Asia. It is
high annual range of temperature of about 25.5 degree
bordered by Vietnam to the east, Cambodia to the south,
celsius..
Thailand to the west, and China to the north. Laos is the
Important Tips only landlocked country in Southeast Asia.
The annual range of temperature is the difference
between the average annual maximum temperature and
the average annual minimum temperature. Cities that
are located in continental interiors tend to have a greater
annual range of temperature than cities that are located
near oceans. This is because oceans tend to moderate the
temperature, making it less extreme.

City Capital Annual Range of temperature


(degree Celsius)
Kuala Malaysia 6.5
Lumpur
Important Tips
New Delhi India 7.5
Double Landlocked Countries- A double landlocked
Shanghai China 5.5
country is a country that is completely surrounded by
other landlocked countries. There are only two double
137. Solution: (c) landlocked countries in the world:
Exp) Option c is the correct answer. Liechtenstein (surrounded by Switzerland and Austria)
Aw - (3) North and South of Amazon Forests. Uzbekistan (surrounded by Afghanistan, Kazakhstan,
The climate type Aw is a tropical savanna climate, which is Kyrgyzstan, Tajikistan, and Turkmenistan).
characterized by wet summers and dry winters.
140. Solution: (d)
Af - (4) Western Equatorial Africa.
Exp) Option d is the correct answer.
The climate type Af is a tropical rainforest climate, which is
characterized by high temperatures and rainfall throughout The Lakshadweep Islands in the Arabian Sea are made up of
the year. small coral atolls and reefs. The Lakshadweep archipelago
is a group of islands located off the southwestern coast of
Cs- (1) Central California.
India. These islands are formed by coral reefs and atolls that
The climate type Cs is a steppe climate, which is characterized have developed over time. Coral atolls are circular or ring-
by hot, dry summers and cold, snowy winters. shaped coral reefs that surround a central lagoon.
Am- (2) Indian Sub-continent.
Important Tips
The climate type Am is a monsoon climate, which is
Andaman and Nicobar Islands:
characterized by a wet summer monsoon and a dry winter
monsoon. Duncan’s passage separates Little Andaman from South
Andaman.
138. Solution: (b) Ten Degree Channel separates The Great Andaman
Exp) Option b is the correct answer. group in the North from the Nicobar group in the south.
Aravallis- Aravalli Range is the oldest mountain range and Port Blair is the capital of Andaman Nicobar Islands.
it rose in the Pre-Cambrian era. Guru Shikhar is the highest Saddle peak (737 m) in North Andaman is the highest
peak within the Aravalli Range. peak. These islands are formed of granitic rocks.
Dharwars- The Dharwar system is considered to be 2500 Major tribes in the Andaman and Nicobar Islands include
million to 1800 million years old and is the result of Onges, Sentinelese, Jarawas, and Shompen.
weathering of the Archaean rocks.
Cuddapah- The Cuddapah system is considered to be 1400- 141. Solution: (a)
600 million years old and is named after the Cuddapah Exp) Option a is the correct answer.
district of Andhra Pradesh. The meridian of Cape of Tasmania forms the boundary
Vindhyans- The Vindhyan Range is considered to come line between the Indian Ocean and the Pacific Ocean. The
after the Cuddapah system and consists of sedimentary Cape of Tasmania is located at the southernmost point of
rocks formed by the deposition of silt in river valleys and Australia, and its meridian, known as the 146°E meridian,
shallow oceans. serves as the dividing line between the two oceans.

333 PYQ Workbook


GENERAL GEOGRAPHY

142. Solution: (b) represent relief features on maps, but they are not as accurate
Exp) Option b is the correct answer. as contours.
Option a is correct- Kolhan is a division in the state of
Jharkhand, India. It is known for its mineral resources, such
as coal, iron ore, and bauxite.
Option b is incorrect- Kutch is a district in the state of
Gujarat, India. It is known for its arid landscape and its
Rann of Kutch, which is a vast salt marsh.
Option c is correct- Malabar Coast is a region in the state
of Kerala, India. It is known for its beautiful beaches, its
backwaters, and its hill stations. The Malabar Coast is also
known for its water resources, such as rivers, lakes, and
estuaries.
Option d is correct- Middle Ganga Plain is a region in the
states of Uttar Pradesh and Bihar, India. It is known for
its fertile soil and its agricultural production. The Middle
Ganga Plain is also known for its mineral resources, such as
coal and iron ore.

143. Solution: (b)


Exp) Option b is the correct answer.
The Manikaran Geothermal Power Plant is located in the
Parvati Valley in the Kullu District of Himachal Pradesh.
The Manikaran geothermal field stretches from the Harihar
temple at the village entrance to the point where the Parbati
River meets the Brahmaganga Nala. In this area, thermal
springs typically emerge either through fissures in quartzite
rock formations or from beneath the terrace gravel deposits. Important Tips
These hot springs in Manikaran have been a source of • Hachures are a traditional method for depicting
relaxation and enjoyment for visitors. They take the form of relief on maps. They convey slope orientation,
spouts and pools, with some reaching temperatures as high steepness, and terrain shape through variations in
as 96°C. In certain springs, you can also observe bubbling line thickness and density. While not as precise as
activity. contour lines, they effectively communicate specific
terrain shapes and are a type of shading distinct from
Important Tips shaded maps.
The six most promising geothermal energy sites in • Layer coloring is a method to represent elevation or
India: depth by using different colors, like varying shades
Cambay Graben in Gujarat of blue to depict water depth. However, it is less
Manikaran in Himachal Pradesh efficient than contour lines as it lacks the precise and
detailed information contour lines provide regarding
Tattapani in Chhattisgarh
elevation changes and the shape of the land.
Surajkund in Jharkhand
• Hill shading is a technique used to represent
Chhumathang in Jammu & Kashmir topographical features on maps by using shading
Puga in Jammu & Kashmir to indicate slopes and terrain variations. While
it provides an intuitive visual experience, it is less
144. Solution: (a)
efficient than contour lines because it lacks the
Exp) Option a is the correct answer. precision and detailed elevation information that
The most accurate method of representing relief features contour lines offer, making it less suitable for precise
on a map is through the use of contour lines. Contour lines analysis and scientific purposes.
are lines on a map that connect points of equal elevation
145. Solution: (d)
above a reference plane, typically meaning sea level. By
using contour lines, mapmakers can accurately depict the Exp) Option d is the correct answer
three-dimensional shape of the land surface, providing a The Indian Institute of Remote Sensing (IIRS) is a premier
detailed and precise representation of relief features such institute for research, higher education, and training in the
as hills, valleys, and mountains. The other three options, field of remote sensing, geoinformatics, and GPS technology
hachures, layer colouring, and hill shading, are also used to for natural resources, environmental, and disaster

PYQ Workbook 334


GENERAL GEOGRAPHY

management under the Indian Space Research Organisation Jawahar Tunnel, also known as Banihal Tunnel, passes
(ISRO). The IIRS was established in 1966 and is located in through the Pir Panjal mountain range in the Union
Dehradun, Uttarakhand. It is one of the institutes that come
Territory of Jammu and Kashmir, India. It is situated at
under the ISRO. The IIRS is responsible for implementing
the National Remote Sensing Policy, which was launched by an elevation of 2,194 m and was constructed between 1954
the Government of India in 1999. and 1956. The tunnel, with a length of 2.85 km, consists
of two parallel tubes allowing one-lane road traffic in each
Important Tips
Institutes of ISRO and their locations: direction. It is situated between Banihāl and Qazigund on

• Department of Space and ISRO HQ - Bengaluru NH 1A that has been renumbered NH 44. It plays a crucial

• Human Space Flight Centre (HSFC) - Bengaluru role in providing year-round road connectivity between

• Indian Institute of Remote Sensing (IIRS) - Dehradun, Srinagar and Jammu.

• ISRO Inertial Systems Unit (IISU) - Important Tips


Thiruvananthapuram
Some important Tunnels in North India:
• ISRO Propulsion Complex (IPRC) - Mahendragiri
Shyama Prasad Mukherjee Tunnel: Originally known as
• ISRO Telemetry, Tracking and Command Network the Chenani-Nashri Tunnel, it is India’s longest highway
(ISTRAC) - Bengaluru tunnel, stretching 9 km. Located in the lower Himalayan
• Laboratory for Electro-Optics Systems (LEOS) - mountain range, it connects Udhampur and Ramban in
Bengaluru Jammu & Kashmir. Notably, it is also Asia’s longest bi-
• Liquid Propulsion Systems Centre (LPSC) - directional highway tunnel.
Thiruvanthapuram Banihal-Qazigund Tunnel: Situated at an elevation of
• Master Control Facility (MCF) - Hassan 1,790 m in the Pir Panjal range, this 8.5 km road tunnel
links Banihal and Qazigund in the union territory of
• National Remote Sensing Centre (NRSC) - Hyderabad
Jammu and Kashmir, enhancing year-round connectivity.
• Satish Dhawan Space Centre (SDSC) SHAR - Nellore
Nandni Tunnels: This series of four highway tunnels
• Space Applications Centre (SAC) - Ahmedabad runs under the Nandni wildlife sanctuary on the Jammu-
• U R Rao Satellite Centre (URSC) - Bengaluru Srinagar National Highway in Udhampur district. With a
• Vikram Sarabhai Space Centre (VSSC) - combined length of 1.4 km, they have significantly reduced
Thiruvanthapuram the distance and travel time between Jammu and Srinagar.
Z-Morh Tunnel: Under construction and located 20 km
146. Solution: (d) from Zojila Pass on the Srinagar-Kargil-Leh highway, this
Exp) Option d is the correct answer. 6.5 km tunnel will directly connect Gagangir to Sonamarg
National Waterway No. 1 (NW-1) is officially known as in Kashmir.
the Ganga-Bhagirathi-Hooghly River Waterway. It spans Zojila Tunnel: An under-construction tunnel, it is set to
a distance of 1,620 kilometers and runs from Allahabad become Asia’s longest road tunnel, measuring 14.2 km.
(Prayagraj) to Haldia, making it the longest national Situated at an altitude of 11,578 feet above sea level, it will
waterway in India. NW-1 covers the Ganges, Bhagirathi, ensure year-round connectivity between Leh, Kargil, and
and Hooghly river system and has fixed terminals at Srinagar.
Haldia, Farrakka, and Patna, as well as floating terminals
at various riverside cities such as Kolkata, Bhagalpur, 148. Solution: (d)
Varanasi, and Allahabad. Exp) Option d is the correct answer.
The 38th parallel border is a line of latitude that roughly
divides the Korean Peninsula in half. It is located 38 degrees
north of the Earth’s equatorial plane. The border was
established in 1945 at the end of World War II, when the
Korean Peninsula was divided into two occupation zones,
one administered by the United States and the other by the
Soviet Union. The 38th parallel was chosen as the dividing
line because it was roughly in the middle of the peninsula.
In 1948, the two occupation zones became independent
147. Solution: (b) countries: North Korea and South Korea. The 38th parallel
Exp) Option b is the correct answer. became the official border between the two countries.

335 PYQ Workbook


GENERAL GEOGRAPHY

The western
Punas Cold dry wind side of Andes
Mountain
Blizzard Cold wind Tundra region
Purga Cold wind Russia
Levanter Cold wind Spain
Norwester Hot wind New Zealand
Santa Ana Hot wind South California
Karaburun
Hot dusty wind Central Asia
(black storm)
Saharan Air Layer
Dust-laden dry
Calima across the Canary
wind
Islands
Moist wind in
Elephanta Malabar coast
monsoon

151. Solution: (b)


149. Solution: (d)
Exp) Option b is the correct answer
Exp) Option d is the correct answer.
Isobaths are utilized to represent depth. These lines on Dakshin Gangotri is an Indian research station in Antarctica.
maps connect points with the same underwater depth,
helping in the visualization and understanding of ocean or It was the first scientific base station of India situated in
lakebed topography and contours. Antarctica, part of the Indian Antarctic Programme. It was
150. Solution: (b)
established during the third Indian expedition to Antarctica
Exp) Option b is the correct answer.
in 1983–84. This was the first time an Indian team spent a
The bora is a strong, cold, and dry wind that primarily
affects regions near the Adriatic Sea, especially in areas of winter in Antarctica to carry out scientific works. The station
Italy and Croatia. It is not related to Siberia.
was built in eight weeks by an 81-member team.
Important Tips
List of Local winds: Important Tips
Name Nature of wind Place India’s second research station ‘Maitri’ is located in
Chinook (Snow The Rockies antarctica. It was built in 1989, shortly before the first
Hot, dry wind station Dakshin Gangotri was buried in ice and abandoned
eaters) mountains
in 1990-91. Maitri research station is in the rocky
Foehn Hot, dry wind The Alps
mountainous region of antarctica called Schirmacher
Khamsin Hot, dry wind Egypt Oasis.
Sahara to the
Siroco Hot, moist wind Mediterranean 152. Solution: (a)
Sea
Sahara to the Exp) Option a is the correct answer.
Solano Hot, moist wind
Iberian Peninsula
Indonesia is a Southeast Asian country, while the other three
Harmattan
(Guinea Hot, dry wind West Africa options, Melanesia, Micronesia, and Australia, are all part of
Doctor)
Oceania.
Blows from
Bora Cold, dry wind Hungary to
Oceania is a geographical region that includes the islands of
North Italy
The Alps and the Pacific Ocean. The names of countries that are part of
Mistral Cold wind
France
Oceania are as follows:

PYQ Workbook 336


GENERAL GEOGRAPHY

Important Tips
Top 5 Biggest Ports in the World in 2023:
Port of Shanghai, Shanghai, China
Cargo Handling Capacity: 744 million tonnes
Port of Singapore, Singapore
Cargo Handling Capacity: 537.6 million tonnes
Port of Tianjin, Tianjin, China
Cargo Handling Capacity: 476 million tonnes
Port of Guangzhou, Guangzhou, China
Cargo Handling Capacity: Over 460 million tonnes
Port of Ningbo, Ningbo-Zhoushan, China
Cargo Handling Capacity: More than 453 million tonnes.

155. Solution: (d)


Exp) Option d is the correct answer.
A. Pavagada - 2. Solar
Pavagada Solar Park is a solar park located in Pavagada taluk,
Tumkur district, Karnataka, India. It has a total installed
capacity of 2,050 MW. The park was commissioned in 2018.
As of 2021, it is the world’s third largest Photovoltaic Solar
Park after Bhadla Solar Park in Rajasthan and Huanghe
Hydropower Hainan Solar Park in China.
153. Solution: (c) B. Tattapani - 3. Geothermal
Exp) Option c is the correct answer. Tattapani geothermal field is located in the Sarguja district
of Chhattisgarh, India. It is one of the most promising
The largest part of our hydrosphere is the Pacific geothermal resources in central India. The field has a
Ocean. Covering about 46% of Earth’s water surface and number of hot springs with temperatures ranging from 50
to 98 degrees Celsius. The heat source for the geothermal
approximately 32% of the planet’s total surface area, the field is believed to be a combination of magmatic intrusion
Pacific Ocean is the largest and deepest of the world’s five and radioactive decay.
C. Muppandal - 1. Wind
oceanic divisions. It extends from the Arctic Ocean in the
The Muppandal Wind Farm is located in the Kanyakumari
north to the Southern Ocean in the south and is bounded district of Tamil Nadu, India. It is the world’s third largest
by the continents of Asia and Oceania in the west and the operational onshore wind farm, with a total installed capacity
of 1,500 MW. It was commissioned in 1986 and has been
Americas in the east. operational for over 30 years.
D. Kakrapar - 4. Atomic
154. Solution: (c)
The Kakrapar Atomic Power Plant (KAPP) is a nuclear
Exp) Option c is the correct answer. power plant located in Kakrapar, Gujarat, India. It is
operated by the Nuclear Power Corporation of India
The largest harbor among the options is the Port of Limited (NPCIL). KAPP has four units, two of which are
Rotterdam, located in Rotterdam, The Netherlands. This 220 MW pressurized heavy water reactors (PHWRs) and two
of which are 700 MW PHWRs.
port holds the distinction of being both the largest port
156. Solution: (d)
in Europe and the sixth largest port globally. It boasts an
Exp) Option d is the correct answer.
impressive cargo handling capacity of 441.5 million tonnes in
Option A matches with option 4: Mount Aconcagua,
2023 and covers a vast area of approximately 12,426 hectares situated on the border of Chile and west-central Argentina,
with a length of 42 kilometers. holds the distinction of being the highest point in South
America and the Western Hemisphere. This imposing peak,
Note: This European port served as the largest port in the soaring over 6,000 feet, also claims the title of the highest
summit in both the Western and Southern Hemispheres.
world for 42 years between 1962 and 2004 before it was
Located within the majestic Andes Mountain range, it
surpassed by Singapore and Shanghai. proudly represents Argentina.

337 PYQ Workbook


GENERAL GEOGRAPHY

Option b matches with option 3: The Meseta Central is a vast


Important Tips
interior plateau nestled in the heart of the Iberian Peninsula,
The tallest mountain we know of in the whole Solar
specifically in central Spain in Europe. Encompassing System is Olympus Mons on Mars, and it’s about 26
an expansive area of 81,000 square miles (210,000 square kilometers high. On Earth, the highest mountain is Mount
Everest, which stands at roughly 8,848 meters.
kilometers) and boasting an average elevation of 2,165 feet
Some major mountain ranges around the world:
(660 meters), this geological formation stands as the oldest
• Mt. Aconcagua: Located in Western South America
and most intricate on the peninsula. Its gentle western slope
• Mt. Albert: Found in Western South America
slopes toward the Atlantic Ocean, forming the basins of the
• Mt. Everest: Situated in South Central Asia
Duero, Tagus, and Guadiana rivers. Notably, at its center lies
• Mt. Kosciusko: Located in Eastern Australia
the vibrant city of Madrid.
• Mt. Vinson Massif: Found in Antarctica
Option c matches with option 2: The Serengeti ecosystem • Mt. Gora Velukha: Located in Central Asia
spans across northern Tanzania in Africa, encompassing a • Mt. Gora Nordnaya: Situated in Central Russia
protected region of approximately 30,000 square kilometers • Mt. Anaimudi: Found in India
(12,000 square miles), including the renowned Serengeti • Galdhopijen: Located in Western Norway
National Park and various game reserves. Within this • Mt. Elbrus (Western Peak): Found in Russia
expanse, the Serengeti plays host to the world’s second- • Mt. Michel: Situated in Eastern USA-Canada
largest terrestrial mammal migration, solidifying its status • Mt. Blanc: Located in Central Europe
as one of the Seven Natural Wonders of Africa. 158. Solution: (a)
Option d matches with option 1: The Gibson Desert, Exp) Option a is the correct answer.
located in Western Australia, remains largely untouched and A. Kosciuszko - 3. Australia
pristine. Covering an impressive 155,000 square kilometers Mount Kosciuszko is the tallest mountain on the Australian
(60,000 square miles), it ranks as Australia’s fifth-largest mainland, reaching an elevation of 2,228 meters.
B. McKinley - 4. North America
desert, following the Great Victoria, Great Sandy, Tanami,
Mount McKinley, now known as Denali, stands as the
and Simpson deserts. The Gibson Desert encompasses both
highest mountain peak in North America, towering at a
an interim Australian bioregion and a desert ecoregion. height of 6,190 meters.
C. Elbrus - 1. Europe
157. Solution: (c)
Mount Elbrus claims the title of the highest mountain in
Exp) Option c is the correct answer. Europe, with an elevation of 5,642 meters. It’s the highest
A. Elbrus - 4. Russia peak in the Caucasus Mountains and is a dormant volcano.
D. Kilimanjaro - 2. Africa
Mount Elbrus is a dormant volcano located in the Caucasus
Mount Kilimanjaro holds the distinction of being the tallest
Mountains in southern Russia. It is the highest mountain in mountain in Africa, rising to a height of 5,895 meters. It
Europe, with an elevation of 5,642 meters. is also the world’s highest free-standing mountain and is
characterized as a dormant volcano.
B. Kilimanjaro - 3. Tanzania
159. Solution: (c)
Mount Kilimanjaro is a dormant volcano located in
Exp) Option c is the correct answer.
Tanzania, East Africa. It is the highest mountain in Africa,
• The Strait of Gibraltar is between the Atlantic Ocean
with an elevation of 5,895 meters.
and the Mediterranean Sea. It separates the continents
C. Mt. Cook - 1. New Zealand of Africa and Europe, and the countries of Morocco and
Spain. The strait is about 14 km wide at its narrowest
Mount Cook is the highest mountain in New Zealand, with point, and has a depth of up to 300 m. The strait is a
an elevation of 3,724 meters. It is located in the Southern major route for ships traveling between Europe, Africa,
Alps, on the South Island. and Asia.
• The Strait of Malacca is between the Indian Ocean and
D. McKinley - 2. USA
the Pacific Ocean. It separates the islands of Sumatra
Mount McKinley, also known as Denali, is the highest (Indonesia) and Peninsular Malaysia. The strait is about
800 km long and varies in width from 65 km to 300 km. It
mountain in North America, with an elevation of 6,190
has a depth of up to 200 m. The strait is one of the busiest
meters. It is located in the Alaska Range, in the state of and most strategic waterways in the world, as it connects
Alaska. China, Japan, India, and other countries in the region.

PYQ Workbook 338


GENERAL GEOGRAPHY

• The Strait of Bering is between the Arctic Ocean and Important Tips
the Pacific Ocean. It separates the continents of Asia
Conditions for coral growth are:
and North America, and the countries of Russia and
the United States. The strait is about 85 km wide at its Sunlight: Corals need to grow in shallow water where
narrowest point, and has a depth of up to 50 m. The strait sunlight can reach them. Corals rarely develop in water
is a natural bridge for animals and humans to migrate deeper than 165 feet (50 meters).
between the two continents. It also has rich marine life Warm water temperature: Corals generally live in water
and natural resources. temperatures of 68–90° F or 20–32° C. They have tolerance
to a very narrow temperature range. Corals thrive in
• The Strait of Hormuz is between the Persian Gulf and
tropical waters between 30°N and 30°S latitudes, where
the Gulf of Oman. It separates the countries of Iran
diurnal and annual temperature ranges are very narrow.
and Oman, and also borders the United Arab Emirates,
Saudi Arabia, Iraq, Kuwait, Bahrain, and Qatar. The Clear salt water: Corals need clear water that lets sunlight
through. They also need saltwater to survive and require a
strait is about 180 km long and varies in width from 55
certain balance in the ratio of salt to water.
km to 95 km. It has a depth of up to 200 m. The strait is a
vital chokepoint for oil and gas exports from the Middle Clean water: Corals are sensitive to pollution and
East to the rest of the world. sediments. Sediment can create cloudy water and be
deposited on corals, blocking out the sun and harming the
160. Solution: (a) polyps.
Exp) Option a is the correct answer. Water movement: Corals need water movement to
The railway line connecting Siliguri and Darjeeling, bring them plankton, oxygen, and other nutrients, and
known as the Darjeeling Himalayan Railway (DHR) or Toy to remove their waste products. Water movement also
Train, has been recognized as a heritage railway line by prevents sediment from settling on corals. However, too
much water movement can damage corals or prevent them
UNESCO. Built between 1879 and 1881, it is about 88 km (55
from feeding.
mi) long. It climbs from about 100 m (330 ft) above sea level
at Siliguri to about 2,200 m (7,200 ft) at Darjeeling, using six 163. Solution: (b)
zig zags and five loops to gain altitude. In 1999, UNESCO
Exp) Option b is the correct answer.
designated it as a World Heritage Site, recognizing its
historical and cultural significance. The Amazon basin, located in northern South America,
is the largest drainage basin in the world. The Amazon
161. Solution: (c) River and its tributaries drain an area of nearly seven million
Exp) Option c is the correct answer square kilometers, which is about 40% of the continent’s
land area. The Amazon basin covers parts of Brazil, Peru,
Country Area (sq km)
Bolivia, Colombia, Ecuador, Venezuela, Guyana, Suriname
Canada 9,984,670 and French Guiana. The Amazon River is also the largest
U.S.A. 9,833,517 river in the world by discharge volume, with an average of
China 9,596,961 224,000 cubic meters per second.
Brazil 8,515,767 Important Tips
Therefore, Canada has the largest area, followed by the The Nile River, which is the longest river in the world at
U.S.A., China, and Brazil, in descending order. 7,088 kilometers, has a much smaller drainage basin of
about 3.2 million square kilometers.
162. Solution: (b)
The Congo River, which is the second largest river by
Exp) Option b is the correct answer. discharge volume and the deepest river in the world, has
Coral reefs are mostly found in the tropical and subtropical a drainage basin of about 3.7 million square kilometers.
regions of the world, where the water temperature is between The Mississippi-Missouri River system, which is the
22°C and 29°C. Coral reefs are sensitive to cold, sediment, fourth longest river in the world and the largest river in
and pollution, so, they are not common in areas where there North America, has a drainage basin of about 3.2 million
are cold currents, large rivers, or industrial activities. Coral square kilometers.
reefs can also grow in some places where warm currents flow
out of the tropics, such as Florida and southern Japan. 164. Solution: (c)

Coral reefs cannot grow on colder seacoasts, because they Exp) Option c is the correct answer.
need warm water to survive. Corals are animals that have a The Caspian Sea is the world’s largest inland body of
symbiotic relationship with algae called zooxanthellae, which water, often described as the world’s largest lake or a full-
provide them with food and oxygen through photosynthesis. fledged sea. An endorheic basin, it lies between Europe and
The algae need sunlight and warm water to perform Asia, covering a surface area of 371,000 km2 and a volume
photosynthesis, so corals cannot live in cold or deep waters. of 78,200 km3. It is bounded by Kazakhstan, Russia,

339 PYQ Workbook


GENERAL GEOGRAPHY

Azerbaijan, Iran, and Turkmenistan. The Caspian Sea has Important Tips
unique features, including low salinity, about a third of the
Sargasso Sea Additional facts:
salinity of average seawater and two deep basins.
The Sargasso sea is named for a kind of sea weed called
165. Solution: (d) “sargassum”. It is basically a “gyre” It lies in the “horse
latitudes” (about 20-35 degrees N) and often has little or
Exp) Option d is the correct answer. no wind.
Kandla Port, officially known as Deendayal Port, is a Sargasso Sea is located within the famous Bermuda
seaport in Kutch district of Gujarat state in Western India, Triangle area with one of its corners Bermuda located on
near the city of Gandhidham. Located on the Gulf of Kutch, its western edge.
it is one of India’s major ports on the west coast. It is Bermuda Triangle has become popularized in horror
about 256 nautical miles southeast of the Port of Karachi in stories as a place where ships have been becalmed for
generations. Actually it poses no threat to navigation or
Pakistan and about 430 nautical miles north-northwest of
shipping.
the Port of Mumbai.
The sea is characterised by weak currents, low
Important Tips precipitation, high evaporation, light winds, warm,
New Names of Ports: saline waters, and depths of 5,000–23,000 feet (1,500–
7,000 m). These factors, combined with the lack of
Ennore Port – Kamarajar Port.
thermal mixing, result in a biological desert that is
Kandla Port – Deendayal Port Trust. largely devoid of plankton, a primary source of food for
Kolkata Port – Dr. Syama Prasad Mukherjee Port. fish.
Nhava Sheva – Jawaharlal Nehru Port. Because of the currents Sargasso Sea has a high degree of
Panambur Port – New Mangalore Port. plastic pollutants in it.
Tuticorin Port – V.O. Chidambaram Port. 167. Solution: (a)

166. Solution: (a) Exp) Option a is the correct answer.

Exp) Option a is the correct answer. The Telegraph Plateau is not located in the Indian Ocean
but rather in the North Atlantic Ocean. It is a region
The Sargasso Sea is a region of the North Atlantic Ocean characterized by relatively flat and shallow terrain in the
that is elliptical in shape, calm, and covered in free- North Atlantic. This plateau includes oceanic crust and
floating seaweed from the species Sargassum. It lies in the a section of the Reykjanes Ridge, situated between the
North Atlantic Ocean, in between the Canary Islands, the Eurasian Plate and the North American Plate, within the
east coast of the United States and the Caribbean Sea. It is Charlie-Gibbs Fracture Zone.
the only sea in the world without a land border and does Important Tips
not have a coastline. The Gulf Stream marks the western The Cocos Ridge is an undersea geological feature in the
limit of the Sargasso Sea, the Canary Current the eastern, eastern Pacific Ocean, extending from Central America
the North Atlantic Current the northern, and the North to the Galapagos Islands.
Equatorial Current and Antilles Current the southern The Walvis Ridge is an aseismic ocean ridge in the
limit. These boundaries are moving in a clockwise direction. southern Atlantic Ocean. More than 3,000 km in length,
They alter according to the Azores High-Pressure Center it extends from the Mid-Atlantic Ridge, near Tristan da
from season to season. Cunha and the Gough Islands, to the African coast (at 18°S
in northern Namibia).
The Agulhas Basin is an oceanic basin located south of
South Africa where the South Atlantic Ocean and south-
western Indian Ocean meet. Part of the African Plate, it
is bounded by the Agulhas Ridge (part of the Agulhas-
Falkland Fracture Zone) to the north and the Southwest
Indian Ridge to the south; by the Meteor Rise to the west
and the Agulhas Plateau to the east.

168. Solution: (a)


Exp) Option a is the correct answer.
The Golden Quadrilateral Project is a vital highway network
connecting Delhi, Mumbai, Kolkata, and Chennai, uniting
India’s major economic and cultural hubs. Initiated in 2001

PYQ Workbook 340


GENERAL GEOGRAPHY

as Phase 1 of the National Highway Development Project, it


Important Tips
comprises 5,846 km of four/six-lane expressways, aimed at
Non-renewable energy resources are resources that
reducing travel time between these cities. This project also cannot be replenished naturally over time. Once they are
includes the North-South corridor connecting Srinagar used up, they are gone. Non-renewable energy resources
(Jammu and Kashmir) and Kanyakumari (Tamil Nadu), as include fossil fuels, such as coal, oil, and natural gas, as
well as nuclear fuel, such as uranium.
well as the East-West corridor connecting Silchar (Assam)
Examples of non-renewable energy resources:
and Porbandar (Gujarat). These highway projects are
Fossil fuels: Fossil fuels are formed from the remains of
implemented by the National Highway Authority of India ancient plants and animals that have been buried and
(NHAI). compressed over millions of years. The three main types
of fossil fuels are coal, oil, and natural gas. Fossil fuels are
used to generate electricity, power vehicles, and produce
a wide range of products, such as plastics, fertilizers, and
pharmaceuticals.
Nuclear fuel: Nuclear fuel is a type of fuel that is used in
nuclear power plants to generate electricity. Nuclear fuel
releases energy when its atoms are split, a process called
nuclear fission. The most common type of nuclear fuel
is uranium.

170. Solution: (b)

Exp) Option b is the correct answer.

Santos is Known as the “Coffee port” of the world. Santos

is a port city in the state of São Paulo, Brazil, located

on the Atlantic coast of Brazil, about 79 kilometers from

the city of São Paulo. This port is a crucial hub for the

shipment of coffee beans from Brazil, which is one of the

169. Solution: (d) world’s largest coffee producers. As such, it has earned the
Exp) Option d is the correct answer. nickname “Coffee port” due to its pivotal role in the coffee
All of the options listed are renewable resources. Renewable
trade. It is the largest port in Latin America. Santos is a
energy is energy that comes from natural sources that can
be replenished naturally over time, such as sunlight, wind, major center for the export of coffee, soybeans, corn, and

rain, tides, waves, and geothermal heat. Renewable energy other agricultural products. It is also a major import port
sources are sustainable and can help to reduce our reliance
for machinery, vehicles, and other manufactured goods.
on fossil fuels, which are finite and polluting.

Option a is correct- Hydropower is a renewable energy 171. Solution: (b)


source that uses the energy of moving water to generate
Exp) Option b is the correct answer.
electricity.

Option b is correct- Solar energy is a renewable energy The line joining the areas having the same rainfall is called
source that uses the energy of the sun to generate electricity an isohyet. Isohyets are used to create isohyetal maps, which
or heat.
show the distribution of rainfall over an area. Isohyetal
Option c is correct- Geothermal energy is a renewable
maps are useful for understanding rainfall patterns and for
energy source that uses the heat generated within the Earth
to generate electricity or heat. predicting future rainfall trends.

341 PYQ Workbook


GENERAL GEOGRAPHY

Important Tips
Different Types of Isolines in Geography:
Isobars: Lines connecting points of equal atmospheric
pressure on a weather map.
Isotherms: Lines connecting points of equal temperature,
helping visualize temperature patterns on maps.
Isobaths: Lines connecting points of equal depths in
oceans or other bodies of water, used in oceanography and
cartography.
Isoseismal: Lines connecting points of equal earthquake
intensity, helping to define the affected areas after an
earthquake.
Isonephs: Lines connecting points of equal cloudiness or
cloud cover, used in meteorology. Important Tips
Isohalines: Lines connecting points of equal salinity in • The Kalahari is a large semi-arid desert covering much
bodies of water, essential in oceanography. of Botswana, as well as parts of Namibia and South
Isogenes: Lines connecting points of equal magnetic Africa.
variation or declination, important for navigation. • The Gobi Desert is found in Mongolia and northern
China.
172. Solution: (c)
• The Thar Desert is arid region of rolling sand hills
Exp) Option c is the correct answer. on the Indian subcontinent. It is located partly in
The country that forms the longest territorial border with Rajasthan state of India and partly in Punjab and Sindh
India is Bangladesh. The India- Bangladesh border is 4,096 provinces of Pakistan.
kilometers.
174. Solution: (b)
The other countries in the list have the following border
lengths with India: Exp) Option b is the correct answer.
China: 3,488 kilometers The correct order from west to east is:
Pakistan: 3,310 kilometers 2. Lisbon, Portugal
Nepal: 1,751 kilometers Lisbon is Portugal capital city. it is hilly, coastal city. Portugal
occupies the westernmost position on continental Europe
and shares its borders with Spain to the north and east,
while the Atlantic Ocean surrounds it to the west and south.
Portugal is most famous for its wine.
1. London, United Kingdom
The capital of the United Kingdom is London. United
Kingdom is a country in northwestern Europe, made up
of England, Scotland, Wales, and Northern Ireland. The
United Kingdom possesses only one land border, which
is the island of Ireland. Since 1994, the English Channel
has connected the UK to France. The United Kingdom has
maritime boundaries that it shares with Belgium, Denmark,
France, Germany, the Netherlands, Norway, and the Faroe
Islands (which is a part of Denmark).
3. Frankfurt, Germany
The Capital of Germany is Frankfurt. Germany’s northern
boundaries are defined by the North Sea, the Baltic Sea,
and Denmark. To the west, it shares borders with the
Netherlands, Belgium, Luxembourg, and France. To the
south, Germany is bordered by Switzerland and Austria,
173. Solution: (c) while to the east, it is bounded by the Czech Republic and
Exp) Option c is the correct answer. Poland.
The largest desert in the world is Antarctica. However, the 4. Beirut, Lebanon
largest hot desert in the world is Sahara. The Sahara Desert The capital of Lebanon is Beirut. Lebanon’s borders are
spreads over eleven countries. Countries like Algeria, Libya, defined by Syria to the north and east, Israel to the south,
Egypt and several others form a part of the Sahara Desert. and the Mediterranean Sea to the west.

PYQ Workbook 342


GENERAL GEOGRAPHY

175. Solution: (d) 178. Solution: (a)


Exp) Option d is the correct answer. Exp) Option a is the correct answer.
The White Mountains constitute a mountain range that The Zaire River, also known as the Congo River, is the only
spans approximately one-quarter of New Hampshire and major river that crosses the equator twice, once in each
extends into a small portion of western Maine in the United
direction. It is the second-longest river in Africa, after the
States. It has an elevation of 6,288 ft. These mountains are
a segment of the northern Appalachian Mountains and are Nile, and the second-largest river in the world by discharge
known for their particularly rugged terrain, making them volume, after the Amazon. It has a total length of 4,370 km
the most challenging mountains in the New England region. (2,715 mi) when measured along with its main tributary, the
Some of the higher peaks within this range feature Alpine Lualaba. It flows through the Congo Rainforest, the world’s
tundra. The White Mountains have close proximity to major second-largest rainforest, and drains into the Atlantic Ocean.
cities such as Boston, New York City, and Montreal.
179. Solution: (b)
Important Tips
Exp) Option b is the correct answer.
Dhaulagiri I is also known as the ‘White Mountain,’. The
Mountain has an elevation of 8,167 meters above sea level, Option 1 is incorrect: Laos is a landlocked country located
ranking as the world’s seventh-highest mountain and in the Southeast Asian region of the continent of Asia.
the tallest mountain entirely contained within Nepal’s Option 2 is correct: Georgia, the country in the South
borders. Positioned 34 kilometres east of Annapurna I, Caucasus region. While it is situated at the crossroads of
which reaches 8,091 meters, the Kali Gandaki River flows Eastern Europe and Western Asia, the majority of its
between them through the world’s deepest gorge, known
territory is located in the South Caucasus region of Asia.
as the Kaligandaki Gorge. Serving as a major regional
Only a small portion of Georgia’s land, specifically the
city and the gateway for climbers and trekkers exploring
western part, is in Europe.
both ranges, Pokhara is also a popular tourist destination
located to the south of the Annapurnas. Option 3 is correct: Turkey is considered a transcontinental
country because it spans two continents, namely Asia and
176. Solution: (b)
Europe. The larger portion of Turkey, about 97% of its land
Exp) Option b is the correct answer. area, is located in Asia (Anatolia), while the remaining 3% is
Chisapani Gorge is a place where the Karnali River, also in Europe (Thrace). Istanbul, one of Turkey’s major cities, is
known as the Ghaghara River in India, emerges from the the only city in the world that straddles two continents, with
Shiwalik Range onto the broad plain and flows through its eastern part in Asia and western part in Europe.
the semi-tropical jungle. Chisapani Gorge is located near
Option 4 is incorrect: Tunisia is a country located in the
the border of Nepal and India and is part of the Karnali
hydroelectric project. Chisapani is also the name of a village North African region of the African continent. It is situated
in Nepal that is a popular trekking destination. Chisapani in the northeastern part of Africa and shares borders with
means “cold water” in Nepali. Algeria to the west and southwest and Libya to the southeast.
To the north and east, Tunisia has a coastline along the
177. Solution: (c)
Mediterranean Sea.
Exp) Option c is the correct answer.
Faroe Islands is derived from old Norse Færeyjar and 180. Solution: (b)
literally means Sheep Islands, a name given by the Norse Exp) Option b is the correct answer.
people that settled these rough and wind-swept isles during The Great Artesian Basin (GAB) is Australia’s most
the Viking Age. The Faroese sheep, a type of Northern
significant hydrogeological system. The GAB contains a vast
European short-tailed sheep, is a small and hardy breed
volume of underground water and is the largest groundwater
that outnumber the human population by roughly 70,000
basin in Australia. It is the only reliable resource of fresh
to 50,000. Sheep have been a vital part of sustaining life on
the Faroe Islands since the first settlers arrived. The Faeroe water throughout much of inland Australia. It is made up of
Islands is a North Atlantic Island group and an autonomous several sedimentary basins.
territory of the Kingdom of Denmark.
181. Solution: (d)
Important Tips Exp) Option d is the correct answer.
Canary Islands are a Spanish archipelago off the coast of Isogonic lines are imaginary lines connecting points on
Africa.
the Earth’s surface where the magnetic declination, the
Madeira Islands are a Portuguese archipelago in the angle between true north and magnetic north, is the same.
Atlantic Ocean.
They help navigate the variations in magnetic fields across
Wellington Islands are a group of islands in the Antarctic different geographic locations, aiding in accurate compass
Peninsula. navigation.

343 PYQ Workbook


GENERAL GEOGRAPHY

Important Tips Hills is Mount Saramati, which has an elevation of 3,826

Different Types of Isolines in Geography: meters.

• Isobar: Points of equal atmospheric pressure. The Chin Hills are located south of the Naga Hills. They
• Isobath: Points of equal water depth. stretch from the Naga Hills in the east to the Arakan Yoma
• Isobathytherm: Points of equal water depth with the in the west. The highest peak in the Chin Hills is Mount
same temperature. Victoria, which has an elevation of 3,035 meters.
• Isochasm: Points of equal aurora occurrence The Lushai Hills are the southernmost of the three
frequency.
ranges. They stretch from the Chin Hills in the east to the
• Isocheim: Points of equal mean winter temperature.
Chittagong Hill Tracts in the west. The highest peak in the
• Isochrone: Points of equal travel time or distance
Lushai Hills is Mount Phawngpui, which has an elevation of
from a specific location.
2,210 meters.
• Isodapane: Points of equal transportation cost for
products from production to markets.
• Isodose: Points of equal intensity of radiation.
• Isodrosotherm: Points of equal dew point.
• Isogeotherm: Points of equal mean temperature.
• Isogloss: Points separating linguistic features.
• Isohaline: Points of equal salinity in the ocean.
• Isohel: Points receiving equal amounts of sunshine.
• Isohume: Points of equal humidity.
• Isohyet: Points of equal precipitation.
• Isoneph: Points of equal cloud cover.
• Isopectic: Points where ice begins to form at the same
time each fall or winter.
• Isophene: Points where biological events occur 183. Solution: (a)
simultaneously, like flowering.
Exp) Option a is the correct answer.
• Isoplat: Points of equal acidity, as in acid precipitation.
Lebanon is the only country out of the four listed in the
• Isopleth: Points of equal numerical value, like
population. question that does not have a land border with the Dead Sea.
• Isopor: Points of equal annual change in magnetic The other four countries, Jordan, Israel, Palestine, all share
declination. borders with the Dead Sea.
• Isostere: Points of equal atmospheric density. The Dead Sea is a Salt Lake located in the Jordan Rift Valley.
• Isotac: Points where ice begins to melt at the same It is bordered by Jordan to the east and Israel and the West
time each spring.
Bank to the west.
• Isotach: Points of equal wind speed.
• Isothere: Points of equal mean summer temperature.
• Isotherm: Points of equal temperature.
• Isotim: Points of equal transport costs from the
source of a raw material.

182. Solution: (d)


Exp) Option d is the correct answer.
The Naga Hills, Chin Hills, and Lushai Hills are three
mountain ranges that form the border between India and
Myanmar. They are part of the Indo-Burmese Ranges, which
are a series of mountain ranges that run along the eastern
edge of the Indian subcontinent.
The Naga Hills are the northernmost of the three ranges.
They stretch from the Patkai Range in the east to the
Manipur Hills in the west. The highest peak in the Naga

PYQ Workbook 344


GENERAL GEOGRAPHY

Important Tips 187. Solution: (b)

• The Dead Sea is one of the saltiest bodies of water in Exp) Option b is the correct answer.
the world, with a salinity of over 30%. The Ninety East Ridge is situated in the Indian Ocean. This
• The Dead Sea is shrinking at an alarming rate, due to mid-ocean ridge stretches approximately 5,000 kilometers
water diversion projects and climate change. in length and is named for its near-parallel alignment
• The Dead Sea is home to a unique ecosystem of bacteria along the 90th meridian in the Eastern Hemisphere. It
and algae that have adapted to the extreme salinity of
extends from the Bay of Bengal southward towards the
the water.
Southeast Indian Ridge (SEIR). The ridge extends between
• The Dead Sea is home to a number of historical and
latitudes 31°S and 9°N and has an average width of 200 km.
archaeological sites, including the ruins of the ancient
city of Sodom. 188. Solution: (c)
• The Dead Sea is a UNESCO World Heritage Site.
Exp) Option c is the correct answer.
184. Solution: (c) The Hindu Kush, the Karakoram Range, the Kunlun
Exp) Option c is the correct answer. Mountains, and the Tien Shan are only a few mountain
Addu Atoll is the southernmost atoll of the Maldives, a ranges extending from the Pamir Mountain system, which
country in the Indian Ocean. The Indian Ocean is the third is centered on the nodal orogenic uplift known as the Pamir
largest of the world’s oceanic divisions, covering about 20% of Knot. Because of this, the Pamir Mountains are known as
the Earth’s water surface. It is bounded by Asia to the north, ‘The Roof of the World’.
Africa to the west, Australia to the east, and Antarctica to
the south. The Indian Ocean is home to many diverse marine 189. Solution: (b)
ecosystems and species, as well as important trade routes and Exp) Option b is the correct answer.
cultural connections.
Canberra is the capital of Australia and is located inland,
185. Solution: (a) about 300 kilometers from the coast. It is not a sea-port
Exp) Option a is the correct answer. city. The other three options, Tokyo, New York, and
The Great Victorian Desert is located at Australia. It is the London are all sea-port cities.
largest desert of Australia stretching from eastern Western
Australia across the western half of South Australia.

Important Tips
Deserts of Australia (in descending order area wise):
• Great Victoria Desert
• Great Sandy Desert
• Tanami Desert
• Simpson Desert
• Gibson Desert

186. Solution: (a)


Exp) Option a is the correct answer.
The Bab-El-Mandeb is a strait between Yemen on the
Arabian Peninsula, Djibouti, and Eritrea in the Horn of
Africa. It connects the Red Sea to the Gulf of Aden and the
Indian Ocean.
190. Solution: (a)
Important Tips
Exp) Option a is the correct answer.
The Hormuz is a strait between Iran and Oman that
connects the Persian Gulf to the Gulf of Oman and the The nearest place from the center of the Earth is at the
Arabian Sea. bottom of the Arctic Sea near the North Pole, which is
The Bosphorus is a strait between Turkey and Europe located 6553 km from the center of the Earth. Earth is not a
that connects the Black Sea to the Sea of Marmara and the perfect sphere, but is slightly flattened at the poles and bulges
Mediterranean Sea. at the equator. This is due to the centrifugal force created by
The Malacca is a strait between Malaysia and Indonesia the Earth’s rotation. The centrifugal force is strongest at the
that connects the Andaman Sea to the South China Sea equator and weakest at the poles. As a result, the North Pole
and the Pacific Ocean. is slightly closer to the center of the Earth than the equator.

345 PYQ Workbook


GENERAL GEOGRAPHY

Important Tips
The highest point above Earth’s center is the summit of Mount Chimborazo, which extends more than 7,000 feet (over
2,100 meters) farther into space from the center of the Earth compared to any of the peaks in the Himalayas, due to the
Earth’s equatorial bulge.
While the South Pole lies on a continental land mass, the North Pole is located in the middle of the Arctic Ocean amid waters
that are almost permanently covered with constantly shifting sea ice. The sea depth at the North Pole has been measured at
4,261 m.

PYQ Workbook 346

You might also like